From svenpran at online.no Sat Nov 1 00:03:04 2008 From: svenpran at online.no (Sven Pran) Date: Sat, 1 Nov 2008 00:03:04 +0100 Subject: [blml] Mr Gladstone suggested [SEC=UNOFFICIAL] In-Reply-To: <2a1c3a560810311251o1b9ea039j39b200467f77c725@mail.gmail.com> References: <1062526051-1225456021-cardhu_decombobulator_blackberry.rim.net-888530924-@bxe065.bisx.produk.on.blackberry> <2a1c3a560810311251o1b9ea039j39b200467f77c725@mail.gmail.com> Message-ID: <000701c93bac$d4c53f00$7e4fbd00$@no> On Behalf Of Wayne Burrows ...................... > I think the way the HUM regulations are written if there exists a hand > that I would open in my methods that is a king or more below average > strength then I am playing a HUM. This interpretation comes from the > word "may" in the HUM regulation "By partnership agreement an opening > bid at the one level may be made with values a king or more below > average strength." > > If I "may" have 7 hcp and it is part of my partnerships methods then I > am playing a HUM. Precisely! (And according to Law 40 this fact is part of your partnerships methods unless "your partner has no more reason to be aware of it than have your opponents".) Regards Sven From svenpran at online.no Sat Nov 1 00:27:41 2008 From: svenpran at online.no (Sven Pran) Date: Sat, 1 Nov 2008 00:27:41 +0100 Subject: [blml] Mr Gladstone suggested [SEC=UNOFFICIAL] In-Reply-To: <000001c93ba4$f34d3b10$d9e7b130$@com> References: <1062526051-1225456021-cardhu_decombobulator_blackberry.rim.net-888530924-@bxe065.bisx.produk.on.blackberry> <2a1c3a560810311251o1b9ea039j39b200467f77c725@mail.gmail.com> <000001c93ba4$f34d3b10$d9e7b130$@com> Message-ID: <000801c93bb0$45467ca0$cfd375e0$@no> On Behalf Of David Burn > [WB] > > If I open 1S to show 0-7 any shape then this is a HUM. > > If I open 1S to show "some 0-7 any shape" then are you saying this is not a > HUM? > > [DALB] > > If there exists a non-empty set of hands in the range 0-7 on which you would > always open 1S, then you are playing a HUM. I am not sure whether that is > what you just asked, but as I said, if Helgemo and Helness open all (or > "almost all") seven-point hands with five hearts 1H in third position, then > they are playing a HUM. No sorry, this must be incorrect! If there exists a non-empty set of hands in the range 0-7 on which you may, according to partnership understanding, occationally open 1S, then you are playing a HUM. And a partnership understanding to this effect is deemed to exist unless your partner has no more reason to be aware of it than have your opponents. (Law 40) ............. > If your convention card says "1S shows 7+ hcp" then you are playing a HUM > (whatever else it says). If your convention card says "1S shows 8+ hcp" but > in fact you open all (or "almost all") 7-point hands 1S then you are playing > a HUM (and you are trying to conceal the fact). This is correct, and you are using CPU! > > If you do not open all (or "almost all") 7-point hands with a bid at the one > level, then: > > If you open ten 7-counts for every one you pass, then you are playing a HUM This is also correct. > (and you may not escape the charge of playing a concealed HUM by exhibiting > the one seven-count you passed). If you pass ten seven-counts for every one > you open, then you are not playing a HUM (and you may escape the charge of > playing a concealed HUM by exhibiting the ten seven-counts you passed). > > This ratio of ten to one is based on nothing but my own intuition - other > people may have a different figure in mind. But some figure there must be, > or (as Wayne correctly says) the WBF policy on psychic bidding will be > ineffective (since it will be meaningless). The important question is not the ratio between opened and passed hands but if your partner has no more reason to be aware of the possibility (that you open at the one-level with less than 8 HCP) than have your opponents. (Again Law 40) > > [WB] > > I think the way the HUM regulations are written if there exists a hand that > I would open in my methods that is a king or more below average strength > then I am playing a HUM. > > This interpretation comes from the word "may" in the HUM regulation "By > partnership agreement an opening bid at the one level may be made with > values a king or more below average strength." > > If I "may" have 7 hcp and it is part of my partnerships methods then I am > playing a HUM. > > [DALB] > > I agree with you. But the important phrase is "in my methods". We have a > plethora of terms for describing the medium by which partners exchange > information in the bidding: "system"; "methods"; "agreements"; > "understandings" etc. And according to Law 40 they all are contributing to your "partnership understanding" > > Herman has tried to distinguish between "agreements" and "understandings" - > not terribly successfully, but you could see what he was driving at. > > I distinguish, as the WBF policy distinguishes, between the normal meaning > of a call (what the "system" or the "methods" say that a call means) and a > "gross violation" of the normal meaning of a call (an "anti-systemic" call). > > Now, some number (perhaps one in ten, perhaps one in a hundred, perhaps one > in a lifetime depending on the RA) of such "gross violations" are permitted > by policy subject to disclosure as to the nature and frequency of the > violations (because the disclosure means that they are also permitted by > Law). Beyond that number (whatever it is) you are playing a HUM (if you > disclose the nature and frequency of the violations) or you have a concealed > partnership understanding (if you do not disclose the nature and frequency > of the violations). The first may be illegal, the second certainly is. > > Sven believes, if I understand him correctly, that once a particular > violation has occurred, it becomes part of the partnership's > "understandings" (or "agreements", or "methods", or "system", or whatever) > since it is part of their shared experience, and must therefore not only be > disclosed but be subject to regulation. He is right about the first part > (for so says the Law) but wrong about the second part (for so says the > policy: "methods of this kind are permissible in any category of event"). Law 40 requires every partnership to disclose to opponents everything needed so that none of their actions during auction or play are based on knowledge that is concealed from (not disclosed to) opponents. Thus once a violation of understanding has occurred against one opponent it may certainly not be repeated against another opponent without disclosure because partner will now have reason to be aware of this possibility. I suppose it may legally be repeated against the same opponents in the same round because they should now be aware of it. I also suppose it may legally be repeated after sufficient time has elapsed so that partner must be expected to having completely forgotten the incident (at least 6 months or so?????) Regards Sven From henk at amsterdamned.org Sat Nov 1 01:01:00 2008 From: henk at amsterdamned.org (Henk Uijterwaal) Date: Sat, 01 Nov 2008 01:01:00 +0100 Subject: [blml] List of BLML Abbreviations Message-ID: (Automated, regular posting) Usenet Bridge Abbreviations ABF Australian Bridge Federation AC Appeals committee ACBL American Contract Bridge League AI Authorised information ArtAS Artificial adjusted score AssAS Assigned adjusted score ATF Across-the-field [matchpointing] ATTNA Appeal to the National Authority BBL British Bridge League [now defunct] BGB Bridge Great Britain BIT Break in Tempo BLML Bridge-laws mailing list BoD Board of directors [ACBL] BoG Board of governors [ACBL] BOOT Bid-Out-Of-Turn CD Convention Disruption C&E Conduct and ethics [often hearings] CC Convention card CHO Center Hand Opponent [ie partner] CoC Conditions of contest COOT Call-Out-Of-Turn CoP Code of practice CPU Concealed partnership understanding CTD Chief Tournament director DBF Danish Bridge Federation DIC Director in charge DP Disciplinary penalty EBL European Bridge League EBU English Bridge Union EHAA Every Hand an Adventure [a system] F2F Face-to-face [to distinguish from Online bridge] FNJ Fit-Non-Jump (A non-jump bid in a new suit that implies a fit for partner's suit). FOLOOT Faced Opening-Lead-Out-Of-Turn FSF Fourth Suit Forcing GCC General Convention Chart [ACBL] HUM Highly Unusual Method IB Insufficient Bid IBLF International Bridge Laws Forum LA Logical alternative L&EC Laws & Ethics Committee [English, Welsh or Scottish] LHO Left hand Opponent Lnn Law number nn LOL Little old lady [may be of either sex] LOOT Lead-Out-Of-Turn MB Misbid ME Misexplanation MI Misinformation MPC Major penalty card mPC Minor penalty card MSC Master Solvers' Club [The Bridge World] NA National Authority NABC ACBL North American Bridge Championships NBB Nederlandse Bridge Bond [Dutch Bridge League] NBO National Bridge organisation NCBO National Contract Bridge organisation NIBU Northern Ireland Bridge Union NO Non-offender NOs Non-offenders NOS Non-offending side OBM Old Black Magic OBOOT Opening-Bid-Out-Of-Turn OKB OKBridge OLB Online bridge [to distinguish from Face-to-face bridge] OLOOT Opening-Lead-Out-Of-Turn OOT Out-Of-Turn Os Offenders OS Offending side pd Partner PLOOT Play-Out-Of-Turn POOT Pass-Out-Of-Turn PP Procedural Penalty PH Passed Hand RA Regulating Authority RGB rec.games.bridge [newsgroup] RGBO rec.games.bridge.okbridge [newsgroup] RHO Right Hand Opponent RLB Real Life Bridge [to distinguish from Online bridge] RoC Rule of coincidence RoW Rest of World [apart from North America] RTFLB Read the [fabulous] Law book! SAYC Standard American Yellow Card SBU Scottish Bridge Union SO Sponsoring organisation TBW The Bridge World [magazine] TD Tournament director TDic Tournament director in charge TFLB The [fabulous] Law book! UI Unauthorised information UPH UnPassed Hand WBF World Bridge Federation WBFLC WBF Laws Committee WBU Welsh Bridge Union YC Young Chelsea ZO Zonal organisation ZT Zero Tolerance [for unacceptable behaviour] Hand diagrams: 3m 3C or 3D [minor] 3M 3H or 3S [Major] ..3H 3H after a hesitation 3H! 3H alerted Cards and bids: H3 A card (3 of hearts) 3H A bid (3 hearts. The above may also be found on David Stevenson's Bridgepage at http://blakjak.com/usenet_br.htm From henk at amsterdamned.org Sat Nov 1 01:01:01 2008 From: henk at amsterdamned.org (Henk Uijterwaal) Date: Sat, 01 Nov 2008 01:01:01 +0100 Subject: [blml] BLML Usage statistics Message-ID: BLML usage statistics for October 2008 Posts From ----- ---- 116 richard.hills (at) immi.gov.au 75 Hermandw (at) skynet.be 69 agot (at) ulb.ac.be 48 grandaeval (at) tiscali.co.uk 28 ehaa (at) starpower.net 25 rfrick (at) rfrick.info 24 wjburrows (at) gmail.com 21 svenpran (at) online.no 20 john (at) asimere.com 17 jfusselman (at) gmail.com 11 dalburn (at) btopenworld.com 10 nigelguthrie (at) talktalk.net 7 ziffbridge (at) t-online.de 7 ardelm (at) optusnet.com.au 6 sater (at) xs4all.nl 6 brian (at) meadows.pair.com 5 richard.willey (at) gmail.com 5 harald.skjaran (at) gmail.com 5 adam (at) tameware.com 4 PeterEidt (at) t-online.de 4 JffEstrsn (at) aol.com 3 t.kooyman (at) worldonline.nl 2 mfrench1 (at) san.rr.com 2 larry (at) charmschool.orangehome.co.uk 2 henk (at) amsterdamned.org 2 geller (at) nifty.com 2 adam (at) irvine.com 2 Gampas (at) aol.com 1 olivier.beauvillain (at) wanadoo.fr 1 lskelso (at) ihug.com.au 1 jean-pierre.rocafort (at) meteo.fr 1 gerben (at) t-online.de 1 daisy_duck (at) btopenworld.com 1 cibor (at) poczta.fm 1 ccw.in.nc (at) gmail.com 1 bobpark (at) connecttime.net 1 axman22 (at) hotmail.com From grandaeval at tiscali.co.uk Sat Nov 1 00:49:28 2008 From: grandaeval at tiscali.co.uk (Grattan) Date: Fri, 31 Oct 2008 23:49:28 -0000 Subject: [blml] New Laws don't allow UI situation to beresolved? [SEC=UNOFFICIAL] References: <2b1e598b0810302300t4f888261j8d7615f57fcdd4ee@mail.gmail.com><000301c93b33$398b9670$aca2c350$@no> Message-ID: <001e01c93bb5$e7c6e1e0$0302a8c0@Mildred> Grattan Endicott To: "Bridge Laws Mailing List" Sent: Friday, October 31, 2008 1:40 PM Subject: Re: [blml] New Laws don't allow UI situation to beresolved? [SEC=UNOFFICIAL] How does the absence of the word "artificial" from 16C2(c) affect anything? The reason "artificial" appears in 16C2(d) is because in that case, the board has not been played and an assigned score, i.e., "a score [that] replaces the score obtained in play" is impossible because there is none. > +=+ <<"Law 12: When the Director is empowered elsewhere in the laws simply to "award an adjusted score" he refers to Law 12 to determine whether this will be an assigned or an artificial adjusted score. Law 12 intends that whenever he is able to award an assigned adjusted score he does so; if Law 12C1(d) or Law 12C2(a) applies the adjusted score is artificial. Note that 12C2(a) does not say "no result has been obtained" but "no result can be obtained", so that if a board is incomplete but has reached a stage when completion of the board can be foreseen an assigned score is appropriate.">> (WBFLC minute, 10th October 08). ~ Grattan ~ +=+ From rfrick at rfrick.info Sat Nov 1 02:18:36 2008 From: rfrick at rfrick.info (Robert Frick) Date: Fri, 31 Oct 2008 20:18:36 -0500 Subject: [blml] New Laws don't allow UI situation to be resolved? [SEC=UNOFFICIAL] In-Reply-To: <001b01c93b5d$4438f2f0$ccaad8d0$@no> References: <2b1e598b0810302300t4f888261j8d7615f57fcdd4ee@mail.gmail.com> <000301c93b33$398b9670$aca2c350$@no> <001b01c93b5d$4438f2f0$ccaad8d0$@no> Message-ID: On Fri, 31 Oct 2008 08:33:31 -0500, Sven Pran wrote: > On Behalf Of Robert Frick > .............. >> My problem is this. When you are playing a board and all of a sudden >> realize that you have useful UI from an extraneous source (not partner), >> you can't play the board normally. You are essentially asking yourself, >> "What would I do if I didn't have the information that I do have?" >> Sometimes you can answer that question. But often you can't, because you >> DO have the information so can't put yourself in that position. >> >> So normal play of the board is now sometimes impossible. But the >> director >> still has to allow normal play of the board? What does that mean? Does >> it >> mean I can use the UI and if it helps me I get to keep my score and the >> opponents are protected with a split score? >> >> Or is there a way to get from L16C3 to L16C2(d)? L16C3 just directs me >> to >> L16C2(c). >> >> That's the major problem I am having. Another problem is that as I read >> this law, it protects me if I make a play suggested by the UI and it >> turns >> out poorly. (That would actually save me 3 matchpoints from Wednesday.) > > Your duty is to attempt playing the board as if you had no extraneous > information. Right. And there is no problem if I can do that. The problem is when I can't. There is a decision making process and I can't do it not using information I have. > > If the director subsequently finds that you have succeeded in that > attempt Right. But what does this mean? What is the director supposed to do? I decide to play for the drop of the queen of spades, which is suggested by my UI but might have been what I would have done anyway -- it is a reasonable line of play. What does the director do now? > then he should let the result stand, otherwise he should award an > adjusted > score (assigned or artificial at his discretion). Exactly what the laws say! But what score? Everyone does something different and that's okay? If you punish me for taking the line of play suggested by the UI, then I am being punished even though I am not the offending side. If you don't punish me for taking the line of play suggested by the UI, then I am free to use the UI. > You cannot expect to be > protected against a bad result unless that result is a consequence of the > extraneous information received. Do I get protection for this? Following a line of play suggested by the UI, I didn't lead a club. Everyone else led club, giving declarer his ninth trick at 3NT. My declarer had to take a heart finesse for his ninth trick. It won, and then hearts broke 3-3, giving him an overtrick and us a cold bottom. I now want the protection you promised me. From bobpark at connecttime.net Sat Nov 1 02:52:25 2008 From: bobpark at connecttime.net (Robert Park) Date: Fri, 31 Oct 2008 21:52:25 -0400 Subject: [blml] Psychs?? Message-ID: <490BB659.1070404@connecttime.net> Hmmmm...Two years ago I psyched a 1H response to 1S. My partner jumped to 4S, which led to us losing a bundle. As he stormed off that night, I heard him mutter that he'd be damned if he'd ever do that again. So...next week, if I see what seems to be a likely hand for a psychic 1H response to 1S, am I barred from making that bid with this partner? Note that I have no agreement or understanding on this topic with this partner. In fact, we have not even spoken to each other since that unfortunate evening...except to arrange for our game next week. This does seem to raise the question as to whether "understandings" must be mutual. Or can unilateral "understandings" be forced on one? : : Now suppose I suspect my partner reported my erratic behavior to another, whom I will play with next month. Am I forever barred from bidding 1S in reply to that partner's 1H when holding short spades, heart support, and weakish values? --Bob Park From dalburn at btopenworld.com Sat Nov 1 09:41:58 2008 From: dalburn at btopenworld.com (David Burn) Date: Sat, 1 Nov 2008 08:41:58 -0000 Subject: [blml] Mr Gladstone suggested [SEC=UNOFFICIAL] In-Reply-To: <2a1c3a560810311545l386979b3vec0c724581100f66@mail.gmail.com> References: <1062526051-1225456021-cardhu_decombobulator_blackberry.rim.net-888530924-@bxe065.bisx.produk.on.blackberry> <2a1c3a560810311251o1b9ea039j39b200467f77c725@mail.gmail.com> <000001c93ba4$f34d3b10$d9e7b130$@com> <2a1c3a560810311545l386979b3vec0c724581100f66@mail.gmail.com> Message-ID: <000101c93bfd$b41bb850$1c5328f0$@com> [WB] I have two problems with this: 1. There is a huge gulf between 10:1 and 1:10 so this does not tell us where the boundary is. [DALB] That's because I don't know where it is. The experiment one would wish to conduct is this: we take a player and we give him Q32 KJ653 J74 52 and see if he opens the bidding in third position, 40 IMPs behind with 16 boards to go. Then we erase all memory of the incident, give him the same hand again and see if he opens the bidding in third position, 40 IMPs ahead with 16 boards to go. We repeat the process on the first board of the 96-board match. If he opens in the second and third cases, he is playing a HUM. If he only opens in the first case, or opens in the second case only because he thinks they might open at the other table 40 IMPs behind, then he is not playing a HUM. Of course, you can't do that in reality, but that is what I mean when I speak of a player opening all, or almost all, seven counts. I don't mean this: [WB] 2. As stated earlier the word "may" suggests strongly to me that the boundary is "if there exists a 7-count on which you would open" then you are playing a HUM - perhaps one could argue that you require no distributional values. [DALB] I'm not sure about this. If you would open the above 7-count only when 40 IMPs behind with 16 to go, or only when needing a top to qualify for the World Pairs Final, then I don't think you are playing a HUM. I think you are playing a HUM only if your methods or your style mean that your partner would expect you to open the above 7-count more often than, or perhaps as often as, you would pass it. [WB] To me this is absurd. Let's say my explicit agreement is 1H = 8-12 5+ hearts. But over time I open fewer and fewer of those hands with 1H and more and more hands 0-7 any distribution with 1H. Eventually I open no hands with 8-12 and 5+ hearts and all hands with 0-7 any. You seem to say this is ok because it is not an explicit agreement but rather an understanding that has developed from repeated violations. [DALB] If that is what I seem to say, I apologise for having expressed myself badly. I said, or at least I thought I said, that there is a limit to the number of times a particular kind of violation may be repeated before it becomes part not only of the methods that must be disclosed (once is enough for that) but the methods that may be regulated (once may be enough for that in the ABCL, more often than "from time to time" would be enough for the WBF). Reading my own words again: "some number (perhaps one in ten, perhaps one in a hundred, perhaps one in a lifetime depending on the RA) of such "gross violations" are permitted by policy subject to disclosure as to the nature and frequency of the violations (because the disclosure means that they are also permitted by Law). Beyond that number (whatever it is) you are playing a HUM (if you disclose the nature and frequency of the violations)" it is perfectly clear to me that this is in fact what I did say - but of course it would be clear to me. David Burn London, England From dalburn at btopenworld.com Sat Nov 1 10:29:05 2008 From: dalburn at btopenworld.com (David Burn) Date: Sat, 1 Nov 2008 09:29:05 -0000 Subject: [blml] Mr Gladstone suggested [SEC=UNOFFICIAL] In-Reply-To: <000801c93bb0$45467ca0$cfd375e0$@no> References: <1062526051-1225456021-cardhu_decombobulator_blackberry.rim.net-888530924-@bxe065.bisx.produk.on.blackberry> <2a1c3a560810311251o1b9ea039j39b200467f77c725@mail.gmail.com> <000001c93ba4$f34d3b10$d9e7b130$@com> <000801c93bb0$45467ca0$cfd375e0$@no> Message-ID: <000201c93c04$492902d0$db7b0870$@com> [SP] If there exists a non-empty set of hands in the range 0-7 on which you may, according to partnership understanding, occasionally open 1S, then you are playing a HUM. And a partnership understanding to this effect is deemed to exist unless your partner has no more reason to be aware of it than have your opponents. (Law 40). [DALB] That's not quite right, because it means that if I disclose my methods to the opponents, they cease to be understandings between my partner and myself. But if I put on my convention card "we may open a seven count in third seat if a long way behind in a match or if needing a top", then [a] I am telling my opponents nothing they don't know already; [b] I do not believe that I am playing a HUM. If in those circumstances I open: Q32 KJ653 J74 52 I believe I am acting in accordance with the WBF policy on psychic bidding, not contravening its regulations on highly unusual methods. I am playing a HUM only if a pass would explicitly deny this hand (which for Helgemo and Helness it would not). [SP] And according to Law 40 they all are contributing to your "partnership understanding" [DALB] The trouble with the Sven argument, and the reason it is close to nonsensical, is this: once Helgemo has opened a seven count, there always exists the possibility that he might do so again. Now, whether Helness actually remembers the last time Helgemo did it or not is irrelevant; Helgemo and Helness will forever be "playing a HUM" even if they never again open the bidding with fewer than the 10 hcp stated on their convention cards. In the next tournament they play together, they must file their system as a HUM not because they *will* open seven counts, but because one of them once *did* open a seven count. Once is not enough. A HUM is a "method"; that is, it is a set of calls whose "normal meanings" involve opening the bidding at the one level with 0-7, or passing with 12+. If one is not playing a HUM, then an occasional gross violation of the normal meaning of a call (a violation that, if repeated often enough to become part of the "normal meaning" of the call, would make the call part of a HUM) does not mean that one suddenly is playing a HUM. Provided the possibility of such violations is properly disclosed, they are permitted (and no Law is contravened). David Burn London, England From svenpran at online.no Sat Nov 1 11:02:13 2008 From: svenpran at online.no (Sven Pran) Date: Sat, 1 Nov 2008 11:02:13 +0100 Subject: [blml] New Laws don't allow UI situation to be resolved? [SEC=UNOFFICIAL] In-Reply-To: References: <2b1e598b0810302300t4f888261j8d7615f57fcdd4ee@mail.gmail.com> <000301c93b33$398b9670$aca2c350$@no> <001b01c93b5d$4438f2f0$ccaad8d0$@no> Message-ID: <000001c93c08$ea3dbf90$beb93eb0$@no> On Behalf Of Robert Frick .............. > Exactly what the laws say! But what score? Everyone does something > different and that's okay? If you punish me for taking the line of play > suggested by the UI, then I am being punished even though I am not the > offending side. If you don't punish me for taking the line of play > suggested by the UI, then I am free to use the UI. > > > > > You cannot expect to be > > protected against a bad result unless that result is a consequence of the > > extraneous information received. > > Do I get protection for this? Following a line of play suggested by the > UI, I didn't lead a club. Everyone else led club, giving declarer his > ninth trick at 3NT. My declarer had to take a heart finesse for his ninth > trick. It won, and then hearts broke 3-3, giving him an overtrick and us a > cold bottom. I now want the protection you promised me. When the director awards an adjusted score under Law 16C he shall consider both sides as not responsible for the irregularity. So if your selected line of play is reasonable it shall be accepted for your side even if it "could have been suggested" by the UI. However, if you had a real choice between alternatives then the adjusted score awarded to your opponents shall be based on an alternative that would be more favorable for them. Regards Sven. From svenpran at online.no Sat Nov 1 12:24:34 2008 From: svenpran at online.no (Sven Pran) Date: Sat, 1 Nov 2008 12:24:34 +0100 Subject: [blml] Mr Gladstone suggested [SEC=UNOFFICIAL] In-Reply-To: <000201c93c04$492902d0$db7b0870$@com> References: <1062526051-1225456021-cardhu_decombobulator_blackberry.rim.net-888530924-@bxe065.bisx.produk.on.blackberry> <2a1c3a560810311251o1b9ea039j39b200467f77c725@mail.gmail.com> <000001c93ba4$f34d3b10$d9e7b130$@com> <000801c93bb0$45467ca0$cfd375e0$@no> <000201c93c04$492902d0$db7b0870$@com> Message-ID: <000101c93c14$6b165d10$41431730$@no> On Behalf Of David Burn > [SP] > > If there exists a non-empty set of hands in the range 0-7 on which you may, > according to partnership understanding, occasionally open 1S, then you are > playing a HUM. > > And a partnership understanding to this effect is deemed to exist unless > your partner has no more reason to be aware of it than have your opponents. > (Law 40). > > [DALB] > > That's not quite right, because it means that if I disclose my methods to > the opponents, they cease to be understandings between my partner and > myself. WHAT ? ! ? ! Are you saying that what you have disclosed to your opponents no longer are understandings between you and your partner? You can't mean that? (They cease to be CONCEALED partnership understandings) > > But if I put on my convention card "we may open a seven count in third seat > if a long way behind in a match or if needing a top", then [a] I am telling > my opponents nothing they don't know already; [b] I do not believe that I am > playing a HUM. You are indeed playing a HUM if you have on your declaration that you can open a seven count with a bid at the one-level. It doesn't matter if that possibility is restricted to when you are way behind in a match, third seat or anything. If in those circumstances I open: > > Q32 KJ653 J74 52 > > I believe I am acting in accordance with the WBF policy on psychic bidding, > not contravening its regulations on highly unusual methods. I am playing a > HUM only if a pass would explicitly deny this hand (which for Helgemo and > Helness it would not). > > [SP] > > And according to Law 40 they all are contributing to your "partnership > understanding" > > [DALB] > > The trouble with the Sven argument, and the reason it is close to > nonsensical, is this: once Helgemo has opened a seven count, there always > exists the possibility that he might do so again. Now, whether Helness > actually remembers the last time Helgemo did it or not is irrelevant; > Helgemo and Helness will forever be "playing a HUM" even if they never again > open the bidding with fewer than the 10 hcp stated on their convention > cards. In the next tournament they play together, they must file their > system as a HUM not because they *will* open seven counts, but because one > of them once *did* open a seven count. > > Once is not enough. A HUM is a "method"; that is, it is a set of calls whose > "normal meanings" involve opening the bidding at the one level with 0-7, or > passing with 12+. If one is not playing a HUM, then an occasional gross > violation of the normal meaning of a call (a violation that, if repeated > often enough to become part of the "normal meaning" of the call, would make > the call part of a HUM) does not mean that one suddenly is playing a HUM. > Provided the possibility of such violations is properly disclosed, they are > permitted (and no Law is contravened). With all these argumentations please remember (and stick to) the basic criteria: "If your partner has more reason to be aware of the situation than have your opponents" you are on thin ice with respect of Law 40 If your declared system is in any way HUM then it is illegal where HUM is forbidden. (And don't mix the two; Law 40 does not concern itself with HUM) Sven From Hermandw at skynet.be Sat Nov 1 13:09:03 2008 From: Hermandw at skynet.be (Herman De Wael) Date: Sat, 01 Nov 2008 13:09:03 +0100 Subject: [blml] New Laws don't allow UI situation to be resolved? In-Reply-To: References: <2b1e598b0810302300t4f888261j8d7615f57fcdd4ee@mail.gmail.com> <000301c93b33$398b9670$aca2c350$@no> <1Kvstx-20Zhi40@fwd08.aul.t-online.de> Message-ID: <490C46DF.5010902@skynet.be> Yes there is. Eric Landau wrote: > > L16C1 obligates a player to call the Director forthwith as soon as he > realizes that he has "accidentally receive[d] unauthorized > information about a board he is playing or has yet to play". It does > not, for obvious reasons, concern itself with receipt of information > about a board the recipient has already played, or will not play. > Overhearing random extraneous information does not obligate the > player to call the TD unless and until he can connect it to "a board > he is playing or has yet to play". That this may mean that the TD is > not called until the hand to which the extraneous information > pertains has already begun does not change the fact that the player > "receive[d] unauthorized information about a board he is [now] > playing... before the auction beg[an]". There is no reason why L16C > should not apply. > Yes there is - since two of the options are no longer available. > > Eric Landau > 1107 Dale Drive > Silver Spring MD 20910 > ehaa at starpower.net > > > > > > _______________________________________________ > blml mailing list > blml at amsterdamned.org > http://www.amsterdamned.org/mailman/listinfo/blml > From Hermandw at skynet.be Sat Nov 1 13:15:11 2008 From: Hermandw at skynet.be (Herman De Wael) Date: Sat, 01 Nov 2008 13:15:11 +0100 Subject: [blml] Mr Gladstone suggested [SEC=UNOFFICIAL] In-Reply-To: <000301c93b8c$97769100$c663b300$@no> References: <1062526051-1225456021-cardhu_decombobulator_blackberry.rim.net-888530924-@bxe065.bisx.produk.on.blackberry> <001a01c93b5c$0393aed0$0abb0c70$@no> <490B1506.4050408@skynet.be> <000301c93b8c$97769100$c663b300$@no> Message-ID: <490C484F.5040906@skynet.be> No Sven. Sven Pran wrote: >>> >> The call may still qualify as a psyche, and it will also be MI. > > Law 40C1 begins with: "A player may deviate from his side's announced > understandings always provided that his partner has no more reason to be > aware of the deviation than have the opponents." > > The consequence of this (and of the remainder of L40C1) is that IF the > partner in question HAS more reason to be aware of the deviation than have > their opponents then the call in question is not a (legal) psyche; it is an > (illegal) concealed partnership understanding. > No Sven. If the partner has more reason then this is not a legal donkey but an illegal one. It is still a donkey. And the legality is the same as if Stayman had not been alerted. MI. There is no such crime as concealed partnership understanding. There is only one crime, and I prefer to call it misinformation. The fact that partner knows something about the donkey does not alter the fact that it is a donkey. When that knowledge is not shared with opponents, there is MI. But the donkey remains a donkey. > But of course the partnership can for instance declare that their opening > bid of 1H, while normally showing 11-19 HCP occasionally can be made with > only 7 HCP. Then this "deviation" will no longer be a psyche, it will be > part of the announced partnership understandings. Their only problem with > such a declaration will in most events be that their system is deemed HUM. > And that is precisely what I am trying to make you see is not true. There is a middle way, with the psyche being still a psyche but the understandings about it revealed to opponents. If there is no middle way, then there are no psyches either - since all psyches carry some amount of partnership understanding. > Sven. > Herman. From Hermandw at skynet.be Sat Nov 1 13:17:35 2008 From: Hermandw at skynet.be (Herman De Wael) Date: Sat, 01 Nov 2008 13:17:35 +0100 Subject: [blml] Mr Gladstone suggested In-Reply-To: <82B397CE-E49C-46F0-84DF-A372659037EB@starpower.net> References: <1429874746-1225461820-cardhu_decombobulator_blackberry.rim.net-2091802381-@bxe065.bisx.produk.on.blackberry> <82B397CE-E49C-46F0-84DF-A372659037EB@starpower.net> Message-ID: <490C48DF.6020508@skynet.be> Eric Landau wrote: > > That works, but only if we're careful what we mean by it. It says > that a partnership may have an understanding that a call which > usually carries its "normal" meaning may carry a second, "grossly > deviant", meaning, provided that the deviation does not occur with a > frequency greater than X. But if you permit "per the understanding" > use of the deviation, you cannot prohibit the pair from making the > call with the deviant meaning whenever the understanding calls for it. > > Which means that X cannot be the empirical relative frequency of the > deviation over any given period of time. It must be the expected > value of the relative frequency over the set of all possible deals, > which, unlike the empirical frequency, is an intrinsic property of > the understanding. IOW, X can only determine *a priori* whether or > not the understanding to deviate from the normal meaning is legal; if > it is, it may be done whenever the understood conditions for it are > met, regardless of how often it may actually occur over the course of > a session, a tournament, a year, or a lifetime. > I do not particularly like the idea of a set value of X, but I do concur completely with what Eric says. Provided the hands are infrequent, one should be allowed to psyche on those hands whenever they appear. If they appear with a greater frequency than expected, a player is allowed to psyche more often than X. > > Eric Landau > 1107 Dale Drive > Silver Spring MD 20910 > ehaa at starpower.net > Herman. From Hermandw at skynet.be Sat Nov 1 13:36:53 2008 From: Hermandw at skynet.be (Herman De Wael) Date: Sat, 01 Nov 2008 13:36:53 +0100 Subject: [blml] Mr Gladstone suggested [SEC=UNOFFICIAL] In-Reply-To: <000001c93ba4$f34d3b10$d9e7b130$@com> References: <1062526051-1225456021-cardhu_decombobulator_blackberry.rim.net-888530924-@bxe065.bisx.produk.on.blackberry> <2a1c3a560810311251o1b9ea039j39b200467f77c725@mail.gmail.com> <000001c93ba4$f34d3b10$d9e7b130$@com> Message-ID: <490C4D65.5090307@skynet.be> David, as usually, makes a lot of sense. However: David Burn wrote: > [WB] > > If I open 1S to show 0-7 any shape then this is a HUM. > > If I open 1S to show "some 0-7 any shape" then are you saying this is not a > HUM? > > [DALB] > > If there exists a non-empty set of hands in the range 0-7 on which you would > always open 1S, then you are playing a HUM. I am not sure whether that is > what you just asked, but as I said, if Helgemo and Helness open all (or > "almost all") seven-point hands with five hearts 1H in third position, then > they are playing a HUM. > > [WB] > > That is if my range includes all 7 counts then you say it is a HUM. > > But if not I can demonstrate even one 7 count that I have not opened then it > is allowable. > > [DALB] > > I am not quite sure what this means: there may be a superfluous "not" in > there somewhere. I will outline my view below. > > If your convention card says "1S shows 7+ hcp" then you are playing a HUM > (whatever else it says). If your convention card says "1S shows 8+ hcp" but > in fact you open all (or "almost all") 7-point hands 1S then you are playing > a HUM (and you are trying to conceal the fact). > Indeed. > If you do not open all (or "almost all") 7-point hands with a bid at the one > level, then: > > If you open ten 7-counts for every one you pass, then you are playing a HUM > (and you may not escape the charge of playing a concealed HUM by exhibiting > the one seven-count you passed). If you pass ten seven-counts for every one > you open, then you are not playing a HUM (and you may escape the charge of > playing a concealed HUM by exhibiting the ten seven-counts you passed). > The problem with this argument is this: H&H may well open only one in ten 7-counts, but they open one in three 7-counts with a singleton next to the five card, and 95% of those where the seven points are constituted of KQ in a three-card suit and five to the Q10. If you only count the 7 points, HH fall below David's threshold, but when you look more closely, they are way above that same threshold. The argument works in the other direction too. David is obviously only thinking of third hand openings, with five hearts. But if one considers all the 7-counts that Helgemo is dealt, he has opened maybe 0.001% of them. > This ratio of ten to one is based on nothing but my own intuition - other > people may have a different figure in mind. But some figure there must be, > or (as Wayne correctly says) the WBF policy on psychic bidding will be > ineffective (since it will be meaningless). > That one is true, but as shown, X can only depend on absolute frequencies, not on relative ones. > [WB] > > I think the way the HUM regulations are written if there exists a hand that > I would open in my methods that is a king or more below average strength > then I am playing a HUM. > > This interpretation comes from the word "may" in the HUM regulation "By > partnership agreement an opening bid at the one level may be made with > values a king or more below average strength." > > If I "may" have 7 hcp and it is part of my partnerships methods then I am > playing a HUM. > > [DALB] > > I agree with you. But the important phrase is "in my methods". We have a > plethora of terms for describing the medium by which partners exchange > information in the bidding: "system"; "methods"; "agreements"; > "understandings" etc. > > Herman has tried to distinguish between "agreements" and "understandings" - > not terribly successfully, but you could see what he was driving at. > Thank you, David. > I distinguish, as the WBF policy distinguishes, between the normal meaning > of a call (what the "system" or the "methods" say that a call means) and a > "gross violation" of the normal meaning of a call (an "anti-systemic" call). > Different words, same idea. > Now, some number (perhaps one in ten, perhaps one in a hundred, perhaps one > in a lifetime depending on the RA) of such "gross violations" are permitted > by policy subject to disclosure as to the nature and frequency of the > violations (because the disclosure means that they are also permitted by > Law). Beyond that number (whatever it is) you are playing a HUM (if you > disclose the nature and frequency of the violations) or you have a concealed > partnership understanding (if you do not disclose the nature and frequency > of the violations). The first may be illegal, the second certainly is. > Different words, same idea. > Sven believes, if I understand him correctly, that once a particular > violation has occurred, it becomes part of the partnership's > "understandings" (or "agreements", or "methods", or "system", or whatever) > since it is part of their shared experience, and must therefore not only be > disclosed but be subject to regulation. He is right about the first part > (for so says the Law) but wrong about the second part (for so says the > policy: "methods of this kind are permissible in any category of event"). > > Herman believes, if I understand him correctly, that the "system" or set of > "agreements" of which a psyche is a "gross violation" is something explicit > and immutable that is unaffected by experience of deviation therefrom. This > is ridiculous, of course, but it embodies at least the sensible notion that > a partnership may psyche more often than once in its existence. > You understand me correctly. Although why it should be immutable is beyond me. > I don't know what Richard believes. I think it had something to do with > elephants. > > David Burn > London, England > David forgets one other thing - the "gross" part. If HH open one in ten 7-counts, one in two 8-counts, 90% of 9-counts and all 10-counts, then the 7 is not a gross violation of the method. Rather, it means that HH employ a different method of hand valuation, in which 10% of seven counts pass the test. That system of hand evaluation is HUM, since it permits the opening of some hands that do not conform to the norm that was set. Herman. From richard.willey at gmail.com Sat Nov 1 13:40:19 2008 From: richard.willey at gmail.com (richard willey) Date: Sat, 1 Nov 2008 08:40:19 -0400 Subject: [blml] Mr Gladstone suggested In-Reply-To: <490C48DF.6020508@skynet.be> References: <1429874746-1225461820-cardhu_decombobulator_blackberry.rim.net-2091802381-@bxe065.bisx.produk.on.blackberry> <82B397CE-E49C-46F0-84DF-A372659037EB@starpower.net> <490C48DF.6020508@skynet.be> Message-ID: <2da24b8e0811010540u2ffd59a8l59caa55fa9996a48@mail.gmail.com> On Sat, Nov 1, 2008 at 8:17 AM, Herman De Wael wrote: > Eric Landau wrote: >> >> That works, but only if we're careful what we mean by it. It says >> that a partnership may have an understanding that a call which >> usually carries its "normal" meaning may carry a second, "grossly >> deviant", meaning, provided that the deviation does not occur with a >> frequency greater than X. But if you permit "per the understanding" >> use of the deviation, you cannot prohibit the pair from making the >> call with the deviant meaning whenever the understanding calls for it. >> >> Which means that X cannot be the empirical relative frequency of the >> deviation over any given period of time. It must be the expected >> value of the relative frequency over the set of all possible deals, >> which, unlike the empirical frequency, is an intrinsic property of >> the understanding. IOW, X can only determine *a priori* whether or >> not the understanding to deviate from the normal meaning is legal; if >> it is, it may be done whenever the understood conditions for it are >> met, regardless of how often it may actually occur over the course of >> a session, a tournament, a year, or a lifetime. >> > > I do not particularly like the idea of a set value of X, but I do concur > completely with what Eric says. Provided the hands are infrequent, one > should be allowed to psyche on those hands whenever they appear. If they > appear with a greater frequency than expected, a player is allowed to > psyche more often than X. Once again: Notice how much easier life becomes if the regulatory system incorporates the idea of a mixed strategy rather than streaching the concept of a psyche to attempt to describe a very different set of behaviors. -- I think back to the halcyon dates of my youth, when indeterminant Hessians had something to do with the Revolutionary War, where conjugate priors were monks who had broken their vows, and the expression (X'X)^-1(X'Y) was greek Those were simpler times From svenpran at online.no Sat Nov 1 16:46:16 2008 From: svenpran at online.no (Sven Pran) Date: Sat, 1 Nov 2008 16:46:16 +0100 Subject: [blml] Mr Gladstone suggested [SEC=UNOFFICIAL] In-Reply-To: <490C484F.5040906@skynet.be> References: <1062526051-1225456021-cardhu_decombobulator_blackberry.rim.net-888530924-@bxe065.bisx.produk.on.blackberry> <001a01c93b5c$0393aed0$0abb0c70$@no> <490B1506.4050408@skynet.be> <000301c93b8c$97769100$c663b300$@no> <490C484F.5040906@skynet.be> Message-ID: <000001c93c38$f9dfd200$ed9f7600$@no> On Behalf Of Herman De Wael ............ > > Law 40C1 begins with: "A player may deviate from his side's announced > > understandings always provided that his partner has no more reason to be > > aware of the deviation than have the opponents." > > > > The consequence of this (and of the remainder of L40C1) is that IF the > > partner in question HAS more reason to be aware of the deviation than have > > their opponents then the call in question is not a (legal) psyche; it is an > > (illegal) concealed partnership understanding. > > > > No Sven. If the partner has more reason then this is not a legal donkey > but an illegal one. It is still a donkey. And the legality is the same > as if Stayman had not been alerted. MI. > > There is no such crime as concealed partnership understanding. > There is only one crime, and I prefer to call it misinformation. > > The fact that partner knows something about the donkey does not alter > the fact that it is a donkey. When that knowledge is not shared with > opponents, there is MI. But the donkey remains a donkey. Why do you not accept Law 40 as it is written? I shall admit you one detail: There is no such thing as "concealed partnership understanding" written in the laws, the laws use the term "undisclosed partnership understanding". Now please do me a favor and explain why the two terms are not synonymous. The go along and explain why Law 40 shall not apply as written. And finally you could explain how a call can be "a deliberate and gross misstatement of honour strength and/or of suit length" once it corresponds with an understanding that has been disclosed. Against what should there be a misstatement if not the disclosed understandings? Sven From rfrick at rfrick.info Sat Nov 1 18:29:37 2008 From: rfrick at rfrick.info (Robert Frick) Date: Sat, 01 Nov 2008 12:29:37 -0500 Subject: [blml] New Laws don't allow UI situation to be resolved? In-Reply-To: <000001c93c08$ea3dbf90$beb93eb0$@no> References: <2b1e598b0810302300t4f888261j8d7615f57fcdd4ee@mail.gmail.com> <000301c93b33$398b9670$aca2c350$@no> <001b01c93b5d$4438f2f0$ccaad8d0$@no> <000001c93c08$ea3dbf90$beb93eb0$@no> Message-ID: On Sat, 01 Nov 2008 05:02:13 -0500, Sven Pran wrote: > On Behalf Of Robert Frick > .............. >> Exactly what the laws say! But what score? Everyone does something >> different and that's okay? If you punish me for taking the line of play >> suggested by the UI, then I am being punished even though I am not the >> offending side. If you don't punish me for taking the line of play >> suggested by the UI, then I am free to use the UI. >> >> >> >> > You cannot expect to be >> > protected against a bad result unless that result is a consequence of > the >> > extraneous information received. >> >> Do I get protection for this? Following a line of play suggested by the >> UI, I didn't lead a club. Everyone else led club, giving declarer his >> ninth trick at 3NT. My declarer had to take a heart finesse for his >> ninth >> trick. It won, and then hearts broke 3-3, giving him an overtrick and >> us a >> cold bottom. I now want the protection you promised me. > > When the director awards an adjusted score under Law 16C he shall > consider > both sides as not responsible for the irregularity. > > So if your selected line of play is reasonable it shall be accepted for > your > side even if it "could have been suggested" by the UI. > > However, if you had a real choice between alternatives then the adjusted > score awarded to your opponents shall be based on an alternative that > would > be more favorable for them. > > Regards Sven. That is clear for what I do as director. Thank you. As player, if I cannot know what my action would have been without the UI, is it ethical to select the choice suggested by the UI? I guess it has to be. From svenpran at online.no Sat Nov 1 18:45:15 2008 From: svenpran at online.no (Sven Pran) Date: Sat, 1 Nov 2008 18:45:15 +0100 Subject: [blml] New Laws don't allow UI situation to be resolved? In-Reply-To: References: <2b1e598b0810302300t4f888261j8d7615f57fcdd4ee@mail.gmail.com> <000301c93b33$398b9670$aca2c350$@no> <001b01c93b5d$4438f2f0$ccaad8d0$@no> <000001c93c08$ea3dbf90$beb93eb0$@no> Message-ID: <000101c93c49$99440220$cbcc0660$@no> On Behalf Of Robert Frick ........................... > > When the director awards an adjusted score under Law 16C he shall > > consider > > both sides as not responsible for the irregularity. > > > > So if your selected line of play is reasonable it shall be accepted for > > your > > side even if it "could have been suggested" by the UI. > > > > However, if you had a real choice between alternatives then the adjusted > > score awarded to your opponents shall be based on an alternative that > > would > > be more favorable for them. > > > > Regards Sven. > > > That is clear for what I do as director. Thank you. > > As player, if I cannot know what my action would have been without the UI, > is it ethical to select the choice suggested by the UI? I guess it has to > be. An ethical player should IMHO NOT select an action suggested by UI received from "other sources" unless he would seriously consider selecting that alternative without the UI. If he does then I realize that it will be impossible for the director to "prove" it so I suppose that this will be a matter for his own conscience. As a director I shall however still be alert on the possibility of a "double shot": He selects an alternative that demonstrably is suggested by the UI while another alternative could be more attractive without the UI, and if his selected alternative fails he then asks for redress. Regards Sven From rfrick at rfrick.info Sat Nov 1 20:21:59 2008 From: rfrick at rfrick.info (Robert Frick) Date: Sat, 01 Nov 2008 14:21:59 -0500 Subject: [blml] New Laws don't allow UI situation to be resolved? In-Reply-To: <000101c93c49$99440220$cbcc0660$@no> References: <2b1e598b0810302300t4f888261j8d7615f57fcdd4ee@mail.gmail.com> <000301c93b33$398b9670$aca2c350$@no> <001b01c93b5d$4438f2f0$ccaad8d0$@no> <000001c93c08$ea3dbf90$beb93eb0$@no> <000101c93c49$99440220$cbcc0660$@no> Message-ID: On Sat, 01 Nov 2008 12:45:15 -0500, Sven Pran wrote: > On Behalf Of Robert Frick > ........................... >> > When the director awards an adjusted score under Law 16C he shall >> > consider >> > both sides as not responsible for the irregularity. >> > >> > So if your selected line of play is reasonable it shall be accepted >> for >> > your >> > side even if it "could have been suggested" by the UI. >> > >> > However, if you had a real choice between alternatives then the >> adjusted >> > score awarded to your opponents shall be based on an alternative that >> > would >> > be more favorable for them. >> > >> > Regards Sven. >> >> >> That is clear for what I do as director. Thank you. >> >> As player, if I cannot know what my action would have been without the >> UI, >> is it ethical to select the choice suggested by the UI? I guess it has >> to >> be. > > An ethical player should IMHO NOT select an action suggested by UI > received > from "other sources" unless he would seriously consider selecting that > alternative without the UI. > > If he does then I realize that it will be impossible for the director to > "prove" it so I suppose that this will be a matter for his own > conscience. If partner supplies the UI, I understand that I cannot and should not choose an LA suggested by the UI. But if the UI is extraneous and it is not my fault that I have it, then it does not seem fair if I am ethically obligated not to select alternatives suggested by the UI. From Hermandw at skynet.be Sat Nov 1 20:25:59 2008 From: Hermandw at skynet.be (Herman De Wael) Date: Sat, 01 Nov 2008 20:25:59 +0100 Subject: [blml] Mr Gladstone suggested [SEC=UNOFFICIAL] In-Reply-To: <000001c93c38$f9dfd200$ed9f7600$@no> References: <1062526051-1225456021-cardhu_decombobulator_blackberry.rim.net-888530924-@bxe065.bisx.produk.on.blackberry> <001a01c93b5c$0393aed0$0abb0c70$@no> <490B1506.4050408@skynet.be> <000301c93b8c$97769100$c663b300$@no> <490C484F.5040906@skynet.be> <000001c93c38$f9dfd200$ed9f7600$@no> Message-ID: <490CAD47.8050907@skynet.be> Sven Pran wrote: >> >> There is no such crime as concealed partnership understanding. >> There is only one crime, and I prefer to call it misinformation. >> >> The fact that partner knows something about the donkey does not alter >> the fact that it is a donkey. When that knowledge is not shared with >> opponents, there is MI. But the donkey remains a donkey. > > Why do you not accept Law 40 as it is written? > Why don't you? But that's not fair. > I shall admit you one detail: There is no such thing as "concealed > partnership understanding" written in the laws, the laws use the term > "undisclosed partnership understanding". > Indeed they do. It's the crime I call MI in short. You OTOH, by using the words CPU in a context where this sounds very "wrong", create the impression that this is a different crime altogether. It isn't. So can we agree on this thing: if the frequency of the psyche is not disclosed to the opponents, then the pair are guilty of MI. The rectification procedure is exactly the same as for a missing alert: we investigate how the auction and play would have gone with complete information and we award an AS on that basis. Is that what you believe as well? > Now please do me a favor and explain why the two terms are not synonymous. > > The go along and explain why Law 40 shall not apply as written. > Of course it shall. L40B4. AS. > And finally you could explain how a call can be "a deliberate and gross > misstatement of honour strength and/or of suit length" once it corresponds > with an understanding that has been disclosed. Against what should there be > a misstatement if not the disclosed understandings? > Indeed. But not all the understandings - not those that deal with the psyche itself, and its frequency. > Sven > Herman. From svenpran at online.no Sat Nov 1 20:26:25 2008 From: svenpran at online.no (Sven Pran) Date: Sat, 1 Nov 2008 20:26:25 +0100 Subject: [blml] New Laws don't allow UI situation to be resolved? In-Reply-To: References: <2b1e598b0810302300t4f888261j8d7615f57fcdd4ee@mail.gmail.com> <000301c93b33$398b9670$aca2c350$@no> <001b01c93b5d$4438f2f0$ccaad8d0$@no> <000001c93c08$ea3dbf90$beb93eb0$@no> <000101c93c49$99440220$cbcc0660$@no> Message-ID: <000201c93c57$bb5d1230$32173690$@no> On Behalf Of Robert Frick > > On Behalf Of Robert Frick > > ........................... > >> > When the director awards an adjusted score under Law 16C he shall > >> > consider > >> > both sides as not responsible for the irregularity. > >> > > >> > So if your selected line of play is reasonable it shall be accepted > >> for > >> > your > >> > side even if it "could have been suggested" by the UI. > >> > > >> > However, if you had a real choice between alternatives then the > >> adjusted > >> > score awarded to your opponents shall be based on an alternative that > >> > would > >> > be more favorable for them. > >> > > >> > Regards Sven. > >> > >> > >> That is clear for what I do as director. Thank you. > >> > >> As player, if I cannot know what my action would have been without the > >> UI, > >> is it ethical to select the choice suggested by the UI? I guess it has > >> to > >> be. > > > > An ethical player should IMHO NOT select an action suggested by UI > > received > > from "other sources" unless he would seriously consider selecting that > > alternative without the UI. > > > > If he does then I realize that it will be impossible for the director to > > "prove" it so I suppose that this will be a matter for his own > > conscience. > > If partner supplies the UI, I understand that I cannot and should not > choose an LA suggested by the UI. But if the UI is extraneous and it is > not my fault that I have it, then it does not seem fair if I am ethically > obligated not to select alternatives suggested by the UI. There is a major difference between extraneous information from your partner and extraneous information from other sources. Regarding extraneous information from your partner it does not matter if you are "innocent" or not, you are not allowed to select among alternatives any alternative that "could have been suggested" by the UI even if that alternative is the one you would have selected anyway. There is no such strong restriction when you have extraneous information (UI) from other sources but you are still supposed to act to the best of your ability not taking advantage of the UI. Regards Sven From grandaeval at tiscali.co.uk Sat Nov 1 20:47:29 2008 From: grandaeval at tiscali.co.uk (Grattan) Date: Sat, 1 Nov 2008 19:47:29 -0000 Subject: [blml] New Laws don't allow UI situation to be resolved? References: <2b1e598b0810302300t4f888261j8d7615f57fcdd4ee@mail.gmail.com><000301c93b33$398b9670$aca2c350$@no><001b01c93b5d$4438f2f0$ccaad8d0$@no><000001c93c08$ea3dbf90$beb93eb0$@no><000101c93c49$99440220$cbcc0660$@no> Message-ID: <001a01c93c5b$6d020830$0302a8c0@Mildred> Grattan Endicott To: "Bridge Laws Mailing List" Sent: Saturday, November 01, 2008 7:21 PM Subject: Re: [blml] New Laws don't allow UI situation to be resolved? If partner supplies the UI, I understand that I cannot and should not choose an LA suggested by the UI. But if the UI is extraneous and it is not my fault that I have it, then it does not seem fair if I am ethically obligated not to select alternatives suggested by the UI. > +=+ Perhaps Robert should look again at Law 16A3. It is not a matter of ethics that he is addressing but a matter of law. ~ Grattan ~ +=+ From grandaeval at tiscali.co.uk Sat Nov 1 20:50:16 2008 From: grandaeval at tiscali.co.uk (Grattan) Date: Sat, 1 Nov 2008 19:50:16 -0000 Subject: [blml] New Laws don't allow UI situation to be resolved? References: <2b1e598b0810302300t4f888261j8d7615f57fcdd4ee@mail.gmail.com> <000301c93b33$398b9670$aca2c350$@no> <001b01c93b5d$4438f2f0$ccaad8d0$@no> <000001c93c08$ea3dbf90$beb93eb0$@no> <000101c93c49$99440220$cbcc0660$@no> <000201c93c57$bb5d1230$32173690$@no> Message-ID: <001b01c93c5b$6d44b680$0302a8c0@Mildred> Grattan Endicott To: "'Bridge Laws Mailing List'" Sent: Saturday, November 01, 2008 7:26 PM Subject: Re: [blml] New Laws don't allow UI situation to be resolved? > There is no such strong restriction when you have extraneous information (UI) from other sources but you are still supposed to act to the best of your ability not taking advantage of the UI. > +=+ But what of Law 16A3? +=+ From rfrick at rfrick.info Sat Nov 1 22:18:47 2008 From: rfrick at rfrick.info (Robert Frick) Date: Sat, 01 Nov 2008 16:18:47 -0500 Subject: [blml] New Laws don't allow UI situation to be resolved? In-Reply-To: <000201c93c57$bb5d1230$32173690$@no> References: <2b1e598b0810302300t4f888261j8d7615f57fcdd4ee@mail.gmail.com> <000301c93b33$398b9670$aca2c350$@no> <001b01c93b5d$4438f2f0$ccaad8d0$@no> <000001c93c08$ea3dbf90$beb93eb0$@no> <000101c93c49$99440220$cbcc0660$@no> <000201c93c57$bb5d1230$32173690$@no> Message-ID: On Sat, 01 Nov 2008 14:26:25 -0500, Sven Pran wrote: > There is no such strong restriction when you have extraneous information > (UI) from other sources but you are still supposed to act to the best of > your ability not taking advantage of the UI. Right, that's the law, but it isn't possible. Roughly, human beings make decisions without knowing what influences their decisions and without being able to control what influences their decisions. The Muller-Lyer illusion is a simple example, but it will go all the way to voting for president. So, if you aren't aware that you are influenced, you will be influenced. You will pick the line with the lines flanking outwards, you will vote for the man who is taller, has a deeper voice, or is more handsome; you will decide on the alternative suggested by the UI. If you know that you are influenced, you are lost, because you don't know the size of the influence. You cannot know which line is longer, you cannot know which person you would have voted for, and you cannot know what you would have selected without the UI. So you go to the director, and explain that it is impossible to decide, and the director says do your best. Which I have already tried to explain is impossible. You have said that if I use the UI in these situations, I will get to keep my result. I am not fond of situations that let a Probst cheat prosper, but okay. But then I thought you were saying I had an ethical obligation not to select alternatives suggested by the UI. That isn't fair, because then I am being treated like I was the offending side. The other choice is that it is ethical for me to use the UI. But I am not fond of that either. From brian at meadows.pair.com Sat Nov 1 22:30:08 2008 From: brian at meadows.pair.com (brian) Date: Sat, 01 Nov 2008 17:30:08 -0400 Subject: [blml] New Laws don't allow UI situation to be resolved? In-Reply-To: <001a01c93c5b$6d020830$0302a8c0@Mildred> References: <2b1e598b0810302300t4f888261j8d7615f57fcdd4ee@mail.gmail.com><000301c93b33$398b9670$aca2c350$@no><001b01c93b5d$4438f2f0$ccaad8d0$@no><000001c93c08$ea3dbf90$beb93eb0$@no><000101c93c49$99440220$cbcc0660$@no> <001a01c93c5b$6d020830$0302a8c0@Mildred> Message-ID: <490CCA60.5060607@meadows.pair.com> Grattan wrote: > > From: "Robert Frick" > > If partner supplies the UI, I understand that I cannot and should not > choose an LA suggested by the UI. But if the UI is extraneous and it is > not my fault that I have it, then it does not seem fair if I am ethically > obligated not to select alternatives suggested by the UI. > +=+ Perhaps Robert should look again at Law 16A3. It is not a > matter of ethics that he is addressing but a matter of law. I hope that I've misunderstood something. Might we consider an example? Say that I hear a loud remark from another table "You should have made that grand, the Queen always lies over the Jack!" I have no idea which board they have just played, nor even whether I'm scheduled to meet it. Some boards later, I open a strong 1NT, partner looks at me in amazement and puts me straight into 7NT (Well, my partner would, anyway!). When partner tables the dummy, I see that I do have to find a missing Queen. Pard S AKQ H Jxx D AJ975 C AK Me S Jx H AKQ D K10864 C QJx Are you telling me that I am obliged by Law 16A3 to start on diamonds by playing a small one to the King, and almost certainly go off in an otherwise 50/50 grand, because of something which I unintentionally overheard from another table? I'm with Robert on this one, it seems grossly unfair to me. Maybe we should all turn up wearing earplugs? Brian. From svenpran at online.no Sat Nov 1 22:57:27 2008 From: svenpran at online.no (Sven Pran) Date: Sat, 1 Nov 2008 22:57:27 +0100 Subject: [blml] Mr Gladstone suggested [SEC=UNOFFICIAL] In-Reply-To: <490CAD47.8050907@skynet.be> References: <1062526051-1225456021-cardhu_decombobulator_blackberry.rim.net-888530924-@bxe065.bisx.produk.on.blackberry> <001a01c93b5c$0393aed0$0abb0c70$@no> <490B1506.4050408@skynet.be> <000301c93b8c$97769100$c663b300$@no> <490C484F.5040906@skynet.be> <000001c93c38$f9dfd200$ed9f7600$@no> <490CAD47.8050907@skynet.be> Message-ID: <000301c93c6c$d4854c90$7d8fe5b0$@no> On Behalf Of Herman De Wael ............. > > Why do you not accept Law 40 as it is written? > > > > Why don't you? > But that's not fair. Do you accept law 40? You definitely argue against it. > > > I shall admit you one detail: There is no such thing as "concealed > > partnership understanding" written in the laws, the laws use the term > > "undisclosed partnership understanding". I withdraw this admission, take a look at Law 73E > > > > Indeed they do. It's the crime I call MI in short. There is a difference between MI and CPU; MI is accidental while CPU is deliberate. > You OTOH, by using the words CPU in a context where this sounds very > "wrong", create the impression that this is a different crime > altogether. It isn't. > > So can we agree on this thing: if the frequency of the psyche is not > disclosed to the opponents, then the pair are guilty of MI. There is only one criterion for judging a psyche and that is given in Law 40C1 > > The rectification procedure is exactly the same as for a missing alert: > we investigate how the auction and play would have gone with complete > information and we award an AS on that basis. > > Is that what you believe as well? > > > Now please do me a favor and explain why the two terms are not synonymous. > > > > The go along and explain why Law 40 shall not apply as written. > > > > Of course it shall. L40B4. AS. Are you kidding? > > > And finally you could explain how a call can be "a deliberate and gross > > misstatement of honour strength and/or of suit length" once it corresponds > > with an understanding that has been disclosed. Against what should there be > > a misstatement if not the disclosed understandings? > > > > Indeed. But not all the understandings - not those that deal with the > psyche itself, and its frequency. Where in the laws do you find any basis for this claim? There is one single good thing about this discussion: You have made it perfectly clear when you make up your own laws instead of adhering to those issued by WBFLC. No sense in fighting the windmill named HdW. Sven From dalburn at btopenworld.com Sat Nov 1 22:59:00 2008 From: dalburn at btopenworld.com (David Burn) Date: Sat, 1 Nov 2008 21:59:00 -0000 Subject: [blml] Mr Gladstone suggested [SEC=UNOFFICIAL] In-Reply-To: <000101c93c14$6b165d10$41431730$@no> References: <1062526051-1225456021-cardhu_decombobulator_blackberry.rim.net-888530924-@bxe065.bisx.produk.on.blackberry> <2a1c3a560810311251o1b9ea039j39b200467f77c725@mail.gmail.com> <000001c93ba4$f34d3b10$d9e7b130$@com> <000801c93bb0$45467ca0$cfd375e0$@no> <000201c93c04$492902d0$db7b0870$@com> <000101c93c14$6b165d10$41431730$@no> Message-ID: <000001c93c6d$0c52bae0$24f830a0$@com> [SP] WHAT ? ! ? ! Are you saying that what you have disclosed to your opponents no longer are understandings between you and your partner? You can't mean that? [DALB] I didn't mean it. Neither did I say it, but you said: "A partnership understanding to this effect is deemed to exist unless your partner has no more reason to be aware of it than have your opponents." which means that if I disclose our understandings to my opponents, so that my partner has no more reason to be aware of our understandings than my opponents do, such understandings are no longer deemed to exist. That may not have been what you meant to say, but it was what you said. What you meant to say was probably this: [SP] They cease to be CONCEALED partnership understandings. [DALB] Indeed they do, which is the point. If I tell my opponents "I may open 1H on a seven count", then when I do open 1H on a seven count, it does not contravene any Laws relating to disclosure. Of course, I may be subject to legislation of another kind: [SP] You are indeed playing a HUM if you have on your declaration that you can open a seven count with a bid at the one-level. [DALB] And here I do not agree with you. If I say "I will open the bidding on all seven counts", then I am playing a HUM. If I say "There exists at least one seven count such that if I am dealt it, I will open the bidding with it", then I am playing a HUM. But if I say "I may open 1H on a seven count" (intending by this that I once psyched 1H with a seven count and that "experience" must be disclosed because it is part of our "understandings"), then I am not playing a HUM. [SP] With all these argumentations please remember (and stick to) the basic criteria: "If your partner has more reason to be aware of the situation than have your opponents you are on thin ice with respect of Law 40." [DALB] Of course. That's why I tell them that I may open 1H on a seven count (not because I am necessarily going to do so, but because I once did so, and they are entitled to know that). [SP] "If your declared system is in any way HUM then it is illegal where HUM is forbidden." [DALB] Of course. But my declared system isn't in any way HUM. You see, last time I opened 1H on a seven count, we got a bad result and my partner made me promise never to do it again. I haven't done it again until today, so that if in the course of this set of boards I do open 1H on a seven count, partner will have *less* reason to be aware of the violation than the opponents will. This does not change the fact that I may open 1H on a seven count. Then again, I may not. David Burn London, England From svenpran at online.no Sat Nov 1 23:07:28 2008 From: svenpran at online.no (Sven Pran) Date: Sat, 1 Nov 2008 23:07:28 +0100 Subject: [blml] New Laws don't allow UI situation to be resolved? In-Reply-To: <001b01c93c5b$6d44b680$0302a8c0@Mildred> References: <2b1e598b0810302300t4f888261j8d7615f57fcdd4ee@mail.gmail.com> <000301c93b33$398b9670$aca2c350$@no> <001b01c93b5d$4438f2f0$ccaad8d0$@no> <000001c93c08$ea3dbf90$beb93eb0$@no> <000101c93c49$99440220$cbcc0660$@no> <000201c93c57$bb5d1230$32173690$@no> <001b01c93c5b$6d44b680$0302a8c0@Mildred> Message-ID: <000401c93c6e$3ac15e30$b0441a90$@no> On Behalf Of Grattan ....... > ----- Original Message ----- > From: "Sven Pran" > There is no such strong restriction when you have extraneous > information (UI) from other sources but you are still supposed > to act to the best of your ability not taking advantage of the UI. > > > +=+ But what of Law 16A3? +=+ Right. But I tend to be more lenient with a player who has received extraneous information from sources other than his partner than I shall be with a player who has received extraneous information from his partner when he claims that his selection among alternatives was not a result of the extraneous information? With UI from partner I adjust if the selected action could have been suggested by the UI, with UI from other sources I shall only adjust if a player demonstrably has used the UI to his advantage. (I can still adjust for opponents in their favor if I find that the choice taken was not obvious.) Regards Sven From svenpran at online.no Sun Nov 2 00:12:06 2008 From: svenpran at online.no (Sven Pran) Date: Sun, 2 Nov 2008 00:12:06 +0100 Subject: [blml] Mr Gladstone suggested [SEC=UNOFFICIAL] In-Reply-To: <000001c93c6d$0c52bae0$24f830a0$@com> References: <1062526051-1225456021-cardhu_decombobulator_blackberry.rim.net-888530924-@bxe065.bisx.produk.on.blackberry> <2a1c3a560810311251o1b9ea039j39b200467f77c725@mail.gmail.com> <000001c93ba4$f34d3b10$d9e7b130$@com> <000801c93bb0$45467ca0$cfd375e0$@no> <000201c93c04$492902d0$db7b0870$@com> <000101c93c14$6b165d10$41431730$@no> <000001c93c6d$0c52bae0$24f830a0$@com> Message-ID: <000501c93c77$427d1c00$c7775400$@no> On Behalf Of David Burn ............ > "A partnership understanding to this effect is deemed to exist unless your > partner has no more reason to be aware of it than have your opponents." > > which means that if I disclose our understandings to my opponents, so that > my partner has no more reason to be aware of our understandings than my > opponents do, such understandings are no longer deemed to exist. That may > not have been what you meant to say, but it was what you said. What you > meant to say was probably this: In the context I was talking about an alleged deviation from partnership understanding. The condition I quoted was from Law 40C1.I agree that taken out of context the quotation can easily be misunderstood. Regards Sven From mfrench1 at san.rr.com Sun Nov 2 01:07:01 2008 From: mfrench1 at san.rr.com (Marvin L French) Date: Sat, 1 Nov 2008 16:07:01 -0800 Subject: [blml] psychs Message-ID: <3032BE6F15B14B4DA5FF8D18E93D4E63@MARVLAPTOP> All this talk about psychs has me confused. Playing with a fairly expert partner for the first time, he opened 1S in first position, favorable vulnerability. Next hand doubled, I redoubled with my flat 13 HCP, and LHO jumped to 3H, raised to 4H by RHO. Unless the oppoents are insane my partner had to have psyched an opening, so I didn't double. That was okay, right? In the same session, with the same vulnerability, the auction was identical and I had the same flat 13 HCP. Are you all telling me I now had to double or be accused of something? If not, I suppose I should Alert all my partner's opening one bids as possible psychs, so I would get a PP for not Alerting 1S in the second case. This is some new game, not the game of bridge I have loved for 60 years. Marv Marvin L French San Diego, CA www.marvinfrench.com From grandaeval at tiscali.co.uk Sun Nov 2 06:44:11 2008 From: grandaeval at tiscali.co.uk (Grattan) Date: Sun, 2 Nov 2008 05:44:11 -0000 Subject: [blml] psychs References: <3032BE6F15B14B4DA5FF8D18E93D4E63@MARVLAPTOP> Message-ID: <000701c93cb6$c03ff060$0302a8c0@Mildred> Grattan Endicott To: "BLML" Sent: Sunday, November 02, 2008 12:07 AM Subject: [blml] psychs > All this talk about psychs has me confused. > > Playing with a fairly expert partner for the first time, he opened > 1S in first position, favorable vulnerability. Next hand doubled, I > redoubled with my flat 13 HCP, and LHO jumped to 3H, raised to 4H by > RHO. > > Unless the oppoents are insane my partner had to have psyched an > opening, so I didn't double. That was okay, right? > > In the same session, with the same vulnerability, the auction was > identical and I had the same flat 13 HCP. Are you all telling me I > now had to double or be accused of something? > > If not, I suppose I should Alert all my partner's opening one bids > as possible psychs, so I would get a PP for not Alerting 1S in the > second case. > > This is some new game, not the game of bridge I have loved for 60 > years. > +=+ I find Marvin's confusion incredible. If he is using authorized information (the cards dealt to him and the legal auction) to identify that partner has psyched, he is acting as the law allows. The ask for the TD would be whether there is significant potential for an alternative possibility in the evidence available to him. ~ Grattan ~ +=+ From grandaeval at tiscali.co.uk Sun Nov 2 07:12:55 2008 From: grandaeval at tiscali.co.uk (Grattan) Date: Sun, 2 Nov 2008 06:12:55 -0000 Subject: [blml] New Laws don't allow UI situation to be resolved? References: <2b1e598b0810302300t4f888261j8d7615f57fcdd4ee@mail.gmail.com><000301c93b33$398b9670$aca2c350$@no><001b01c93b5d$4438f2f0$ccaad8d0$@no><000001c93c08$ea3dbf90$beb93eb0$@no><000101c93c49$99440220$cbcc0660$@no> <001a01c93c5b$6d020830$0302a8c0@Mildred> <490CCA60.5060607@meadows.pair.com> Message-ID: <000801c93cb6$c082c5c0$0302a8c0@Mildred> Grattan Endicott To: "Bridge Laws Mailing List" Sent: Saturday, November 01, 2008 9:30 PM Subject: Re: [blml] New Laws don't allow UI situation to be resolved? > Grattan wrote: >> >> From: "Robert Frick" >> >> If partner supplies the UI, I understand that I cannot and should not >> choose an LA suggested by the UI. But if the UI is extraneous and it is >> not my fault that I have it, then it does not seem fair if I am >> ethically >> obligated not to select alternatives suggested by the UI. >> +=+ Perhaps Robert should look again at Law 16A3. It is not a >> matter of ethics that he is addressing but a matter of law. > > > I hope that I've misunderstood something. Might we consider an example? > > Say that I hear a loud remark from another table "You should have made > that grand, the Queen always lies over the Jack!" I have no idea which > board they have just played, nor even whether I'm scheduled to meet it. > > Some boards later, I open a strong 1NT, partner looks at me in > amazement and puts me straight into 7NT (Well, my partner would, > anyway!). When partner tables the dummy, I see that I do have to find > a missing Queen. > > Pard > S AKQ > H Jxx > D AJ975 > C AK > > > Me > S Jx > H AKQ > D K10864 > C QJx > > > Are you telling me that I am obliged by Law 16A3 to start on diamonds > by playing a small one to the King, and almost certainly go off in an > otherwise 50/50 grand, because of something which I unintentionally > overheard from another table? I'm with Robert on this one, it seems > grossly unfair to me. Maybe we should all turn up wearing earplugs? > +=+ No. The Director, whom you will have called when you received the UI in the first place, will have ruled that the UI "could interfere with normal play" and have applied Law 16C2. If he resorts to Law 16C2(d) both sides are non-offending. ~ G ~ +=+ From Hermandw at skynet.be Sun Nov 2 09:49:14 2008 From: Hermandw at skynet.be (Herman De Wael) Date: Sun, 02 Nov 2008 09:49:14 +0100 Subject: [blml] Mr Gladstone suggested [SEC=UNOFFICIAL] In-Reply-To: <000301c93c6c$d4854c90$7d8fe5b0$@no> References: <1062526051-1225456021-cardhu_decombobulator_blackberry.rim.net-888530924-@bxe065.bisx.produk.on.blackberry> <001a01c93b5c$0393aed0$0abb0c70$@no> <490B1506.4050408@skynet.be> <000301c93b8c$97769100$c663b300$@no> <490C484F.5040906@skynet.be> <000001c93c38$f9dfd200$ed9f7600$@no> <490CAD47.8050907@skynet.be> <000301c93c6c$d4854c90$7d8fe5b0$@no> Message-ID: <490D698A.20903@skynet.be> Sven Pran wrote: > On Behalf Of Herman De Wael > ............. >>> Why do you not accept Law 40 as it is written? >>> >> Why don't you? >> But that's not fair. > > Do you accept law 40? You definitely argue against it. > Of course I don't - we may not agree on what it means, but we're both reading the same text. It's clear you have not understood what I'm saying. But again, that's not fair - maybe I haven't clearly understood you. Let's try again. >>> I shall admit you one detail: There is no such thing as "concealed >>> partnership understanding" written in the laws, the laws use the term >>> "undisclosed partnership understanding". > > I withdraw this admission, take a look at Law 73E > What has that got to do with it? >> Indeed they do. It's the crime I call MI in short. > > There is a difference between MI and CPU; MI is accidental while CPU is > deliberate. > And this is where you are wrong - not teling the frequency of a psyche is far less deliberate than forgetting to alert. >> You OTOH, by using the words CPU in a context where this sounds very >> "wrong", create the impression that this is a different crime >> altogether. It isn't. >> >> So can we agree on this thing: if the frequency of the psyche is not >> disclosed to the opponents, then the pair are guilty of MI. > > There is only one criterion for judging a psyche and that is given in Law > 40C1 > Yes, but you are wrong in what to do next. The law on disclosing psyches "partner is no more aware than opponents" is a similar law to the one on all disclosure "duty to make available to opponents". >> The rectification procedure is exactly the same as for a missing alert: >> we investigate how the auction and play would have gone with complete >> information and we award an AS on that basis. >> >> Is that what you believe as well? >> >>> Now please do me a favor and explain why the two terms are not > synonymous. >>> The go along and explain why Law 40 shall not apply as written. >>> >> Of course it shall. L40B4. AS. > > Are you kidding? > No, are you? Please tell me, if you believe CPU and MI are two different things, how you deal with CPU. >>> And finally you could explain how a call can be "a deliberate and gross >>> misstatement of honour strength and/or of suit length" once it > corresponds >>> with an understanding that has been disclosed. Against what should there > be >>> a misstatement if not the disclosed understandings? >>> >> Indeed. But not all the understandings - not those that deal with the >> psyche itself, and its frequency. > > Where in the laws do you find any basis for this claim? > Nowhere in the laws, merely in logic. If any understanding about it makes a psyche a piece of system, then there are no psyches. > There is one single good thing about this discussion: You have made it > perfectly clear when you make up your own laws instead of adhering to those > issued by WBFLC. > No, there is one thing clear: you refuse to listen to other possible interpretations than you own. You are, like so many readers on blml, absolutely convinced about your being right. When you hear dissenting opinions, you read my arguments but refuse to think about them. You don't counter my arguments, but merely repeat your own. Please answer my question: if CPU and MI are two different crimes, what is then the ruling on CPUs - please refer me to law numbers. > No sense in fighting the windmill named HdW. > No sense in fighting against quixotic people like SP. > Sven > Herman. From Hermandw at skynet.be Sun Nov 2 09:55:44 2008 From: Hermandw at skynet.be (Herman De Wael) Date: Sun, 02 Nov 2008 09:55:44 +0100 Subject: [blml] psychs In-Reply-To: <000701c93cb6$c03ff060$0302a8c0@Mildred> References: <3032BE6F15B14B4DA5FF8D18E93D4E63@MARVLAPTOP> <000701c93cb6$c03ff060$0302a8c0@Mildred> Message-ID: <490D6B10.8030905@skynet.be> Grattan wrote: > > Grattan Endicott also ************************************** > "Let us then be up and doing, > With a heart for any fate, > Still achieving, still pursuing, > Learn to labour and to wait." > [H.W. Longfellow] > > ''''''''''''''''''''''''''''''''''''''''''''''''''''''''''''''''''''''''''''''''''''''''''''''''' > > > > ----- Original Message ----- > From: "Marvin L French" > To: "BLML" > Sent: Sunday, November 02, 2008 12:07 AM > Subject: [blml] psychs > > >> All this talk about psychs has me confused. >> >> Playing with a fairly expert partner for the first time, he opened >> 1S in first position, favorable vulnerability. Next hand doubled, I >> redoubled with my flat 13 HCP, and LHO jumped to 3H, raised to 4H by >> RHO. >> >> Unless the oppoents are insane my partner had to have psyched an >> opening, so I didn't double. That was okay, right? >> >> In the same session, with the same vulnerability, the auction was >> identical and I had the same flat 13 HCP. Are you all telling me I >> now had to double or be accused of something? >> >> If not, I suppose I should Alert all my partner's opening one bids >> as possible psychs, so I would get a PP for not Alerting 1S in the >> second case. >> >> This is some new game, not the game of bridge I have loved for 60 >> years. >> > +=+ I find Marvin's confusion incredible. If he is using authorized > information (the cards dealt to him and the legal auction) to identify > that partner has psyched, he is acting as the law allows. The ask > for the TD would be whether there is significant potential for an > alternative possibility in the evidence available to him. > ~ Grattan ~ +=+ > No Grattan, this is not incredible. Marv is extrapolating from what some of the posters on blml seem to be saying. If a psyche is only legal if partner has no more reason to believe it is possible than opponents, then there are two possible conclusions: a) after one psyche, all psyches become illegal, or b) after one psyche, all openings must be alerted. If, OTOH, a psyche is treated as any other piece of partnership understanding, which means that although the opponents are entitled to know about it, the penalty for them not being told is an adjustment if such absent knowledge caused damage, then the game (as Marv has played and loved it for 60 years) can be played as before. It is not Marv's confusion which is incredible, it is the posts of Sven's and others that caused the confusion which is incredible. Herman. From Hermandw at skynet.be Sun Nov 2 09:58:15 2008 From: Hermandw at skynet.be (Herman De Wael) Date: Sun, 02 Nov 2008 09:58:15 +0100 Subject: [blml] New Laws don't allow UI situation to be resolved? In-Reply-To: <000801c93cb6$c082c5c0$0302a8c0@Mildred> References: <2b1e598b0810302300t4f888261j8d7615f57fcdd4ee@mail.gmail.com><000301c93b33$398b9670$aca2c350$@no><001b01c93b5d$4438f2f0$ccaad8d0$@no><000001c93c08$ea3dbf90$beb93eb0$@no><000101c93c49$99440220$cbcc0660$@no> <001a01c93c5b$6d020830$0302a8c0@Mildred> <490CCA60.5060607@meadows.pair.com> <000801c93cb6$c082c5c0$0302a8c0@Mildred> Message-ID: <490D6BA7.30607@skynet.be> Grattan wrote: > > Grattan Endicott also ************************************** > "Let us then be up and doing, > With a heart for any fate, > Still achieving, still pursuing, > Learn to labour and to wait." > [H.W. Longfellow] > > ''''''''''''''''''''''''''''''''''''''''''''''''''''''''''''''''''''''''''''''''''''''''''''''''' > ----- Original Message ----- > From: "brian" > To: "Bridge Laws Mailing List" > Sent: Saturday, November 01, 2008 9:30 PM > Subject: Re: [blml] New Laws don't allow UI situation to be resolved? > > >> Grattan wrote: >>> From: "Robert Frick" >>> >>> If partner supplies the UI, I understand that I cannot and should not >>> choose an LA suggested by the UI. But if the UI is extraneous and it is >>> not my fault that I have it, then it does not seem fair if I am >>> ethically >>> obligated not to select alternatives suggested by the UI. >>> +=+ Perhaps Robert should look again at Law 16A3. It is not a >>> matter of ethics that he is addressing but a matter of law. >> >> I hope that I've misunderstood something. Might we consider an example? >> >> Say that I hear a loud remark from another table "You should have made >> that grand, the Queen always lies over the Jack!" I have no idea which >> board they have just played, nor even whether I'm scheduled to meet it. >> >> Some boards later, I open a strong 1NT, partner looks at me in >> amazement and puts me straight into 7NT (Well, my partner would, >> anyway!). When partner tables the dummy, I see that I do have to find >> a missing Queen. >> >> Pard >> S AKQ >> H Jxx >> D AJ975 >> C AK >> >> >> Me >> S Jx >> H AKQ >> D K10864 >> C QJx >> >> >> Are you telling me that I am obliged by Law 16A3 to start on diamonds >> by playing a small one to the King, and almost certainly go off in an >> otherwise 50/50 grand, because of something which I unintentionally >> overheard from another table? I'm with Robert on this one, it seems >> grossly unfair to me. Maybe we should all turn up wearing earplugs? >> > +=+ No. The Director, whom you will have called when you > received the UI in the first place, will have ruled that the UI "could > interfere with normal play" and have applied Law 16C2. If he resorts > to Law 16C2(d) both sides are non-offending. > ~ G ~ +=+ > That is just plain silly, Grattan. The laws have been altered so as to allow for more boards to be played. If you call the TD on the first occasion, he will no doubt apply (c) allow play to continue. Then you call him again a second time - now what? Herman. From brian at meadows.pair.com Sun Nov 2 11:29:29 2008 From: brian at meadows.pair.com (brian) Date: Sun, 02 Nov 2008 05:29:29 -0500 Subject: [blml] New Laws don't allow UI situation to be resolved? In-Reply-To: <000801c93cb6$c082c5c0$0302a8c0@Mildred> References: <2b1e598b0810302300t4f888261j8d7615f57fcdd4ee@mail.gmail.com><000301c93b33$398b9670$aca2c350$@no><001b01c93b5d$4438f2f0$ccaad8d0$@no><000001c93c08$ea3dbf90$beb93eb0$@no><000101c93c49$99440220$cbcc0660$@no> <001a01c93c5b$6d020830$0302a8c0@Mildred> <490CCA60.5060607@meadows.pair.com> <000801c93cb6$c082c5c0$0302a8c0@Mildred> Message-ID: <490D8109.3040609@meadows.pair.com> Grattan wrote: > > Grattan Endicott also ************************************** > "Let us then be up and doing, > With a heart for any fate, > Still achieving, still pursuing, > Learn to labour and to wait." > [H.W. Longfellow] > > ''''''''''''''''''''''''''''''''''''''''''''''''''''''''''''''''''''''''''''''''''''''''''''''''' > ----- Original Message ----- > From: "brian" > To: "Bridge Laws Mailing List" > Sent: Saturday, November 01, 2008 9:30 PM > Subject: Re: [blml] New Laws don't allow UI situation to be resolved? > > >> Grattan wrote: >>> From: "Robert Frick" >>> >>> If partner supplies the UI, I understand that I cannot and should not >>> choose an LA suggested by the UI. But if the UI is extraneous and it is >>> not my fault that I have it, then it does not seem fair if I am >>> ethically >>> obligated not to select alternatives suggested by the UI. >>> +=+ Perhaps Robert should look again at Law 16A3. It is not a >>> matter of ethics that he is addressing but a matter of law. >> >> I hope that I've misunderstood something. Might we consider an example? >> >> Say that I hear a loud remark from another table "You should have made >> that grand, the Queen always lies over the Jack!" I have no idea which >> board they have just played, nor even whether I'm scheduled to meet it. >> >> Some boards later, I open a strong 1NT, partner looks at me in >> amazement and puts me straight into 7NT (Well, my partner would, >> anyway!). When partner tables the dummy, I see that I do have to find >> a missing Queen. >> >> Pard >> S AKQ >> H Jxx >> D AJ975 >> C AK >> >> >> Me >> S Jx >> H AKQ >> D K10864 >> C QJx >> >> >> Are you telling me that I am obliged by Law 16A3 to start on diamonds >> by playing a small one to the King, and almost certainly go off in an >> otherwise 50/50 grand, because of something which I unintentionally >> overheard from another table? I'm with Robert on this one, it seems >> grossly unfair to me. Maybe we should all turn up wearing earplugs? >> > +=+ No. The Director, whom you will have called when you > received the UI in the first place, will have ruled that the UI "could > interfere with normal play" and have applied Law 16C2. If he resorts > to Law 16C2(d) both sides are non-offending. OK, I can handle the idea of awarding an artificial adjusted score, no problem with that one. But what if the TD DOESN'T reach 16C2(d), and decides instead to go for 16C2(c), and asks me to play the board out? Am I, or am I not, allowed to play for DQ sitting over DJ, as suggested by the extraneous UI, or does Law 16A3 compel me to play the DK on the first round of diamonds? And if the answer is that this situation is so obvious (I tried to make it so) that an adjusted score would be automatic, then please consider a slightly less obvious situation, where I again have an unavoidable choice made easy by extraneous UI. The same question applies. It seems to me that Law 16A3, which you referenced above, means that if you are asked to play out a board where you have extraneous UI, you must take a line not suggested by the UI. If that's the case, and you know you have to get something wrong to comply, is the award of an adjusted score going to be (almost) automatic? I have visions of some TDs saying "No, you got it wrong, you keep your score". Brian. From nigelguthrie at talktalk.net Sun Nov 2 12:42:23 2008 From: nigelguthrie at talktalk.net (Nigel Guthrie) Date: Sun, 02 Nov 2008 11:42:23 +0000 Subject: [blml] psychs In-Reply-To: <000701c93cb6$c03ff060$0302a8c0@Mildred> References: <3032BE6F15B14B4DA5FF8D18E93D4E63@MARVLAPTOP> <000701c93cb6$c03ff060$0302a8c0@Mildred> Message-ID: <490D921F.2060705@talktalk.net> [Marvin L French] All this talk about psychs has me confused. Playing with a fairly expert partner for the first time, he opened 1S in first position, favorable vulnerability. Next hand doubled, I redoubled with my flat 13 HCP, and LHO jumped to 3H, raised to 4H by RHO. Unless the oppoents are insane my partner had to have psyched an opening, so I didn't double. That was okay, right? In the same session, with the same vulnerability, the auction was identical and I had the same flat 13 HCP. Are you all telling me I now had to double or be accused of something? If not, I suppose I should Alert all my partner's opening one bids as possible psychs, so I would get a PP for not Alerting 1S in the second case. This is some new game, not the game of bridge I have loved for 60 years. [Grattan Endicott] +=+ I find Marvin's confusion incredible. If he is using authorized information (the cards dealt to him and the legal auction) to identify that partner has psyched, he is acting as the law allows. The ask for the TD would be whether there is significant potential for an alternative possibility in the evidence available to him. [Nigel[ Marvin appears to consider only *two* alternatives - Partner has psyched *or* - Both opponents are insane. I judge other explanations to be more likely. For example ... - Somebody has made a mistake. This is at least twice as likely to be an opponent as an expert partner. - Opponents hold shapely fitting hands. Everybody has appropriate values. In my experience, the latter is the most common explanation. We have all played contracts that make with few high cards. Sometimes, both sides can make a game or even a slam. If partner has already psyched in an identical bidding sequence in the same session, you might infer that this is a repetition. You might regard your suspicion as a nascent partnership understanding and lean over backwards to act as if partner has his bid. You might worry that any "fielding" behaviour might be prompted by rationalised self-interest. Marvin and I do seem to have been playing quite different games :) From grandaeval at tiscali.co.uk Sun Nov 2 11:41:56 2008 From: grandaeval at tiscali.co.uk (Grattan) Date: Sun, 2 Nov 2008 10:41:56 -0000 Subject: [blml] New Laws don't allow UI situation to be resolved? References: <2b1e598b0810302300t4f888261j8d7615f57fcdd4ee@mail.gmail.com><000301c93b33$398b9670$aca2c350$@no><001b01c93b5d$4438f2f0$ccaad8d0$@no><000001c93c08$ea3dbf90$beb93eb0$@no><000101c93c49$99440220$cbcc0660$@no> <001a01c93c5b$6d020830$0302a8c0@Mildred> <490CCA60.5060607@meadows.pair.com><000801c93cb6$c082c5c0$0302a8c0@Mildred> <490D6BA7.30607@skynet.be> Message-ID: <002c01c93ce1$221094f0$0302a8c0@Mildred> Grattan Endicott To: "Bridge Laws Mailing List" Sent: Sunday, November 02, 2008 8:58 AM Subject: Re: [blml] New Laws don't allow UI situation to be resolved? < +=+ No. The Director, whom you will have called when you received the UI in the first place, will have ruled that the UI "could interfere with normal play" and have applied Law 16C2. If he resorts to Law 16C2(d) both sides are non-offending. ~ G ~ +=+ >> > Herman. That is just plain silly, Grattan. The laws have been altered so as to allow for more boards to be played. If you call the TD on the first occasion, he will no doubt apply (c) allow play to continue. Then you call him again a second time - now what? > > +=+ "Just plain silly", if you say so, but nevertheless it is the law. Brian should have informed the Director when he heard the remark, not when he came to play the board (Law 16C1). The Director would then have control of the matter before Brian had the board in front of him. Having by then discovered the nature of the hand and the significance of the UI only a much inferior Director would have allowed play of the board. In the situation Brian reached, through oversight of his Law 16C1 duty, one could expect the Director to award an adjusted score because he judged (Law 12A2) that the UI was such as not to permit normal play of the board, but in addition to apply a penalty to Brian for his breach of law. ~ Grattan ~ +=+ From grandaeval at tiscali.co.uk Sun Nov 2 11:51:51 2008 From: grandaeval at tiscali.co.uk (Grattan) Date: Sun, 2 Nov 2008 10:51:51 -0000 Subject: [blml] Mr Gladstone suggested [SEC=UNOFFICIAL] References: <1062526051-1225456021-cardhu_decombobulator_blackberry.rim.net-888530924-@bxe065.bisx.produk.on.blackberry> <001a01c93b5c$0393aed0$0abb0c70$@no> <490B1506.4050408@skynet.be> <000301c93b8c$97769100$c663b300$@no> <490C484F.5040906@skynet.be> <000001c93c38$f9dfd200$ed9f7600$@no> <490CAD47.8050907@skynet.be><000301c93c6c$d4854c90$7d8fe5b0$@no> <490D698A.20903@skynet.be> Message-ID: <002d01c93ce1$22534340$0302a8c0@Mildred> Grattan Endicott To: "Bridge Laws Mailing List" Sent: Sunday, November 02, 2008 8:49 AM Subject: Re: [blml] Mr Gladstone suggested [SEC=UNOFFICIAL] > No, there is one thing clear: you refuse to listen to other possible interpretations than your own. You are, like so many readers on blml, absolutely convinced about your being right. When you hear dissenting opinions, you read my arguments but refuse to think about them. You don't counter my arguments, but merely repeat your own. << +=+ The pot calling the kettle black.+=+ From nigelguthrie at talktalk.net Sun Nov 2 13:10:52 2008 From: nigelguthrie at talktalk.net (Nigel Guthrie) Date: Sun, 02 Nov 2008 12:10:52 +0000 Subject: [blml] New Laws don't allow UI situation to be resolved? In-Reply-To: <002c01c93ce1$221094f0$0302a8c0@Mildred> References: <2b1e598b0810302300t4f888261j8d7615f57fcdd4ee@mail.gmail.com><000301c93b33$398b9670$aca2c350$@no><001b01c93b5d$4438f2f0$ccaad8d0$@no><000001c93c08$ea3dbf90$beb93eb0$@no><000101c93c49$99440220$cbcc0660$@no> <001a01c93c5b$6d020830$0302a8c0@Mildred> <490CCA60.5060607@meadows.pair.com><000801c93cb6$c082c5c0$0302a8c0@Mildred> <490D6BA7.30607@skynet.be> <002c01c93ce1$221094f0$0302a8c0@Mildred> Message-ID: <490D98CC.6040908@talktalk.net> [Herman de Wael] That is just plain silly, Grattan. The laws have been altered so as to allow for more boards to be played. If you call the TD on the first occasion, he will no doubt apply (c) allow play to continue. Then you call him again a second time - now what? [Grattan Endicott] +=+ "Just plain silly", if you say so, but nevertheless it is the law. Brian should have informed the Director when he heard the remark, not when he came to play the board (Law 16C1). The Director would then have control of the matter before Brian had the board in front of him. Having by then discovered the nature of the hand and the significance of the UI only a much inferior Director would have allowed play of the board. In the situation Brian reached, through oversight of his Law 16C1 duty, one could expect the Director to award an adjusted score because he judged (Law 12A2) that the UI was such as not to permit normal play of the board, but in addition to apply a penalty to Brian for his breach of law. [Nige1] Again, in this case, the law according to Grattan seems fair and sensible; is Grattan brain-washing us? From brian at meadows.pair.com Sun Nov 2 13:40:28 2008 From: brian at meadows.pair.com (brian) Date: Sun, 02 Nov 2008 07:40:28 -0500 Subject: [blml] New Laws don't allow UI situation to be resolved? In-Reply-To: <002c01c93ce1$221094f0$0302a8c0@Mildred> References: <2b1e598b0810302300t4f888261j8d7615f57fcdd4ee@mail.gmail.com><000301c93b33$398b9670$aca2c350$@no><001b01c93b5d$4438f2f0$ccaad8d0$@no><000001c93c08$ea3dbf90$beb93eb0$@no><000101c93c49$99440220$cbcc0660$@no> <001a01c93c5b$6d020830$0302a8c0@Mildred> <490CCA60.5060607@meadows.pair.com><000801c93cb6$c082c5c0$0302a8c0@Mildred> <490D6BA7.30607@skynet.be> <002c01c93ce1$221094f0$0302a8c0@Mildred> Message-ID: <490D9FBC.5080300@meadows.pair.com> Grattan wrote: > > +=+ "Just plain silly", if you say so, but nevertheless it is the > law. Brian should have informed the Director when he heard > the remark, not when he came to play the board (Law 16C1). > The Director would then have control of the matter before > Brian had the board in front of him. Having by then discovered > the nature of the hand and the significance of the UI only a much > inferior Director would have allowed play of the board. > In the situation Brian reached, through oversight of his Law > 16C1 duty, one could expect the Director to award an adjusted > score because he judged (Law 12A2) that the UI was such as > not to permit normal play of the board, but in addition to apply > a penalty to Brian for his breach of law. I thought that the consensus was that you called when you knew that the information applied to a board you were about to play? Are you saying that this isn't the case, and that a player has a duty to call the TD immediately for any remark heard which COULD be relevant to a board that player MIGHT yet have to play, even if the player can't identify the source of the remark? Brian. From nigelguthrie at talktalk.net Sun Nov 2 14:40:01 2008 From: nigelguthrie at talktalk.net (Nigel Guthrie) Date: Sun, 02 Nov 2008 13:40:01 +0000 Subject: [blml] New Laws don't allow UI situation to be resolved? In-Reply-To: <490D9FBC.5080300@meadows.pair.com> References: <2b1e598b0810302300t4f888261j8d7615f57fcdd4ee@mail.gmail.com><000301c93b33$398b9670$aca2c350$@no><001b01c93b5d$4438f2f0$ccaad8d0$@no><000001c93c08$ea3dbf90$beb93eb0$@no><000101c93c49$99440220$cbcc0660$@no> <001a01c93c5b$6d020830$0302a8c0@Mildred> <490CCA60.5060607@meadows.pair.com><000801c93cb6$c082c5c0$0302a8c0@Mildred> <490D6BA7.30607@skynet.be> <002c01c93ce1$221094f0$0302a8c0@Mildred> <490D9FBC.5080300@meadows.pair.com> Message-ID: <490DADB1.1020002@talktalk.net> [brian meadows] I thought that the consensus was that you called when you knew that the information applied to a board you were about to play? Are you saying that this isn't the case, and that a player has a duty to call the TD immediately for any remark heard which COULD be relevant to a board that player MIGHT yet have to play, even if the player can't identify the source of the remark? [nigel] A fair law would stipulate that a player called the director *immediately*, whenever a table conducts a revealing post-mortem, loud enough for other tables to overhear. This would allow the director - - to establish which players are responsible and to penalize the offenders in an attempt to prevent more boards being compromised. - to investigate which other tables may be in receipt of the same UI. Circumstances permitting, the director may even cancel or replace the board. From grandaeval at tiscali.co.uk Sun Nov 2 14:52:58 2008 From: grandaeval at tiscali.co.uk (Grattan) Date: Sun, 2 Nov 2008 13:52:58 -0000 Subject: [blml] New Laws don't allow UI situation to be resolved? References: <2b1e598b0810302300t4f888261j8d7615f57fcdd4ee@mail.gmail.com><000301c93b33$398b9670$aca2c350$@no><001b01c93b5d$4438f2f0$ccaad8d0$@no><000001c93c08$ea3dbf90$beb93eb0$@no><000101c93c49$99440220$cbcc0660$@no> <001a01c93c5b$6d020830$0302a8c0@Mildred> <490CCA60.5060607@meadows.pair.com><000801c93cb6$c082c5c0$0302a8c0@Mildred> <490D6BA7.30607@skynet.be><002c01c93ce1$221094f0$0302a8c0@Mildred> <490D9FBC.5080300@meadows.pair.com> Message-ID: <001a01c93cf2$dae1c0b0$0302a8c0@Mildred> Grattan Endicott To: "Bridge Laws Mailing List" Sent: Sunday, November 02, 2008 12:40 PM Subject: Re: [blml] New Laws don't allow UI situation to be resolved? > Grattan wrote: >> +=+ "Just plain silly", if you say so, but nevertheless it is the law. Brian should have informed the Director when he heard the remark, not when he came to play the board (Law 16C1). The Director would then have control of the matter before Brian had the board in front of him. Having by then discovered the nature of the hand and the significance of the UI only a much inferior Director would have allowed play of the board. In the situation Brian reached, through oversight of his Law 16C1 duty, one could expect the Director to award an adjusted score because he judged (Law 12A2) that the UI was such as not to permit normal play of the board, but in addition to apply a penalty to Brian for his breach of law. > > Brian. I thought that the consensus was that you called when you knew that the information applied to a board you were about to play? Are you saying that this isn't the case, and that a player has a duty to call the TD immediately for any remark heard which COULD be relevant to a board that player MIGHT yet have to play, even if the player can't identify the source of the remark? > +=+ The law is uncompromising. If the UI turns out to relate to a board yet to be played the player will be in the wrong if he has not told the Director about it. It is not often that remarks heard during a session do not relate to boards in that session. So the player is wise to inform the Director of what he has heard if he thinks it at all possible it could relate to a board he has yet to play. ~ Grattan ~ +=+ From Hermandw at skynet.be Sun Nov 2 15:23:33 2008 From: Hermandw at skynet.be (Herman De Wael) Date: Sun, 02 Nov 2008 15:23:33 +0100 Subject: [blml] Mr Gladstone suggested [SEC=UNOFFICIAL] In-Reply-To: <002d01c93ce1$22534340$0302a8c0@Mildred> References: <1062526051-1225456021-cardhu_decombobulator_blackberry.rim.net-888530924-@bxe065.bisx.produk.on.blackberry> <001a01c93b5c$0393aed0$0abb0c70$@no> <490B1506.4050408@skynet.be> <000301c93b8c$97769100$c663b300$@no> <490C484F.5040906@skynet.be> <000001c93c38$f9dfd200$ed9f7600$@no> <490CAD47.8050907@skynet.be><000301c93c6c$d4854c90$7d8fe5b0$@no> <490D698A.20903@skynet.be> <002d01c93ce1$22534340$0302a8c0@Mildred> Message-ID: <490DB7E5.3030608@skynet.be> Grattan wrote: > > Grattan Endicott also ************************************** > "Let us then be up and doing, > With a heart for any fate, > Still achieving, still pursuing, > Learn to labour and to wait." > [H.W. Longfellow] > > ''''''''''''''''''''''''''''''''''''''''''''''''''''''''''''''''''''''''''''''''''''''''''''''''' > ----- Original Message ----- > From: "Herman De Wael" > To: "Bridge Laws Mailing List" > Sent: Sunday, November 02, 2008 8:49 AM > Subject: Re: [blml] Mr Gladstone suggested [SEC=UNOFFICIAL] > > > No, there is one thing clear: you refuse to listen to other > possible interpretations than your own. > You are, like so many readers on blml, absolutely convinced > about your being right. When you hear dissenting opinions, > you read my arguments but refuse to think about them. You > don't counter my arguments, but merely repeat your own. > << > +=+ The pot calling the kettle black.+=+ > Grattan, I have continuously given argumentation for my views, and I see no refutations on those arguments. I have twice today asked Sven what the ruling is in his hypothetical third crime. I have not yet received an answer, but maybe there is still one coming. In the past however, I have often felt that my arguments go unanswered. OTOH, I have never shied away from answering other people's arguments and showing why they are wrong. I really don't feel like a kettle, or a pot. Herman. From Hermandw at skynet.be Sun Nov 2 15:29:12 2008 From: Hermandw at skynet.be (Herman De Wael) Date: Sun, 02 Nov 2008 15:29:12 +0100 Subject: [blml] New Laws don't allow UI situation to be resolved? In-Reply-To: <002c01c93ce1$221094f0$0302a8c0@Mildred> References: <2b1e598b0810302300t4f888261j8d7615f57fcdd4ee@mail.gmail.com><000301c93b33$398b9670$aca2c350$@no><001b01c93b5d$4438f2f0$ccaad8d0$@no><000001c93c08$ea3dbf90$beb93eb0$@no><000101c93c49$99440220$cbcc0660$@no> <001a01c93c5b$6d020830$0302a8c0@Mildred> <490CCA60.5060607@meadows.pair.com><000801c93cb6$c082c5c0$0302a8c0@Mildred> <490D6BA7.30607@skynet.be> <002c01c93ce1$221094f0$0302a8c0@Mildred> Message-ID: <490DB938.8010807@skynet.be> Grattan wrote: > > Grattan Endicott also ************************************** > "Let us then be up and doing, > With a heart for any fate, > Still achieving, still pursuing, > Learn to labour and to wait." > [H.W. Longfellow] > > ''''''''''''''''''''''''''''''''''''''''''''''''''''''''''''''''''''''''''''''''''''''''''''''''' > > > > ----- Original Message ----- > From: "Herman De Wael" > To: "Bridge Laws Mailing List" > Sent: Sunday, November 02, 2008 8:58 AM > Subject: Re: [blml] New Laws don't allow UI situation to be resolved? > < > +=+ No. The Director, whom you will have called when you > received the UI in the first place, will have ruled that the UI "could > interfere with normal play" and have applied Law 16C2. If he resorts > to Law 16C2(d) both sides are non-offending. > ~ G ~ +=+ >> Herman. > That is just plain silly, Grattan. > The laws have been altered so as to allow for more boards > to be played. If you call the TD on the first occasion, he will > no doubt apply (c) allow play to continue. > Then you call him again a second time - now what? >> > +=+ "Just plain silly", if you say so, but nevertheless it is the > law. Brian should have informed the Director when he heard > the remark, not when he came to play the board (Law 16C1). Come on Grattan, have you ever tried this in real life? "I have heard that there is a queen over a jack in a seven contract." What TD is going to spend time finding out what board it is, or when the hearer is going to play it? He'll just say "play on and call me back if you recognize the board". And even if he knew it was the board he's starting next, the TD will not divulge this. Perhaps the player will not recognize it. But even so, the TD has the right (in the 2007 laws) to allow the board to continue, so to say, as you do : > The Director would then have control of the matter before > Brian had the board in front of him. Having by then discovered > the nature of the hand and the significance of the UI only a much > inferior Director would have allowed play of the board. is negating the laws that now specifically allow the board to be played. Even if you believe that in this case only an inferior director would allow the board to start, there must be other cases where the start of the board is allowed (or your work in writing L16C2c has no sense at all). > In the situation Brian reached, through oversight of his Law > 16C1 duty, one could expect the Director to award an adjusted > score because he judged (Law 12A2) that the UI was such as > not to permit normal play of the board, but in addition to apply > a penalty to Brian for his breach of law. > ~ Grattan ~ +=+ > OK, so tell us what the TD shall do if he's of the opinion that the L16C1 oversight did not change anything - as he would have allowe the board to start anyway. Herman. From rfrick at rfrick.info Sun Nov 2 16:32:40 2008 From: rfrick at rfrick.info (Robert Frick) Date: Sun, 02 Nov 2008 10:32:40 -0500 Subject: [blml] New Laws don't allow UI situation to be resolved? In-Reply-To: <002c01c93ce1$221094f0$0302a8c0@Mildred> References: <2b1e598b0810302300t4f888261j8d7615f57fcdd4ee@mail.gmail.com> <000301c93b33$398b9670$aca2c350$@no> <001b01c93b5d$4438f2f0$ccaad8d0$@no> <000001c93c08$ea3dbf90$beb93eb0$@no> <000101c93c49$99440220$cbcc0660$@no> <001a01c93c5b$6d020830$0302a8c0@Mildred> <490CCA60.5060607@meadows.pair.com> <000801c93cb6$c082c5c0$0302a8c0@Mildred> <490D6BA7.30607@skynet.be> <002c01c93ce1$221094f0$0302a8c0@Mildred> Message-ID: On Sun, 02 Nov 2008 05:41:56 -0500, Grattan wrote: > > > Grattan Endicott also ************************************** > "Let us then be up and doing, > With a heart for any fate, > Still achieving, still pursuing, > Learn to labour and to wait." > [H.W. Longfellow] > > ''''''''''''''''''''''''''''''''''''''''''''''''''''''''''''''''''''''''''''''''''''''''''''''''' > > > > ----- Original Message ----- > From: "Herman De Wael" > To: "Bridge Laws Mailing List" > Sent: Sunday, November 02, 2008 8:58 AM > Subject: Re: [blml] New Laws don't allow UI situation to be resolved? > < > +=+ No. The Director, whom you will have called when you > received the UI in the first place, will have ruled that the UI "could > interfere with normal play" and have applied Law 16C2. If he resorts > to Law 16C2(d) both sides are non-offending. > ~ G ~ +=+ >>> >> Herman. > That is just plain silly, Grattan. > The laws have been altered so as to allow for more boards > to be played. If you call the TD on the first occasion, he will > no doubt apply (c) allow play to continue. > Then you call him again a second time - now what? >> >> > +=+ "Just plain silly", if you say so, but nevertheless it is the > law. Brian should have informed the Director when he heard > the remark, not when he came to play the board (Law 16C1). > The Director would then have control of the matter before > Brian had the board in front of him. Having by then discovered > the nature of the hand and the significance of the UI only a much > inferior Director would have allowed play of the board. > In the situation Brian reached, through oversight of his Law > 16C1 duty, one could expect the Director to award an adjusted > score because he judged (Law 12A2) that the UI was such as > not to permit normal play of the board, but in addition to apply > a penalty to Brian for his breach of law. > ~ Grattan ~ +=+ At a World Championship, I assume that it is feasible to notify a director whenever you have any advance knowledge of a board. At a club game, this is COMPLETELY impractical. Most of the time, you get some information that you can't tell if it is useful or not. And you don't know which board it applies to; it could even be a board you played. And it happens a lot. If everyone calls the director every time this happens, the game grinds to a halt. If I call the director every time this happens, the director tells me in polite terms to stop annoying her. Then, as Herman noted, if you do call the director, the director moves straight to L16C2(c), try to play the board normally. So it was a waste of time to notify the director anyway. There is more. If I hear a snippet of information that might be useful, my best strategy is to try to ignore that information. If I can ignore it, I can probably forget it. The worst strategy for forgetting it is to process the information to decide if it might be useful or not. And another horrible strategy is to think about what board it is on to then decide whether or not I have played it. From svenpran at online.no Sun Nov 2 16:01:42 2008 From: svenpran at online.no (Sven Pran) Date: Sun, 2 Nov 2008 16:01:42 +0100 Subject: [blml] Mr Gladstone suggested [SEC=UNOFFICIAL] In-Reply-To: <490DB7E5.3030608@skynet.be> References: <1062526051-1225456021-cardhu_decombobulator_blackberry.rim.net-888530924-@bxe065.bisx.produk.on.blackberry> <001a01c93b5c$0393aed0$0abb0c70$@no> <490B1506.4050408@skynet.be> <000301c93b8c$97769100$c663b300$@no> <490C484F.5040906@skynet.be> <000001c93c38$f9dfd200$ed9f7600$@no> <490CAD47.8050907@skynet.be><000301c93c6c$d4854c90$7d8fe5b0$@no> <490D698A.20903@skynet.be> <002d01c93ce1$22534340$0302a8c0@Mildred> <490DB7E5.3030608@skynet.be> Message-ID: <000301c93cfb$eafff2b0$c0ffd810$@no> On Behalf Of Herman De Wael .............. > Grattan, I have continuously given argumentation for my views, and I see > no refutations on those arguments. Your argumentation is apparently (and according to your own words) based on what you think is the meaning of the law, even when that meaning is directly in conflict with the law text. Latest example: Law 40 has an absolute condition for a psyche to be legal that partner of the psyching player has no more reason than have opponents to be aware of the psyche. There is absolutely no provision anywhere in the laws for establishing any probability or frequency of psyching below which the psyche becomes legal. Still you maintain such a criterion to be significant. > > I have twice today asked Sven what the ruling is in his hypothetical > third crime. I have not yet received an answer, but maybe there is still > one coming. No, there will not be any. As I have said I find it futile to add anything more to this thread. Sven From svenpran at online.no Sun Nov 2 16:52:09 2008 From: svenpran at online.no (Sven Pran) Date: Sun, 2 Nov 2008 16:52:09 +0100 Subject: [blml] New Laws don't allow UI situation to be resolved? In-Reply-To: References: <2b1e598b0810302300t4f888261j8d7615f57fcdd4ee@mail.gmail.com> <000301c93b33$398b9670$aca2c350$@no> <001b01c93b5d$4438f2f0$ccaad8d0$@no> <000001c93c08$ea3dbf90$beb93eb0$@no> <000101c93c49$99440220$cbcc0660$@no> <001a01c93c5b$6d020830$0302a8c0@Mildred> <490CCA60.5060607@meadows.pair.com> <000801c93cb6$c082c5c0$0302a8c0@Mildred> <490D6BA7.30607@skynet.be> <002c01c93ce1$221094f0$0302a8c0@Mildred> Message-ID: <000401c93d02$f6ddcdd0$e4996970$@no> I once had a situation similar to the object of this thread: While intensely concentrating on the question whether I should proceed for a slam contract or just settle in game (it was a borderline decision and the auction had reached the critical point) I suddenly heard a remark from a neighboring table: "Six is cold". That remark simply made me completely unable to make any serious decision and I demanded (and received) A+/A+ on the ground that the board had been destroyed. It is part of the story that to this day I have no idea if the remark applied to the board I was just playing, to a different board in the same round or even to a board played in the previous round! (We play barometer so all tables play the same boards in the same round; it was revealed afterwards that all three alternatives were possible.) And I still feel that A+ both ways was the only sensible ruling unless we should have awarded a split score; the most advantageous alternative of game made or slam made independently for each side. This was of course under the 1997 laws, but I feel the same today. Note that this "ruling" was based directly on the "fact" that it was no longer possible to obtain a result on the board by "normal" play because we felt it impossible to continue the auction in any "normal" fashion. Regards Sven From grandaeval at tiscali.co.uk Sun Nov 2 17:19:04 2008 From: grandaeval at tiscali.co.uk (Grattan) Date: Sun, 2 Nov 2008 16:19:04 -0000 Subject: [blml] Mr Gladstone suggested [SEC=UNOFFICIAL] References: <1062526051-1225456021-cardhu_decombobulator_blackberry.rim.net-888530924-@bxe065.bisx.produk.on.blackberry> <001a01c93b5c$0393aed0$0abb0c70$@no> <490B1506.4050408@skynet.be> <000301c93b8c$97769100$c663b300$@no> <490C484F.5040906@skynet.be> <000001c93c38$f9dfd200$ed9f7600$@no> <490CAD47.8050907@skynet.be><000301c93c6c$d4854c90$7d8fe5b0$@no> <490D698A.20903@skynet.be> <002d01c93ce1$22534340$0302a8c0@Mildred><490DB7E5.3030608@skynet.be> <000301c93cfb$eafff2b0$c0ffd810$@no> Message-ID: <002701c93d06$bbd534e0$0302a8c0@Mildred> Grattan Endicott On Behalf Of Herman De Wael > .............. >> Grattan, I have continuously given argumentation for my views, and I see >> no refutations on those arguments. > > Your argumentation is apparently (and according to your own words) based > on > what you think is the meaning of the law, even when that meaning is > directly > in conflict with the law text. > > Latest example: Law 40 has an absolute condition for a psyche to be legal > that partner of the psyching player has no more reason than have opponents > to be aware of the psyche. There is absolutely no provision anywhere in > the > laws for establishing any probability or frequency of psyching below which > the psyche becomes legal. Still you maintain such a criterion to be > significant. > +=+ I do not know where Herman saw me arguing this. I would like him to point to *my* words saying it. In fact my position is that the law provides that a psyche becomes part of system if there is a partnership understanding attached to it, and as part of system it may be regulated and will be unlawful if there is a violation of the system regulations. Frequency may create an implicit partnership understanding - something for the Director to assess. Subscribers will note that in general I cite positions under the laws, leaving others to argue their personal opinions. I have not expressed any personal opinion as to when the line is crossed from psyche to partnership understanding: in fact my opinion is that it is the task, case by case, of the Director and AC (with such guidance as the RA may give) to judge when the line is crossed. ~ Grattan ~ +=+ From Hermandw at skynet.be Sun Nov 2 17:42:35 2008 From: Hermandw at skynet.be (Herman De Wael) Date: Sun, 02 Nov 2008 17:42:35 +0100 Subject: [blml] Mr Gladstone suggested [SEC=UNOFFICIAL] In-Reply-To: <000301c93cfb$eafff2b0$c0ffd810$@no> References: <1062526051-1225456021-cardhu_decombobulator_blackberry.rim.net-888530924-@bxe065.bisx.produk.on.blackberry> <001a01c93b5c$0393aed0$0abb0c70$@no> <490B1506.4050408@skynet.be> <000301c93b8c$97769100$c663b300$@no> <490C484F.5040906@skynet.be> <000001c93c38$f9dfd200$ed9f7600$@no> <490CAD47.8050907@skynet.be><000301c93c6c$d4854c90$7d8fe5b0$@no> <490D698A.20903@skynet.be> <002d01c93ce1$22534340$0302a8c0@Mildred> <490DB7E5.3030608@skynet.be> <000301c93cfb$eafff2b0$c0ffd810$@no> Message-ID: <490DD87B.3080004@skynet.be> Sven, again: Sven Pran wrote: > On Behalf Of Herman De Wael > .............. >> Grattan, I have continuously given argumentation for my views, and I see >> no refutations on those arguments. > > Your argumentation is apparently (and according to your own words) based on > what you think is the meaning of the law, even when that meaning is directly > in conflict with the law text. > Well Sven, the least you could do is award me the courtesy of trying to imagine what I am trying to do. You have told me twice already that I have not read L40, and I have twoice replied that I have. > Latest example: Law 40 has an absolute condition for a psyche to be legal > that partner of the psyching player has no more reason than have opponents > to be aware of the psyche. There is absolutely no provision anywhere in the > laws for establishing any probability or frequency of psyching below which > the psyche becomes legal. Still you maintain such a criterion to be > significant. > Sven, that same law has the same condition for the use of a convention. Yet when a convention is employed that is not on the SC, or not alerted, or not correctly explained, we do not rule that convention to be illegal, but we correct the resulting damage. I believe (and you may still disagree - but then please do so) that the same applies to psyches: if the condition in L40 is not met, then there will be an adjustment if there was resulting damage. >> I have twice today asked Sven what the ruling is in his hypothetical >> third crime. I have not yet received an answer, but maybe there is still >> one coming. > > No, there will not be any. As I have said I find it futile to add anything > more to this thread. > See? I give arguments, and when they cannot win any more, they give up. I ahve asked a very pertinent question - I have even given the answer I expected - but rather than answering or even merely agreeing with my answer, Sven says it is futile. Do you see why I keep badgering you? - you refuse to admit that I'm right, even after you have exhausted all argumentation. But I'm the bad one, of course. Well. I consider it futile to try and convince Sven. It's like fighting windmills. > Sven > Herman. From Hermandw at skynet.be Sun Nov 2 17:43:28 2008 From: Hermandw at skynet.be (Herman De Wael) Date: Sun, 02 Nov 2008 17:43:28 +0100 Subject: [blml] Mr Gladstone suggested [SEC=UNOFFICIAL] In-Reply-To: <002701c93d06$bbd534e0$0302a8c0@Mildred> References: <1062526051-1225456021-cardhu_decombobulator_blackberry.rim.net-888530924-@bxe065.bisx.produk.on.blackberry> <001a01c93b5c$0393aed0$0abb0c70$@no> <490B1506.4050408@skynet.be> <000301c93b8c$97769100$c663b300$@no> <490C484F.5040906@skynet.be> <000001c93c38$f9dfd200$ed9f7600$@no> <490CAD47.8050907@skynet.be><000301c93c6c$d4854c90$7d8fe5b0$@no> <490D698A.20903@skynet.be> <002d01c93ce1$22534340$0302a8c0@Mildred><490DB7E5.3030608@skynet.be> <000301c93cfb$eafff2b0$c0ffd810$@no> <002701c93d06$bbd534e0$0302a8c0@Mildred> Message-ID: <490DD8B0.60006@skynet.be> Grattan wrote: > >> > +=+ I do not know where Herman saw me arguing this. I have no recollection of you argueing anything like this, or of me suggesting that you did. Herman. From mfrench1 at san.rr.com Sun Nov 2 18:58:49 2008 From: mfrench1 at san.rr.com (Marvin L French) Date: Sun, 2 Nov 2008 09:58:49 -0800 Subject: [blml] psychs References: <3032BE6F15B14B4DA5FF8D18E93D4E63@MARVLAPTOP><000701c93cb6$c03ff060$0302a8c0@Mildred> <490D921F.2060705@talktalk.net> Message-ID: <4E674A6861864A729C1A16BB86EC8652@MARVLAPTOP> Nigel wrote: > If partner has already psyched in an identical bidding sequence in > the same session, you might infer that this is a repetition. You > might regard your suspicion as a nascent partnership understanding > and lean over backwards to act as if partner has his bid. You > might worry that any "fielding" behaviour might be prompted by > rationalised self-interest. And therefore commit suicide. My understanding comes from general knowledge and experience, and no amount of partnership experience can change that if I don't let it. I do "lean over backwards," straining to make any logical call with a partner who psychs occasionally. If I would conclude a psych playing with a strange partner, how could it be wrong to so conclude with any partner? > > Marvin and I do seem to have been playing quite different games :) My game is bridge. Please tell me in detail how psych and falsecard possibilities should be disclosed. "We psych a lot," one young pair announced to us at an NABC. I was happy to face opponents who played the whole game of bridge, not just a namby-pamby subset. Marv Marvin L French San Diego, CA www.marvinfrench.com From mfrench1 at san.rr.com Sun Nov 2 19:24:04 2008 From: mfrench1 at san.rr.com (Marvin L French) Date: Sun, 2 Nov 2008 10:24:04 -0800 Subject: [blml] psychs References: <3032BE6F15B14B4DA5FF8D18E93D4E63@MARVLAPTOP><000701c93cb6$c03ff060$0302a8c0@Mildred> <490D6B10.8030905@skynet.be> Message-ID: Herman wrote: > > If, OTOH, a psyche is treated as any other piece of partnership > understanding, which means that although the opponents are > entitled to > know about it, the penalty for them not being told is an > adjustment if > such absent knowledge caused damage, then the game (as Marv has > played > and loved it for 60 years) can be played as before. > But how do I inform opponents that we play the total game, which includes an occasional psych that will not affect a partner's bidding unless there is conclusive evidence (acknowledgeable by any experienced player) that a psych must have been perpetrated? Marv From svenpran at online.no Sun Nov 2 19:33:31 2008 From: svenpran at online.no (Sven Pran) Date: Sun, 2 Nov 2008 19:33:31 +0100 Subject: [blml] Mr Gladstone suggested [SEC=UNOFFICIAL] In-Reply-To: <490DD87B.3080004@skynet.be> References: <1062526051-1225456021-cardhu_decombobulator_blackberry.rim.net-888530924-@bxe065.bisx.produk.on.blackberry> <001a01c93b5c$0393aed0$0abb0c70$@no> <490B1506.4050408@skynet.be> <000301c93b8c$97769100$c663b300$@no> <490C484F.5040906@skynet.be> <000001c93c38$f9dfd200$ed9f7600$@no> <490CAD47.8050907@skynet.be><000301c93c6c$d4854c90$7d8fe5b0$@no> <490D698A.20903@skynet.be> <002d01c93ce1$22534340$0302a8c0@Mildred> <490DB7E5.3030608@skynet.be> <000301c93cfb$eafff2b0$c0ffd810$@no> <490DD87B.3080004@skynet.be> Message-ID: <000501c93d19$81da8bb0$858fa310$@no> On Behalf Of Herman De Wael ....................... > Sven, that same law has the same condition for the use of a convention. > Yet when a convention is employed that is not on the SC, or not alerted, > or not correctly explained, we do not rule that convention to be > illegal, but we correct the resulting damage. OK then, (rather against my better wisdom to try yet another time): If opponents have incorrect (or no) information on some relevant partnership understanding then of course we have a situation of misinformation. This has little or nothing to do with psychic calls. The primary condition for having a call accepted (by the director) to be a psyche is that it must be a deliberate and gross deviation from partnership understandings; this implies that the meaning of the call can in no way be part of the partnership understandings, a fact that is further emphasized in Law 40; Partner must not have more reason than have opponents to suspect this deviation from partnership understandings. A partnership is of course free to (explicitly or implicitly) include in their understandings (which then must be disclosed to opponents in the prescribed way) that they can make what they call a psychic call for instance with a certain predefined probability. However, once they have specified that then this has become part of their partnership understanding so when they subsequently psyches (in their own mind) they really do not psyche but they call according to their partnership understanding. A possible statement that the "psyche" will occur extremely seldom is irrelevant; I have in my partnership understandings (and I believe most players have) specifications on situations that can be expected less than once a year. They still are part of my partnership understandings, Then we have the second question: Are such understandings legal? Yes, as long as the understandings are disclosed all such understandings are legal with regard of the laws, but they may be illegal with regards of regulations. (This is where for instance HUM enters the case.) So the complete conditions for a psychic call to be legal is that partner must not have more reason than opponents have to be aware of the deviation from understandings, and also that the call is indeed a gross violation of partnership understandings. (This last condition is obviously not met if the "psyche" has in any way been disclosed with the partnership understandings!) So what are the consequences in various situations? South deliberately makes a gross deviation from partnership understandings, opponents have correct description of understandings and there is no doubt that North was at least as unaware of the situation as were opponents: We have a genuine psyche and there is no rectification or other reaction. The same, but actions by North (or other circumstances) raise suspicion that he could be aware of the (attempted) psyche: Depending on his judgment the director can accept or reject the call as a genuine psyche. If he rejects it the consequence should be PP against NS in addition to redress for opponents. In this case there can be further reactions from the attempted psyche found to violate regulations (e.g. HUM). Opponents have incorrect description of understandings: They will receive redress if they can show damage (Note that this case is independent of South bidding according to or contrary to partnership understandings!) This is the ordinary MI situation which should result in redress for opponents if they are found damaged, but which also at the director's judgment can result in PP against NS, Sven From dalburn at btopenworld.com Sun Nov 2 19:50:04 2008 From: dalburn at btopenworld.com (David Burn) Date: Sun, 2 Nov 2008 18:50:04 -0000 Subject: [blml] New Laws don't allow UI situation to be resolved? In-Reply-To: <001a01c93cf2$dae1c0b0$0302a8c0@Mildred> References: <2b1e598b0810302300t4f888261j8d7615f57fcdd4ee@mail.gmail.com><000301c93b33$398b9670$aca2c350$@no><001b01c93b5d$4438f2f0$ccaad8d0$@no><000001c93c08$ea3dbf90$beb93eb0$@no><000101c93c49$99440220$cbcc0660$@no> <001a01c93c5b$6d020830$0302a8c0@Mildred> <490CCA60.5060607@meadows.pair.com><000801c93cb6$c082c5c0$0302a8c0@Mildred> <490D6BA7.30607@skynet.be><002c01c93ce1$221094f0$0302a8c0@Mildred> <490D9FBC.5080300@meadows.pair.com> <001a01c93cf2$dae1c0b0$0302a8c0@Mildred> Message-ID: <000901c93d1b$d1d72900$75857b00$@com> Haven't been paying much attention to this, but in Brian's case it seems to me that: If you hear a remark that amounts to "you could have made 7NT if you had played for the queen to be over the jack", you should call the TD at once (unless you know that it relates to a board you have already played or will never play). That is what L16C1 seems to me to say. If it turns out that the remark related to a board played by the remarker's partner last week, at least no harm will have been done. The TD might then determine on which board you could make 7NT by playing for the queen to be over the jack and prevent you from playing that board (ruling under L12A2). Or, he might let you play the board to see whether or not you reached 7NT (or recognised during the auction that this was the board in question). If you did, he would award both sides average plus; if not, he might award you some percentage of 12 tricks and some percentage of 13 tricks in 6NT, or whatever you did reach. But if you do not call the Director as soon as you receive the information, it seems to me that you place your side at some risk. Perhaps, for the sake of clarity, "has yet to play" might be amended to "may yet have to play" in the next version of the Laws. David Burn London, England From Hermandw at skynet.be Sun Nov 2 20:23:07 2008 From: Hermandw at skynet.be (Herman De Wael) Date: Sun, 02 Nov 2008 20:23:07 +0100 Subject: [blml] Mr Gladstone suggested [SEC=UNOFFICIAL] In-Reply-To: <000501c93d19$81da8bb0$858fa310$@no> References: <1062526051-1225456021-cardhu_decombobulator_blackberry.rim.net-888530924-@bxe065.bisx.produk.on.blackberry> <001a01c93b5c$0393aed0$0abb0c70$@no> <490B1506.4050408@skynet.be> <000301c93b8c$97769100$c663b300$@no> <490C484F.5040906@skynet.be> <000001c93c38$f9dfd200$ed9f7600$@no> <490CAD47.8050907@skynet.be><000301c93c6c$d4854c90$7d8fe5b0$@no> <490D698A.20903@skynet.be> <002d01c93ce1$22534340$0302a8c0@Mildred> <490DB7E5.3030608@skynet.be> <000301c93cfb$eafff2b0$c0ffd810$@no> <490DD87B.3080004@skynet.be> <000501c93d19$81da8bb0$858fa310$@no> Message-ID: <490DFE1B.1070002@skynet.be> Hello Sven, thanks for the consideration. Sven Pran wrote: > On Behalf Of Herman De Wael > ....................... >> Sven, that same law has the same condition for the use of a convention. >> Yet when a convention is employed that is not on the SC, or not alerted, >> or not correctly explained, we do not rule that convention to be >> illegal, but we correct the resulting damage. > > OK then, (rather against my better wisdom to try yet another time): > > If opponents have incorrect (or no) information on some relevant partnership > understanding then of course we have a situation of misinformation. This has > little or nothing to do with psychic calls. > I agree, but the similarity will become obvious if you read L40A3. There too, it is said that one may not make a call that is not understood. Exactly the same construction: the bidding pair is not allowed to ... > The primary condition for having a call accepted (by the director) to be a > psyche is that it must be a deliberate and gross deviation from partnership > understandings; Agreed, with one condition: the Director need not (as a primary duty) determine if something is a psyche or not. But I digress. > this implies that the meaning of the call can in no way be > part of the partnership understandings, a fact that is further emphasized in > Law 40; No, it cannot be part of the partnership agreements. Said very often. > Partner must not have more reason than have opponents to suspect > this deviation from partnership understandings. > Indeed - but it does say "no more" not "no". That means that the laws do acknowledge that a player may be aware that his partner sometimes psyches. As long as the opponents also know that the player sometimes psyches, the condition of L40C1 is met. > A partnership is of course free to (explicitly or implicitly) include in > their understandings (which then must be disclosed to opponents in the > prescribed way) that they can make what they call a psychic call for > instance with a certain predefined probability. > Thank you. Do you agree that this knowledge is part of partnership understandings - apparently you do. So you agree with me that there can be partnership understandings about psyches. Great. > However, once they have specified that then this has become part of their > partnership understanding so when they subsequently psyches (in their own > mind) they really do not psyche but they call according to their partnership > understanding. A possible statement that the "psyche" will occur extremely > seldom is irrelevant; I have in my partnership understandings (and I believe > most players have) specifications on situations that can be expected less > than once a year. They still are part of my partnership understandings, > And there we must part ways. In your opinion, there are no more psyches. That is the same as forbidding them. Which is not what the laws intend to do. Your argumentation cannot be the correct one. > Then we have the second question: Are such understandings legal? > Indeed a second question. In two parts. Are they legal, and are they HUM (if weak) > Yes, as long as the understandings are disclosed all such understandings are > legal with regard of the laws, but they may be illegal with regards of > regulations. (This is where for instance HUM enters the case.) > Indeed the correct answer to the first question. > So the complete conditions for a psychic call to be legal is that partner > must not have more reason than opponents have to be aware of the deviation > from understandings, and also that the call is indeed a gross violation of > partnership understandings. (This last condition is obviously not met if the > "psyche" has in any way been disclosed with the partnership understandings!) > > So what are the consequences in various situations? > Aha. > South deliberately makes a gross deviation from partnership understandings, > opponents have correct description of understandings and there is no doubt > that North was at least as unaware of the situation as were opponents: We > have a genuine psyche and there is no rectification or other reaction. > OK, although I fail to see how this can be achieved if the player has psyched ever before, under your definition of a psyche. But let's continue. > The same, but actions by North (or other circumstances) raise suspicion that > he could be aware of the (attempted) psyche: Depending on his judgment the > director can accept or reject the call as a genuine psyche. If he rejects it > the consequence should be PP against NS in addition to redress for > opponents. In this case there can be further reactions from the attempted > psyche found to violate regulations (e.g. HUM). > And here is your answer to the question I was posing: a PP to the offenders. Please tell me where in the laws you find the reason for this PP. Remember that all they have so far done is provide less information to their opponents than the director deems necessary. But I note one important thing: you use the MI laws for the remainder. So you do see that (what you call) CPU and MI are one and the same! > Opponents have incorrect description of understandings: They will receive > redress if they can show damage (Note that this case is independent of South > bidding according to or contrary to partnership understandings!) This is the > ordinary MI situation which should result in redress for opponents if they > are found damaged, but which also at the director's judgment can result in > PP against NS, > Now you have me confused - what is the difference between the second and third case? Partner acting on the psyche before it should be known ? that is the second crime I called cheating in some previous post. A PP would be too lenient IMO. Then your third case is the third crime I alluded to. And you give only an AS if damage in that case. In what way is that then any different than MI? Sven, we are not so far apart, you and I. Do you agree with me that the normal simple psyche, when considered to be breaking L40C1 (partner being more aware than opponents) is completely analogous to a case of MI? > Sven > Herman. From grandaeval at tiscali.co.uk Sun Nov 2 20:23:04 2008 From: grandaeval at tiscali.co.uk (Grattan) Date: Sun, 2 Nov 2008 19:23:04 -0000 Subject: [blml] Mr Gladstone suggested [SEC=UNOFFICIAL] References: <1062526051-1225456021-cardhu_decombobulator_blackberry.rim.net-888530924-@bxe065.bisx.produk.on.blackberry> <001a01c93b5c$0393aed0$0abb0c70$@no> <490B1506.4050408@skynet.be> <000301c93b8c$97769100$c663b300$@no> <490C484F.5040906@skynet.be> <000001c93c38$f9dfd200$ed9f7600$@no> <490CAD47.8050907@skynet.be><000301c93c6c$d4854c90$7d8fe5b0$@no> <490D698A.20903@skynet.be> <002d01c93ce1$22534340$0302a8c0@Mildred><490DB7E5.3030608@skynet.be> <000301c93cfb$eafff2b0$c0ffd810$@no><002701c93d06$bbd534e0$0302a8c0@Mildred> <490DD8B0.60006@skynet.be> Message-ID: <000e01c93d20$71a38740$0302a8c0@Mildred> Grattan Endicott To: "Bridge Laws Mailing List" Sent: Sunday, November 02, 2008 4:43 PM Subject: Re: [blml] Mr Gladstone suggested [SEC=UNOFFICIAL] > Grattan wrote: >> >>> >> +=+ I do not know where Herman saw me arguing this. > > I have no recollection of you argueing anything like this, or of me > suggesting that you did. > > Herman. > +=+ Thank you, Herman. Apparently someone was misquoting you. ~ Grattan ~ +=+ From svenpran at online.no Sun Nov 2 21:39:16 2008 From: svenpran at online.no (Sven Pran) Date: Sun, 2 Nov 2008 21:39:16 +0100 Subject: [blml] Mr Gladstone suggested [SEC=UNOFFICIAL] In-Reply-To: <490DFE1B.1070002@skynet.be> References: <1062526051-1225456021-cardhu_decombobulator_blackberry.rim.net-888530924-@bxe065.bisx.produk.on.blackberry> <001a01c93b5c$0393aed0$0abb0c70$@no> <490B1506.4050408@skynet.be> <000301c93b8c$97769100$c663b300$@no> <490C484F.5040906@skynet.be> <000001c93c38$f9dfd200$ed9f7600$@no> <490CAD47.8050907@skynet.be><000301c93c6c$d4854c90$7d8fe5b0$@no> <490D698A.20903@skynet.be> <002d01c93ce1$22534340$0302a8c0@Mildred> <490DB7E5.3030608@skynet.be> <000301c93cfb$eafff2b0$c0ffd810$@no> <490DD87B.3080004@skynet.be> <000501c93d19$81da8bb0$858fa310$@no> <490DFE1B.1070002@skynet.be> Message-ID: <000701c93d2b$13455a10$39d00e30$@no> On Behalf Of Herman De Wael ................. > > A partnership is of course free to (explicitly or implicitly) include in > > their understandings (which then must be disclosed to opponents in the > > prescribed way) that they can make what they call a psychic call for > > instance with a certain predefined probability. > > > > Thank you. Do you agree that this knowledge is part of partnership > understandings - apparently you do. So you agree with me that there can > be partnership understandings about psyches. Great. I wrote "what they call a psychic call". That is not to say that the call should be accepted by the director as a psychic call. One important feature with psychic calls is that the player making the call is claiming that his call is a psyche. His claim is then either accepted or rejected. > > > However, once they have specified that then this has become part of their > > partnership understanding so when they subsequently psyches (in their own > > mind) they really do not psyche but they call according to their partnership > > understanding. A possible statement that the "psyche" will occur extremely > > seldom is irrelevant; I have in my partnership understandings (and I believe > > most players have) specifications on situations that can be expected less > > than once a year. They still are part of my partnership understandings, > > > > And there we must part ways. In your opinion, there are no more psyches. > That is the same as forbidding them. Which is not what the laws intend > to do. Your argumentation cannot be the correct one. I didn't say that there are no more psyches, nor do I have any such opinion. But as I often say for simplicity: For a call to be acceptable as a psyche it must come as a complete surprise for the caller's partner. > > Then we have the second question: Are such understandings legal? > > > > Indeed a second question. In two parts. Are they legal, and are they HUM > (if weak) > > > Yes, as long as the understandings are disclosed all such understandings are > > legal with regard of the laws, but they may be illegal with regards of > > regulations. (This is where for instance HUM enters the case.) > > > > Indeed the correct answer to the first question. > > > So the complete conditions for a psychic call to be legal is that partner > > must not have more reason than opponents have to be aware of the deviation > > from understandings, and also that the call is indeed a gross violation of > > partnership understandings. (This last condition is obviously not met if the > > "psyche" has in any way been disclosed with the partnership understandings!) > > > > So what are the consequences in various situations? > > > > Aha. > > > South deliberately makes a gross deviation from partnership understandings, > > opponents have correct description of understandings and there is no doubt > > that North was at least as unaware of the situation as were opponents: We > > have a genuine psyche and there is no rectification or other reaction. > > > > OK, although I fail to see how this can be achieved if the player has > psyched ever before, under your definition of a psyche. But let's continue. I very seldom meet psychic calls, but at the Norwegian bridge festivals there are always some. I believe we have established there a standard routine whenever we meet a genuine psyche: "OK, we accept this as a psyche, but don't repeat it during this week, the next time it will not be accepted!". > > > The same, but actions by North (or other circumstances) raise suspicion that > > he could be aware of the (attempted) psyche: Depending on his judgment the > > director can accept or reject the call as a genuine psyche. If he rejects it > > the consequence should be PP against NS in addition to redress for > > opponents. In this case there can be further reactions from the attempted > > psyche found to violate regulations (e.g. HUM). > > > > And here is your answer to the question I was posing: a PP to the > offenders. Please tell me where in the laws you find the reason for this > PP. Remember that all they have so far done is provide less information > to their opponents than the director deems necessary. What is wrong with using L90? The player has violated correct procedure as prescribed in L40. > > But I note one important thing: you use the MI laws for the remainder. > So you do see that (what you call) CPU and MI are one and the same! MI is (usually) accidental, CPU is intentional. > > > Opponents have incorrect description of understandings: They will receive > > redress if they can show damage (Note that this case is independent of South > > bidding according to or contrary to partnership understandings!) This is the > > ordinary MI situation which should result in redress for opponents if they > > are found damaged, but which also at the director's judgment can result in > > PP against NS, > > > > Now you have me confused - what is the difference between the second and > third case? Partner acting on the psyche before it should be known ? The difference is L73E. > that is the second crime I called cheating in some previous post. A PP > would be too lenient IMO. Then your third case is the third crime I > alluded to. And you give only an AS if damage in that case. In what way > is that then any different than MI? > > Sven, we are not so far apart, you and I. > > Do you agree with me that the normal simple psyche, when considered to > be breaking L40C1 (partner being more aware than opponents) is > completely analogous to a case of MI? No, there is a difference (as the girl said when she saw her naked brother).. Sven From svenpran at online.no Sun Nov 2 21:44:07 2008 From: svenpran at online.no (Sven Pran) Date: Sun, 2 Nov 2008 21:44:07 +0100 Subject: [blml] Mr Gladstone suggested [SEC=UNOFFICIAL] In-Reply-To: <000e01c93d20$71a38740$0302a8c0@Mildred> References: <1062526051-1225456021-cardhu_decombobulator_blackberry.rim.net-888530924-@bxe065.bisx.produk.on.blackberry> <001a01c93b5c$0393aed0$0abb0c70$@no> <490B1506.4050408@skynet.be> <000301c93b8c$97769100$c663b300$@no> <490C484F.5040906@skynet.be> <000001c93c38$f9dfd200$ed9f7600$@no> <490CAD47.8050907@skynet.be><000301c93c6c$d4854c90$7d8fe5b0$@no> <490D698A.20903@skynet.be> <002d01c93ce1$22534340$0302a8c0@Mildred><490DB7E5.3030608@skynet.be> <000301c93cfb$eafff2b0$c0ffd810$@no><002701c93d06$bbd534e0$0302a8c0@Mildred> <490DD8B0.60006@skynet.be> <000e01c93d20$71a38740$0302a8c0@Mildred> Message-ID: <000801c93d2b$c07ceb30$4176c190$@no> On Behalf Of Grattan > >> +=+ I do not know where Herman saw me arguing this. > > > > I have no recollection of you argueing anything like this, or of me > > suggesting that you did. > > > > Herman. > > > +=+ Thank you, Herman. Apparently someone was misquoting you. > ~ Grattan ~ > +=+ I suspect what can have happened was Grattan reading my comments to Herman and mistakenly believe that they were Herman's comments to Grattan? If so then I regret causing that confusion. Regards Sven From richard.hills at immi.gov.au Sun Nov 2 23:01:25 2008 From: richard.hills at immi.gov.au (richard.hills at immi.gov.au) Date: Mon, 3 Nov 2008 09:01:25 +1100 Subject: [blml] Mr Gladstone suggested [SEC=UNOFFICIAL] In-Reply-To: <490B0F0D.7060300@skynet.be> Message-ID: Benjamin Disraeli (1804-1881): "Mr Gladstone not only appeared but rushed into the debate ... The new Members trembled and fluttered like small birds when a hawk is in the air." Herman De Wael asserted: [snip] >This thing is indeed a psyche - and it is also >a part of partnership understanding. [snip] >Psyches are gross misstatements of partnership >agreements, with agreement in its non-legal >sense (the word does not appear in the 2007 >laws anyway). Richard Hills quibbles: The word "agreement" appears many times in the 2007 Lawbook. Three particularly pertinent instances are: Law 40A heading - Players' Systemic Agreements Law 40B1(b), first sentence - Whether explicit or implicit an agreement between partners is a partnership understanding. Law 40B1(b), second sentence - A convention is included, unless the Regulating Authority decides otherwise, among the agreements and treatments that constitute special partnership understandings as is the case with any call that has an artificial meaning. WBF LC minutes, October 2008: "LAW 75C "The phrase 'they have no claim to an accurate description of the N-S hands' first appeared in the 1975 laws of the game. It was accompanied then as now by the injunction forbidding the Director to alter the table result. It was entered primarily to establish beyond doubt that that the partnership agreement must be described accurately in response to lawful enquiry and that the explanation given must not aim to describe what the explainer believes as to the contents of either hand. It was continued in those terms in the 1987 law book, while for 2007 NBOs were invited to say whether the example or the wording should be updated. Among replies received there was a general consensus for retaining them as they had been previously, whilst moving the statements from a footnote into the body of the Law." Richard Hills: The WBF LC minutes treat "partnership agreement" as synonymous with "partnership understanding", consistent with Law 40B1(b). The WBF LC minutes also note that "the partnership agreement must be described accurately". So if a call which Herman De Wael gives the label "psyche" (or, as Herman De Wael suggested, the alternative label "donkey") is a part of a partnership understanding, then it must be disclosed. In that sense, a "disclosable donkey" is not a contradiction in terms. Sven Pran and myself give a different definition to "psyche" and "mistaken call". We define what we label a "psyche" (or, as I suggested, the alternative label "elephant") as a deliberate misstatement of partnership understanding, and a Law 75C "mistaken call" as an unintentional misstatement of partnership understanding. In that sense, a "disclosable elephant" is definitely a contradiction in terms. But however Sven, Herman, DALB and myself play with semantics it is clear that the WBF views partnership agreements / understandings as disclosable, special partnership understandings as regulable, and non-partnership understandings as not disclosable. What's the problem? Best wishes Richard James Hills Recruitment Section, Level 3 Blue, workstation 15 (first on left) Department of Immigration and Citizenship Telephone: 02 6223 8453 Email: richard.hills at immi.gov.au -------------------------------------------------------------------- Important Notice: If you have received this email by mistake, please advise the sender and delete the message and attachments immediately. This email, including attachments, may contain confidential, sensitive, legally privileged and/or copyright information. Any review, retransmission, dissemination or other use of this information by persons or entities other than the intended recipient is prohibited. DIAC respects your privacy and has obligations under the Privacy Act 1988. The official departmental privacy policy can be viewed on the department's website at www.immi.gov.au. See: http://www.immi.gov.au/functional/privacy.htm --------------------------------------------------------------------- From Hermandw at skynet.be Mon Nov 3 01:02:34 2008 From: Hermandw at skynet.be (Herman De Wael) Date: Mon, 03 Nov 2008 01:02:34 +0100 Subject: [blml] Mr Gladstone suggested [SEC=UNOFFICIAL] In-Reply-To: References: Message-ID: <490E3F9A.204@skynet.be> richard.hills at immi.gov.au wrote: > > Richard Hills: > > The WBF LC minutes treat "partnership agreement" > as synonymous with "partnership understanding", > consistent with Law 40B1(b). > > The WBF LC minutes also note that "the partnership > agreement must be described accurately". > > So if a call which Herman De Wael gives the label > "psyche" (or, as Herman De Wael suggested, the > alternative label "donkey") is a part of a > partnership understanding, then it must be > disclosed. In that sense, a "disclosable donkey" > is not a contradiction in terms. > > > But however Sven, Herman, DALB and myself play > with semantics it is clear that the WBF views > partnership agreements / understandings as > disclosable, special partnership understandings > as regulable, and non-partnership understandings > as not disclosable. > > What's the problem? > The fact that I specifically used the word agreement in a non-legal sense as that from which the psyche deviates. By reintroducing the word agreement as a synonym of understanding, Richard deliberately goes around my argument again. I dare not use the word system, or he'll find some other way of twisting the words there. Let's use the word "something". Richard: a psyche is a deliberate deviation from something. Somethings form a subset of understandings. Somethings cannot be synonymous with understandings. What the somethings are is for the Director to decide, but if he rules that all understandings are somethings, then he has misinterpreted the definition of a psyche. What's the problem? > > Best wishes > > Richard James Hills Herman. From Hermandw at skynet.be Mon Nov 3 01:06:16 2008 From: Hermandw at skynet.be (Herman De Wael) Date: Mon, 03 Nov 2008 01:06:16 +0100 Subject: [blml] Mr Gladstone suggested [SEC=UNOFFICIAL] In-Reply-To: <000701c93d2b$13455a10$39d00e30$@no> References: <1062526051-1225456021-cardhu_decombobulator_blackberry.rim.net-888530924-@bxe065.bisx.produk.on.blackberry> <001a01c93b5c$0393aed0$0abb0c70$@no> <490B1506.4050408@skynet.be> <000301c93b8c$97769100$c663b300$@no> <490C484F.5040906@skynet.be> <000001c93c38$f9dfd200$ed9f7600$@no> <490CAD47.8050907@skynet.be><000301c93c6c$d4854c90$7d8fe5b0$@no> <490D698A.20903@skynet.be> <002d01c93ce1$22534340$0302a8c0@Mildred> <490DB7E5.3030608@skynet.be> <000301c93cfb$eafff2b0$c0ffd810$@no> <490DD87B.3080004@skynet.be> <000501c93d19$81da8bb0$858fa310$@no> <490DFE1B.1070002@skynet.be> <000701c93d2b$13455a10$39d00e30$@no> Message-ID: <490E4078.1060602@skynet.be> Sven Pran wrote: > > I didn't say that there are no more psyches, nor do I have any such opinion. > But as I often say for simplicity: For a call to be acceptable as a psyche > it must come as a complete surprise for the caller's partner. > Completely wrong, Sven: it must come as no more surprise to partner than to opponents. If your partner psyches, and you say it came as a complete surprise to you, then you are lying. It cannot be a complete surprise. All it can be is a certain level of surprise. As long as that level is known to opponents, all is well. Herman. From Hermandw at skynet.be Mon Nov 3 01:08:58 2008 From: Hermandw at skynet.be (Herman De Wael) Date: Mon, 03 Nov 2008 01:08:58 +0100 Subject: [blml] Mr Gladstone suggested [SEC=UNOFFICIAL] In-Reply-To: <000701c93d2b$13455a10$39d00e30$@no> References: <1062526051-1225456021-cardhu_decombobulator_blackberry.rim.net-888530924-@bxe065.bisx.produk.on.blackberry> <001a01c93b5c$0393aed0$0abb0c70$@no> <490B1506.4050408@skynet.be> <000301c93b8c$97769100$c663b300$@no> <490C484F.5040906@skynet.be> <000001c93c38$f9dfd200$ed9f7600$@no> <490CAD47.8050907@skynet.be><000301c93c6c$d4854c90$7d8fe5b0$@no> <490D698A.20903@skynet.be> <002d01c93ce1$22534340$0302a8c0@Mildred> <490DB7E5.3030608@skynet.be> <000301c93cfb$eafff2b0$c0ffd810$@no> <490DD87B.3080004@skynet.be> <000501c93d19$81da8bb0$858fa310$@no> <490DFE1B.1070002@skynet.be> <000701c93d2b$13455a10$39d00e30$@no> Message-ID: <490E411A.1090006@skynet.be> Sven Pran wrote: > > I very seldom meet psychic calls, but at the Norwegian bridge festivals > there are always some. I believe we have established there a standard > routine whenever we meet a genuine psyche: "OK, we accept this as a psyche, > but don't repeat it during this week, the next time it will not be > accepted!". > I don't think the London Bridge players would feel comfortable with that level of permitted psyching. I think therefore that Sven's expertise in these matters is tainted. Perhaps he should listen to more experienced psychers. Not that I am one. Note that the level which is apparently acceptable in Norway (one per week) is far higher than the level with which I perform my so-much-maligned H1H. Again, we see that the only problem with my psyching habits is that I am being too honest. Herman. From Hermandw at skynet.be Mon Nov 3 01:10:04 2008 From: Hermandw at skynet.be (Herman De Wael) Date: Mon, 03 Nov 2008 01:10:04 +0100 Subject: [blml] Mr Gladstone suggested [SEC=UNOFFICIAL] In-Reply-To: <000701c93d2b$13455a10$39d00e30$@no> References: <1062526051-1225456021-cardhu_decombobulator_blackberry.rim.net-888530924-@bxe065.bisx.produk.on.blackberry> <001a01c93b5c$0393aed0$0abb0c70$@no> <490B1506.4050408@skynet.be> <000301c93b8c$97769100$c663b300$@no> <490C484F.5040906@skynet.be> <000001c93c38$f9dfd200$ed9f7600$@no> <490CAD47.8050907@skynet.be><000301c93c6c$d4854c90$7d8fe5b0$@no> <490D698A.20903@skynet.be> <002d01c93ce1$22534340$0302a8c0@Mildred> <490DB7E5.3030608@skynet.be> <000301c93cfb$eafff2b0$c0ffd810$@no> <490DD87B.3080004@skynet.be> <000501c93d19$81da8bb0$858fa310$@no> <490DFE1B.1070002@skynet.be> <000701c93d2b$13455a10$39d00e30$@no> Message-ID: <490E415C.40709@skynet.be> Sven Pran wrote: >>> >> And here is your answer to the question I was posing: a PP to the >> offenders. Please tell me where in the laws you find the reason for this >> PP. Remember that all they have so far done is provide less information >> to their opponents than the director deems necessary. > > What is wrong with using L90? The player has violated correct procedure as > prescribed in L40. > Do you often hand out penalties for forgotten alerts? If you do, then your PP in this case is as meaningless as that one. Herman. From Hermandw at skynet.be Mon Nov 3 01:12:49 2008 From: Hermandw at skynet.be (Herman De Wael) Date: Mon, 03 Nov 2008 01:12:49 +0100 Subject: [blml] Mr Gladstone suggested [SEC=UNOFFICIAL] In-Reply-To: <000701c93d2b$13455a10$39d00e30$@no> References: <1062526051-1225456021-cardhu_decombobulator_blackberry.rim.net-888530924-@bxe065.bisx.produk.on.blackberry> <001a01c93b5c$0393aed0$0abb0c70$@no> <490B1506.4050408@skynet.be> <000301c93b8c$97769100$c663b300$@no> <490C484F.5040906@skynet.be> <000001c93c38$f9dfd200$ed9f7600$@no> <490CAD47.8050907@skynet.be><000301c93c6c$d4854c90$7d8fe5b0$@no> <490D698A.20903@skynet.be> <002d01c93ce1$22534340$0302a8c0@Mildred> <490DB7E5.3030608@skynet.be> <000301c93cfb$eafff2b0$c0ffd810$@no> <490DD87B.3080004@skynet.be> <000501c93d19$81da8bb0$858fa310$@no> <490DFE1B.1070002@skynet.be> <000701c93d2b$13455a10$39d00e30$@no> Message-ID: <490E4201.7060700@skynet.be> Sven Pran wrote: > >> But I note one important thing: you use the MI laws for the remainder. >> So you do see that (what you call) CPU and MI are one and the same! > > MI is (usually) accidental, CPU is intentional. > No it's not. It's things like forgetting to tell opponents every time I open a 1H in third seat that I have done that on 3 points once a year for the past ten years. Hardly intentional! Well, if you want to call something intentional a CPU, go ahead, but then my partner's offences are not CPU's. Which is not what I want, because I believe my opponents are entitled to the information. Don't you see that I'm trying to help my opponents - not hide behind "my partner doesn't know". Herman. From Hermandw at skynet.be Mon Nov 3 01:14:07 2008 From: Hermandw at skynet.be (Herman De Wael) Date: Mon, 03 Nov 2008 01:14:07 +0100 Subject: [blml] Mr Gladstone suggested [SEC=UNOFFICIAL] In-Reply-To: <000701c93d2b$13455a10$39d00e30$@no> References: <1062526051-1225456021-cardhu_decombobulator_blackberry.rim.net-888530924-@bxe065.bisx.produk.on.blackberry> <001a01c93b5c$0393aed0$0abb0c70$@no> <490B1506.4050408@skynet.be> <000301c93b8c$97769100$c663b300$@no> <490C484F.5040906@skynet.be> <000001c93c38$f9dfd200$ed9f7600$@no> <490CAD47.8050907@skynet.be><000301c93c6c$d4854c90$7d8fe5b0$@no> <490D698A.20903@skynet.be> <002d01c93ce1$22534340$0302a8c0@Mildred> <490DB7E5.3030608@skynet.be> <000301c93cfb$eafff2b0$c0ffd810$@no> <490DD87B.3080004@skynet.be> <000501c93d19$81da8bb0$858fa310$@no> <490DFE1B.1070002@skynet.be> <000701c93d2b$13455a10$39d00e30$@no> Message-ID: <490E424F.1080707@skynet.be> Sven Pran wrote: >>> >> Now you have me confused - what is the difference between the second and >> third case? Partner acting on the psyche before it should be known ? > > The difference is L73E. > But what is the penalty for breaking L73E? It doesn't say. IMO, it's just MI. Herman. From nigelguthrie at talktalk.net Mon Nov 3 01:22:44 2008 From: nigelguthrie at talktalk.net (Nigel Guthrie) Date: Mon, 03 Nov 2008 00:22:44 +0000 Subject: [blml] psychs In-Reply-To: <4E674A6861864A729C1A16BB86EC8652@MARVLAPTOP> References: <3032BE6F15B14B4DA5FF8D18E93D4E63@MARVLAPTOP><000701c93cb6$c03ff060$0302a8c0@Mildred> <490D921F.2060705@talktalk.net> <4E674A6861864A729C1A16BB86EC8652@MARVLAPTOP> Message-ID: <490E4454.3050608@talktalk.net> [Marvin French] Please tell me in detail how psych and falsecard possibilities should be disclosed. "We psych a lot," one young pair announced to us at an NABC. I was happy to face opponents who played the whole game of bridge, not just a namby-pamby subset. [Nige1] Currently, I don't see a simple way that you can declare psyching propensities but I do have a few relevant comments. Sloppy nomenclature hampers discussion of disclosure issues. Several of us have attempted to tighten the definitions of basic terms but, predictably, we attract the same criticism as "Humpty Dumpty". Unnecessary sophisticated rules make the situation worse. For example complex disclosure protocols and system restrictions. But we must endure these for the forseeable future. Incidentally, I don't understand why our concern is with *gross* deviation. Agreements about *minor* deviations are more common, confer a similar advantage, and are harder to nail down. If you often pysch then you tend to favour certain kinds. Thus it is hard for partner to avoid inducing a pattern. Examples - Opening light third in hand. - Overcalling comic notrumps. - psyching a suit after LHO doubles an opening bid. - so-called "tactical" bids (false trial bids, cue-bids, exclusion bids and so on). - strange overcalls when losing a match by more than a certain margin. If you diagnose such a pattern it is hard for you to disclose it and anyway opponents may not know where to find it on your system card. Years ago, On BLML, I suggested a basis on which a parterrship might agree to randomise "psychs". For example: "Psych" 1C if you hold 4 deuces, 1D if you hold 4 treys, and so on. If you don't declare this then your partnership may have an advantage. For example if partner opens 1C and you hold any deuce, you know his call is genuine. Personally, I see no harm in this, if properly declared (although it may fall foul of encryption rules). Currently, however, I see no alternative to taking partner's actions at face value unless your peers would agree with you that there is overwhelming legally acquired evidence to the contrary. From richard.hills at immi.gov.au Mon Nov 3 02:29:49 2008 From: richard.hills at immi.gov.au (richard.hills at immi.gov.au) Date: Mon, 3 Nov 2008 12:29:49 +1100 Subject: [blml] Mr Gladstone suggested [SEC=UNOFFICIAL] In-Reply-To: <490E3F9A.204@skynet.be> Message-ID: Henry Labouchere (1831-1912), on Gladstone: "I do not object to the old man always having a card up his sleeve, but I do object to his insinuating that the Almighty has placed it there." Richard Hills: >>>But however Sven, Herman, DALB and myself play >>>with semantics it is clear that the WBF views >>>partnership agreements / understandings as >>>disclosable, special partnership understandings >>>as regulable, and non-partnership understandings >>>as not disclosable. >>> >>>What's the problem? Herman De Wael: >>Richard: a psyche is a deliberate deviation from >>something. >> >>Somethings form a subset of understandings. >> >>Somethings cannot be synonymous with >>understandings. >> >>What the somethings are is for the Director to >>decide, but if he rules that all understandings >>are somethings, then he has misinterpreted the >>definition of a psyche. >> >>What's the problem? Grattan Endicott: >...In fact my position is that the law provides >that a psyche becomes part of system if there is >a partnership understanding attached to it, and >as part of system it may be regulated and will be >unlawful if there is a violation of the system >regulations. Frequency may create an implicit >partnership understanding - something for the >Director to assess. > >Subscribers will note that in general I cite >positions under the laws, leaving others to >argue their personal opinions. I have not >expressed any personal opinion as to when the >line is crossed from psyche to partnership >understanding: in fact my opinion is that it is >the task, case by case, of the Director and AC >(with such guidance as the RA may give) to judge >when the line is crossed. > ~ Grattan ~ +=+ Law 17D2 extract plus footnote: "...the Director shall award artificial adjusted scores when offender's substituted call differs* from his cancelled call... * For example, a substituted call differs if its meaning is much different or if it is psychic." Richard Hills: Okay, I retract my previous assertion that Sven, Herman, DALB and I are merely playing with semantics. Since the word "psychic" appears in the Law 17 footnote, correct interpretation of what the Lawbook means in its Definition of "psychic call" is of primary importance in a Director giving a ruling under Law 17D2. Only of secondary importance is "when the line is crossed from psyche to partnership understanding". Best wishes Richard James Hills Recruitment Section, Level 3 Blue, workstation 15 (first on left) Department of Immigration and Citizenship Telephone: 02 6223 8453 Email: richard.hills at immi.gov.au -------------------------------------------------------------------- Important Notice: If you have received this email by mistake, please advise the sender and delete the message and attachments immediately. This email, including attachments, may contain confidential, sensitive, legally privileged and/or copyright information. Any review, retransmission, dissemination or other use of this information by persons or entities other than the intended recipient is prohibited. DIAC respects your privacy and has obligations under the Privacy Act 1988. The official departmental privacy policy can be viewed on the department's website at www.immi.gov.au. See: http://www.immi.gov.au/functional/privacy.htm --------------------------------------------------------------------- From rfrick at rfrick.info Mon Nov 3 03:20:51 2008 From: rfrick at rfrick.info (Robert Frick) Date: Sun, 02 Nov 2008 21:20:51 -0500 Subject: [blml] Mr Gladstone suggested [SEC=UNOFFICIAL] In-Reply-To: <002701c93d06$bbd534e0$0302a8c0@Mildred> References: <1062526051-1225456021-cardhu_decombobulator_blackberry.rim.net-888530924-@bxe065.bisx.produk.on.blackberry> <001a01c93b5c$0393aed0$0abb0c70$@no> <490B1506.4050408@skynet.be> <000301c93b8c$97769100$c663b300$@no> <490C484F.5040906@skynet.be> <000001c93c38$f9dfd200$ed9f7600$@no> <490CAD47.8050907@skynet.be> <000301c93c6c$d4854c90$7d8fe5b0$@no> <490D698A.20903@skynet.be> <002d01c93ce1$22534340$0302a8c0@Mildred> <490DB7E5.3030608@skynet.be> <000301c93cfb$eafff2b0$c0ffd810$@no> <002701c93d06$bbd534e0$0302a8c0@Mildred> Message-ID: Perhaps saying the obvious, but a line is crossed when you think your psyche (or any intentional misdescription) is safer with your regular partner than with an unknown partner. I know that is probably unenforceable. From richard.hills at immi.gov.au Mon Nov 3 04:15:23 2008 From: richard.hills at immi.gov.au (richard.hills at immi.gov.au) Date: Mon, 3 Nov 2008 14:15:23 +1100 Subject: [blml] Mr Lincoln suggested [SEC=UNOFFICIAL] In-Reply-To: Message-ID: Law 72B2: "There is no obligation to draw attention to an infraction of law committed by one's own side (but see Law 20F for a mistaken explanation and see Laws 62A and 79A2)." Robert Frick: >Perhaps saying the obvious, but a line is crossed when >you think your psyche (or any intentional >misdescription) is safer with your regular partner than >with an unknown partner. I know that is probably >unenforceable. Richard Hills: No, self-enforceable. Law 72B2 requires you to call the Director against yourself when mature reflection causes you to realise that your intended psyche had actually crossed the line into being a concealed implicit mutual partnership understanding. Annabel Crabb, Sydney Morning Herald, November 1st 2008: [big snip] There's a great story about Abraham Lincoln, who in 1860 was also a youngish new face from Illinois in eager pursuit of the presidency. John Scripps, of the Chicago Tribune, pressed Lincoln for autobiographical details to fill out a formal campaign biography. Lincoln told the pressman that there was nothing to offer but "the short and simple annals of the poor". Scripps persisted, and when Lincoln received an advance copy of the biography, he noticed that it credited him with reading Plutarch, among other things. Lincoln returned the copy, telling Scripps that he had not, in fact, read Plutarch before, but since reading the biography had read some Plutarch, so as to render the portrayal accurate. "A scrupulous teller of the truth," concluded Scripps later, "too exact in his notions to suit the atmosphere of Washington as it is now." Best wishes Richard James Hills Recruitment Section, Level 3 Blue, workstation 15 (first on left) Department of Immigration and Citizenship Telephone: 02 6223 8453 Email: richard.hills at immi.gov.au -------------------------------------------------------------------- Important Notice: If you have received this email by mistake, please advise the sender and delete the message and attachments immediately. This email, including attachments, may contain confidential, sensitive, legally privileged and/or copyright information. Any review, retransmission, dissemination or other use of this information by persons or entities other than the intended recipient is prohibited. DIAC respects your privacy and has obligations under the Privacy Act 1988. The official departmental privacy policy can be viewed on the department's website at www.immi.gov.au. See: http://www.immi.gov.au/functional/privacy.htm --------------------------------------------------------------------- From Hermandw at skynet.be Mon Nov 3 09:31:41 2008 From: Hermandw at skynet.be (Herman De Wael) Date: Mon, 03 Nov 2008 09:31:41 +0100 Subject: [blml] Mr Lincoln suggested [SEC=UNOFFICIAL] In-Reply-To: References: Message-ID: <490EB6ED.7080805@skynet.be> richard.hills at immi.gov.au wrote: > Law 72B2: > > "There is no obligation to draw attention to an > infraction of law committed by one's own side (but see > Law 20F for a mistaken explanation and see Laws 62A and > 79A2)." > > Robert Frick: > >> Perhaps saying the obvious, but a line is crossed when >> you think your psyche (or any intentional >> misdescription) is safer with your regular partner than >> with an unknown partner. I know that is probably >> unenforceable. > > Richard Hills: > > No, self-enforceable. Law 72B2 requires you to call the > Director against yourself when mature reflection causes > you to realise that your intended psyche had actually > crossed the line into being a concealed implicit mutual > partnership understanding. > Quite fair. But Richard, try saying it like this: Law 72B2 requires you to call the Director against yourself when mature reflection causes you to realise that your psyche had actually crossed the line into being a psyche with undisclosed partnership understanding. Same meaning, other words, not so negative. By calling something a "concealed implicit mutual partnership understanding" you imply that you should be punished for making the call in the first place. Rather, we should focus on what the director will do after you call her: check if the opponents have been damaged through the non-disclosure, and award an AS if she believes they have. Or what do you think the penalty for a "concealed implicit mutual partnership understanding" ought to be - disbarment? Herman. From grandaeval at tiscali.co.uk Sat Nov 1 01:04:37 2008 From: grandaeval at tiscali.co.uk (Grattan) Date: Sat, 1 Nov 2008 00:04:37 -0000 Subject: [blml] WBFLC Minutes Oct 10th 08. Message-ID: <001f01c93bb5$e85abd70$0302a8c0@Mildred> Grattan Endicott Hi BLMLers, A player makes a call and then asks for the TD saying: "I made a mistake and would like to change my call." The TD deems, and the player agree, that the call was not unintended (Law 25A), but the player would like to try another call, hoping LHO will agree (Law 25B). 1) Must the TD allow the new call, explaining consequences of Law 25B ? 2) If the player just says " I made a mistake (not unintended), does the TD should offer this option: "You can try another call, but..(Law 25B)." ? 3) If your answer is yes, is this option already on after LHO subsequently calls ? Thx in advance. Laval Du Breuil Quebec City -------------- next part -------------- An HTML attachment was scrubbed... URL: http://www.amsterdamned.org/pipermail/blml/attachments/20081102/1378f09f/attachment.htm From agot at ulb.ac.be Mon Nov 3 09:59:39 2008 From: agot at ulb.ac.be (Alain Gottcheiner) Date: Mon, 03 Nov 2008 09:59:39 +0100 Subject: [blml] psychs In-Reply-To: <3032BE6F15B14B4DA5FF8D18E93D4E63@MARVLAPTOP> References: <3032BE6F15B14B4DA5FF8D18E93D4E63@MARVLAPTOP> Message-ID: <490EBD7B.5080708@ulb.ac.be> Marvin L French a ?crit : > All this talk about psychs has me confused. > > Playing with a fairly expert partner for the first time, he opened > 1S in first position, favorable vulnerability. Next hand doubled, I > redoubled with my flat 13 HCP, and LHO jumped to 3H, raised to 4H by > RHO. > > Unless the oppoents are insane my partner had to have psyched an > opening, so I didn't double. That was okay, right? > > In the same session, with the same vulnerability, the auction was > identical and I had the same flat 13 HCP. Are you all telling me I > now had to double or be accused of something? > AG : unless something very special happened, I'd say you should call the TD and tell him you already played this deal. But, apart from that, if your partner is prone to do the same psyche twice in a session, this should be written in bold and framed on your SC. Remember, a bid is a psyche only if the amount of surprise is as great for you as for your opponents. Best regards Alain From agot at ulb.ac.be Mon Nov 3 10:31:18 2008 From: agot at ulb.ac.be (Alain Gottcheiner) Date: Mon, 03 Nov 2008 10:31:18 +0100 Subject: [blml] psychs In-Reply-To: References: <3032BE6F15B14B4DA5FF8D18E93D4E63@MARVLAPTOP><000701c93cb6$c03ff060$0302a8c0@Mildred> <490D6B10.8030905@skynet.be> Message-ID: <490EC4E6.8060200@ulb.ac.be> Marvin L French a ?crit : > Herman wrote: > >> If, OTOH, a psyche is treated as any other piece of partnership >> understanding, which means that although the opponents are >> entitled to >> know about it, the penalty for them not being told is an >> adjustment if >> such absent knowledge caused damage, then the game (as Marv has >> played >> and loved it for 60 years) can be played as before. >> >> > But how do I inform opponents that we play the total game, which > includes an occasional psych that will not affect a partner's > bidding unless there is conclusive evidence (acknowledgeable by any > experienced player) that a psych must have been perpetrated? > AG : unless you psyche equally every bid, you should disclose your partnership's psychic tendencies (which bids and under which circumstances). There is a space for this on the SC. For examples, some never psyche an opening, but might psyche a 1/1 response. I even know of a pair who wrote "only when angry". Now that they rached 4H, psyche notwithstanding, they still have a right to know whether it's more probable that partner psyched his opening, or that you psyched your redouble. Both are possible, the answer depends on the partnership's experience and they are entitled to know it. Since asking you would be embarrassing whetehr they get the right answer or not, the SC seems the right solution. Best regards Alain From PeterEidt at t-online.de Mon Nov 3 11:16:07 2008 From: PeterEidt at t-online.de (Peter Eidt) Date: Mon, 03 Nov 2008 11:16:07 +0100 Subject: [blml] =?iso-8859-15?q?Law_25B?= In-Reply-To: <000001c93d39$aa784a10$ff68de30$@com> References: <000001c93d39$aa784a10$ff68de30$@com> Message-ID: <1KwwTf-1KRvBg0@fwd10.aul.t-online.de> From: "Picatou" > A player makes a call and then asks ?for the TD saying: "I made a > mistake and would like to change my call." The TD ?deems, and > the player agree, that the call was not unintended (Law 25A), but > the player would like to try another call, hoping LHO will agree > (Law 25B). > > 1) ? ? ?Must the TD allow the new call, explaining consequences of > Law 25B ? NO. In fact: he must not! > 2) ? ? ?If the player just says " I made a mistake (not unintended), > does the TD should offer this option: "You can try another call, but ... (Law 25B)." ? NO > 3) ? ? ?If your answer is yes, is this option already on after LHO > subsequently calls ? No YES, no answer ;-) ? Peter > Thx in advance. > Laval Du Breuil From grandaeval at tiscali.co.uk Mon Nov 3 11:43:36 2008 From: grandaeval at tiscali.co.uk (Grattan) Date: Mon, 3 Nov 2008 10:43:36 -0000 Subject: [blml] Mr Gladstone suggested [SEC=UNOFFICIAL] References: <490E3F9A.204@skynet.be> Message-ID: <001401c93da1$0882d230$0302a8c0@Mildred> Grattan Endicott To: "Bridge Laws Mailing List" Sent: Monday, November 03, 2008 12:02 AM Subject: Re: [blml] Mr Gladstone suggested [SEC=UNOFFICIAL] > richard.hills at immi.gov.au wrote: >> >> Richard Hills: >> >> The WBF LC minutes treat "partnership agreement" >> as synonymous with "partnership understanding", >> consistent with Law 40B1(b). >> >> The WBF LC minutes also note that "the partnership >> agreement must be described accurately". >> >> So if a call which Herman De Wael gives the label >> "psyche" (or, as Herman De Wael suggested, the >> alternative label "donkey") is a part of a >> partnership understanding, then it must be >> disclosed. In that sense, a "disclosable donkey" >> is not a contradiction in terms. >> >> >> But however Sven, Herman, DALB and myself play >> with semantics it is clear that the WBF views >> partnership agreements / understandings as >> disclosable, special partnership understandings >> as regulable, and non-partnership understandings >> as not disclosable. >> >> What's the problem? >> > > The fact that I specifically used the word agreement in a non-legal > sense as that from which the psyche deviates. By reintroducing the word > agreement as a synonym of understanding, Richard deliberately goes > around my argument again. > > I dare not use the word system, or he'll find some other way of twisting > the words there. Let's use the word "something". > > Richard: a psyche is a deliberate deviation from something. > Somethings form a subset of understandings. > Somethings cannot be synonymous with understandings. > What the somethings are is for the Director to decide, but if he rules > that all understandings are somethings, then he has misinterpreted the > definition of a psyche. > > What's the problem? > >> >> Best wishes >> >> Richard James Hills > > Herman. > > _______________________________________________ > blml mailing list > blml at amsterdamned.org > http://www.amsterdamned.org/mailman/listinfo/blml From grandaeval at tiscali.co.uk Mon Nov 3 12:46:27 2008 From: grandaeval at tiscali.co.uk (Grattan) Date: Mon, 3 Nov 2008 11:46:27 -0000 Subject: [blml] Mr Burn suggested References: <48FC56AB.1050505@skynet.be> <02A30BE1-5E95-41F3-AE74-1936A0913D29@starpower.net> <48FCAB1E.8010201@skynet.be> <2a1c3a560810201658h1877df31n5886171a8165db7c@mail.gmail.com> <2a1c3a560810201952j563d60f9ua2096c951f2885b8@mail.gmail.com> <48FDDBFF.7070300@connecttime.net> Message-ID: <002301c93dab$79209450$0302a8c0@Mildred> Grattan Endicott <48FC56AB.1050505@skynet.be> <02A30BE1-5E95-41F3-AE74-1936A0913D29@starpower.net> <48FCAB1E.8010201@skynet.be> <2a1c3a560810201658h1877df31n5886171a8165db7c@mail.gmail.com> <2a1c3a560810201952j563d60f9ua2096c951f2885b8@mail.gmail.com> <48FDDBFF.7070300@connecttime.net> Message-ID: <002401c93dab$7960f8b0$0302a8c0@Mildred> Grattan Endicott Partner's ability to describe the method to opponent demonstrates the existence of a partnership understanding. This may be regulated by the RA which may allow it, disallow it, or allow it conditionally. ~ Grattan ~ +=+ From grandaeval at tiscali.co.uk Mon Nov 3 13:25:43 2008 From: grandaeval at tiscali.co.uk (Grattan) Date: Mon, 3 Nov 2008 12:25:43 -0000 Subject: [blml] Control of methods Message-ID: <003f01c93daf$4df0af00$0302a8c0@Mildred> Grattan Endicott References: <003f01c93daf$4df0af00$0302a8c0@Mildred> Message-ID: <490EF4BA.6080705@skynet.be> Grattan wrote: > > Grattan Endicott also ************************************** > "Let us then be up and doing, > With a heart for any fate, > Still achieving, still pursuing, > Learn to labour and to wait." > [H.W. Longfellow] > > ''''''''''''''''''''''''''''''''''''''''''''''''''''''''''''''''''''''''''''''''''''''''''''''''' > > But isn't "then form part of the partnership's methods" exactly the > same as "become a part of system"? > > No it doesn't - not in the sense of system regulations. IMO. > > +=+ The foregoing remarks seem to imply that we will find in the > laws a reference to 'system regulations'. I do not recall that there > is one. In the 2007 Laws it is on partnership understandings that > controls are authorized, and in this regard the Regulating Authority > is given absolute discretion as to what it will allow and what it will > not allow. > ~ Grattan ~ +=+ > Fully correct Grattan, but: If a RA decides that for the purpose of their system regulations all partnership understandings are part of the regulated system, then they will find that they have written a set of regulations that ban (some) psyches. Such a set of regulations would be contrary to the laws. In order for a set of system regulations to be workable, there HAS to be a difference between the set of partnership understandings and the set of partnership agreements, with only the last (sub)set being subject to the regulations in force. I believe that the WBF system regulations make such a distinction. Herman. From Hermandw at skynet.be Mon Nov 3 13:57:40 2008 From: Hermandw at skynet.be (Herman De Wael) Date: Mon, 03 Nov 2008 13:57:40 +0100 Subject: [blml] Mr Gladstone suggested [SEC=UNOFFICIAL] In-Reply-To: <001401c93da1$0882d230$0302a8c0@Mildred> References: <490E3F9A.204@skynet.be> <001401c93da1$0882d230$0302a8c0@Mildred> Message-ID: <490EF544.3030606@skynet.be> Grattan wrote: > > Grattan Endicott also ************************************** > "Let us then be up and doing, > With a heart for any fate, > Still achieving, still pursuing, > Learn to labour and to wait." > [H.W. Longfellow] > > ''''''''''''''''''''''''''''''''''''''''''''''''''''''''''''''''''''''''''''''''''''''''''''''''' > +=+ This topic has become unreal. The language and the discussion > are so confused that it is virtually impossible to make any sense of it. > As it is I doubt that it is of the least value to subscribers generally. > ~ G ~ +=+ > I apologise for the confusion created. I should not have hammered on Richard and Sven for their use of language. Instead, I should have focused on the penalties these esteemed contributors would hand out for their so-called "illegal concealed partnership mutual understandings". I shall attempt to do so in a new thread called "if it quacks". Herman. From Hermandw at skynet.be Mon Nov 3 14:15:17 2008 From: Hermandw at skynet.be (Herman De Wael) Date: Mon, 03 Nov 2008 14:15:17 +0100 Subject: [blml] If it quacks Message-ID: <490EF965.6070004@skynet.be> like a duck, it is most probably a duck. We've compared psyches to elephants and donkeys already, maybe we should try ducks. Consider an opening of 1H, third in hand, on less than normally expected values. Now Sven and Richard call this thing a teal. I prefer to call it a duck. After all, it quacks. For non-native english speakers, a teal is a type of duck, considered by some to be a separate animal but in reality one of several species within the genus Anas (ducks). For some days, I have been trying to convince Sven and Richard that this thing is actually a duck. I have not succeeded, and Grattan recently called the topic unreal. I believe he is right. There is no sense in discussing the names - all we need to do is see what we shall do with a teal. So we call the Director. We point out that the hand that opened 1H does not conform to the normal expectations of such an opening. Let's assume that a deviation is easily seen. So the Director starts an investigation. As a first step, she will find out, presumably, that the opener has done this before. She will attempt to form an idea of the frequency with which this teal has happened before. As part of that first step, she may try to establish if partner knew what this frequency was. I don't believe, however, that this information is relevant. It is my firm opinion that the deviation frequency is part of the information to which the opponents are entitled, even when the partner does not actually know that frequency. The fact that he could know it should suffice. Then in a second step, the Director will investigate how much the opponents knew about the frequency. She will add: - any mention on the SC; - any mention at the table by the partner; - general bridge knowledge and form an idea about the level to which the opponents could have suspected that a deviation might take place. Then she compares these two levels. If she finds that the partner had no more reason to suspect that a deviation had occured than opponents, she rules that all is well and good. So let's suppose she finds that opponents did not have all the information they should have had. Now for the third step. I believe that step is to check for damage. The Director will ask the opponents if they would have acted differently if they had know about the heightened expectancy of a deviation. Of course the question is not "if you had known that it was this weak ..." but rather "if you had know that it could have been this weak more often than ...". Quite a lot of deviations will be judged to be without damage. If however, the damage is real, the Director will start the fourth step and find an adjusted score based on what would have happened with complete information. Now, in my opinion, this animal quacks like a piece of MI. I therefor suggested to call it a piece of MI. Sven and Richard may continue to call it a ICMPU, or a teal. But do they (and you) agree that the MI rules are the ones to use? Or do they really believe a teal is not a duck? Herman. From svenpran at online.no Mon Nov 3 15:17:31 2008 From: svenpran at online.no (Sven Pran) Date: Mon, 3 Nov 2008 15:17:31 +0100 Subject: [blml] If it quacks In-Reply-To: <490EF965.6070004@skynet.be> References: <490EF965.6070004@skynet.be> Message-ID: <001d01c93dbe$e95d82b0$bc188810$@no> On Behalf Of Herman De Wael > like a duck, it is most probably a duck. > > We've compared psyches to elephants and donkeys already, maybe we should > try ducks. > > Consider an opening of 1H, third in hand, on less than normally expected > values. > > Now Sven and Richard call this thing a teal. I can't speak for Richard, but I have said this before and apparently must say it again: Don't tell me what I think; you are not a qualified mind reader. And I have never used the word "teal" for any purpose. A psyche is a well-defined term in the bridge laws; it is a call with some special properties. The collection of all misbids form a subset of all calls and the collection of all psyches form a subset of all misbids. Therefore a psyche is also a misbid, but a misbid is not necessarily a psyche just as a misbid is also a call but a call is not necessarily a misbid. Misinformation is not a call at all; it is an incorrect (or missing where required) disclosure of partnership understanding. Sven (the rest of this rubbish deliberatly snipped) From picatou at picatou.com Mon Nov 3 15:19:10 2008 From: picatou at picatou.com (Picatou) Date: Mon, 3 Nov 2008 09:19:10 -0500 Subject: [blml] Law 25B In-Reply-To: <1KwwTf-1KRvBg0@fwd10.aul.t-online.de> References: <000001c93d39$aa784a10$ff68de30$@com> <1KwwTf-1KRvBg0@fwd10.aul.t-online.de> Message-ID: <000001c93dbf$2422fb50$6c68f1f0$@com> Thx Peter, I have seen the same answer in the ACBL Bulletin this morning. The ACBL Law Commission said NO. Laval Du Breuil -----Message d'origine----- De?: blml-bounces at amsterdamned.org [mailto:blml-bounces at amsterdamned.org] De la part de Peter Eidt Envoy??: 3 novembre 2008 05:16 ??: Bridge Laws Mailing List Objet?: Re: [blml] Law 25B From: "Picatou" > A player makes a call and then asks ?for the TD saying: "I made a > mistake and would like to change my call." The TD ?deems, and > the player agree, that the call was not unintended (Law 25A), but > the player would like to try another call, hoping LHO will agree > (Law 25B). > > 1) ? ? ?Must the TD allow the new call, explaining consequences of > Law 25B ? NO. In fact: he must not! > 2) ? ? ?If the player just says " I made a mistake (not unintended), > does the TD should offer this option: "You can try another call, but ... (Law 25B)." ? NO > 3) ? ? ?If your answer is yes, is this option already on after LHO > subsequently calls ? No YES, no answer ;-) ? Peter > Thx in advance. > Laval Du Breuil _______________________________________________ blml mailing list blml at amsterdamned.org http://www.amsterdamned.org/mailman/listinfo/blml From ehaa at starpower.net Mon Nov 3 15:28:58 2008 From: ehaa at starpower.net (Eric Landau) Date: Mon, 3 Nov 2008 09:28:58 -0500 Subject: [blml] Mr Gladstone suggested In-Reply-To: <2a1c3a560810311545l386979b3vec0c724581100f66@mail.gmail.com> References: <1062526051-1225456021-cardhu_decombobulator_blackberry.rim.net-888530924-@bxe065.bisx.produk.on.blackberry> <2a1c3a560810311251o1b9ea039j39b200467f77c725@mail.gmail.com> <000001c93ba4$f34d3b10$d9e7b130$@com> <2a1c3a560810311545l386979b3vec0c724581100f66@mail.gmail.com> Message-ID: <88BF54A6-5114-40E9-9677-A7E861FFDB59@starpower.net> On Oct 31, 2008, at 6:45 PM, Wayne Burrows wrote: > 2008/11/1 David Burn : > >> If your convention card says "1S shows 7+ hcp" then you are >> playing a HUM >> (whatever else it says). If your convention card says "1S shows 8+ >> hcp" but >> in fact you open all (or "almost all") 7-point hands 1S then you >> are playing >> a HUM (and you are trying to conceal the fact). >> >> If you do not open all (or "almost all") 7-point hands with a bid >> at the one >> level, then: >> >> If you open ten 7-counts for every one you pass, then you are >> playing a HUM >> (and you may not escape the charge of playing a concealed HUM by >> exhibiting >> the one seven-count you passed). If you pass ten seven-counts for >> every one >> you open, then you are not playing a HUM (and you may escape the >> charge of >> playing a concealed HUM by exhibiting the ten seven-counts you >> passed). >> >> This ratio of ten to one is based on nothing but my own intuition >> - other >> people may have a different figure in mind. But some figure there >> must be, >> or (as Wayne correctly says) the WBF policy on psychic bidding >> will be >> ineffective (since it will be meaningless). > > I have two problems with this: > > 1. There is a huge gulf between 10:1 and 1:10 so this does not tell > us where the boundary is. I see no possibility of a "bright line" here, so any boundary would necessarily be subjective. But the appropriate subjective boundary would be pretty obvious: allow opening those 7-counts for which the player can offer a sensible justification for treating them as the playing-strength equivalent of at least an average 8-count. (My unsupported intuition is that something around 10-20% of 7-counts would qualify, but the point is that the criterion not depend on the actual frequency.) Eric Landau 1107 Dale Drive Silver Spring MD 20910 ehaa at starpower.net From agot at ulb.ac.be Mon Nov 3 15:30:39 2008 From: agot at ulb.ac.be (Alain Gottcheiner) Date: Mon, 03 Nov 2008 15:30:39 +0100 Subject: [blml] If it quacks In-Reply-To: <490EF965.6070004@skynet.be> References: <490EF965.6070004@skynet.be> Message-ID: <490F0B0F.6040404@ulb.ac.be> Herman De Wael a ?crit : > Now for the third step. I believe that step is to check for damage. The > Director will ask the opponents if they would have acted differently if > they had know about the heightened expectancy of a deviation. Of course > the question is not "if you had known that it was this weak ..." but > rather "if you had know that it could have been this weak more often > than ...". Quite a lot of deviations will be judged to be without damage. > AG : perhaps it should be that way ; but you know as well as I do that in Belgium, a light opening not adhering to L/18 will be considered to be, not a psyche, but use of HUM. Which leads to penalties even without damage, and demands to change systems. Whence we're in a ridiculous situation where you may make gross misstatements, but not milder ones. NB : you'll like this one. Two female students asked for a demo of some computer feature. So I showed them while they were standing slightly behind my chair, on the left side (I'm left-handed). When I mumbled 'me voil? avec deux slips', there were some anxious looks... Best regards Alain From agot at ulb.ac.be Mon Nov 3 15:41:45 2008 From: agot at ulb.ac.be (Alain Gottcheiner) Date: Mon, 03 Nov 2008 15:41:45 +0100 Subject: [blml] If it quacks In-Reply-To: <001d01c93dbe$e95d82b0$bc188810$@no> References: <490EF965.6070004@skynet.be> <001d01c93dbe$e95d82b0$bc188810$@no> Message-ID: <490F0DA9.8070105@ulb.ac.be> Sven Pran a ?crit : > > The collection of all misbids form a subset of all calls and the collection > of all psyches form a subset of all misbids. > No, Sir. To misbid is to bid *involuntarily* in a way that doesn't correspond to the system. This can be seen from the prefix 'mis' , which carries the idea of 'wrong'. A psyche is made on purpose, it isn't an error, so it doesn't pertain to the set of misbids. However, psyches form a subset of non-systemic bids. Regards Alain From ehaa at starpower.net Mon Nov 3 15:57:08 2008 From: ehaa at starpower.net (Eric Landau) Date: Mon, 3 Nov 2008 09:57:08 -0500 Subject: [blml] New Laws don't allow UI situation to be resolved? In-Reply-To: References: <2b1e598b0810302300t4f888261j8d7615f57fcdd4ee@mail.gmail.com> <000301c93b33$398b9670$aca2c350$@no> <001b01c93b5d$4438f2f0$ccaad8d0$@no> <000001c93c08$ea3dbf90$beb93eb0$@no> <000101c93c49$99440220$cbcc0660$@no> Message-ID: <8550C64B-0545-409B-B0A7-EFA5C73BA03F@starpower.net> On Nov 1, 2008, at 3:21 PM, Robert Frick wrote: > If partner supplies the UI, I understand that I cannot and should not > choose an LA suggested by the UI. But if the UI is extraneous and > it is > not my fault that I have it, then it does not seem fair if I am > ethically > obligated not to select alternatives suggested by the UI. Suggested LAs and all that are exclusive to L16B. When the UI does not come from partner, L16C governs, and your only obligation is to call the Director. Eric Landau 1107 Dale Drive Silver Spring MD 20910 ehaa at starpower.net From Hermandw at skynet.be Mon Nov 3 16:01:54 2008 From: Hermandw at skynet.be (Herman De Wael) Date: Mon, 03 Nov 2008 16:01:54 +0100 Subject: [blml] If it quacks In-Reply-To: <001d01c93dbe$e95d82b0$bc188810$@no> References: <490EF965.6070004@skynet.be> <001d01c93dbe$e95d82b0$bc188810$@no> Message-ID: <490F1262.5090703@skynet.be> Sven Pran wrote: > > Sven > > (the rest of this rubbish deliberatly snipped) > No more needs to be said, then. I shall from now on treat everything Sven writes as rubbish as well. He needs no longer to wait for any answer. Please allow me to make one thing clear - not responding to something by Sven does not mean I agree, from now on. Herman. From ehaa at starpower.net Mon Nov 3 16:56:27 2008 From: ehaa at starpower.net (Eric Landau) Date: Mon, 3 Nov 2008 10:56:27 -0500 Subject: [blml] New Laws don't allow UI situation to be resolved? In-Reply-To: <490D98CC.6040908@talktalk.net> References: <2b1e598b0810302300t4f888261j8d7615f57fcdd4ee@mail.gmail.com><000301c93b33$398b9670$aca2c350$@no><001b01c93b5d$4438f2f0$ccaad8d0$@no><000001c93c08$ea3dbf90$beb93eb0$@no><000101c93c49$99440220$cbcc0660$@no> <001a01c93c5b$6d020830$0302a8c0@Mildred> <490CCA60.5060607@meadows.pair.com><000801c93cb6$c082c5c0$0302a8c0@Mildred> <490D6BA7.30607@skynet.be> <002c01c93ce1$221094f0$0302a8c0@Mildred> <490D98CC.6040908@talktalk.net> Message-ID: <5AAF2C28-908F-43D3-95E2-5767DA109E8D@starpower.net> On Nov 2, 2008, at 7:10 AM, Nigel Guthrie wrote: > [Herman de Wael] > That is just plain silly, Grattan. The laws have been altered so as to > allow for more boards to be played. If you call the TD on the first > occasion, he will no doubt apply (c) allow play to continue. Then you > call him again a second time - now what? > > [Grattan Endicott] > +=+ "Just plain silly", if you say so, but nevertheless it is the > law. Brian should have informed the Director when he heard > the remark, not when he came to play the board (Law 16C1). The > Director would then have control of the matter before > Brian had the board in front of him. Having by then discovered the > nature of the hand and the significance of the UI only a much > inferior Director would have allowed play of the board. In the > situation Brian reached, through oversight of his Law > 16C1 duty, one could expect the Director to award an adjusted score > because he judged (Law 12A2) that the UI was such as > not to permit normal play of the board, but in addition to apply a > penalty to Brian for his breach of law. > > [Nige1] > Again, in this case, the law according to Grattan seems fair and > sensible; is Grattan brain-washing us? L16C1 requires calling the director "when a player accidentally receives unauthorized information about a board he is playing or has yet to play". The authors of that law could have omitted the words after "board" had they chosen to. Brian received unauthorized information about a board. Subsequently, but immediately upon realizing that it could be information "about a board he is playing or has yet to play" he called the director. It strikes me as neither fair nor sensible to "apply a penalty to Brian for his breach of law" in not calling the director sooner. That would mean that any overheard remark, even if totally meaningless out of context, must be reported immediately lest one eventually discover that it might have something to do with a board one was, at the time, yet to play. Eric Landau 1107 Dale Drive Silver Spring MD 20910 ehaa at starpower.net From adam at irvine.com Mon Nov 3 18:25:00 2008 From: adam at irvine.com (Adam Beneschan) Date: Mon, 03 Nov 2008 09:25:00 -0800 Subject: [blml] New Laws don't allow UI situation to be resolved? In-Reply-To: Your message of "Sun, 02 Nov 2008 18:50:04 GMT." <000901c93d1b$d1d72900$75857b00$@com> Message-ID: <200811031723.JAA26098@mailhub.irvine.com> David Burn wrote: > Haven't been paying much attention to this, but in Brian's case it seems to > me that: > > If you hear a remark that amounts to "you could have made 7NT if you had > played for the queen to be over the jack", you should call the TD at once > (unless you know that it relates to a board you have already played or will > never play). That is what L16C1 seems to me to say. If it turns out that the > remark related to a board played by the remarker's partner last week, at > least no harm will have been done. > > The TD might then determine on which board you could make 7NT by playing for > the queen to be over the jack and prevent you from playing that board > (ruling under L12A2). Or, he might let you play the board to see whether or > not you reached 7NT (or recognised during the auction that this was the > board in question). If you did, he would award both sides average plus; if > not, he might award you some percentage of 12 tricks and some percentage of > 13 tricks in 6NT, or whatever you did reach. > > But if you do not call the Director as soon as you receive the information, > it seems to me that you place your side at some risk. Perhaps, for the sake > of clarity, "has yet to play" might be amended to "may yet have to play" in > the next version of the Laws. Since you haven't paid that much attention, I'll reiterate the situation I faced that caused me a problem (I'm the one who started this thread on BLML). Someone called the director and said audibly (and fairly loudly, since I think he was upset), "He hesitated and then bid six spades." I heard it, but in all likelihood I was concentrating on some other bidding or play problem at the time. It did not occur to me that the remark might be relevant to anything. The importance became clear only during the next round, when I saw the actual hand that must have caused the problem. If your theory about how the Laws are to be intepreted is correct, please consider what sort of requirement you are imposing on players (and in particular what sort of requirement would have been imposed on me in that situation). It means that while they are trying to concentrate on playing the game, they must also keep their ears (and maybe eyes) open to any information they may get that could prove to be relevant later. So when that other guy made the loud remark, I would have needed to stop my train of thought about the hand I was playing, whatever it was; think about the remark I just heard, determine whether it was something that I would need to call the director about, and then called him. Or, if it was a remark that I determined couldn't possibly provide any later UI, I would still have to stop thinking and consider the remark I just heard to see if it was something I'd need to call the director about, and then once I determined it was not, then I could go back and try to figure out what it was about the hand I was playing that I was thinking about. I think that's pretty bad. But at least everyone was using bidding boxes. Consider the possible problems if someone at the next table was unable to use a bid box because of severe arthritis, and I was able to hear some of the bids they were making---do I need to call the Director every time I hear a bid, on the possible chance that I might later pick up a hand and that somehow the information I heard at the other table might suddenly come together in a way that made it clear what was going on on that hand? Consider also that in my original situation, when the person who called the director made the loud remark, the responsibility you seem to be imposing would require about twenty players to all call the director simultaneously. What would happen then? What would the Director (who had already been called to one table when someone complained about the hesitation) need to do? Sorry, I just don't see how this interpretation of the Laws could work in practice. -- Adam From ehaa at starpower.net Mon Nov 3 18:27:50 2008 From: ehaa at starpower.net (Eric Landau) Date: Mon, 3 Nov 2008 12:27:50 -0500 Subject: [blml] Mr Gladstone suggested In-Reply-To: <490DFE1B.1070002@skynet.be> References: <1062526051-1225456021-cardhu_decombobulator_blackberry.rim.net-888530924-@bxe065.bisx.produk.on.blackberry> <001a01c93b5c$0393aed0$0abb0c70$@no> <490B1506.4050408@skynet.be> <000301c93b8c$97769100$c663b300$@no> <490C484F.5040906@skynet.be> <000001c93c38$f9dfd200$ed9f7600$@no> <490CAD47.8050907@skynet.be><000301c93c6c$d4854c90$7d8fe5b0$@no> <490D698A.20903@skynet.be> <002d01c93ce1$22534340$0302a8c0@Mildred> <490DB7E5.3030608@skynet.be> <000301c93cfb$eafff2b0$c0ffd810$@no> <490DD87B.3080004@skynet.be> <000501c93d19$81da8bb0$858fa310$@no> <490DFE1B.1070002@skynet.be> Message-ID: <05F81EEC-2B6E-4D01-847F-D5CA24957815@starpower.net> On Nov 2, 2008, at 2:23 PM, Herman De Wael wrote: > Hello Sven, thanks for the consideration. > > Sven Pran wrote: > >> The primary condition for having a call accepted (by the director) >> to be a >> psyche is that it must be a deliberate and gross deviation from >> partnership >> understandings; > > Agreed, with one condition: the Director need not (as a primary duty) > determine if something is a psyche or not. But I digress. > >> this implies that the meaning of the call can in no way be >> part of the partnership understandings, a fact that is further >> emphasized in >> Law 40; > > No, it cannot be part of the partnership agreements. Said very often. > >> Partner must not have more reason than have opponents to suspect >> this deviation from partnership understandings. > > Indeed - but it does say "no more" not "no". That means that the > laws do > acknowledge that a player may be aware that his partner sometimes > psyches. As long as the opponents also know that the player sometimes > psyches, the condition of L40C1 is met. Bridge players sometimes psych. What's more, that bridge players sometimes psych is a "matter[] generally known to bridge players" [L40B6(a)]. Eric Landau 1107 Dale Drive Silver Spring MD 20910 ehaa at starpower.net From jfusselman at gmail.com Mon Nov 3 19:03:35 2008 From: jfusselman at gmail.com (Jerry Fusselman) Date: Mon, 3 Nov 2008 12:03:35 -0600 Subject: [blml] New Laws don't allow UI situation to be resolved? In-Reply-To: <200811031723.JAA26098@mailhub.irvine.com> References: <000901c93d1b$d1d72900$75857b00$@com> <200811031723.JAA26098@mailhub.irvine.com> Message-ID: <2b1e598b0811031003lde57aa4o89a9041ac6dc20c5@mail.gmail.com> Adam Beneschan: > > Consider also that in my original situation, when the person who > called the director made the loud remark, the responsibility you seem > to be imposing would require about twenty players to all call the > director simultaneously. What would happen then? What would the > Director (who had already been called to one table when someone > complained about the hesitation) need to do? > > Sorry, I just don't see how this interpretation of the Laws could work > in practice. > You forgot to end with QED. (I.e., I consider Adam's point proven.) -Jerry Fusselman From wjburrows at gmail.com Mon Nov 3 19:17:28 2008 From: wjburrows at gmail.com (Wayne Burrows) Date: Tue, 4 Nov 2008 07:17:28 +1300 Subject: [blml] Mr Gladstone suggested In-Reply-To: <88BF54A6-5114-40E9-9677-A7E861FFDB59@starpower.net> References: <1062526051-1225456021-cardhu_decombobulator_blackberry.rim.net-888530924-@bxe065.bisx.produk.on.blackberry> <2a1c3a560810311251o1b9ea039j39b200467f77c725@mail.gmail.com> <000001c93ba4$f34d3b10$d9e7b130$@com> <2a1c3a560810311545l386979b3vec0c724581100f66@mail.gmail.com> <88BF54A6-5114-40E9-9677-A7E861FFDB59@starpower.net> Message-ID: <2a1c3a560811031017r62868df6i6a69773ea092540d@mail.gmail.com> 2008/11/4 Eric Landau : > On Oct 31, 2008, at 6:45 PM, Wayne Burrows wrote: > >> 2008/11/1 David Burn : >> >>> If your convention card says "1S shows 7+ hcp" then you are >>> playing a HUM >>> (whatever else it says). If your convention card says "1S shows 8+ >>> hcp" but >>> in fact you open all (or "almost all") 7-point hands 1S then you >>> are playing >>> a HUM (and you are trying to conceal the fact). >>> >>> If you do not open all (or "almost all") 7-point hands with a bid >>> at the one >>> level, then: >>> >>> If you open ten 7-counts for every one you pass, then you are >>> playing a HUM >>> (and you may not escape the charge of playing a concealed HUM by >>> exhibiting >>> the one seven-count you passed). If you pass ten seven-counts for >>> every one >>> you open, then you are not playing a HUM (and you may escape the >>> charge of >>> playing a concealed HUM by exhibiting the ten seven-counts you >>> passed). >>> >>> This ratio of ten to one is based on nothing but my own intuition >>> - other >>> people may have a different figure in mind. But some figure there >>> must be, >>> or (as Wayne correctly says) the WBF policy on psychic bidding >>> will be >>> ineffective (since it will be meaningless). >> >> I have two problems with this: >> >> 1. There is a huge gulf between 10:1 and 1:10 so this does not tell >> us where the boundary is. > > I see no possibility of a "bright line" here, so any boundary would > necessarily be subjective. But the appropriate subjective boundary > would be pretty obvious: allow opening those 7-counts for which the > player can offer a sensible justification for treating them as the > playing-strength equivalent of at least an average 8-count. (My > unsupported intuition is that something around 10-20% of 7-count > would qualify, but the point is that the criterion not depend on the > actual frequency.) > To me this boundary is far from obvious. You and I may have different boundaries. Indeed completely different subsets. Your justification may be sensible to you and mine sensible to me. We should both be allowed to exercise our judgement - after all bridge is a game of judgement and if you don't let me exercise my bad judgement you will not be able to beat me or at least you will not be able to be me by as much as you deserve to. To me the fundamental problem here is that the regulation uses a form of judgement - milton work points are a way of judging how good a hand is - and the boundary set - " a king below average" - is close to the normal set of hands opened. Although having said that there is likely to be a problem whereever the boundary is set. Once we set boundaries it is only natural for (some) players to explore those boundaries. The boundaries between opening hand and not opening hand (or some weak opening) are not rigid but fuzzy. Bridge literature (especially online) and discussions are often about these fuzzy boundaries - "What should I bid with this hand?" "Should I open or should I pass?" etc etc. Milton Work Points have the advantage of being clear. (And I imagine almost all bridge players count their Milton Work Points as an initial assessment of strength.) However "points schmoints" these same Milton Work Points don't serve well when players play systems near the boundary as there own systemic fuzzy boundaries are likely to lie either side of the boundary - Q32 Q32 Q32 J432 will have very different evaluations to A1098765 K109876 - - and yet both evaluate identically on the "king or more below average strength" criterion. Or more absurdly replace the jack for the queen in the first example and the first hand is an allowable opening and the second is not. So there is both a need for flexibility in boundaries. But there is also a need for rigidity. Otherwise I see Helgemo opening some 7 hcp hand and think why is he allowed to do that when I am not. Maybe you tell me he is exercising his judgement and that particular 7 hcp is work 9.3 points on some fancy scale. So I start exercising my judgement and the local director stamps on me. To me any regulation (or law) that attempts to regulate judgement is fundamentally flawed and will always cause these sorts of problems. Judgement is not universal so we will always have serious anomolies when the regulations are written in terms of some sort of judgement and disregard other methods of judgement. This means it is impossible to have a boundary of this type and for the rule to be sensible and fair. I suppose if the rule e.g. "a king or more below average" was treated strictly then it would be fair but it would be far from sensible. The regulators and/or law makers seem to have some notion that light opening bids are "bad". This may or may not be a correct view. But their efforts to provide a boundary between allowable and disallowed bids is nonsensical in theory (when judgement is fuzzy) and in practice (when Helgemo can open his choice of 7-counts but I cannot open my choice of 7-counts). -- Wayne Burrows Palmerston North New Zealand From ehaa at starpower.net Mon Nov 3 20:55:54 2008 From: ehaa at starpower.net (Eric Landau) Date: Mon, 3 Nov 2008 14:55:54 -0500 Subject: [blml] Mr Gladstone suggested In-Reply-To: <2a1c3a560811031017r62868df6i6a69773ea092540d@mail.gmail.com> References: <1062526051-1225456021-cardhu_decombobulator_blackberry.rim.net-888530924-@bxe065.bisx.produk.on.blackberry> <2a1c3a560810311251o1b9ea039j39b200467f77c725@mail.gmail.com> <000001c93ba4$f34d3b10$d9e7b130$@com> <2a1c3a560810311545l386979b3vec0c724581100f66@mail.gmail.com> <88BF54A6-5114-40E9-9677-A7E861FFDB59@starpower.net> <2a1c3a560811031017r62868df6i6a69773ea092540d@mail.gmail.com> Message-ID: On Nov 3, 2008, at 1:17 PM, Wayne Burrows wrote: > 2008/11/4 Eric Landau : > >> I see no possibility of a "bright line" here, so any boundary would >> necessarily be subjective. But the appropriate subjective boundary >> would be pretty obvious: allow opening those 7-counts for which the >> player can offer a sensible justification for treating them as the >> playing-strength equivalent of at least an average 8-count. (My >> unsupported intuition is that something around 10-20% of 7-count >> would qualify, but the point is that the criterion not depend on the >> actual frequency.) > > To me this boundary is far from obvious. You and I may have different > boundaries. Indeed completely different subsets. Your justification > may be sensible to you and mine sensible to me. We should both be > allowed to exercise our judgement - after all bridge is a game of > judgement and if you don't let me exercise my bad judgement you will > not be able to beat me or at least you will not be able to be me by as > much as you deserve to. > > To me the fundamental problem here is that the regulation uses a form > of judgement - milton work points are a way of judging how good a hand > is - and the boundary set - " a king below average" - is close to the > normal set of hands opened. Although having said that there is likely > to be a problem whereever the boundary is set. Once we set boundaries > it is only natural for (some) players to explore those boundaries. > > The boundaries between opening hand and not opening hand (or some weak > opening) are not rigid but fuzzy. Bridge literature (especially > online) and discussions are often about these fuzzy boundaries - "What > should I bid with this hand?" "Should I open or should I pass?" etc > etc. Note that the "necessarily subjective" judgment I am calling for is *not* about whether a particular hand is worth an opening bid, but rather about whether the player making it has a "sensible justification" for deeming it so. IOW, we investigate to satisfy ourselves that the player is only opening occasional hands that *he* considers worth a significant degree of promotion (regardless of what we might think), and not, say, opening random bad hands willy-nilly and finding alleged "justification" only when challenged. > Milton Work Points have the advantage of being clear. (And I imagine > almost all bridge players count their Milton Work Points as an initial > assessment of strength.) However "points schmoints" these same Milton > Work Points don't serve well when players play systems near the > boundary as there own systemic fuzzy boundaries are likely to lie > either side of the boundary - Q32 Q32 Q32 J432 will have very > different evaluations to A1098765 K109876 - - and yet both evaluate > identically on the "king or more below average strength" criterion. > Or more absurdly replace the jack for the queen in the first example > and the first hand is an allowable opening and the second is not. I note that Q32/Q32/Q32/Q432 -- a legal opening bid -- has eight losers (by LTC), our typical minimum opening bid has six losers, and A1098765/K109876 has four losers. That's what I would call "a 'sensible justification'". > So there is both a need for flexibility in boundaries. But there is > also a need for rigidity. Otherwise I see Helgemo opening some 7 hcp > hand and think why is he allowed to do that when I am not. Maybe you > tell me he is exercising his judgement and that particular 7 hcp is > work 9.3 points on some fancy scale. So I start exercising my > judgement and the local director stamps on me. Well, in theory "9.3 points on some fancy scale" and "it just felt right" are both potentially acceptable justifications, but it wouldn't surprise me if a director investigating cold was a lot more sympathetic to the "sensibleness" of the first than the second. > To me any regulation (or law) that attempts to regulate judgement is > fundamentally flawed and will always cause these sorts of problems. > Judgement is not universal so we will always have serious anomolies > when the regulations are written in terms of some sort of judgement > and disregard other methods of judgement. This means it is impossible > to have a boundary of this type and for the rule to be sensible and > fair. I suppose if the rule e.g. "a king or more below average" was > treated strictly then it would be fair but it would be far from > sensible. > > The regulators and/or law makers seem to have some notion that light > opening bids are "bad". This may or may not be a correct view. But > their efforts to provide a boundary between allowable and disallowed > bids is nonsensical in theory (when judgement is fuzzy) and in > practice (when Helgemo can open his choice of 7-counts but I cannot > open my choice of 7-counts). "...a regulation must not restrict [a player's] style and judgment, only method" [L40B2(a)]. A TD cannot impose his own style or his own judgment. But he can, and sometimes must, make a subjective determination as to whether a particular deviation was in fact predicated on the player's style or judgment, or was not, and so may have infracted some regulation. Eric Landau 1107 Dale Drive Silver Spring MD 20910 ehaa at starpower.net From grandaeval at tiscali.co.uk Mon Nov 3 17:56:39 2008 From: grandaeval at tiscali.co.uk (Grattan) Date: Mon, 3 Nov 2008 16:56:39 -0000 Subject: [blml] Control of methods References: <003f01c93daf$4df0af00$0302a8c0@Mildred> <490EF4BA.6080705@skynet.be> Message-ID: <000101c93df6$045af9d0$0302a8c0@Mildred> Grattan Endicott To: "Bridge Laws Mailing List" Sent: Monday, November 03, 2008 12:55 PM Subject: Re: [blml] Control of methods > Grattan wrote: >> +=+ The foregoing remarks seem to imply that we will find in the >> laws a reference to 'system regulations'. I do not recall that there >> is one. In the 2007 Laws it is on partnership understandings that >> controls are authorized, and in this regard the Regulating Authority >> is given absolute discretion as to what it will allow and what it will >> not allow. >> ~ Grattan ~ +=+ >> > > Fully correct Grattan, but: > > If a RA decides that for the purpose of their system regulations all > partnership understandings are part of the regulated system, then they > will find that they have written a set of regulations that ban (some) > psyches. Such a set of regulations would be contrary to the laws. > > In order for a set of system regulations to be workable, there HAS to be > a difference between the set of partnership understandings and the set > of partnership agreements, with only the last (sub)set being subject to > the regulations in force. > > I believe that the WBF system regulations make such a distinction. > +=+ The laws say that any partnership agreement constitutes a partnership understanding. Any partnership understanding may be declared a special understanding and regulated. If a 'psyche', as defined in the law book, is the subject of a partnership understanding it may be regulated - and that includes the power to disallow it. A psyche which has no element of partnership understanding attached is not subject to regulation (unless Law 40B2(d) applies). In regulating partnership understandings it is not incumbent on other RAs to adopt any distinctions the WBF regulations may make in respect of deviant actions. ~ G ~ +=+ From grandaeval at tiscali.co.uk Mon Nov 3 21:42:51 2008 From: grandaeval at tiscali.co.uk (Grattan) Date: Mon, 3 Nov 2008 20:42:51 -0000 Subject: [blml] New Laws don't allow UI situation to be resolved? References: <2b1e598b0810302300t4f888261j8d7615f57fcdd4ee@mail.gmail.com><000301c93b33$398b9670$aca2c350$@no><001b01c93b5d$4438f2f0$ccaad8d0$@no><000001c93c08$ea3dbf90$beb93eb0$@no><000101c93c49$99440220$cbcc0660$@no> <001a01c93c5b$6d020830$0302a8c0@Mildred> <490CCA60.5060607@meadows.pair.com><000801c93cb6$c082c5c0$0302a8c0@Mildred> <490D6BA7.30607@skynet.be><002c01c93ce1$221094f0$0302a8c0@Mildred><490D98CC.6040908@talktalk.net> <5AAF2C28-908F-43D3-95E2-5767DA109E8D@starpower.net> Message-ID: <000201c93df6$04ae7100$0302a8c0@Mildred> Grattan Endicott To: "Bridge Laws Mailing List" Sent: Monday, November 03, 2008 3:56 PM Subject: Re: [blml] New Laws don't allow UI situation to be resolved? > On Nov 2, 2008, at 7:10 AM, Nigel Guthrie wrote: > >> [Herman de Wael] >> That is just plain silly, Grattan. The laws have been altered so as to >> allow for more boards to be played. If you call the TD on the first >> occasion, he will no doubt apply (c) allow play to continue. Then you >> call him again a second time - now what? >> >> [Grattan Endicott] >> +=+ "Just plain silly", if you say so, but nevertheless it is the >> law. Brian should have informed the Director when he heard >> the remark, not when he came to play the board (Law 16C1). The >> Director would then have control of the matter before >> Brian had the board in front of him. Having by then discovered the >> nature of the hand and the significance of the UI only a much >> inferior Director would have allowed play of the board. In the >> situation Brian reached, through oversight of his Law >> 16C1 duty, one could expect the Director to award an adjusted score >> because he judged (Law 12A2) that the UI was such as >> not to permit normal play of the board, but in addition to apply a >> penalty to Brian for his breach of law. >> >> [Nige1] >> Again, in this case, the law according to Grattan seems fair and >> sensible; is Grattan brain-washing us? > > L16C1 requires calling the director "when a player accidentally > receives unauthorized information about a board he is playing or has > yet to play". The authors of that law could have omitted the words > after "board" had they chosen to. Brian received unauthorized > information about a board. Subsequently, but immediately upon > realizing that it could be information "about a board he is playing > or has yet to play" he called the director. It strikes me as neither > fair nor sensible to "apply a penalty to Brian for his breach of law" > in not calling the director sooner. That would mean that any > overheard remark, even if totally meaningless out of context, must be > reported immediately lest one eventually discover that it might have > something to do with a board one was, at the time, yet to play. > +=+ Eric, we have differing appreciations of the level of bridge nous that Brian displayed and, given the nature of what he heard, of the potential significance of placing the honour. ~ Grattan ~ +=+ From grandaeval at tiscali.co.uk Mon Nov 3 21:47:47 2008 From: grandaeval at tiscali.co.uk (Grattan) Date: Mon, 3 Nov 2008 20:47:47 -0000 Subject: [blml] Quacky fits References: <490EF965.6070004@skynet.be> Message-ID: <000301c93df6$04f39050$0302a8c0@Mildred> Grattan Endicott To: "Bridge Laws Mailing List" Sent: Monday, November 03, 2008 1:15 PM Subject: [blml] If it quacks > like a duck, it is most probably a duck. > > We've compared psyches to elephants and donkeys already, maybe we should > try ducks. > > Consider an opening of 1H, third in hand, on less than normally expected > values. > > Now Sven and Richard call this thing a teal. > I prefer to call it a duck. After all, it quacks. > > For non-native english speakers, a teal is a type of duck, considered by > some to be a separate animal but in reality one of several species > within the genus Anas (ducks). > > For some days, I have been trying to convince Sven and Richard that this > thing is actually a duck. I have not succeeded, and Grattan recently > called the topic unreal. I believe he is right. > > There is no sense in discussing the names - all we need to do is see > what we shall do with a teal. > > So we call the Director. > > We point out that the hand that opened 1H does not conform to the normal > expectations of such an opening. Let's assume that a deviation is easily > seen. > > So the Director starts an investigation. > > As a first step, she will find out, presumably, that the opener has done > this before. She will attempt to form an idea of the frequency with > which this teal has happened before. > > As part of that first step, she may try to establish if partner knew > what this frequency was. I don't believe, however, that this information > is relevant. It is my firm opinion that the deviation frequency is part > of the information to which the opponents are entitled, even when the > partner does not actually know that frequency. The fact that he could > know it should suffice. > > Then in a second step, the Director will investigate how much the > opponents knew about the frequency. She will add: > - any mention on the SC; > - any mention at the table by the partner; > - general bridge knowledge > and form an idea about the level to which the opponents could have > suspected that a deviation might take place. > Then she compares these two levels. If she finds that the partner had no > more reason to suspect that a deviation had occured than opponents, she > rules that all is well and good. > > So let's suppose she finds that opponents did not have all the > information they should have had. > > Now for the third step. I believe that step is to check for damage. The > Director will ask the opponents if they would have acted differently if > they had know about the heightened expectancy of a deviation. Of course > the question is not "if you had known that it was this weak ..." but > rather "if you had know that it could have been this weak more often > than ...". Quite a lot of deviations will be judged to be without damage. > > If however, the damage is real, the Director will start the fourth step > and find an adjusted score based on what would have happened with > complete information. > > Now, in my opinion, this animal quacks like a piece of MI. I therefor > suggested to call it a piece of MI. Sven and Richard may continue to > call it a ICMPU, or a teal. > > But do they (and you) agree that the MI rules are the ones to use? > +=+ Broadly Herman is stating a truth, although the Director's mode of enquiry is a matter for her and the RA.. However misinformation is not the only potential crime. The *use* of the call may lack authority under Law 40A3. The full extent of the law is available to the Director, who may have guidance on policy from the RA. Possibilities to examine are those in Laws 40B4, 40B5, and 40B6(b) and, if making the call was unlawful, also 12A2. ~ Grattan ~ +=+ From brian at meadows.pair.com Mon Nov 3 23:09:18 2008 From: brian at meadows.pair.com (brian) Date: Mon, 03 Nov 2008 17:09:18 -0500 Subject: [blml] New Laws don't allow UI situation to be resolved? In-Reply-To: <000201c93df6$04ae7100$0302a8c0@Mildred> References: <2b1e598b0810302300t4f888261j8d7615f57fcdd4ee@mail.gmail.com><000301c93b33$398b9670$aca2c350$@no><001b01c93b5d$4438f2f0$ccaad8d0$@no><000001c93c08$ea3dbf90$beb93eb0$@no><000101c93c49$99440220$cbcc0660$@no> <001a01c93c5b$6d020830$0302a8c0@Mildred> <490CCA60.5060607@meadows.pair.com><000801c93cb6$c082c5c0$0302a8c0@Mildred> <490D6BA7.30607@skynet.be><002c01c93ce1$221094f0$0302a8c0@Mildred><490D98CC.6040908@talktalk.net> <5AAF2C28-908F-43D3-95E2-5767DA109E8D@starpower.net> <000201c93df6$04ae7100$0302a8c0@Mildred> Message-ID: <490F768E.1010106@meadows.pair.com> Grattan wrote: > >> > +=+ Eric, we have differing appreciations of the level of bridge nous > that Brian displayed and, given the nature of what he heard, of the > potential significance of placing the honour. My example hand was very obviously (I hope) contrived so as to make the relevance of the UI as clearly as possible related to a particular board. Posting it was prompted by your reference to Law 16A3 as regards whether a player could take into account the UI when presented with a 50-50 choice. So OK, the answer was to call the director first - but I would still like to hear two things from you. 1) The answer as to whether, *if* the TD should instruct me to play on, I can use that information to decide which honour to play first or not. 2) The answer to the point made by another poster (Eric I think, sorry if I have it wrong) about the *practicality* of requiring a player to call as soon as he overhears a remark, how do you expect a playing club TD to deal with a dozen or so simultaneous director calls because of the same over-loud discussion from a table? And this time, no comments about the level of "nous" which I displayed in a situation which (obviously, IMO) was contrived to hopefully illustrate a point rather than something which actually happened? Brian. From dalburn at btopenworld.com Mon Nov 3 23:40:16 2008 From: dalburn at btopenworld.com (David Burn) Date: Mon, 3 Nov 2008 22:40:16 -0000 Subject: [blml] New Laws don't allow UI situation to be resolved? In-Reply-To: <490F768E.1010106@meadows.pair.com> References: <2b1e598b0810302300t4f888261j8d7615f57fcdd4ee@mail.gmail.com><000301c93b33$398b9670$aca2c350$@no><001b01c93b5d$4438f2f0$ccaad8d0$@no><000001c93c08$ea3dbf90$beb93eb0$@no><000101c93c49$99440220$cbcc0660$@no> <001a01c93c5b$6d020830$0302a8c0@Mildred> <490CCA60.5060607@meadows.pair.com><000801c93cb6$c082c5c0$0302a8c0@Mildred> <490D6BA7.30607@skynet.be><002c01c93ce1$221094f0$0302a8c0@Mildred><490D98CC.6040908@talktalk.net> <5AAF2C28-908F-43D3-95E2-5767DA109E8D@starpower.net> <000201c93df6$04ae7100$0302a8c0@Mildred> <490F768E.1010106@meadows.pair.com> Message-ID: <000001c93e05$2add3af0$8097b0d0$@com> [Brian] My example hand was very obviously (I hope) contrived so as to make the relevance of the UI as clearly as possible related to a particular board. [DALB] Indeed, which is why I am a bit puzzled by people asking "Am I supposed to call the Director any time I hear anything at all that might vaguely be related to a board I might or might not be about to play?" The answer to that is "No", but the answer to this: [Brian] 2) The answer to the point made by another poster (Eric I think, sorry if I have it wrong) about the *practicality* of requiring a player to call as soon as he overhears a remark, how do you expect a playing club TD to deal with a dozen or so simultaneous director calls because of the same over-loud discussion from a table? [DALB] is that I expect the Director to rule the board unplayable for the remainder of the session, to award a PP against the over-loud discussers in proportion to the number of times the board can no longer be played, and later to award averages at each table where the board has not yet been played. If the Director has the time and the inclination, he may decide to introduce a substitute board for the remainder of the session and score accordingly (to avoid the paying customers having to sit out for half a round). Of course, the impracticality of calling the Director for any and all pieces of overheard data, whether or not they constitute information, does occasionally place a considerable onus on players themselves to act ethically. Some of them won't. Too bad. David Burn London, England From john at asimere.com Tue Nov 4 01:24:09 2008 From: john at asimere.com (John (MadDog) Probst) Date: Tue, 4 Nov 2008 00:24:09 -0000 Subject: [blml] Mr Gladstone suggested [SEC=UNOFFICIAL] References: <1062526051-1225456021-cardhu_decombobulator_blackberry.rim.net-888530924-@bxe065.bisx.produk.on.blackberry><001a01c93b5c$0393aed0$0abb0c70$@no> <490B1506.4050408@skynet.be><000301c93b8c$97769100$c663b300$@no> <490C484F.5040906@skynet.be><000001c93c38$f9dfd200$ed9f7600$@no> <490CAD47.8050907@skynet.be><000301c93c6c$d4854c90$7d8fe5b0$@no> <490D698A.20903@skynet.be><002d01c93ce1$22534340$0302a8c0@Mildred><490DB7E5.3030608@skynet.be> <000301c93cfb$eafff2b0$c0ffd810$@no><002701c93d06$bbd534e0$0302a8c0@Mildred> Message-ID: <37DA741301334558B895AA493AF4BDAC@JOHN> ----- Original Message ----- From: "Robert Frick" To: "Bridge Laws Mailing List" Sent: Monday, November 03, 2008 2:20 AM Subject: Re: [blml] Mr Gladstone suggested [SEC=UNOFFICIAL] > Perhaps saying the obvious, but a line is crossed when you think your > psyche (or any intentional misdescription) is safer with your regular > partner than with an unknown partner. I know that is probably > unenforceable. That's a nice definition. I'd add unkown partner of similar standard to your regular partner. We have to know he's gotten dry behind the ears. > > > > _______________________________________________ > blml mailing list > blml at amsterdamned.org > http://www.amsterdamned.org/mailman/listinfo/blml From john at asimere.com Tue Nov 4 01:36:10 2008 From: john at asimere.com (John (MadDog) Probst) Date: Tue, 4 Nov 2008 00:36:10 -0000 Subject: [blml] Mr Burn suggested References: <48FC56AB.1050505@skynet.be> <02A30BE1-5E95-41F3-AE74-1936A0913D29@starpower.net> <48FCAB1E.8010201@skynet.be> <2a1c3a560810201658h1877df31n5886171a8165db7c@mail.gmail.com> <2a1c3a560810201952j563d60f9ua2096c951f2885b8@mail.gmail.com><48FDDBFF.7070300@connecttime.net> <002401c93dab$7960f8b0$0302a8c0@Mildred> Message-ID: ----- Original Message ----- From: "Grattan" To: "Bridge Laws Mailing List" Sent: Monday, November 03, 2008 11:56 AM Subject: Re: [blml] Mr Burn suggested > > > Grattan Endicott also ************************************** > "Let us then be up and doing, > With a heart for any fate, > Still achieving, still pursuing, > Learn to labour and to wait." > [H.W. Longfellow] > > ''''''''''''''''''''''''''''''''''''''''''''''''''''''''''''''''''''''''''''''''''''''''''''''''' > > Someone wrote > < what my method does is to keep my 1NT call to a point where 1) > it has lowish frequency compared with the natural call; 2) has lowish > frequency as a call showing H and a bad hand.since most of these > hands get passed 3) every now and then I bid 1NT with spades or > diamonds or clubs, which my opponents moan about because I've > psyched my psych. Partner does alert my 1NT and does explain > "15-17 and sometimes //about 10% of these// when he holds a long > H suit and a weak hand he does it and sometimes he does it on > other hands too". I am confident I am home free using this method.> > > Partner's ability to describe the method to opponent demonstrates > the existence of a partnership understanding. This may be regulated > by the RA which may allow it, disallow it, or allow it conditionally. I wrote it. The last three times partner explained it as such it was a psych with diamonds (the opponents moaned about MI) and twice natural. Partner has NO idea except observation as to what my 1NT psych is. He does know, from frequency analysis, that I do bid 1NT with H some of the time, and statistically (given the number of hands we've played together) about 10-15% of the hands that might fall into the category ARE so overcalled. This is disclosure; not partnership understanding. It could just as well be a different psyche or natural. and partner HAS noticed it seems to have H more often, so he discloses it. Of all the 1NT psyches no more than about 35% are H based. Grattan, There is NO formally discernable pattern; I'm still home free. Partner CAN'T PREDICT my psych. ; > > ~ Grattan ~ +=+ > > > > _______________________________________________ > blml mailing list > blml at amsterdamned.org > http://www.amsterdamned.org/mailman/listinfo/blml From grandaeval at tiscali.co.uk Tue Nov 4 02:20:02 2008 From: grandaeval at tiscali.co.uk (Grattan) Date: Tue, 4 Nov 2008 01:20:02 -0000 Subject: [blml] New Laws don't allow UI situation to be resolved? References: <2b1e598b0810302300t4f888261j8d7615f57fcdd4ee@mail.gmail.com><000301c93b33$398b9670$aca2c350$@no><001b01c93b5d$4438f2f0$ccaad8d0$@no><000001c93c08$ea3dbf90$beb93eb0$@no><000101c93c49$99440220$cbcc0660$@no> <001a01c93c5b$6d020830$0302a8c0@Mildred> <490CCA60.5060607@meadows.pair.com><000801c93cb6$c082c5c0$0302a8c0@Mildred> <490D6BA7.30607@skynet.be><002c01c93ce1$221094f0$0302a8c0@Mildred><490D98CC.6040908@talktalk.net> <5AAF2C28-908F-43D3-95E2-5767DA109E8D@starpower.net><000201c93df6$04ae7100$0302a8c0@Mildred> <490F768E.1010106@meadows.pair.com> Message-ID: <002401c93e1b$7a20d160$0302a8c0@Mildred> Grattan Endicott To: "Bridge Laws Mailing List" Sent: Monday, November 03, 2008 10:09 PM Subject: Re: [blml] New Laws don't allow UI situation to be resolved? And this time, no comments about the level of "nous" which I displayed in a situation which (obviously, IMO) was contrived to hopefully illustrate a point rather than something which actually happened? +=+ I understood the nature of your presentation, but I dealt with it as it was presented. +=+ << So OK, the answer was to call the director first - but I would still like to hear two things from you. > 1) The answer as to whether, *if* the TD should instruct me to play on, I can use that information to decide which honour to play first or not. > +=+ If the Director realizes that the UI you have is significant and gives you potential advantage he is stupid if he allows the play to continue. Whether you take that advantage or not he has the duty of protecting two non-offending sides and he should, indeed must, return the score to equity (which will almost certainly mean an artificial adjusted score with each side enjoying the benefit of its innocence). If the truth slams home to him only after letting play continue his duty is to adjust the score on that same basis. All roads should lead to Rome. +=+ << 2) The answer to the point made by another poster (Eric I think, sorry if I have it wrong) about the *practicality* of requiring a player to call as soon as he overhears a remark, how do you expect a playing club TD to deal with a dozen or so simultaneous director calls because of the same over-loud discussion from a table? > +=+ Two answers: 1. a real bridge player should usually identify if he has heard something that could well be significant. He should assume it applies to a board currently in play. In these circumstances he should protect his position by telling the Director. (In many cases the emotion of the speaker will wake him up.) and: 2. half a dozen plus complaints are a likely indication that the board can no longer stay in play. If a substitute board can be introduced it may be possible to score each of them among those who played it, scoring them like a fouled board (Law 87B). +=+ From grandaeval at tiscali.co.uk Tue Nov 4 02:24:46 2008 From: grandaeval at tiscali.co.uk (Grattan) Date: Tue, 4 Nov 2008 01:24:46 -0000 Subject: [blml] Mr Burn suggested References: <48FC56AB.1050505@skynet.be> <02A30BE1-5E95-41F3-AE74-1936A0913D29@starpower.net> <48FCAB1E.8010201@skynet.be> <2a1c3a560810201658h1877df31n5886171a8165db7c@mail.gmail.com> <2a1c3a560810201952j563d60f9ua2096c951f2885b8@mail.gmail.com><48FDDBFF.7070300@connecttime.net><002401c93dab$7960f8b0$0302a8c0@Mildred> Message-ID: <002d01c93e1c$28b1ff10$0302a8c0@Mildred> Grattan Endicott To: "Bridge Laws Mailing List" Sent: Tuesday, November 04, 2008 12:36 AM Subject: Re: [blml] Mr Burn suggested > > Grattan, There is NO formally discernable pattern; > I'm still home free. Partner CAN'T PREDICT my psych. > +=+ Not good enough. Partner is still aware of the increased potential for a psyche to occur. He has more reason to be aware of the deviation than have opponents. ~ G ~ +=+ From wjburrows at gmail.com Tue Nov 4 03:11:11 2008 From: wjburrows at gmail.com (Wayne Burrows) Date: Tue, 4 Nov 2008 15:11:11 +1300 Subject: [blml] Mr Burn suggested In-Reply-To: <002d01c93e1c$28b1ff10$0302a8c0@Mildred> References: <02A30BE1-5E95-41F3-AE74-1936A0913D29@starpower.net> <48FCAB1E.8010201@skynet.be> <2a1c3a560810201658h1877df31n5886171a8165db7c@mail.gmail.com> <2a1c3a560810201952j563d60f9ua2096c951f2885b8@mail.gmail.com> <48FDDBFF.7070300@connecttime.net> <002401c93dab$7960f8b0$0302a8c0@Mildred> <002d01c93e1c$28b1ff10$0302a8c0@Mildred> Message-ID: <2a1c3a560811031811p4e0597e3s5c30da7a72dba451@mail.gmail.com> 2008/11/4 Grattan : > > > Grattan Endicott also ************************************** > "Let us then be up and doing, > With a heart for any fate, > Still achieving, still pursuing, > Learn to labour and to wait." > [H.W. Longfellow] > > ''''''''''''''''''''''''''''''''''''''''''''''''''''''''''''''''''''''''''''''''''''''''''''''''' > > > > ----- Original Message ----- > From: "John (MadDog) Probst" > To: "Bridge Laws Mailing List" > Sent: Tuesday, November 04, 2008 12:36 AM > Subject: Re: [blml] Mr Burn suggested > > >> >> Grattan, There is NO formally discernable pattern; >> I'm still home free. Partner CAN'T PREDICT my psych. >> > +=+ Not good enough. Partner is still aware of the increased > potential for a psyche to occur. He has more reason to be > aware of the deviation than have opponents. > ~ G ~ +=+ > I would describe this situation as "more reason to be aware of a deviation" rather than "more reason to be aware of the deviation" which relates to this specific deviation. An increased awareness of a potential deviation does not mean more aware of any particular actual deviation which is what is required by the use of the definite article. -- Wayne Burrows Palmerston North New Zealand From rfrick at rfrick.info Tue Nov 4 04:18:27 2008 From: rfrick at rfrick.info (Robert Frick) Date: Mon, 03 Nov 2008 22:18:27 -0500 Subject: [blml] New Laws don't allow UI situation to be resolved? In-Reply-To: <002401c93e1b$7a20d160$0302a8c0@Mildred> References: <2b1e598b0810302300t4f888261j8d7615f57fcdd4ee@mail.gmail.com> <000301c93b33$398b9670$aca2c350$@no> <001b01c93b5d$4438f2f0$ccaad8d0$@no> <000001c93c08$ea3dbf90$beb93eb0$@no> <000101c93c49$99440220$cbcc0660$@no> <001a01c93c5b$6d020830$0302a8c0@Mildred> <490CCA60.5060607@meadows.pair.com> <000801c93cb6$c082c5c0$0302a8c0@Mildred> <490D6BA7.30607@skynet.be> <002c01c93ce1$221094f0$0302a8c0@Mildred> <490D98CC.6040908@talktalk.net> <5AAF2C28-908F-43D3-95E2-5767DA109E8D@starpower.net> <000201c93df6$04ae7100$0302a8c0@Mildred> <490F768E.1010106@meadows.pair.com> <002401c93e1b$7a20d160$0302a8c0@Mildred> Message-ID: On Mon, 03 Nov 2008 20:20:02 -0500, Grattan wrote: > > > Grattan Endicott also ************************************** > "Let us then be up and doing, > With a heart for any fate, > Still achieving, still pursuing, > Learn to labour and to wait." > [H.W. Longfellow] > > ''''''''''''''''''''''''''''''''''''''''''''''''''''''''''''''''''''''''''''''''''''''''''''''''' > > > > ----- Original Message ----- > From: "brian" > To: "Bridge Laws Mailing List" > Sent: Monday, November 03, 2008 10:09 PM > Subject: Re: [blml] New Laws don't allow UI situation to be resolved? > > > And this time, no comments about the level of "nous" which I displayed > in a situation which (obviously, IMO) was contrived to hopefully > illustrate a point rather than something which actually happened? > +=+ > I understood the nature of your presentation, but I dealt with it as > it was presented. > +=+ > << > So OK, the answer was to call the director first - but I would still > like to hear two things from you. >> > 1) The answer as to whether, *if* the TD should instruct me to play > on, I can use that information to decide which honour to play first or > not. >> > +=+ If the Director realizes that the UI you have is significant and > gives you potential advantage he is stupid if he allows the play to > continue. Whether you take that advantage or not he has the duty > of protecting two non-offending sides and he should, indeed must, > return the score to equity (which will almost certainly mean an > artificial adjusted score with each side enjoying the benefit of its > innocence). If the truth slams home to him only after letting play > continue his duty is to adjust the score on that same basis. All > roads should lead to Rome. +=+ > << > 2) The answer to the point made by another poster (Eric I think, > sorry if I have it wrong) about the *practicality* of requiring a > player to call as soon as he overhears a remark, how do you > expect a playing club TD to deal with a dozen or so simultaneous > director calls because of the same over-loud discussion from a > table? >> > +=+ Two answers: 1. a real bridge player should usually identify if > he has heard something that could well be significant. He should > assume it applies to a board currently in play. In these circumstances > he should protect his position by telling the Director. (In many cases > the emotion of the speaker will wake him up.) > and: 2. half a dozen plus complaints are a likely indication > that > the board can no longer stay in play. If a substitute board can be > introduced it may be possible to score each of them among those > who played it, scoring them like a fouled board (Law 87B). +=+ I pick up my hand and it is in order from left to right, the KQ of spades, the AQ of clubs, two more clubs and a spade, 3 hearts, and then two diamonds, and finally a club. Is this information that "could well be significant"? I can just shuffle my cards and hope I forget, or I can think about it. You seem to be saying that I should think about it. Well, it looks like it could be an opening club lead, declarer runs the red suits, and I win my high cards at the end. I have managed to turn this into some possibly useful information. Should I contact the director now? The problem is, the hand could have been shuffled before being put back in the board and I am trying to find patterns in the clouds. If not, maybe I am reading it wrong. Maybe we will win the hand, or my partner will be on lead, or it will be a different contract. The odds are very likely that this snippet will not be useful. It makes a lot more sense to see if I actually have a decision that this information influences, then I contact the director. Then I can also skip the part about figuring out how the information could be useful to me. From ardelm at optusnet.com.au Tue Nov 4 04:28:34 2008 From: ardelm at optusnet.com.au (Tony Musgrove) Date: Tue, 04 Nov 2008 14:28:34 +1100 Subject: [blml] Mr Burn suggested In-Reply-To: References: <48FC56AB.1050505@skynet.be> <02A30BE1-5E95-41F3-AE74-1936A0913D29@starpower.net> <48FCAB1E.8010201@skynet.be> <2a1c3a560810201658h1877df31n5886171a8165db7c@mail.gmail.com> <2a1c3a560810201952j563d60f9ua2096c951f2885b8@mail.gmail.com> <48FDDBFF.7070300@connecttime.net> <002401c93dab$7960f8b0$0302a8c0@Mildred> Message-ID: <6.1.0.6.2.20081104142750.01db0ec0@mail.optusnet.com.au> >Grattan, There is NO formally discernable pattern; >I'm still home free. Partner CAN'T PREDICT my psych. I bet that like NZ sheep, they're nervous though Cheers Tony (Sydney) >; > > > > ~ Grattan ~ +=+ > > > > > > > > _______________________________________________ > > blml mailing list > > blml at amsterdamned.org > > http://www.amsterdamned.org/mailman/listinfo/blml > > > >_______________________________________________ >blml mailing list >blml at amsterdamned.org >http://www.amsterdamned.org/mailman/listinfo/blml From grandaeval at tiscali.co.uk Tue Nov 4 08:23:47 2008 From: grandaeval at tiscali.co.uk (Grattan) Date: Tue, 4 Nov 2008 07:23:47 -0000 Subject: [blml] Mr Burn suggested References: <02A30BE1-5E95-41F3-AE74-1936A0913D29@starpower.net><48FCAB1E.8010201@skynet.be><2a1c3a560810201658h1877df31n5886171a8165db7c@mail.gmail.com><2a1c3a560810201952j563d60f9ua2096c951f2885b8@mail.gmail.com><48FDDBFF.7070300@connecttime.net><002401c93dab$7960f8b0$0302a8c0@Mildred><002d01c93e1c$28b1ff10$0302a8c0@Mildred> <2a1c3a560811031811p4e0597e3s5c30da7a72dba451@mail.gmail.com> Message-ID: <001301c93e4e$4a819fb0$0302a8c0@Mildred> Grattan Endicott To: "Bridge Laws Mailing List" Sent: Tuesday, November 04, 2008 2:11 AM Subject: Re: [blml] Mr Burn suggested > 2008/11/4 Grattan : >> >> >> Grattan Endicott> also > ************************************** >> "Let us then be up and doing, >> With a heart for any fate, >> Still achieving, still pursuing, >> Learn to labour and to wait." >> [H.W. Longfellow] >> >> ''''''''''''''''''''''''''''''''''''''''''''''''''''''''''''''''''''''''''''''''''''''''''''''''' >> >> >> >> ----- Original Message ----- >> From: "John (MadDog) Probst" >> To: "Bridge Laws Mailing List" >> Sent: Tuesday, November 04, 2008 12:36 AM >> Subject: Re: [blml] Mr Burn suggested >> >> >>> >>> Grattan, There is NO formally discernable pattern; >>> I'm still home free. Partner CAN'T PREDICT my psych. >>> >> +=+ Not good enough. Partner is still aware of the increased >> potential for a psyche to occur. He has more reason to be >> aware of the deviation than have opponents. >> ~ G ~ +=+ >> > > I would describe this situation as "more reason to be aware of a > deviation" rather than "more reason to be aware of the deviation" > which relates to this specific deviation. An increased awareness of a > potential deviation does not mean more aware of any particular actual > deviation which is what is required by the use of the definite > article. > +=+ The deviation occurs when he leaves the path of announced meanings and specification of what path he then may take is a separate issue. Opponents are entitled to have the same access as partner to information that opens up the possibility of a deviation of some kind. They also have the right to equal access with partner to any information that suggests what the nature of the deviation might be. ~ Grattan ~ +=+ From Hermandw at skynet.be Tue Nov 4 09:08:46 2008 From: Hermandw at skynet.be (Herman De Wael) Date: Tue, 04 Nov 2008 09:08:46 +0100 Subject: [blml] Mr Burn suggested In-Reply-To: <002d01c93e1c$28b1ff10$0302a8c0@Mildred> References: <48FC56AB.1050505@skynet.be> <02A30BE1-5E95-41F3-AE74-1936A0913D29@starpower.net> <48FCAB1E.8010201@skynet.be> <2a1c3a560810201658h1877df31n5886171a8165db7c@mail.gmail.com> <2a1c3a560810201952j563d60f9ua2096c951f2885b8@mail.gmail.com><48FDDBFF.7070300@connecttime.net><002401c93dab$7960f8b0$0302a8c0@Mildred> <002d01c93e1c$28b1ff10$0302a8c0@Mildred> Message-ID: <4910030E.2010905@skynet.be> Grattan wrote: > > Grattan Endicott also ************************************** > "Let us then be up and doing, > With a heart for any fate, > Still achieving, still pursuing, > Learn to labour and to wait." > [H.W. Longfellow] > > ''''''''''''''''''''''''''''''''''''''''''''''''''''''''''''''''''''''''''''''''''''''''''''''''' > > > > ----- Original Message ----- > From: "John (MadDog) Probst" > To: "Bridge Laws Mailing List" > Sent: Tuesday, November 04, 2008 12:36 AM > Subject: Re: [blml] Mr Burn suggested > > >> Grattan, There is NO formally discernable pattern; >> I'm still home free. Partner CAN'T PREDICT my psych. >> > +=+ Not good enough. Partner is still aware of the increased > potential for a psyche to occur. He has more reason to be > aware of the deviation than have opponents. > ~ G ~ +=+ > Well, he does tell them - so that's correcting some. But Grattan, please tell us: What is the ruling on John's psyche if you do find his partner has more reason to be aware of it? Herman. From Hermandw at skynet.be Tue Nov 4 09:16:27 2008 From: Hermandw at skynet.be (Herman De Wael) Date: Tue, 04 Nov 2008 09:16:27 +0100 Subject: [blml] Quacky fits In-Reply-To: <000301c93df6$04f39050$0302a8c0@Mildred> References: <490EF965.6070004@skynet.be> <000301c93df6$04f39050$0302a8c0@Mildred> Message-ID: <491004DB.70002@skynet.be> Grattan wrote: > >> >> But do they (and you) agree that the MI rules are the ones to use? >> > +=+ Broadly Herman is stating a truth, although the Director's mode of > enquiry is a matter for her and the RA.. However misinformation is not > the only potential crime. The *use* of the call may lack authority under > Law 40A3. The full extent of the law is available to the Director, who > may have guidance on policy from the RA. Possibilities to examine > are those in Laws 40B4, 40B5, and 40B6(b) and, if making the call > was unlawful, also 12A2. > ~ Grattan ~ +=+ > Grattan, please be very precise here. The *use* of the law may lack authority under L40A3. What is the ruling the TD ought to give if she decides this is the case? Herman. From grandaeval at tiscali.co.uk Tue Nov 4 09:24:21 2008 From: grandaeval at tiscali.co.uk (Grattan) Date: Tue, 4 Nov 2008 08:24:21 -0000 Subject: [blml] Mr Burn suggested References: <48FC56AB.1050505@skynet.be><02A30BE1-5E95-41F3-AE74-1936A0913D29@starpower.net><48FCAB1E.8010201@skynet.be><2a1c3a560810201658h1877df31n5886171a8165db7c@mail.gmail.com><2a1c3a560810201952j563d60f9ua2096c951f2885b8@mail.gmail.com><48FDDBFF.7070300@connecttime.net><002401c93dab$7960f8b0$0302a8c0@Mildred> <6.1.0.6.2.20081104142750.01db0ec0@mail.optusnet.com.au> Message-ID: <001001c93e56$c21363d0$0302a8c0@Mildred> Grattan Endicott To: "Bridge Laws Mailing List" Sent: Tuesday, November 04, 2008 3:28 AM Subject: Re: [blml] Mr Burn suggested > >>Grattan, There is NO formally discernable pattern; >>I'm still home free. Partner CAN'T PREDICT my psych. > > I bet that like NZ sheep, they're nervous though > > Cheers > > Tony (Sydney) > +=+ Sorry Tony, but what you say falls short of the requirement. Partner's awareness that you may well psyche in a given situation is something opponents are entitled to share even if the nature of the psyche is unpredictable (which unpredictability is apt to be a questionable assertion anyway, in the light of partnership experience, but is a separate issue). ~ Grattan ~ +=+ From grandaeval at tiscali.co.uk Tue Nov 4 09:42:51 2008 From: grandaeval at tiscali.co.uk (Grattan) Date: Tue, 4 Nov 2008 08:42:51 -0000 Subject: [blml] New Laws don't allow UI situation to be resolved? References: <2b1e598b0810302300t4f888261j8d7615f57fcdd4ee@mail.gmail.com><000301c93b33$398b9670$aca2c350$@no><001b01c93b5d$4438f2f0$ccaad8d0$@no><000001c93c08$ea3dbf90$beb93eb0$@no><000101c93c49$99440220$cbcc0660$@no> <001a01c93c5b$6d020830$0302a8c0@Mildred> <490CCA60.5060607@meadows.pair.com><000801c93cb6$c082c5c0$0302a8c0@Mildred> <490D6BA7.30607@skynet.be><002c01c93ce1$221094f0$0302a8c0@Mildred><490D98CC.6040908@talktalk.net> <5AAF2C28-908F-43D3-95E2-5767DA109E8D@starpower.net> <000201c93df6$04ae7100$0302a8c0@Mildred><490F768E.1010106@meadows.pair.com> <000001c93e05$2add3af0$8097b0d0$@com> Message-ID: <002001c93e59$56434370$0302a8c0@Mildred> Grattan Endicott To: "'Bridge Laws Mailing List'" Sent: Monday, November 03, 2008 10:40 PM Subject: Re: [blml] New Laws don't allow UI situation to be resolved? > > [DALB] > Indeed, which is why I am a bit puzzled by people asking "Am I supposed to call the Director any time I hear anything at all that might vaguely be related to a board I might or might not be about to play?" The answer to that is "No", > +=+ However, if the information turns out to be significant the Director, the AC, and public opinion, may well question whether you should have realized it could well be. And the law is clear enough that the Director should be notified forthwith when a player accidentally receives unauthorized information about a board he is playing or has yet to play. The condition for notifying the Director is one of fact and there is an onus on the player to recognize the possibility when it can reasonably be held he should have done so. ~ Grattan ~ +=+ From grandaeval at tiscali.co.uk Tue Nov 4 10:58:08 2008 From: grandaeval at tiscali.co.uk (Grattan) Date: Tue, 4 Nov 2008 09:58:08 -0000 Subject: [blml] Quacky fits References: <490EF965.6070004@skynet.be><000301c93df6$04f39050$0302a8c0@Mildred> <491004DB.70002@skynet.be> Message-ID: <000a01c93e64$34a4ff50$0302a8c0@Mildred> Grattan Endicott To: "Bridge Laws Mailing List" Sent: Tuesday, November 04, 2008 8:16 AM Subject: Re: [blml] Quacky fits > Grattan wrote: >> >>> >>> But do they (and you) agree that the MI rules are the ones to use? >>> >> +=+ Broadly Herman is stating a truth, although the Director's mode of >> enquiry is a matter for her and the RA.. However misinformation is not >> the only potential crime. The *use* of the call may lack authority under >> Law 40A3. The full extent of the law is available to the Director, who >> may have guidance on policy from the RA. Possibilities to examine >> are those in Laws 40B4, 40B5, and 40B6(b) and, if making the call >> was unlawful, also 12A2. >> ~ Grattan ~ +=+ >> > > Grattan, please be very precise here. > The *use* of the law may lack authority under L40A3. > What is the ruling the TD ought to give if she decides this is the case? > +=+ Different RAs will issue different guidelines on dealing with use of illegal calls. If it results in a favourable outcome for opponents then obviously no score adjustment. If advantage accrues to the offender then score adjustment should occur. Dependent on degree of aggravation then a warning to the offender may suffice, or a PP is an available sanction. Repetition is a serious aggravation and should be more harshly sanctioned. The EBU White Book will provide an example of one possible approach to the subject - look for 'illegal method' under 'procedural penalties'. ~ Grattan ~ +=+ From grandaeval at tiscali.co.uk Tue Nov 4 11:00:36 2008 From: grandaeval at tiscali.co.uk (Grattan) Date: Tue, 4 Nov 2008 10:00:36 -0000 Subject: [blml] Mr Burn suggested References: <48FC56AB.1050505@skynet.be> <02A30BE1-5E95-41F3-AE74-1936A0913D29@starpower.net> <48FCAB1E.8010201@skynet.be> <2a1c3a560810201658h1877df31n5886171a8165db7c@mail.gmail.com> <2a1c3a560810201952j563d60f9ua2096c951f2885b8@mail.gmail.com><48FDDBFF.7070300@connecttime.net><002401c93dab$7960f8b0$0302a8c0@Mildred> <002d01c93e1c$28b1ff10$0302a8c0@Mildred> <4910030E.2010905@skynet.be> Message-ID: <000b01c93e64$34e7d4b0$0302a8c0@Mildred> Grattan Endicott To: "Bridge Laws Mailing List" Sent: Tuesday, November 04, 2008 8:08 AM Subject: Re: [blml] Mr Burn suggested > Grattan wrote: >> >> Grattan Endicott> also > ************************************** >> "Let us then be up and doing, >> With a heart for any fate, >> Still achieving, still pursuing, >> Learn to labour and to wait." >> [H.W. Longfellow] >> >> ''''''''''''''''''''''''''''''''''''''''''''''''''''''''''''''''''''''''''''''''''''''''''''''''' >> >> >> >> ----- Original Message ----- >> From: "John (MadDog) Probst" >> To: "Bridge Laws Mailing List" >> Sent: Tuesday, November 04, 2008 12:36 AM >> Subject: Re: [blml] Mr Burn suggested >> >> >>> Grattan, There is NO formally discernable pattern; >>> I'm still home free. Partner CAN'T PREDICT my psych. >>> >> +=+ Not good enough. Partner is still aware of the increased >> potential for a psyche to occur. He has more reason to be >> aware of the deviation than have opponents. >> ~ G ~ +=+ >> > > Well, he does tell them - so that's correcting some. > But Grattan, please tell us: > What is the ruling on John's psyche if you do find his > partner has more reason to be aware of it? > > Herman. > +=+ Treat as use of an illegal method. ~ G ~ +=+ From wjburrows at gmail.com Tue Nov 4 11:11:31 2008 From: wjburrows at gmail.com (Wayne Burrows) Date: Tue, 4 Nov 2008 23:11:31 +1300 Subject: [blml] Mr Burn suggested In-Reply-To: <001301c93e4e$4a819fb0$0302a8c0@Mildred> References: <2a1c3a560810201658h1877df31n5886171a8165db7c@mail.gmail.com> <2a1c3a560810201952j563d60f9ua2096c951f2885b8@mail.gmail.com> <48FDDBFF.7070300@connecttime.net> <002401c93dab$7960f8b0$0302a8c0@Mildred> <002d01c93e1c$28b1ff10$0302a8c0@Mildred> <2a1c3a560811031811p4e0597e3s5c30da7a72dba451@mail.gmail.com> <001301c93e4e$4a819fb0$0302a8c0@Mildred> Message-ID: <2a1c3a560811040211we3724e9r5666f5193166f293@mail.gmail.com> 2008/11/4 Grattan : > > > Grattan Endicott also ************************************** > "Let us then be up and doing, > With a heart for any fate, > Still achieving, still pursuing, > Learn to labour and to wait." > [H.W. Longfellow] > > ''''''''''''''''''''''''''''''''''''''''''''''''''''''''''''''''''''''''''''''''''''''''''''''''' > > > > ----- Original Message ----- > From: "Wayne Burrows" > To: "Bridge Laws Mailing List" > Sent: Tuesday, November 04, 2008 2:11 AM > Subject: Re: [blml] Mr Burn suggested > > >> 2008/11/4 Grattan : >>> >>> >>> Grattan Endicott>> also >> ************************************** >>> "Let us then be up and doing, >>> With a heart for any fate, >>> Still achieving, still pursuing, >>> Learn to labour and to wait." >>> [H.W. Longfellow] >>> >>> ''''''''''''''''''''''''''''''''''''''''''''''''''''''''''''''''''''''''''''''''''''''''''''''''' >>> >>> >>> >>> ----- Original Message ----- >>> From: "John (MadDog) Probst" >>> To: "Bridge Laws Mailing List" >>> Sent: Tuesday, November 04, 2008 12:36 AM >>> Subject: Re: [blml] Mr Burn suggested >>> >>> >>>> >>>> Grattan, There is NO formally discernable pattern; >>>> I'm still home free. Partner CAN'T PREDICT my psych. >>>> >>> +=+ Not good enough. Partner is still aware of the increased >>> potential for a psyche to occur. He has more reason to be >>> aware of the deviation than have opponents. >>> ~ G ~ +=+ >>> >> >> I would describe this situation as "more reason to be aware of a >> deviation" rather than "more reason to be aware of the deviation" >> which relates to this specific deviation. An increased awareness of a >> potential deviation does not mean more aware of any particular actual >> deviation which is what is required by the use of the definite >> article. >> > +=+ The deviation occurs when he leaves the path of announced > meanings and specification of what path he then may take is a > separate issue. Opponents are entitled to have the same access > as partner to information that opens up the possibility of a deviation > of some kind. They also have the right to equal access with partner > to any information that suggests what the nature of the deviation > might be. > ~ Grattan ~ +=+ > Grattan the law does not say be aware of the possibility of a deviation of some kind. It says you are only restricted if you are aware of this particular deviation. "A player may deviate from his side's announced understandings always provided that his partner has no more reason to be aware of the deviation than have the opponents." Your meaning and interpretation would be more consistent with one of the drafts that was rejected as the word "anticipate" was crossed out and "be aware" was substituted. It is clear that in common language that "the deviation" refers to the same thing that is referred to by "may deviate" earlier in the same sentence. -- Wayne Burrows Palmerston North New Zealand From Hermandw at skynet.be Tue Nov 4 13:19:20 2008 From: Hermandw at skynet.be (Herman De Wael) Date: Tue, 04 Nov 2008 13:19:20 +0100 Subject: [blml] Mr Burn suggested In-Reply-To: <000b01c93e64$34e7d4b0$0302a8c0@Mildred> References: <48FC56AB.1050505@skynet.be> <02A30BE1-5E95-41F3-AE74-1936A0913D29@starpower.net> <48FCAB1E.8010201@skynet.be> <2a1c3a560810201658h1877df31n5886171a8165db7c@mail.gmail.com> <2a1c3a560810201952j563d60f9ua2096c951f2885b8@mail.gmail.com><48FDDBFF.7070300@connecttime.net><002401c93dab$7960f8b0$0302a8c0@Mildred> <002d01c93e1c$28b1ff10$0302a8c0@Mildred> <4910030E.2010905@skynet.be> <000b01c93e64$34e7d4b0$0302a8c0@Mildred> Message-ID: <49103DC8.6010308@skynet.be> Grattan wrote: > >>>> >>> +=+ Not good enough. Partner is still aware of the increased >>> potential for a psyche to occur. He has more reason to be >>> aware of the deviation than have opponents. >>> ~ G ~ +=+ >>> >> Well, he does tell them - so that's correcting some. >> But Grattan, please tell us: >> What is the ruling on John's psyche if you do find his >> partner has more reason to be aware of it? >> >> Herman. >> > +=+ Treat as use of an illegal method. ~ G ~ +=+ > Which means? Sorry, Grattan, but this answer is not enough. You say first "this is an illegal method" and as answer to my question "treat as an illegal method". I would really like to know how one treats an illegal method. Please, with law references. I am serious, Grattan. I do think I know what you are suggesting, but I cannot be certain of anything anymore. Which is why I want you to spell it out for me. And for the rest of the world. Herman. From Hermandw at skynet.be Tue Nov 4 13:21:39 2008 From: Hermandw at skynet.be (Herman De Wael) Date: Tue, 04 Nov 2008 13:21:39 +0100 Subject: [blml] Quacky fits In-Reply-To: <000a01c93e64$34a4ff50$0302a8c0@Mildred> References: <490EF965.6070004@skynet.be><000301c93df6$04f39050$0302a8c0@Mildred> <491004DB.70002@skynet.be> <000a01c93e64$34a4ff50$0302a8c0@Mildred> Message-ID: <49103E53.2030304@skynet.be> Grattan wrote: > > Grattan Endicott also ************************************** > "Let us then be up and doing, > With a heart for any fate, > Still achieving, still pursuing, > Learn to labour and to wait." > [H.W. Longfellow] > > ''''''''''''''''''''''''''''''''''''''''''''''''''''''''''''''''''''''''''''''''''''''''''''''''' > > > > ----- Original Message ----- > From: "Herman De Wael" > To: "Bridge Laws Mailing List" > Sent: Tuesday, November 04, 2008 8:16 AM > Subject: Re: [blml] Quacky fits > > >> Grattan wrote: >>>> But do they (and you) agree that the MI rules are the ones to use? >>>> >>> +=+ Broadly Herman is stating a truth, although the Director's mode of >>> enquiry is a matter for her and the RA.. However misinformation is not >>> the only potential crime. The *use* of the call may lack authority under >>> Law 40A3. The full extent of the law is available to the Director, who >>> may have guidance on policy from the RA. Possibilities to examine >>> are those in Laws 40B4, 40B5, and 40B6(b) and, if making the call >>> was unlawful, also 12A2. >>> ~ Grattan ~ +=+ >>> >> Grattan, please be very precise here. >> The *use* of the law may lack authority under L40A3. >> What is the ruling the TD ought to give if she decides this is the case? >> > +=+ Different RAs will issue different guidelines on dealing > with use of illegal calls. No Grattan, you will not get away with that. You are applying L40C3. That is not subject to RA guidelines. Or at least it should not be. > If it results in a favourable outcome for > opponents then obviously no score adjustment. If advantage > accrues to the offender then score adjustment should occur. AS by L12 - based on what? > Dependent on degree of aggravation then a warning to the > offender may suffice, or a PP is an available sanction. > Repetition is a serious aggravation and should be more > harshly sanctioned. That goes without saying. > The EBU White Book will provide an example of one > possible approach to the subject - look for 'illegal method' > under 'procedural penalties'. > ~ Grattan ~ +=+ > No Grattan, I am not in England, I have no reason to read the EBY White Book. International Laws please. Herman. From grandaeval at tiscali.co.uk Tue Nov 4 13:34:08 2008 From: grandaeval at tiscali.co.uk (Grattan) Date: Tue, 4 Nov 2008 12:34:08 -0000 Subject: [blml] Mr Burn suggested References: <2a1c3a560810201658h1877df31n5886171a8165db7c@mail.gmail.com><2a1c3a560810201952j563d60f9ua2096c951f2885b8@mail.gmail.com><48FDDBFF.7070300@connecttime.net><002401c93dab$7960f8b0$0302a8c0@Mildred><002d01c93e1c$28b1ff10$0302a8c0@Mildred><2a1c3a560811031811p4e0597e3s5c30da7a72dba451@mail.gmail.com><001301c93e4e$4a819fb0$0302a8c0@Mildred> <2a1c3a560811040211we3724e9r5666f5193166f293@mail.gmail.com> Message-ID: <007101c93e7b$d53b1b90$0302a8c0@Mildred> Grattan Endicott To: "Bridge Laws Mailing List" Sent: Tuesday, November 04, 2008 10:11 AM Subject: Re: [blml] Mr Burn suggested > > It says you are only restricted if you are aware of this particular > deviation. > > "A player may deviate from his side's announced > understandings always provided that his partner has no > more reason to be aware of the deviation than > have the opponents." > > Your meaning and interpretation would be more consistent with one of > the drafts that was rejected as the word "anticipate" was crossed out > and "be aware" was substituted. > > It is clear that in common language that "the deviation" refers to the > same thing that is referred to by "may deviate" earlier in the same > sentence. > +=+ I disagree. I disagree absolutely. The act of deviation is the act of diverging from the normal meaning, no matter in what direction you diverge. 'May deviate' has the same meaning, i.e. that he may diverge from. There is absolutely no specification of a need to identify in what particular direction he will go when he leaves the path. If he leaves the path of normality in any direction he has diverged, has deviated. Opponents are entitled to know as much as partner about the potential for deviation. As an entirely separate issue opponents are equally entitled to know as much as partner about the possible form deviation may take. ~ Grattan ~ +=+ From nigelguthrie at talktalk.net Tue Nov 4 14:31:18 2008 From: nigelguthrie at talktalk.net (Nigel Guthrie) Date: Tue, 04 Nov 2008 13:31:18 +0000 Subject: [blml] Mr Burn suggested In-Reply-To: <2a1c3a560811040211we3724e9r5666f5193166f293@mail.gmail.com> References: <2a1c3a560810201658h1877df31n5886171a8165db7c@mail.gmail.com> <2a1c3a560810201952j563d60f9ua2096c951f2885b8@mail.gmail.com> <48FDDBFF.7070300@connecttime.net> <002401c93dab$7960f8b0$0302a8c0@Mildred> <002d01c93e1c$28b1ff10$0302a8c0@Mildred> <2a1c3a560811031811p4e0597e3s5c30da7a72dba451@mail.gmail.com> <001301c93e4e$4a819fb0$0302a8c0@Mildred> <2a1c3a560811040211we3724e9r5666f5193166f293@mail.gmail.com> Message-ID: <49104EA6.1060200@talktalk.net> Whenever Herman holds 0-3 HCP in 3rd seat, he opens 1H. Have I got that right Herman?. IMO - - This 1H opener isn't a psych. It's a *convention*. - If Herman's partner doesn't do the same, then, unless the licensing authority allow *asymmetric* systems, he's breaking the law. - If Herman's *licensing* authority doesn't permit this convention, then he's breaking the law. - Unless Herman *discloses* this convention to opponents, he's breaking the law. To appreciate that hand *frequency* is irrelevant, consider another example. Suppose that you open 2C - whenever you hold 5-10 HCP and 6 diamonds; - and also when you hold 20+HCP and 4441 shape. Hands that satisfy the latter requirement are infrequent But you must still disclose that possibility. How can Bridge sink to a state, where such basic issues cause endless controversy and confusion? IMO there are two main reasons - - Bridge laws and regulations are overly sophisticated and subjective. - The WBFLC have built a tower of Babel by delegating their responsibilities to local legislators. For instance, the WBFLC should lay down a *default* disclosure protocol that is simple but complete. From grandaeval at tiscali.co.uk Tue Nov 4 14:37:34 2008 From: grandaeval at tiscali.co.uk (Grattan) Date: Tue, 4 Nov 2008 13:37:34 -0000 Subject: [blml] Quacky fits References: <490EF965.6070004@skynet.be><000301c93df6$04f39050$0302a8c0@Mildred> <491004DB.70002@skynet.be><000a01c93e64$34a4ff50$0302a8c0@Mildred> <49103E53.2030304@skynet.be> Message-ID: <008801c93e82$95c3fd90$0302a8c0@Mildred> Grattan Endicott To: "Bridge Laws Mailing List" Sent: Tuesday, November 04, 2008 12:21 PM Subject: Re: [blml] Quacky fits > Grattan wrote: >> >> Grattan Endicott> also > ************************************** >> "Let us then be up and doing, >> With a heart for any fate, >> Still achieving, still pursuing, >> Learn to labour and to wait." >> [H.W. Longfellow] >> >> ''''''''''''''''''''''''''''''''''''''''''''''''''''''''''''''''''''''''''''''''''''''''''''''''' >> >> >> >> ----- Original Message ----- >> From: "Herman De Wael" >> To: "Bridge Laws Mailing List" >> Sent: Tuesday, November 04, 2008 8:16 AM >> Subject: Re: [blml] Quacky fits >> >> >>> Grattan wrote: >>>>> But do they (and you) agree that the MI rules are the ones to use? >>>>> >>>> +=+ Broadly Herman is stating a truth, although the Director's mode of >>>> enquiry is a matter for her and the RA.. However misinformation is not >>>> the only potential crime. The *use* of the call may lack authority >>>> under >>>> Law 40A3. The full extent of the law is available to the Director, who >>>> may have guidance on policy from the RA. Possibilities to examine >>>> are those in Laws 40B4, 40B5, and 40B6(b) and, if making the call >>>> was unlawful, also 12A2. >>>> ~ Grattan ~ +=+ >>>> >>> Grattan, please be very precise here. >>> The *use* of the law may lack authority under L40A3. >>> What is the ruling the TD ought to give if she decides this is the case? >>> >> +=+ Different RAs will issue different guidelines on dealing >> with use of illegal calls. > > No Grattan, you will not get away with that. You are applying L40C3. > That is not subject to RA guidelines. Or at least it should not be. > >> If it results in a favourable outcome for >> opponents then obviously no score adjustment. If advantage >> accrues to the offender then score adjustment should occur. > > AS by L12 - based on what? > >> Dependent on degree of aggravation then a warning to the >> offender may suffice, or a PP is an available sanction. >> Repetition is a serious aggravation and should be more >> harshly sanctioned. > > That goes without saying. > >> The EBU White Book will provide an example of one >> possible approach to the subject - look for 'illegal method' >> under 'procedural penalties'. >> ~ Grattan ~ +=+ >> > > No Grattan, I am not in England, I have no reason to read > the EBU White Book. International Laws please. > > > Herman. > +=+ Herman, You have my answer. If you want to find out more you will have to do your own research on the guidance given by RAs on the Director's treatment of use of illegal methods. That guidance will tell the Director the RA's preferred course of action within his discretion to apply Laws 40B4, 40B5, and 40B6(b), 12A2 and, of course, Laws 12A1, 90 and 91. I invited you to examine what the EBU says - it is likely to adopt a less harsh attitude than some - but if you are not inclined to do that I see no reason to discuss the subject any further. ~ Grattan ~ +=+ From Hermandw at skynet.be Tue Nov 4 15:28:30 2008 From: Hermandw at skynet.be (Herman De Wael) Date: Tue, 04 Nov 2008 15:28:30 +0100 Subject: [blml] Quacky fits In-Reply-To: <008801c93e82$95c3fd90$0302a8c0@Mildred> References: <490EF965.6070004@skynet.be><000301c93df6$04f39050$0302a8c0@Mildred> <491004DB.70002@skynet.be><000a01c93e64$34a4ff50$0302a8c0@Mildred> <49103E53.2030304@skynet.be> <008801c93e82$95c3fd90$0302a8c0@Mildred> Message-ID: <49105C0E.7050003@skynet.be> Grattan wrote: > >>> >> No Grattan, I am not in England, I have no reason to read >> the EBU White Book. International Laws please. >> >> >> Herman. >> > +=+ Herman, > You have my answer. If you want to find out more you will > have to do your own research on the guidance given by RAs on > the Director's treatment of use of illegal methods. That guidance > will tell the Director the RA's preferred course of action within > his discretion to apply Laws 40B4, 40B5, and 40B6(b), 12A2 > and, of course, Laws 12A1, 90 and 91. I invited you to examine > what the EBU says - it is likely to adopt a less harsh attitude than > some - but if you are not inclined to do that I see no reason to > discuss the subject any further. > ~ Grattan ~ +=+ > Does anyone else than me consider this a non-answer? What do we do? Cancel the board or award an AS based on the opponents being fully informed about the possibilities of the deviation? I still don't know. Well, I know what I would do, but I don't even know if I'm being considered a crazy Belgian about it or not! Herman. From Hermandw at skynet.be Tue Nov 4 15:36:47 2008 From: Hermandw at skynet.be (Herman De Wael) Date: Tue, 04 Nov 2008 15:36:47 +0100 Subject: [blml] Mr Burn suggested In-Reply-To: <49104EA6.1060200@talktalk.net> References: <2a1c3a560810201658h1877df31n5886171a8165db7c@mail.gmail.com> <2a1c3a560810201952j563d60f9ua2096c951f2885b8@mail.gmail.com> <48FDDBFF.7070300@connecttime.net> <002401c93dab$7960f8b0$0302a8c0@Mildred> <002d01c93e1c$28b1ff10$0302a8c0@Mildred> <2a1c3a560811031811p4e0597e3s5c30da7a72dba451@mail.gmail.com> <001301c93e4e$4a819fb0$0302a8c0@Mildred> <2a1c3a560811040211we3724e9r5666f5193166f293@mail.gmail.com> <49104EA6.1060200@talktalk.net> Message-ID: <49105DFF.4090106@skynet.be> Nigel Guthrie wrote: > Whenever Herman holds 0-3 HCP in 3rd seat, he opens 1H. Have I got that > right Herman?. IMO - > Not completely, but it's close enough to be the basis of this discussion. > - This 1H opener isn't a psych. It's a *convention*. Well, if you state it like that, I'm sure to have to agree ;) > - If Herman's partner doesn't do the same, then, unless the licensing > authority allow *asymmetric* systems, he's breaking the law. Only if it is indeed systemic. > - If Herman's *licensing* authority doesn't permit this convention, > then he's breaking the law. Only if it is indeed systemic. > - Unless Herman *discloses* this convention to opponents, he's breaking > the law. > Which I (try to) do. (disclosing, I mean, not breaking the law) > To appreciate that hand *frequency* is irrelevant, consider another > example. Suppose that you open 2C Well, if you state it like that, I'm sure to have to agree ;) > - whenever you hold 5-10 HCP and 6 diamonds; > - and also when you hold 20+HCP and 4441 shape. > Hands that satisfy the latter requirement are infrequent But you must > still disclose that possibility. > Of course. But don't you see the difference between the two cases? In one, I have no method for describing my hand afterwards - in the other, I'm bound to have some agreement with partner for showing either of the possible holdings. Nigel, nothing you say here is new. It's been said several times over the past few years. I have always answered it. If you consider that by saying that I have performed a particular deviation some times before in the past, I am now no longer able to perform that same deviation, then you belong to the camp that says that psyches are allowed provide you never actually do one. That cannot be the true state of affairs. I am trying to be honest here, by telling my opponents which psyches I have performed in the past and which I might do again. If you forbid me to perform those psyches then you will never ever hear anybody admit to having psyched once before. Is that the state of play you wish to see? I don't. > How can Bridge sink to a state, where such basic issues cause endless > controversy and confusion? IMO there are two main reasons - > - Bridge laws and regulations are overly sophisticated and subjective. Perhaps yes. > - The WBFLC have built a tower of Babel by delegating their > responsibilities to local legislators. For instance, the WBFLC should > lay down a *default* disclosure protocol that is simple but complete. > Indeed. A third reason is that people like you never read what I write, never think about it for some time longer than to say "crazy Belgian", and never want to change their opinion about anything. If you would simply allow me to psyche as I have done in the past, nothing is wrong. Herman. From john at asimere.com Tue Nov 4 17:31:51 2008 From: john at asimere.com (John (MadDog) Probst) Date: Tue, 4 Nov 2008 16:31:51 -0000 Subject: [blml] Mr Burn suggested References: <48FC56AB.1050505@skynet.be> <02A30BE1-5E95-41F3-AE74-1936A0913D29@starpower.net> <48FCAB1E.8010201@skynet.be> <2a1c3a560810201658h1877df31n5886171a8165db7c@mail.gmail.com> <2a1c3a560810201952j563d60f9ua2096c951f2885b8@mail.gmail.com><48FDDBFF.7070300@connecttime.net><002401c93dab$7960f8b0$0302a8c0@Mildred> <002d01c93e1c$28b1ff10$0302a8c0@Mildred> Message-ID: <1326DFF27E07403B8DDB13650C0B578F@JOHN> ----- Original Message ----- From: "Grattan" To: "Bridge Laws Mailing List" Sent: Tuesday, November 04, 2008 1:24 AM Subject: Re: [blml] Mr Burn suggested > > > Grattan Endicott also ************************************** > "Let us then be up and doing, > With a heart for any fate, > Still achieving, still pursuing, > Learn to labour and to wait." > [H.W. Longfellow] > > ''''''''''''''''''''''''''''''''''''''''''''''''''''''''''''''''''''''''''''''''''''''''''''''''' > > > > ----- Original Message ----- > From: "John (MadDog) Probst" > To: "Bridge Laws Mailing List" > Sent: Tuesday, November 04, 2008 12:36 AM > Subject: Re: [blml] Mr Burn suggested > > >> >> Grattan, There is NO formally discernable pattern; >> I'm still home free. Partner CAN'T PREDICT my psych. >> > +=+ Not good enough. Partner is still aware of the increased > potential for a psyche to occur. He has more reason to be > aware of the deviation than have opponents. which is why he discloses it. But it does not enter his head to expect i've done it. John > ~ G ~ +=+ > > > _______________________________________________ > blml mailing list > blml at amsterdamned.org > http://www.amsterdamned.org/mailman/listinfo/blml From john at asimere.com Tue Nov 4 17:33:25 2008 From: john at asimere.com (John (MadDog) Probst) Date: Tue, 4 Nov 2008 16:33:25 -0000 Subject: [blml] Mr Burn suggested References: <48FC56AB.1050505@skynet.be><02A30BE1-5E95-41F3-AE74-1936A0913D29@starpower.net><48FCAB1E.8010201@skynet.be><2a1c3a560810201658h1877df31n5886171a8165db7c@mail.gmail.com><2a1c3a560810201952j563d60f9ua2096c951f2885b8@mail.gmail.com><48FDDBFF.7070300@connecttime.net><002401c93dab$7960f8b0$0302a8c0@Mildred> <6.1.0.6.2.20081104142750.01db0ec0@mail.optusnet.com.au> Message-ID: ----- Original Message ----- From: "Tony Musgrove" To: "Bridge Laws Mailing List" Sent: Tuesday, November 04, 2008 3:28 AM Subject: Re: [blml] Mr Burn suggested > >>Grattan, There is NO formally discernable pattern; >>I'm still home free. Partner CAN'T PREDICT my psych. > > I bet that like NZ sheep, they're nervous though so are my opponents :) > > Cheers > > Tony (Sydney) > > > >>; >> > >> > ~ Grattan ~ +=+ >> > >> > >> > >> > _______________________________________________ >> > blml mailing list >> > blml at amsterdamned.org >> > http://www.amsterdamned.org/mailman/listinfo/blml >> >> >> >>_______________________________________________ >>blml mailing list >>blml at amsterdamned.org >>http://www.amsterdamned.org/mailman/listinfo/blml > > > _______________________________________________ > blml mailing list > blml at amsterdamned.org > http://www.amsterdamned.org/mailman/listinfo/blml From john at asimere.com Tue Nov 4 17:35:01 2008 From: john at asimere.com (John (MadDog) Probst) Date: Tue, 4 Nov 2008 16:35:01 -0000 Subject: [blml] Mr Burn suggested References: <48FC56AB.1050505@skynet.be><02A30BE1-5E95-41F3-AE74-1936A0913D29@starpower.net><48FCAB1E.8010201@skynet.be><2a1c3a560810201658h1877df31n5886171a8165db7c@mail.gmail.com><2a1c3a560810201952j563d60f9ua2096c951f2885b8@mail.gmail.com><48FDDBFF.7070300@connecttime.net><002401c93dab$7960f8b0$0302a8c0@Mildred><6.1.0.6.2.20081104142750.01db0ec0@mail.optusnet.com.au> <001001c93e56$c21363d0$0302a8c0@Mildred> Message-ID: <58770ECEBA8A464F9F6948B8A01F8FE4@JOHN> ----- Original Message ----- From: "Grattan" To: "Bridge Laws Mailing List" Sent: Tuesday, November 04, 2008 8:24 AM Subject: Re: [blml] Mr Burn suggested > > > Grattan Endicott also ************************************** > "Let us then be up and doing, > With a heart for any fate, > Still achieving, still pursuing, > Learn to labour and to wait." > [H.W. Longfellow] > > ''''''''''''''''''''''''''''''''''''''''''''''''''''''''''''''''''''''''''''''''''''''''''''''''' > > > > ----- Original Message ----- > From: "Tony Musgrove" > To: "Bridge Laws Mailing List" > Sent: Tuesday, November 04, 2008 3:28 AM > Subject: Re: [blml] Mr Burn suggested > > >> >>>Grattan, There is NO formally discernable pattern; >>>I'm still home free. Partner CAN'T PREDICT my psych. >> >> I bet that like NZ sheep, they're nervous though >> >> Cheers >> >> Tony (Sydney) >> > +=+ Sorry Tony, but what you say falls short of the requirement. > Partner's awareness that you may well psyche in a given situation > is something opponents are entitled to share even if the nature > of the psyche is unpredictable (which unpredictability is apt to > be a questionable assertion anyway, in the light of partnership > experience, but is a separate issue). Indeed, the unpredictability is a cornerstone of my argument. John > ~ Grattan ~ +=+ > > > > _______________________________________________ > blml mailing list > blml at amsterdamned.org > http://www.amsterdamned.org/mailman/listinfo/blml From john at asimere.com Tue Nov 4 17:37:52 2008 From: john at asimere.com (John (MadDog) Probst) Date: Tue, 4 Nov 2008 16:37:52 -0000 Subject: [blml] Mr Burn suggested References: <48FC56AB.1050505@skynet.be> <02A30BE1-5E95-41F3-AE74-1936A0913D29@starpower.net> <48FCAB1E.8010201@skynet.be> <2a1c3a560810201658h1877df31n5886171a8165db7c@mail.gmail.com> <2a1c3a560810201952j563d60f9ua2096c951f2885b8@mail.gmail.com><48FDDBFF.7070300@connecttime.net><002401c93dab$7960f8b0$0302a8c0@Mildred> <002d01c93e1c$28b1ff10$0302a8c0@Mildred><4910030E.2010905@skynet.be> <000b01c93e64$34e7d4b0$0302a8c0@Mildred> Message-ID: <224A3AA133794C8298368C7E5AD9949E@JOHN> ----- Original Message ----- From: "Grattan" To: "Bridge Laws Mailing List" Sent: Tuesday, November 04, 2008 10:00 AM Subject: Re: [blml] Mr Burn suggested > > > Grattan Endicott also ************************************** > "Let us then be up and doing, > With a heart for any fate, > Still achieving, still pursuing, > Learn to labour and to wait." > [H.W. Longfellow] > > ''''''''''''''''''''''''''''''''''''''''''''''''''''''''''''''''''''''''''''''''''''''''''''''''' > > > > ----- Original Message ----- > From: "Herman De Wael" > To: "Bridge Laws Mailing List" > Sent: Tuesday, November 04, 2008 8:08 AM > Subject: Re: [blml] Mr Burn suggested > > >> Grattan wrote: >>> >>> Grattan Endicott>> also >> ************************************** >>> "Let us then be up and doing, >>> With a heart for any fate, >>> Still achieving, still pursuing, >>> Learn to labour and to wait." >>> [H.W. Longfellow] >>> >>> ''''''''''''''''''''''''''''''''''''''''''''''''''''''''''''''''''''''''''''''''''''''''''''''''' >>> >>> >>> >>> ----- Original Message ----- >>> From: "John (MadDog) Probst" >>> To: "Bridge Laws Mailing List" >>> Sent: Tuesday, November 04, 2008 12:36 AM >>> Subject: Re: [blml] Mr Burn suggested >>> >>> >>>> Grattan, There is NO formally discernable pattern; >>>> I'm still home free. Partner CAN'T PREDICT my psych. >>>> >>> +=+ Not good enough. Partner is still aware of the increased >>> potential for a psyche to occur. He has more reason to be >>> aware of the deviation than have opponents. >>> ~ G ~ +=+ >>> >> >> Well, he does tell them - so that's correcting some. >> But Grattan, please tell us: >> What is the ruling on John's psyche if you do find his >> partner has more reason to be aware of it? >> >> Herman. >> > +=+ Treat as use of an illegal method. ~ G ~ +=+ and now I pass my 6 card H suit over 1 minor with a 4 count. ? (which I do most of the time, partner having alerted my 1NT overcall on the previous board and finding me with a flat 16 and a stopper.) if I were an opponent, I'd want an adjusted score for my NOT having overcalled 1NT. > > > _______________________________________________ > blml mailing list > blml at amsterdamned.org > http://www.amsterdamned.org/mailman/listinfo/blml From wjburrows at gmail.com Tue Nov 4 19:49:15 2008 From: wjburrows at gmail.com (Wayne Burrows) Date: Wed, 5 Nov 2008 07:49:15 +1300 Subject: [blml] Mr Burn suggested In-Reply-To: <007101c93e7b$d53b1b90$0302a8c0@Mildred> References: <2a1c3a560810201952j563d60f9ua2096c951f2885b8@mail.gmail.com> <48FDDBFF.7070300@connecttime.net> <002401c93dab$7960f8b0$0302a8c0@Mildred> <002d01c93e1c$28b1ff10$0302a8c0@Mildred> <2a1c3a560811031811p4e0597e3s5c30da7a72dba451@mail.gmail.com> <001301c93e4e$4a819fb0$0302a8c0@Mildred> <2a1c3a560811040211we3724e9r5666f5193166f293@mail.gmail.com> <007101c93e7b$d53b1b90$0302a8c0@Mildred> Message-ID: <2a1c3a560811041049s35c62024ha3ee7bbf05387ce2@mail.gmail.com> 2008/11/5 Grattan : > > > Grattan Endicott also ************************************** > "Let us then be up and doing, > With a heart for any fate, > Still achieving, still pursuing, > Learn to labour and to wait." > [H.W. Longfellow] > > ''''''''''''''''''''''''''''''''''''''''''''''''''''''''''''''''''''''''''''''''''''''''''''''''' > > > > ----- Original Message ----- > From: "Wayne Burrows" > To: "Bridge Laws Mailing List" > Sent: Tuesday, November 04, 2008 10:11 AM > Subject: Re: [blml] Mr Burn suggested >> >> It says you are only restricted if you are aware of this particular >> deviation. >> >> "A player may deviate from his side's announced >> understandings always provided that his partner has no >> more reason to be aware of the deviation than >> have the opponents." >> >> Your meaning and interpretation would be more consistent with one of >> the drafts that was rejected as the word "anticipate" was crossed out >> and "be aware" was substituted. >> >> It is clear that in common language that "the deviation" refers to the >> same thing that is referred to by "may deviate" earlier in the same >> sentence. >> > +=+ I disagree. I disagree absolutely. The act of deviation is the act > of diverging from the normal meaning, no matter in what direction you > diverge. 'May deviate' has the same meaning, i.e. that he may diverge > from. There is absolutely no specification of a need to identify in what > particular direction he will go when he leaves the path. If he leaves > the path of normality in any direction he has diverged, has deviated. > Opponents are entitled to know as much as partner about the > potential for deviation. As an entirely separate issue opponents are > equally entitled to know as much as partner about the possible form > deviation may take. > ~ Grattan ~ +=+ > You miss my point completely. You "may deviate" provided partner is no more aware of "the deviation". This is not the same as partner is no more aware of "the potential for deviation". This language requires that partner must be more aware of this particular deviation. That is that a deviation has occurred with this bid. At the moment that my partner psyches 1S 3rd in hand I am not more aware of the deviation than the opponents. I have no arrangement to psyche on this particular board and have no knowledge of my partner's hand so how can I be more aware of "the deviation" that has just occurred. This fact gives my partner licence to "deviate" since I am not "aware" of "the deviation" even if I am "aware" of "the potential to deviate". -- Wayne Burrows Palmerston North New Zealand From richard.hills at immi.gov.au Tue Nov 4 22:15:25 2008 From: richard.hills at immi.gov.au (richard.hills at immi.gov.au) Date: Wed, 5 Nov 2008 08:15:25 +1100 Subject: [blml] Mr Lincoln suggested [SEC=UNOFFICIAL] In-Reply-To: <490EB6ED.7080805@skynet.be> Message-ID: Abraham Lincoln, March 17th 1865: "Whenever I hear anyone arguing for slavery, I feel a strong impulse to see it tried on him personally." Herman De Wael asked: [big snip] >Or what do you think the penalty for a "concealed implicit >mutual partnership understanding" ought to be - disbarment? Law 40C3(b): "Repeated violations of requirements to disclose partnership understandings may be penalized." Richard Hills: Under the language of the new Lawbook, a "penalty" refers only to a Law 90 procedural penalty and/or a Law 91 disciplinary penalty. So if a Director is using Law 40C3(b) as the basis for her ruling, she has the discretion to consequently apply either Law 90 and/or Law 91, depending upon her judgement of the gravity of the Law 40C3(b) infraction and/or any mitigating circumstances. The Director may judge that if the offending player chooses a Lincolnesque call for the TD against his own interests (when the non-offending side is unaware of the infraction), then that is very much a mitigating circumstance. What's the problem? Best wishes Richard James Hills Recruitment Section, Level 3 Blue, workstation 15 (first on left) Department of Immigration and Citizenship Telephone: 02 6223 8453 Email: richard.hills at immi.gov.au -------------------------------------------------------------------- Important Notice: If you have received this email by mistake, please advise the sender and delete the message and attachments immediately. This email, including attachments, may contain confidential, sensitive, legally privileged and/or copyright information. Any review, retransmission, dissemination or other use of this information by persons or entities other than the intended recipient is prohibited. DIAC respects your privacy and has obligations under the Privacy Act 1988. The official departmental privacy policy can be viewed on the department's website at www.immi.gov.au. See: http://www.immi.gov.au/functional/privacy.htm --------------------------------------------------------------------- From nigelguthrie at talktalk.net Wed Nov 5 00:47:05 2008 From: nigelguthrie at talktalk.net (Nigel Guthrie) Date: Tue, 04 Nov 2008 23:47:05 +0000 Subject: [blml] Mr Burn suggested In-Reply-To: <49105DFF.4090106@skynet.be> References: <2a1c3a560810201658h1877df31n5886171a8165db7c@mail.gmail.com> <2a1c3a560810201952j563d60f9ua2096c951f2885b8@mail.gmail.com> <48FDDBFF.7070300@connecttime.net> <002401c93dab$7960f8b0$0302a8c0@Mildred> <002d01c93e1c$28b1ff10$0302a8c0@Mildred> <2a1c3a560811031811p4e0597e3s5c30da7a72dba451@mail.gmail.com> <001301c93e4e$4a819fb0$0302a8c0@Mildred> <2a1c3a560811040211we3724e9r5666f5193166f293@mail.gmail.com> <49104EA6.1060200@talktalk.net> <49105DFF.4090106@skynet.be> Message-ID: <4910DEF9.8070602@talktalk.net> [Nige1] To appreciate that hand *frequency* is irrelevant, consider another example. Suppose that you open 2C - whenever you hold 5-10 HCP and 6 diamonds; - and also when you hold 20+HCP and 4441 shape. Hands that satisfy the latter requirement are infrequent But you must still disclose that possibility. [Herman] Of course. But don't you see the difference between the two cases? In one, I have no method for describing my hand afterwards - in the other, I'm bound to have some agreement with partner for showing either of the possible holdings. [Nige2] IMO, this difference is spurious. - My conventional bid shows a 4441 20+HCP hand or a weak 2 in diamonds. Hence I *need* methods to describe my hand further. - Herman's conventional bid shows 0-3 HCP or a normal opener. If he holds the former, then he need not refine that messsage opposite his passed partner. Nevertheless, opponents' actions and Herman's eloquent passes may help his partner to distinguish which of the two conventional hand-types he holds. From richard.hills at immi.gov.au Wed Nov 5 07:25:53 2008 From: richard.hills at immi.gov.au (richard.hills at immi.gov.au) Date: Wed, 5 Nov 2008 17:25:53 +1100 Subject: [blml] Mr Lincoln suggested [SEC=UNOFFICIAL] In-Reply-To: <49105C0E.7050003@skynet.be> Message-ID: Abraham Lincoln (1809-1865): "He can compress the most words into the smallest idea of any man I ever met." Herman De Wael: >>>No Grattan, I am not in England, I have no reason to read >>>the EBU White Book. International Laws please. Grattan Endicott: >>+=+ Herman, >> You have my answer. If you want to find out more >>you will have to do your own research on the guidance given >>by RAs on the Director's treatment of use of illegal >>methods. That guidance will tell the Director the RA's >>preferred course of action within his discretion to apply >>Laws 40B4, 40B5, and 40B6(b), 12A2 and, of course, Laws >>12A1, 90 and 91. I invited you to examine what the EBU says >>- it is likely to adopt a less harsh attitude than some - >>but if you are not inclined to do that I see no reason to >>discuss the subject any further. >> ~ Grattan ~ +=+ Herman De Wael asked: >Does anyone else than me consider this a non-answer? Abraham Lincoln asked: "If I were two-faced, would I be wearing this one?" Herman De Wael: >What do we do? Cancel the board or award an AS based on the >opponents being fully informed about the possibilities of the >deviation? Richard Hills: Neither, a false dichotomy. Under both the EBU White Book and the International Laws a cancellation of the board due to what is colloquially known as an "illegal convention" would be a Director's Error. Law 40B5: "When a side is damaged by an opponent's use of a special partnership understanding that does not comply with the regulations governing the tournament the score **shall** be adjusted. A side in breach of those regulations may be subject to a procedural penalty." Herman De Wael: >I still don't know. > >Well, I know what I would do, but I don't even know if I'm >being considered a crazy Belgian about it or not! Richard Hills; I am not in England, but this crazy Aussie does have reason to read the EBU White Book, and also to read the EBU Laws and Ethics Committee minutes. Although these documents are not legally binding in God's own country, from sea to shining sea, they are useful examples of what another Regulating Authority does, and the reasoning that the RA uses. The English RA guidance on a Law 40B5 "illegal convention" is that the non-offending side deserves rectification of damage via an artificial adjusted score of Ave+ (but of course the NOS keeps its table score if better than Ave+, since the EBU rules that then the NOS has not been damaged). The offending side gets Ave- or its table score, whichever is worse. No procedural penalty for a revealed illegal convention, but a standard 10% of a top PP for any concealed partnership understanding (illegal convention or otherwise) -- which has the colourful misnomer in EBU regulation of "Red Psyche". In the most recent EBU L&EC minutes, the EBU justifies its decision to award ArtAS (rather than AssAS) for Law 40B5 "illegal convention" rulings under Law 12C1(d): "If the possibilities are numerous or not obvious, the Director may award an artificial adjusted score", so therefore the point is that a legal auction would not involve the "illegal convention" being _explained_, but rather the "illegal convention" not ever being _used_. Thus an alternative hypothetically legal auction would involve the assumption that the offending side instead used one of several zillion different legal methods, hence the possibilities are just a tad numerous. What's the problem? Best wishes Richard James Hills Recruitment Section, Level 3 Blue, workstation 15 (first on left) Department of Immigration and Citizenship Telephone: 02 6223 8453 Email: richard.hills at immi.gov.au -------------------------------------------------------------------- Important Notice: If you have received this email by mistake, please advise the sender and delete the message and attachments immediately. This email, including attachments, may contain confidential, sensitive, legally privileged and/or copyright information. Any review, retransmission, dissemination or other use of this information by persons or entities other than the intended recipient is prohibited. DIAC respects your privacy and has obligations under the Privacy Act 1988. The official departmental privacy policy can be viewed on the department's website at www.immi.gov.au. See: http://www.immi.gov.au/functional/privacy.htm --------------------------------------------------------------------- From grandaeval at tiscali.co.uk Wed Nov 5 09:32:49 2008 From: grandaeval at tiscali.co.uk (Grattan) Date: Wed, 5 Nov 2008 08:32:49 -0000 Subject: [blml] Mr Lincoln suggested [SEC=UNOFFICIAL] References: Message-ID: <001001c93f21$1d00eb80$0302a8c0@Mildred> Grattan Endicott To: "Bridge Laws Mailing List" Sent: Wednesday, November 05, 2008 6:25 AM Subject: Re: [blml] Mr Lincoln suggested [SEC=UNOFFICIAL] > Abraham Lincoln (1809-1865): > > "He can compress the most words into the smallest idea of any > man I ever met." > > Herman De Wael: > >>>>No Grattan, I am not in England, I have no reason to read >>>>the EBU White Book. International Laws please. > > Grattan Endicott: > >>>+=+ Herman, >>> You have my answer. If you want to find out more >>>you will have to do your own research on the guidance given >>>by RAs on the Director's treatment of use of illegal >>>methods. That guidance will tell the Director the RA's >>>preferred course of action within his discretion to apply >>>Laws 40B4, 40B5, and 40B6(b), 12A2 and, of course, Laws >>>12A1, 90 and 91. I invited you to examine what the EBU says >>>- it is likely to adopt a less harsh attitude than some - >>>but if you are not inclined to do that I see no reason to >>>discuss the subject any further. >>> ~ Grattan ~ +=+ > > Herman De Wael asked: > >>Does anyone else than me consider this a non-answer? > > Abraham Lincoln asked: > > "If I were two-faced, would I be wearing this one?" > > Herman De Wael: > >>What do we do? Cancel the board or award an AS based on the >>opponents being fully informed about the possibilities of the >>deviation? > > Richard Hills: > > Neither, a false dichotomy. > > Under both the EBU White Book and the International Laws a > cancellation of the board due to what is colloquially known as > an "illegal convention" would be a Director's Error. > > Law 40B5: > > "When a side is damaged by an opponent's use of a special > partnership understanding that does not comply with the > regulations governing the tournament the score **shall** be > adjusted. A side in breach of those regulations may be subject > to a procedural penalty." > > Herman De Wael: > >>I still don't know. >> >>Well, I know what I would do, but I don't even know if I'm >>being considered a crazy Belgian about it or not! > > Richard Hills; > > I am not in England, but this crazy Aussie does have reason to > read the EBU White Book, and also to read the EBU Laws and > Ethics Committee minutes. > > Although these documents are not legally binding in God's own > country, from sea to shining sea, they are useful examples of > what another Regulating Authority does, and the reasoning that > the RA uses. > > The English RA guidance on a Law 40B5 "illegal convention" is > that the non-offending side deserves rectification of damage > via an artificial adjusted score of Ave+ (but of course the > NOS keeps its table score if better than Ave+, since the EBU > rules that then the NOS has not been damaged). The offending > side gets Ave- or its table score, whichever is worse. No > procedural penalty for a revealed illegal convention, but a > standard 10% of a top PP for any concealed partnership > understanding (illegal convention or otherwise) -- which has > the colourful misnomer in EBU regulation of "Red Psyche". > > In the most recent EBU L&EC minutes, the EBU justifies its > decision to award ArtAS (rather than AssAS) for Law 40B5 > "illegal convention" rulings under > > Law 12C1(d): > > "If the possibilities are numerous or not obvious, the > Director may award an artificial adjusted score", > > so therefore the point is that a legal auction would not > involve the "illegal convention" being _explained_, but rather > the "illegal convention" not ever being _used_. Thus an > alternative hypothetically legal auction would involve the > assumption that the offending side instead used one of several > zillion different legal methods, hence the possibilities are > just a tad numerous. > > What's the problem? > +=+ The problem I had was that Herman took my statement and broke it up into segments and then demanded a detailed exposition of each. He did not take my statement as a rounded whole and then examine for himself the use of the powers that the laws give to the Director to deal with such a case, noting the EBU example illustrating how within the laws the RA may issue guidelines to its directors. I was not prepared to have the daunting task of educating Herman thrust upon me when he was not prepared to do any homework on the subject. Richard Hills has now done the homework for him and he should be eternally grateful. Perhaps he will confound my expectations by exhibiting just such gratitude. ~ Grattan ~ +=+ From jfusselman at gmail.com Wed Nov 5 09:38:46 2008 From: jfusselman at gmail.com (Jerry Fusselman) Date: Wed, 5 Nov 2008 02:38:46 -0600 Subject: [blml] Mr Lincoln suggested [SEC=UNOFFICIAL] In-Reply-To: References: <49105C0E.7050003@skynet.be> Message-ID: <2b1e598b0811050038o4b8534b3t3f50f7fa4454493f@mail.gmail.com> Richard Hills started with one of his standard strings of gratuitous insults, then wrote: > > Law 40B5: > > "When a side is damaged by an opponent's use of a special > partnership understanding that does not comply with the > regulations governing the tournament the score **shall** be > adjusted. But this is vague about the adjustment amount and criteria. See below. > > The English RA guidance on a Law 40B5 "illegal convention" is > that the non-offending side deserves rectification of damage > via an artificial adjusted score of Ave+ (but of course the > NOS keeps its table score if better than Ave+, since the EBU > rules that then the NOS has not been damaged). The offending > side gets Ave- or its table score, whichever is worse. No > procedural penalty for a revealed illegal convention, but a > standard 10% of a top PP for any concealed partnership > understanding (illegal convention or otherwise) -- which has > the colourful misnomer in EBU regulation of "Red Psyche". > This seems quite incomplete to me. For example, how is damage defined? Is it automatic that you get Ave+ at least---or do you need to prove damage? Damage might include: unpreparedness; superiority of the disallowed methods; confusion from the disallowed methods; actions that could be considered absurd but are forgiven under the circumstances. And does damage to the NOS go away if the NOS did something absurd before or after the illegal convention came up? Also, if one part of the method is disallowed, but the other, related part that is not illegal comes up, is it an illegal method, or not? For example, suppose 1H-pass-1NT showing spades is legal, but 1H-pass-1S artificial forcing one round, is not legal. (I.e., they switched the 1S and 1NT forcing responses.) If 1H-pass-1S happens, it is an illegal convention, obviously. But what if 1H-pass-1NT happens? It is legal, but a pair might not be prepared for it, because it is not part of any useful legal system. Is damage possible deemed possible (or certain) after 1H-pass-1NT, or not? These questions seem to me unanswered by Richard and Grattan. Jerry Fusselman From grandaeval at tiscali.co.uk Wed Nov 5 10:44:22 2008 From: grandaeval at tiscali.co.uk (Grattan) Date: Wed, 5 Nov 2008 09:44:22 -0000 Subject: [blml] Mr Lincoln suggested [SEC=UNOFFICIAL] References: <49105C0E.7050003@skynet.be> <2b1e598b0811050038o4b8534b3t3f50f7fa4454493f@mail.gmail.com> Message-ID: <000601c93f2b$16e9e260$0302a8c0@Mildred> Grattan Endicott To: "Bridge Laws Mailing List" Sent: Wednesday, November 05, 2008 8:38 AM Subject: Re: [blml] Mr Lincoln suggested [SEC=UNOFFICIAL] > > For example, how is damage defined? > +=+ Damage is defined in Law 12B1. +=+ From Hermandw at skynet.be Wed Nov 5 11:12:25 2008 From: Hermandw at skynet.be (Herman De Wael) Date: Wed, 05 Nov 2008 11:12:25 +0100 Subject: [blml] Mr Burn suggested In-Reply-To: <4910DEF9.8070602@talktalk.net> References: <2a1c3a560810201658h1877df31n5886171a8165db7c@mail.gmail.com> <2a1c3a560810201952j563d60f9ua2096c951f2885b8@mail.gmail.com> <48FDDBFF.7070300@connecttime.net> <002401c93dab$7960f8b0$0302a8c0@Mildred> <002d01c93e1c$28b1ff10$0302a8c0@Mildred> <2a1c3a560811031811p4e0597e3s5c30da7a72dba451@mail.gmail.com> <001301c93e4e$4a819fb0$0302a8c0@Mildred> <2a1c3a560811040211we3724e9r5666f5193166f293@mail.gmail.com> <49104EA6.1060200@talktalk.net> <49105DFF.4090106@skynet.be> <4910DEF9.8070602@talktalk.net> Message-ID: <49117189.7020209@skynet.be> Nigel Guthrie wrote: > [Nige1] > To appreciate that hand *frequency* is irrelevant, consider another > example. Suppose that you open 2C > - whenever you hold 5-10 HCP and 6 diamonds; > - and also when you hold 20+HCP and 4441 shape. > Hands that satisfy the latter requirement are infrequent But you must > still disclose that possibility. > > [Herman] > Of course. But don't you see the difference between the two cases? In one, I have > no method for describing my hand afterwards - in the other, I'm bound to > have some agreement with partner for showing either of the possible > holdings. > > [Nige2] > IMO, this difference is spurious. > - My conventional bid shows a 4441 20+HCP hand or a weak 2 in diamonds. Hence I *need* methods to describe my hand further. > - Herman's conventional bid shows 0-3 HCP or a normal opener. If he holds the former, then he need not refine that messsage opposite his passed partner. Nevertheless, opponents' actions and Herman's eloquent passes may help his partner to distinguish which of the two conventional hand-types he holds. > I agree, but only to the extent that it is up to the director to determine that I don't in effect have a systemic way of dealing with the deviation. I maintain that there is a difference between a deviation and a two-way bid, and that one should not call just any psyche systemic because it could be considered as a two-way bid. If you does that, you end up with just banning psyche altogether, since every single psyche can be considered as a two-way bid: it describes either the systemic meaning or the particular 13 cards precisely held by the bidder. Herman. From Hermandw at skynet.be Wed Nov 5 11:19:12 2008 From: Hermandw at skynet.be (Herman De Wael) Date: Wed, 05 Nov 2008 11:19:12 +0100 Subject: [blml] Mr Lincoln suggested [SEC=UNOFFICIAL] In-Reply-To: References: Message-ID: <49117320.1020105@skynet.be> richard.hills at immi.gov.au wrote: > Abraham Lincoln, March 17th 1865: > > "Whenever I hear anyone arguing for slavery, I feel a strong > impulse to see it tried on him personally." > > Herman De Wael asked: > > [big snip] > >> Or what do you think the penalty for a "concealed implicit >> mutual partnership understanding" ought to be - disbarment? > > Law 40C3(b): > > "Repeated violations of requirements to disclose partnership > understandings may be penalized." > > Richard Hills: > > Under the language of the new Lawbook, a "penalty" refers > only to a Law 90 procedural penalty and/or a Law 91 > disciplinary penalty. So if a Director is using Law 40C3(b) > as the basis for her ruling, she has the discretion to > consequently apply either Law 90 and/or Law 91, depending > upon her judgement of the gravity of the Law 40C3(b) > infraction and/or any mitigating circumstances. > > The Director may judge that if the offending player chooses a > Lincolnesque call for the TD against his own interests (when > the non-offending side is unaware of the infraction), then > that is very much a mitigating circumstance. > > What's the problem? > The problem is that you too, Richard, refuses to give me a clear answer. I did not want to spell out the two possibilities that I see, for fear of missing a third, or of pre-empting your responses, but I see two ways in which a ruling can be made on the infraction that we both recognise exists, even if we call it by different names: A player makes (what he believes to be) a psyche. The TD establishes that the partner had more reason to expect this than did the opponents. (infraction of L40C1). Furthermore, the TD establishes that the opponents are damaged. What is his ruling? a) an AS according to L12C1a or c or d (or e in the ACBL), based on what the opponents would have done differently if they had received the information that the partner apparently had; or: b) a AAS based on A-/A+, ruling that the call should not have been made in the first place. Which one is it? > > Best wishes > > Richard James Hills > Recruitment Section, Level 3 Blue, workstation 15 (first on left) > Department of Immigration and Citizenship > Telephone: 02 6223 8453 > Email: richard.hills at immi.gov.au > > > > -------------------------------------------------------------------- > Important Notice: If you have received this email by mistake, please advise the sender and delete the message and attachments immediately. This email, including attachments, may contain confidential, sensitive, legally privileged and/or copyright information. Any review, retransmission, dissemination or other use of this information by persons or entities other than the intended recipient is prohibited. DIAC respects your privacy and has obligations under the Privacy Act 1988. The official departmental privacy policy can be viewed on the department's website at www.immi.gov.au. See: http://www.immi.gov.au/functional/privacy.htm > > --------------------------------------------------------------------- > > > _______________________________________________ > blml mailing list > blml at amsterdamned.org > http://www.amsterdamned.org/mailman/listinfo/blml > From Hermandw at skynet.be Wed Nov 5 11:21:24 2008 From: Hermandw at skynet.be (Herman De Wael) Date: Wed, 05 Nov 2008 11:21:24 +0100 Subject: [blml] Mr Lincoln suggested [SEC=UNOFFICIAL] In-Reply-To: References: Message-ID: <491173A4.4050702@skynet.be> richard.hills at immi.gov.au wrote: > Abraham Lincoln (1809-1865): > > "He can compress the most words into the smallest idea of any > man I ever met." > > Herman De Wael: > >>>> No Grattan, I am not in England, I have no reason to read >>>> the EBU White Book. International Laws please. > > Grattan Endicott: > >>> +=+ Herman, >>> You have my answer. If you want to find out more >>> you will have to do your own research on the guidance given >>> by RAs on the Director's treatment of use of illegal >>> methods. That guidance will tell the Director the RA's >>> preferred course of action within his discretion to apply >>> Laws 40B4, 40B5, and 40B6(b), 12A2 and, of course, Laws >>> 12A1, 90 and 91. I invited you to examine what the EBU says >>> - it is likely to adopt a less harsh attitude than some - >>> but if you are not inclined to do that I see no reason to >>> discuss the subject any further. >>> ~ Grattan ~ +=+ > > Herman De Wael asked: > >> Does anyone else than me consider this a non-answer? > > Abraham Lincoln asked: > > "If I were two-faced, would I be wearing this one?" > > Herman De Wael: > >> What do we do? Cancel the board or award an AS based on the >> opponents being fully informed about the possibilities of the >> deviation? > > Richard Hills: > > Neither, a false dichotomy. > > Under both the EBU White Book and the International Laws a > cancellation of the board due to what is colloquially known as > an "illegal convention" would be a Director's Error. > > Law 40B5: > > "When a side is damaged by an opponent's use of a special > partnership understanding that does not comply with the > regulations governing the tournament the score **shall** be > adjusted. A side in breach of those regulations may be subject > to a procedural penalty." > > Herman De Wael: > >> I still don't know. >> >> Well, I know what I would do, but I don't even know if I'm >> being considered a crazy Belgian about it or not! > > Richard Hills; > > I am not in England, but this crazy Aussie does have reason to > read the EBU White Book, and also to read the EBU Laws and > Ethics Committee minutes. > > Although these documents are not legally binding in God's own > country, from sea to shining sea, they are useful examples of > what another Regulating Authority does, and the reasoning that > the RA uses. > > The English RA guidance on a Law 40B5 "illegal convention" is > that the non-offending side deserves rectification of damage > via an artificial adjusted score of Ave+ (but of course the > NOS keeps its table score if better than Ave+, since the EBU > rules that then the NOS has not been damaged). The offending > side gets Ave- or its table score, whichever is worse. No > procedural penalty for a revealed illegal convention, but a > standard 10% of a top PP for any concealed partnership > understanding (illegal convention or otherwise) -- which has > the colourful misnomer in EBU regulation of "Red Psyche". > > In the most recent EBU L&EC minutes, the EBU justifies its > decision to award ArtAS (rather than AssAS) for Law 40B5 > "illegal convention" rulings under > > Law 12C1(d): > > "If the possibilities are numerous or not obvious, the > Director may award an artificial adjusted score", > > so therefore the point is that a legal auction would not > involve the "illegal convention" being _explained_, but rather > the "illegal convention" not ever being _used_. Thus an > alternative hypothetically legal auction would involve the > assumption that the offending side instead used one of several > zillion different legal methods, hence the possibilities are > just a tad numerous. > > What's the problem? None any more, consider my important question answered. I am glad we are in agreement over this one, Richard. Now do you see why I was talking about ducks? If it quacks like MI, and it is ruled like MI, why not simply call it MI. But that is not important. > > > Best wishes > > Richard James Hills Herman. From Hermandw at skynet.be Wed Nov 5 11:23:48 2008 From: Hermandw at skynet.be (Herman De Wael) Date: Wed, 05 Nov 2008 11:23:48 +0100 Subject: [blml] Mr Lincoln suggested [SEC=UNOFFICIAL] In-Reply-To: <001001c93f21$1d00eb80$0302a8c0@Mildred> References: <001001c93f21$1d00eb80$0302a8c0@Mildred> Message-ID: <49117434.2030208@skynet.be> Grattan wrote: > >> What's the problem? >> > +=+ The problem I had was that Herman took my statement > and broke it up into segments and then demanded a detailed > exposition of each. He did not take my statement as a rounded > whole and then examine for himself the use of the powers that > the laws give to the Director to deal with such a case, noting > the EBU example illustrating how within the laws the RA may > issue guidelines to its directors. I was not prepared to have > the daunting task of educating Herman thrust upon me when > he was not prepared to do any homework on the subject. > Richard Hills has now done the homework for him and > he should be eternally grateful. Perhaps he will confound my > expectations by exhibiting just such gratitude. > ~ Grattan ~ +=+ > I think I have shown my gratitude in my reply to Richard. Forgive me for asking things that appear obvious, but I have so often in the past believed in obvious things that turn out to be completely obvious to every one else - in a different manner. I am glad that we are in agreement over this one. Herman. From grandaeval at tiscali.co.uk Wed Nov 5 12:32:19 2008 From: grandaeval at tiscali.co.uk (Grattan) Date: Wed, 5 Nov 2008 11:32:19 -0000 Subject: [blml] Fw: Mr Lincoln suggested [SEC=UNOFFICIAL] Message-ID: <000601c93f3a$89c40db0$0302a8c0@Mildred> Grattan Endicott A player makes (what he believes to be) a psyche. The TD establishes > that the partner had more reason to expect this than did the opponents. > (infraction of L40C1). Furthermore, the TD establishes that the > opponents are damaged. What is his ruling? > a) an AS according to L12C1a or c or d (or e in the ACBL), based on what > the opponents would have done differently if they had received the > information that the partner apparently had; or: > b) a AAS based on A-/A+, ruling that the call should not have been made > in the first place. > +=+ Richard, I do not see where you are going here. The illegal bid is disallowed, so no result can be based on events that involve the use of it. The only available course is (b) - with, invariably I would believe in a matter of this gravity, also the imposition of a penalty for the violation of law. ~ Grattan ~ +=+ From agot at ulb.ac.be Wed Nov 5 12:58:32 2008 From: agot at ulb.ac.be (Alain Gottcheiner) Date: Wed, 05 Nov 2008 12:58:32 +0100 Subject: [blml] Fw: Mr Lincoln suggested [SEC=UNOFFICIAL] In-Reply-To: <000601c93f3a$89c40db0$0302a8c0@Mildred> References: <000601c93f3a$89c40db0$0302a8c0@Mildred> Message-ID: <49118A68.30602@ulb.ac.be> Grattan a ?crit : > > >> A player makes (what he believes to be) a psyche. The TD establishes >> that the partner had more reason to expect this than did the opponents. >> (infraction of L40C1). Furthermore, the TD establishes that the >> opponents are damaged. What is his ruling? >> a) an AS according to L12C1a or c or d (or e in the ACBL), based on what >> the opponents would have done differently if they had received the >> information that the partner apparently had; or: >> b) a AAS based on A-/A+, ruling that the call should not have been made >> in the first place. >> >> > +=+ Richard, > > I do not see where you are going here. The illegal bid is > disallowed, so no result can be based on events that involve the use of it. > The only available course is (b) - with, invariably I would believe > in a matter of this gravity, also the imposition of a penalty for the violation > of law. > ~ Grattan ~ +=+ > > > AG : in circumstances when L/18 doesn't rear its baroque head, which part of TFL allows us to decide there was CPU (deliberately refusing to mention an agreement), rather than MI (omitting to mention an agreement) ? The difference is one of intent, and that's always difficult to ascertain. Of course, there might be an unconscious agreement (this has been discussed before), but C (for concealing) needs a decision to act. To the contrary, incomplete disclosure is nearly always unintended. 'Oh, I forgot to tell them, indeed, partner occasionally psyched this resopnse' Do you rule the player deliberately hid this information ? (provided there was no fielding it, of course) Some don't even know they have to disclose this, whence surely they didn't conceal it (i.e. intentionally refused to mention it). They infracted, to be sure, and they one should decide on MI grounds, and perhaps give a PP, but CPU it can't be. Best regards Alain From Hermandw at skynet.be Wed Nov 5 13:29:01 2008 From: Hermandw at skynet.be (Herman De Wael) Date: Wed, 05 Nov 2008 13:29:01 +0100 Subject: [blml] Fw: Mr Lincoln suggested [SEC=UNOFFICIAL] In-Reply-To: <000601c93f3a$89c40db0$0302a8c0@Mildred> References: <000601c93f3a$89c40db0$0302a8c0@Mildred> Message-ID: <4911918D.6030309@skynet.be> Grattan wrote: > > Grattan Endicott also ************************************** > "Let us then be up and doing, > With a heart for any fate, > Still achieving, still pursuing, > Learn to labour and to wait." > [H.W. Longfellow] > > ''''''''''''''''''''''''''''''''''''''''''''''''''''''''''''''''''''''''''''''''''''''''''''''''' > > >> A player makes (what he believes to be) a psyche. The TD establishes >> that the partner had more reason to expect this than did the opponents. >> (infraction of L40C1). Furthermore, the TD establishes that the >> opponents are damaged. What is his ruling? >> a) an AS according to L12C1a or c or d (or e in the ACBL), based on what >> the opponents would have done differently if they had received the >> information that the partner apparently had; or: >> b) a AAS based on A-/A+, ruling that the call should not have been made >> in the first place. >> > +=+ Richard, No, Herman. > I do not see where you are going here. The illegal bid is > disallowed, > so no result can be based on events that involve the use of it. > The only available course is (b) - with, invariably I would believe > in > a matter of this gravity, also the imposition of a penalty for the violation > of > law. > ~ Grattan ~ +=+ > See? Merely 1 hour after I said I was glad we agreed - Grattan tells me we actually disagree. OK Grattan, now tell me this: Helgemo opens 1He. on 8 points. We forget for the moment the fact that this is a HUM, but we do find that on his SC it says "10-19". We also find that his partner expects only 8, and that the opponents have not been informed by this. Do we: a) rule misinformation, with an AS; or b) rule illegal deviation, with an AAS. How do we decide, when in fact both L40C and L40A could be said to apply? As always, Grattan has not thought everything through to its logical conclusions. I doubt if he is alone in that, as well. Herman. From rfrick at rfrick.info Wed Nov 5 14:26:03 2008 From: rfrick at rfrick.info (Robert Frick) Date: Wed, 05 Nov 2008 08:26:03 -0500 Subject: [blml] New Laws don't allow UI situation to be resolved? In-Reply-To: <490D98CC.6040908@talktalk.net> References: <2b1e598b0810302300t4f888261j8d7615f57fcdd4ee@mail.gmail.com> <000301c93b33$398b9670$aca2c350$@no> <001b01c93b5d$4438f2f0$ccaad8d0$@no> <000001c93c08$ea3dbf90$beb93eb0$@no> <000101c93c49$99440220$cbcc0660$@no> <001a01c93c5b$6d020830$0302a8c0@Mildred> <490CCA60.5060607@meadows.pair.com> <000801c93cb6$c082c5c0$0302a8c0@Mildred> <490D6BA7.30607@skynet.be> <002c01c93ce1$221094f0$0302a8c0@Mildred> <490D98CC.6040908@talktalk.net> Message-ID: I am trying to separate the question of what a player or director should do when a player receives a snippet of information, from what the laws currently require. I have the following. Is this right? 1. The player is required to report any snippet of information as soon as it is received. Reporting all snippets of information seems to be completely impractical at the club level, and probably any non-major tournament. So I assume that this law must be infracted, with unspoken approval from the director. 2. Once this law is infracted, L16C1 and L16C3 don't apply any more, giving the player considerable freedom. For example, the player apparently is free to either try to ignore the information or to process the information (think about meaning, find out which board is involved). 3. If the information seems at a later time to be relevant, L16A3 now applies -- the player cannot base his call or play on the UI. But this will often be impossible, psychologically (because people essentially do not have the ability to ignore information they possess). 4. When faced with this situation, the player may notify the director. But he need not (L72B2). Is this true? 5. The director will apply L16C2(c). But is there any legal way to get to this? (The initial conditions of L16C2 and L16C3 are not met.) I had the idea that maybe the director could rectify for the initial failure to report the reception of information (L84A and Law82B1?). Then, if the initial reception of information had been reported, L16C2(c) would now be in force, so maybe L16C2 would be an appropriate rectification? From richard.hills at immi.gov.au Wed Nov 5 23:18:40 2008 From: richard.hills at immi.gov.au (richard.hills at immi.gov.au) Date: Thu, 6 Nov 2008 09:18:40 +1100 Subject: [blml] Netiquette (was Mr Lincoln) [SEC=UNOFFICIAL] In-Reply-To: <2b1e598b0811050038o4b8534b3t3f50f7fa4454493f@mail.gmail.com> Message-ID: Abraham Lincoln (1809-1865): "He can compress the most words into the smallest idea of any man I ever met." Jerry Fusselman asserted: >Richard Hills started with one of his standard strings of >gratuitous insults, then wrote: [big snip] Richard Hills quibbles: An American presidential campaign slogan 60 years ago was: "To err is Truman." So occasionally I err, overstepping the mark from bantering to gratuitous insult - although not as stylish as the very witty insults from David Burn: >>Disraeli called Gladstone a "sophisticated rhetorician >>intoxicated with the exuberance of his own verbosity". This >>was a bit unfair on Gladstone, but Richard wasn't alive at >>the time, so Disraeli chose the only target he could. Richard Hills quibbles: I was not offended by David Burn's gratuitous insult, since not only did it inspire me to start the "Mr Gladstone" thread, I also managed to insult David Burn back in the best possible way - an insult so obscure that DALB did not know he was being insulted when he was compared to the late and great Australian Prime Minister, William McMahon. And in the same posting of mine Jerry took umbrage with, Herman De Wael and I were exchanging self-deprecating insults, he referring to himself as a crazy Belgian, and me reciprocating by referring to myself as a crazy Aussie. So is Jerry being over-precious, more Catholic than the Pope? I conclude by repeating advice on blml netiquette, originally posted on June 17th 2004: As some day it must happen that a victim must be found, I've got a little list - I've got a little list Of blml offenders who might well be under ground And who never would be missed - who never would be missed! There's the pestilential nuisances who write large monographs, Those posters who write flabby text in endless paragraphs - All pedants who obscurify, and "autochthon" you flat - All persons who in quoting you, misquote your posts like *that* - And all rude writers who on four-letter words insist - They'd none of 'em be missed - they'd none of 'em be missed! CHORUS. He's got 'em on the list - he's got 'em on the list; And they'll none of 'em be missed - they'll none of 'em be missed. There's the threading serenader, and the others of their race, And the pre-prandialist - I've got them on the list! And the people posting polemics, who flame it in your face, They never would be missed - they never would be missed! Then the idiot who praises, with enthusiastic tone, All likely Laws but ours, and every country but his own; Plus the weirdo from the provinces, who writes rubbish as a crank, And who doesn't read the Lawbook, but still RAs will spank; And that singular anomaly, the pseudo-humorist - I don't think they'd be missed - I'm sure they'd not be missed! Best pseudo-witty wishes Richard James Hills, pre-prandialist and threading serenader -------------------------------------------------------------------- Important Notice: If you have received this email by mistake, please advise the sender and delete the message and attachments immediately. This email, including attachments, may contain confidential, sensitive, legally privileged and/or copyright information. Any review, retransmission, dissemination or other use of this information by persons or entities other than the intended recipient is prohibited. DIAC respects your privacy and has obligations under the Privacy Act 1988. The official departmental privacy policy can be viewed on the department's website at www.immi.gov.au. See: http://www.immi.gov.au/functional/privacy.htm --------------------------------------------------------------------- From grandaeval at tiscali.co.uk Thu Nov 6 00:03:55 2008 From: grandaeval at tiscali.co.uk (Grattan) Date: Wed, 5 Nov 2008 23:03:55 -0000 Subject: [blml] one short Message-ID: <000801c93f9a$cde489b0$0302a8c0@Mildred> Grattan Endicott Message-ID: <200811052322.PAA15009@mailhub.irvine.com> Grattan wrote: > Help! I have lost an electoral college vote! > Obama 349 called > McCain 162 called > Dow 486 down > Missouri 11 not yet called > North Carolina 15 not yet called > So where is the 538th ? > ~ Grattan ~ [All right, the Dow was my addition...] You didn't lose the vote, your source did. CNN has 349 and 163 ( http://www.cnn.com/ELECTION/2008/results/president/ ). -- Adam From dalburn at btopenworld.com Thu Nov 6 00:53:08 2008 From: dalburn at btopenworld.com (David Burn) Date: Wed, 5 Nov 2008 23:53:08 -0000 Subject: [blml] one short In-Reply-To: <000801c93f9a$cde489b0$0302a8c0@Mildred> References: <000801c93f9a$cde489b0$0302a8c0@Mildred> Message-ID: <001501c93fa1$a7f75aa0$f7e60fe0$@com> [GE] So where is the 538th? [DALB] A victim of cuts at the BBC, whose election map shows McCain with 162 votes, but if you add up the number of votes in the states he has won, it comes to 163. Perhaps he was fined one vote by the Director General for psyching during the campaign. David Burn London, England From richard.hills at immi.gov.au Thu Nov 6 00:54:09 2008 From: richard.hills at immi.gov.au (richard.hills at immi.gov.au) Date: Thu, 6 Nov 2008 10:54:09 +1100 Subject: [blml] one short [SEC=UNOFFICIAL] In-Reply-To: <000801c93f9a$cde489b0$0302a8c0@Mildred> Message-ID: Help! I have lost an electoral college vote! Obama 349 called McCain 162 called Missouri 11 not yet called North Carolina 15 not yet called So where is the 538th ? ~ Grattan ~ Winner-take-all of a state's electoral college votes is not a constitutional requirement, but merely a practice which developed over the years (similar to celibacy of Roman Catholic clergy not being a matter of dogma, but rather an administrative arrangement, originally adopted in the Middle Ages to prevent church lands from being ceded to children of clergy). In the nineteenth century the larger states adopted winner-take-all to increase their influence in affecting the outcome of presidential elections and thus gain greater access to pork-barrelling promises from the rival candidates. The smaller states then perforce followed suit in adopting winner-take-all, to maximise what little influence they had. But recently the states of Maine and Nebraska opted for a more democratic arrangement. Only two of their electoral college votes went to the state-wide winner, with their remaining electoral college votes being allocated by congressional district. Nebraska has been Republican heartland since 1964, so John McCain easily won the two state-wide electoral college votes, plus two votes allocated to two of its three congressional districts. But the third district is somewhat trendy and left-wing, so the electoral college vote for that Nebraska district is still undecided. What's the problem? The problem for John McCain was that Barack Obama left nothing to chance in the campaign, as Obama chose to open a campaign office in uber-Republican Nebraska to chase this one short vote. Comparable to my partner, holding 42 of trumps in dummy, choosing to ruff with the four - and, sure enough, RHO wanted to over-ruff but held the singleton trey of trumps. Best wishes Richard James Hills Recruitment Section, Level 3 Blue, workstation 15 (first on left) Department of Immigration and Citizenship Telephone: 02 6223 8453 Email: richard.hills at immi.gov.au -------------------------------------------------------------------- Important Notice: If you have received this email by mistake, please advise the sender and delete the message and attachments immediately. This email, including attachments, may contain confidential, sensitive, legally privileged and/or copyright information. Any review, retransmission, dissemination or other use of this information by persons or entities other than the intended recipient is prohibited. DIAC respects your privacy and has obligations under the Privacy Act 1988. The official departmental privacy policy can be viewed on the department's website at www.immi.gov.au. See: http://www.immi.gov.au/functional/privacy.htm --------------------------------------------------------------------- From james.boyce at oracle.com Thu Nov 6 00:57:18 2008 From: james.boyce at oracle.com (James Boyce) Date: Wed, 05 Nov 2008 15:57:18 -0800 Subject: [blml] one short In-Reply-To: <000801c93f9a$cde489b0$0302a8c0@Mildred> References: <000801c93f9a$cde489b0$0302a8c0@Mildred> Message-ID: <491232DE.8070401@oracle.com> Grattan wrote: > Grattan Endicott also ************************************** > "Let us then be up and doing, > With a heart for any fate, > Still achieving, still pursuing, > Learn to labour and to wait." > [H.W. Longfellow] > > ''''''''''''''''''''''''''''''''''''''''''''''''''''''''''''''''''''''''''''''''''''''''''''''''' > Help! I have lost an electoral college vote! > Obama 349 called > McCain 162 called > Missouri 11 not yet called > North Carolina 15 not yet called > So where is the 538th ? > ~ Grattan ~ > > > > _______________________________________________ > blml mailing list > blml at amsterdamned.org > http://www.amsterdamned.org/mailman/listinfo/blml > The odd vote is likely to be the vote from the 2nd district in Nebraska. (Two states, Nebraska and Maine, are not "winner-take-all".) It is possible that some sources have called that district for McCain. It is also possible that some sources are treating the state incorrectly. From geller at nifty.com Thu Nov 6 00:58:26 2008 From: geller at nifty.com (Robert Geller) Date: Thu, 06 Nov 2008 08:58:26 +0900 Subject: [blml] one short In-Reply-To: <000801c93f9a$cde489b0$0302a8c0@Mildred> References: <000801c93f9a$cde489b0$0302a8c0@Mildred> Message-ID: <200811052358.AA16400@geller204.nifty.com> Nebraska is the only state that doesn't follow winner-take-all. Two electors are awarded to the overall victor, and one to the winner of each congressional district. I haven't been following the details really closely, but I recall reading that the second congressional district of Nebraska is still too close to call. -Bob Grattan ????????: > > >Grattan Endicottalso ************************************** >"Let us then be up and doing, >With a heart for any fate, >Still achieving, still pursuing, >Learn to labour and to wait." > [H.W. Longfellow] > >''''''''''''''''''''''''''''''''''''''''''''''''''''''''''''''''' '''''''''''''''''''''''''''''''' >Help! I have lost an electoral college vote! >Obama 349 called >McCain 162 called >Missouri 11 not yet called >North Carolina 15 not yet called >So where is the 538th ? > ~ Grattan ~ > > > >_______________________________________________ >blml mailing list >blml at amsterdamned.org >http://www.amsterdamned.org/mailman/listinfo/blml ----------------------------------------------------- Robert (Bob) Geller, Tokyo, Japan geller at nifty.com From PeterEidt at t-online.de Thu Nov 6 01:08:29 2008 From: PeterEidt at t-online.de (Peter Eidt) Date: Thu, 06 Nov 2008 01:08:29 +0100 Subject: [blml] =?iso-8859-15?q?one_short?= In-Reply-To: <200811052322.PAA15009@mailhub.irvine.com> References: <200811052322.PAA15009@mailhub.irvine.com> Message-ID: <1KxsQH-1vELmC0@fwd04.aul.t-online.de> From: Adam Beneschan > Grattan wrote: > > Help! ?I have lost an electoral college vote! > > Obama 349 called > > McCain 162 called > > Dow 486 down > > Missouri 11 not yet called > > North Carolina 15 not yet called > > So where is the 538th ? > > ? ? ? ? ? ? ? ? ~ Grattan ~ > > [All right, the Dow was my addition...] > > You didn't lose the vote, your source did. ?CNN has 349 and 163 > ( http://www.cnn.com/ELECTION/2008/results/president/ ). Yes, it seems to be an error in addition (but it's the same error in Germany - we also have 349 and 162 (??)) AK (3) + AZ (10) + UT (5) + ID (4) + MT (3) + WY (3) + ND (3) + + SD (3) + NE (5) + KS (6) + OK (7) + TX (34) + LA (9) + AR (6) + + MS (6) + AL (9) + GA (15) + SC (8) + TN (11) + KY (8) + WV (5) adds up to 163. Peter From dalburn at btopenworld.com Thu Nov 6 02:57:54 2008 From: dalburn at btopenworld.com (David Burn) Date: Thu, 6 Nov 2008 01:57:54 -0000 Subject: [blml] One long In-Reply-To: <200811052358.AA16400@geller204.nifty.com> References: <000801c93f9a$cde489b0$0302a8c0@Mildred> <200811052358.AA16400@geller204.nifty.com> Message-ID: <000001c93fb3$160cc000$42264000$@com> [BG] Grattan ???????? [DALB] Crikey. Is that the Japanese for "wrote"? David Burn London, England From geller at nifty.com Thu Nov 6 03:27:29 2008 From: geller at nifty.com (Robert Geller) Date: Thu, 06 Nov 2008 11:27:29 +0900 Subject: [blml] One long In-Reply-To: <000001c93fb3$160cc000$42264000$@com> References: <000001c93fb3$160cc000$42264000$@com> Message-ID: <200811060227.AA16401@geller204.nifty.com> David Burn ????????: >[BG] > >Grattan ???????? > >[DALB] > >Crikey. Is that the Japanese for "wrote"? Yes. > >David Burn >London, England > > >_______________________________________________ >blml mailing list >blml at amsterdamned.org >http://www.amsterdamned.org/mailman/listinfo/blml ----------------------------------------------------- Robert (Bob) Geller, Tokyo, Japan geller at nifty.com From john at asimere.com Thu Nov 6 04:34:47 2008 From: john at asimere.com (John (MadDog) Probst) Date: Thu, 6 Nov 2008 03:34:47 -0000 Subject: [blml] One long References: <000801c93f9a$cde489b0$0302a8c0@Mildred><200811052358.AA16400@geller204.nifty.com> <000001c93fb3$160cc000$42264000$@com> Message-ID: <18F382E9F7524489B90EFEECABFEAA12@JOHN> ----- Original Message ----- From: "David Burn" To: "'Bridge Laws Mailing List'" Sent: Thursday, November 06, 2008 1:57 AM Subject: [blml] One long > [BG] > > Grattan $B$5$s$O=q$-$^$7$?(B well there's an honorific "-san" which is the first two characters after Grattan. Then we get up steam for the verb which starts Ha-something-ki and then we have the wind down ma-shi-ta which sticks it into the past tense AND makes it more formal.; so most of that is just being being polite and it can't really be left out. It is almost impossible to be rude in Japanese as most of the constructions are of the form honorable idiot or stupid wife - you get the drift. cheers John > > [DALB] > > Crikey. Is that the Japanese for "wrote"? > > David Burn > London, England > > > _______________________________________________ > blml mailing list > blml at amsterdamned.org > http://www.amsterdamned.org/mailman/listinfo/blml From ardelm at optusnet.com.au Thu Nov 6 06:01:56 2008 From: ardelm at optusnet.com.au (Tony Musgrove) Date: Thu, 06 Nov 2008 16:01:56 +1100 Subject: [blml] one short In-Reply-To: <000801c93f9a$cde489b0$0302a8c0@Mildred> References: <000801c93f9a$cde489b0$0302a8c0@Mildred> Message-ID: <6.1.0.6.2.20081106160114.01bd5a60@mail.optusnet.com.au> At 10:03 AM 6/11/2008, you wrote: >Grattan Endicottalso ************************************** >"Let us then be up and doing, >With a heart for any fate, >Still achieving, still pursuing, >Learn to labour and to wait." > [H.W. Longfellow] > >''''''''''''''''''''''''''''''''''''''''''''''''''''''''''''''''''''''''''''''''''''''''''''''''' >Help! I have lost an electoral college vote! >Obama 349 called >McCain 162 called >Missouri 11 not yet called >North Carolina 15 not yet called >So where is the 538th ? > ~ Grattan ~ > > > Perhaps if they gave the extra vote to BLML, they might get Bush back in? Cheers, Tony (Sydney) >_______________________________________________ >blml mailing list >blml at amsterdamned.org >http://www.amsterdamned.org/mailman/listinfo/blml From grandaeval at tiscali.co.uk Thu Nov 6 07:48:43 2008 From: grandaeval at tiscali.co.uk (Grattan) Date: Thu, 6 Nov 2008 06:48:43 -0000 Subject: [blml] Fw: one short Message-ID: <001101c93fdb$b7b219a0$0302a8c0@Mildred> Grattan Endicott To: "Jeff Ford" Sent: Thursday, November 06, 2008 6:47 AM Subject: Re: [blml] one short > > > Grattan Endicott also ************************************** > "Let us then be up and doing, > With a heart for any fate, > Still achieving, still pursuing, > Learn to labour and to wait." > [H.W. Longfellow] > > ''''''''''''''''''''''''''''''''''''''''''''''''''''''''''''''''''''''''''''''''''''''''''''''''' > > > > ----- Original Message ----- > From: "Jeff Ford" > To: "Grattan Endicott" > Sent: Wednesday, November 05, 2008 11:23 PM > Subject: Re: [blml] one short > > >> On Wed, Nov 5, 2008 at 3:03 PM, Grattan Endicott >> wrote: >>> Help! I have lost an electoral college vote! >>> Obama 349 called >>> McCain 162 called >>> Missouri 11 not yet called >>> North Carolina 15 not yet called >>> So where is the 538th ? >> >> Nebraska does not award all five electoral votes to the state winner, >> giving two to the state winner, and one to the winner in each of the >> three districts. I believe that while the state is called for McCain, >> one district is still in doubt. >> >> Jeff >> > +=+ Checking on the detail for Nebraska I find only 4 votes called. > Thank you one and all. > ~G ~ +=+ From Hermandw at skynet.be Thu Nov 6 09:32:28 2008 From: Hermandw at skynet.be (Herman De Wael) Date: Thu, 06 Nov 2008 09:32:28 +0100 Subject: [blml] only on blml Message-ID: <4912AB9C.3080004@skynet.be> Could one find Englishmen having a discussion about a single vote in an American election won by almost two-to-one leading into the intricacies of Japanese grammar! I love it! Herman. From grandaeval at tiscali.co.uk Thu Nov 6 12:54:06 2008 From: grandaeval at tiscali.co.uk (Grattan) Date: Thu, 6 Nov 2008 11:54:06 -0000 Subject: [blml] one short References: <000801c93f9a$cde489b0$0302a8c0@Mildred> <200811052358.AA16400@geller204.nifty.com> Message-ID: <002001c94006$f2a857b0$0302a8c0@Mildred> Grattan Endicott To: "Bridge Laws Mailing List" Sent: Wednesday, November 05, 2008 11:58 PM Subject: Re: [blml] one short > Nebraska is the only state that doesn't follow winner-take-all. > Two electors are awarded to the overall victor, and one to the > winner of each congressional district. I haven't been following > the details really closely, but I recall reading that the second > congressional district of Nebraska is still too close to call. > -Bob >> +=+ As an electoral method this has an improved feel by comparison with the winner-takes-all grab in the great majority of states. I wonder what the effect would be if it applied throughout. I note someone says it is done this way also in Maine. ~ Grattan ~ +=+ From grandaeval at tiscali.co.uk Thu Nov 6 12:58:03 2008 From: grandaeval at tiscali.co.uk (Grattan) Date: Thu, 6 Nov 2008 11:58:03 -0000 Subject: [blml] only on blml References: <4912AB9C.3080004@skynet.be> Message-ID: <002101c94006$f2e8bc10$0302a8c0@Mildred> Grattan Endicott To: "Bridge Laws Mailing List" Sent: Thursday, November 06, 2008 8:32 AM Subject: [blml] only on blml Could one find Englishmen having a discussion about a single vote in an American election won by almost two- to-one leading into the intricacies of Japanese grammar! > I love it! > Herman. > +=+ Not so droll really. It helps the blood pressure by comparison with the usual trade on here. ~ G ~ +=+ From agot at ulb.ac.be Thu Nov 6 13:47:49 2008 From: agot at ulb.ac.be (Alain Gottcheiner) Date: Thu, 06 Nov 2008 13:47:49 +0100 Subject: [blml] only on blml In-Reply-To: <4912AB9C.3080004@skynet.be> References: <4912AB9C.3080004@skynet.be> Message-ID: <4912E775.1050500@ulb.ac.be> Herman De Wael a ?crit : > Could one find Englishmen having a discussion about a single vote in an > American election won by almost two-to-one leading into the intricacies > of Japanese grammar! > If you like linguistical intricacies, may I add that the form 'you wrote', followed by the complete former message, isn't a creation of e-mail. In the Assyrian Empire, the custom was to reply to a letter by writing (er, carving) either av? ispuram (my father wrote) for your superior ah? ispuram (my brother wrote) for your equal followed by the meaasge you recieved, then only your response. Apparently, it was considered a sign of respect to take a while to cut umpteen symbols that weren't needed ; or perhaps mail took such a long time that one could have forgotten what one had written ? I occasionnally use those formulae in lieu of the ordinary 'you wrote', but few seem to appreciate it :-P Best regards Alain From Hermandw at skynet.be Thu Nov 6 14:00:06 2008 From: Hermandw at skynet.be (Herman De Wael) Date: Thu, 06 Nov 2008 14:00:06 +0100 Subject: [blml] only on blml In-Reply-To: <002101c94006$f2e8bc10$0302a8c0@Mildred> References: <4912AB9C.3080004@skynet.be> <002101c94006$f2e8bc10$0302a8c0@Mildred> Message-ID: <4912EA56.4040701@skynet.be> Grattan wrote: > > > Could one find Englishmen having a discussion about a > single vote in an American election won by almost two- > to-one leading into the intricacies of Japanese grammar! > I love it! > Herman. > +=+ Not so droll really. It helps the blood pressure by > comparison with the usual trade on here. ~ G ~ +=+ > Meanwhile, I find no reaction to yesterday's revelation that you Grattan have a radically different opinion on a very important issue with regards to Richard and myself. Not as interesting as the news about Nebraska, certainly, but still ... Herman. From Hermandw at skynet.be Thu Nov 6 14:04:48 2008 From: Hermandw at skynet.be (Herman De Wael) Date: Thu, 06 Nov 2008 14:04:48 +0100 Subject: [blml] only on blml In-Reply-To: <4912AB9C.3080004@skynet.be> References: <4912AB9C.3080004@skynet.be> Message-ID: <4912EB70.20009@skynet.be> McCain has wont the Nebraska second district by 569 votes (126303 - 125734) http://www.sos.ne.gov/elec/2008/ElectNight/electoralcollegeresults.pdf I'm sure he'll be very pleased with that. Herman De Wael wrote: > Could one find Englishmen having a discussion about a single vote in an > American election won by almost two-to-one leading into the intricacies > of Japanese grammar! > > I love it! > > Herman. > > _______________________________________________ > blml mailing list > blml at amsterdamned.org > http://www.amsterdamned.org/mailman/listinfo/blml > From Hermandw at skynet.be Thu Nov 6 14:10:14 2008 From: Hermandw at skynet.be (Herman De Wael) Date: Thu, 06 Nov 2008 14:10:14 +0100 Subject: [blml] only on blml In-Reply-To: <4912E775.1050500@ulb.ac.be> References: <4912AB9C.3080004@skynet.be> <4912E775.1050500@ulb.ac.be> Message-ID: <4912ECB6.2020401@skynet.be> Alain Gottcheiner ahi ispuram: > Herman De Wael a ?crit : >> Could one find Englishmen having a discussion about a single vote in an >> American election won by almost two-to-one leading into the intricacies >> of Japanese grammar! >> > If you like linguistical intricacies, may I add that the form 'you > wrote', followed by the complete former message, isn't a creation of e-mail. > > In the Assyrian Empire, the custom was to reply to a letter by writing > (er, carving) either > av? ispuram (my father wrote) for your superior > ah? ispuram (my brother wrote) for your equal > followed by the meaasge you recieved, then only your response. > > Apparently, it was considered a sign of respect to take a while to cut > umpteen symbols that weren't needed ; or perhaps mail took such a long > time that one could have forgotten what one had written ? > > I occasionnally use those formulae in lieu of the ordinary 'you wrote', > but few seem to appreciate it :-P > > Best regards > > Alain > Why did you not on your mail? How can we know what you think of us if you don't use the appropriate form in Assyrian? For that matter, how do we know it if you don't do it in French (writing "a ?crit" in the third person rather than the second, so that we could see the plural/singular. Given the usual level of insults, I think most of us would agree to be tutoyated. Except David, Grattan and Richard of course, who regularly refer to Presidents and Prime Ministers. Only polite plural is acceptable in those instances. Herman. From dalburn at btopenworld.com Thu Nov 6 14:51:33 2008 From: dalburn at btopenworld.com (dalburn at btopenworld.com) Date: Thu, 6 Nov 2008 13:51:33 +0000 Subject: [blml] only on blml Message-ID: <2105780810-1225979535-cardhu_decombobulator_blackberry.rim.net-899727981-@bxe065.bisx.produk.on.blackberry> [HdW] Meanwhile, I find no reaction to yesterday's revelation that you Grattan have a radically different opinion on a very important issue with regards to Richard and myself. [DALB] I have not read a rotten page Of "Sex Hate" or "The Social Test". And here comes "Husks" and "Heritage" - O Moses! Give us all a rest. "Ethics of Empire" - I protest. I will not even cut the strings; I'll read "Jack Redskin on the Quest" And feed my brain with better things. G K Chesterton, Ballade of a Furious Reviewer Sent from my BlackBerry? wireless device From grandaeval at tiscali.co.uk Thu Nov 6 15:58:27 2008 From: grandaeval at tiscali.co.uk (Grattan) Date: Thu, 6 Nov 2008 14:58:27 -0000 Subject: [blml] What did I say? References: <4912AB9C.3080004@skynet.be><002101c94006$f2e8bc10$0302a8c0@Mildred> <4912EA56.4040701@skynet.be> Message-ID: <003601c94020$22ab69c0$0302a8c0@Mildred> Grattan Endicott To: "Bridge Laws Mailing List" Sent: Thursday, November 06, 2008 1:00 PM Subject: Re: [blml] only on blml > Meanwhile, I find no reaction to yesterday's revelation that you Grattan have a radically different opinion on a very important issue with regards to Richard and myself. > Not as interesting as the news about Nebraska, certainly, but still ... > > Herman. < +=+ I must have overlooked it. And now you tell me, just as I have expunged yesterday's and prior contents of my 'blml' folder. Ah well, not all that earth-shattering if no-one got excited. But it is not so unusual for my reactions to be cast in a different mould from those of others who bring forth issue here. ~ Ancient Radical ~ +=+ From grandaeval at tiscali.co.uk Thu Nov 6 16:04:16 2008 From: grandaeval at tiscali.co.uk (Grattan) Date: Thu, 6 Nov 2008 15:04:16 -0000 Subject: [blml] only on blml References: <2105780810-1225979535-cardhu_decombobulator_blackberry.rim.net-899727981-@bxe065.bisx.produk.on.blackberry> Message-ID: <003d01c94020$f9711a40$0302a8c0@Mildred> Grattan Endicott To: "Bridge Laws Mailing List" Sent: Thursday, November 06, 2008 1:51 PM Subject: Re: [blml] only on blml [HdW] Meanwhile, I find no reaction to yesterday's revelation that you Grattan have a radically different opinion on a very important issue with regards to Richard and myself. [DALB] I have not read a rotten page Of "Sex Hate" or "The Social Test". And here comes "Husks" and "Heritage" - O Moses! Give us all a rest. "Ethics of Empire" - I protest. I will not even cut the strings; I'll read "Jack Redskin on the Quest" And feed my brain with better things. G K Chesterton, Ballade of a Furious Reviewer Sent from my BlackBerry? wireless device [GE] +=+ Dear David, I thought of you even as I read those words. But can you imagine what might be "a very important issue with regard to Richard and myself'"- or am I not reading that right? ~ G ~ +=+ From brian at meadows.pair.com Thu Nov 6 16:19:23 2008 From: brian at meadows.pair.com (brian) Date: Thu, 06 Nov 2008 10:19:23 -0500 Subject: [blml] one short In-Reply-To: <002001c94006$f2a857b0$0302a8c0@Mildred> References: <000801c93f9a$cde489b0$0302a8c0@Mildred> <200811052358.AA16400@geller204.nifty.com> <002001c94006$f2a857b0$0302a8c0@Mildred> Message-ID: <49130AFB.7040904@meadows.pair.com> Grattan wrote: > > +=+ As an electoral method this has an improved feel by > comparison with the winner-takes-all grab in the great > majority of states. I wonder what the effect would be if > it applied throughout. I note someone says it is done this > way also in Maine. Maine does divide up its electoral college votes, but I seem to remember them taking a decision to do it that way sometime in the last five years, or thereabouts. That's presumably why Bob wasn't aware of it. I have no idea of what would change if the sharing was applied throughout, I think Obama probably has enough of a majority to be safe from any such changes, but it would be interesting re-work the 2000 election in this fashion. Since Gore (narrowly) won the overall vote for President, you would have to suspect that it could well change the result. Brian. From jfusselman at gmail.com Thu Nov 6 16:28:19 2008 From: jfusselman at gmail.com (Jerry Fusselman) Date: Thu, 6 Nov 2008 09:28:19 -0600 Subject: [blml] only on blml In-Reply-To: <2105780810-1225979535-cardhu_decombobulator_blackberry.rim.net-899727981-@bxe065.bisx.produk.on.blackberry> References: <2105780810-1225979535-cardhu_decombobulator_blackberry.rim.net-899727981-@bxe065.bisx.produk.on.blackberry> Message-ID: <2b1e598b0811060728x5bc1b111j1a9c873da179376d@mail.gmail.com> > [DALB] > > I have not read a rotten page > Of "Sex Hate" or "The Social Test". > And here comes "Husks" and "Heritage" - > O Moses! Give us all a rest. > "Ethics of Empire" - I protest. > I will not even cut the strings; > I'll read "Jack Redskin on the Quest" > And feed my brain with better things. > G K Chesterton, Ballade of a Furious Reviewer > Sent from my BlackBerry(R) wireless device Anyone willing to 'splain any of this to me? If anyone but DALB had written this, I would assume it was complete trash, but paraphrasing Russell Ziskey, I am willing to learn. From agot at ulb.ac.be Thu Nov 6 16:50:12 2008 From: agot at ulb.ac.be (Alain Gottcheiner) Date: Thu, 06 Nov 2008 16:50:12 +0100 Subject: [blml] one short In-Reply-To: <49130AFB.7040904@meadows.pair.com> References: <000801c93f9a$cde489b0$0302a8c0@Mildred> <200811052358.AA16400@geller204.nifty.com> <002001c94006$f2a857b0$0302a8c0@Mildred> <49130AFB.7040904@meadows.pair.com> Message-ID: <49131234.9050602@ulb.ac.be> brian a ?crit : > Grattan wrote: > >> +=+ As an electoral method this has an improved feel by >> comparison with the winner-takes-all grab in the great >> majority of states. I wonder what the effect would be if >> it applied throughout. I note someone says it is done this >> way also in Maine. >> > > Maine does divide up its electoral college votes, but I seem to > remember them taking a decision to do it that way sometime in the last > five years, or thereabouts. That's presumably why Bob wasn't aware of it. > > I have no idea of what would change if the sharing was applied > throughout, I think Obama probably has enough of a majority to be safe > from any such changes, but it would be interesting re-work the 2000 > election in this fashion. Since Gore (narrowly) won the overall vote > for President, you would have to suspect that it could well change the > result. > AG : counting per district tends to minimize small majorities more than big ones. That is, if you get 70% of the votes, districting will usually give you all districts, hence not differ from 'winner-takes-it-all', while it might be that two, or even three, out of five districts be awarded to a candidate getting only 45% of the votes. The main reason for not districting is to avoid gerrymandering. Overall, a big majority can't be overturned by districting, or even by one-districting (i.e. adding all votes), so we're on safer ground than in 2004. Best regards Alain From adam at irvine.com Thu Nov 6 17:13:51 2008 From: adam at irvine.com (Adam Beneschan) Date: Thu, 06 Nov 2008 08:13:51 -0800 Subject: [blml] one short In-Reply-To: Your message of "Thu, 06 Nov 2008 10:19:23 EST." <49130AFB.7040904@meadows.pair.com> Message-ID: <200811061612.IAA23613@mailhub.irvine.com> Brian wrote: > Grattan wrote: > > > > +=+ As an electoral method this has an improved feel by > > comparison with the winner-takes-all grab in the great > > majority of states. I wonder what the effect would be if > > it applied throughout. I note someone says it is done this > > way also in Maine. > > Maine does divide up its electoral college votes, but I seem to > remember them taking a decision to do it that way sometime in the last > five years, or thereabouts. That's presumably why Bob wasn't aware of it. Actually, according to Wikipedia, that method has been in use in Maine since 1972, and in Nebraska since 1992. [look up "Electoral College (United States)"] I actually wasn't aware that Nebraska split its votes like that until this thread came up; I did know about Maine. > I have no idea of what would change if the sharing was applied > throughout, I think Obama probably has enough of a majority to be safe > from any such changes, but it would be interesting re-work the 2000 > election in this fashion. Since Gore (narrowly) won the overall vote > for President, you would have to suspect that it could well change the > result. Unfortunately, if that method had been in use in every state, the likely result would have been that we wouldn't have had a recount battle just in one state, but in every Congressional district where the vote was close. Not fun. -- Adam From rfrick at rfrick.info Thu Nov 6 17:47:07 2008 From: rfrick at rfrick.info (Robert Frick) Date: Thu, 06 Nov 2008 11:47:07 -0500 Subject: [blml] New Laws don't allow UI situation to be resolved? In-Reply-To: References: <2b1e598b0810302300t4f888261j8d7615f57fcdd4ee@mail.gmail.com> <000301c93b33$398b9670$aca2c350$@no> <001b01c93b5d$4438f2f0$ccaad8d0$@no> <000001c93c08$ea3dbf90$beb93eb0$@no> <000101c93c49$99440220$cbcc0660$@no> <001a01c93c5b$6d020830$0302a8c0@Mildred> <490CCA60.5060607@meadows.pair.com> <000801c93cb6$c082c5c0$0302a8c0@Mildred> <490D6BA7.30607@skynet.be> <002c01c93ce1$221094f0$0302a8c0@Mildred> <490D98CC.6040908@talktalk.net> Message-ID: On Wed, 05 Nov 2008 08:26:03 -0500, Robert Frick wrote: > I am trying to separate the question of what a player or director > should do when a player receives a snippet of information, from what the > laws currently require. I have the following. Is this right? > > 1. The player is required to report any snippet of information as soon as > it is received. Reporting all snippets of information seems to be > completely impractical at the club level, and probably any non-major > tournament. So I assume that this law must be infracted, with unspoken > approval from the director. > > 2. Once this law is infracted, L16C1 and L16C3 don't apply any more, > giving the player considerable freedom. For example, the player > apparently > is free to either try to ignore the information or to process the > information (think about meaning, find out which board is involved). > > 3. If the information seems at a later time to be relevant, L16A3 now > applies -- the player cannot base his call or play on the UI. But this > will often be impossible, psychologically (because people essentially do > not have the ability to ignore information they possess). > > 4. When faced with this situation, the player may notify the director. > But > he need not (L72B2). Is this true? > > 5. The director will apply L16C2(c). But is there any legal way to get to > this? (The initial conditions of L16C2 and L16C3 are not met.) I had the > idea that maybe the director could rectify for the initial failure to > report the reception of information (L84A and Law82B1?). Then, if the > initial reception of information had been reported, L16C2(c) would now be > in force, so maybe L16C2 would be an appropriate rectification? There is a question of what ethical players at a club actually do when faced with extraneous information from an outside source before the play of the hand. I am assuming that they make no effort to attend to or process the snippet of information. When it seems large, they notify the director beforehand. (In doing so, they will learn what the director wants to hear about and what the director does not want to hear about.) The director almost surely will direct that the board be played and that the director should be notified if there is a problem. If there is, the director treats both sides as nonoffending. (Or else that player never again reports relevant UI.) To bridge the gap between this practice and the actual law, perhaps it could be said that upon receipt of extraneous information, there is an implicit director call. The director then implicitly says to play the board and call me if there is a problem. That creates an implicit instruction from the director to be notified if the information becomes relevant; both sides must be treated as nonoffending; and the director now is legally at L16C2(c) From Hermandw at skynet.be Thu Nov 6 18:01:59 2008 From: Hermandw at skynet.be (Herman De Wael) Date: Thu, 06 Nov 2008 18:01:59 +0100 Subject: [blml] What did I say? In-Reply-To: <003601c94020$22ab69c0$0302a8c0@Mildred> References: <4912AB9C.3080004@skynet.be><002101c94006$f2e8bc10$0302a8c0@Mildred> <4912EA56.4040701@skynet.be> <003601c94020$22ab69c0$0302a8c0@Mildred> Message-ID: <49132307.2010804@skynet.be> I have gone back to Richard's answer to my question and I must confess I misread it. Having seen one sentence that I agreed with actually made me think I was getting the answer I expected. Not so, apparently. Richard will award, when a deviation occurs and the partner seems to have more reason to expect it than opponents do, a score of A+/A-. He bases this on the guidelines of the EBY, not that he believes they apply in Australia or in any other part of the world except England (and Wales, IIRC). Grattan apparently agrees with this. I disagree (so what's new). Alain has also already pointed out why this approach must be wrong. Is there anyone else who will have a go before I explain in detail why it must be wrong? Herman. From brian at meadows.pair.com Thu Nov 6 20:29:21 2008 From: brian at meadows.pair.com (brian) Date: Thu, 06 Nov 2008 14:29:21 -0500 Subject: [blml] one short In-Reply-To: <200811061612.IAA23613@mailhub.irvine.com> References: <200811061612.IAA23613@mailhub.irvine.com> Message-ID: <49134591.1010504@meadows.pair.com> Adam Beneschan wrote: > Brian wrote: > >> Grattan wrote: >>> +=+ As an electoral method this has an improved feel by >>> comparison with the winner-takes-all grab in the great >>> majority of states. I wonder what the effect would be if >>> it applied throughout. I note someone says it is done this >>> way also in Maine. >> Maine does divide up its electoral college votes, but I seem to >> remember them taking a decision to do it that way sometime in the last >> five years, or thereabouts. That's presumably why Bob wasn't aware of it. > > Actually, according to Wikipedia, that method has been in use in Maine > since 1972, and in Nebraska since 1992. [look up "Electoral College > (United States)"] I actually wasn't aware that Nebraska split its > votes like that until this thread came up; I did know about Maine. > Interesting. I recall an extended discussion with my wife on the merits of the two systems, because this whole electoral college concept is just such a strange idea to me, having been brought up on the British system. Since I didn't move to the USA until 1997, it obviously can't have been the changeover in either state. Now you've got me wondering what it was that DID happen, I'm sure it was something involving Maine's voting system. I suppose I should have checked with Google before I posted, though, serves me right for not doing so. > >> I have no idea of what would change if the sharing was applied >> throughout, I think Obama probably has enough of a majority to be safe >> from any such changes, but it would be interesting re-work the 2000 >> election in this fashion. Since Gore (narrowly) won the overall vote >> for President, you would have to suspect that it could well change the >> result. > > Unfortunately, if that method had been in use in every state, the > likely result would have been that we wouldn't have had a recount > battle just in one state, but in every Congressional district where > the vote was close. Not fun. > For sure - but then I think the question was only of academic interest, I can't imagine that any sizeable number of the states are going to switch to the Nebraska/Maine model any time soon. For now, at least, I'm just waiting for the dust to settle - Pennsylvania is a strange state, Democratic on the eastern and western edges, but solidly Republican up the middle. McCain won about 65% of the vote in this county. Brian. From grandaeval at tiscali.co.uk Thu Nov 6 21:36:28 2008 From: grandaeval at tiscali.co.uk (Grattan) Date: Thu, 6 Nov 2008 20:36:28 -0000 Subject: [blml] What did I say? References: <4912AB9C.3080004@skynet.be><002101c94006$f2e8bc10$0302a8c0@Mildred> <4912EA56.4040701@skynet.be><003601c94020$22ab69c0$0302a8c0@Mildred> <49132307.2010804@skynet.be> Message-ID: <001d01c9404f$5a791ad0$0302a8c0@Mildred> Grattan Endicott To: "Bridge Laws Mailing List" Sent: Thursday, November 06, 2008 5:01 PM Subject: Re: [blml] What did I say? >I have gone back to Richard's answer to my question and I must confess I > misread it. Having seen one sentence that I agreed with actually made me > think I was getting the answer I expected. > > Not so, apparently. > > Richard will award, when a deviation occurs and the partner seems to > have more reason to expect it than opponents do, a score of A+/A-. He > bases this on the guidelines of the EBU, not that he believes they apply > in Australia or in any other part of the world except England (and > Wales, IIRC). > > Grattan apparently agrees with this. > > I disagree (so what's new). > > Alain has also already pointed out why this approach must be wrong. > > Is there anyone else who will have a go before I explain in detail why > it must be wrong? > +=+ Wales and England have their respective NBOs.. They may or may not regulate similarly; there is an element of liaison between them. If use of a call is illegal the call is disallowed and score adjustment may not take account of any result that might be obtained by way of the disallowed call. A deviation of which partner has more reason to be aware than have opponents is an illegal call. It is an infraction to make such a call. ~ Grattan ~ +=+ From ehaa at starpower.net Thu Nov 6 21:40:58 2008 From: ehaa at starpower.net (Eric Landau) Date: Thu, 6 Nov 2008 15:40:58 -0500 Subject: [blml] New Laws don't allow UI situation to be resolved? In-Reply-To: References: <2b1e598b0810302300t4f888261j8d7615f57fcdd4ee@mail.gmail.com> <000301c93b33$398b9670$aca2c350$@no> <001b01c93b5d$4438f2f0$ccaad8d0$@no> <000001c93c08$ea3dbf90$beb93eb0$@no> <000101c93c49$99440220$cbcc0660$@no> <001a01c93c5b$6d020830$0302a8c0@Mildred> <490CCA60.5060607@meadows.pair.com> <000801c93cb6$c082c5c0$0302a8c0@Mildred> <490D6BA7.30607@skynet.be> <002c01c93ce1$221094f0$0302a8c0@Mildred> <490D98CC.6040908@talktalk.net> Message-ID: <570ECB04-6224-4CDE-AD70-1BCED5662B6D@starpower.net> On Nov 6, 2008, at 11:47 AM, Robert Frick wrote: > On Wed, 05 Nov 2008 08:26:03 -0500, Robert Frick > wrote: > >> I am trying to separate the question of what a player or director >> should do when a player receives a snippet of information, from >> what the >> laws currently require. I have the following. Is this right? >> >> 1. The player is required to report any snippet of information as >> soon as >> it is received. Reporting all snippets of information seems to be >> completely impractical at the club level, and probably any non-major >> tournament. So I assume that this law must be infracted, with >> unspoken >> approval from the director. >> >> 2. Once this law is infracted, L16C1 and L16C3 don't apply any more, >> giving the player considerable freedom. For example, the player >> apparently >> is free to either try to ignore the information or to process the >> information (think about meaning, find out which board is involved). >> >> 3. If the information seems at a later time to be relevant, L16A3 now >> applies -- the player cannot base his call or play on the UI. But >> this >> will often be impossible, psychologically (because people >> essentially do >> not have the ability to ignore information they possess). >> >> 4. When faced with this situation, the player may notify the >> director. >> But >> he need not (L72B2). Is this true? >> >> 5. The director will apply L16C2(c). But is there any legal way to >> get to >> this? (The initial conditions of L16C2 and L16C3 are not met.) I >> had the >> idea that maybe the director could rectify for the initial failure to >> report the reception of information (L84A and Law82B1?). Then, if the >> initial reception of information had been reported, L16C2(c) would >> now be >> in force, so maybe L16C2 would be an appropriate rectification? > > There is a question of what ethical players at a club actually do when > faced with extraneous information from an outside source before the > play > of the hand. > > I am assuming that they make no effort to attend to or process the > snippet > of information. When it seems large, they notify the director > beforehand. > (In doing so, they will learn what the director wants to hear about > and > what the director does not want to hear about.) The director almost > surely > will direct that the board be played and that the director should be > notified if there is a problem. If there is, the director treats both > sides as nonoffending. (Or else that player never again reports > relevant > UI.) > > To bridge the gap between this practice and the actual law, perhaps it > could be said that upon receipt of extraneous information, there is an > implicit director call. The director then implicitly says to play the > board and call me if there is a problem. That creates an implicit > instruction from the director to be notified if the information > becomes > relevant; both sides must be treated as nonoffending; and the > director now > is legally at L16C2(c) I think "call me back if there's a problem" gives the player too much leeway. My instruction normally would be to call me back if he has any reason to suspect that the board he's on might be the one he has the EI about. Eric Landau 1107 Dale Drive Silver Spring MD 20910 ehaa at starpower.net From ehaa at starpower.net Thu Nov 6 21:56:19 2008 From: ehaa at starpower.net (Eric Landau) Date: Thu, 6 Nov 2008 15:56:19 -0500 Subject: [blml] one short In-Reply-To: <49134591.1010504@meadows.pair.com> References: <200811061612.IAA23613@mailhub.irvine.com> <49134591.1010504@meadows.pair.com> Message-ID: <0A7BB797-B229-4F7D-B1A2-E6946360C195@starpower.net> On Nov 6, 2008, at 2:29 PM, brian wrote: > Adam Beneschan wrote: >> Brian wrote: > > Interesting. I recall an extended discussion with my wife on the > merits of the two systems, because this whole electoral college > concept is just such a strange idea to me, having been brought up on > the British system. Since I didn't move to the USA until 1997, it > obviously can't have been the changeover in either state. Now you've > got me wondering what it was that DID happen, I'm sure it was > something involving Maine's voting system. I suppose I should have > checked with Google before I posted, though, serves me right for not > doing so. > >>> I have no idea of what would change if the sharing was applied >>> throughout, I think Obama probably has enough of a majority to be >>> safe >>> from any such changes, but it would be interesting re-work the 2000 >>> election in this fashion. Since Gore (narrowly) won the overall vote >>> for President, you would have to suspect that it could well >>> change the >>> result. >> >> Unfortunately, if that method had been in use in every state, the >> likely result would have been that we wouldn't have had a recount >> battle just in one state, but in every Congressional district where >> the vote was close. Not fun. > > For sure - but then I think the question was only of academic > interest, I can't imagine that any sizeable number of the states are > going to switch to the Nebraska/Maine model any time soon. That will never happen. It would eliminate the primary effect of the Electoral College system, which is to overweight the voting in the smaller states. If the American polity chose to do away with that particular effect, they would simply eliminate the Electoral College altogether and elect the President by direct popular vote. There is a substantial, but definitely minority, consituency for doing that; we hear from them at length every time the popular and electoral vote winners diverge, as happened in 2000 (after the U.S. Supreme Court settled the electoral vote count). > For now, at least, I'm just waiting for the dust to settle - > Pennsylvania is a strange state, Democratic on the eastern and western > edges, but solidly Republican up the middle. McCain won about 65% of > the vote in this county. Not so strange. Southeastern Pennsylvania is metropolitan Philadelphia and the heavily industrialized Lehigh Valley, and southwestern Pennsylvania is metropolitan Pittsburgh and its surrounding industrial towns. The "red T" in the center and north is rural, mostly bible-belt country, with the Amish concentrated in the south central part of the state. Eric Landau 1107 Dale Drive Silver Spring MD 20910 ehaa at starpower.net From grandaeval at tiscali.co.uk Thu Nov 6 22:38:17 2008 From: grandaeval at tiscali.co.uk (Grattan) Date: Thu, 6 Nov 2008 21:38:17 -0000 Subject: [blml] one short References: <200811061612.IAA23613@mailhub.irvine.com> <49134591.1010504@meadows.pair.com> Message-ID: <002601c94057$fd88e770$0302a8c0@Mildred> Grattan Endicott To: "Bridge Laws Mailing List" Sent: Thursday, November 06, 2008 7:29 PM Subject: Re: [blml] one short > > For sure - but then I think the question was only of academic > interest, I can't imagine that any sizeable number of the states are > going to switch to the Nebraska/Maine model any time soon. > +=+ I agree. However, I suspect that the Nebraska/Maine model could more accurately reflect the proportionality of the minority votes state by state. ~ Grattan ~ +=+ From richard.hills at immi.gov.au Thu Nov 6 22:41:50 2008 From: richard.hills at immi.gov.au (richard.hills at immi.gov.au) Date: Fri, 7 Nov 2008 08:41:50 +1100 Subject: [blml] only on blml [SEC=UNOFFICIAL] In-Reply-To: <4912ECB6.2020401@skynet.be> Message-ID: Herman De Wael: [snip] >Except David, Grattan and Richard of course, who regularly >refer to Presidents and Prime Ministers. Only polite plural >is acceptable in those instances. On behalf of the Australian people, Prime Minister John Howard politely prayed for Obama, February 10th 2007: "If I was running al-Qaeda in Iraq, I would put a circle around March 2008, and pray, as many times as possible, for a victory not only for Obama, but also for the Democrats." On behalf of the American people, Senator Obama politely accepted John Howard's flattery, February 11th 2007: "I think it's flattering that one of George Bush's allies on the other side of the world started attacking me the day after I announced. I would also note that we have close to 140,000 troops in Iraq, and my understanding is Mr Howard has deployed 1400, so if he is ... to fight the good fight in Iraq, I would suggest that he calls up another 20,000 Australians and sends them to Iraq. Otherwise it's just a bunch of empty rhetoric." Herman De Wael: >I have gone back to Richard's answer to my question and I >must confess I misread it. Having seen one sentence that I >agreed with actually made me think I was getting the answer >I expected. > >Not so, apparently. > >Richard will award, when a deviation occurs and the partner >seems to have more reason to expect it than opponents do, a >score of A+/A-. He bases this on the guidelines of the EBU, >not that he believes they apply in Australia or in any other >part of the world except England (and Wales, IIRC). Richard Hills quibbles: As a point of clarification, I do not blindly follow EBU guidance when I deem EBU guidance to be pig-headed and/or unLawful. For example, totally unLawful was Orange Book 3B10: "If a player has knowledge that partner tends to forget a particular agreement that tendency must neither be disclosed nor acted upon. This is an exception to the principle explained in 3A2 and is a change in policy with this Orange Book." Fortunately the most recent meeting of the EBU L&EC finally realised that 3B10 was unLawful, abolishing the clause with immediate effect. Furthermore, while I support EBU reasoning in deeming that Ave+/Ave- is appropriate for an infraction of Law 40B5 (colloquially known as "illegal convention"), I differ from EBU policy on some Law 40B4 rulings. That is, if a concealed partnership understanding (misnamed by the EBU as a "Red Psyche" or "Fielded Misbid") would be colloquially known as a "legal convention" if revealed, then I award an assigned -- not artificial -- score to rectify damage, based on a weighting of what would have happened if the non-offending side had been aware of the offending side's understanding. Grattan Endicott: >>.....If use of a call is illegal the call is disallowed and >>score adjustment may not take account of any result that >>might be obtained by way of the disallowed call. >> A deviation of which partner has more reason to be >>aware than have opponents is an illegal call. It is an >>infraction to make such a call. >> ~ Grattan ~ +=+ Richard Hills: In my opinion, Grattan's statements above are (or should be) universal principles of International Law, not merely local principles of EBU interpretation. What's the problem? Best wishes Richard James Hills Recruitment Section, Level 3 Blue, workstation 15 (first on left) Department of Immigration and Citizenship Telephone: 02 6223 8453 Email: richard.hills at immi.gov.au -------------------------------------------------------------------- Important Notice: If you have received this email by mistake, please advise the sender and delete the message and attachments immediately. This email, including attachments, may contain confidential, sensitive, legally privileged and/or copyright information. Any review, retransmission, dissemination or other use of this information by persons or entities other than the intended recipient is prohibited. DIAC respects your privacy and has obligations under the Privacy Act 1988. The official departmental privacy policy can be viewed on the department's website at www.immi.gov.au. See: http://www.immi.gov.au/functional/privacy.htm --------------------------------------------------------------------- From JffEstrsn at aol.com Fri Nov 7 00:53:28 2008 From: JffEstrsn at aol.com (Jeff Easterson) Date: Fri, 07 Nov 2008 00:53:28 +0100 Subject: [blml] Richard Hills quotes John Howard Message-ID: <49138378.6030106@aol.com> Puzzling. Why March 2008? What happens then? As far as I know the election was two days ago and investiture before March. (But I could be mistaken in the latter.) And doesn't anyone use the subjunctive any more? Howard: "If I was...." instead of "If I were...." Ciao, JE From richard.hills at immi.gov.au Fri Nov 7 02:25:36 2008 From: richard.hills at immi.gov.au (richard.hills at immi.gov.au) Date: Fri, 7 Nov 2008 12:25:36 +1100 Subject: [blml] Richard Hills quotes John Howard [SEC=UNOFFICIAL] In-Reply-To: <49138378.6030106@aol.com> Message-ID: Richard Hills quotes another Aussie Prime Minister, William McMahon (writing in the 1971 Reader's Digest): "The world has suddenly grown small as it spins furiously down the ringing grooves of change." A better use of metaphor in 1971 Aussie politics occurred when McMahon was plotting against his own party leader, John Gorton, despite McMahon notionally being on vacation at the holiday town Isle of Capri. Future Prime Minister Gough Whitlam provided the political quote of the year, saying that McMahon: "sat there on the Isle of Capri plotting his [John Gorton's] destruction - Tiberius with a telephone". Jeff Easterson: >Puzzling. Why March 2008? What happens then? As far >as I know the election was two days ago and >investiture before March. (But I could be mistaken in >the latter.) Richard Hills: Franklin Delano Roosevelt was first elected President on November 8th 1932, but was not inaugurated until March 4th 1933. This caused some problems, as the doctrinaire Hoover Administration remained in lame duck power for four months during the height of the Great Depression. The Democrat controlled Congress had foreseen the problem, proposing the Twentieth Amendment to the Constitution on March 2nd 1932. But, alas, insufficient States had ratified the Amendment by its internal deadline of the next October 15th, so the new inauguration date for a President of January 20th (only two-and-a-half months of lame duckery) did not happen until 1937. On the other hand, Nobel Peace Prize winner Jimmy Carter used his November 1980 to January 1981 period as a lame duck as a negotiating tool with the government of Iran. Carter pointed out to the Iranians that they might get a better deal from him about ransom for the American hostages than the Iranians might get from his enthusiastic ("we start bombing in five minutes") successor in the Presidency. Sure enough, the hostages were released by Iran on January 20th 1981. As for March 2008, when Barack Obama announced his presidential candidacy at the beginning of 2007, the Democrats had just won the mid-term Congressional elections. Senator Barack Obama campaigned on the idea of his Congressional colleagues mandating a phased withdrawal from Iraq to conclude in March 2008. The precedent was the phased withdrawal from Vietnam imposed upon Republican President Gerald Ford by a Democrat Congress in 1975. But the Democrat success in the 2006 midterms was much smaller than the landslide (due to the stench of Watergate) in the 1974 midterms. Hence the Republicans were able to easily outmanoeuvre the Democrats on Iraq in 2007 by various parliamentary tactics, notably the filibuster. Jeff Easterson: >And doesn't anyone use the subjunctive any more? >Howard: "If I was...." instead of "If I were...." The Naughty Preposition, by Morris Bishop, in *The New Yorker*, 1947: I lately lost a preposition; It hid, I thought, beneath my chair; And angrily I cried,"Perdition! Up from out of in under there!" Correctness is my vade mecum, And straggling phrases I abhor, And yet I wonder, "What should he come Up from out of in under for?" Best wishes Richard James Hills Recruitment Section, Level 3 Blue, workstation 15 (first on left) Department of Immigration and Citizenship Telephone: 02 6223 8453 Email: richard.hills at immi.gov.au -------------------------------------------------------------------- Important Notice: If you have received this email by mistake, please advise the sender and delete the message and attachments immediately. This email, including attachments, may contain confidential, sensitive, legally privileged and/or copyright information. Any review, retransmission, dissemination or other use of this information by persons or entities other than the intended recipient is prohibited. DIAC respects your privacy and has obligations under the Privacy Act 1988. The official departmental privacy policy can be viewed on the department's website at www.immi.gov.au. See: http://www.immi.gov.au/functional/privacy.htm --------------------------------------------------------------------- From grandaeval at tiscali.co.uk Fri Nov 7 02:53:03 2008 From: grandaeval at tiscali.co.uk (Grattan) Date: Fri, 7 Nov 2008 01:53:03 -0000 Subject: [blml] The Nebraskan issue Message-ID: <000801c9407b$b54c2070$0302a8c0@Mildred> Grattan Endicott Message-ID: Abraham Lincoln, letter to long-time friend and slave-holder Joshua F. Speed, August 24th 1855: As a nation, we began by declaring that "all men are created equal". We now practically read it "all men are created equal, except negroes". When the Know-Nothings get control, it will read "all men are created equal, except negroes, and foreigners, and catholics". When it comes to this I should prefer emigrating to some country where they make no pretence of loving liberty - to Russia, for instance, where despotism can be take pure, and without the base alloy of hypocracy. Richard Hills: The neologism "hypocracy" was Honest Abe's own invention. Perhaps he was fusing the two standard words "hypocrite" and "democracy", i.e. that government of the hypocrites, by the hypocrites, for the hypocrites, should perish from the earth. Jerry Fusselman correctly asked (It is not the act of a know- nothing to seek to be informed. The so-called "Know-Nothing" Party of the mid-19th century was given that name not just because its members were Palinesque know-nothings, but also because its non-mainstream origin was that of a xenophobic secret society. Since a secret society whose members tell all is a contradiction in terms, members were instructed to respond to all questions with the answer, "I know nothing".): >For example, how is damage defined? > >Is it automatic that you get Ave+ at least---or do you need >to prove damage? Richard Hills replies (since there are no stupid questions, only stupid answers): A false dichotomy -- for many moons now the Lawbook has defined **both** clauses of Jerry's second question as true. 1997 LAW 88 - AWARD OF INDEMNITY POINTS In a pair or individual event, when a non-offending contestant is required to take an artificial adjusted score through no fault or choice of his own, such contestant shall be awarded a minimum of 60% of the matchpoints available to him on that board, or the percentage of matchpoints he earned on boards actually played during the session if that percentage was greater than 60%. 2007 LAW 88 - AWARD OF INDEMNITY POINTS See Law 12C2. 2007 LAW 12C2: 2. (a) When owing to an irregularity no result can be obtained [and see C1(d)] the Director awards an artificial adjusted score according to responsibility for the irregularity: average minus (at most 40% of the available matchpoints in pairs) to a contestant directly at fault, average (50% in pairs) to a contestant only partly at fault, and average plus (at least 60% in pairs) to a contestant in no way at fault. (b) When the Director awards an artificial adjusted score of average plus or minus at international match points that score is normally plus or minus 3 imps, but this may be varied as Law 86A allows. (c) The foregoing is modified for a non-offending contestant that obtains a session score exceeding 60% of the available matchpoints or for an offending contestant that obtains a session score that is less than 40% of the available matchpoints (or the equivalent in imps). Such contestants are awarded the percentage obtained (or the equivalent in imps) on the other boards of that session. What's the problem? Well, there is a problem with 2007 Law 12C2(c). Suppose that in a 26-board session a non-offending pair scores a top on the first board, then (through no fault of their own) the other 25 boards are fouled only at their table (not at any other table) due to a computer glitch. Does that non-offending pair always get a session score of 100%? Best wishes Richard James Hills Recruitment Section, Level 3 Blue, workstation 15 (first on left) Department of Immigration and Citizenship Telephone: 02 6223 8453 Email: richard.hills at immi.gov.au -------------------------------------------------------------------- Important Notice: If you have received this email by mistake, please advise the sender and delete the message and attachments immediately. This email, including attachments, may contain confidential, sensitive, legally privileged and/or copyright information. Any review, retransmission, dissemination or other use of this information by persons or entities other than the intended recipient is prohibited. DIAC respects your privacy and has obligations under the Privacy Act 1988. The official departmental privacy policy can be viewed on the department's website at www.immi.gov.au. See: http://www.immi.gov.au/functional/privacy.htm --------------------------------------------------------------------- From richard.hills at immi.gov.au Fri Nov 7 07:24:47 2008 From: richard.hills at immi.gov.au (richard.hills at immi.gov.au) Date: Fri, 7 Nov 2008 17:24:47 +1100 Subject: [blml] Mr Gladstone suggested [SEC=UNOFFICIAL] In-Reply-To: <1429874746-1225461820-cardhu_decombobulator_blackberry.rim.net-2091802381-@bxe065.bisx.produk.on.blackberry> Message-ID: W.E. Gladstone, House of Commons, 21st January 1886: "I would tell them of my intention to keep my counsel ... and I will venture to recommend them, as an old Parliamentary hand, to do the same." David Burn asserted: >Where you seem to be stuck is that you believe that any such >disclosure automatically makes the deviation part of the >methods. It does not, provided that the deviation occurs >with less than a frequency X. As I have said, it is for the >Regulating Authorities to determine X for events within >their jurisdictions. Richard Hills quibbles: Where David Burn seems to be stuck is that Sven Pran and I instead believe the converse -- that a call which is part of the partnership's methods must therefore be disclosed. Law 40C1 (second sentence): "Repeated deviations lead to implicit understandings which then form part of the partnership's methods and must be disclosed in accordance with the regulations governing disclosure of system." Richard Hills: Sven Pran and I also believe that a call which is _not_ part of the partnership's methods need _not_ be disclosed. Law 40C2: "Other than the above no player has any obligation to disclose to opponents that he has deviated from his announced methods." Law 75C, WBF LC interpretation, October 10th 2008: "The phrase 'they have no claim to an accurate description of the N-S hands' first appeared in the 1975 laws of the game. It was accompanied then as now by the injunction forbidding the Director to alter the table result. It was entered primarily to establish beyond doubt that that the partnership agreement must be described accurately in response to lawful enquiry and that the explanation given must not aim to describe what the explainer believes as to the contents of either hand. It was continued in those terms in the 1987 law book, while for 2007 NBOs were invited to say whether the example or the wording should be updated. Among replies received there was a general consensus for retaining them as they had been previously, whilst moving the statements from a footnote into the body of the Law." David Burn asserted: >Precisely. That's why the WBF policy specifies that there >must be disclosure of the nature of the deviation. Then, >partner will have no more reason than the opponents to be >aware of the possibility of and the nature of the deviation. Richard Hills quibbles: The WBF policy David Burn refers to was created by the WBF Systems Committee. But the WBF Systems Committee lacks the power to overrule the WBF Laws Committee on interpretation of the Lawbook in general and of Law 75C in particular. (Indeed, the WBF Systems Committee has failed to revisit its policies since the 2007 Lawbook took effect.) What's the problem? Best wishes Richard James Hills Recruitment Section, Level 3 Blue, workstation 15 (first on left) Department of Immigration and Citizenship Telephone: 02 6223 8453 Email: richard.hills at immi.gov.au -------------------------------------------------------------------- Important Notice: If you have received this email by mistake, please advise the sender and delete the message and attachments immediately. This email, including attachments, may contain confidential, sensitive, legally privileged and/or copyright information. Any review, retransmission, dissemination or other use of this information by persons or entities other than the intended recipient is prohibited. DIAC respects your privacy and has obligations under the Privacy Act 1988. The official departmental privacy policy can be viewed on the department's website at www.immi.gov.au. See: http://www.immi.gov.au/functional/privacy.htm --------------------------------------------------------------------- From ardelm at optusnet.com.au Fri Nov 7 04:31:46 2008 From: ardelm at optusnet.com.au (Tony Musgrove) Date: Fri, 07 Nov 2008 14:31:46 +1100 Subject: [blml] The Nebraskan issue In-Reply-To: <000801c9407b$b54c2070$0302a8c0@Mildred> References: <000801c9407b$b54c2070$0302a8c0@Mildred> Message-ID: <6.1.0.6.2.20081107143112.01d68980@mail.optusnet.com.au> At 12:53 PM 7/11/2008, you wrote: >Grattan Endicottalso ************************************** >Which way up is the Universe? >''''''''''''''''''''''''''''''''''''''''''''''''''''''''''''''''''''''''''''''''''''''''''''''''' > >In the Southern Hemisphere water swirls in antipodean fashion. >In Nebraska blue donkeys walk to the left of red elephants, >while in England the red political spectrum is found to the left >of the blue. All perception. > > > >With the US election arousing more interest than a H1H bid, and with reference to a previous quote of Adlai Stevenson, I append the following: Cheers, Tony > > > >61e35c4.jpg > > > > -------------- next part -------------- An HTML attachment was scrubbed... URL: http://www.amsterdamned.org/pipermail/blml/attachments/20081107/68b60a0c/attachment-0001.htm -------------- next part -------------- A non-text attachment was scrubbed... Name: 61e35c4.jpg Type: image/jpeg Size: 303555 bytes Desc: not available Url : http://www.amsterdamned.org/pipermail/blml/attachments/20081107/68b60a0c/attachment-0001.jpg From grandaeval at tiscali.co.uk Fri Nov 7 09:46:10 2008 From: grandaeval at tiscali.co.uk (Grattan) Date: Fri, 7 Nov 2008 08:46:10 -0000 Subject: [blml] Mr Gladstone suggested [SEC=UNOFFICIAL] References: Message-ID: <000f01c940b5$4a0c2d20$0302a8c0@Mildred> Grattan Endicott To: "Bridge Laws Mailing List" Sent: Friday, November 07, 2008 6:24 AM Subject: Re: [blml] Mr Gladstone suggested [SEC=UNOFFICIAL] (Indeed, the WBF Systems Committee has failed to revisit its policies since the 2007 Lawbook took effect.) > +=+ It is more true to say that what it did in Beijing has not yet seen the light of day. It is in the pipeline for publication on the website. ~ Grattan ~ +=+ From Hermandw at skynet.be Fri Nov 7 10:20:19 2008 From: Hermandw at skynet.be (Herman De Wael) Date: Fri, 07 Nov 2008 10:20:19 +0100 Subject: [blml] Mr Gladstone suggested [SEC=UNOFFICIAL] In-Reply-To: References: Message-ID: <49140853.60107@skynet.be> richard.hills at immi.gov.au wrote: > W.E. Gladstone, House of Commons, 21st January 1886: > > "I would tell them of my intention to keep my counsel ... and > I will venture to recommend them, as an old Parliamentary > hand, to do the same." > > David Burn asserted: > >> Where you seem to be stuck is that you believe that any such >> disclosure automatically makes the deviation part of the >> methods. It does not, provided that the deviation occurs >> with less than a frequency X. As I have said, it is for the >> Regulating Authorities to determine X for events within >> their jurisdictions. > > Richard Hills quibbles: > > Where David Burn seems to be stuck is that Sven Pran and I > instead believe the converse -- that a call which is part of > the partnership's methods must therefore be disclosed. > Where Richard seems to be struck is in thinking that these two things are antagonistic. David is talking about "methods" not "understandings". David has the correct view: things can be understandings, and must be disclosed, while they are not yet methods, and therefore not subject to system regulation. If Richard sees this difference too, he'll become unstuck. Herman. From Hermandw at skynet.be Fri Nov 7 10:25:43 2008 From: Hermandw at skynet.be (Herman De Wael) Date: Fri, 07 Nov 2008 10:25:43 +0100 Subject: [blml] What did I say? In-Reply-To: <001d01c9404f$5a791ad0$0302a8c0@Mildred> References: <4912AB9C.3080004@skynet.be><002101c94006$f2e8bc10$0302a8c0@Mildred> <4912EA56.4040701@skynet.be><003601c94020$22ab69c0$0302a8c0@Mildred> <49132307.2010804@skynet.be> <001d01c9404f$5a791ad0$0302a8c0@Mildred> Message-ID: <49140997.90002@skynet.be> Grattan wrote: > > If use of a call is illegal the call is disallowed and score > adjustment > may not take account of any result that might be obtained by way of > the disallowed call. > A deviation of which partner has more reason to be aware than > have opponents is an illegal call. It is an infraction to make such a call. > ~ Grattan ~ +=+ > OK Grattan, then now tell me how we distinguish between a less-than fully revealed deviation and a misexplanation. Take the Helgemo 1H as an example: is his 8HCP opening a deviation of a 10-19 system, or is it a misrepresented 8-19 system? How do we tell the difference? Herman. From grandaeval at tiscali.co.uk Fri Nov 7 14:41:03 2008 From: grandaeval at tiscali.co.uk (Grattan) Date: Fri, 7 Nov 2008 13:41:03 -0000 Subject: [blml] Deviations from announced system. Message-ID: <003501c940de$dfd49c60$0302a8c0@Mildred> Grattan Endicott References: <003501c940de$dfd49c60$0302a8c0@Mildred> Message-ID: <4914532F.1080100@skynet.be> Grattan wrote: > > < > Herman: > "David is talking about "methods" not "understandings". > David has the correct view: things can be understandings, > and must be disclosed, while they are not yet methods, > and therefore not subject to system regulation." > < > +=+ David, I imagine, will feel able to speak for himself. > However, Herman is just plain wrong in suggesting that > not everything which is a partnership understanding is > part of the partnership's methods. Let's get one thing straight here: as long as you keep using words that have no legal standing in quasi legal meanings, we shall not get anywhere on this. A deviation is a deviation. From something. That something needs a name. Call it methods, as David did, and Grattan drags up the word method from the 2007 Laws. Call it agreements, as I did, and someone drags up the 1997 legal meaning of that word. Call it elephants, as Richard did, and people start with ridicule. Whatever we call it, a deviation is a deviation from something. There can be partnership understanding regarding when and how deviations are likely to occur, but these understandings are not part of the "something" that the deviation deviates from. This seems like pure and simple logic to me. > The law says that > "repeated deviations lead to implicit understandings which > then form part of the partnership's methods and must be > disclosed in accordance with the regulations governing > disclosure of system". Yes Grattan, but even repeated deviations are still deviations from something. The fact that the repeating creates understandings about them does not change the something they are deviations from. > The law also says that the Regulating > Authority "is empowered without restriction to allow, > disallow, or allow conditionally, any special partnership > understanding". (A partnership understanding translates to > a special partnership understanding on the say-so of the > Regulating Authority when it decides to regulate it.) +=+ Indeed - this seems to mean that a RA can ban all psyches if it wants to - which cannot be correct. > .................................................................................. > +=+ So much, this much. However, there is a further > point to clarify. I wrote: > " If use of a call is illegal the call is disallowed and score > adjustment may not take account of any result that might > be obtained by way of the disallowed call. > A deviation of which partner has more reason to be > aware than have opponents is an illegal call. It is an > infraction to make such a call." I am willing to accept that, but if I do, it is only out of higher authority. And before you set this in stone, Grattan, would you please listen to a few arguments which say that this may not be the best way to deal with these things? > So when a player makes a deviant call that is > based on a partnership understanding that has not been > disclosed he commits an infraction in making the call. > The call is disallowed and in adjusting the score the > Director disregards any possible outcome that might > be reached via the disallowed score. His adjustment > will not remove any advantage that opponents happen > to have gained. In this there is no conflict with the final > statement in Law 40C1. Herman's enquiry whether the > matter could be remedied by considering possible > outcomes had disclosure occurred is a non-starter since > it would reintroduce a disallowed call that can no longer > be taken into consideration. Score adjustment returns > the equity to the point before the infraction occurred.+=+ > ............................................................................ > ~ Grattan ~ +=+ > Grattan, please think about the following arguments: a) While your interpretation is a possible one, it is not the only one. L40C1 says nothing more than "he shall adjust the score". There is no reference to L12, but it has to be assumed that L12 is to be used. L12 says nothing about how such an adjustment shall take place. Of course we adjust to the result "had the irregularity not occured". It is your interpretation only that says that the bid is the irregularity, not the non-disclosure. A second interpretation is certainly viable. b) Compare all this to a "normal" non-disclosure, such as a failure to alert. The 2007 laws talk of a duty to disclose, but the previous laws all talked about "you may not utilize methods that your opponents are unaware of". At no time was an adjustment made with the non-disclosed call absent. Surely if the 2007 laws were meant to change the way deviations are treated, the WBFLC would have specified this? c) When ruling on a non-disclosed deviation, how shall the director decide if something is a deviation or part of a system? If you consider Helgemo's 1H a deviation from "10-19", he will be judged to have passed. But if you decide he actually plays "8-19", he will get an adjustment with his 1He in place. How is the TD to distinguish between the two? d) "Repeated deviations lead to implicit understandings that form part of the methods". Does this not mean that these are now to be treated as methods which are non-disclosed, with the simple MI adjustments of L40B4? e) do we really need two different kinds of adjustment for what is basically the same crime - not telling your opponents all they are entitled to? Why should the frequency of a psyche be any different from the set of hands that a player is likely to jump to 3NT with (as opposed to the set of hands he asks Stayman with). Both are merely indications of what style of bidding a player uses. f) what is a deviation for one pair might be a perfectly written-out method for another. I never open 1NT with 5 of a major, but I did it last Tuesday. My heart suit was K9xxx. Aren't my opponents entitled to know that this is the only time I would do so? Does that entitlement differ if it stems from "hand evaluation" or from "written system notes". I think Grattan opens a can of worms if he really decides to have the WBF go down this route. Herman. From agot at ulb.ac.be Fri Nov 7 15:45:21 2008 From: agot at ulb.ac.be (Alain Gottcheiner) Date: Fri, 07 Nov 2008 15:45:21 +0100 Subject: [blml] Deviations from announced system. In-Reply-To: <003501c940de$dfd49c60$0302a8c0@Mildred> References: <003501c940de$dfd49c60$0302a8c0@Mildred> Message-ID: <49145481.9000008@ulb.ac.be> Grattan a ?crit : > Grattan Endicott also ************************************** > > David Burn: > "Where you seem to be stuck is that you believe that any > such disclosure automatically makes the deviation part of > the methods. It does not, provided that the deviation occurs > with less than a frequency X. As I have said, it is for the > Regulating Authorities to determine X for events within > AG: I've no qualms with the general idea (frequency differenciates between psyches and parts of the system), but I'm wondering whether there is any way to ascertain frequencies. Few pairs maintain a detailed account of their bidding sequences, and many will know what their interest is when asked about frequency. Furthermore, you'll have to split hairs very fine to make sure that situations are well-defined. For example, a player may have a moderately long history of psyching a 1NT opening, NV and first-in-hand, but "I never thought he'd do it 2nd-in-hand" could be a fair description of the element of surprise that indeed ensures the opening was a psyche. So, you'll need to make distinctions about seat, vulnerability, type of competition, moment in the competition, opponents' personalities and style ('I psyched an opening already twice against him, because he overcalls so light and his partner won't be able to know who's the joker') etc. Furthermore, this might be dependent on table incidents. The last two times I psyched an opening (and that's the best I can remember, because it already puts us several years back), it was because LHO's behaviour made it obvious that he hald a strong hand. Try putting that into the equation.. Instead, I'd find it more practical to demand that a pair who psyche more than one in their sams?ra state as faithfully as possible which kind of psychic bids are possible, an adverb describing frequency (or a % statement), and what parameters will be taken into account. (and also which bids might be non-psyche, but systemically spectacularly light). If they aren't able to do this in a few lines, this isn't necessarily bad news : it could mean that they haven't any pattern after all. Best regards Alain From john at asimere.com Fri Nov 7 15:55:11 2008 From: john at asimere.com (John (MadDog) Probst) Date: Fri, 7 Nov 2008 14:55:11 -0000 Subject: [blml] Mr Gladstone suggested [SEC=UNOFFICIAL] References: <49140853.60107@skynet.be> Message-ID: ----- Original Message ----- From: "Herman De Wael" To: "Bridge Laws Mailing List" Sent: Friday, November 07, 2008 9:20 AM Subject: Re: [blml] Mr Gladstone suggested [SEC=UNOFFICIAL] > richard.hills at immi.gov.au wrote: >> W.E. Gladstone, House of Commons, 21st January 1886: >> >> "I would tell them of my intention to keep my counsel ... and >> I will venture to recommend them, as an old Parliamentary >> hand, to do the same." I realised I had a problem when one of my partners alerted my 1NT overcall, and I held the H hand. With that partner I tend to keep off the heart suit now so that his perception of my 1NT overcall has started to change. It was about this time that my opponents moaned about MI as to which suit I held "He's psyched his psych" they said. I went via 2H as well, just so they'd get full value :) It still isn't an agreement whatever I do as it is unpredictable even to me and if I can't predict whether I'll psych (at least until I have inspected my cards, checked the sweep hand of my watch and observed the sex of the player sitting E at table 7.) how can partner be in any position to state we have an agreement, even though he is still obligated to disclose? "15-17 usually, or weak with H if the parity of the lowest cards in the other 3 suits are all even, or weak with a non H suit if the parity of the lowest card in all 4 suits is the same, or any another hand with sufficient frequency to ensure I can't predict it's a suit when it is psychic and psychic with an overall frequency to ensure it pays neither you nor me especially to cater for the possibility" would be close (but not correct). I'm not prepared to divulge my algorithm. I am in complete agreement with Grattan that if I do psych and if it is always the same psyche and I always do it with this hand then it is probable that it has become part of the partnership method and is thus regulatable; and that leads me to the Herman heart. I say "probably become part of", but if the absolute frequency is very low and the relative frequency to the normal use of the call is also very low then, probably, it is not disclosable since partner is not expecting it with higher expectation than opponents. Nonetheless I would disclose as an actively ethical player. Certainly the Probst NT if it were always hearts WOULD be an illegal method if it were done on all or even most H hands and on no others as it has higher absolute frequency than the rara avis Herman Heart, and for this reason I choose to vary it making calls I perceive to have less merit and keeping the H frequency lower than ideally I would like to. I see very little difference in what I am permitted to do between the 2007 code and the 1997 code in this respect. >> >> David Burn asserted: >> >>> Where you seem to be stuck is that you believe that any such >>> disclosure automatically makes the deviation part of the >>> methods. It does not, provided that the deviation occurs >>> with less than a frequency X. As I have said, it is for the >>> Regulating Authorities to determine X for events within >>> their jurisdictions. >> >> Richard Hills quibbles: >> >> Where David Burn seems to be stuck is that Sven Pran and I >> instead believe the converse -- that a call which is part of >> the partnership's methods must therefore be disclosed. >> > > Where Richard seems to be struck is in thinking that these two things > are antagonistic. > > David is talking about "methods" not "understandings". David has the > correct view: things can be understandings, and must be disclosed, while > they are not yet methods, and therefore not subject to system regulation. > > If Richard sees this difference too, he'll become unstuck. > > Herman. > > _______________________________________________ > blml mailing list > blml at amsterdamned.org > http://www.amsterdamned.org/mailman/listinfo/blml From agot at ulb.ac.be Fri Nov 7 16:08:52 2008 From: agot at ulb.ac.be (Alain Gottcheiner) Date: Fri, 07 Nov 2008 16:08:52 +0100 Subject: [blml] Deviations from announced system. In-Reply-To: <4914532F.1080100@skynet.be> References: <003501c940de$dfd49c60$0302a8c0@Mildred> <4914532F.1080100@skynet.be> Message-ID: <49145A04.2000707@ulb.ac.be> Herman De Wael a ?crit : > Let's get one thing straight here: as long as you keep using words that > have no legal standing in quasi legal meanings, we shall not get > anywhere on this. > A deviation is a deviation. From something. That something needs a name. > Agreed up to now, and I prefer the word 'agreements'. > Call it methods, as David did, and Grattan drags up the word method from > the 2007 Laws. Call it agreements, as I did, and someone drags up the > 1997 legal meaning of that word. Call it elephants, as Richard did, and > people start with ridicule. > Whatever we call it, a deviation is a deviation from something. There > can be partnership understanding regarding when and how deviations are > likely to occur, but these understandings are not part of the > "something" that the deviation deviates from. > This seems like pure and simple logic to me. > > AG : but don't agree with this. If you take a test tube containing some acid, and start adding (with caution) a strong base, there is some point after which the contents of the tube won't be an acid anymore, and the transition is nearly instantaneous. What TFLB says is : - every pair has a set of agreements - it is allowed to make bids that deviate from this set of agreements - after you made it a certain number of times, your set of agreements have changed. There is still a deviation from your old agreements, but the bid is part of the (now changed) present agreements, not a deviation from them. Since you have to disclose your present agreements, what you have to disclose has changed and the next time you make the bid your exlpanation will include any changes prompted by that new occurrence of deviation.. Furthermore, it is possible that the bid be a deviation of your announced agreements, but that your actual agreements be different, so that the bid isn't a deviation from your actual agreements - but you've committed either MI or CPU. I don't see any lack of ligic in this. Informaticians may think of this as IF (Number_of_observed_cases_of_light_opening > N) THEN Lowerrange = Lowerrange - 1. The end of this statement (Lowerrange = Lowerrange - 1) is ridiculous, but the whole sentence doesn't lack logic in any way. It means that the set of variables values (your agreements) has changed and this will be taken into account from now on. Best regards Alain From Hermandw at skynet.be Fri Nov 7 17:12:55 2008 From: Hermandw at skynet.be (Herman De Wael) Date: Fri, 07 Nov 2008 17:12:55 +0100 Subject: [blml] Deviations from announced system. In-Reply-To: <49145A04.2000707@ulb.ac.be> References: <003501c940de$dfd49c60$0302a8c0@Mildred> <4914532F.1080100@skynet.be> <49145A04.2000707@ulb.ac.be> Message-ID: <49146907.9020204@skynet.be> Alain is correct, but! Alain Gottcheiner wrote: > Herman De Wael a ?crit : >> Let's get one thing straight here: as long as you keep using words that >> have no legal standing in quasi legal meanings, we shall not get >> anywhere on this. >> A deviation is a deviation. From something. That something needs a name. >> > Agreed up to now, and I prefer the word 'agreements'. > >> Call it methods, as David did, and Grattan drags up the word method from >> the 2007 Laws. Call it agreements, as I did, and someone drags up the >> 1997 legal meaning of that word. Call it elephants, as Richard did, and >> people start with ridicule. >> Whatever we call it, a deviation is a deviation from something. There >> can be partnership understanding regarding when and how deviations are >> likely to occur, but these understandings are not part of the >> "something" that the deviation deviates from. >> This seems like pure and simple logic to me. >> >> > AG : but don't agree with this. > > If you take a test tube containing some acid, and start adding (with > caution) a strong base, there is some point after which the contents of > the tube won't be an acid anymore, and the transition is nearly > instantaneous. > > What TFLB says is : > - every pair has a set of agreements > - it is allowed to make bids that deviate from this set of agreements > - after you made it a certain number of times, your set of agreements > have changed. There is still a deviation from your old agreements, but > the bid is part of the (now changed) present agreements, not a deviation > from them. Since you have to disclose your present agreements, what you > have to disclose has changed and the next time you make the bid your > exlpanation will include any changes prompted by that new occurrence of > deviation.. > But: Then it is no longer a deviation! Which is actually my point. If you call the deviation part of your agreements (or your elephants), then it is no longer a deviation from those agreements, but an agreement in itself. To which L40C cannot apply! > Furthermore, it is possible that the bid be a deviation of your > announced agreements, but that your actual agreements be different, so > that the bid isn't a deviation from your actual agreements - but you've > committed either MI or CPU. > Well, if the TD decides that the bid is not a deviation, but actually part of the agreements, then the crime is MI. And that is why I don't believe there is such a crime as CPU. > I don't see any lack of ligic in this. > Nor do I. > Informaticians may think of this as IF > (Number_of_observed_cases_of_light_opening > N) THEN Lowerrange = > Lowerrange - 1. > The end of this statement (Lowerrange = Lowerrange - 1) is ridiculous, > but the whole sentence doesn't lack logic in any way. It means that the > set of variables values (your agreements) has changed and this will be > taken into account from now on. > > Best regards > > Alain > Herman. From agot at ulb.ac.be Fri Nov 7 17:35:09 2008 From: agot at ulb.ac.be (Alain Gottcheiner) Date: Fri, 07 Nov 2008 17:35:09 +0100 Subject: [blml] Deviations from announced system. In-Reply-To: <49146907.9020204@skynet.be> References: <003501c940de$dfd49c60$0302a8c0@Mildred> <4914532F.1080100@skynet.be> <49145A04.2000707@ulb.ac.be> <49146907.9020204@skynet.be> Message-ID: <49146E3D.8050904@ulb.ac.be> Herman De Wael a ?crit : > But: > > Then it is no longer a deviation! > Which is actually my point. If you call the deviation part of your > agreements (or your elephants), then it is no longer a deviation from > those agreements, but an agreement in itself. > > Indeed. > To which L40C cannot apply! > L40B will do, thanks. > >> Furthermore, it is possible that the bid be a deviation of your >> announced agreements, but that your actual agreements be different, so >> that the bid isn't a deviation from your actual agreements - but you've >> committed either MI or CPU. >> >> > > Well, if the TD decides that the bid is not a deviation, but actually > part of the agreements, then the crime is MI. > And that is why I don't believe there is such a crime as CPU. > ISTM that summary definitions could be : MI : not to give all information they're entitled to. CPU use : to take steps to hide information from opponents. ('we'll use 14-16 NT but tell them 15-17') Whence all CPU use is a specific kind of MI, that is, MI that's intentional and agreed upon to be given. CPU can't be an offence, because it's neither an act nor an abstention to act. Using CPUs is an offence. (there is also the possibility that you decided to give MI on purpose but partner doesn't know. That's not CPU use, but still a severe offence) The penalty for MI is a PP (along with score adjustment, which isn't a penalty). The penalty for CPU use can be banishment. Best regards Alain. From ehaa at starpower.net Fri Nov 7 19:35:28 2008 From: ehaa at starpower.net (Eric Landau) Date: Fri, 7 Nov 2008 13:35:28 -0500 Subject: [blml] The Nebraskan issue In-Reply-To: <000801c9407b$b54c2070$0302a8c0@Mildred> References: <000801c9407b$b54c2070$0302a8c0@Mildred> Message-ID: <3234F09B-9E47-4EE2-94C9-91CE772A4B7B@starpower.net> On Nov 6, 2008, at 8:53 PM, Grattan wrote: > In the Southern Hemisphere water swirls in antipodean fashion. > In Nebraska blue donkeys walk to the left of red elephants, > while in England the red political spectrum is found to the left > of the blue. All perception. I know the cold war is over, but I do wonder how and when the "red" segment of the American politcal spectrum became anything other than the leftmost extreme. Eric Landau 1107 Dale Drive Silver Spring MD 20910 ehaa at starpower.net From Hermandw at skynet.be Fri Nov 7 20:51:14 2008 From: Hermandw at skynet.be (Herman De Wael) Date: Fri, 07 Nov 2008 20:51:14 +0100 Subject: [blml] Deviations from announced system. In-Reply-To: <49146E3D.8050904@ulb.ac.be> References: <003501c940de$dfd49c60$0302a8c0@Mildred> <4914532F.1080100@skynet.be> <49145A04.2000707@ulb.ac.be> <49146907.9020204@skynet.be> <49146E3D.8050904@ulb.ac.be> Message-ID: <49149C32.7050005@skynet.be> I really don't see why we're argueing. Alain is spot on: Alain Gottcheiner wrote: > Herman De Wael a ?crit : >> But: >> >> Then it is no longer a deviation! >> Which is actually my point. If you call the deviation part of your >> agreements (or your elephants), then it is no longer a deviation from >> those agreements, but an agreement in itself. >> >> > Indeed. > >> To which L40C cannot apply! >> > > L40B will do, thanks. Of course - but the adjustment according to L40B are AS based on correct information, not based on the deviant call not being made! >> >>> Furthermore, it is possible that the bid be a deviation of your >>> announced agreements, but that your actual agreements be different, so >>> that the bid isn't a deviation from your actual agreements - but you've >>> committed either MI or CPU. >>> >>> >> Well, if the TD decides that the bid is not a deviation, but actually >> part of the agreements, then the crime is MI. >> And that is why I don't believe there is such a crime as CPU. >> > ISTM that summary definitions could be : > > MI : not to give all information they're entitled to. > CPU use : to take steps to hide information from opponents. ('we'll use > 14-16 NT but tell them 15-17') > But that is what I've always been calling the second crime - it's cheating. And of course it exists. But it's not what this discussion is all about. Our discussion is about whether there exists a third crime - the one Richard calls CPU (erroneously IMO) and which should really be called Undisclosed Deviation Tendencies. I now admit that L40C does mean that such a third crime exists, as it is punished not just by an adjusted score but also by a PP. But I still maintain that the only logical adjustment for a L40C offence is the AS that we all know as the MI one. Not the Grattan one. > Whence all CPU use is a specific kind of MI, that is, MI that's > intentional and agreed upon to be given. > > CPU can't be an offence, because it's neither an act nor an abstention > to act. > Using CPUs is an offence. > (there is also the possibility that you decided to give MI on purpose > but partner doesn't know. That's not CPU use, but still a severe offence) > > The penalty for MI is a PP (along with score adjustment, which isn't a > penalty). > The penalty for CPU use can be banishment. > Of course, but that was never in any doubt. > Best regards > > Alain. > Herman. From grandaeval at tiscali.co.uk Sat Nov 8 11:51:42 2008 From: grandaeval at tiscali.co.uk (Grattan) Date: Sat, 8 Nov 2008 10:51:42 -0000 Subject: [blml] Deviations from announced system. References: <003501c940de$dfd49c60$0302a8c0@Mildred> <4914532F.1080100@skynet.be> <49145A04.2000707@ulb.ac.be> <49146907.9020204@skynet.be><49146E3D.8050904@ulb.ac.be> <49149C32.7050005@skynet.be> Message-ID: <003701c94190$28575d40$0302a8c0@Mildred> Grattan Endicott To: "Bridge Laws Mailing List" Sent: Friday, November 07, 2008 7:51 PM Subject: Re: [blml] Deviations from announced system. : >> >> Then it is no longer a deviation! >> Which is actually my point. If you call the deviation part of your >> agreements (or your elephants), then it is no longer a deviation from >> those agreements, but an agreement in itself. >> +=+ As soon as it becomes an agreement it is subject to the system regulations for the tournament. In some instances these will declare it to be an illegal call. That is disallowed etc. Score adjustment cannot then take account of the possibility of the disallowed call. This is a situation from which no amount of wriggling will let you escape. ~ Grattan ~ +=+ From grandaeval at tiscali.co.uk Sat Nov 8 11:56:21 2008 From: grandaeval at tiscali.co.uk (Grattan) Date: Sat, 8 Nov 2008 10:56:21 -0000 Subject: [blml] The Nebraskan issue References: <000801c9407b$b54c2070$0302a8c0@Mildred> <3234F09B-9E47-4EE2-94C9-91CE772A4B7B@starpower.net> Message-ID: <003c01c94190$a4376680$0302a8c0@Mildred> Grattan Endicott To: "Bridge Laws Mailing List" Sent: Friday, November 07, 2008 6:35 PM Subject: Re: [blml] The Nebraskan issue > On Nov 6, 2008, at 8:53 PM, Grattan wrote: > >> In the Southern Hemisphere water swirls in antipodean fashion. >> In Nebraska blue donkeys walk to the left of red elephants, >> while in England the red political spectrum is found to the left >> of the blue. All perception. > > I know the cold war is over, but I do wonder how and when the "red" > segment of the American politcal spectrum became anything other than > the leftmost extreme. > > +=+ I wonder how the most conservative elements of the elephant party could allow the GOP to adopt red for its colour all over the electoral map. ~ G ~ +=+ From geller at nifty.com Sat Nov 8 13:21:34 2008 From: geller at nifty.com (Robert Geller) Date: Sat, 08 Nov 2008 21:21:34 +0900 Subject: [blml] The Nebraskan issue In-Reply-To: <003c01c94190$a4376680$0302a8c0@Mildred> References: <003c01c94190$a4376680$0302a8c0@Mildred> Message-ID: <200811081221.AA16430@geller204.nifty.com> According to The Washington Post, the terms [red states and blue state] were coined by Tim Russert during his televised coverage of the 2000 presidential election. This was by no means the first election in which the news media used colored maps to graphically depict voter preferences in the various states, but it was the first time a "standard" color scheme took hold. (Excerpted from the following:) http://en.wikipedia.org/wiki/Red_states_and_blue_states -Bob Grattan ????????: > > >Grattan Endicottalso ************************************** >"Let us then be up and doing, >With a heart for any fate, >Still achieving, still pursuing, >Learn to labour and to wait." > [H.W. Longfellow] > >''''''''''''''''''''''''''''''''''''''''''''''''''''''''''''''''''''''''''''''''''''''''' '''''''' > > > >----- Original Message ----- >From: "Eric Landau" >To: "Bridge Laws Mailing List" >Sent: Friday, November 07, 2008 6:35 PM >Subject: Re: [blml] The Nebraskan issue > > >> On Nov 6, 2008, at 8:53 PM, Grattan wrote: >> >>> In the Southern Hemisphere water swirls in antipodean fashion. >>> In Nebraska blue donkeys walk to the left of red elephants, >>> while in England the red political spectrum is found to the left >>> of the blue. All perception. >> >> I know the cold war is over, but I do wonder how and when the "red" >> segment of the American politcal spectrum became anything other than >> the leftmost extreme. >> >> >+=+ I wonder how the most conservative elements of the elephant > party could allow the GOP to adopt red for its colour all over > the electoral map. > ~ G ~ +=+ > > >_______________________________________________ >blml mailing list >blml at amsterdamned.org >http://www.amsterdamned.org/mailman/listinfo/blml ----------------------------------------------------- Robert (Bob) Geller, Tokyo, Japan geller at nifty.com From Hermandw at skynet.be Sun Nov 9 12:24:43 2008 From: Hermandw at skynet.be (Herman De Wael) Date: Sun, 09 Nov 2008 12:24:43 +0100 Subject: [blml] Deviations from announced system. In-Reply-To: <003701c94190$28575d40$0302a8c0@Mildred> References: <003501c940de$dfd49c60$0302a8c0@Mildred> <4914532F.1080100@skynet.be> <49145A04.2000707@ulb.ac.be> <49146907.9020204@skynet.be><49146E3D.8050904@ulb.ac.be> <49149C32.7050005@skynet.be> <003701c94190$28575d40$0302a8c0@Mildred> Message-ID: <4916C87B.1080105@skynet.be> Grattan wrote: > > : >>> Then it is no longer a deviation! >>> Which is actually my point. If you call the deviation part of your >>> agreements (or your elephants), then it is no longer a deviation from >>> those agreements, but an agreement in itself. >>> > +=+ As soon as it becomes an agreement it is subject to the system > regulations for the tournament. In some instances these will declare > it to be an illegal call. That is disallowed etc. Score adjustment cannot > then take account of the possibility of the disallowed call. This is > a situation from which no amount of wriggling will let you escape. > ~ Grattan ~ +=+ > No Grattan, now you are throwing together two quite different rulings. If the call is deemed systemic (at whatever level) _and_ if the system regulations tell us they apply (not necessarily the case IMO) _and_ if the call falls within those banned by the system regulations (normally the case for weak calls), THEN the call is banned by the system regulations. I have no problems with that - except with all the provisos (but that's another discussion). If all the above is satisfied, then indeed the call ought not to have been made and an adjustment is based on it not being made (leading in most but not all instances to an artificial AS). What we are talking about is a call being infractious of L40C1 - it being a deviation of which the partner is deemed to be more aware than the opponents. You seemed to be saying that under this law alone, a call can be deemed illegal, and an adjustment also based on the call not being made. I disagree with that view because when you deem that a deviation has become "expected", you yourself insist that it becomes partnership understanding. Well, that partnership understanding has not been revealed and this infraction should be punished in the same way as any missing alert or alike, that is with an adjustment being based on the call being properly disclosed. Please tell me if I understood your previous posts correctly and you do indeed believe as I stated, or whether I am right after all. > > Herman. From grandaeval at tiscali.co.uk Sun Nov 9 16:00:36 2008 From: grandaeval at tiscali.co.uk (Grattan) Date: Sun, 9 Nov 2008 15:00:36 -0000 Subject: [blml] Deviations from announced system. References: <003501c940de$dfd49c60$0302a8c0@Mildred> <4914532F.1080100@skynet.be> <49145A04.2000707@ulb.ac.be> <49146907.9020204@skynet.be><49146E3D.8050904@ulb.ac.be> <49149C32.7050005@skynet.be><003701c94190$28575d40$0302a8c0@Mildred> <4916C87B.1080105@skynet.be> Message-ID: <000901c9427b$f28876d0$0302a8c0@Mildred> Grattan Endicott To: "Bridge Laws Mailing List" Sent: Sunday, November 09, 2008 11:24 AM Subject: Re: [blml] Deviations from announced system. No Grattan, now you are throwing together two quite different rulings. If the call is deemed systemic (at whatever level) _and_ if the system regulations tell us they apply (not necessarily the case IMO) _and_ if the call falls within those banned by the system regulations (normally the case for weak calls), THEN the call is banned by the system regulations. > +=+ I am not really mixing into the irrelevant 'discussion' that you are keeping alive. I am simply issuing constant reminders that if there is a breach of Law 40A3 because the player makes a call that is based on an undisclosed partnership understanding, that call is illegal. The player will not then be allowed use of the call and score adjustment will not, I say again *will not*, take any account of possible actions ensuing by way of the illegal call. It is too late and absolutely against the law for the Director to take account of what could have happened if opponents had been told of the partnership understanding. ~ Grattan ~ +=+ From grandaeval at tiscali.co.uk Sun Nov 9 16:18:55 2008 From: grandaeval at tiscali.co.uk (Grattan) Date: Sun, 9 Nov 2008 15:18:55 -0000 Subject: [blml] Deviations from announced system. References: <003501c940de$dfd49c60$0302a8c0@Mildred> <4914532F.1080100@skynet.be> <49145A04.2000707@ulb.ac.be> <49146907.9020204@skynet.be><49146E3D.8050904@ulb.ac.be> <49149C32.7050005@skynet.be><003701c94190$28575d40$0302a8c0@Mildred> <4916C87B.1080105@skynet.be> Message-ID: <001701c9427e$7e4d08a0$0302a8c0@Mildred> Grattan Endicott To: "Bridge Laws Mailing List" Sent: Sunday, November 09, 2008 11:24 AM Subject: Re: [blml] Deviations from announced system. You seemed to be saying that under this law alone, a call can be deemed illegal, and an adjustment also based on the call not being made. I disagree with that view because when you deem that a deviation has become "expected", you yourself insist that it becomes partnership understanding. Well, that partnership understanding has not been revealed and this infraction should be punished in the same way as any missing alert or alike, that is with an adjustment being based on the call being properly disclosed. > +=+Herman, while I note your 'disagreement' I discount it because it is based upon ignorance of the law. We have to deal with the law in the book, not with Herman's fantasy law. It is an infraction to make a call that violates Law 40A3. I say again *to make* the call. Rectification must take the action back to the point immediately before the call is made because the violation to be rectified is the making of the illegal call. ~ Grattan ~ +=+ From harsanyi at t-online.de Sun Nov 9 19:18:23 2008 From: harsanyi at t-online.de (Josef Harsanyi) Date: Sun, 9 Nov 2008 19:18:23 +0100 Subject: [blml] AC in Amsterdam In-Reply-To: <000901c9427b$f28876d0$0302a8c0@Mildred> References: <003501c940de$dfd49c60$0302a8c0@Mildred> <4914532F.1080100@skynet.be> <49145A04.2000707@ulb.ac.be> <49146907.9020204@skynet.be><49146E3D.8050904@ulb.ac.be> <49149C32.7050005@skynet.be><003701c94190$28575d40$0302a8c0@Mildred> <4916C87B.1080105@skynet.be> <000901c9427b$f28876d0$0302a8c0@Mildred> Message-ID: <000901c94297$8dc606b0$a9521410$@de> http://www.eurobridge.org/bulletin/08_3%20Amsterdam/pdf/Bul_04.pdf You are south: 3s-p-p-x P -? D5 9 AJ1096(4) K942 LA= pass / 3NT/ 4s/4NT, ? Which kind of extraneous information(s) could be propagated by a slow double? Which alternative calls are suggested for East by a delayed double (if any)? The singular alternative calls :pass/ 3NT/ 4s/4NT, in which portion are influenced by a slow double? How to rule as a TD? How should (if at all) be adjusted the decision of TD by the AC, if the TD decided for score stands? Thanks for the answers in advance. Josef From richard.hills at immi.gov.au Mon Nov 10 00:59:15 2008 From: richard.hills at immi.gov.au (richard.hills at immi.gov.au) Date: Mon, 10 Nov 2008 10:59:15 +1100 Subject: [blml] AC in Amsterdam [SEC=UNOFFICIAL] In-Reply-To: <000901c94297$8dc606b0$a9521410$@de> Message-ID: Deal rotated 90 degrees for convenience Imps Dlr: East Vul: Both The bidding has gone: WEST NORTH EAST SOUTH --- --- 3S Pass Pass ...X Pass ? You, South, hold: Q5 9 AJT964 K942 Josef Harsanyi asked: >LA= pass / 3NT/ 4s/4NT, ? >Which kind of extraneous information(s) could be propagated >by a slow double? >Which alternative calls are suggested for South by a delayed >double (if any)? >The singular alternative calls :pass/ 3NT/ 4s/4NT, in which >portion are influenced by a slow double? >How to rule as a TD? >How should (if at all) be adjusted the decision of TD by >the AC, if the TD decided for score stands? > >Thanks for the answers in advance. >Josef Richard Hills: (1) In my opinion a Pass of the takeout double is not an LA with only two spades and an easy alternative of bidding the six-card diamond suit. Given the disparity in strength and length of South's minors, in my opinion a unilateral 5D is superior to a choice-of-minors 4NT (especially since 5D in a 6-2 fit is likely to play at least one trick better than 5C in a 4-3 fit). A conservative 4D also has merit in my opinion. (2) A slow takeout double transmits UI that pard does not have an obvious takeout double, thus suggesting that pard is nowhere near the classical 1-4-4-4 shape, but might instead hold, for example, 2-5-2-4 shape with what pard thinks is a wasted doubleton king of spades. (3) 3NT by South would be silly if pard had doubled with the classical 1-4-4-4 shape and pard's singleton was the jack or lower. But a slow double suggests pard is more likely to hold some spade length (e.g. Jxx) or spade strength (e.g. Kx). (4) As TD I adjust the score from 3NT making +600 to a weighted score of 25% of 4D making +130 and 75% of 5D failing -100. (5) While an AC should initially presume that the TD has made the correct ruling, the AC does have the power to over-rule the TD's assessment of fact if the AC gains more evidence than was available to the TD. See the WBF Code of Practice for Appeals Committees. Best wishes Richard James Hills Recruitment Section, Level 3 Blue, workstation 15 (first on left) Department of Immigration and Citizenship Telephone: 02 6223 8453 Email: richard.hills at immi.gov.au -------------------------------------------------------------------- Important Notice: If you have received this email by mistake, please advise the sender and delete the message and attachments immediately. This email, including attachments, may contain confidential, sensitive, legally privileged and/or copyright information. Any review, retransmission, dissemination or other use of this information by persons or entities other than the intended recipient is prohibited. DIAC respects your privacy and has obligations under the Privacy Act 1988. The official departmental privacy policy can be viewed on the department's website at www.immi.gov.au. See: http://www.immi.gov.au/functional/privacy.htm --------------------------------------------------------------------- From richard.hills at immi.gov.au Mon Nov 10 02:20:18 2008 From: richard.hills at immi.gov.au (richard.hills at immi.gov.au) Date: Mon, 10 Nov 2008 12:20:18 +1100 Subject: [blml] New Laws don't allow UI situation to be resolved? [SEC=UNOFFICIAL] In-Reply-To: <001a01c93c5b$6d020830$0302a8c0@Mildred> Message-ID: Robert Frick: >>If partner supplies the UI, I understand that I cannot and >>should not choose an LA suggested by the UI. But if the UI >>is extraneous and it is not my fault that I have it, then >>it does not seem fair if I am ethically obligated not to >>select alternatives suggested by the UI. Grattan Endicott: >+=+ Perhaps Robert should look again at Law 16A3. It is not >a matter of ethics that he is addressing but a matter of >law. > ~ Grattan ~ +=+ Richard Hills: Perhaps Grattan should look again at Law 16A3 -> "No player may base a call or play on other information (such information being designated extraneous).' It seems to me player may choose a call ***suggested*** by EI not supplied by partner provided that the player does not ***base*** that call on the EI. That is, if: (a) there are two logical alternatives A and B, and (b) the player would normally choose logical alternative A, and (c) the unauthorised information demonstrably suggests A, then (d1) if the UI came from partner B must be selected, but (d2) if the EI came from the next table A may be selected, since calling A is not "based" on the EI but rather "based" on what the player would normally choose. Of course, in case (d2) the TD should award an adjusted score under Law 16C2(c), ruling both sides as nonoffending. What's the problem? Best wishes Richard James Hills Recruitment Section, Level 3 Blue, workstation 15 (first on left) Department of Immigration and Citizenship Telephone: 02 6223 8453 Email: richard.hills at immi.gov.au -------------------------------------------------------------------- Important Notice: If you have received this email by mistake, please advise the sender and delete the message and attachments immediately. This email, including attachments, may contain confidential, sensitive, legally privileged and/or copyright information. Any review, retransmission, dissemination or other use of this information by persons or entities other than the intended recipient is prohibited. DIAC respects your privacy and has obligations under the Privacy Act 1988. The official departmental privacy policy can be viewed on the department's website at www.immi.gov.au. See: http://www.immi.gov.au/functional/privacy.htm --------------------------------------------------------------------- From richard.hills at immi.gov.au Mon Nov 10 08:27:50 2008 From: richard.hills at immi.gov.au (richard.hills at immi.gov.au) Date: Mon, 10 Nov 2008 18:27:50 +1100 Subject: [blml] Mr Gladstone suggested [SEC=UNOFFICIAL] In-Reply-To: <490DB7E5.3030608@skynet.be> Message-ID: Michael Foot (1913- ), former British Labour leader: "Disraeli was my favourite Tory. He was an adventurer pure and simple, or impure and complex. I'm glad to say Gladstone got the better of him." Grattan Endicott: >>+=+ The pot calling the kettle black.+=+ Herman De Wael: [snip] >In the past however, I have often felt that my arguments >go unanswered. Richard Hills: It is true that Herman's repeated requests for an official WBF Laws Committee interpretation of the 1997 Law 75D2 (now the 2007 Law 20F5(a)) went unanswered for a decade. But last month the WBF LC finally elected to publish a comprehensive answer -- alas, not the answer that Herman De Wael wanted. Herman De Wael: >OTOH, I have never shied away from answering other >people's arguments and showing why they are wrong. > >I really don't feel like a kettle, or a pot. Richard Hills: Petitio principii, begging the question. Merely because I might think I have proved Herman wrong does not mean that I have actually proved Herman wrong. Likewise, merely because Herman might think he has proved me wrong does not mean that Herman has actually proved me wrong. Indeed, on this thread's particular topic of deviation from partnership understandings Herman De Wael's logic has been truly impeccable, satisfying all the requirements of Euclidean and Aristotelean proof. But I am still a pot calling a kettle black, arguing that I and Sven Pran have shown Herman De Wael to be wrong. Walt Whitman (1819-1892): "Do I contradict myself? Very well then I contradict myself, (I am large, I contain multitudes.)" Richard Hills: Despite Steve Willner also approving of the internal consistency of Herman De Wael's logic, ultimately all logical reasoning about a particular topic is based on an unprovable set of axioms. Sven Pran and I prefer to have our unprovable set of axioms aligned with the intent of the Drafting Committee. Which is why Sven Pran and I have the unprovable belief that we are right, and that Steve Willner and Herman De Wael are wrong. Steve Willner, 11th February 2008: [snip] >>To start that off, I'm going to list six "schools" that >>Laws or interpretations could establish for the situation >>when one player has given MI. [snip] >>If the DSC is serious about issuing an interpretation, I >>_hope_ filling out a table like this one is how they are >>going about it. Sven Pran, 11th February 2008: >Had the submission below come from Herman I would just have >ignored it, but it worries me when such issues come from >other sources. > >There is an established principle at least 75 years old that >a partnership shall always give their opponents full and >correct description of any of their methods. This is only >crossed by the rule that a player may not inform his partner >that he (the partner) has (in the player's opinion) given >incorrect information to opponents. Various laws for bridge >have expressed this principle essentially unchanged since >1932. [snip] >No player should ever worry about UI communicated to his >partner when answering questions from opponents. Such UI is >partner's worry and nobody else's. > >The activities by Herman who I understand is a senior officer >in the Belgian NBOs are extremely dangerous just because of >his position, as they can mislead other players to >deliberately violate some of the most essential laws for >bridge. Such players may claim ignorance of the laws, but that >ignorance is inexcusable and unforgivable, and Herman should >really be the first person to teach players the laws rather >than promoting his own conflicting ideas to them. Best wishes Richard James Hills Recruitment Section, Level 3 Blue, workstation 15 (first on left) Department of Immigration and Citizenship Telephone: 02 6223 8453 Email: richard.hills at immi.gov.au -------------------------------------------------------------------- Important Notice: If you have received this email by mistake, please advise the sender and delete the message and attachments immediately. This email, including attachments, may contain confidential, sensitive, legally privileged and/or copyright information. Any review, retransmission, dissemination or other use of this information by persons or entities other than the intended recipient is prohibited. DIAC respects your privacy and has obligations under the Privacy Act 1988. The official departmental privacy policy can be viewed on the department's website at www.immi.gov.au. See: http://www.immi.gov.au/functional/privacy.htm --------------------------------------------------------------------- From grandaeval at tiscali.co.uk Mon Nov 10 08:28:39 2008 From: grandaeval at tiscali.co.uk (Grattan) Date: Mon, 10 Nov 2008 07:28:39 -0000 Subject: [blml] AC in Amsterdam References: <003501c940de$dfd49c60$0302a8c0@Mildred> <4914532F.1080100@skynet.be> <49145A04.2000707@ulb.ac.be> <49146907.9020204@skynet.be><49146E3D.8050904@ulb.ac.be> <49149C32.7050005@skynet.be><003701c94190$28575d40$0302a8c0@Mildred> <4916C87B.1080105@skynet.be><000901c9427b$f28876d0$0302a8c0@Mildred> <000901c94297$8dc606b0$a9521410$@de> Message-ID: <000801c94305$f9b42500$0302a8c0@Mildred> Grattan Endicott To: "'Bridge Laws Mailing List'" Sent: Sunday, November 09, 2008 6:18 PM Subject: [blml] AC in Amsterdam > http://www.eurobridge.org/bulletin/08_3%20Amsterdam/pdf/Bul_04.pdf > You are south: > 3s-p-p-x > P -? > > D5 > 9 > AJ1096(4) > K942 > > LA= pass / 3NT/ 4s/4NT, ? > Which kind of extraneous information(s) could be propagated by a slow > double? > Which alternative calls are suggested for East by a delayed double (if > any)? > The singular alternative calls :pass/ 3NT/ 4s/4NT, in which portion are > influenced by a slow double? > How to rule as a TD? > How should (if at all) be adjusted the decision of TD by the AC, if the > TD > decided for score stands? > > Thanks for the answers in advance. > Josef > +=+ The question is tricky. I would expect a hesitant double to suggest questionable shape for a take-out, could include maybe four shabby Spades and a suit shortage somewhere else. Pass not acceptable - if 4NT is for minors OK, more likely some quantity of diamonds - maybe five or a more conservative four. If Spades turn out to have trump tricks a Pass would show too much mutual understanding. Not 3NT. Score adjustment if required? Maybe weighted between 4/5 diamonds (but on some holdings consider whether doubler would remove 4D and what might follow). ~ Grattan ~ +=+ From harsanyi at t-online.de Mon Nov 10 08:29:28 2008 From: harsanyi at t-online.de (Josef Harsanyi) Date: Mon, 10 Nov 2008 08:29:28 +0100 Subject: [blml] AC in Amsterdam [SEC=UNOFFICIAL] In-Reply-To: References: <000901c94297$8dc606b0$a9521410$@de> Message-ID: <000701c94306$11156790$334036b0$@de> Thank for the answer Richard, After 2s -pass - pass -dbl pass -? The pass is a LA, but only if the years of the pair is above 135. After 4s--pass - pass -dbl pass -? The pass is very probable also from pairs with a common age of 40 years Our case 3s--pass - pass -dbl pass -? Is very case sensitive I think. Partnerships with over 120 years together will seriously deal with the thought of a penalty pass. A Taylor-made score should consider also the bidding style of the players? Some experienced players are eager to write 800 against youngsters. Best regards Josef -----Urspr?ngliche Nachricht----- Von: blml-bounces at amsterdamned.org [mailto:blml-bounces at amsterdamned.org] Im Auftrag von richard.hills at immi.gov.au Gesendet: Montag, 10. November 2008 00:59 An: Bridge Laws Mailing List Betreff: Re: [blml] AC in Amsterdam [SEC=UNOFFICIAL] Deal rotated 90 degrees for convenience Imps Dlr: East Vul: Both The bidding has gone: WEST NORTH EAST SOUTH --- --- 3S Pass Pass ...X Pass ? You, South, hold: Q5 9 AJT964 K942 Josef Harsanyi asked: >LA= pass / 3NT/ 4s/4NT, ? >Which kind of extraneous information(s) could be propagated >by a slow double? >Which alternative calls are suggested for South by a delayed >double (if any)? >The singular alternative calls :pass/ 3NT/ 4s/4NT, in which >portion are influenced by a slow double? >How to rule as a TD? >How should (if at all) be adjusted the decision of TD by >the AC, if the TD decided for score stands? > >Thanks for the answers in advance. >Josef Richard Hills: (1) In my opinion a Pass of the takeout double is not an LA with only two spades and an easy alternative of bidding the six-card diamond suit. Given the disparity in strength and length of South's minors, in my opinion a unilateral 5D is superior to a choice-of-minors 4NT (especially since 5D in a 6-2 fit is likely to play at least one trick better than 5C in a 4-3 fit). A conservative 4D also has merit in my opinion. (2) A slow takeout double transmits UI that pard does not have an obvious takeout double, thus suggesting that pard is nowhere near the classical 1-4-4-4 shape, but might instead hold, for example, 2-5-2-4 shape with what pard thinks is a wasted doubleton king of spades. (3) 3NT by South would be silly if pard had doubled with the classical 1-4-4-4 shape and pard's singleton was the jack or lower. But a slow double suggests pard is more likely to hold some spade length (e.g. Jxx) or spade strength (e.g. Kx). (4) As TD I adjust the score from 3NT making +600 to a weighted score of 25% of 4D making +130 and 75% of 5D failing -100. (5) While an AC should initially presume that the TD has made the correct ruling, the AC does have the power to over-rule the TD's assessment of fact if the AC gains more evidence than was available to the TD. See the WBF Code of Practice for Appeals Committees. Best wishes Richard James Hills Recruitment Section, Level 3 Blue, workstation 15 (first on left) Department of Immigration and Citizenship Telephone: 02 6223 8453 Email: richard.hills at immi.gov.au -------------------------------------------------------------------- Important Notice: If you have received this email by mistake, please advise the sender and delete the message and attachments immediately. This email, including attachments, may contain confidential, sensitive, legally privileged and/or copyright information. Any review, retransmission, dissemination or other use of this information by persons or entities other than the intended recipient is prohibited. DIAC respects your privacy and has obligations under the Privacy Act 1988. The official departmental privacy policy can be viewed on the department's website at www.immi.gov.au. See: http://www.immi.gov.au/functional/privacy.htm --------------------------------------------------------------------- _______________________________________________ blml mailing list blml at amsterdamned.org http://www.amsterdamned.org/mailman/listinfo/blml From JffEstrsn at aol.com Mon Nov 10 10:25:45 2008 From: JffEstrsn at aol.com (Jeff Easterson) Date: Mon, 10 Nov 2008 10:25:45 +0100 Subject: [blml] Ac in Amasterdam Message-ID: <4917FE19.9010405@aol.com> blmlers who commented on the AC Amsterdam case might be interested to hear that, according to the reports I have: 1) the hesitation was not "discovered" until after the round/session had ended. 2) A TD was not called; at least there was no TD decision. 3) The appeal was only registered after two other appeals by the Italians had been rejected. 4) The AC first said it could only make a decision after hearing the report of the table TD. 5) Some time later the AC announced its decision, changing the score (that is, accepting the argument of the Italians). The Germans were not heard, in fact they were not even informed that the AC was deliberating. The AC decided without a report from the table TD since he apparently was never called and was unaware of the case. The Germans were told that in such a case they had no right to be heard. Would any of this influence you or change your opinion? (I agree that 3NT is suggested by the hesitation, if there was a hesitation. But since when does an AC decide on the basis of statements by one side without hearing the other or a TD?) I give no guarantee that the "facts" as stated above. They are, as I recall, what Harsanyi and Grom?ller reported. Perhaps one of them could read this and make, if necessary, corrections. JE From Harsanyi at t-online.de Mon Nov 10 10:50:05 2008 From: Harsanyi at t-online.de (Harsanyi at t-online.de) Date: Mon, 10 Nov 2008 10:50:05 +0100 Subject: [blml] =?iso-8859-15?q?Ac_in_Amasterdam?= In-Reply-To: <4917FE19.9010405@aol.com> References: <4917FE19.9010405@aol.com> Message-ID: <1KzTPJ-1coLfE0@fwd00.aul.t-online.de> An HTML attachment was scrubbed... URL: http://www.amsterdamned.org/pipermail/blml/attachments/20081110/052ae2d2/attachment.htm From t.kooyman at worldonline.nl Mon Nov 10 10:59:10 2008 From: t.kooyman at worldonline.nl (ton) Date: Mon, 10 Nov 2008 10:59:10 +0100 Subject: [blml] Ac in Amasterdam In-Reply-To: <4917FE19.9010405@aol.com> Message-ID: blmlers who commented on the AC Amsterdam case might be interested to hear that, according to the reports I have: 1) the hesitation was not "discovered" until after the round/session had ended. 2) A TD was not called; at least there was no TD decision. 3) The appeal was only registered after two other appeals by the Italians had been rejected. 4) The AC first said it could only make a decision after hearing the report of the table TD. 5) Some time later the AC announced its decision, changing the score (that is, accepting the argument of the Italians). The Germans were not heard, in fact they were not even informed that the AC was deliberating. The AC decided without a report from the table TD since he apparently was never called and was unaware of the case. The Germans were told that in such a case they had no right to be heard. Would any of this influence you or change your opinion? (I agree that 3NT is suggested by the hesitation, if there was a hesitation. But since when does an AC decide on the basis of statements by one side without hearing the other or a TD?) I give no guarantee that the "facts" as stated above. They are, as I recall, what Harsanyi and Grom?ller reported. Perhaps one of them could read this and make, if necessary, corrections. JE ton: I guess that the source for this information is Grom?ller and do not expect corrections from him. Since I was present my description of the facts could be worth something. Yes, the Italian team reported this case late (to me), but still within the protest time. At that moment the appeal committee was dealing with two other appeals for which both sides were heard at that same moment, both TD's being present. There was no decision made on any of those yet (first wrong statement). After finishing the hearing I told the AC about this third case and I sent both teams to the AC for a further hearing (second wrong statement). That hearing took place and both sides got the possibility to give their opinion. Nobody at that stage told the Germans that they had no right to be heard (third wrong statement). Then the AC asked the TD's and me whether it was necessary to have a decision from the TD's on this case before they could handle it. Our answer was 'no'. Only then the AC started discussing all three cases. The outcome was that they changed the result on the third case as reported. After being informed the Germans wanted to appeal and at that moment I told them that a further step in this case was not possible, the decision of the AC being final. ton From grandaeval at tiscali.co.uk Mon Nov 10 11:08:36 2008 From: grandaeval at tiscali.co.uk (Grattan) Date: Mon, 10 Nov 2008 10:08:36 -0000 Subject: [blml] Ac in Amasterdam References: <4917FE19.9010405@aol.com> Message-ID: <001f01c9431c$94ea0600$0302a8c0@Mildred> Grattan Endicott To: "Bridge Laws Mailing List" Sent: Monday, November 10, 2008 9:25 AM Subject: [blml] Ac in Amasterdam blmlers who commented on the AC Amsterdam case might be interested to hear that, according to the reports I have: 1) the hesitation was not "discovered" until after the round/session had ended. 2) A TD was not called; at least there was no TD decision. 3) The appeal was only registered after two other appeals by the Italians had been rejected. 4) The AC first said it could only make a decision after hearing the report of the table TD. 5) Some time later the AC announced its decision, changing the score (that is, accepting the argument of the Italians). The Germans were not heard, in fact they were not even informed that the AC was deliberating. The AC decided without a report from the table TD since he apparently was never called and was unaware of the case. The Germans were told that in such a case they had no right to be heard. Would any of this influence you or change your opinion? (I agree that 3NT is suggested by the hesitation, if there was a hesitation. But since when does an AC decide on the basis of statements by one side without hearing the other or a TD?) I give no guarantee that the "facts" as stated above. They are, as I recall, what Harsanyi and Grom?ller reported. Perhaps one of them could read this and make, if necessary, corrections. JE +=+ An appeal is an appeal of a Director's ruling. If there is no Director's ruling there is nothing to appeal. An appeals committee has no power to make a ruling of first instance. See Law 92C; the AC may have misdirected itself as to its power to hear the appeal and the decision should be overturned if this were the case, upon further appeal to the Regulating Authority. I do not know the timing of events, whatever the events were. The question will arise as to whether the appeal was through the Director and in time. ~ Grattan ~ +=+ From Hermandw at skynet.be Mon Nov 10 11:14:51 2008 From: Hermandw at skynet.be (Herman De Wael) Date: Mon, 10 Nov 2008 11:14:51 +0100 Subject: [blml] Mr Gladstone suggested [SEC=UNOFFICIAL] In-Reply-To: References: Message-ID: <4918099B.8050600@skynet.be> Hello Richard. Please don't go turning my words around, and don't use false logic. richard.hills at immi.gov.au wrote: > > Herman De Wael: > >> In the past however, I have often felt that my arguments >> go unanswered. > > Richard Hills: > > It is true that Herman's repeated requests for an official > WBF Laws Committee interpretation of the 1997 Law 75D2 > (now the 2007 Law 20F5(a)) went unanswered for a decade. > Not exactly what I meant, but OK. > But last month the WBF LC finally elected to publish a > comprehensive answer -- alas, not the answer that Herman > De Wael wanted. > Absolutely not comprehensive. I doubt if the meeting took more than a minute to approve the text Grattan had obviously prepared. And indeed not what I wanted, but also not what is needed. I see no reason to change my intended habit if it ever came along at my particular table. No sense in saying what a player should do if there is no penalty to back it up. > Herman De Wael: > >> OTOH, I have never shied away from answering other >> people's arguments and showing why they are wrong. >> >> I really don't feel like a kettle, or a pot. > > Richard Hills: > > Petitio principii, begging the question. Merely because I > might think I have proved Herman wrong does not mean that > I have actually proved Herman wrong. Likewise, merely > because Herman might think he has proved me wrong does not > mean that Herman has actually proved me wrong. > Richard, you have NEVER, ever, even attempted to prove me wrong. You have repeated the same arguments over and over again, and I have over and over countered your arguments. When I give arguments of my own, you have often not even responded. I'll just wait for the next spin of the wheel to point you to some of my answers that you haven't responded to. But of course that is only what I think of the issue. > Indeed, on this thread's particular topic of deviation from > partnership understandings Herman De Wael's logic has been > truly impeccable, satisfying all the requirements of > Euclidean and Aristotelean proof. But I am still a pot > calling a kettle black, arguing that I and Sven Pran have > shown Herman De Wael to be wrong. > How? > Walt Whitman (1819-1892): > > "Do I contradict myself? > Very well then I contradict myself, > (I am large, I contain multitudes.)" > > Richard Hills: > > Despite Steve Willner also approving of the internal > consistency of Herman De Wael's logic, ultimately all > logical reasoning about a particular topic is based on an > unprovable set of axioms. Sven Pran and I prefer to have > our unprovable set of axioms aligned with the intent of > the Drafting Committee. Which is why Sven Pran and I have > the unprovable belief that we are right, and that Steve > Willner and Herman De Wael are wrong. > I would very much like to get to the bottom of this Richard, and pull awy the foundations of the axioms that you think you are using. Neither of us can actually use any axioms, since there aren't any. But even without them, a construction can be so wobbly as to fall apart when a storm hits it. > Steve Willner, 11th February 2008: > > [snip] > >>> To start that off, I'm going to list six "schools" that >>> Laws or interpretations could establish for the situation >>> when one player has given MI. > > [snip] > >>> If the DSC is serious about issuing an interpretation, I >>> _hope_ filling out a table like this one is how they are >>> going about it. > > Sven Pran, 11th February 2008: > >> Had the submission below come from Herman I would just have >> ignored it, but it worries me when such issues come from >> other sources. >> >> There is an established principle at least 75 years old that >> a partnership shall always give their opponents full and >> correct description of any of their methods. This is only >> crossed by the rule that a player may not inform his partner >> that he (the partner) has (in the player's opinion) given >> incorrect information to opponents. Various laws for bridge >> have expressed this principle essentially unchanged since >> 1932. > Is this trying to prove me right? It is exactly the basis of what I would say. > [snip] > >> No player should ever worry about UI communicated to his >> partner when answering questions from opponents. Such UI is >> partner's worry and nobody else's. >> Which is a direct antithesis of the sentence above. >> This is only >> crossed by the rule that a player may not inform his partner >> that he (the partner) has (in the player's opinion) given >> incorrect information to opponents. Is precisely the reason why the player should in fact worry about UI communicated. The WBF tells him not to do so. Of course he should worry about doing so. >> The activities by Herman who I understand is a senior officer >> in the Belgian NBOs are extremely dangerous just because of >> his position, as they can mislead other players to >> deliberately violate some of the most essential laws for >> bridge. Such players may claim ignorance of the laws, but that >> ignorance is inexcusable and unforgivable, and Herman should >> really be the first person to teach players the laws rather >> than promoting his own conflicting ideas to them. > The only reason you are writing this is because we disagree. Do I state that I worry about the Australian Bridge players who are obliged to divulge UI to their partners. Richard, have you even thought of the following three questions: 1) How do you feel about a law that obliges you to give UI to your partner? 2) As TD, if there is a law that obliges someone to give UI to his partner, how do you rule if the player in front of you has refused to do so? 3) Quiz question: how many laws in the 2007 Lawbook explicitely forbid giving a piece of UI to partner? And how many oblige a player to do so? Answers on a postcard, please. > > Best wishes > > Richard James Hills Herman. From Hermandw at skynet.be Mon Nov 10 11:16:23 2008 From: Hermandw at skynet.be (Herman De Wael) Date: Mon, 10 Nov 2008 11:16:23 +0100 Subject: [blml] Deviations from announced system. In-Reply-To: <000901c9427b$f28876d0$0302a8c0@Mildred> References: <003501c940de$dfd49c60$0302a8c0@Mildred> <4914532F.1080100@skynet.be> <49145A04.2000707@ulb.ac.be> <49146907.9020204@skynet.be><49146E3D.8050904@ulb.ac.be> <49149C32.7050005@skynet.be><003701c94190$28575d40$0302a8c0@Mildred> <4916C87B.1080105@skynet.be> <000901c9427b$f28876d0$0302a8c0@Mildred> Message-ID: <491809F7.7060803@skynet.be> Grattan, you told us three times that a call is inadmissible, and I asked you four times what the ruling is. You have never replied that question, and again not here. Herman. Grattan wrote: > > Grattan Endicott also ************************************** > "Let us then be up and doing, > With a heart for any fate, > Still achieving, still pursuing, > Learn to labour and to wait." > [H.W. Longfellow] > > ''''''''''''''''''''''''''''''''''''''''''''''''''''''''''''''''''''''''''''''''''''''''''''''''' > > > > ----- Original Message ----- > From: "Herman De Wael" > To: "Bridge Laws Mailing List" > Sent: Sunday, November 09, 2008 11:24 AM > Subject: Re: [blml] Deviations from announced system. > > > No Grattan, now you are throwing together two quite > different rulings. If the call is deemed systemic (at whatever > level) _and_ if the system regulations tell us they apply (not > necessarily the case IMO) _and_ if the call falls within those > banned by the system regulations (normally the case for weak > calls), THEN the call is banned by the system regulations. > +=+ I am not really mixing into the irrelevant 'discussion' that > you are keeping alive. I am simply issuing constant reminders > that if there is a breach of Law 40A3 because the player makes > a call that is based on an undisclosed partnership understanding, > that call is illegal. The player will not then be allowed use of > the call and score adjustment will not, I say again *will not*, > take any account of possible actions ensuing by way of the > illegal call. It is too late and absolutely against the law for the > Director to take account of what could have happened if > opponents had been told of the partnership understanding. > ~ Grattan ~ +=+ > From Hermandw at skynet.be Mon Nov 10 11:21:27 2008 From: Hermandw at skynet.be (Herman De Wael) Date: Mon, 10 Nov 2008 11:21:27 +0100 Subject: [blml] Deviations from announced system. In-Reply-To: <001701c9427e$7e4d08a0$0302a8c0@Mildred> References: <003501c940de$dfd49c60$0302a8c0@Mildred> <4914532F.1080100@skynet.be> <49145A04.2000707@ulb.ac.be> <49146907.9020204@skynet.be><49146E3D.8050904@ulb.ac.be> <49149C32.7050005@skynet.be><003701c94190$28575d40$0302a8c0@Mildred> <4916C87B.1080105@skynet.be> <001701c9427e$7e4d08a0$0302a8c0@Mildred> Message-ID: <49180B27.9030002@skynet.be> Grattan wrote: > > Grattan Endicott also ************************************** > "Let us then be up and doing, > With a heart for any fate, > Still achieving, still pursuing, > Learn to labour and to wait." > [H.W. Longfellow] > > ''''''''''''''''''''''''''''''''''''''''''''''''''''''''''''''''''''''''''''''''''''''''''''''''' > > > > ----- Original Message ----- > From: "Herman De Wael" > To: "Bridge Laws Mailing List" > Sent: Sunday, November 09, 2008 11:24 AM > Subject: Re: [blml] Deviations from announced system. > > > You seemed to be saying that under this law alone, a call > can be deemed illegal, and an adjustment also based on > the call not being made. I disagree with that view because > when you deem that a deviation has become "expected", > you yourself insist that it becomes partnership understanding. > Well, that partnership understanding has not been revealed > and this infraction should be punished in the same way as any > missing alert or alike, that is with an adjustment being based > on the call being properly disclosed. > +=+Herman, while I note your 'disagreement' I discount it > because it is based upon ignorance of the law. We have to > deal with the law in the book, not with Herman's fantasy law. > It is an infraction to make a call that violates Law 40A3. I say > again *to make* the call. Rectification must take the action > back to the point immediately before the call is made because > the violation to be rectified is the making of the illegal call. > ~ Grattan ~ +=+ > OK Grattan, I accept. Now will you please tell us in what class Helgemo's 1H is supposed to be? Is it a system that is badly explained (10-19 in stead of 8-19)? Or is it a illegal deviation from a system? In any case suppose that Helness knows Helgemo can open on 8 points and the opponents don't. According to you, the adjustments in both cases are radically different. How shall the TD decide which of the two laws to use? Please answer this one, Grattan, since it is important. And please stop belittling me. I understand as much of the laws as I am told. This is absolutely unclear in the laws, and just reading them leads to at least two different interpretations. So far, I have not yet criticised the WBF for this, as I realize this is a tough job. But then when you have failed to make something clear, don't criticise us for not understanding immediately what you have in your mind. Herman. From agot at ulb.ac.be Mon Nov 10 11:42:06 2008 From: agot at ulb.ac.be (Alain Gottcheiner) Date: Mon, 10 Nov 2008 11:42:06 +0100 Subject: [blml] AC in Amsterdam [SEC=UNOFFICIAL] In-Reply-To: References: Message-ID: <49180FFE.4020304@ulb.ac.be> richard.hills at immi.gov.au a ?crit : > Deal rotated 90 degrees for convenience > Imps > Dlr: East > Vul: Both > > The bidding has gone: > > WEST NORTH EAST SOUTH > --- --- 3S Pass > Pass ...X Pass ? > > You, South, hold: > > Q5 > 9 > AJT964 > K942 > > Josef Harsanyi asked: > > >> LA= pass / 3NT/ 4s/4NT, ? >> Surely 5D, too. Probably the choice is between the greedy 3NT, the simple 5D and the ambitious (a bit too ambitious IMHO) 4NT followed by 5D over 5C (but what over 5D ?) Even 4D may be, if one fears 7222 spades and a lightish reopening. Passing doesn't seem right, but perhaps the criteria for 'logical alternative' will be met. >> Which kind of extraneous information(s) could be propagated >> by a slow double? >> Very few. Umpteen reasons may exist for a solw double : a- weakish hand, fearing too much enthousiasm from partner b- ideas about 3NT, perhaps 3433 pattern without a stopper c- spade void, fearing a leave-in d- too much for a 3NT bid e- strange pattern e.g. 1426, but no convenient bid (say if 4C reopening is 55 in their system) A slow middle-of-the-road, nebulous, action seldom suggests anything specific, whence L16 won't be needed, and most takeout doubles pertain to this class. >> Which alternative calls are suggested for South by a delayed >> double (if any)? >> None is suggested. Perhaps 4D would have good chances, to let partner off the hook in a), and let him express his hand in d) and e), BTA it would be a bad choice in b) and c), so it's marginal. Or perhaps 3NT would be successful if partner has an habit of having difficult choices between double and NT (playing him for the likes of Kxx-AKxx-xxx-Axx). But we aren't told that. But they don't meet the criteria for 'unmistakably suggested'. Best regards Alain From grandaeval at tiscali.co.uk Mon Nov 10 11:41:56 2008 From: grandaeval at tiscali.co.uk (Grattan) Date: Mon, 10 Nov 2008 10:41:56 -0000 Subject: [blml] Ac in Amasterdam References: <4917FE19.9010405@aol.com> <1KzTPJ-1coLfE0@fwd00.aul.t-online.de> Message-ID: <003601c94320$f5d5be10$0302a8c0@Mildred> Grattan Endicott Message-ID: +=+ An appeal is an appeal of a Director's ruling. If there is no Director's ruling there is nothing to appeal. An appeals committee has no power to make a ruling of first instance. See Law 92C; the AC may have misdirected itself as to its power to hear the appeal and the decision should be overturned if this were the case, upon further appeal to the Regulating Authority. I do not know the timing of events, whatever the events were. The question will arise as to whether the appeal was through the Director and in time. ~ Grattan ~ +=+ ton: There was no appeal. But law 92C does not say that the AC has no power to hear a ruling of first instance. It just states that appeals should be brought to the appeals committee by the TD. Imo we are dealing with L83 here, with a liberal interpretation. The TD's decided not to make a decision (which is a decision in itself) and referred the case to the AC. The reason was that any decision taken would be appealed anyway, that nobody was longing for more wasted time, not the teams involved either. ton From grandaeval at tiscali.co.uk Mon Nov 10 14:18:54 2008 From: grandaeval at tiscali.co.uk (Grattan) Date: Mon, 10 Nov 2008 13:18:54 -0000 Subject: [blml] Deviations from announced system. References: <003501c940de$dfd49c60$0302a8c0@Mildred> <4914532F.1080100@skynet.be> <49145A04.2000707@ulb.ac.be> <49146907.9020204@skynet.be><49146E3D.8050904@ulb.ac.be> <49149C32.7050005@skynet.be><003701c94190$28575d40$0302a8c0@Mildred> <4916C87B.1080105@skynet.be><001701c9427e$7e4d08a0$0302a8c0@Mildred> <49180B27.9030002@skynet.be> Message-ID: <005c01c94336$fb4a1100$0302a8c0@Mildred> Grattan Endicott To: "Bridge Laws Mailing List" Sent: Monday, November 10, 2008 10:21 AM Subject: Re: [blml] Deviations from announced system. << > Now will you please tell us in what class Helgemo's 1H is supposed to be? > Is it a system that is badly explained (10-19 in stead of 8-19)? > Or is it a illegal deviation from a system? > In any case suppose that Helness knows Helgemo can open on 8 points and > the opponents don't. > > According to you, the adjustments in both cases are radically different. > How shall the TD decide which of the two laws to use? > > Please answer this one, Grattan, since it is important. > ============================================ +=+ Well Herman, Let's try to look at the essentials. What do we know about the Helgemo 1H? Is it a regular occurrence? What are its features? What does the partnership disclose about it (in particular on the System Card)? What do we think the partnership knows about it? Having established this much, what regulations are in force for the particular tournament? This can be a key question. So can we have a set of parameters to consider? ~ G ~ +=+ From grandaeval at tiscali.co.uk Mon Nov 10 14:45:30 2008 From: grandaeval at tiscali.co.uk (Grattan) Date: Mon, 10 Nov 2008 13:45:30 -0000 Subject: [blml] Ac in Amasterdam References: <48DA26571AEAE6BF@mail-10-uk.mail.tiscali.sys> (added by postmaster@mail-10.uk.tiscali.com) Message-ID: <007201c9433a$9a302360$0302a8c0@Mildred> Grattan Endicott To: "'Bridge Laws Mailing List'" Sent: Monday, November 10, 2008 11:16 AM Subject: Re: [blml] Ac in Amasterdam > There was no appeal. But law 92C does not say that the AC has no power to hear a ruling of first instance. It just states that appeals should be brought to the appeals committee by the TD. > Imo we are dealing with L83 here, with a liberal interpretation. The TD's decided not to make a decision (which is a decision in itself) and referred the case to the AC. The reason was that any decision taken would be appealed anyway, that nobody was longing for more wasted time, not the teams involved either. > > ton > +=+ I agree that if the Director refused to consider a request for a ruling that is a Director's decision that may be appealed. I do not agree that an AC has a power to make a decision of first instance if no application for a ruling has been made to a Director. Nor do I agree that the laws admit of any appeal that is not brought via the Director (93C is clear on this). An AC should ensure that the Director's function is not usurped, and it has been established previously by the WBFLC that a where a positive authority is not stated in the laws it is not to be assumed. The laws say with total clarity that an appeal must be an appeal of a Director's ruling. This is a point that Kojak has repeatedly addressed and I agree with him. It is not the function of the AC to stand in for a Director who has been bypassed. These points aside, the authoritative account of what occurred that ton has provided is significantly different from what we had read earlier. I have no direct knowledge of the case. ~ Grattan ~ +=+ From henk at ripe.net Mon Nov 10 15:09:35 2008 From: henk at ripe.net (Henk Uijterwaal) Date: Mon, 10 Nov 2008 15:09:35 +0100 Subject: [blml] Ac in Amasterdam In-Reply-To: <007201c9433a$9a302360$0302a8c0@Mildred> References: <48DA26571AEAE6BF@mail-10-uk.mail.tiscali.sys> (added by postmaster@mail-10.uk.tiscali.com) <007201c9433a$9a302360$0302a8c0@Mildred> Message-ID: <4918409F.5030009@ripe.net> Grattan wrote: > From: "ton" > The reason was that any decision taken would be appealed > anyway, that nobody was longing for more wasted time, not > the teams involved either. > +=+ I agree that if the Director refused to consider a request > for a ruling that is a Director's decision that may be appealed. > I do not agree that an AC has a power to make a decision of > first instance if no application for a ruling has been made to a > Director. Nor do I agree that the laws admit of any appeal > that is not brought via the Director (93C is clear on this). I fail to see the difference between the decision made by the TD here ("I'm not going to rule on this") or any other decision made by the TD. Whatever the TD decided, he knew one team would appeal this anyway. Henk ------------------------------------------------------------------------------ Henk Uijterwaal Email: henk.uijterwaal(at)ripe.net RIPE Network Coordination Centre http://www.amsterdamned.org/~henk P.O.Box 10096 Singel 258 Phone: +31.20.5354414 1001 EB Amsterdam 1016 AB Amsterdam Fax: +31.20.5354445 The Netherlands The Netherlands Mobile: +31.6.55861746 ------------------------------------------------------------------------------ Ceterum censeo Asplain esse delendam (Cato & Henk) From ehaa at starpower.net Mon Nov 10 15:16:52 2008 From: ehaa at starpower.net (Eric Landau) Date: Mon, 10 Nov 2008 09:16:52 -0500 Subject: [blml] Mr Gladstone suggested In-Reply-To: References: Message-ID: On Nov 7, 2008, at 1:24 AM, richard.hills at immi.gov.au wrote: > Law 40C1 (second sentence): > > "Repeated deviations lead to implicit understandings which > then form part of the partnership's methods and must be > disclosed in accordance with the regulations governing > disclosure of system." > > Richard Hills: > > Sven Pran and I also believe that a call which is _not_ part > of the partnership's methods need _not_ be disclosed. > > Law 40C2: > > "Other than the above no player has any obligation to disclose > to opponents that he has deviated from his announced methods." > > Law 75C, WBF LC interpretation, October 10th 2008: > > "The phrase 'they have no claim to an accurate description of > the N-S hands' first appeared in the 1975 laws of the game. It > was accompanied then as now by the injunction forbidding the > Director to alter the table result. It was entered primarily > to establish beyond doubt that that the partnership agreement > must be described accurately in response to lawful enquiry and > that the explanation given must not aim to describe what the > explainer believes as to the contents of either hand. It was > continued in those terms in the 1987 law book, while for 2007 > NBOs were invited to say whether the example or the wording > should be updated. Among replies received there was a general > consensus for retaining them as they had been previously, > whilst moving the statements from a footnote into the body of > the Law." > > David Burn asserted: > >> Precisely. That's why the WBF policy specifies that there >> must be disclosure of the nature of the deviation. Then, >> partner will have no more reason than the opponents to be >> aware of the possibility of and the nature of the deviation. > > Richard Hills quibbles: > > The WBF policy David Burn refers to was created by the WBF > Systems Committee. But the WBF Systems Committee lacks the > power to overrule the WBF Laws Committee on interpretation of > the Lawbook in general and of Law 75C in particular. (Indeed, > the WBF Systems Committee has failed to revisit its policies > since the 2007 Lawbook took effect.) > > What's the problem? The problem is that this requires a player considering a psych against an unknown pair to somehow determine the extent to which his opponents "have... reason... to be aware of the possibility of and the nature of the deviation". As a practical matter, that requires the relevant authority to provide a basis for some arbitrary presumption. IOW, for the law to be meaningful, someone must decide to what extent the possibility of a psych, or even of its "nature" in well-known "baby psych" positions, is a "matter[] generally known to bridge players". If you are the ACBL, and you have mounted a 30-year campaign to convince your membership that neither good nor ethical players ever psych, you wind up with the infamous one-psych-per-partnership-per- lifetime rule (and even that first one may require you to defend your action in front of a committee). If you live in a jurisdiction that recognizes psychs as a legitimate element of the game of bridge, matters are not so simple. Eric Landau 1107 Dale Drive Silver Spring MD 20910 ehaa at starpower.net From ehaa at starpower.net Mon Nov 10 16:07:35 2008 From: ehaa at starpower.net (Eric Landau) Date: Mon, 10 Nov 2008 10:07:35 -0500 Subject: [blml] Deviations from announced system. In-Reply-To: <000901c9427b$f28876d0$0302a8c0@Mildred> References: <003501c940de$dfd49c60$0302a8c0@Mildred> <4914532F.1080100@skynet.be> <49145A04.2000707@ulb.ac.be> <49146907.9020204@skynet.be><49146E3D.8050904@ulb.ac.be> <49149C32.7050005@skynet.be><003701c94190$28575d40$0302a8c0@Mildred> <4916C87B.1080105@skynet.be> <000901c9427b$f28876d0$0302a8c0@Mildred> Message-ID: <1A14B6E7-FDF1-44A8-A6FE-7F805260E029@starpower.net> On Nov 9, 2008, at 10:00 AM, Grattan wrote: > ----- Original Message ----- > From: "Herman De Wael" > > No Grattan, now you are throwing together two quite > different rulings. If the call is deemed systemic (at whatever > level) _and_ if the system regulations tell us they apply (not > necessarily the case IMO) _and_ if the call falls within those > banned by the system regulations (normally the case for weak > calls), THEN the call is banned by the system regulations. > > +=+ I am not really mixing into the irrelevant 'discussion' that > you are keeping alive. I am simply issuing constant reminders > that if there is a breach of Law 40A3 because the player makes > a call that is based on an undisclosed partnership understanding, > that call is illegal. The player will not then be allowed use of > the call and score adjustment will not, I say again *will not*, > take any account of possible actions ensuing by way of the > illegal call. It is too late and absolutely against the law for the > Director to take account of what could have happened if > opponents had been told of the partnership understanding. That is extremely counter-intuitive. It says that in making an adjustment consequent to a call or play based on an undisclosed partnership understanding, the adjudicator need not (indeed, must not) consider whether the understanding would or would not be allowed in the event had it been properly disclosed. Is that really right? Eric Landau 1107 Dale Drive Silver Spring MD 20910 ehaa at starpower.net From Harsanyi at t-online.de Mon Nov 10 16:16:50 2008 From: Harsanyi at t-online.de (Harsanyi at t-online.de) Date: Mon, 10 Nov 2008 16:16:50 +0100 Subject: [blml] =?iso-8859-15?q?Ac_in_Amasterdam?= In-Reply-To: <003601c94320$f5d5be10$0302a8c0@Mildred> References: <4917FE19.9010405@aol.com> <1KzTPJ-1coLfE0@fwd00.aul.t-online.de> <003601c94320$f5d5be10$0302a8c0@Mildred> Message-ID: <1KzYVW-0RlwG00@fwd00.aul.t-online.de> Dear Grattan, to collect the facts, is not any more possible, as the Law prescribes - despite of the 100% sportmanship of the four players. It will be written a lot of sentences about the unsatisfactorily circumstances. The analysis of this case is complex, the hunger in stomach and limited time for understanding and deciding makes every AC nervous, esspecially if the taxi for the AC-members is waiting in front of the House. I am sure, with more concentration the BLML-Colleagues shall make a stronger friendship with the 3NT bid - as Alains excellent mail shows it. All other questions will be cleared, documented and negotiated at the proper forums to raise the value of an European Champions Cup.. Thank you Josef -----Original Message----- Date: Mon, 10 Nov 2008 11:41:56 +0100 Subject: Re: [blml] Ac in Amasterdam From: "Grattan" To: "Bridge Laws Mailing List" ? ? ? ? Grattan Endicott Links: ------ [1] mailto:grandaeval at tiscali.co.uk [2] mailto:gesta at tiscali.co.uk [3] mailto:Harsanyi at t-online.de [4] mailto:blml at amsterdamned.org From Hermandw at skynet.be Mon Nov 10 16:18:56 2008 From: Hermandw at skynet.be (Herman De Wael) Date: Mon, 10 Nov 2008 16:18:56 +0100 Subject: [blml] Deviations from announced system. In-Reply-To: <005c01c94336$fb4a1100$0302a8c0@Mildred> References: <003501c940de$dfd49c60$0302a8c0@Mildred> <4914532F.1080100@skynet.be> <49145A04.2000707@ulb.ac.be> <49146907.9020204@skynet.be><49146E3D.8050904@ulb.ac.be> <49149C32.7050005@skynet.be><003701c94190$28575d40$0302a8c0@Mildred> <4916C87B.1080105@skynet.be><001701c9427e$7e4d08a0$0302a8c0@Mildred> <49180B27.9030002@skynet.be> <005c01c94336$fb4a1100$0302a8c0@Mildred> Message-ID: <491850E0.4000003@skynet.be> Yes Grattan, I will gladly answer everything I know about this case: Grattan wrote: > > ----- Original Message ----- > From: "Herman De Wael" >> Now will you please tell us in what class Helgemo's 1H is supposed to be? >> Is it a system that is badly explained (10-19 in stead of 8-19)? >> Or is it a illegal deviation from a system? >> In any case suppose that Helness knows Helgemo can open on 8 points and >> the opponents don't. >> >> According to you, the adjustments in both cases are radically different. >> How shall the TD decide which of the two laws to use? >> >> Please answer this one, Grattan, since it is important. >> > ============================================ > +=+ Well Herman, > Let's try to look at the essentials. > What do we know about the Helgemo 1H? > Is it a regular occurrence? What is regular? We know "in Norway they bid like that". In order for there to have to be a ruling we must assume that it is regular - Helness is not surprised, nor angry. > What are its features? third in hand, non-vul, 5332 shape. > What does the partnership disclose about it (in > particular on the System Card)? Well, for there to be a ruling, let's assume: nothing. > What do we think the partnership knows about > it? We assume in the ruling that partner knows it's possible. > Having established this much, what regulations > are in force for the particular tournament? This > can be a key question. I don't think it is - since I am not talking about system regulations at all, merely about L40C. Let's assume that the rule of 18 is not to be taken into account. If you think that is an impossible situation, then just add 2 points to the hands and assume they are playing 12-19. Or that he opens a strong two on a 16-count without additional distribution. Or anything to which the system regulations are immaterial. > So can we have a set of parameters to consider? > ~ G ~ +=+ > Anything else you wish to assume or which determines what ruling there is going to be, you can safely add to your ruling. My question is still the same: is this a "CPU" according to L40C or "MI" according to L40A1b and L40B4. How shall we tell the difference? Why should there need to be a difference? Herman. From wjburrows at gmail.com Mon Nov 10 16:48:57 2008 From: wjburrows at gmail.com (Wayne Burrows) Date: Tue, 11 Nov 2008 04:48:57 +1300 Subject: [blml] AC in Amsterdam In-Reply-To: <000901c94297$8dc606b0$a9521410$@de> References: <003501c940de$dfd49c60$0302a8c0@Mildred> <4914532F.1080100@skynet.be> <49145A04.2000707@ulb.ac.be> <49146907.9020204@skynet.be> <49146E3D.8050904@ulb.ac.be> <49149C32.7050005@skynet.be> <003701c94190$28575d40$0302a8c0@Mildred> <4916C87B.1080105@skynet.be> <000901c9427b$f28876d0$0302a8c0@Mildred> <000901c94297$8dc606b0$a9521410$@de> Message-ID: <2a1c3a560811100748s9500254j288a30e3ba3ed87f@mail.gmail.com> 2008/11/10 Josef Harsanyi : > http://www.eurobridge.org/bulletin/08_3%20Amsterdam/pdf/Bul_04.pdf > You are south: > 3s-p-p-x > P -? > > D5 > 9 > AJ1096(4) > K942 > > LA= pass / 3NT/ 4s/4NT, ? > Which kind of extraneous information(s) could be propagated by a slow > double? > Which alternative calls are suggested for East by a delayed double (if any)? > The singular alternative calls :pass/ 3NT/ 4s/4NT, in which portion are > influenced by a slow double? > How to rule as a TD? > How should (if at all) be adjusted the decision of TD by the AC, if the TD > decided for score stands? > > Thanks for the answers in advance. > Josef > > I agree with Grattan this is a tricky question. A slow double could be: 1. Stretching in values - do I compete or not? 2. A non-classical shape for a takeout double - will I live with partner bidding my doubleton? This could be in combination with marginal values or there could be an alternative of an overcall. 3. a marginal decision between an overcall and a double and bid hand. In many cases the UI or at least what is demonstrably suggested by it is minimal or non-existant. For a particular partnership there could be a pattern that slowness strongly suggests one action or another. In terms of frequency of hand type where a problem might occur stretching in values is more likely simply because hands with around average strength (9-11 hcp or so) are more frequent than any other similar range. In general a slow action usually creates much less UI and fewer demonstrably suggested alternatives than a slow pass which almost always suggests action on marginal hands. The most common significant exception is when partner makes a slow penalty double. Even then depending on the auction partner's decision could have been between pass and double or bid and double reducing or possibly even removing the demonstrably suggested options. -- Wayne Burrows Palmerston North New Zealand From svenpran at online.no Mon Nov 10 17:06:35 2008 From: svenpran at online.no (Sven Pran) Date: Mon, 10 Nov 2008 17:06:35 +0100 Subject: [blml] Deviations from announced system. In-Reply-To: <491850E0.4000003@skynet.be> References: <003501c940de$dfd49c60$0302a8c0@Mildred> <4914532F.1080100@skynet.be> <49145A04.2000707@ulb.ac.be> <49146907.9020204@skynet.be><49146E3D.8050904@ulb.ac.be> <49149C32.7050005@skynet.be><003701c94190$28575d40$0302a8c0@Mildred> <4916C87B.1080105@skynet.be><001701c9427e$7e4d08a0$0302a8c0@Mildred> <49180B27.9030002@skynet.be> <005c01c94336$fb4a1100$0302a8c0@Mildred> <491850E0.4000003@skynet.be> Message-ID: <002001c9434e$4e6742b0$eb35c810$@no> On Behalf Of Herman De Wael ............. > What is regular? We know "in Norway they bid like that". Before presenting allegations like that you should make certain that you know what you are talking about. Do you have any indication that the hearsay to this effect that we have seen is a correct description of facts? A single comment from some spectator "in Norway they bid like that" carries no value at all. .............. > My question is still the same: is this a "CPU" according to L40C or "MI" > according to L40A1b and L40B4. How shall we tell the difference? What evidence do you have of either alternative? Could there be a third? Sven From Hermandw at skynet.be Mon Nov 10 17:39:53 2008 From: Hermandw at skynet.be (Herman De Wael) Date: Mon, 10 Nov 2008 17:39:53 +0100 Subject: [blml] Deviations from announced system. In-Reply-To: <002001c9434e$4e6742b0$eb35c810$@no> References: <003501c940de$dfd49c60$0302a8c0@Mildred> <4914532F.1080100@skynet.be> <49145A04.2000707@ulb.ac.be> <49146907.9020204@skynet.be><49146E3D.8050904@ulb.ac.be> <49149C32.7050005@skynet.be><003701c94190$28575d40$0302a8c0@Mildred> <4916C87B.1080105@skynet.be><001701c9427e$7e4d08a0$0302a8c0@Mildred> <49180B27.9030002@skynet.be> <005c01c94336$fb4a1100$0302a8c0@Mildred> <491850E0.4000003@skynet.be> <002001c9434e$4e6742b0$eb35c810$@no> Message-ID: <491863D9.2090008@skynet.be> Sven Pran wrote: > On Behalf Of Herman De Wael > ............. >> What is regular? We know "in Norway they bid like that". > > Before presenting allegations like that you should make certain that you > know what you are talking about. Do you have any indication that the hearsay > to this effect that we have seen is a correct description of facts? > > A single comment from some spectator "in Norway they bid like that" carries > no value at all. > Sven, this is a hypothetical. OK? I want Grattan to answer me how a ruling should be made if the TD finds there is a problem. So of course I need to state that there is a problem. Please don't interfere if you have not understood what the thread has been about for the past five messages. Herman. From ziffbridge at t-online.de Mon Nov 10 17:51:48 2008 From: ziffbridge at t-online.de (Matthias Berghaus) Date: Mon, 10 Nov 2008 17:51:48 +0100 Subject: [blml] Ac in Amasterdam Message-ID: <491866A4.6010800@t-online.de> > ton: > I guess that the source for this information is Grom?ller and do not expect > corrections from him. Since Michael Grom?ller is not a member of this list I translate his report on the German list DOUBL (shortened and paraphrased, to get only the facts as presented by him). According to Michael two cases were heard, the players concerned had left the AC room. 5 minutes later the German players were told (presumably so were the Italian players, he does not mention that) that there was another appeal and they had to enter the AC room again. According to Michael the Italians asked the AC for a ruling about an alleged hesitation. The TD had not been called during the play. The AC gathered the facts and told the players that the TD would have to rule first. The players were sent away again. He does not say whether the TD made a ruling or not, presumably because he doesn`t know. The German players were not given a TD decision. 10 minutes later the TD gave the team all three AC rulings. The score on the third appeal was changed, giving the Italian team enough IMPs to win. The German team wanted a chance to tell the AC their arguments for not changing the score. This was denied, as the decision of the AC was final. These are the facts as presented by Michael Grom?ller (after filtering some anger and frustration). Apparently a TD denied the Germans a changed score earlier in the match, because the TD was called on the side of the screen where the hesitation happened. Now he asks why this appeal was not denied after the Italian on the other side of the screen did not call the director. Good question, actually, but I do not know what facts the AC gathered, so I refrain from commenting on this, but I can see why the German team is baffled and angry. They got what feels to them like two completely different decisions. Again, no comment without facts, but every TD knows players who are angry because they do not understand a decision, even when that decision was correct. Maybe the facts will get filled in later, so we can form an opinion. Best regards Matthias > > Since I was present my description of the facts could be worth something. > Yes, the Italian team reported this case late (to me), but still within the > protest time. At that moment the appeal committee was dealing with two other > appeals for which both sides were heard at that same moment, both TD's > being present. There was no decision made on any of those yet (first wrong > statement). > After finishing the hearing I told the AC about this third case and I sent > both teams to the AC for a further hearing (second wrong statement). That > hearing took place and both sides got the possibility to give their opinion. > Nobody at that stage told the Germans that they had no right to be heard > (third wrong statement). > Then the AC asked the TD's and me whether it was necessary to have a > decision from the TD's on this case before they could handle it. Our answer > was 'no'. > > Only then the AC started discussing all three cases. The outcome was that > they changed the result on the third case as reported. > After being informed the Germans wanted to appeal and at that moment I told > them that a further step in this case was not possible, the decision of the > AC being final. > > ton > > > > > _______________________________________________ > blml mailing list > blml at amsterdamned.org > http://www.amsterdamned.org/mailman/listinfo/blml > > From t.kooyman at worldonline.nl Mon Nov 10 17:53:56 2008 From: t.kooyman at worldonline.nl (ton) Date: Mon, 10 Nov 2008 17:53:56 +0100 Subject: [blml] Ac in Amasterdam In-Reply-To: <1KzYVW-0RlwG00@fwd00.aul.t-online.de> Message-ID: Dear Grattan, to collect the facts, is not any more possible, as the Law prescribes - despite of the 100% sportmanship of the four players. It will be written a lot of sentences about the unsatisfactorily circumstances. The analysis of this case is complex, the hunger in stomach and limited time for understanding and deciding makes every AC nervous, esspecially if the taxi for the AC-members is waiting in front of the House. ton: This is the not uncommon level of debate with mean suggestions and insinuations. It is not easy to show sportmanship when you loose a match. The AC met from 19.45 till 21.00 (forgive me a difference of one or two minutes) to handle these appeals. Only when the three appeals were dealt with and communicated with the teams involved taxis were called. They appeared at 21.15h. ton From jkljkl at gmx.de Mon Nov 10 17:59:17 2008 From: jkljkl at gmx.de (Stefan Filonardi) Date: Mon, 10 Nov 2008 17:59:17 +0100 Subject: [blml] Ac in Amasterdam In-Reply-To: <20081110104248.7850gmx1@mx033.gmx.net> References: <20081110104248.7850gmx1@mx033.gmx.net> Message-ID: <49186865.7040208@gmx.de> Hello, ton schrieb: (recapitulation by Jeff Easterson snipped) > ton: > I guess that the source for this information is Grom?ller and do not expect > corrections from him. I have no idea about the facts. But since I am reading both mailing lists, that are discussing about what happened in Amsterdam, I would like to put to your attention that I consider Jeff's recapitulation of Mr. Grom?ller's long email a little bit sloppy in some points. I will put to Mr. Grom?ll's attention that blml is discussing the case too and if he is interested to submit a recapitulation of the facts as seen by him or if he allows me to forward his email. with best regards stefan filonardi From harsanyi at t-online.de Mon Nov 10 19:34:31 2008 From: harsanyi at t-online.de (Josef Harsanyi) Date: Mon, 10 Nov 2008 19:34:31 +0100 Subject: [blml] Ac in Amasterdam References: <1KzYVW-0RlwG00@fwd00.aul.t-online.de> Message-ID: <005c01c94362$f8abbb70$ea033250$@de> Sorry Ton, I want to take back all of mine suggestions and insinuations, if my information about the happenings in Amsterdam were not correct. Only within the limits of sportsmanship and absolutely correctness can I imagine to find the answers on the questions regarding the ruling of AC which made the German Team from winner to "not winner". Sorry in advance, if the members of AC made their job without any deviation from the regulations. Josef to collect the facts, is not any more possible, as the Law prescribes - despite of the 100% sportmanship of the four players. It will be written a lot of sentences about the unsatisfactorily circumstances. The analysis of this case is complex, the hunger in stomach and limited time for understanding and deciding makes every AC nervous, esspecially if the taxi for the AC-members is waiting in front of the House. ton: This is the not uncommon level of debate with mean suggestions and insinuations. It is not easy to show sportmanship when you loose a match. The AC met from 19.45 till 21.00 (forgive me a difference of one or two minutes) to handle these appeals. Only when the three appeals were dealt with and communicated with the teams involved taxis were called. They appeared at 21.15h. ton _______________________________________________ blml mailing list blml at amsterdamned.org http://www.amsterdamned.org/mailman/listinfo/blml From schoderb at msn.com Mon Nov 10 22:05:45 2008 From: schoderb at msn.com (WILLIAM SCHODER) Date: Mon, 10 Nov 2008 16:05:45 -0500 Subject: [blml] Ac in Amasterdam In-Reply-To: <48DA26571AEAE6BF@mail-10-uk.mail.tiscali.sys> (added bypostmaster@mail-10.uk.tiscali.com) <007201c9433a$9a302360$0302a8c0@Mildred> References: <48DA26571AEAE6BF@mail-10-uk.mail.tiscali.sys> (added bypostmaster@mail-10.uk.tiscali.com) <007201c9433a$9a302360$0302a8c0@Mildred> Message-ID: "....... Only then the AC started discussing all three cases......" (Ton's latest quote that I have) seems kind of strange to me. Why were the three appeals grouped in discussion? It would seem to me that each case would be decided and completed on its own merits, before going on to the next one. Yet I find no other reason to modify what I drafted this AM, which follows: Not having all the facts; Ton's "liberal interpretation" of Law 83 is wrong, Why bother to follow the strictures of the Laws when there is an alternate less demanding route you can take? Law 83 clearly states ".....if the Director feels that a REVIEW OF HIS DECISION......ETC....." - I fail to see how this authorizes anyone to throw the appeals process out the window. Smells to me like politics running rampant. Even in English this means you have to make a decision. There was no ruling? Why was there no ruling? I don't buy the argument about being sure one side or the other would appeal anyhow? "......... There was no appeal........." So whatever happened at the table stood BY LAW I would think. To say ".....Law 92C does not say that the AC has no power to hear a ruling of first instance....." cleverly plants the idea that players and ACs can do what they please in adjudicating disagreements without the Director. That is wrong. Rejection of the Code of Practice for Appeals Committees is just one example of 'I know what is right, I don't need those things'. I'm well familiar with the attitude of those Regulating Authorities who find Directors' decisions are just a waste of time, and are eager to get the 'case' to a group of individuals whose knowledge of the laws is often far below their abilities to analyze and play bridge. ACBL, for example, cancels the Director's ruling at the beginning of the hearing, and the AC makes its own ruling! The ruling is not REVIEWED, it is CANCELLED, and the AC starts from scratch. I don't see this kind of action in other sports, -- the usual philosophy is to uphold the TD, Referee, Umpire, Judge, Arbiter, Official, etc. decisions unless there is substantial reason to change them. Seems like others have greater faith in their trained officials than some bridge organizations are willing to accept. Respect for Laws, Rules, Regulations, and adherence thereto restricts the flights of fancy of the know-it-alls who work hard to destroy our game. In my experience in WBF and many WBF zones and NBOs, I've often heard AC chairmen refuse to add some other situation to a case when players attempted to do so. Bravo! Take into account each item as it occurred, decide on the review of that ruling, and then go to the next item. It makes the overall result a lot more palatable, creates less suspicion, less contention and argument when the simple, direct Laws of Duplicate Bridge 2007 are followed. For an AC to preempt the Directors duties throws a wrench into the machinery and goes a long way to get the "do what feels good (to me) at the time" ambiance that some people thrive upon. To refuse to make a ruling when it was requested in accordance with the laws is something no competent Director should ever consider - it simply boggles my mind! It puts the scenario into a realm where anything goes and breaks down a tested, established, and valuable process. I'd like to have all the FACTS of course, but it appears they remain in dispute. Kojak > From: "ton" > There was no appeal. But law 92C does not say that the AC > has no power to hear a ruling of first instance. It just states that > appeals should be brought to the appeals committee by the TD. > > > Imo we are dealing with L83 here, with a liberal interpretation. > The TD's decided not to make a decision (which is a decision > in itself) and referred the case to the AC. > The reason was that any decision taken would be appealed > anyway, that nobody was longing for more wasted time, not > the teams involved either. > > > > ton >From Grattan: > +=+ I agree that if the Director refused to consider a request > for a ruling that is a Director's decision that may be appealed. > I do not agree that an AC has a power to make a decision of > first instance if no application for a ruling has been made to a > Director. Nor do I agree that the laws admit of any appeal > that is not brought via the Director (93C is clear on this). An > AC should ensure that the Director's function is not usurped, > and it has been established previously by the WBFLC that a > where a positive authority is not stated in the laws it is not to > be assumed. > The laws say with total clarity that an appeal must be an > appeal of a Director's ruling. This is a point that Kojak has > repeatedly addressed and I agree with him. It is not the function > of the AC to stand in for a Director who has been bypassed. > These points aside, the authoritative account of what > occurred that ton has provided is significantly different from > what we had read earlier. I have no direct knowledge of the > case. > ~ Grattan ~ +=+ > > > _______________________________________________ > blml mailing list > blml at amsterdamned.org > http://www.amsterdamned.org/mailman/listinfo/blml > From richard.hills at immi.gov.au Mon Nov 10 22:29:29 2008 From: richard.hills at immi.gov.au (richard.hills at immi.gov.au) Date: Tue, 11 Nov 2008 08:29:29 +1100 Subject: [blml] Mr Gladstone suggested [SEC=UNOFFICIAL] In-Reply-To: <4918099B.8050600@skynet.be> Message-ID: Peter Hennessy (1947- ), English historian: "The model of a modern Prime Minister would be a kind of grotesque composite freak -- someone with the dedication to duty of a Peel, the physical energy of a Gladstone, the detachment of a Salisbury, the brains of an Asquith, the balls of a Lloyd George, the word-power of a Churchill, the administrative gifts of an Attlee, the style of a Macmillan, the managerialism of a Heath, the erudition of an Endicott, and the sleep requirements of a Thatcher. Human beings do not come like that." Herman De Wael asked: >Richard, have you even thought of the following three >questions: > >1) How do you feel about a law that obliges you to give UI to >your partner? >2) As TD, if there is a law that obliges someone to give UI >to his partner, how do you rule if the player in front of you >has refused to do so? >3) Quiz question: how many laws in the 2007 Lawbook >explicitly forbid giving a piece of UI to partner? And how >many oblige a player to do so? >Answers on a postcard, please. Postcard size answer from WBF LC minutes, 10th October 2008: "There is no infraction when a correct explanation discloses that partner's prior explanation was mistaken. The words 'nor may he indicate in any manner that a mistake has been made' (in Law 20F5(a)) do not refer to compliance with the overriding requirement of the laws always to respond to enquiries under Law 20F with correct explanations of the partnership understandings." Herman De Wael asserted: >And indeed not what I wanted, but also not what is needed. I >see no reason to change my intended habit if it ever came >along at my particular table. No sense in saying what a player >should do if there is no penalty to back it up. I see a reason to change in Law 72B1: "A player must not infringe a law intentionally, even if there is a prescribed rectification he is willing to accept." I see a reason to change in Law 40C3(b): "Repeated violations of requirements to disclose partnership understandings may be penalized." I see a penalty to back it up in Law 91B - Right to Disqualify: "The Director is empowered to disqualify a contestant for cause, subject to approval by the Tournament Organizer." What's the problem? Best wishes Richard James Hills Recruitment Section, Level 3 Blue, workstation 15 (first on left) Department of Immigration and Citizenship Telephone: 02 6223 8453 Email: richard.hills at immi.gov.au -------------------------------------------------------------------- Important Notice: If you have received this email by mistake, please advise the sender and delete the message and attachments immediately. This email, including attachments, may contain confidential, sensitive, legally privileged and/or copyright information. Any review, retransmission, dissemination or other use of this information by persons or entities other than the intended recipient is prohibited. DIAC respects your privacy and has obligations under the Privacy Act 1988. The official departmental privacy policy can be viewed on the department's website at www.immi.gov.au. See: http://www.immi.gov.au/functional/privacy.htm --------------------------------------------------------------------- From svenpran at online.no Mon Nov 10 22:53:48 2008 From: svenpran at online.no (Sven Pran) Date: Mon, 10 Nov 2008 22:53:48 +0100 Subject: [blml] Deviations from announced system. In-Reply-To: <491863D9.2090008@skynet.be> References: <003501c940de$dfd49c60$0302a8c0@Mildred> <4914532F.1080100@skynet.be> <49145A04.2000707@ulb.ac.be> <49146907.9020204@skynet.be><49146E3D.8050904@ulb.ac.be> <49149C32.7050005@skynet.be><003701c94190$28575d40$0302a8c0@Mildred> <4916C87B.1080105@skynet.be><001701c9427e$7e4d08a0$0302a8c0@Mildred> <49180B27.9030002@skynet.be> <005c01c94336$fb4a1100$0302a8c0@Mildred> <491850E0.4000003@skynet.be> <002001c9434e$4e6742b0$eb35c810$@no> <491863D9.2090008@skynet.be> Message-ID: <000601c9437e$cfe03f10$6fa0bd30$@no> On Behalf Of Herman De Wael ............. > > Before presenting allegations like that you should make certain that you > > know what you are talking about. Do you have any indication that the hearsay > > to this effect that we have seen is a correct description of facts? > > > > A single comment from some spectator "in Norway they bid like that" carries > > no value at all. > > > > Sven, this is a hypothetical. OK? It is not hypothetical when you continue repeating slander against the Norwegian Bridge playing society with no other foundation than having heard or read some comments from a single spectator. Sven From wrgptfan at gmail.com Tue Nov 11 01:03:33 2008 From: wrgptfan at gmail.com (David Kent) Date: Mon, 10 Nov 2008 19:03:33 -0500 Subject: [blml] Ac in Amasterdam In-Reply-To: References: <48DA26571AEAE6BF@mail-10-uk.mail.tiscali.sys> (added bypostmaster@mail-10.uk.tiscali.com) <007201c9433a$9a302360$0302a8c0@Mildred> Message-ID: I very rarely post here although I do read - or at least skim - most postings. I mostly play in ACBL national events, the occasional international event, the Canadian National Team Chanpionships and a very occasional regional. Having said that, I find that the never- ending discussion about Helgemo's 1H opener in 3rd hand with - God forbid - only 7 HCP totally ludicrous. It would never occur to me as his opponent to have a second thought about this - let alone think about calling the director if my partnership were unable to handle this "gross" violation. Similarly, if Herman was to psyche a 1H opener in 3rd seat or if Zia made a tactical bid against me that happened to work, I would applaud their initiative not feeling that I had been hard done by. However when a situation like that which occurred in Amsterdam happens, I feel compelled to speak out. If indeed the situation as described by Ton et al actually occurred, I am in 100% agreement with Kojak. It is incredible that this happened in a tournament of this importance. I have no personal feeling as to the ruling itself (well I actually do, but that is beside the point) but I feel that the process followed is intrinsically wrong. At this level of competition, following correct procedure is at least important as getting the ruling correct. If a ruling by an AC was incorrect, there is usually at least some sympathy for the ruling. However there should be no sympathy for failing to follow proper procedure. I apologize for my blathering as all I originally intended to say was "Right on, Kojak". Sent from my iPhone On Nov 10, 2008, at 4:05 PM, "WILLIAM SCHODER" wrote: > > "....... Only then the AC started discussing all three > cases......" (Ton's > latest > quote that I have) seems kind of strange to me. Why were the three > appeals grouped in discussion? It would seem to me that each case > would be > decided and completed on its own merits, before going on to the next > one. > > Yet I find no other reason to modify what I drafted this AM, which > follows: > > Not having all the facts; > > Ton's "liberal interpretation" of Law 83 is wrong, Why bother to > follow the > strictures of the Laws when there is an alternate less demanding > route you > can take? Law 83 clearly states ".....if the Director feels that a > REVIEW OF > HIS DECISION......ETC....." - I fail to see how this authorizes > anyone to > throw the appeals process out the window. Smells to me like politics > running > rampant. Even in English this means you have to make a decision. > There was > no ruling? Why was there no ruling? I don't buy the argument > about being sure one side or the other would appeal anyhow? > > "......... There was no appeal........." So whatever happened at the > table > stood BY LAW I would think. To say ".....Law 92C does not say that > the AC > has no power to hear a ruling > of first instance....." cleverly plants the idea that players and > ACs can > do what they please in adjudicating disagreements without the > Director. That > is wrong. Rejection of the Code of Practice for Appeals Committees > is just > one example of 'I know what is right, I don't need those things'. > > I'm well familiar with the attitude of those Regulating Authorities > who find > Directors' decisions are just a waste of time, and are eager to get > the > 'case' to a group of individuals whose knowledge of the laws is > often far > below their abilities to analyze and play bridge. ACBL, for example, > cancels > the Director's ruling at the beginning of the hearing, and the AC > makes its > own ruling! The ruling is not REVIEWED, it is CANCELLED, and the AC > starts > from scratch. > > I don't see this kind of action in other sports, -- the usual > philosophy is > to uphold the TD, Referee, Umpire, Judge, Arbiter, Official, etc. > decisions > unless there is substantial reason to change them. Seems like others > have > greater faith in their trained officials than some bridge > organizations are > willing to accept. Respect for Laws, Rules, Regulations, and adherence > thereto restricts the flights of fancy of the know-it-alls who work > hard to > destroy our game. > > In my experience in WBF and many WBF zones and NBOs, I've often > heard AC > chairmen refuse to add some other situation to a case when players > attempted > to do so. Bravo! Take into account each item as it occurred, decide > on the > review of that ruling, and then go to the next item. It makes the > overall > result a lot more palatable, creates less suspicion, less contention > and > argument when the simple, direct Laws of Duplicate Bridge 2007 are > followed. > > For an AC to preempt the Directors duties throws a wrench > into the machinery and goes a long way to get the "do what feels > good (to > me) at the > time" ambiance that some people thrive upon. To refuse to make a > ruling when > it was requested in accordance with the laws is something no competent > Director should ever consider - it simply boggles my mind! It puts the > scenario into a realm where > anything goes and breaks down a tested, established, and valuable > process. > > I'd like to have all the FACTS of course, but it appears they remain > in > dispute. > > Kojak > > >> From: "ton" > >> There was no appeal. But law 92C does not say that the AC >> has no power to hear a ruling of first instance. It just states that >> appeals should be brought to the appeals committee by the TD. >>> >> Imo we are dealing with L83 here, with a liberal interpretation. >> The TD's decided not to make a decision (which is a decision >> in itself) and referred the case to the AC. >> The reason was that any decision taken would be appealed >> anyway, that nobody was longing for more wasted time, not >> the teams involved either. >>> >>> ton > >> From Grattan: > >> +=+ I agree that if the Director refused to consider a request >> for a ruling that is a Director's decision that may be appealed. >> I do not agree that an AC has a power to make a decision of >> first instance if no application for a ruling has been made to a >> Director. Nor do I agree that the laws admit of any appeal >> that is not brought via the Director (93C is clear on this). An >> AC should ensure that the Director's function is not usurped, >> and it has been established previously by the WBFLC that a >> where a positive authority is not stated in the laws it is not to >> be assumed. >> The laws say with total clarity that an appeal must be an >> appeal of a Director's ruling. This is a point that Kojak has >> repeatedly addressed and I agree with him. It is not the function >> of the AC to stand in for a Director who has been bypassed. >> These points aside, the authoritative account of what >> occurred that ton has provided is significantly different from >> what we had read earlier. I have no direct knowledge of the >> case. >> ~ Grattan ~ +=+ >> >> >> _______________________________________________ >> blml mailing list >> blml at amsterdamned.org >> http://www.amsterdamned.org/mailman/listinfo/blml >> > > > _______________________________________________ > blml mailing list > blml at amsterdamned.org > http://www.amsterdamned.org/mailman/listinfo/blml From richard.hills at immi.gov.au Tue Nov 11 01:46:21 2008 From: richard.hills at immi.gov.au (richard.hills at immi.gov.au) Date: Tue, 11 Nov 2008 11:46:21 +1100 Subject: [blml] Deviations from announced system. [SEC=UNOFFICIAL] In-Reply-To: <1A14B6E7-FDF1-44A8-A6FE-7F805260E029@starpower.net> Message-ID: Grattan Endicott asserted: [snip] >>if there is a breach of Law 40A3 because the player makes >>a call that is based on an undisclosed partnership >>understanding, that call is illegal. [snip] >>It is too late and absolutely against the law for the >>Director to take account of what could have happened if >>opponents had been told of the partnership understanding. >> ~ Grattan ~ +=+ Eric Landau asked: >That is extremely counter-intuitive. It says that in >making an adjustment consequent to a call or play based on >an undisclosed partnership understanding, the adjudicator >need not (indeed, must not) consider whether the >understanding would or would not be allowed in the event >had it been properly disclosed. Is that really right? Richard Hills quibbles: No, in my opinion Grattan is really wrong. Law 40A3: "A player may make any call or play without prior announcement provided that such call or play is not based on an undisclosed partnership understanding (see Law 40C1)." Law 40B4: "A side that is damaged as a consequence of its opponents' failure to provide disclosure of the meaning of a call or play as these laws require, is entitled to rectification through the award of an adjusted score." Richard Hills quibbles: It seems to me that an infraction of Law 40A3 is identical to an infraction of Law 40B4. In both cases the infractions seem to me to be simply standard misinformation (MI). (Of course, if the MI conceals an intrinsically unLawful understanding - a so-called "illegal convention" - then Law 40B5 would take precedence over Law 40B4.) And the standard resolution for a MI ruling is not to deem that the undisclosed call was never made, but rather to determine what would have happened if the non-offending side had been informed at the appropriate time. What's the problem? Yes, I know that disagreeing with Grattan is the problem, and there is a better than 50% chance I will have to retract my interpretation once Grattan unveils an obvious point that I have missed in the new Law 40. Best wishes Richard James Hills Recruitment Section, Level 3 Blue, workstation 15 (first on left) Department of Immigration and Citizenship Telephone: 02 6223 8453 Email: richard.hills at immi.gov.au -------------------------------------------------------------------- Important Notice: If you have received this email by mistake, please advise the sender and delete the message and attachments immediately. This email, including attachments, may contain confidential, sensitive, legally privileged and/or copyright information. Any review, retransmission, dissemination or other use of this information by persons or entities other than the intended recipient is prohibited. DIAC respects your privacy and has obligations under the Privacy Act 1988. The official departmental privacy policy can be viewed on the department's website at www.immi.gov.au. See: http://www.immi.gov.au/functional/privacy.htm --------------------------------------------------------------------- From grandaeval at tiscali.co.uk Tue Nov 11 02:30:10 2008 From: grandaeval at tiscali.co.uk (Grattan) Date: Tue, 11 Nov 2008 01:30:10 -0000 Subject: [blml] Ac in Amasterdam References: <48DA26571A5EB8C1@mail-8-uk.mail.tiscali.sys> (added by postmaster@mail-8.uk.tiscali.com) Message-ID: <000a01c9439d$0bcb9e70$0302a8c0@Mildred> Grattan Endicott To: "'Bridge Laws Mailing List'" Sent: Monday, November 10, 2008 4:53 PM Subject: Re: [blml] Ac in Amasterdam > > > Dear Grattan, > > to collect the facts, is not any more possible, as the Law prescribes - > despite of the 100% sportmanship of the four players. It will be written a > lot of sentences about the unsatisfactorily circumstances. The analysis > of > this case is complex, the hunger in stomach and limited time for > understanding and deciding makes every AC nervous, esspecially if the taxi > for the AC-members is waiting in front of the House. > > > ton: > This is the not uncommon level of debate with mean suggestions and > insinuations. It is not easy to show sportmanship when you loose a match. > > The AC met from 19.45 till 21.00 (forgive me a difference of one or two > minutes) to handle these appeals. Only when the three appeals were dealt > with and communicated with the teams involved taxis were called. They > appeared at 21.15h. > > ton > +=+ Yes, Ton, When I read your evidently objective account of the matter I did immediately regret that I had allowed a prejudiced report to suck me into discussion. The procedural angles are relatively minor and if the TD was present I consider this to indicate that the appeal has been presented through the Director, satisfying 92C. I am under attack from a virus at this time, causing inflammation in joints and quite painful. So I think I am less wary than usual under the stress of it. ~ Grattan ~ +=+ From gro at bridgepro.de Tue Nov 11 03:53:17 2008 From: gro at bridgepro.de (Gro) Date: Tue, 11 Nov 2008 03:53:17 +0100 Subject: [blml] Ac in Amasterdam Message-ID: <4918F39D.8070708@bridgepro.de> Here are the facts from my point of view: All starts when germany made an appeal against Italy. There was hesitation on both sides of the screen and the TD was called. For one hesitation the "right" side made the call , that the tray was very long at the other side. The TD accepted this. For the second hesitation the "wrong side" called -- so- the side where the hesitation took place. The TD did NOT accept this, cuase he told the players, that the wrong side called and so they don't take this facts and also will not rule about this. So the TD only decides the call from the right side and his decision was, that the bid is normal and the score stands. Against this decision germany appealed. This appeal was lost by germany, cause AC followed the TD-decision. after 4 segments of play the result was, that germany has won against italy at the table. Now the TD comes and said: italy has 2 appeals and we should come to the AC immediately. The AC told us, that both appeals will be handled in a row. So the normal procedure starts, TD gives the facts and why he ruled- the players were asked statements and had to answer questions from the AC. With the second appeal it was exactly the same. Now both could go out and wait for the decisions. After 5 minutes the TD tells us, to come in again, cause italy has a 3^rd appeal. The TD shows the hand .now italy was asked why they make this appeal. They said, that there was a hesitation and with this hesitation the bid from the other side is not "possible". The german were asked, if they agree, that there was a hesitation and they agreed, saying it was a very difficult decision, which needs time.Now the AC says, that they will ask the TD to make a decision and afterwards will decide about that appeal.. Both were send out now. After about 15 minutes the TD comes and gives us the decisions: appeal 1: itlay lost- score stands appeal 2: italy lost, score stands appeal 3: italy wins, score corrected- and italy wins the whole match now! Then I asked the TD: "what was the decision from you?" He said: "there was no decision from the TD- we gave the finding of the decision to the AC.". I asked : " why couldn't we say something about that? The AC said to us, that they wait on the TD decision and then the appeal can happen." TD said: " yes- but we told the AC that the TD will not decide, cause any other will appeal in any case- so we let the decion to the AC himself." I said: " and why couldn't we add something before the appeal? we thought we can say some words after the TD-decision and get heared what we can add and say." Td said: " you have no right to get heared!" I asked him: "what? we have no right to get heared anymore? That's an european championship- and we speaking about reaching the final or not- and you sure we can add nothing?" Then Mr. Koiman said: "no- you are out- you had no right of getting heared- and they want to close now here!" I said: "I am sorry- I cant believe this -- we are out now, cause we couldn't tell the AC something about this 3^rd appeal? We could not know, that its impossible to say something after you send us out with the words: first the TD must take a decision." " and now you telling us, that you didn't decide anything and we are also not allowed to say something???? " Koiman said: "you out. the AC decided- and against this decision you cant do anything! " I said: " oh..I want to appeal this! (not against the ruling of the AC- but against the procedure of how it comes to this ruling)" . Mr Melander from sweden was standing byside and hears all the conversation and I think he felt very uncomfortable now about his own decision , so he finally asked Mr. koiman, if we can appeal against this. Mr. Koiman said: " no- but if you find someone in amsterdam now , you can give him your appeal- I have to take my taxi now, bye!" .............. I have no idea, if the TD can just give the right of decision to the AC ? But I just cannot understand, why the 3^rd appeal could ever happen. With our appeal, the TD told us, that he will take no notice for the one hesitation, cause the wrong side was calling! And now, this 3^rd appeal was: Italy comes at the end of the session t the TD and reports, that in 1 board was a hesitation at one side of the screen. This must have beeen the german player. The TD took this note. And at the end this was the 3^rd appeal. What I now don't understand: in our appeal we were not allowed to use inforamtion , cause wrong side called. in italy appeal, NOTHING was said at the table -- NO TD was called -- and NOW the TD accepts this???? Without knowing if the wrong or right side "called" even there was NO call at all...??..is that ok and logical?.. If I understand this right, then we were punished, cause wrong side called---and with italy, NOBODY called and the the TD says: ok- we take your point? in my sense of logic, this appeal could never happened at all. At the next day I said to the TD before we started: "we play this match with protest. We wanted appeal yesterday night but were not allowed to do so.!" "We didn't want to appeal agianst the decision, cause its not possible, but againts the procedure before. We think, if we could tell this facts to the AC ( the facts, that there was no TD and no call at all- and therefore the appeal should never been allowed at all), then the AC might have found another decion- maybe the decision, that they will not talk about this at all, cause of no call at the table and no call to the TD. that were all the facts. _Now my personal meaning:_ Even if the TD are allowed to transfer a decision to the AC, its very poor to do it in an european Halffinal. Even if the players have no technical right to say some words, they should have heared the players- cause again its an european halffinal- it would have taken about 5 minutes. Its NOT our fault, that the italians made 2 appeals that took a lot of time. what can we do now? The german team is very frustrated about this happening. loosing an appeal without getting heared- loosing a final, cause TD let it through, that the opps made a statement at the end of session without sayiing anything at the table before abut this hesitation. We just rember last year championcup: italy made an appeal against us with hesitation again and won it- this decisoin last year was very very close.And with this deciosn we lost by 5, instead of winning the match. Now we loose again with 5 cause of AC committee decision. We asked Mr. Latala about this appeal and the decision of the AC. He said. " that decision is a big scandal! And you can tell this with my name on it to the AC ! "..... so..not only the appeal was doubful to come in -- also the decision itself was very doubtful also..... And as it might happen: we have nothing more to do, then to look around and accept all this without the hope of an appeal against that . That makes as very upset !!! And I really cant understand, why Ton says, it needs really sportsmanship when you loose a match. If he meant our team, I think with this report most of you might understand, why we were not satisfied to loose the match. I am allways (!) a good sportsman - but if results from a match comes not from the bridgetable but from , lets say, some very unlucky circumstansces around the bridgetable - yes- then its not so eays to show good sportsmanship- cause we were not given this sportmanship before at all- by not hearing us and with all what I wrote. Michael Grom?ller, german captain. From wjburrows at gmail.com Tue Nov 11 04:19:46 2008 From: wjburrows at gmail.com (Wayne Burrows) Date: Tue, 11 Nov 2008 16:19:46 +1300 Subject: [blml] Ac in Amasterdam In-Reply-To: <4918F39D.8070708@bridgepro.de> References: <4918F39D.8070708@bridgepro.de> Message-ID: <2a1c3a560811101919x623ac545uce2bdc6e9c437e68@mail.gmail.com> 2008/11/11 Gro : > Here are the facts from my point of view: > > All starts when germany made an appeal against Italy. There was > hesitation on both sides of the screen and the TD was called. For one > hesitation the "right" side made the call , that the tray was very long > at the other side. The TD accepted this. For the second hesitation the > "wrong side" called -- so- the side where the hesitation took place. The > TD did NOT accept this, cuase he told the players, that the wrong side > called and so they don't take this facts and also will not rule about > this. So the TD only decides the call from the right side and his > decision was, that the bid is normal and the score stands. Against this > decision germany appealed. This appeal was lost by germany, cause AC > followed the TD-decision. > > after 4 segments of play the result was, that germany has won against > italy at the table. > > Now the TD comes and said: italy has 2 appeals and we should come to the > AC immediately. > > The AC told us, that both appeals will be handled in a row. So the > normal procedure starts, TD gives the facts and why he ruled- the > players were asked statements and had to answer questions from the AC. > With the second appeal it was exactly the same. Now both could go out > and wait for the decisions. > > After 5 minutes the TD tells us, to come in again, cause italy has a > 3^rd appeal. The TD shows the hand .now italy was asked why they make > this appeal. They said, that there was a hesitation and with this > hesitation the bid from the other side is not "possible". The german > were asked, if they agree, that there was a hesitation and they agreed, > saying it was a very difficult decision, which needs time.Now the AC > says, that they will ask the TD to make a decision and afterwards will > decide about that appeal.. Both were send out now. > > After about 15 minutes the TD comes and gives us the decisions: > > appeal 1: itlay lost- score stands > > appeal 2: italy lost, score stands > > appeal 3: italy wins, score corrected- and italy wins the whole match now! > > Then I asked the TD: "what was the decision from you?" He said: "there > was no decision from the TD- we gave the finding of the decision to the > AC.". > > I asked : " why couldn't we say something about that? The AC said to us, > that they wait on the TD decision and then the appeal can happen." TD > said: " yes- but we told the AC that the TD will not decide, cause any > other will appeal in any case- so we let the decion to the AC himself." > > I said: " and why couldn't we add something before the appeal? we > thought we can say some words after the TD-decision and get heared what > we can add and say." > > Td said: " you have no right to get heared!" > > I asked him: "what? we have no right to get heared anymore? That's an > european championship- and we speaking about reaching the final or not- > and you sure we can add nothing?" Then Mr. Koiman said: "no- you are > out- you had no right of getting heared- and they want to close now here!" > > I said: "I am sorry- I cant believe this -- we are out now, cause we > couldn't tell the AC something about this 3^rd appeal? We could not > know, that its impossible to say something after you send us out with > the words: first the TD must take a decision." " and now you telling us, > that you didn't decide anything and we are also not allowed to say > something???? " > > Koiman said: "you out. the AC decided- and against this decision you > cant do anything! " I said: > > " oh..I want to appeal this! (not against the ruling of the AC- but > against the procedure of how it comes to this ruling)" . Mr Melander > from sweden was standing byside and hears all the conversation and I > think he felt very uncomfortable now about his own decision , so he > finally asked Mr. koiman, if we can appeal against this. Mr. Koiman > said: " no- but if you find someone in amsterdam now , you can give him > your appeal- I have to take my taxi now, bye!" > > .............. > > I have no idea, if the TD can just give the right of decision to the AC ? > > But I just cannot understand, why the 3^rd appeal could ever happen. > With our appeal, the TD told us, that he will take no notice for the one > hesitation, cause the wrong side was calling! > > And now, this 3^rd appeal was: Italy comes at the end of the session t > the TD and reports, that in 1 board was a hesitation at one side of the > screen. This must have beeen the german player. > > The TD took this note. And at the end this was the 3^rd appeal. > > What I now don't understand: > > in our appeal we were not allowed to use inforamtion , cause wrong side > called. > > in italy appeal, NOTHING was said at the table -- NO TD was called -- > and NOW the TD accepts this???? Without knowing if the wrong or right > side "called" even there was NO call at all...??..is that ok and logical?.. > > If I understand this right, then we were punished, cause wrong side > called---and with italy, NOBODY called and the the TD says: ok- we take > your point? > > in my sense of logic, this appeal could never happened at all. > > At the next day I said to the TD before we started: "we play this match > with protest. We wanted appeal yesterday night but were not allowed to > do so.!" "We didn't want to appeal agianst the decision, cause its not > possible, but againts the procedure before. We think, if we could tell > this facts to the AC ( the facts, that there was no TD and no call at > all- and therefore the appeal should never been allowed at all), then > the AC might have found another decion- maybe the decision, that they > will not talk about this at all, cause of no call at the table and no > call to the TD. > > that were all the facts. > > _Now my personal meaning:_ > > Even if the TD are allowed to transfer a decision to the AC, its very > poor to do it in an european Halffinal. > > Even if the players have no technical right to say some words, they > should have heared the players- cause again its an european halffinal- > it would have taken about 5 minutes. Its NOT our fault, that the > italians made 2 appeals that took a lot of time. > > what can we do now? The german team is very frustrated about this > happening. loosing an appeal without getting heared- loosing a final, > cause TD let it through, that the opps made a statement at the end of > session without sayiing anything at the table before abut this hesitation. > > We just rember last year championcup: italy made an appeal against us > with hesitation again and won it- this decisoin last year was very very > close.And with this deciosn we lost by 5, instead of winning the match. > > Now we loose again with 5 cause of AC committee decision. We asked Mr. > Latala about this appeal and the decision of the AC. He said. " that > decision is a big scandal! And you can tell this with my name on it to > the AC ! "..... > > so..not only the appeal was doubful to come in -- also the decision > itself was very doubtful also..... > > > And as it might happen: we have nothing more to do, then to look around > and accept all this without the hope of an appeal against that . That > makes as very upset !!! > And I really cant understand, why Ton says, it needs really > sportsmanship when you loose a match. If he meant our team, I think with > this report most of you might understand, why > we were not satisfied to loose the match. I am allways (!) a good > sportsman - but if results from a match comes not from the bridgetable > but from , lets say, some very unlucky circumstansces around the > bridgetable - yes- > then its not so eays to show good sportsmanship- cause we were not given > this sportmanship before at all- by not hearing us and with all what I > wrote. > > Michael Grom?ller, german captain. > > If the facts are anything at all like how Michael Grom?ller reports them then I hope that the TD gets fired. I would expect that he would resign before then. > > > _______________________________________________ > blml mailing list > blml at amsterdamned.org > http://www.amsterdamned.org/mailman/listinfo/blml > -- Wayne Burrows Palmerston North New Zealand From harsanyi at t-online.de Tue Nov 11 06:42:09 2008 From: harsanyi at t-online.de (Josef Harsanyi) Date: Tue, 11 Nov 2008 06:42:09 +0100 Subject: [blml] Ac in Amasterdam In-Reply-To: <4918F39D.8070708@bridgepro.de> References: <4918F39D.8070708@bridgepro.de> Message-ID: <000001c943c0$3da81fe0$b8f85fa0$@de> The facts described by Michael is confirmed by members of the German Team. The German Bridge Federation shall submit a complaint to the Standing Appeals Committee of the EBL applying for a consequent regulation of the case. All of us are eagerly awaiting the cleaning up of this case, to get the right judgment and to finish the conflict, because we want to enjoy the strong play of our opponents at the tables, enjoy tournaments without reminiscence of this scandal. Best regards Josef Harsanyi -----Urspr?ngliche Nachricht----- Von: blml-bounces at amsterdamned.org [mailto:blml-bounces at amsterdamned.org] Im Auftrag von Gro Gesendet: Dienstag, 11. November 2008 03:53 An: blml at amsterdamned.org Betreff: [blml] Ac in Amasterdam Here are the facts from my point of view: All starts when germany made an appeal against Italy. There was hesitation on both sides of the screen and the TD was called. For one hesitation the "right" side made the call , that the tray was very long at the other side. The TD accepted this. For the second hesitation the "wrong side" called -- so- the side where the hesitation took place. The TD did NOT accept this, cuase he told the players, that the wrong side called and so they don't take this facts and also will not rule about this. So the TD only decides the call from the right side and his decision was, that the bid is normal and the score stands. Against this decision germany appealed. This appeal was lost by germany, cause AC followed the TD-decision. after 4 segments of play the result was, that germany has won against italy at the table. Now the TD comes and said: italy has 2 appeals and we should come to the AC immediately. The AC told us, that both appeals will be handled in a row. So the normal procedure starts, TD gives the facts and why he ruled- the players were asked statements and had to answer questions from the AC. With the second appeal it was exactly the same. Now both could go out and wait for the decisions. After 5 minutes the TD tells us, to come in again, cause italy has a 3^rd appeal. The TD shows the hand .now italy was asked why they make this appeal. They said, that there was a hesitation and with this hesitation the bid from the other side is not "possible". The german were asked, if they agree, that there was a hesitation and they agreed, saying it was a very difficult decision, which needs time.Now the AC says, that they will ask the TD to make a decision and afterwards will decide about that appeal.. Both were send out now. After about 15 minutes the TD comes and gives us the decisions: appeal 1: itlay lost- score stands appeal 2: italy lost, score stands appeal 3: italy wins, score corrected- and italy wins the whole match now! Then I asked the TD: "what was the decision from you?" He said: "there was no decision from the TD- we gave the finding of the decision to the AC.". I asked : " why couldn't we say something about that? The AC said to us, that they wait on the TD decision and then the appeal can happen." TD said: " yes- but we told the AC that the TD will not decide, cause any other will appeal in any case- so we let the decion to the AC himself." I said: " and why couldn't we add something before the appeal? we thought we can say some words after the TD-decision and get heared what we can add and say." Td said: " you have no right to get heared!" I asked him: "what? we have no right to get heared anymore? That's an european championship- and we speaking about reaching the final or not- and you sure we can add nothing?" Then Mr. Koiman said: "no- you are out- you had no right of getting heared- and they want to close now here!" I said: "I am sorry- I cant believe this -- we are out now, cause we couldn't tell the AC something about this 3^rd appeal? We could not know, that its impossible to say something after you send us out with the words: first the TD must take a decision." " and now you telling us, that you didn't decide anything and we are also not allowed to say something???? " Koiman said: "you out. the AC decided- and against this decision you cant do anything! " I said: " oh..I want to appeal this! (not against the ruling of the AC- but against the procedure of how it comes to this ruling)" . Mr Melander from sweden was standing byside and hears all the conversation and I think he felt very uncomfortable now about his own decision , so he finally asked Mr. koiman, if we can appeal against this. Mr. Koiman said: " no- but if you find someone in amsterdam now , you can give him your appeal- I have to take my taxi now, bye!" .............. I have no idea, if the TD can just give the right of decision to the AC ? But I just cannot understand, why the 3^rd appeal could ever happen. With our appeal, the TD told us, that he will take no notice for the one hesitation, cause the wrong side was calling! And now, this 3^rd appeal was: Italy comes at the end of the session t the TD and reports, that in 1 board was a hesitation at one side of the screen. This must have beeen the german player. The TD took this note. And at the end this was the 3^rd appeal. What I now don't understand: in our appeal we were not allowed to use inforamtion , cause wrong side called. in italy appeal, NOTHING was said at the table -- NO TD was called -- and NOW the TD accepts this???? Without knowing if the wrong or right side "called" even there was NO call at all...??..is that ok and logical?.. If I understand this right, then we were punished, cause wrong side called---and with italy, NOBODY called and the the TD says: ok- we take your point? in my sense of logic, this appeal could never happened at all. At the next day I said to the TD before we started: "we play this match with protest. We wanted appeal yesterday night but were not allowed to do so.!" "We didn't want to appeal agianst the decision, cause its not possible, but againts the procedure before. We think, if we could tell this facts to the AC ( the facts, that there was no TD and no call at all- and therefore the appeal should never been allowed at all), then the AC might have found another decion- maybe the decision, that they will not talk about this at all, cause of no call at the table and no call to the TD. that were all the facts. _Now my personal meaning:_ Even if the TD are allowed to transfer a decision to the AC, its very poor to do it in an european Halffinal. Even if the players have no technical right to say some words, they should have heared the players- cause again its an european halffinal- it would have taken about 5 minutes. Its NOT our fault, that the italians made 2 appeals that took a lot of time. what can we do now? The german team is very frustrated about this happening. loosing an appeal without getting heared- loosing a final, cause TD let it through, that the opps made a statement at the end of session without sayiing anything at the table before abut this hesitation. We just rember last year championcup: italy made an appeal against us with hesitation again and won it- this decisoin last year was very very close.And with this deciosn we lost by 5, instead of winning the match. Now we loose again with 5 cause of AC committee decision. We asked Mr. Latala about this appeal and the decision of the AC. He said. " that decision is a big scandal! And you can tell this with my name on it to the AC ! "..... so..not only the appeal was doubful to come in -- also the decision itself was very doubtful also..... And as it might happen: we have nothing more to do, then to look around and accept all this without the hope of an appeal against that . That makes as very upset !!! And I really cant understand, why Ton says, it needs really sportsmanship when you loose a match. If he meant our team, I think with this report most of you might understand, why we were not satisfied to loose the match. I am allways (!) a good sportsman - but if results from a match comes not from the bridgetable but from , lets say, some very unlucky circumstansces around the bridgetable - yes- then its not so eays to show good sportsmanship- cause we were not given this sportmanship before at all- by not hearing us and with all what I wrote. Michael Grom?ller, german captain. _______________________________________________ blml mailing list blml at amsterdamned.org http://www.amsterdamned.org/mailman/listinfo/blml From mustikka at charter.net Tue Nov 11 07:14:54 2008 From: mustikka at charter.net (raija) Date: Mon, 10 Nov 2008 22:14:54 -0800 Subject: [blml] Ac in Amasterdam References: <48DA26571AEAE6BF@mail-10-uk.mail.tiscali.sys> (addedbypostmaster@mail-10.uk.tiscali.com)<007201c9433a$9a302360$0302a8c0@Mildred> Message-ID: <8B48661EFEF146658361E5ADB7BB30CF@DFYXB361> I still lurk on blml every now and then though I do not post any more. But I must express opinion on this one. Kojak is right. When there is no TD ruling (and that fact appears to be _not in dispute_; so stated by Ton Koojiman, by the German team captain, and one German player) there can be no appeal and no AC decision. ----- Original Message ----- From: "WILLIAM SCHODER" To: "Bridge Laws Mailing List" Sent: Monday, November 10, 2008 1:05 PM Subject: Re: [blml] Ac in Amasterdam > > "....... Only then the AC started discussing all three cases......" (Ton's > latest > quote that I have) seems kind of strange to me. Why were the three > appeals grouped in discussion? It would seem to me that each case would be > decided and completed on its own merits, before going on to the next one. > > Yet I find no other reason to modify what I drafted this AM, which > follows: > > Not having all the facts; > > Ton's "liberal interpretation" of Law 83 is wrong, Why bother to follow > the > strictures of the Laws when there is an alternate less demanding route you > can take? Law 83 clearly states ".....if the Director feels that a REVIEW > OF > HIS DECISION......ETC....." - I fail to see how this authorizes anyone to > throw the appeals process out the window. Smells to me like politics > running > rampant. Even in English this means you have to make a decision. There was > no ruling? Why was there no ruling? I don't buy the argument > about being sure one side or the other would appeal anyhow? > > "......... There was no appeal........." So whatever happened at the table > stood BY LAW I would think. To say ".....Law 92C does not say that the > AC > has no power to hear a ruling > of first instance....." cleverly plants the idea that players and ACs can > do what they please in adjudicating disagreements without the Director. > That > is wrong. Rejection of the Code of Practice for Appeals Committees is just > one example of 'I know what is right, I don't need those things'. > > I'm well familiar with the attitude of those Regulating Authorities who > find > Directors' decisions are just a waste of time, and are eager to get the > 'case' to a group of individuals whose knowledge of the laws is often far > below their abilities to analyze and play bridge. ACBL, for example, > cancels > the Director's ruling at the beginning of the hearing, and the AC makes > its > own ruling! The ruling is not REVIEWED, it is CANCELLED, and the AC > starts > from scratch. > > I don't see this kind of action in other sports, -- the usual philosophy > is > to uphold the TD, Referee, Umpire, Judge, Arbiter, Official, etc. > decisions > unless there is substantial reason to change them. Seems like others have > greater faith in their trained officials than some bridge organizations > are > willing to accept. Respect for Laws, Rules, Regulations, and adherence > thereto restricts the flights of fancy of the know-it-alls who work hard > to > destroy our game. > > In my experience in WBF and many WBF zones and NBOs, I've often heard AC > chairmen refuse to add some other situation to a case when players > attempted > to do so. Bravo! Take into account each item as it occurred, decide on the > review of that ruling, and then go to the next item. It makes the overall > result a lot more palatable, creates less suspicion, less contention and > argument when the simple, direct Laws of Duplicate Bridge 2007 are > followed. > > For an AC to preempt the Directors duties throws a wrench > into the machinery and goes a long way to get the "do what feels good (to > me) at the > time" ambiance that some people thrive upon. To refuse to make a ruling > when > it was requested in accordance with the laws is something no competent > Director should ever consider - it simply boggles my mind! It puts the > scenario into a realm where > anything goes and breaks down a tested, established, and valuable process. > > I'd like to have all the FACTS of course, but it appears they remain in > dispute. > > Kojak > > >> From: "ton" > >> There was no appeal. But law 92C does not say that the AC >> has no power to hear a ruling of first instance. It just states that >> appeals should be brought to the appeals committee by the TD. >> > >> Imo we are dealing with L83 here, with a liberal interpretation. >> The TD's decided not to make a decision (which is a decision >> in itself) and referred the case to the AC. >> The reason was that any decision taken would be appealed >> anyway, that nobody was longing for more wasted time, not >> the teams involved either. >> > >> > ton > >>From Grattan: > >> +=+ I agree that if the Director refused to consider a request >> for a ruling that is a Director's decision that may be appealed. >> I do not agree that an AC has a power to make a decision of >> first instance if no application for a ruling has been made to a >> Director. Nor do I agree that the laws admit of any appeal >> that is not brought via the Director (93C is clear on this). An >> AC should ensure that the Director's function is not usurped, >> and it has been established previously by the WBFLC that a >> where a positive authority is not stated in the laws it is not to >> be assumed. >> The laws say with total clarity that an appeal must be an >> appeal of a Director's ruling. This is a point that Kojak has >> repeatedly addressed and I agree with him. It is not the function >> of the AC to stand in for a Director who has been bypassed. >> These points aside, the authoritative account of what >> occurred that ton has provided is significantly different from >> what we had read earlier. I have no direct knowledge of the >> case. >> ~ Grattan ~ +=+ >> >> >> _______________________________________________ >> blml mailing list >> blml at amsterdamned.org >> http://www.amsterdamned.org/mailman/listinfo/blml >> > > > _______________________________________________ > blml mailing list > blml at amsterdamned.org > http://www.amsterdamned.org/mailman/listinfo/blml From richard.hills at immi.gov.au Tue Nov 11 08:10:30 2008 From: richard.hills at immi.gov.au (richard.hills at immi.gov.au) Date: Tue, 11 Nov 2008 18:10:30 +1100 Subject: [blml] AC in Amsterdam [SEC=UNOFFICIAL] In-Reply-To: Message-ID: William Schoder ("Kojak"): [big snip] >To refuse to make a ruling when it was requested in accordance >with the laws is something no competent Director should ever >consider - it simply boggles my mind! It puts the scenario >into a realm where anything goes and breaks down a tested, >established, and valuable process. > >I'd like to have all the FACTS of course, but it appears they >remain in dispute. Richard Hills: It seems to me that Ton Kooijman does not dispute the particular fact that the relevant Director refused to make a ruling. In my opinion such refusal is not merely incompetence, but an actual Director's error unintentionally infracting Law, due to unLawful advice from the author of the forthcoming Appendix to the Laws. Ton Kooijman: [snip] >>Then the AC asked the TD's and me whether it was necessary to >>have a decision from the TD's on this case before they could >>handle it. Our answer was 'no'. [snip] Law 81B2: "The Director applies, **and is bound by**, these Laws and supplementary regulations announced under authority given in these Laws." Law 81C, first sentence: "The Director (not the players) has the **responsibility** for rectifying irregularities and redressing damage." Richard Hills: As Kojak has already noted, in order for a Law 83 review of a TD's decision to take place, first the TD must make a decision so that it may be reviewed. Another point is that this international championship must have had a regulation about appeals without merit. How could an Appeals Committee sensibly rule an appeal against a Director's decision to be without merit when there was no Director's decision in the first place? On the other hand..... Law 81D - Delegation of Duties: "The Director may delegate any of his duties to assistants, but he is not thereby relieved of responsibility for their correct performance." Richard Hills: It is Lawful for the Director to nominate all members of the Appeals Committee as her assistants. But in that case the AC members change their status to become Assistant Directors, hence their decision can be appealed to a "real" Appeals Committee. What's the problem? Well, a problem is one co-author of the new Lawbook giving unLawful advice, and another co-author of the new Lawbook stating "The procedural angles are relatively minor". Best wishes Richard James Hills Recruitment Section, Level 3 Blue, workstation 15 (first on left) Department of Immigration and Citizenship Telephone: 02 6223 8453 Email: richard.hills at immi.gov.au -------------------------------------------------------------------- Important Notice: If you have received this email by mistake, please advise the sender and delete the message and attachments immediately. This email, including attachments, may contain confidential, sensitive, legally privileged and/or copyright information. Any review, retransmission, dissemination or other use of this information by persons or entities other than the intended recipient is prohibited. DIAC respects your privacy and has obligations under the Privacy Act 1988. The official departmental privacy policy can be viewed on the department's website at www.immi.gov.au. See: http://www.immi.gov.au/functional/privacy.htm --------------------------------------------------------------------- From nigelguthrie at talktalk.net Tue Nov 11 08:51:10 2008 From: nigelguthrie at talktalk.net (Nigel Guthrie) Date: Tue, 11 Nov 2008 07:51:10 +0000 Subject: [blml] Deviations from announced system. In-Reply-To: <000601c9437e$cfe03f10$6fa0bd30$@no> References: <003501c940de$dfd49c60$0302a8c0@Mildred> <4914532F.1080100@skynet.be> <49145A04.2000707@ulb.ac.be> <49146907.9020204@skynet.be><49146E3D.8050904@ulb.ac.be> <49149C32.7050005@skynet.be><003701c94190$28575d40$0302a8c0@Mildred> <4916C87B.1080105@skynet.be><001701c9427e$7e4d08a0$0302a8c0@Mildred> <49180B27.9030002@skynet.be> <005c01c94336$fb4a1100$0302a8c0@Mildred> <491850E0.4000003@skynet.be> <002001c9434e$4e6742b0$eb35c810$@no> <491863D9.2090008@skynet.be> <000601c9437e$cfe03f10$6fa0bd30$@no> Message-ID: <4919396E.6090005@talktalk.net> [Sven Pran] It is not hypothetical when you continue repeating slander against the. Norwegian Bridge playing society with no other foundation than having heard or read some comments from a single spectator. [Nige1] If this occurred during an on-line viewgraph session, then both the table-incident and commentator-statements are a matter of public record. Please would somebody post the relevant system-card, hand-record, and chat-log to BLML so we can judge for ourselves whether there is a case that demands further investigation by the WBF. In any case, examination of previous public deal records should make it easy for Norway to nip any slander in the bud. IMO Methods should be properly disclosed but there shouldn't be any system restrictions; In the meantime, however, elite players are not unofficially exempted from rules with which ordinary players comply From grandaeval at tiscali.co.uk Tue Nov 11 02:46:32 2008 From: grandaeval at tiscali.co.uk (Grattan) Date: Tue, 11 Nov 2008 01:46:32 -0000 Subject: [blml] Deviations from announced system. [SEC=UNOFFICIAL] References: Message-ID: <000501c943d7$d6ce6550$0302a8c0@Mildred> Grattan Endicott To: "Bridge Laws Mailing List" Sent: Tuesday, November 11, 2008 12:46 AM Subject: Re: [blml] Deviations from announced system. [SEC=UNOFFICIAL] > > What's the problem? > > Yes, I know that disagreeing with Grattan is the problem, > and there is a better than 50% chance I will have to retract > my interpretation once Grattan unveils an obvious point that > I have missed in the new Law 40. > +=+ The only way to deal with this is to go back to an elementary tutorial on the Director's procedure for dealing with illegal calls. If I wake up relatively pain-free tomorrow I will examine what clarification I may add to this misunderstood subject. Making an illegal call is an infraction. If there is damage arising from it the adjusted score must return to the equity prior to the infraction i.e.the illegal call. This pre-empts any suggestion about misinformation. A significant part of the determination of a call's illegality is played by the applicable regulations on disclosure and on what partnership understandings are allowed and not allowed. When a call is disallowed score adjustment disregards entirely any outcome that might be reached via that call. ~ G ~ +=+ From grandaeval at tiscali.co.uk Tue Nov 11 09:53:54 2008 From: grandaeval at tiscali.co.uk (Grattan) Date: Tue, 11 Nov 2008 08:53:54 -0000 Subject: [blml] AC in Amsterdam [SEC=UNOFFICIAL] References: Message-ID: <001401c943db$107ac570$0302a8c0@Mildred> Grattan Endicott To: "Bridge Laws Mailing List" Sent: Tuesday, November 11, 2008 7:10 AM Subject: Re: [blml] AC in Amsterdam [SEC=UNOFFICIAL] > > What's the problem? Well, a problem is one co-author of the new > Lawbook giving unLawful advice, and another co-author of the new > Lawbook stating "The procedural angles are relatively minor". > +=+ Do not misunderstand. Minor in relationship to the mish-mash of conflicting statements about the facts and the procedure. They remain errors of procedure in conflict with the law book, and exercise of powers the laws do not grant, if they are as they appear to be. I do think the organizers have landed themselves in a mess, and I am concerned about a situation in which a contestant that has lost two appeals then appears to find grounds for a third that was not on the table at the start. I am surprised it was in time and that could be a reason for the Director to refuse to rule on it. ~ Grattan ~ +=+ From JffEstrsn at aol.com Tue Nov 11 10:01:52 2008 From: JffEstrsn at aol.com (Jeff Easterson) Date: Tue, 11 Nov 2008 10:01:52 +0100 Subject: [blml] AC in Amsterdam Message-ID: <49194A00.1010007@aol.com> I am puzzled. It is apparently not disputed that an earlier attempt to appeal was rejected on formal/technical grounds: that the appeal was made (or at least the TD called) by a player on the "wrong" side of the screen, that is, the side where the alleged hesitation took place. Thus I deduce that there was a regulation in force saying that an appeal (or TD call?) in such cases can only be made from the "other" side of the screen. This regulation is new to me but that is irrelevant. More relevant might be the fact that I don't understand the logic behind this regulation. When the tray comes back after a delay both sides of the screen know there has been a delay, thus probable hesitation. One side doesn't know who caused the delay; that side can act (appeal, call TD) the other side knows who caused the delay; they are not allowed to act. Can anyone explain to me the reason for this? But, to move to case 3. Apparently the call/appeal was made after play had concluded and the opponents had left the table. Thus it was at least theoretically possible that the player on the side of the screen where a hesitation was alleged (and confirmed later) told his partner: I was on the wrong side of the screen, I thus can't do anything. Okay, you call/appeal, you may. This is apparently legal. (?) Did anyone investigate why this appeal was not lodged until (apparently) long after play concluded? Generally when a hesitation occurs it is noticed immediately. Why was this one apparently only noticed after such a long time? JE From grandaeval at tiscali.co.uk Tue Nov 11 10:14:01 2008 From: grandaeval at tiscali.co.uk (Grattan) Date: Tue, 11 Nov 2008 09:14:01 -0000 Subject: [blml] AC in Amsterdam [SEC=UNOFFICIAL] References: Message-ID: <003101c943dd$d8881480$0302a8c0@Mildred> Grattan Endicott To: "Bridge Laws Mailing List" Sent: Tuesday, November 11, 2008 7:10 AM Subject: Re: [blml] AC in Amsterdam [SEC=UNOFFICIAL] > William Schoder ("Kojak"): > > [big snip] > >>To refuse to make a ruling when it was requested in accordance >>with the laws is something no competent Director should ever >>consider - it simply boggles my mind! It puts the scenario >>into a realm where anything goes and breaks down a tested, >>established, and valuable process. >> >>I'd like to have all the FACTS of course, but it appears they >>remain in dispute. > > Richard Hills: > > It seems to me that Ton Kooijman does not dispute the particular > fact that the relevant Director refused to make a ruling. In my > opinion such refusal is not merely incompetence, but an actual > Director's error unintentionally infracting Law, due to unLawful > advice from the author of the forthcoming Appendix to the Laws. > +=+ Slow down. We have not heard from the Director why he refused. He could have said, for example, "you are too late to make yet another appeal" - in which case the appeal would be about whether the appeal was in time. If the AC decided it was, the matter should be referred back to the Director for him to make a ruling. I absolutely insist that the laws do not give ACs the power to make rulings of first instance. Any appeal is an appeal from the ruling of a Director. ~ Grttan ~ +=+ From geller at nifty.com Tue Nov 11 10:15:52 2008 From: geller at nifty.com (Robert Geller) Date: Tue, 11 Nov 2008 18:15:52 +0900 Subject: [blml] AC in Amsterdam In-Reply-To: <49194A00.1010007@aol.com> References: <49194A00.1010007@aol.com> Message-ID: <200811110915.AA16465@geller204.nifty.com> Jeff Easterson writes: >I am puzzled. It is apparently not disputed that an earlier attempt to >appeal was rejected on formal/technical grounds: that the appeal was >made (or at least the TD called) by a player on the "wrong" side of the >screen, that is, the side where the alleged hesitation took place. Thus >I deduce that there was a regulation in force saying that an appeal (or >TD call?) in such cases can only be made from the "other" side of the >screen. This regulation is new to me but that is irrelevant. More >relevant might be the fact that I don't understand the logic behind this >regulation. When the tray comes back after a delay both sides of the >screen know there has been a delay, thus probable hesitation. One side >doesn't know who caused the delay; that side can act (appeal, call TD) >the other side knows who caused the delay; they are not allowed to act. > Can anyone explain to me the reason for this? -> I wasn't at all involved in the drafting of the regulation, but having played frequently with screens the reason seems obvious. A minor hesitation is quite perceptible to the two players on the same side of the screen where the BIT occurred, but when the tray is passed to the other side (where the receiver of (alleged) UI is located) the hesitation is not deemed worthy of noting unless a player on that side (the side oppositie to the two calls that were made before the tray was passed back) considers the BIT to have been sufficiently noteworthy to warrant calling the Director. -Bob ----------------------------------------------------- Robert (Bob) Geller, Tokyo, Japan geller at nifty.com From Hermandw at skynet.be Tue Nov 11 10:48:05 2008 From: Hermandw at skynet.be (Herman De Wael) Date: Tue, 11 Nov 2008 10:48:05 +0100 Subject: [blml] Mr Gladstone suggested [SEC=UNOFFICIAL] In-Reply-To: References: Message-ID: <491954D5.3070104@skynet.be> Richard, when you started this thread you criticized me for complaining that my arguments fon't get answered. So I ask three questions and what do I get - one non-responsive answer. Please do me the immense favor of answering yet: richard.hills at immi.gov.au wrote: > > Herman De Wael asked: > >> Richard, have you even thought of the following three >> questions: >> >> 1) How do you feel about a law that obliges you to give UI to >> your partner? >> Answers on a postcard, please. > > Postcard size answer from WBF LC minutes, 10th October 2008: > > "There is no infraction when a correct explanation discloses > that partner's prior explanation was mistaken. The words 'nor > may he indicate in any manner that a mistake has been made' (in > Law 20F5(a)) do not refer to compliance with the overriding > requirement of the laws always to respond to enquiries under > Law 20F with correct explanations of the partnership > understandings." > Allow me one comment: my question was: How do _YOU_ _FEEL_ (two separate pieces of emphasis) about a law that _obliges_ you to give UI. The WBF minutes say "there is no infraction". _You_ translate this into "you must" (I don't). How do you feel about such an obligation. And there were two other questions: >> 2) As TD, if there is a law that obliges someone to give UI >> to his partner, how do you rule if the player in front of you >> has refused to do so? >> 3) Quiz question: how many laws in the 2007 Lawbook >> explicitly forbid giving a piece of UI to partner? And how >> many oblige a player to do so? I would really like to hear your answer to that last one? > Herman De Wael asserted: > >> And indeed not what I wanted, but also not what is needed. I >> see no reason to change my intended habit if it ever came >> along at my particular table. No sense in saying what a player >> should do if there is no penalty to back it up. > > I see a reason to change in Law 72B1: > > "A player must not infringe a law intentionally, even if there > is a prescribed rectification he is willing to accept." > Posed 27 times in the past 10 years and answered 28 times. > I see a reason to change in Law 40C3(b): > > "Repeated violations of requirements to disclose partnership > understandings may be penalized." > A new one - but since I have never yet had a chance to explicitely act according to DWS, there can be no repetition. And after all, that was not what this law was intended for, was it. Richard, you were the one talking about axioms and foundations. Don't go looking for twigs in the thatched roof then. > I see a penalty to back it up in Law 91B - Right to Disqualify: > > "The Director is empowered to disqualify a contestant for > cause, subject to approval by the Tournament Organizer." > Well of course - I shall never be able to play bridge in Australia - I don't agree with Richard Hills, so I won't be able to enter the country in the first place! > What's the problem? > > > Best wishes > > Richard James Hills > Department of Immigration and Citizenship Herman. From grandaeval at tiscali.co.uk Tue Nov 11 10:47:44 2008 From: grandaeval at tiscali.co.uk (Grattan) Date: Tue, 11 Nov 2008 09:47:44 -0000 Subject: [blml] AC in Amsterdam References: <49194A00.1010007@aol.com> <200811110915.AA16465@geller204.nifty.com> Message-ID: <004a01c943e2$8ec317f0$0302a8c0@Mildred> Grattan Endicott To: "Bridge Laws Mailing List" Sent: Tuesday, November 11, 2008 9:15 AM Subject: Re: [blml] AC in Amsterdam -> I wasn't at all involved in the drafting of the regulation, but having played frequently with screens the reason seems obvious. A minor hesitation is quite perceptible to the two players on the same side of the screen where the BIT occurred, but when the tray is passed to the other side (where the receiver of (alleged) UI is located) the hesitation is not deemed worthy of noting unless a player on that side (the side oppositie to the two calls that were made before the tray was passed back) considers the BIT to have been sufficiently noteworthy to warrant calling the Director. > +=+ Even some longer delays are not particularly noticed on the receiving side of the screen. At WBF level even before we incorporated it into the CoCs the Directors did observe that if the delay were drawn by the side where it occurred to the attention of the receiving side who had apparently not noticed it there was little evidence that it could have influenced the action on the receiving side. ~ Grattan ~ +=+ From Hermandw at skynet.be Tue Nov 11 10:50:34 2008 From: Hermandw at skynet.be (Herman De Wael) Date: Tue, 11 Nov 2008 10:50:34 +0100 Subject: [blml] Deviations from announced system. [SEC=UNOFFICIAL] In-Reply-To: References: Message-ID: <4919556A.9020308@skynet.be> It is possible folks! richard.hills at immi.gov.au wrote: > Grattan Endicott asserted: > > [snip] > >>> if there is a breach of Law 40A3 because the player makes >>> a call that is based on an undisclosed partnership >>> understanding, that call is illegal. > > [snip] > >>> It is too late and absolutely against the law for the >>> Director to take account of what could have happened if >>> opponents had been told of the partnership understanding. >>> ~ Grattan ~ +=+ > > > Richard Hills quibbles: > > No, in my opinion Grattan is really wrong. > > Law 40A3: > > "A player may make any call or play without prior > announcement provided that such call or play is not based on > an undisclosed partnership understanding (see Law 40C1)." > > Law 40B4: > > "A side that is damaged as a consequence of its opponents' > failure to provide disclosure of the meaning of a call or > play as these laws require, is entitled to rectification > through the award of an adjusted score." > > Richard Hills quibbles: > > It seems to me that an infraction of Law 40A3 is identical > to an infraction of Law 40B4. In both cases the infractions > seem to me to be simply standard misinformation (MI). > I fully agree with Richard. > (Of course, if the MI conceals an intrinsically unLawful > understanding - a so-called "illegal convention" - then Law > 40B5 would take precedence over Law 40B4.) > Of course, but that's another ruling entirely. > And the standard resolution for a MI ruling is not to deem > that the undisclosed call was never made, but rather to > determine what would have happened if the non-offending side > had been informed at the appropriate time. > > What's the problem? > > Yes, I know that disagreeing with Grattan is the problem, No, having Herman agree must be a kiss of death. > and there is a better than 50% chance I will have to retract > my interpretation once Grattan unveils an obvious point that > I have missed in the new Law 40. > > > Best wishes > > Richard James Hills Herman. From Hermandw at skynet.be Tue Nov 11 10:55:24 2008 From: Hermandw at skynet.be (Herman De Wael) Date: Tue, 11 Nov 2008 10:55:24 +0100 Subject: [blml] Deviations from announced system. [SEC=UNOFFICIAL] In-Reply-To: <000501c943d7$d6ce6550$0302a8c0@Mildred> References: <000501c943d7$d6ce6550$0302a8c0@Mildred> Message-ID: <4919568C.70708@skynet.be> Grattan wrote: > >> > +=+ The only way to deal with this is to go back to an elementary > tutorial on the Director's procedure for dealing with illegal calls. > If I wake up relatively pain-free tomorrow I will examine what > clarification I may add to this misunderstood subject. > Making an illegal call is an infraction. If there is damage arising > from it the adjusted score must return to the equity prior to the > infraction i.e.the illegal call. This pre-empts any suggestion about > misinformation. A significant part of the determination of a call's > illegality is played by the applicable regulations on disclosure and > on what partnership understandings are allowed and not allowed. > When a call is disallowed score adjustment disregards entirely > any outcome that might be reached via that call. > ~ G ~ +=+ > Yes Grattan, we understand that. But the laws have for 20 years used the same construction for simple MI cases "it is illegal to use a system that opponents don't know", yet we have never (well, some tried) used this to go back to the situation before the undisclosed call. If the WBF have now intended to invent a new class of MI, with a new kind of adjustment, then they should have said so, and not simply rely on the change of wording in a another law to explain a change of interpretation in this one. And if the WBF really wanted there to be two radically different adjustments to two quite similar infractions, then they should at least have begun to spell out what cases are to be treated in one way and what cases in the other one. Really Grattan, I believe you have misinterpreted the text you wrote yourself. And if your interpretation was what you wanted all along, then you have written it very badly. Herman. From Hermandw at skynet.be Tue Nov 11 11:07:55 2008 From: Hermandw at skynet.be (Herman De Wael) Date: Tue, 11 Nov 2008 11:07:55 +0100 Subject: [blml] Ac in Amasterdam In-Reply-To: References: <48DA26571AEAE6BF@mail-10-uk.mail.tiscali.sys> (added bypostmaster@mail-10.uk.tiscali.com) <007201c9433a$9a302360$0302a8c0@Mildred> Message-ID: <4919597B.4040105@skynet.be> WILLIAM SCHODER wrote: > "....... Only then the AC started discussing all three cases......" (Ton's > latest > quote that I have) seems kind of strange to me. Why were the three > appeals grouped in discussion? It would seem to me that each case would be > decided and completed on its own merits, before going on to the next one. > Maybe they did, and Ton can't tell us because he was not in there with them? And are you going to disallow an AC to discuss the cases in the order they want them to? Maybe they had different opinions about cases one and two and moved on to case three - when they agreed on that one they saw they no longer needed to deal with the other two and gave a majority ruling without further discussion. That is their right. As long as both sides had a chance to have their say to the committee, I see no reason to criticize or even discuss what happened behind closed walls. > Yet I find no other reason to modify what I drafted this AM, which > follows: > > Not having all the facts; > > Ton's "liberal interpretation" of Law 83 is wrong, Why bother to follow the > strictures of the Laws when there is an alternate less demanding route you > can take? Law 83 clearly states ".....if the Director feels that a REVIEW OF > HIS DECISION......ETC....." - I fail to see how this authorizes anyone to > throw the appeals process out the window. Smells to me like politics running > rampant. Even in English this means you have to make a decision. There was > no ruling? Why was there no ruling? I don't buy the argument > about being sure one side or the other would appeal anyhow? > Bill, if I were the director coming with this case, and you were on the AC, and you objected to my not making a ruling, I would take out a coin, toss, say "Italy (or Germany) win the ruling" and then ask the others if they want to appeal. I will be very glad if you tell me that my ruling method was wrong, but I'll have gained valuable minutes of YOUR time. > "......... There was no appeal........." So whatever happened at the table > stood BY LAW I would think. To say ".....Law 92C does not say that the AC > has no power to hear a ruling > of first instance....." cleverly plants the idea that players and ACs can > do what they please in adjudicating disagreements without the Director. That > is wrong. Rejection of the Code of Practice for Appeals Committees is just > one example of 'I know what is right, I don't need those things'. > > I'm well familiar with the attitude of those Regulating Authorities who find > Directors' decisions are just a waste of time, and are eager to get the > 'case' to a group of individuals whose knowledge of the laws is often far > below their abilities to analyze and play bridge. ACBL, for example, cancels > the Director's ruling at the beginning of the hearing, and the AC makes its > own ruling! The ruling is not REVIEWED, it is CANCELLED, and the AC starts > from scratch. > In that sense of course, you are right. Ton should have made a ruling (even if only by the toss of a coin) so that the AC could, in fine, decide to go with the director if they are 50/50 as well. An EBLAC once did that - having no idea whatsoever and finally just siding with the director. That was impossible here, but what's the problem if the two sides agree to go the AC in stead. They too want to go to dinnenr and not have to tell everything twice. But you're right. The AC should have asked the TD, after hearing the case, to give a ruling. > I don't see this kind of action in other sports, -- the usual philosophy is > to uphold the TD, Referee, Umpire, Judge, Arbiter, Official, etc. decisions > unless there is substantial reason to change them. Seems like others have > greater faith in their trained officials than some bridge organizations are > willing to accept. Respect for Laws, Rules, Regulations, and adherence > thereto restricts the flights of fancy of the know-it-alls who work hard to > destroy our game. > > In my experience in WBF and many WBF zones and NBOs, I've often heard AC > chairmen refuse to add some other situation to a case when players attempted > to do so. Bravo! Take into account each item as it occurred, decide on the > review of that ruling, and then go to the next item. It makes the overall > result a lot more palatable, creates less suspicion, less contention and > argument when the simple, direct Laws of Duplicate Bridge 2007 are followed. > > For an AC to preempt the Directors duties throws a wrench > into the machinery and goes a long way to get the "do what feels good (to > me) at the > time" ambiance that some people thrive upon. To refuse to make a ruling when > it was requested in accordance with the laws is something no competent > Director should ever consider - it simply boggles my mind! It puts the > scenario into a realm where > anything goes and breaks down a tested, established, and valuable process. > I agree with you on principle. I agree with Ton in practice. It is somewhat strange to see the American go with strict application of procedure and the European going with the flow of things. Maybe the American is not familiar with this tournament: it is the finals of the European Teams cup. The National Champions of 10 countries come together and play over a long weekend. Very high level bridge in very pleasant surroundings, with dinners galore. > I'd like to have all the FACTS of course, but it appears they remain in > dispute. > > Kojak > Herman. From Hermandw at skynet.be Tue Nov 11 11:23:50 2008 From: Hermandw at skynet.be (Herman De Wael) Date: Tue, 11 Nov 2008 11:23:50 +0100 Subject: [blml] Ac in Amasterdam In-Reply-To: <4918F39D.8070708@bridgepro.de> References: <4918F39D.8070708@bridgepro.de> Message-ID: <49195D36.7090201@skynet.be> Hello Michael, I completely understand your frustrations. Indeed, after hearing both your and Ton's comments, I can see what went wrong. When the third case came about, the TD decided to go directly into the AC room. There is nothing wrong there. There, the facts were presented. Again, nothing there. But then of course, in practice, the TD should have made a decision, asked you if you wanted to appeal and only then have the case discussed by the AC. Now surely you can see that coming out after 5 minutes, ask whomever was losing if they wanted to appeal and then going back in would have been only a formality - surely you would both have appealed the decision. So don't feel terribly hard about that, Michael. Also the fact that the TD, in the end, tells you that he did not in fact make a (anyway preliminary) decision, should not be terribly frustrating for you - after all, if you had insisted against me, I would have tossed a coin. What you are - possibly correctly - angry about is the fact that you may have been unaware that you would only be heard once. If the AC did not make this clear to you, you might have reason to complain. But if the hearing was conducted by the AC Chairman, and not by the TD, you could have known that this was a once-off. Also, I am quite certain the AC asked you before leaving "have you anything to add?" and allowed you to speak. To complain that you then did not have another chance to speak 15 minutes later is not OK. All in all, there have been some procedural errors, but I don't think the Germans (or Italians) were damaged as a result. Allow me to congratulate you, Michael, and your team-mates, for producing what should rank as a top performance against most probably the best team in the world. Herman (the non-German) Gro wrote: > Here are the facts from my point of view: > [I'll snip them] From Hermandw at skynet.be Tue Nov 11 11:28:20 2008 From: Hermandw at skynet.be (Herman De Wael) Date: Tue, 11 Nov 2008 11:28:20 +0100 Subject: [blml] AC in Amsterdam In-Reply-To: <49194A00.1010007@aol.com> References: <49194A00.1010007@aol.com> Message-ID: <49195E44.9070300@skynet.be> Good point Jeff. Yes, the EBL screen regulations say that a TDcall made by the screen-mate of the hesitator shall be regarded with more suspicion. I am quite certain the TDs involved followed this regulation, and that they also followed the (non-written) corollary. A TD Call from the wrong side can be taken into account if all the players agree that the hesitation was there. If it takes 5 minutes, I don't care who calls the TD. I'm confident the TD and AC took this into account. And mind you, I don't think the Germans were forbidden to appeal the ruling made on the call from the wrong side. Only that they got no adjustment since the TD could not ascertain that there was any UI. Jeff Easterson wrote: > I am puzzled. It is apparently not disputed that an earlier attempt to > appeal was rejected on formal/technical grounds: that the appeal was > made (or at least the TD called) by a player on the "wrong" side of the > screen, that is, the side where the alleged hesitation took place. Thus > I deduce that there was a regulation in force saying that an appeal (or > TD call?) in such cases can only be made from the "other" side of the > screen. This regulation is new to me but that is irrelevant. More > relevant might be the fact that I don't understand the logic behind this > regulation. When the tray comes back after a delay both sides of the > screen know there has been a delay, thus probable hesitation. One side > doesn't know who caused the delay; that side can act (appeal, call TD) > the other side knows who caused the delay; they are not allowed to act. > Can anyone explain to me the reason for this? > But, to move to case 3. Apparently the call/appeal was made after play > had concluded and the opponents had left the table. Thus it was at > least theoretically possible that the player on the side of the screen > where a hesitation was alleged (and confirmed later) told his partner: I > was on the wrong side of the screen, I thus can't do anything. Okay, you > call/appeal, you may. This is apparently legal. (?) > Did anyone investigate why this appeal was not lodged until (apparently) > long after play concluded? Generally when a hesitation occurs it is > noticed immediately. Why was this one apparently only noticed after > such a long time? JE > > _______________________________________________ > blml mailing list > blml at amsterdamned.org > http://www.amsterdamned.org/mailman/listinfo/blml > From wjburrows at gmail.com Tue Nov 11 11:51:54 2008 From: wjburrows at gmail.com (Wayne Burrows) Date: Tue, 11 Nov 2008 23:51:54 +1300 Subject: [blml] Deviations from announced system. In-Reply-To: <4919396E.6090005@talktalk.net> References: <003501c940de$dfd49c60$0302a8c0@Mildred> <4916C87B.1080105@skynet.be> <001701c9427e$7e4d08a0$0302a8c0@Mildred> <49180B27.9030002@skynet.be> <005c01c94336$fb4a1100$0302a8c0@Mildred> <491850E0.4000003@skynet.be> <002001c9434e$4e6742b0$eb35c810$@no> <491863D9.2090008@skynet.be> <000601c9437e$cfe03f10$6fa0bd30$@no> <4919396E.6090005@talktalk.net> Message-ID: <2a1c3a560811110251n324ffe1fmee30153381be6e3d@mail.gmail.com> 2008/11/11 Nigel Guthrie : > [Sven Pran] > It is not hypothetical when you continue repeating slander against the. > Norwegian Bridge playing society with no other foundation than having > heard or read some comments from a single spectator. > > > [Nige1] > If this occurred during an on-line viewgraph session, then both the > table-incident and commentator-statements are a matter of public record. > Please would somebody post the relevant system-card, hand-record, and > chat-log to BLML so we can judge for ourselves whether there is a case > that demands further investigation by the WBF. In any case, examination > of previous public deal records should make it easy for Norway to nip > any slander in the bud. > > IMO Methods should be properly disclosed but there shouldn't be any > system restrictions; In the meantime, however, elite players are not > unofficially exempted from rules with which ordinary players comply > I believe that I have posted the hand, the table commentary and some private commentary that I had and at some point quoted any relevant sections I could find on their system card. -- Wayne Burrows Palmerston North New Zealand From t.kooyman at worldonline.nl Tue Nov 11 12:52:39 2008 From: t.kooyman at worldonline.nl (ton) Date: Tue, 11 Nov 2008 12:52:39 +0100 Subject: [blml] Ac in Amasterdam In-Reply-To: Message-ID: Kojak: "....... Only then the AC started discussing all three cases......" (Ton's latest quote that I have) seems kind of strange to me. Why were the three appeals grouped in discussion? It would seem to me that each case would be decided and completed on its own merits, before going on to the next one. ton: Why?, do you find this in the laws too? Kojak: Yet I find no other reason to modify what I drafted this AM, which follows: Not having all the facts; Ton's "liberal interpretation" of Law 83 is wrong, Why bother to follow the strictures of the Laws when there is an alternate less demanding route you can take? Law 83 clearly states ".....if the Director feels that a REVIEW OF HIS DECISION......ETC....." - I fail to see how this authorizes anyone to throw the appeals process out the window. Smells to me like politics running rampant. Even in English this means you have to make a decision. There was no ruling? Why was there no ruling? I don't buy the argument about being sure one side or the other would appeal anyhow? ton: In the past years Kojak has so often decided that I was wrong were it appeared that I wasn't, that I tend to be happy with such statement. The first question goes to Kojak, Grattan, Stefan (with a similar strong opinion): What do you do if the TD decides for one reason or another not to take a decision? Don't try with: 'then we search for another TD'. Second question (with a more pragmatic approach): What makes the TD producing such decision superior to taking the case itself ('it' being the AC)? ton From henk at ripe.net Tue Nov 11 13:12:04 2008 From: henk at ripe.net (Henk Uijterwaal) Date: Tue, 11 Nov 2008 13:12:04 +0100 Subject: [blml] Ac in Amasterdam In-Reply-To: <20081111115256.49E3E2F583@herring.ripe.net> References: <20081111115256.49E3E2F583@herring.ripe.net> Message-ID: <49197694.204@ripe.net> ton wrote: > Kojak: > > "....... Only then the AC started discussing all three cases......" (Ton's > latest quote that I have) seems kind of strange to me. Why were the three > appeals grouped in discussion? It would seem to me that each case would be > decided and completed on its own merits, before going on to the next one. In general, I think that the AC should have the freedom to organize their work in the most efficient way. In this specific case: There was 1 AC, all 3 appeals were from the same match, with the same teams involved. It was known that as soon as the Italian team won one of the 3 appeals, then they'd win the match, regardless of the decision on the other cases. Finally, it was quite late in the evening. First hearing all appeals before starting the delibrations, does make sense to me. Henk -- ------------------------------------------------------------------------------ Henk Uijterwaal Email: henk.uijterwaal(at)ripe.net RIPE Network Coordination Centre http://www.amsterdamned.org/~henk P.O.Box 10096 Singel 258 Phone: +31.20.5354414 1001 EB Amsterdam 1016 AB Amsterdam Fax: +31.20.5354445 The Netherlands The Netherlands Mobile: +31.6.55861746 ------------------------------------------------------------------------------ Ceterum censeo Asplain esse delendam (Cato & Henk) From gro at bridgepro.de Tue Nov 11 13:34:00 2008 From: gro at bridgepro.de (Gro) Date: Tue, 11 Nov 2008 13:34:00 +0100 Subject: [blml] Ac in Amasterdam References: 4918F39D.8070708@bridgepro.de Message-ID: <49197BB8.1030507@bridgepro.de> Hi Herman, Herman wrote: "What you are - possibly correctly - angry about is the fact that you may have been unaware that you would only be heard once. If the AC did not make this clear to you, you might have reason to complain. But if the hearing was conducted by the AC Chairman, and not by the TD, you could have known that this was a once-off. Also, I am quite certain the AC asked you before leaving "have you anything to add?" and allowed you to speak. To complain that you then did not have another chance to speak 15 minutes later is not OK." Michael: They did not make that clear, cause they said: Before we start with an appeal we need a decision from the TD. . Then we were send out. Nobody say before we got out: "anyone wants to say something, last chance!?" Cause in THAT moment there was NO TD-Decison at all, to which we could add something. So it was impossible for us to think we will NOT get heared again AFTER the TD-decison, cause the "normal procedure" is to get heared AFTER the TD-decison...or? And you really think that our complaint 15 minutes later, without having a TD-decison before is not ok?? I cant imagine that. Herman: " All in all, there have been some procedural errors, but I don't think the Germans (or Italians) were damaged as a result." Michael: we were not damaged? I told in my statement, that about in this 3rd appeal there was no call at the table and no TD was there, taking the facts from that hesitation. The TD was informed, after that segment, without beeing called at the table. There are of course possibilties, where you can call a TD after the session behind screeens- as an example that they were differnet explanations behind the screens. BUT to inform TD for a probable hesitation AFTER the segement- looks impossible for us! Especially when in our first appeal the ruling from the same TD was: you are NOT allowed to use the hesitation, cause you called from the wrong side -- and now it doesnt matter at all ??? thats very unlocigal and ununderstandable for us..or I am so wrong? And..after the TD was informed about this hesitation -AND if this is allowed at all ( what we dont believe)..why there was no decision on that from the TD ? Our players were not informed about the call form the italians afterwards. And of course also not heared by the TD if the facts are correct. Me as a captain was told form the TD: Italians complained about a board on that session about hesitaion. I was not told which board- I did not get a written TD-decison - I did not get a decison at all - cause there was no decison ( and as we know now- there was TD-decison till the end at all!) thank you for your gratulation. We beat Italy in 2006, 2007 and 2008 in the championscup halffinal. We won the tourney in 2006 in 2007 and 2008 the AC tooks away the imps and we both lost with 5 imps! in 2007 all the procedures were correct and the decison of the AC was close, but the decision was quite ok- it costs as 6 imps... in 2008 the most of the procedure was not correct AND the decison itself was very dubious. Today the german bridge federation appealed against the technical/formal circumstances at the EBL-AC greetings Michael P.s. one of our players spoke with EBL-tournament director Latala today again, and he just says, that he said to us yesterday about the decison looks like a scandal only for the ears from our team ! He didnt want to adress that to the AC. So I herebye correct this! From t.kooyman at worldonline.nl Tue Nov 11 14:10:07 2008 From: t.kooyman at worldonline.nl (ton) Date: Tue, 11 Nov 2008 14:10:07 +0100 Subject: [blml] AC in Amsterdam [SEC=UNOFFICIAL] In-Reply-To: <003101c943dd$d8881480$0302a8c0@Mildred> Message-ID: I absolutely insist that the laws do not give ACs the power to make rulings of first instance. Any appeal is an appeal from the ruling of a Director. ~ Grttan ~ +=+ ton: I have the feeling that you (Grattan) still miss the point. Of course an appeal is an appeal from a ruling of a director. This is wel known rhetoric. The question is whether an appeal committee may hear and decide upon a case in which the TD did not actively give his ruling, but just referred the case to that committee. And my answer on this question still is 'yes'. Appeal committees do this regularly without anybody ever denying that power in cases of procedural or disciplinary penalties. The TD informs the committee about unsportmanship behavior without exercising his rights under L 90A and asks the AC to take action. It is my opinion that people denying this possible procedural way are splitting hairs and are not focussed on the main job a TD has: running the tournament smoothly and striving for the best possible decisions in case of technical irregularities. I do know several cases in EBL and WBF events were the procedure was the same. I need more content oriented arguments to continue this discussion ton From matthias.schueller at gmx.de Tue Nov 11 15:47:25 2008 From: matthias.schueller at gmx.de (=?ISO-8859-1?Q?Matthias_Sch=FCller?=) Date: Tue, 11 Nov 2008 15:47:25 +0100 Subject: [blml] Ac in Amasterdam In-Reply-To: <49197BB8.1030507@bridgepro.de> References: 4918F39D.8070708@bridgepro.de <49197BB8.1030507@bridgepro.de> Message-ID: <49199AFD.3050904@gmx.de> Gro wrote: > Hi Herman, > > Herman wrote: > "What you are - possibly correctly - angry about is the fact that you may > have been unaware that you would only be heard once. If the AC did not > make this clear to you, you might have reason to complain. But if the > hearing was conducted by the AC Chairman, and not by the TD, you could > have known that this was a once-off. > Also, I am quite certain the AC asked you before leaving "have you > anything to add?" and allowed you to speak. To complain that you then > did not have another chance to speak 15 minutes later is not OK." > > Michael: > They did not make that clear, cause they said: > Before we start with an appeal we need a decision from the TD. . > Then we were send out. > Nobody say before we got out: "anyone wants to say something, last chance!?" > Cause in THAT moment there was NO TD-Decison at all, to which we could > add something. > So it was impossible for us to think we will NOT get heared again AFTER > the TD-decison, cause > the "normal procedure" is to get heared AFTER the TD-decison...or? > And you really think that our complaint 15 minutes later, without having > a TD-decison before is not ok?? > I cant imagine that. > > Herman: > " All in all, there have been some procedural errors, but I don't think > the Germans (or Italians) were damaged as a result." > > Michael: > we were not damaged? > I told in my statement, that about in this 3rd appeal there was no call > at the table and no TD was there, taking the facts from that hesitation. > The TD was informed, after that segment, without beeing called at the table. > There are of course possibilties, where you can call a TD after the > session behind screeens- > as an example that they were differnet explanations behind the screens. > > BUT to inform TD for a probable hesitation AFTER the segement- looks > impossible for us! > Especially when in our first appeal the ruling from the same TD was: you > are NOT allowed to use the hesitation, cause you called > from the wrong side -- and now it doesnt matter at all ??? thats very > unlocigal and ununderstandable for us..or I am so wrong? > > And..after the TD was informed about this hesitation -AND if this is > allowed at all ( what we dont believe)..why there was no decision on > that from the TD ? Of course it is allowed to ask for a ruling -- this is an absolute right within the correction period, ie 30 minutes after the official score has been made available (in the absence of a regulation specifying a different time frame). A ruling will always be given; but the timing of the director call can materially affect the ruling (easy examples can be found in the technical laws, e.g. the point from which a revoke is not rectified by L64A but by L64C). There are some irregularities, most notably different explanations behind screens, that will often be discovered only after the session. And as long as the facts can still be established, the TD's ruling will be the same regardless of when he was called. But for other irregularities this is not true, and the possible use of UI (particularly through breaks in tempo) is one of them. This has to do with two things: a) it will typically be more difficult to establish the relevant facts later; b) the thing about "calling the TD from the wrong side of the screen". Now, a call from the "wrong" side of the screen, i.e. the side where the break in tempo happened, does not mean that no ruling will be given. The TD will still establish the facts -- but the score will be adjusted only if it is clear beyond all doubt that a) a very significant break in tempo occured; b) the break in tempo must have been obvious to the receiving side of the screen; c) only one of the players could have been responsible for the break in tempo. While all these things are always important, the fact that the player on the receiving side of the screen (correctly) calls the TD definitely points to a) and b) being true, i.e. the ruling will much more often be based on the assumption that there was a noticeable break in tempo. As soon as the players have (or could have) talked about the situation, it is, of course, immaterial who calls the TD in the end. I have often been called to a table where North had (possible) broken tempo; after the bidding East told West "Call the TD please; I can't do it". Now, this will of course be treated like a call from the "wrong" side -- i.e. a ruling will still be given, but the score will be adjusted in very obvious cases only. And the same principle will be followed if the TD was first notified of the problem after the end of the session. Matthias > Our players were not informed about the call form the italians > afterwards. And of course also not heared by the TD if the facts are > correct. > Me as a captain was told form the TD: Italians complained about a board > on that session about hesitaion. > I was not told which board- I did not get a written TD-decison - I did > not get a decison at all - cause there was no decison ( and as we know > now- there was TD-decison till the end at all!) > > > > thank you for your gratulation. > We beat Italy in 2006, 2007 and 2008 in the championscup halffinal. > We won the tourney in 2006 > in 2007 and 2008 the AC tooks away the imps and we both lost with 5 imps! > in 2007 all the procedures were correct and the decison of the AC was > close, but the decision was quite ok- it costs as 6 imps... > > in 2008 the most of the procedure was not correct AND the decison > itself was very dubious. > Today the german bridge federation appealed against the technical/formal > circumstances at the EBL-AC > > greetings > Michael > > P.s. > one of our players spoke with EBL-tournament director Latala today > again, and he just says, that he said to us yesterday about the decison > looks like a scandal only for the ears from our team ! > He didnt want to adress that to the AC. So I herebye correct this! From schoderb at msn.com Tue Nov 11 16:38:37 2008 From: schoderb at msn.com (WILLIAM SCHODER) Date: Tue, 11 Nov 2008 10:38:37 -0500 Subject: [blml] Ac in Amasterdam In-Reply-To: References: Message-ID: Answers to Ton using his posting by snipping. Kojak: > "....... Only then the AC started discussing all three cases......" (Ton's > latest quote that I have) seems kind of strange to me. Why were the three > appeals grouped in discussion? It would seem to me that each case would be > decided and completed on its own merits, before going on to the next one. > > ton: > Why?, do you find this in the laws too? Kojak: When a normal procedure is followed by an appeal, a hearing, and a decision all based on the facts and circumstances of that PARTICULAR appeal without any possible shading caused by other appeals most ACs find it proper to do so. It avoids the appearance of 'managing' the totality of results to obtain a joint effect on the game. That is not in the laws, that is just common sense. > ton: > > The first question goes to Kojak, Grattan, Stefan (with a similar strong > opinion): > What do you do if the TD decides for one reason or another not to take a > decision? Don't try with: 'then we search for another TD'. Kojak: To answer that question properly you have to tell me "who" I am. A Tournament Organizer? A Regulating Authority? A TD in charge? The Chairman of the AC? The affected player? Team Captain? >Ton: > Second question (with a more pragmatic approach): > > What makes the TD producing such decision superior to taking the case > itself > ('it' being the AC)? > Kojak: Besides being a violation of the Laws which are supposedly in force, when there is a TD decision ACs can follow a normal procedure which includes having the TD present the Facts WITH THE PLAYERS PRESENT. Both sides have the opportunity to state their positions and reasons for their actions, and the AC then arrives at a decision about the TD ruling. It is not a case of being superior, it is a case of being an accepted procedure which makes it fair for everyone. -- the players, the TD, AND THE AC. If I read the postings correctly one side was not given the opportunity to present their reasoning and explanation for their actions on appeal (which I'm told in other postings really never happened) # 3. It continues to boggle my mind that any official administering a serious high level event would allow this to happen. It opens the door to suspicions, accusations of bias, perceptions of favoritism, etc. I am not saying that these were evident in this case, I'm simply saying that a proper procedure would allay them. And it is particularly important to do so in international competition. Kojak additional: The WBF does not disallow an appeal for a ruling when the "wrong" side calls attention to a hesitation with screens. It says that in the establishment of fact it is heavily weighted that the other side of the screen apparently did not consider the possibility of UI being present and call the Director. Please see the Code of Practice for Appeals Committees for a detailed discussion. And: The only time I have ever gone to an AC without a Director's ruling is when it was impossible to determine the facts. In that case my presentation to the AC was "If the facts are xxxxxxx, then my ruling is xxxxxx. If the facts are YYYYYYY then my ruling is YYYYYYYY." Since application of the proper result depends on vital facts I could not determine I've asked the AC to help me determine what to rule. Kojak > _______________________________________________ > blml mailing list > blml at amsterdamned.org > http://www.amsterdamned.org/mailman/listinfo/blml > From schoderb at msn.com Tue Nov 11 19:27:30 2008 From: schoderb at msn.com (WILLIAM SCHODER) Date: Tue, 11 Nov 2008 13:27:30 -0500 Subject: [blml] amsterdam Message-ID: I'm still confused about the sequence of events. Is it correct that the Director told the Germans that the Italians had asked for a ruling on board 13? That there was no ruling, but the Director went directly to the AC for a decision? That the Germans were never given the opportunity to address the AC about this matter and were told that the Director gave the AC the facts and they ruled as they did without any input from the Germans as to facts or reasons for their actions? Kojak From gro at bridgepro.de Tue Nov 11 20:56:03 2008 From: gro at bridgepro.de (Gro) Date: Tue, 11 Nov 2008 20:56:03 +0100 Subject: [blml] amsterdam In-Reply-To: References: Message-ID: <4919E353.2000302@bridgepro.de> hi, WILLIAM SCHODER schrieb: > I'm still confused about the sequence of events. > > Is it correct that the Director told the Germans that the Italians had > asked for a ruling on board 13? yes > That there was no ruling, but the Director > went directly to the AC for a decision? yes > That the Germans were never given > the opportunity to address the AC about this matter yes > and were told that the > Director gave the AC the facts and they ruled as they did without any input > from the Germans as to facts or reasons for their actions? > no as we got in , the AC said, that there is another 3rd appeal from the italians. Now the facts were cleared in the way, that the italainas were asked, why they want to do this appeal .The italians pointed, that after hesitation a 3NT call is impossible. Mr.Elinescu was asked from the AC, if and why he hesitated. He said, that the hesitation is true. He had to find a good bid with his hand, what was unfortunatly not clear cut. I as a captain asked the italians, what they think what special information was gone to the partner with that hesitation- that he has Spade King?? Then the AC said: ok- now we first have to have a TD -decision before we continue- and they sended us out. the rest you know. and I still wonder, why the italians got the right to point AFTER the segment, that there was a hesitation- cause nobody says a word at the table and no TD was called either. In our first appeal we didnt get the right to use information, cause wrong side called. And NOW nobodys called and its ok !!?? Exactly this fact we would have given the AC after considering all this facts. In the moemnt we were in and in that moment we get informed about this Board and the probable appeal we didnt consider that fact, cause we had no decision from TD before- nothing wirtten before to prepare to the appeal.. Michael > Kojak > > > From jkljkl at gmx.de Tue Nov 11 21:13:40 2008 From: jkljkl at gmx.de (Stefan Filonardi) Date: Tue, 11 Nov 2008 21:13:40 +0100 Subject: [blml] amsterdam In-Reply-To: References: Message-ID: <4919E774.5020901@gmx.de> Hello, WILLIAM SCHODER schrieb: > I'm still confused about the sequence of events. > > Is it correct that the Director told the Germans that the > Italians had asked for a ruling on board 13? That there > was no ruling, but the Director went directly to the AC > for a decision? That the Germans were never given the > opportunity to address the AC about this matter and were > told that the Director gave the AC the facts and they > ruled as they did without any input from the Germans as > to facts or reasons for their actions? My guess from what I read so far. It is hard not to be confused, both sides agree that there was a questioning about one action, the hesitation. So Ton can comfortably say that an hearing took place. While it seems that the german Team (and maybe even a part of the AC) was not aware that, although there was no TD decision to be appealed, the hearing would be treated as concluded while the AC was consulting if they could make a verdict at all about the matter. Ton Kooijman | Michael Grom?ller | That hearing took place |they (The AC) said: and both sides got the |Before we start with an appeal possibility to give their |we need a decision from the TD opinion. |Then we were send out. Maybe Ton could tell us how the german players should have recognized at what stage of the procedure they were when a completely different procedure, compared to a normal appeal, was taking place? And can we have the names of the people sitting in the actual AC? With best regards, stefan From henk at ripe.net Tue Nov 11 22:21:23 2008 From: henk at ripe.net (Henk Uijterwaal) Date: Tue, 11 Nov 2008 22:21:23 +0100 Subject: [blml] Amsterdam (2) Message-ID: <4919F753.9040806@ripe.net> I just read on the site of the IMP magazine that the German Bridge Federation has filed a formal appeal to the EBL about the events in Amsterdam. They basically claim that the AC authorized to make a decision on the matter, ask that the table result is restored and that a new final is played between the Germans and the Dutch. The hand in question is now also available online, see: http://www.imp-bridge.nl/manasite/mas/imp/inhoud/erratum/32654-1164.pdf Henk -- ------------------------------------------------------------------------------ Henk Uijterwaal Email: henk.uijterwaal(at)ripe.net RIPE Network Coordination Centre http://www.amsterdamned.org/~henk P.O.Box 10096 Singel 258 Phone: +31.20.5354414 1001 EB Amsterdam 1016 AB Amsterdam Fax: +31.20.5354445 The Netherlands The Netherlands Mobile: +31.6.55861746 ------------------------------------------------------------------------------ Ceterum censeo Asplain esse delendam (Cato & Henk) From henk at ripe.net Tue Nov 11 22:24:44 2008 From: henk at ripe.net (Henk Uijterwaal) Date: Tue, 11 Nov 2008 22:24:44 +0100 Subject: [blml] Amsterdam (2) In-Reply-To: <4919F753.9040806@ripe.net> References: <4919F753.9040806@ripe.net> Message-ID: <4919F81C.7020806@ripe.net> Henk Uijterwaal wrote: > I just read on the site of the IMP magazine that the German Bridge > Federation has filed a formal appeal to the EBL about the events in > Amsterdam. They basically claim that the AC authorized to make a "authorized" -> "was not authorized" > decision on the matter, ask that the table result is restored and > that a new final is played between the Germans and the Dutch. > > The hand in question is now also available online, see: > > http://www.imp-bridge.nl/manasite/mas/imp/inhoud/erratum/32654-1164.pdf > > Henk -- ------------------------------------------------------------------------------ Henk Uijterwaal Email: henk.uijterwaal(at)ripe.net RIPE Network Coordination Centre http://www.amsterdamned.org/~henk P.O.Box 10096 Singel 258 Phone: +31.20.5354414 1001 EB Amsterdam 1016 AB Amsterdam Fax: +31.20.5354445 The Netherlands The Netherlands Mobile: +31.6.55861746 ------------------------------------------------------------------------------ Ceterum censeo Asplain esse delendam (Cato & Henk) From richard.hills at immi.gov.au Tue Nov 11 22:59:44 2008 From: richard.hills at immi.gov.au (richard.hills at immi.gov.au) Date: Wed, 12 Nov 2008 08:59:44 +1100 Subject: [blml] Mr Chesterton suggested [SEC=UNOFFICIAL] In-Reply-To: <491954D5.3070104@skynet.be> Message-ID: G.K. Chesterton (1874-1936): "The word 'good' has many meanings. For example, if a man were to shoot his mother at a range of five hundred yards, I should call him a good shot, but not necessarily a good man." Sub-postcard size answer from WBF LC minutes, 10th October 2008: ".....compliance with the overriding requirement of the laws always to respond to enquiries under Law 20F with correct explanations of the partnership understandings." Herman De Wael: >Allow me one comment: my question was: >How do _YOU_ _FEEL_ (two separate pieces of emphasis) about a >law that _obliges_ you to give UI. >The WBF minutes say "there is no infraction". _You_ translate >this into "you must" (I don't). Richard Hills: "Overriding requirement" seems to me to be an obvious synonym of "you must". Herman De Wael: >How do you feel about such an obligation? Richard Hills: How do I feel about gravity? "Gravity is not just a good idea, it is the Law". But, if Herman is seeking my personal opinion, he already knows it. When I was a wet-behind-the-ears newbie on blml, one of the first postings I saw was Herman advocating deliberately incorrect explanations of partnership understandings. I ventured a posting of disbelief, only to be advised by other senior blmlers that Herman was Herman. Ergo, when the WBF LC takes submissions for the 2018 Lawbook, one of my proposals will be this revised 2018 Law 20F5: "As soon as a player realizes that her partner's explanation was erroneous or incomplete she must call the Director immediately. The Director applies Law 21B or Law 40B4." Best wishes Richard James Hills Recruitment Section, Level 3 Blue, workstation 15 (first on left) Department of Immigration and Citizenship Telephone: 02 6223 8453 Email: richard.hills at immi.gov.au -------------------------------------------------------------------- Important Notice: If you have received this email by mistake, please advise the sender and delete the message and attachments immediately. This email, including attachments, may contain confidential, sensitive, legally privileged and/or copyright information. Any review, retransmission, dissemination or other use of this information by persons or entities other than the intended recipient is prohibited. DIAC respects your privacy and has obligations under the Privacy Act 1988. The official departmental privacy policy can be viewed on the department's website at www.immi.gov.au. See: http://www.immi.gov.au/functional/privacy.htm --------------------------------------------------------------------- From richard.hills at immi.gov.au Wed Nov 12 04:27:22 2008 From: richard.hills at immi.gov.au (richard.hills at immi.gov.au) Date: Wed, 12 Nov 2008 14:27:22 +1100 Subject: [blml] Amsterdam (2) [SEC=UNOFFICIAL] In-Reply-To: <4919F81C.7020806@ripe.net> Message-ID: Solon (c.630 - c.555 BCE): "Laws are like spider's webs: if some poor weak creature come up against them, it is caught; but a bigger one can break through and get away." Ton Kooijman asked: >The first question goes to Kojak, Grattan, Stefan (with a >similar strong opinion): > >What do you do if the TD decides for one reason or another >not to take a decision? Don't try with: 'then we search for >another TD'. Law 81C, first sentence: "The Director (not the players) has the responsibility for rectifying irregularities and redressing damage." Law 81C7: "The Director's duties and powers normally include also the following: to refer any matter to an appropriate committee." Richard Hills: In my opinion these two clauses of Law 81C do not contradict themselves. The Director has Law 12 adjusted score responsibility to rectify and redress, hence the TD choosing to immediately flick-pass a Law 12 ruling (before ruling herself) to an Appeals Committee would be referring the "matter to an _inappropriate_ committee". But if instead there is a prima facie case of a Law 73B2 "gravest possible offence" then that is a discipline issue, not a score adjustment issue. Since the Director lacks the power to suspend the apparently guilty pair for three years, but the Regulating Authority does have that power, the TD choosing to immediately flick-pass a Law 73B2 ruling (before ruling herself) to the Conduct and Ethics Committee of the Regulating Authority would indeed be referring the "matter to an appropriate committee". Ton Kooijman: >Second question (with a more pragmatic approach): > >What makes the TD producing such decision superior to taking >the case itself ('it' being the AC)? Richard Hills: I pragmatically note that, since this was an international tournament, the WBF Code of Practice was a binding regulation. The Code does not require Appeals Committees to provide guidance to Directors; rather the Code requires Directors to provide guidance to Appeals Committees. WBF Code of Practice, page 6: "The expectation is that each appeal committee will presume initially that the Director's ruling is correct. The ruling is overturned only on the basis of evidence presented. For this reason the Director must inform the committee if a ruling in favour of the non-offending side reflects a margin of doubt that continues to exist after the appropriate consultation procedure." Richard Hills: A blmler (claiming to be an International Director) frivolously asserted that, if he was the Director in question, he would have tossed a coin to determine his ruling, thus giving the Appeals Committee the legal basis to hear an appeal against his ruling. I agree that that International Director blmler is indeed a tosser, since tossing a coin to make a ruling shows a lack of inclination to read the binding regulation which is the WBF Code of Practice. WBF Code of Practice, page 6: "It is the function of the Director to make a ruling in a judgemental matter, having consulted appropriately, that executes most accurately the intention of the laws." What's the problem? Best wishes Richard James Hills Recruitment Section, Level 3 Blue, workstation 15 (first on left) Department of Immigration and Citizenship Telephone: 02 6223 8453 Email: richard.hills at immi.gov.au -------------------------------------------------------------------- Important Notice: If you have received this email by mistake, please advise the sender and delete the message and attachments immediately. This email, including attachments, may contain confidential, sensitive, legally privileged and/or copyright information. Any review, retransmission, dissemination or other use of this information by persons or entities other than the intended recipient is prohibited. DIAC respects your privacy and has obligations under the Privacy Act 1988. The official departmental privacy policy can be viewed on the department's website at www.immi.gov.au. See: http://www.immi.gov.au/functional/privacy.htm --------------------------------------------------------------------- From richard.hills at immi.gov.au Wed Nov 12 08:15:47 2008 From: richard.hills at immi.gov.au (richard.hills at immi.gov.au) Date: Wed, 12 Nov 2008 18:15:47 +1100 Subject: [blml] Amsterdam (2) [SEC=UNOFFICIAL] In-Reply-To: <4919F81C.7020806@ripe.net> Message-ID: Solon (c.630 - c.555 BCE): "Laws are like spider's webs: if some poor weak creature come up against them, it is caught; but a bigger one can break through and get away." Henk Uijterwaal: >I just read on the site of the IMP magazine that the >German Bridge Federation has filed a formal appeal to the >EBL about the events in Amsterdam. They basically claim >that the AC was not authorized to make a decision on the >matter, Richard Hills: Correct. Henk Uijterwaal: >ask that the table result is restored and that a new >final is played between the Germans and the Dutch. Richard Hills: Non sequitur. A Law 16 infraction does not become a legal action simply because a process is flawed. All that the German Bridge Federation is entitled to seek is a re- hearing on the issue of whether or not a German player did infract Law 16. The appropriate body for this re-hearing is presumably the European Standing Appeals Committee, the relevant sub- committee of the European Regulating Authority for the purposes of the second sentence of Law 93C2: "The Regulating Authority has authority to resolve any matter finally." What's the problem? Well, there was and is a problem with the admirable time-saving aim of Ton Kooijman and his colleagues by taking a short cut which avoided the Laws and the Code of Practice. J.R.R. Tolkien: "Short cuts make long delays." Best wishes Richard James Hills Recruitment Section, Level 3 Blue, workstation 15 (first on left) Department of Immigration and Citizenship Telephone: 02 6223 8453 Email: richard.hills at immi.gov.au -------------------------------------------------------------------- Important Notice: If you have received this email by mistake, please advise the sender and delete the message and attachments immediately. This email, including attachments, may contain confidential, sensitive, legally privileged and/or copyright information. Any review, retransmission, dissemination or other use of this information by persons or entities other than the intended recipient is prohibited. DIAC respects your privacy and has obligations under the Privacy Act 1988. The official departmental privacy policy can be viewed on the department's website at www.immi.gov.au. See: http://www.immi.gov.au/functional/privacy.htm --------------------------------------------------------------------- From Hermandw at skynet.be Wed Nov 12 09:32:43 2008 From: Hermandw at skynet.be (Herman De Wael) Date: Wed, 12 Nov 2008 09:32:43 +0100 Subject: [blml] Mr Chesterton suggested [SEC=UNOFFICIAL] In-Reply-To: References: Message-ID: <491A94AB.2030805@skynet.be> Will you all please note that I asked three questions and that Richard only answered two? But then this pot is as black as any kettle. Herman. richard.hills at immi.gov.au wrote: > G.K. Chesterton (1874-1936): > > "The word 'good' has many meanings. For example, if a man were > to shoot his mother at a range of five hundred yards, I should > call him a good shot, but not necessarily a good man." > > Sub-postcard size answer from WBF LC minutes, 10th October 2008: > > ".....compliance with the overriding requirement of the laws > always to respond to enquiries under Law 20F with correct > explanations of the partnership understandings." > > Herman De Wael: > >> Allow me one comment: my question was: >> How do _YOU_ _FEEL_ (two separate pieces of emphasis) about a >> law that _obliges_ you to give UI. >> The WBF minutes say "there is no infraction". _You_ translate >> this into "you must" (I don't). > > Richard Hills: > > "Overriding requirement" seems to me to be an obvious synonym of > "you must". > > Herman De Wael: > >> How do you feel about such an obligation? > > Richard Hills: > > How do I feel about gravity? "Gravity is not just a good idea, > it is the Law". > > But, if Herman is seeking my personal opinion, he already knows > it. When I was a wet-behind-the-ears newbie on blml, one of the > first postings I saw was Herman advocating deliberately incorrect > explanations of partnership understandings. I ventured a posting > of disbelief, only to be advised by other senior blmlers that > Herman was Herman. > > Ergo, when the WBF LC takes submissions for the 2018 Lawbook, one > of my proposals will be this revised 2018 Law 20F5: > > "As soon as a player realizes that her partner's explanation was > erroneous or incomplete she must call the Director immediately. > The Director applies Law 21B or Law 40B4." > > > Best wishes > > Richard James Hills > Recruitment Section, Level 3 Blue, workstation 15 (first on left) > Department of Immigration and Citizenship > Telephone: 02 6223 8453 > Email: richard.hills at immi.gov.au > > > > -------------------------------------------------------------------- > Important Notice: If you have received this email by mistake, please advise the sender and delete the message and attachments immediately. This email, including attachments, may contain confidential, sensitive, legally privileged and/or copyright information. Any review, retransmission, dissemination or other use of this information by persons or entities other than the intended recipient is prohibited. DIAC respects your privacy and has obligations under the Privacy Act 1988. The official departmental privacy policy can be viewed on the department's website at www.immi.gov.au. See: http://www.immi.gov.au/functional/privacy.htm > > --------------------------------------------------------------------- > > > _______________________________________________ > blml mailing list > blml at amsterdamned.org > http://www.amsterdamned.org/mailman/listinfo/blml > From Hermandw at skynet.be Wed Nov 12 09:52:39 2008 From: Hermandw at skynet.be (Herman De Wael) Date: Wed, 12 Nov 2008 09:52:39 +0100 Subject: [blml] Ac in Amasterdam In-Reply-To: <49197BB8.1030507@bridgepro.de> References: 4918F39D.8070708@bridgepro.de <49197BB8.1030507@bridgepro.de> Message-ID: <491A9957.4060905@skynet.be> Hello Michael, Gro wrote: > Hi Herman, > > Herman wrote: > "What you are - possibly correctly - angry about is the fact that you may > have been unaware that you would only be heard once. If the AC did not > make this clear to you, you might have reason to complain. But if the > hearing was conducted by the AC Chairman, and not by the TD, you could > have known that this was a once-off. > Also, I am quite certain the AC asked you before leaving "have you > anything to add?" and allowed you to speak. To complain that you then > did not have another chance to speak 15 minutes later is not OK." > > Michael: > They did not make that clear, cause they said: > Before we start with an appeal we need a decision from the TD. . > Then we were send out. OK, that seems to be an error on their part. > Nobody say before we got out: "anyone wants to say something, last chance!?" > Cause in THAT moment there was NO TD-Decison at all, to which we could > add something. But what could you have added after a TD ruling that you could not have said before it? > So it was impossible for us to think we will NOT get heared again AFTER > the TD-decison, cause > the "normal procedure" is to get heared AFTER the TD-decison...or? Indeed, but what else were you going to say? Surely they discussed the case at length? > And you really think that our complaint 15 minutes later, without having > a TD-decison before is not ok?? > I cant imagine that. > Well, as I said, I cannot imagine there is anything to be said 15 minutes later that you could not have said to the same committee 15 minutes earlier. > Herman: > " All in all, there have been some procedural errors, but I don't think > the Germans (or Italians) were damaged as a result." > > Michael: > we were not damaged? Not by the procedural errors, no, not in my opinion. Of course you were damaged in not being able to play in the final, but that is the consequence of the decision, not of the procedural errors. > I told in my statement, that about in this 3rd appeal there was no call > at the table and no TD was there, taking the facts from that hesitation. > The TD was informed, after that segment, without beeing called at the table. > There are of course possibilties, where you can call a TD after the > session behind screeens- > as an example that they were differnet explanations behind the screens. > > BUT to inform TD for a probable hesitation AFTER the segement- looks > impossible for us! And to me, you're right. But it only looks impossible. I can imagine a lot of circumstances in which it would be possible. If the pause was 5 minutes, for example, in which case sensible players would not feel the need to call the director at that time, realizing that their opponents will not question the pause at any later moment. I cannot imagine that your team is questioning that there has been a pause. Because I cannot imagine an AC ruling on this without absolute certainty that there had been a pause. And you would be screaming blue murder if they had - and you'd be right. > Especially when in our first appeal the ruling from the same TD was: you > are NOT allowed to use the hesitation, cause you called > from the wrong side -- and now it doesnt matter at all ??? thats very > unlocigal and ununderstandable for us..or I am so wrong? > If the hesitation was, say, 25 seconds, then it is imperative that the TD establishes that the pause was noticed at the other side. In that case, if it is the wrong side who calls, the TD cannot establish this. If, OTOH, the hesitation was 5 minutes, and all four players agree to this, then the TD has established that the pause was noticed on the other side. Those two things are not the same and it is therefore possible that two quite distinct rulings are given in what you think are similar cases. But I should not be assuming things I have no direct knowledge about. Please do not interpret this as saying that you have thought for 5 minutes - that was just an example of how this case could be understandable. There might be other reasons why the AC ruled as they did, equally correct. > And..after the TD was informed about this hesitation -AND if this is > allowed at all ( what we dont believe)..why there was no decision on > that from the TD ? > Our players were not informed about the call form the italians > afterwards. And of course also not heared by the TD if the facts are > correct. I interpret the hearing as both a hearing by the TD and by the AC. I think your players were heard. > Me as a captain was told form the TD: Italians complained about a board > on that session about hesitaion. > I was not told which board- I did not get a written TD-decison - I did > not get a decison at all - cause there was no decison ( and as we know > now- there was TD-decison till the end at all!) > > > > thank you for your gratulation. > We beat Italy in 2006, 2007 and 2008 in the championscup halffinal. > We won the tourney in 2006 > in 2007 and 2008 the AC tooks away the imps and we both lost with 5 imps! > in 2007 all the procedures were correct and the decison of the AC was > close, but the decision was quite ok- it costs as 6 imps... > > in 2008 the most of the procedure was not correct AND the decison > itself was very dubious. > Today the german bridge federation appealed against the technical/formal > circumstances at the EBL-AC > I should stop commenting then. > greetings > Michael > > P.s. > one of our players spoke with EBL-tournament director Latala today > again, and he just says, that he said to us yesterday about the decison > looks like a scandal only for the ears from our team ! > He didnt want to adress that to the AC. So I herebye correct this! > > It is very wrong to comment on cases one hears from from only one side. Or to believe those comments as fully formed opinions. I am not commenting on the case, since I have no full knowledge about it. I am merely trying to allow you to see there is more than one facet to any story, and your view is obviously tainted by the sad result. > > Herman. From Hermandw at skynet.be Wed Nov 12 09:56:06 2008 From: Hermandw at skynet.be (Herman De Wael) Date: Wed, 12 Nov 2008 09:56:06 +0100 Subject: [blml] amsterdam In-Reply-To: References: Message-ID: <491A9A26.5010206@skynet.be> WILLIAM SCHODER wrote: > I'm still confused about the sequence of events. > Aren't we all? > Is it correct that the Director told the Germans that the Italians had > asked for a ruling on board 13? That there was no ruling, but the Director > went directly to the AC for a decision? That is what I gathered too. > That the Germans were never given > the opportunity to address the AC about this matter and were told that the > Director gave the AC the facts and they ruled as they did without any input > from the Germans as to facts or reasons for their actions? > I very much doubt if that is true. If it is, lock away the whole AC, and the whole of the Netherlands with it, and throw away the key. I think I gathered from Ton's account that the Germans were present at a hearing in which the facts were established. I have heard nothing in Michael's statement to suggest that they weren't heard, only that they did not realize they would not get a second chance to speak. > Kojak > Herman. From grandaeval at tiscali.co.uk Wed Nov 12 11:06:43 2008 From: grandaeval at tiscali.co.uk (Grattan) Date: Wed, 12 Nov 2008 10:06:43 -0000 Subject: [blml] amsterdam References: Message-ID: <000d01c944ae$5f2b20b0$0302a8c0@Mildred> Grattan Endicott To: "blml" Sent: Tuesday, November 11, 2008 6:27 PM Subject: [blml] amsterdam > I'm still confused about the sequence of events. > > Is it correct that the Director told the Germans that the Italians had > asked for a ruling on board 13? That there was no ruling, but the Director > went directly to the AC for a decision? That the Germans were never given > the opportunity to address the AC about this matter and were told that the > Director gave the AC the facts and they ruled as they did without any > input from the Germans as to facts or reasons for their actions? > > Kojak +=+ Having trapped myself into comment on some half facts originally circulating here I, too, am trying to make sense of all that I am reading. The Netherlands people are so precise in the way they normally do things that I can hardly believe what I am reading, and I feel sure I must be misunderstanding what is being said. This has now achieved 'cause celebre' status and it will be very difficult to unwind. ~ Grattan ~ +=+ From grandaeval at tiscali.co.uk Wed Nov 12 11:16:03 2008 From: grandaeval at tiscali.co.uk (Grattan) Date: Wed, 12 Nov 2008 10:16:03 -0000 Subject: [blml] Ac in Amasterdam References: 4918F39D.8070708@bridgepro.de <49197BB8.1030507@bridgepro.de> <491A9957.4060905@skynet.be> Message-ID: <001c01c944af$cfe114d0$0302a8c0@Mildred> Grattan Endicott To: "Bridge Laws Mailing List" Sent: Wednesday, November 12, 2008 8:52 AM Subject: Re: [blml] Ac in Amasterdam > >> Today the german bridge federation appealed against the technical/formal circumstances at the EBL-AC >> +=+ What was the status of the tournament? Would the EBL Standing AC have jurisdiction? ~ Grattan ~ +=+ From gro at bridgepro.de Wed Nov 12 12:22:30 2008 From: gro at bridgepro.de (Gro) Date: Wed, 12 Nov 2008 12:22:30 +0100 Subject: [blml] Ac in Amasterdam In-Reply-To: <491A9957.4060905@skynet.be> References: 4918F39D.8070708@bridgepro.de <49197BB8.1030507@bridgepro.de> <491A9957.4060905@skynet.be> Message-ID: <491ABC76.3000307@bridgepro.de> Hi again, Herman De Wael schrieb: > OK, that seems to be an error on their part. >> Nobody say before we got out: "anyone wants to say something, last chance!?" >> Cause in THAT moment there was NO TD-Decison at all, to which we could >> add something. >> > > But what could you have added after a TD ruling that you could not have > said before it? > Michael: Our players were informed about this appeal in the moment they were going in. There the facts were discussed, why our player hesitated. Assume there Would have been a ruling from TD, lets say: 5 Diamonds minus 1 . what is the normal procedure now? then we would have heared, if we could say something, why in our opinion this ruling is wrong!.. then we could have suggested, that 3NT is possible, cause....pass is possible, cause.....5D is impossible, cuase...and so on! and we would have also suggested, that no call was made and no TD was asking our players about if this is right at all. Also we were not informed before, that italy makes an appeal- we were informed 3-5 minutes before going in- so...not a longh time to think about all the facts, which we could say. normally you have some time to prepare yourself for an appeal- cuase you get a written formular form the TD- all this did not happen! unfortunatly there was no decisoin, to which we could say anything!!! We just could agree to he facts and not to any decisions, cuase there was none! thats my answer to your question, what we would have added, what we partly couldnt know during the appeal! > >> So it was impossible for us to think we will NOT get heared again AFTER >> the TD-decison, cause >> the "normal procedure" is to get heared AFTER the TD-decison...or? >> > > Indeed, but what else were you going to say? Surely they discussed the > case at length? > > >> And you really think that our complaint 15 minutes later, without having >> a TD-decison before is not ok?? >> I cant imagine that. >> >> > > Well, as I said, I cannot imagine there is anything to be said 15 > minutes later that you could not have said to the same committee 15 > minutes earlier. > > >> Herman: >> " All in all, there have been some procedural errors, but I don't think >> the Germans (or Italians) were damaged as a result." >> >> Michael: >> we were not damaged? >> > > Not by the procedural errors, no, not in my opinion. > Of course you were damaged in not being able to play in the final, but > that is the consequence of the decision, not of the procedural errors. > Michael: not? the italians complained after the segment, that there was a hesitaion somewhere. I got informed by the TD as a captain. ( just that italy complains- no board number!) Our players were not informed at all! what needs a TD normally to do, when a pair comes and says, that there was something wrong? The TD has to ask the other side, if they agree..or? nobody was asked, if they agree - first time our players heared that was at the AC. There was no decision and no appeal for that. But I will stop now, cuase this are facts, we will give to the standing AC of EBL. Michael From grandaeval at tiscali.co.uk Wed Nov 12 12:52:30 2008 From: grandaeval at tiscali.co.uk (Grattan) Date: Wed, 12 Nov 2008 11:52:30 -0000 Subject: [blml] Amsterdam (2) [SEC=UNOFFICIAL] References: Message-ID: <004301c944bd$28ccade0$0302a8c0@Mildred> Grattan Endicott To: "Bridge Laws Mailing List" Sent: Wednesday, November 12, 2008 3:27 AM Subject: Re: [blml] Amsterdam (2) [SEC=UNOFFICIAL] " Ton Kooijman: Second question (with a more pragmatic approach): What makes the TD producing such decision superior to taking the case itself ('it' being the AC)?" < +=+ Richard quotes a remark allegedly made by Ton. I just do not believe he can possibly have asked this question. The chairman of the WBF Laws Committee does not go around suggesting that it is OK to disregard the laws of the game. The answer to the question, whoever did ask it, is simply that this is the law. The authors of the laws had reason to believe that it was desirable for the Director to establish the facts and to make the ruling - this being, for one thing, what he is appointed to do. Now, what I have not heard is who the Director in charge was. I do not recall seeing any statement by him as to when the appeal was made, whether it was in time and properly made to the Director, and if so how it was dealt with. Is the said Director in hiding or something? Why are we not hearing from him as a key person? ~ Grattan ~ +=+ From Hermandw at skynet.be Wed Nov 12 13:35:22 2008 From: Hermandw at skynet.be (Herman De Wael) Date: Wed, 12 Nov 2008 13:35:22 +0100 Subject: [blml] Amsterdam (2) [SEC=UNOFFICIAL] In-Reply-To: <004301c944bd$28ccade0$0302a8c0@Mildred> References: <004301c944bd$28ccade0$0302a8c0@Mildred> Message-ID: <491ACD8A.8080000@skynet.be> Grattan wrote: > Now, what I have not heard is who the Director in > charge was. I do not recall seeing any statement by him > as to when the appeal was made, whether it was in time > and properly made to the Director, and if so how it was > dealt with. Is the said Director in hiding or something? > Why are we not hearing from him as a key person? Maybe because this is not an official inquiry? As to the ruling being made in time, I am certain both the TD and the AC considered this, and must have reached the conclusion that it was in time. Nothing in the German's account suggests they doubt that it was in time, either. > ~ Grattan ~ +=+ > Herman. From gro at bridgepro.de Wed Nov 12 13:41:33 2008 From: gro at bridgepro.de (Gro) Date: Wed, 12 Nov 2008 13:41:33 +0100 Subject: [blml] Amsterdam (2) [SEC=UNOFFICIAL] In-Reply-To: <491ACD8A.8080000@skynet.be> References: <004301c944bd$28ccade0$0302a8c0@Mildred> <491ACD8A.8080000@skynet.be> Message-ID: <491ACEFD.3090004@bridgepro.de> hi, right, no suggestion from our side till now... but the suggestion will follow - but not in public :) if we would not have good suggestions about in our opinion _a lot of incorrect _happenings, our appeal at the EBL would be ridiculous. Michael Herman De Wael schrieb: > Grattan wrote: > > >> Now, what I have not heard is who the Director in >> charge was. I do not recall seeing any statement by him >> as to when the appeal was made, whether it was in time >> and properly made to the Director, and if so how it was >> dealt with. Is the said Director in hiding or something? >> Why are we not hearing from him as a key person? >> > > Maybe because this is not an official inquiry? > As to the ruling being made in time, I am certain both the TD and the AC > considered this, and must have reached the conclusion that it was in time. > Nothing in the German's account suggests they doubt that it was in time, > either. > > >> ~ Grattan ~ +=+ >> >> > > Herman. > > _______________________________________________ > blml mailing list > blml at amsterdamned.org > http://www.amsterdamned.org/mailman/listinfo/blml > > > From grandaeval at tiscali.co.uk Wed Nov 12 15:56:06 2008 From: grandaeval at tiscali.co.uk (Grattan) Date: Wed, 12 Nov 2008 14:56:06 -0000 Subject: [blml] Amsterdam (2) [SEC=UNOFFICIAL] References: <004301c944bd$28ccade0$0302a8c0@Mildred> <491ACD8A.8080000@skynet.be> Message-ID: <000a01c944d6$dd3a10b0$0302a8c0@Mildred> Grattan Endicott To: "Bridge Laws Mailing List" Sent: Wednesday, November 12, 2008 12:35 PM Subject: Re: [blml] Amsterdam (2) [SEC=UNOFFICIAL] > Grattan wrote: > >> Now, what I have not heard is who the Director in >> charge was. I do not recall seeing any statement by him >> as to when the appeal was made, whether it was in time >> and properly made to the Director, and if so how it was >> dealt with. Is the said Director in hiding or something? >> Why are we not hearing from him as a key person? > > Maybe because this is not an official inquiry? > As to the ruling being made in time, I am certain both the TD and the AC > considered this, and must have reached the conclusion that it was in time. > Nothing in the German's account suggests they doubt that it was in time, > either. > +=+ Indeed, and this underlines my view that all our waffle here is valueless without the input of the most important authority, the CTD. My question about the timing stems from my understanding that the Director was not summoned at the time as Law 16B3 urges and the appeal has all the marks of an afterthought. ~ Grattan ~ +=+ From t.kooyman at worldonline.nl Wed Nov 12 15:57:20 2008 From: t.kooyman at worldonline.nl (ton) Date: Wed, 12 Nov 2008 15:57:20 +0100 Subject: [blml] Amsterdam (2) [SEC=UNOFFICIAL] In-Reply-To: <004301c944bd$28ccade0$0302a8c0@Mildred> Message-ID: Now, what I have not heard is who the Director in charge was. I do not recall seeing any statement by him as to when the appeal was made, whether it was in time and properly made to the Director, and if so how it was dealt with. Is the said Director in hiding or something? Why are we not hearing from him as a key person? ~ Grattan ~ +=+ ton: What incredible statement this is!!! I will not adress you in any of your official functions, just as mister Grattan Endicott. Has this forum now become the tribunal where people are called to justify themselves? You must have become .....whatever. ton From Hermandw at skynet.be Wed Nov 12 17:03:34 2008 From: Hermandw at skynet.be (Herman De Wael) Date: Wed, 12 Nov 2008 17:03:34 +0100 Subject: [blml] Amsterdam (2) [SEC=UNOFFICIAL] In-Reply-To: <000a01c944d6$dd3a10b0$0302a8c0@Mildred> References: <004301c944bd$28ccade0$0302a8c0@Mildred> <491ACD8A.8080000@skynet.be> <000a01c944d6$dd3a10b0$0302a8c0@Mildred> Message-ID: <491AFE56.3050907@skynet.be> Grattan wrote: > >> > +=+ Indeed, and this underlines my view that all our waffle here > is valueless without the input of the most important authority, the > CTD. My question about the timing stems from my understanding > that the Director was not summoned at the time as Law 16B3 > urges and the appeal has all the marks of an afterthought. > ~ Grattan ~ +=+ > And you somehow assume that neither the Director nor the AC took those things into account? Just because some things are not being said does not mean they were disregarded. Rather, since they are not said, no party is claiming they were disregarded, and we should assume they were taken into consideration. From grandaeval at tiscali.co.uk Wed Nov 12 17:41:28 2008 From: grandaeval at tiscali.co.uk (Grattan) Date: Wed, 12 Nov 2008 16:41:28 -0000 Subject: [blml] Amsterdam (2) [SEC=UNOFFICIAL] References: <48DA265735941B28@mail-7-uk.mail.tiscali.sys> (added by postmaster@mail-7.uk.tiscali.com) Message-ID: <001101c944e5$84fea410$0302a8c0@Mildred> Grattan Endicott To: "'Bridge Laws Mailing List'" Sent: Wednesday, November 12, 2008 2:57 PM Subject: Re: [blml] Amsterdam (2) [SEC=UNOFFICIAL] > Now, what I have not heard is who the Director in > charge was. I do not recall seeing any statement by him > as to when the appeal was made, whether it was in time > and properly made to the Director, and if so how it was > dealt with. Is the said Director in hiding or something? > Why are we not hearing from him as a key person? > ~ Grattan ~ +=+ > > ton: What incredible statement this is!!! I will not adress you in any of your official functions, just as mister Grattan Endicott. Has this forum now become the tribunal where people are called to justify themselves? You must have become .....whatever. > ton < +=+ Ah, my dear Ton, I am sorry if I have ruffled your feathers a little. I am simply surveying a series of ill-coordinated contributions which have no official status and pointing to the absence of anything from the official whose remarks would have crucial importance. Not that I think there is any right to expect that we would hear from him here, but more to emphasize the utter futility of all our chatter in this forum without it. I am surprised beyond belief that a Netherlands based event should produce such a shambles since it is wholly contrary to past experience of the careful adhesion of Netherlands folk to principle and procedure. ~ Grattan ~ +=+ From grandaeval at tiscali.co.uk Wed Nov 12 18:05:37 2008 From: grandaeval at tiscali.co.uk (Grattan) Date: Wed, 12 Nov 2008 17:05:37 -0000 Subject: [blml] Amsterdam (2) [SEC=UNOFFICIAL] References: <004301c944bd$28ccade0$0302a8c0@Mildred> <491ACD8A.8080000@skynet.be><000a01c944d6$dd3a10b0$0302a8c0@Mildred> <491AFE56.3050907@skynet.be> Message-ID: <001a01c944e9$e21b52c0$0302a8c0@Mildred> Grattan Endicott To: "Bridge Laws Mailing List" Sent: Wednesday, November 12, 2008 4:03 PM Subject: Re: [blml] Amsterdam (2) [SEC=UNOFFICIAL] > Grattan wrote: >> >>> >> +=+ Indeed, and this underlines my view that all our waffle here >> is valueless without the input of the most important authority, the >> CTD. My question about the timing stems from my understanding >> that the Director was not summoned at the time as Law 16B3 >> urges and the appeal has all the marks of an afterthought. >> ~ Grattan ~ +=+ >> > > And you somehow assume that neither the Director nor the AC > took those things into account? > > Just because some things are not being said does not mean they > were disregarded. Rather, since they are not said, no party is > claiming they were disregarded, and we should assume they were > taken into consideration. > +=+ I assume nothing. The crucial evidence is absent. As for 'no party is claiming they were disregarded' I am unsure after what I have read whether all parties were in a position to know. ~ G ~ +=+ From Harsanyi at t-online.de Wed Nov 12 18:38:12 2008 From: Harsanyi at t-online.de (Harsanyi at t-online.de) Date: Wed, 12 Nov 2008 18:38:12 +0100 Subject: [blml] =?iso-8859-15?q?Amsterdam_=282=29_=5BSEC=3DUNOFFICIAL=5D?= In-Reply-To: References: Message-ID: <1L0JfQ-0IQYD20@fwd03.aul.t-online.de> An HTML attachment was scrubbed... URL: http://www.amsterdamned.org/pipermail/blml/attachments/20081112/de50c9e6/attachment.htm From john at asimere.com Wed Nov 12 16:31:38 2008 From: john at asimere.com (John (MadDog) Probst) Date: Wed, 12 Nov 2008 15:31:38 -0000 Subject: [blml] Ac in Amasterdam References: Message-ID: <0FC109DDB4594FE2B5A8945C60F24A8C@JOHN> ----- Original Message ----- From: "WILLIAM SCHODER" To: "Bridge Laws Mailing List" Sent: Tuesday, November 11, 2008 3:38 PM Subject: Re: [blml] Ac in Amasterdam > Answers to Ton using his posting by snipping. > > Kojak: > > "....... Only then the AC started discussing all three cases......" > (Ton's >> latest quote that I have) seems kind of strange to me. Why were the three >> appeals grouped in discussion? It would seem to me that each case would >> be >> decided and completed on its own merits, before going on to the next one. >> >> ton: Ir seems fundamental to me that natural justice is not obtained unless both parties have been given an opportunity to present their case. If the rules of a sport do not conform to natural justice then the sport is so seriously flawed it should not survive. A TD ruling followed by appeal to an AC allows for both the Napoleonc code of examining magistrate and the English adversarial testing of the other side's assertions before a judgement is given. This is enough to satisfy both the Frogs and the Rosbifs; and if you can do THAT you're probably doing it ok. To leave out some steps in this process engenders some doubt in the process and is a step down the road to the Dodgsonian Queen of Hearts and her incessant "Off with his head". This is a road I don't want to travel. I don't need to look in the Lawbook at all if I'm told that a decision has been made without both parties having been able to present a case. I've no doubt the Sports Court in Geneva won't need to either. I'm appalled. Every now and then sports decisions are made often with good intention and natural justice falls victim. These decsions have to be annulled and proper judgements made to over-ride them. That the proper judgement may not relieve the damge (failing to progress in a tournament for example) is not relevant. The decision itself HAS to be SEEN to be lawful. John >> Why?, do you find this in the laws too? > > Kojak: > When a normal procedure is followed by an appeal, a hearing, and a > decision all based on the facts and circumstances of that PARTICULAR > appeal without any possible shading caused by other appeals most ACs find > it proper to do so. It avoids the appearance of 'managing' the totality of > results to obtain a joint effect on the game. That is not in the laws, > that is just common sense. > > >> ton: >> > The first question goes to Kojak, Grattan, Stefan (with a similar >> > strong >> opinion): > >> What do you do if the TD decides for one reason or another not to take a >> decision? Don't try with: 'then we search for another TD'. > > Kojak: > > To answer that question properly you have to tell me "who" I am. A > Tournament Organizer? A Regulating Authority? A TD in charge? The Chairman > of the AC? The affected player? Team Captain? > >>Ton: >> Second question (with a more pragmatic approach): >> >> What makes the TD producing such decision superior to taking the case >> itself >> ('it' being the AC)? >> > > Kojak: Besides being a violation of the Laws which are supposedly in > force, when there is a TD decision ACs can follow a normal procedure which > includes having the TD present the Facts WITH THE PLAYERS PRESENT. Both > sides have the opportunity to state their positions and reasons for their > actions, and the AC then arrives at a decision about the TD ruling. It is > not a case of being superior, it is a case of being an accepted procedure > which makes it fair for everyone. -- the players, the TD, AND THE AC. > > If I read the postings correctly one side was not given the opportunity to > present their reasoning and explanation for their actions on appeal (which > I'm told in other postings really never happened) # 3. It continues to > boggle my mind that any official administering a serious high level event > would allow this to happen. It opens the door to suspicions, accusations > of bias, perceptions of favoritism, etc. I am not saying that these were > evident in this case, I'm simply saying that a proper procedure would > allay them. And it is particularly important to do so in international > competition. > > Kojak additional: The WBF does not disallow an appeal for a ruling when > the "wrong" side calls attention to a hesitation with screens. It says > that in the establishment of fact it is heavily weighted that the other > side of the screen apparently did not consider the possibility of UI being > present and call the Director. Please see the Code of Practice for Appeals > Committees for a detailed discussion. > > And: > > The only time I have ever gone to an AC without a Director's ruling is > when it was impossible to determine the facts. In that case my > presentation to the AC was "If the facts are xxxxxxx, then my ruling is > xxxxxx. If the facts are YYYYYYY then my ruling is YYYYYYYY." Since > application of the proper result depends on vital facts I could not > determine I've asked the AC to help me determine what to rule. > > Kojak > >> _______________________________________________ >> blml mailing list >> blml at amsterdamned.org >> http://www.amsterdamned.org/mailman/listinfo/blml >> > > _______________________________________________ > blml mailing list > blml at amsterdamned.org > http://www.amsterdamned.org/mailman/listinfo/blml From john at asimere.com Wed Nov 12 16:34:15 2008 From: john at asimere.com (John (MadDog) Probst) Date: Wed, 12 Nov 2008 15:34:15 -0000 Subject: [blml] Mr Chesterton suggested [SEC=UNOFFICIAL] References: Message-ID: <2CF77EB18250470CB513261A89089258@JOHN> ----- Original Message ----- From: To: "Bridge Laws Mailing List" Sent: Tuesday, November 11, 2008 9:59 PM Subject: Re: [blml] Mr Chesterton suggested [SEC=UNOFFICIAL] > G.K. Chesterton (1874-1936): > > "The word 'good' has many meanings. For example, if a man were > to shoot his mother at a range of five hundred yards, I should > call him a good shot, but not necessarily a good man." > > Sub-postcard size answer from WBF LC minutes, 10th October 2008: > > ".....compliance with the overriding requirement of the laws > always to respond to enquiries under Law 20F with correct > explanations of the partnership understandings." > > Herman De Wael: > >>Allow me one comment: my question was: >>How do _YOU_ _FEEL_ (two separate pieces of emphasis) about a >>law that _obliges_ you to give UI. Perfectly comfortable. The giving of UI is a necessary evil. John >>The WBF minutes say "there is no infraction". _You_ translate >>this into "you must" (I don't). > > Richard Hills: > > "Overriding requirement" seems to me to be an obvious synonym of > "you must". > > Herman De Wael: > >>How do you feel about such an obligation? > > Richard Hills: > > How do I feel about gravity? "Gravity is not just a good idea, > it is the Law". > > But, if Herman is seeking my personal opinion, he already knows > it. When I was a wet-behind-the-ears newbie on blml, one of the > first postings I saw was Herman advocating deliberately incorrect > explanations of partnership understandings. I ventured a posting > of disbelief, only to be advised by other senior blmlers that > Herman was Herman. > > Ergo, when the WBF LC takes submissions for the 2018 Lawbook, one > of my proposals will be this revised 2018 Law 20F5: > > "As soon as a player realizes that her partner's explanation was > erroneous or incomplete she must call the Director immediately. > The Director applies Law 21B or Law 40B4." > > > Best wishes > > Richard James Hills > Recruitment Section, Level 3 Blue, workstation 15 (first on left) > Department of Immigration and Citizenship > Telephone: 02 6223 8453 > Email: richard.hills at immi.gov.au > > > > -------------------------------------------------------------------- > Important Notice: If you have received this email by mistake, please > advise the sender and delete the message and attachments immediately. > This email, including attachments, may contain confidential, sensitive, > legally privileged and/or copyright information. Any review, > retransmission, dissemination or other use of this information by persons > or entities other than the intended recipient is prohibited. DIAC > respects your privacy and has obligations under the Privacy Act 1988. The > official departmental privacy policy can be viewed on the department's > website at www.immi.gov.au. See: > http://www.immi.gov.au/functional/privacy.htm > > --------------------------------------------------------------------- > > > _______________________________________________ > blml mailing list > blml at amsterdamned.org > http://www.amsterdamned.org/mailman/listinfo/blml From t.kooyman at worldonline.nl Wed Nov 12 19:40:05 2008 From: t.kooyman at worldonline.nl (ton) Date: Wed, 12 Nov 2008 19:40:05 +0100 Subject: [blml] Amsterdam (2) [SEC=UNOFFICIAL] In-Reply-To: <001101c944e5$84fea410$0302a8c0@Mildred> Message-ID: > Now, what I have not heard is who the Director in > charge was. I do not recall seeing any statement by him > as to when the appeal was made, whether it was in time > and properly made to the Director, and if so how it was > dealt with. Is the said Director in hiding or something? > Why are we not hearing from him as a key person? > ~ Grattan ~ +=+ > > ton: What incredible statement this is!!! I will not adress you in any of your official functions, just as mister Grattan Endicott. Has this forum now become the tribunal where people are called to justify themselves? You must have become .....whatever. > ton < +=+ Ah, my dear Ton, I am sorry if I have ruffled your feathers a little. I am simply surveying a series of ill-coordinated contributions which have no official status and pointing to the absence of anything from the official whose remarks would have crucial importance. Not that I think there is any right to expect that we would hear from him here, but more to emphasize the utter futility of all our chatter in this forum without it. I am surprised beyond belief that a Netherlands based event should produce such a shambles since it is wholly contrary to past experience of the careful adhesion of Netherlands folk to principle and procedure. ~ Grattan ~ +=+ ton: Imo you are contributing to this ill-coordination and futile chatter. Somewhat below the level I expected from you. Combining the sentences: 'Is the said Director in hiding or something? Why are we not hearing from him as a key person?' with: 'Not that I think there is any right to expect that we would hear from him here' sounds like the bla bla of a drunken sailsman. Well, enjoy yourself ton From Hermandw at skynet.be Wed Nov 12 23:38:53 2008 From: Hermandw at skynet.be (Herman De Wael) Date: Wed, 12 Nov 2008 23:38:53 +0100 Subject: [blml] Mr Chesterton suggested [SEC=UNOFFICIAL] In-Reply-To: <2CF77EB18250470CB513261A89089258@JOHN> References: <2CF77EB18250470CB513261A89089258@JOHN> Message-ID: <491B5AFD.8000604@skynet.be> John, you too, responded to only one question, not all three. John (MadDog) Probst wrote: >> >> Herman De Wael: >> >>> Allow me one comment: my question was: >>> How do _YOU_ _FEEL_ (two separate pieces of emphasis) about a >>> law that _obliges_ you to give UI. > > Perfectly comfortable. The giving of UI is a necessary evil. John > I'm not talking about giving UI, I'm talking about being _obliged_ to give UI. Everything I do at the bridge table is an attempt to keep from giving UI to my partner, from my poker face to my somewhat hasty bids. And here is a law that would tell me that I am forced to give UI. It goes against my nature. So I don't do it. Which leads me to my second question: You're the TD, and I have "broken" this law telling me to give UI. I have not given UI. What adjustment do you propose? And there was still my third "quiz" question. How many laws in the lawbook explicitely forbid a player to give a piece of UI to his partner? And how many oblige him to do so? Herman. (pot or kettle, I don't care: you're all black) From nigelguthrie at talktalk.net Thu Nov 13 00:22:29 2008 From: nigelguthrie at talktalk.net (Nigel Guthrie) Date: Wed, 12 Nov 2008 23:22:29 +0000 Subject: [blml] Amsterdam (2) [SEC=UNOFFICIAL] Message-ID: <491B6535.5020201@talktalk.net> [ton] Imo you are contributing to this ill-coordination and futile chatter. Somewhat below the level I expected from you. Combining the sentences: 'Is the said Director in hiding or something? Why are we not hearing from him as a key person?' with: 'Not that I think there is any right to expect that we would hear from him here' sounds like the bla bla of a drunken sailsman. Well, enjoy yourself [nige1] Grattan doesn't seem to be criticising, he appears to be expressing our curiosity about the official version of events. For example, it's hard to understand why the name of the CTD is cloaked in secrecy :) From grandaeval at tiscali.co.uk Thu Nov 13 02:15:08 2008 From: grandaeval at tiscali.co.uk (Grattan) Date: Thu, 13 Nov 2008 01:15:08 -0000 Subject: [blml] Amsterdam (2) [SEC=UNOFFICIAL] References: <491B6535.5020201@talktalk.net> Message-ID: <001a01c9452d$46ddddc0$0302a8c0@Mildred> Grattan Endicott To: "Bridge Laws Mailing List" Sent: Wednesday, November 12, 2008 11:22 PM Subject: Re: [blml] Amsterdam (2) [SEC=UNOFFICIAL] > [ton] > Imo you are contributing to this ill-coordination and futile chatter. Somewhat below the level I expected from you. Combining the sentences: 'Is the said Director in hiding or something? Why are we not hearing from him as a key person?' with: 'Not that I think there is any right to expect that we would hear from him here' sounds like the bla bla of a drunken sailsman. Well, enjoy yourself > > [nige1] > Grattan doesn't seem to be criticising, he appears to be expressing our curiosity about the official version of events. For example, it's hard to understand why the name of the CTD is cloaked in secrecy :) > +=+ Nigel, the problem is that we have seen a number of accounts of the matter, mostly rather opaque and certainly demonstrating anomalies and divergences. As far as I can tell the question whether the responders to the appeal were fully and properly heard is a real one. Certainly I hear their allegations that they were not and the anger, emotion, that is voiced about it. Neither do we know what view was taken of the lack of protest at the time of the alleged irregularity and the desire latterly for a ruling (or rulings) when there was opportunity to turn a narrow defeat into victory. As has been observed, on blml we are not entitled to demand to know, although there has been some defence of what occurred. So the correspondence carries an odour of justice not seen to be done, a lack of transparency as to exactly how matters were handled, and what we do not know about the Director in charge and his actions contributes to this. One can see the scope for suspicion that the request for a ruling may have been moved past the Director without waiting for his appropriate action - and one can recognize the anxiety some may feel as to the possibilities for challenging the validity of the process if it were to appear flawed. We should not leap to judgement, but the questions will linger. ~ Grattan ~ +=+ From richard.hills at immi.gov.au Thu Nov 13 03:34:06 2008 From: richard.hills at immi.gov.au (richard.hills at immi.gov.au) Date: Thu, 13 Nov 2008 13:34:06 +1100 Subject: [blml] Amsterdam (2) [SEC=UNOFFICIAL] In-Reply-To: <001101c944e5$84fea410$0302a8c0@Mildred> Message-ID: Grattan Endicott, 12th November 2008: [snip] >I am surprised beyond belief that a Netherlands based event >should produce such a shambles since it is wholly contrary >to past experience of the careful adhesion of Netherlands >folk to principle and procedure. > ~ Grattan ~ +=+ BBO posting transcribed to blml, 4th September 2007: [snip] >>>In the Aug/Sept issue of the magazine, Roald Ramer defends >>>the position of the AC, while Hans van Staveren (until >>>recently member of the Dutch law committee), Ton Kooijman >>>(chairman of the WBF law committee) as well as the Belgian >>>bridge law expert Herman de Wael attack the AC's position >>>vigorously. [snip] >>>In the September issue of the BF magazine "Bridge", Berry >>>Westra defends the AC's position. He argues that the right >>>to misbid is from a time when psyches were common while >>>conventions were not. He thinks that there is a need to >>>protect the game against the growing convention madness >>>("steeds verder oprukkende conventiegekte"). >>> >>>The board of directors of the BF has announced that until >>>further notice, the AC must follow the laws, [snip] Richard Hills, 13th November 2008: So, provided the Netherlands Bridge Federation board of directors give notice, the Netherlands Appeals Committee is then again empowered to flout the Laws? Hans van Staveren, 4th September 2007: >>The TD did not rule against the player. The TD investigated, >>found out that all information supplied was according to >>system, and let the table result stand. >>The other side, knowing the tendencies of the Dutch AC, went >>into appeal and found a very sympathetic ear. >> >>As Herman knows, and as stated in the article, I am >>currently a Don Quichot like figure in the Netherlands. I >>resigned from my official position just to be able to shout >>at everybody until they stop this nonsense. >>In particular it is my position that the Dutch Bridge >>Federation should have fired the AC. Any AC that knowingly >>rules against the laws of bridge should go. Richard Hills, 13th November 2008: Sure, "careful adhesion of Netherlands folk to principle and procedure" would require sacking the Dutch National Appeals Committee. But much simpler is instead again carelessly violating Lawbook principles by Hans van Staveren, 5th December 2007: >...the high powers of the Dutch Bridge Federation want to >make the following change: > >Any agreement in non fixed partnerships, or any agreement >less than one year old in fixed partnerships becomes >essentially a non-agreement. Getting it wrong would >automatically lead to a ruling of misinformation. >Furthermore, all agreements older than a year in fixed >partnerships would need additional documentation, not just >the name on the convention card. >Directors would be forced to check for this, not just when >they are unsure of the misinformation/misbid situation. >... >Is this legal? Richard Hills, 13th November 2008: No. I am surprised beyond belief about such a shambles of an unLawful regulation which in effect rewords two immutable Laws in the 2007 Lawbook to read -> 2007 Law 40C1 (Dutch version): A player may not deviate from his side's announced understandings in a partnership of less than one year's standing and/or in a partnership with no system notes, notwithstanding that his partner has no more reason to be aware of the deviation than have the opponents. 2007 Law 75C (Dutch version): The partnership agreement is not as explained - 2D is not strong and artificial; because the mistake in South's call automatically cancels the partnership agreement. Here there is an infraction of Law, since East-West did not receive an accurate description of the North-South agreement since such an agreement by definition must be identical to an accurate description of the North-South hands. William Schoder ("Kojak"), 5th December 2007: [big snip] >or do I smell the 'control freak syndrome'? I hope not. > >Kojak > >P.S. Control Freak Syndrome = I've been given the power to >alter something, but if I never do so, I have been given >nothing. Very common disease in political hierarchies. Best wishes Richard James Hills Recruitment Section, Level 3 Blue, workstation 15 (first on left) Department of Immigration and Citizenship Telephone: 02 6223 8453 Email: richard.hills at immi.gov.au -------------------------------------------------------------------- Important Notice: If you have received this email by mistake, please advise the sender and delete the message and attachments immediately. This email, including attachments, may contain confidential, sensitive, legally privileged and/or copyright information. Any review, retransmission, dissemination or other use of this information by persons or entities other than the intended recipient is prohibited. DIAC respects your privacy and has obligations under the Privacy Act 1988. The official departmental privacy policy can be viewed on the department's website at www.immi.gov.au. See: http://www.immi.gov.au/functional/privacy.htm --------------------------------------------------------------------- From rfrick at rfrick.info Thu Nov 13 05:02:49 2008 From: rfrick at rfrick.info (Robert Frick) Date: Wed, 12 Nov 2008 23:02:49 -0500 Subject: [blml] New Laws don't allow UI situation to be resolved? [SEC=UNOFFICIAL] In-Reply-To: References: Message-ID: On Sun, 09 Nov 2008 20:20:18 -0500, wrote: > Robert Frick: > >>> If partner supplies the UI, I understand that I cannot and >>> should not choose an LA suggested by the UI. But if the UI >>> is extraneous and it is not my fault that I have it, then >>> it does not seem fair if I am ethically obligated not to >>> select alternatives suggested by the UI. > > Grattan Endicott: > >> +=+ Perhaps Robert should look again at Law 16A3. It is not >> a matter of ethics that he is addressing but a matter of >> law. >> ~ Grattan ~ +=+ > > Richard Hills: > > Perhaps Grattan should look again at Law 16A3 -> > > "No player may base a call or play on other information > (such information being designated extraneous).' > > It seems to me player may choose a call ***suggested*** by EI > not supplied by partner provided that the player does not > ***base*** that call on the EI. That is, if: > > (a) there are two logical alternatives A and B, > > and > > (b) the player would normally choose logical alternative A, > > and > > (c) the unauthorised information demonstrably suggests A, > > then > > (d1) if the UI came from partner B must be selected, > > but > > (d2) if the EI came from the next table A may be selected, > since calling A is not "based" on the EI but rather "based" > on what the player would normally choose. > > Of course, in case (d2) the TD should award an adjusted > score under Law 16C2(c), ruling both sides as nonoffending. > > What's the problem? You assume we can know what the player would do without the EI (extraneous information). If the player's decision is just following rules (I have 13 HCP and a 5-card spade suit so I open 1 Spade), this can be done. When it gets to judgments (should I open this garbage, what should I lead?), then there is no way to know. (E.g., you can't answer, "We missed a cold slam, tell me how you would bid this hand and ignore that I told you about the slam.") But L16A3 is still useful for EI. First, it covers the situation where a rule-based decision is possible. Second, suppose I decide to use the EI and lead a club. (The director should have stopped play, but did not.) If the club lead then works out poorly, can I seek redress from Law 16C2(c), that the unauthorized information influenced the result. But I would have to argue that the EI influenced my choice, which it wasn't supposed to according to L16A3. So I use the EI only at my own risk. Bob, who realized that blml is a book club -- we all read the same book and then discuss it. From richard.hills at immi.gov.au Thu Nov 13 07:19:19 2008 From: richard.hills at immi.gov.au (richard.hills at immi.gov.au) Date: Thu, 13 Nov 2008 17:19:19 +1100 Subject: [blml] Mr Chesterton suggested [SEC=UNOFFICIAL] In-Reply-To: <2CF77EB18250470CB513261A89089258@JOHN> Message-ID: G.K. Chesterton (1874-1936), describing Father Brown: "His head was always most valuable when he had lost it. In such moments he put two and two together and made four million." Sub-postcard size answer from WBF LC minutes, 10th October 2008: ".....compliance with the overriding requirement of the laws always to respond to enquiries under Law 20F with correct explanations of the partnership understandings." Herman De Wael: >>>Allow me one comment: my question was: >>>How do _YOU_ _FEEL_ (two separate pieces of emphasis) about a >>>law that _obliges_ you to give UI. John (MadDog) Probst: >>Perfectly comfortable. The giving of UI is a necessary evil. Herman De Wael: >And there was still my third "quiz" question. How many laws in >the lawbook explicitly forbid a player to give a piece of UI to >his partner? And how many oblige him to do so? Law 73D - Variations in Tempo or Manner: 1. It is desirable, though **not always** required, for players to maintain steady tempo and unvarying manner. However, players should be particularly careful when variations may work to the benefit of their side. Otherwise, **unintentionally** to vary the tempo or manner in which a call or play is made is **not in itself** an infraction. Inferences from such variation may appropriately be drawn only by an opponent, and at his own risk. 2. A player may not **attempt to mislead** an opponent by means of remark or gesture, by the haste or hesitancy of a call or play (as in **hesitating before playing a singleton**), the manner in which a call or play is made or by any **purposeful** deviation from correct procedure. Richard Hills: Correctly describing your partnership understanding in response to an opponent's question is not: (a) a purposeful deviation, since the purpose is imposed by an overriding requirement of the Laws, nor (b) an attempt to mislead the opponents about the partnership understanding, since it is a correct description, nor (c) an intentional variation from an unvarying manner of impassive silence, since an opponent has ordered you to speak, and Law 20F mandates a response. What's the problem? See below. Herman De Wael: >I'm not talking about giving UI, I'm talking about being _obliged_ >to give UI. > >Everything I do at the bridge table is an attempt to keep from >giving UI to my partner, from my poker face to my somewhat hasty >bids. > >And here is a law that would tell me that I am forced to give UI. >It goes against my nature. Richard Hills: C'est la vie. Herman De Wael: >So I don't do it. Richard Hills: Non sequitur. Some aspects of Duplicate Bridge Law go against my nature. For example I believe that the 2007 Law 79C2 has the wrong default, which should be changed from the current wording: "Regulations may provide for circumstances in which a scoring error may be corrected....." to this 2018 wording: "Unless the Regulating Authority decides otherwise, a scoring error may be corrected....." But when I direct I do not contemplate violating Law 79C2 or any other Law. In my opinion a Director who deliberately ignores the Lawbook is a contradiction in terms. Herman De Wael: >Which leads me to my second question: You're the TD, and I have >"broken" this law telling me to give UI. I have not given UI. What >adjustment do you propose? Richard Hills: The adjustment I suggest for Herman is that he might wish to set up a home game or private bridge club in which his heterodox variations on the Duplicate Laws were the House Rules. (Indeed, many so-called duplicate bridge clubs use a House Rules Lawbook -- although in most cases the variation is the repeal of Law 16.) But when playing or directing under the official WBF Duplicate Lawbook, Interpretations and Code of Practice, in my opinion Herman De Wael has an obligation to try to obey those official rules. G.K. Chesterton, On Detective Novels (1928): "Of one preliminary principle I am pretty certain. The whole point of a sensational story is that the secret should be simple. The whole story exists for the moment of surprise; and it should be a moment. It should not be something that it takes twenty minutes to explain, and twenty-four hours to learn by heart, for fear of forgetting it. The best way of testing it is to make an imaginative picture in the mind of some dramatic moment. Imagine a dark garden at twilight, and a terrible voice crying out in the distance, and coming nearer and nearer along the serpentine garden paths until the words become dreadfully distinct; a cry coming from some sinister yet familiar figure in the story, a stranger or a servant from whom we sub- consciously expect some such rending revelation. Now, it is clear that the cry which breaks from him must be something short and simple in itself, as, 'The butler is his father', or 'The Archdeacon is Bloody Bill' ..." Ultimately my secret of this debate about principle comes down to my simple point: "In my opinion a Director who deliberately ignores the Lawbook is a contradiction in terms." >Herman. (pot or kettle, I don't care: you're all black) Best wishes Richard James Hills Pot-valiant and pretty kettle of fish -------------------------------------------------------------------- Important Notice: If you have received this email by mistake, please advise the sender and delete the message and attachments immediately. This email, including attachments, may contain confidential, sensitive, legally privileged and/or copyright information. Any review, retransmission, dissemination or other use of this information by persons or entities other than the intended recipient is prohibited. DIAC respects your privacy and has obligations under the Privacy Act 1988. The official departmental privacy policy can be viewed on the department's website at www.immi.gov.au. See: http://www.immi.gov.au/functional/privacy.htm --------------------------------------------------------------------- From dalburn at btopenworld.com Thu Nov 13 08:19:46 2008 From: dalburn at btopenworld.com (David Burn) Date: Thu, 13 Nov 2008 07:19:46 -0000 Subject: [blml] Mr Chesterton suggested [SEC=UNOFFICIAL] In-Reply-To: <491B5AFD.8000604@skynet.be> References: <2CF77EB18250470CB513261A89089258@JOHN> <491B5AFD.8000604@skynet.be> Message-ID: <000001c94560$3567ecb0$a037c610$@com> [HdW] And there was still my third "quiz" question. How many laws in the lawbook explicitely forbid a player to give a piece of UI to his partner? [DALB] One, Law 73, in certain circumstances only that could easily be avoided. [Hdw] And how many oblige him to do so? [DALB] One, Law 20, in certain circumstances only that could easily be avoided. Do I win? No, I thought not. David Burn London, England From sater at xs4all.nl Thu Nov 13 08:28:17 2008 From: sater at xs4all.nl (Hans van Staveren) Date: Thu, 13 Nov 2008 08:28:17 +0100 Subject: [blml] Amsterdam (2) [SEC=UNOFFICIAL] In-Reply-To: References: <001101c944e5$84fea410$0302a8c0@Mildred> Message-ID: <002e01c94561$661116b0$32334410$@nl> Here I have to step in: Whatever you may think of the Dutch Bridge Federation, and the less than ideal way the last AC crisis was handled, this has nothing whatsoever to do with what happened in Amsterdam. This was a European tournament. I do not know who the acting TD was, I do not know who the AC was, but it was most surely not the Dutch National AC. Don't make this mess bigger. All this crazy speculation is leading to trial by mob. Just wait for the handling of the official appeal the Germans are about to make. Hans van Staveren From geller at nifty.com Thu Nov 13 08:49:50 2008 From: geller at nifty.com (Robert Geller) Date: Thu, 13 Nov 2008 16:49:50 +0900 Subject: [blml] Amsterdam (2) [SEC=UNOFFICIAL] In-Reply-To: <002e01c94561$661116b0$32334410$@nl> References: <002e01c94561$661116b0$32334410$@nl> Message-ID: <200811130749.AA16483@geller204.nifty.com> Just for information, what was the name of the event and when was it held? This was probably noted upthread, but I don't want to search through all the mails again. Thanks. -Bob Hans van Staveren wrote: >Here I have to step in: > >Whatever you may think of the Dutch Bridge Federation, and the less than >ideal way the last AC crisis was handled, this has nothing whatsoever to do >with what happened in Amsterdam. This was a European tournament. I do not >know who the acting TD was, I do not know who the AC was, but it was most >surely not the Dutch National AC. > >Don't make this mess bigger. > >All this crazy speculation is leading to trial by mob. Just wait for the >handling of the official appeal the Germans are about to make. > >Hans van Staveren > > > > >_______________________________________________ >blml mailing list >blml at amsterdamned.org >http://www.amsterdamned.org/mailman/listinfo/blml From sater at xs4all.nl Thu Nov 13 08:57:00 2008 From: sater at xs4all.nl (Hans van Staveren) Date: Thu, 13 Nov 2008 08:57:00 +0100 Subject: [blml] Amsterdam (2) [SEC=UNOFFICIAL] In-Reply-To: <200811130749.AA16483@geller204.nifty.com> References: <002e01c94561$661116b0$32334410$@nl> <200811130749.AA16483@geller204.nifty.com> Message-ID: <002f01c94565$6ac61670$40524350$@nl> European Champions Cup, last weekend. It was a European tournament, organized by the EBL, hosted by the Netherlands. Here is the link to the appeal: http://www.bridge.nl/documenten/appaelccup.pdf Hans -----Original Message----- From: blml-bounces at amsterdamned.org [mailto:blml-bounces at amsterdamned.org] On Behalf Of Robert Geller Sent: donderdag 13 november 2008 8:50 To: Bridge Laws Mailing List Subject: Re: [blml] Amsterdam (2) [SEC=UNOFFICIAL] Just for information, what was the name of the event and when was it held? This was probably noted upthread, but I don't want to search through all the mails again. Thanks. -Bob Hans van Staveren wrote: >Here I have to step in: > >Whatever you may think of the Dutch Bridge Federation, and the less than >ideal way the last AC crisis was handled, this has nothing whatsoever to do >with what happened in Amsterdam. This was a European tournament. I do not >know who the acting TD was, I do not know who the AC was, but it was most >surely not the Dutch National AC. > >Don't make this mess bigger. > >All this crazy speculation is leading to trial by mob. Just wait for the >handling of the official appeal the Germans are about to make. > >Hans van Staveren > > > > >_______________________________________________ >blml mailing list >blml at amsterdamned.org >http://www.amsterdamned.org/mailman/listinfo/blml _______________________________________________ blml mailing list blml at amsterdamned.org http://www.amsterdamned.org/mailman/listinfo/blml From schoderb at msn.com Thu Nov 13 02:16:57 2008 From: schoderb at msn.com (WILLIAM SCHODER) Date: Wed, 12 Nov 2008 20:16:57 -0500 Subject: [blml] Amsterdam (2) [SEC=UNOFFICIAL] In-Reply-To: <491B6535.5020201@talktalk.net> References: <491B6535.5020201@talktalk.net> Message-ID: The real professional is the one who acknowledges error on his part, and learns from it. That is what establishes respect from those he deals with. I have no comments on the AC decision on "appeal" 3 -- I do have serious reservations about departing from the laws, and then not having the guts to stand up and be counted. I continue to deplore creating situations which demean the role of the Director and the Laws, simply because "it feels right." "it saves time" " it makes no difference" "either side will appeal." This will quickly lead to destruction of respect for the game, the laws, the Directors, and the Administrators. Again, let me be clear, I don't comment on the decision arrived at by the AC -- I do comment upon the departure from normal procedure by both the Director and the AC in the apparent handling of a serious case, where the players deserved better treatment than they received. Seems we will never get an "official" version of the events. and, if by chance, the Director did not want to make an appealable decision which might change the winner of a match, then that individual would, In the American idiom, "be better off selling shoes" rather than hiding behind the AC to do his job for him, and profanely lashing out against those who task him for his screw up. I have yet to get an answer from Ton on "who am I?" when he asked me what I would have done, but regardless of his answer my position is, I hope, clear. Kojak ----- Original Message ----- From: Nigel Guthrie To: Bridge Laws Mailing List Sent: Wednesday, November 12, 2008 6:22 PM Subject: Re: [blml] Amsterdam (2) [SEC=UNOFFICIAL] [ton] Imo you are contributing to this ill-coordination and futile chatter. Somewhat below the level I expected from you. Combining the sentences: 'Is the said Director in hiding or something? Why are we not hearing from him as a key person?' with: 'Not that I think there is any right to expect that we would hear from him here' sounds like the bla bla of a drunken sailsman. Well, enjoy yourself [nige1] Grattan doesn't seem to be criticising, he appears to be expressing our curiosity about the official version of events. For example, it's hard to understand why the name of the CTD is cloaked in secrecy :) _______________________________________________ blml mailing list blml at amsterdamned.org http://www.amsterdamned.org/mailman/listinfo/blml -------------- next part -------------- An HTML attachment was scrubbed... URL: http://www.amsterdamned.org/pipermail/blml/attachments/20081112/86d61e7a/attachment.htm From sater at xs4all.nl Thu Nov 13 09:05:47 2008 From: sater at xs4all.nl (Hans van Staveren) Date: Thu, 13 Nov 2008 09:05:47 +0100 Subject: [blml] Amsterdam (2) [SEC=UNOFFICIAL] In-Reply-To: <1L0JfQ-0IQYD20@fwd03.aul.t-online.de> References: <1L0JfQ-0IQYD20@fwd03.aul.t-online.de> Message-ID: <003001c94566$a5395af0$efac10d0$@nl> I just saw the name of Josef under the letter the DBF sent to the EBL. Josef seems to be the vice president sports of the DBF. Josef, If you sent a letter of protest to the EBL on November 11, and then say "correct me if I am wrong" on BLML(of all places) on November 12, you could be accused of jumping to conclusions and behavior not worthy of a vice president. But apart from this, it is my strong feeling that the way this is handled here on BLML is now way beyond what is still acceptable in a polite society. Something happened in Amsterdam, there are unhappy people. An enquiry will be made, a result will be forthcoming. It is fitting to accept that and not make things worse. And no, I was not involved. Hans From: blml-bounces at amsterdamned.org [mailto:blml-bounces at amsterdamned.org] On Behalf Of Harsanyi at t-online.de Sent: woensdag 12 november 2008 18:38 To: Bridge Laws Mailing List Subject: Re: [blml] Amsterdam (2) [SEC=UNOFFICIAL] Solon (c.630 - c.555 BCE): "Laws are like spider's webs: if some poor weak creature come up against them, it is caught; but a bigger one can break through and get away." It has nothing to do with the law, if a player do not registrate any delay and do not call the TD during the bidding and playing of a board, but later, after talking about the hands with his team mates goes to an organiser, or to the TD and says, that in the second board of the last match came the tray too slow back, my partner on the other side of the screen is ready to confirm, that the opponent did bid slowly. There was some UI and it was used, please change the score! The TD is not ready to decide with such a case, and puts the work into the hands of the AC. The AC decides without hearing the accused side, and penalise them in a KO match. So makes the AC winner from the looser. It is not a joke, it is something, wich can not be tolerated. I was informed about this case by a non english speaking player at this table. Please correct me if I am wrong. Josef -------------- next part -------------- An HTML attachment was scrubbed... URL: http://www.amsterdamned.org/pipermail/blml/attachments/20081113/cdb794bd/attachment.htm From geller at nifty.com Thu Nov 13 09:17:59 2008 From: geller at nifty.com (Robert Geller) Date: Thu, 13 Nov 2008 17:17:59 +0900 Subject: [blml] Amsterdam (2) [SEC=UNOFFICIAL] In-Reply-To: <002f01c94565$6ac61670$40524350$@nl> References: <002f01c94565$6ac61670$40524350$@nl> Message-ID: <200811130817.AA16486@geller204.nifty.com> The Daily Bulletions (see below) http://www.eurobridge.org/competitions/08Amsterdam/Bulletins.htm don't mention the name of the DIC. No reason they necessarily should, but isn't this usually done? At least we usually do this for the Daily Bulletins of the NEC tourney in Japn or the PABF (zonal) championships. -Bob Hans van Staveren writes: >European Champions Cup, last weekend. >It was a European tournament, organized by the EBL, hosted by the >Netherlands. > >Here is the link to the appeal: >http://www.bridge.nl/documenten/appaelccup.pdf > >Hans > >-----Original Message----- >From: blml-bounces at amsterdamned.org [mailto:blml-bounces at amsterdamned.org] >On Behalf Of Robert Geller >Sent: donderdag 13 november 2008 8:50 >To: Bridge Laws Mailing List >Subject: Re: [blml] Amsterdam (2) [SEC=UNOFFICIAL] > >Just for information, what was the name of the event and when was it held? >This was probably noted upthread, but I don't want to search through all the >mails again. Thanks. >-Bob > >Hans van Staveren wrote: >>Here I have to step in: >> >>Whatever you may think of the Dutch Bridge Federation, and the less than >>ideal way the last AC crisis was handled, this has nothing whatsoever to do >>with what happened in Amsterdam. This was a European tournament. I do not >>know who the acting TD was, I do not know who the AC was, but it was most >>surely not the Dutch National AC. >> >>Don't make this mess bigger. >> >>All this crazy speculation is leading to trial by mob. Just wait for the >>handling of the official appeal the Germans are about to make. >> >>Hans van Staveren >> >> >> >> >>_______________________________________________ >>blml mailing list >>blml at amsterdamned.org >>http://www.amsterdamned.org/mailman/listinfo/blml > >_______________________________________________ >blml mailing list >blml at amsterdamned.org >http://www.amsterdamned.org/mailman/listinfo/blml > > >_______________________________________________ >blml mailing list >blml at amsterdamned.org >http://www.amsterdamned.org/mailman/listinfo/blml From Hermandw at skynet.be Thu Nov 13 10:02:41 2008 From: Hermandw at skynet.be (Herman De Wael) Date: Thu, 13 Nov 2008 10:02:41 +0100 Subject: [blml] Mr Chesterton suggested [SEC=UNOFFICIAL] In-Reply-To: <000001c94560$3567ecb0$a037c610$@com> References: <2CF77EB18250470CB513261A89089258@JOHN> <491B5AFD.8000604@skynet.be> <000001c94560$3567ecb0$a037c610$@com> Message-ID: <491BED31.40509@skynet.be> Thanks David, for the reply - i'll await (probably in vain) for other answers before telling you how you've done. Herman. David Burn wrote: > [HdW] > > And there was still my third "quiz" question. How many laws in the lawbook > explicitely forbid a player to give a piece of UI to his > partner? > > [DALB] > > One, Law 73, in certain circumstances only that could easily be avoided. > > [Hdw] > > And how many oblige him to do so? > > [DALB] > > One, Law 20, in certain circumstances only that could easily be avoided. > > Do I win? No, I thought not. > > David Burn > London, England > > > _______________________________________________ > blml mailing list > blml at amsterdamned.org > http://www.amsterdamned.org/mailman/listinfo/blml > From Hermandw at skynet.be Thu Nov 13 10:05:45 2008 From: Hermandw at skynet.be (Herman De Wael) Date: Thu, 13 Nov 2008 10:05:45 +0100 Subject: [blml] Mr Chesterton suggested [SEC=UNOFFICIAL] In-Reply-To: References: Message-ID: <491BEDE9.1080804@skynet.be> Once again, Richard chooses only some of my words to pick upon. He bluntly refuses to answer two of my questions. Who was calling who black? Herman. [endless drivel snipped] From Hermandw at skynet.be Thu Nov 13 10:08:44 2008 From: Hermandw at skynet.be (Herman De Wael) Date: Thu, 13 Nov 2008 10:08:44 +0100 Subject: [blml] Amsterdam (2) [SEC=UNOFFICIAL] In-Reply-To: <491B6535.5020201@talktalk.net> References: <491B6535.5020201@talktalk.net> Message-ID: <491BEE9C.5060301@skynet.be> Nigel Guthrie wrote: > [ton] Imo you are contributing to this ill-coordination and futile > chatter. Somewhat below the level I expected from you. Combining the > sentences: 'Is the said Director in hiding or something? Why are we > not hearing from him as a key person?' with: 'Not that I think there > is any right to expect that we would hear from him here' sounds like > the bla bla of a drunken sailsman. Well, enjoy yourself > > [nige1] Grattan doesn't seem to be criticising, he appears to be > expressing our curiosity about the official version of events. For > example, it's hard to understand why the name of the CTD is cloaked > in secrecy :) I don't see any cloak of secrecy. I don't see any reason why the director should be known. This is not an official inquiry. For a person of Grattan's stature to use insinuating language like that is indeed very remarkable. And if you are reading Grattan's words as "curiosity" not "criticism", then we don't have the same mother language :) Herman. From grandaeval at tiscali.co.uk Thu Nov 13 10:46:15 2008 From: grandaeval at tiscali.co.uk (Grattan) Date: Thu, 13 Nov 2008 09:46:15 -0000 Subject: [blml] Amsterdam (2) [SEC=UNOFFICIAL] References: <001101c944e5$84fea410$0302a8c0@Mildred> <002e01c94561$661116b0$32334410$@nl> Message-ID: <003501c94574$fb66f000$0302a8c0@Mildred> Grattan Endicott To: "'Bridge Laws Mailing List'" Sent: Thursday, November 13, 2008 7:28 AM Subject: Re: [blml] Amsterdam (2) [SEC=UNOFFICIAL] > Here I have to step in: > Whatever you may think of the Dutch Bridge Federation, and the less than ideal way the last AC crisis was handled, this has nothing whatsoever to do with what happened in Amsterdam. This was a European tournament. I do not know who the acting TD was, I do not know who the AC was, but it was most surely not the Dutch National AC. > +=+ I am more than a little surprised that only now does anyone think it helpful to tell us this. Not that it alters the questions, but it does place them away from any Netherlands connection. If the matter was opened up on blml by someone who was taking the case to further appeal I deprecate that action. At the very least I would have expected him to make that information known. From the nature of the question as it was raised I was under the impression that we were discussing a closed matter, not one still live. I have looked at the case on the web site. The report does not include statements on the procedure and of course I have no intention of commenting currently on the actual AC decision. Are not the identities of officials at the event matters of public knowledge? ~ Grattan ~ +=+ From geller at nifty.com Thu Nov 13 10:54:53 2008 From: geller at nifty.com (Robert Geller) Date: Thu, 13 Nov 2008 18:54:53 +0900 Subject: [blml] Amsterdam (2) [SEC=UNOFFICIAL] In-Reply-To: <003501c94574$fb66f000$0302a8c0@Mildred> References: <003501c94574$fb66f000$0302a8c0@Mildred> Message-ID: <200811130954.AA16494@geller204.nifty.com> Grattan wrote: >Are not the identities of officials at the event matters of public >knowledge? "... and the DIC's face is always well-hidden, and a hard, a hard, a hard rain's gonna fall." (with apologies to Bob Dylan) -Bob ----------------------------------------------------- Robert (Bob) Geller, Tokyo, Japan geller at nifty.com From agot at ulb.ac.be Thu Nov 13 11:16:47 2008 From: agot at ulb.ac.be (Alain Gottcheiner) Date: Thu, 13 Nov 2008 11:16:47 +0100 Subject: [blml] Mr Chesterton suggested [SEC=UNOFFICIAL] In-Reply-To: <491B5AFD.8000604@skynet.be> References: <2CF77EB18250470CB513261A89089258@JOHN> <491B5AFD.8000604@skynet.be> Message-ID: <491BFE8F.7080105@ulb.ac.be> Herman De Wael a ?crit : > John, you too, responded to only one question, not all three. > > John (MadDog) Probst wrote: > >>> Herman De Wael: >>> >>> >>>> Allow me one comment: my question was: >>>> How do _YOU_ _FEEL_ (two separate pieces of emphasis) about a >>>> law that _obliges_ you to give UI. >>>> >> Perfectly comfortable. The giving of UI is a necessary evil. John >> >> > > I'm not talking about giving UI, I'm talking about being _obliged_ to > give UI. > AG : that's the point indeed. The giving of UI is only necessary because of the MS interpretation. I admit that it is perfectly clear from the rules and several notices, but this doesn't mean it is optimal. My teacher had a motto : "always try to save the board". That is, do everything possible to get an unaffected result. Surely this influences me. Surely, too, there has been a step in the right direction wuith the new revoke law. Creating UI will give you very little change to do so : either the player in possession of UI will "bend backwards" and shoot himself in the foot ; or he won't do it, and there will be an adjusted score. Other ways to cope with the problem give us a better chance to do so. The MS / dWS antagonism isn't about MI vs UI ; it is about "tell the truth" vs "save the board". The present state is that, because players aren't allowed to take steps to "save the board", and because calling the TD would in a way create the same UI, players don't have any choice. But some of us think there are better ways to handle the problem. The ideal way to proceed would be to request that explanations be written, but this takes some time. Best regards Alain From grandaeval at tiscali.co.uk Thu Nov 13 11:24:54 2008 From: grandaeval at tiscali.co.uk (Grattan) Date: Thu, 13 Nov 2008 10:24:54 -0000 Subject: [blml] Amsterdam (2) [SEC=UNOFFICIAL] References: <491B6535.5020201@talktalk.net> <491BEE9C.5060301@skynet.be> Message-ID: <004501c9457a$1abb77f0$0302a8c0@Mildred> Grattan Endicott To: "Bridge Laws Mailing List" Sent: Thursday, November 13, 2008 9:08 AM Subject: Re: [blml] Amsterdam (2) [SEC=UNOFFICIAL] I don't see any cloak of secrecy. I don't see any reason why the director should be known. This is not an official inquiry. For a person of Grattan's stature to use insinuating language like that is indeed very remarkable. > And if you are reading Grattan's words as "curiosity" not "criticism", then we don't have the same mother language :) > Herman. > +=+ I do not regard what I said as either criticism or curiosity, but rather as observation of fact. Insofar as our knowledge goes I do have criticisms of the apparent failure to comply with the Laws and of any suggestion that any such failure was of little consequence. In this respect I share wholly Kojak's concerns and do not accept that a member of the WBF Laws Committee may condone it. When there is a request for a ruling the Director's training, professionalism, experience, practice, and obligation to consult, are not to be discounted - and especially not by the Director himself. I can tolerate the thought that I do not enjoy the same mother tongue as Herman; I feel this may not be the only occasion on which we have had evidence of the fact. However, as to 'cloak of secrecy' what is lacking is not so much the identity of the Director in charge as a record of his action on receiving the request for a ruling and his decision with regard to it - if, as it seems, he allowed the table score to stand, what were his reasons (which the AC determined they had persuasive evidence to overturn)? ~ Grattan ~ +=+ From Hermandw at skynet.be Thu Nov 13 13:08:32 2008 From: Hermandw at skynet.be (Herman De Wael) Date: Thu, 13 Nov 2008 13:08:32 +0100 Subject: [blml] Amsterdam (2) [SEC=UNOFFICIAL] In-Reply-To: <004501c9457a$1abb77f0$0302a8c0@Mildred> References: <491B6535.5020201@talktalk.net> <491BEE9C.5060301@skynet.be> <004501c9457a$1abb77f0$0302a8c0@Mildred> Message-ID: <491C18C0.7050008@skynet.be> Hello Grattan, Grattan wrote: > > Grattan Endicott also ************************************ > "We desire truth and find within > ourselves only uncertainty." > [Pascal] > ''''''''''''''''''''''''''''''''''''''''''''''''''''''''''''''''''''''''''''''''''''''''''''''''' > > > > ----- Original Message ----- > From: "Herman De Wael" > To: "Bridge Laws Mailing List" > Sent: Thursday, November 13, 2008 9:08 AM > Subject: Re: [blml] Amsterdam (2) [SEC=UNOFFICIAL] > > > I don't see any cloak of secrecy. > I don't see any reason why the director should be known. This is not an > official inquiry. > For a person of Grattan's stature to use insinuating language like that > is indeed very remarkable. > And if you are reading Grattan's words as "curiosity" not "criticism", > then we don't have the same mother language :) > Herman. > +=+ I do not regard what I said as either criticism or curiosity, but > rather as observation of fact. Insofar as our knowledge goes I do > have criticisms of the apparent failure to comply with the Laws and > of any suggestion that any such failure was of little consequence. In > this respect I share wholly Kojak's concerns and do not accept that > a member of the WBF Laws Committee may condone it. > When there is a request for a ruling the Director's training, > professionalism, experience, practice, and obligation to consult, are > not to be discounted - and especially not by the Director himself. Your concerns are noted. They are correct as well. Perhaps we needed to be there to know why the Directors thought they could get away with a (IMO) minor deviation of the laws. I have once assisted to this final and it is a very pleasant matter, where even appeals are handled in a friendly athmosphere between players who know one another quite well. > I can tolerate the thought that I do not enjoy the same mother > tongue as Herman; I was commenting on Nigel's comment who thought your opinions were of curiosity - I did not read them as such, but then again I don't share the same mother tongue with Nigel. > I feel this may not be the only occasion on which > we have had evidence of the fact. However, as to 'cloak of secrecy' > what is lacking is not so much the identity of the Director in charge > as a record of his action on receiving the request for a ruling and his > decision with regard to it - if, as it seems, he allowed the table score > to stand, what were his reasons (which the AC determined they had > persuasive evidence to overturn)? > ~ Grattan ~ +=+ > But the answer is the same. Why should we be in possession of anything? We are quite lucky, here on blml, to have first-hand accounts of two of the main protagonists. Since these are from different sides of the issue, this enables us to form an unbiased opinion of what likely happened. But we are not the official inquiry (not yet) and we should not demand things we have no right to. Nor should we look for sinister complots simply because we have no word from the Italians or the AC Chairman. And we don't even know if the AC overturned the TD's decision to allow the table score to stand - as was said, the TD did not take a position in the end. Maybe the laws insist that he should. If they do, then IMO any decision (including the tossing of a coin) is then enough. Surely we don't want directors to give decisions that way - isn't it better then to allow them to say that since the matter is already in the hands of the AC, no TD decision is necessary? Herman. From Hermandw at skynet.be Thu Nov 13 13:11:43 2008 From: Hermandw at skynet.be (Herman De Wael) Date: Thu, 13 Nov 2008 13:11:43 +0100 Subject: [blml] Amsterdam (2) [SEC=UNOFFICIAL] In-Reply-To: <003501c94574$fb66f000$0302a8c0@Mildred> References: <001101c944e5$84fea410$0302a8c0@Mildred> <002e01c94561$661116b0$32334410$@nl> <003501c94574$fb66f000$0302a8c0@Mildred> Message-ID: <491C197F.9090106@skynet.be> Grattan wrote: > > Grattan Endicott also ************************************ > "We desire truth and find within > ourselves only uncertainty." > [Pascal] > ''''''''''''''''''''''''''''''''''''''''''''''''''''''''''''''''''''''''''''''''''''''''''''''''' > > > > ----- Original Message ----- > From: "Hans van Staveren" > To: "'Bridge Laws Mailing List'" > Sent: Thursday, November 13, 2008 7:28 AM > Subject: Re: [blml] Amsterdam (2) [SEC=UNOFFICIAL] > > >> Here I have to step in: >> > Whatever you may think of the Dutch Bridge Federation, and > the less than ideal way the last AC crisis was handled, this has > nothing whatsoever to do with what happened in Amsterdam. > This was a European tournament. I do not know who the acting > TD was, I do not know who the AC was, but it was most > surely not the Dutch National AC. > +=+ I am more than a little surprised that only now does anyone > think it helpful to tell us this. Not that it alters the questions, but > it does place them away from any Netherlands connection. > If the matter was opened up on blml by someone who was > taking the case to further appeal I deprecate that action. At the > very least I would have expected him to make that information > known. From the nature of the question as it was raised I was > under the impression that we were discussing a closed matter, > not one still live. > I have looked at the case on the web site. The report does > not include statements on the procedure and of course I have > no intention of commenting currently on the actual AC decision. > Are not the identities of officials at the event matters of public > knowledge? > ~ Grattan ~ +=+ > Yes they are, and if we ask nicely, perhaps Ton will tell us who they are. But does that matter? Should not someone like yourself start off by saying that the AC must have had good reasons to do as they did? Rather than the reverse: seek for complot theories because the AC does not speak in its defence? Why should they have to defend themselves, I would rather say! Herman. From schoderb at msn.com Thu Nov 13 13:15:40 2008 From: schoderb at msn.com (WILLIAM SCHODER) Date: Thu, 13 Nov 2008 07:15:40 -0500 Subject: [blml] Amsterdam (2) [SEC=UNOFFICIAL] In-Reply-To: <491B6535.5020201@talktalk.net> <491BEE9C.5060301@skynet.be><004501c9457a$1abb77f0$0302a8c0@Mildred> <491C18C0.7050008@skynet.be> References: <491B6535.5020201@talktalk.net> <491BEE9C.5060301@skynet.be><004501c9457a$1abb77f0$0302a8c0@Mildred> <491C18C0.7050008@skynet.be> Message-ID: Herman wrote: Surely we > don't want directors to give decisions that way - isn't it better then > to allow them to say that since the matter is already in the hands of > the AC, no TD decision is necessary? > > Herman. > No. Kojak From blml at bridgescore.de Thu Nov 13 14:01:16 2008 From: blml at bridgescore.de (Christian Farwig (BLML)) Date: Thu, 13 Nov 2008 14:01:16 +0100 Subject: [blml] Amsterdam (2) [SEC=UNOFFICIAL] In-Reply-To: <491C197F.9090106@skynet.be> References: <001101c944e5$84fea410$0302a8c0@Mildred> <002e01c94561$661116b0$32334410$@nl> <003501c94574$fb66f000$0302a8c0@Mildred> <491C197F.9090106@skynet.be> Message-ID: <491C251C.2070807@bridgescore.de> Hi, in the Amsterdam case, which has huge discrepancies in the description of events between the parties involved, the statement of the director in charge would help a lot in shedding light on the issues. Until then, we are basically stuck. This seems so totally obvious to me that I am really astonished about the heated discussion b/o this. Best regards, Christian Herman De Wael schrieb: > Grattan wrote: > >> Grattan Endicott> also > ************************************ >> "We desire truth and find within >> ourselves only uncertainty." >> [Pascal] >> ''''''''''''''''''''''''''''''''''''''''''''''''''''''''''''''''''''''''''''''''''''''''''''''''' >> >> >> >> ----- Original Message ----- >> From: "Hans van Staveren" >> To: "'Bridge Laws Mailing List'" >> Sent: Thursday, November 13, 2008 7:28 AM >> Subject: Re: [blml] Amsterdam (2) [SEC=UNOFFICIAL] >> >> >> >>> Here I have to step in: >>> >>> >> Whatever you may think of the Dutch Bridge Federation, and >> the less than ideal way the last AC crisis was handled, this has >> nothing whatsoever to do with what happened in Amsterdam. >> This was a European tournament. I do not know who the acting >> TD was, I do not know who the AC was, but it was most >> surely not the Dutch National AC. >> +=+ I am more than a little surprised that only now does anyone >> think it helpful to tell us this. Not that it alters the questions, but >> it does place them away from any Netherlands connection. >> If the matter was opened up on blml by someone who was >> taking the case to further appeal I deprecate that action. At the >> very least I would have expected him to make that information >> known. From the nature of the question as it was raised I was >> under the impression that we were discussing a closed matter, >> not one still live. >> I have looked at the case on the web site. The report does >> not include statements on the procedure and of course I have >> no intention of commenting currently on the actual AC decision. >> Are not the identities of officials at the event matters of public >> knowledge? >> ~ Grattan ~ +=+ >> >> > > Yes they are, and if we ask nicely, perhaps Ton will tell us who they are. > But does that matter? > Should not someone like yourself start off by saying that the AC must > have had good reasons to do as they did? Rather than the reverse: seek > for complot theories because the AC does not speak in its defence? > Why should they have to defend themselves, I would rather say! > > Herman. > > > > _______________________________________________ > blml mailing list > blml at amsterdamned.org > http://www.amsterdamned.org/mailman/listinfo/blml > > From Hermandw at skynet.be Thu Nov 13 14:06:53 2008 From: Hermandw at skynet.be (Herman De Wael) Date: Thu, 13 Nov 2008 14:06:53 +0100 Subject: [blml] Amsterdam (2) [SEC=UNOFFICIAL] In-Reply-To: References: <491B6535.5020201@talktalk.net> <491BEE9C.5060301@skynet.be><004501c9457a$1abb77f0$0302a8c0@Mildred> <491C18C0.7050008@skynet.be> Message-ID: <491C266D.7050806@skynet.be> WILLIAM SCHODER wrote: > Herman wrote: > > > Surely we >> don't want directors to give decisions that way - isn't it better then >> to allow them to say that since the matter is already in the hands of >> the AC, no TD decision is necessary? >> >> Herman. >> > No. Kojak > You prefer the TD to give the decision by a toss of a coin then. Well, I guess you're right at that. Herman. From Hermandw at skynet.be Thu Nov 13 14:12:57 2008 From: Hermandw at skynet.be (Herman De Wael) Date: Thu, 13 Nov 2008 14:12:57 +0100 Subject: [blml] A player's comment Message-ID: <491C27D9.4010501@skynet.be> Heard yesterday from a (top-level) player in Belgium. She was playing in a club tournament, and the bidding went: 1NT - 2Di - pass - 2Sp pass- 3NT - pass - 4Di pass- 4Sp (there may have been some doubles in there) 2Di was a multi, and the player had long hearts. It was not alerted. 2Sp was obviously intended as natural, and the player had long spades. In the Multi-system, 2Sp shows heart support. 2Sp was alerted. Since 4Sp went lots down, there was no TD call any more. But what she complained most about was the alert on 2Sp. She considered this most unethical, on an equal footing with saying "2Di was Multi". I told her that I agreed with her, but that most of you would act the same way. I shall not repeat the words she used for all of you. Herman. From john at asimere.com Thu Nov 13 14:14:10 2008 From: john at asimere.com (John (MadDog) Probst) Date: Thu, 13 Nov 2008 13:14:10 -0000 Subject: [blml] Mr Chesterton suggested [SEC=UNOFFICIAL] References: <2CF77EB18250470CB513261A89089258@JOHN> <491B5AFD.8000604@skynet.be> Message-ID: <7A05D24604AF42DFAC4C29F8329E46C9@JOHN> ----- Original Message ----- From: "Herman De Wael" To: "Bridge Laws Mailing List" Sent: Wednesday, November 12, 2008 10:38 PM Subject: Re: [blml] Mr Chesterton suggested [SEC=UNOFFICIAL] > John, you too, responded to only one question, not all three. > > John (MadDog) Probst wrote: >>> >>> Herman De Wael: >>> >>>> Allow me one comment: my question was: >>>> How do _YOU_ _FEEL_ (two separate pieces of emphasis) about a >>>> law that _obliges_ you to give UI. >> >> Perfectly comfortable. The giving of UI is a necessary evil. John >> > > I'm not talking about giving UI, I'm talking about being _obliged_ to > give UI. > Everything I do at the bridge table is an attempt to keep from giving UI > to my partner, from my poker face to my somewhat hasty bids. > And here is a law that would tell me that I am forced to give UI. It > goes against my nature. So I don't do it. > > Which leads me to my second question: You're the TD, and I have "broken" > this law telling me to give UI. I have not given UI. What adjustment do > you propose? I'll issue a PP. > > And there was still my third "quiz" question. How many laws in the > lawbook explicitely forbid a player to give a piece of UI to his > partner? And how many oblige him to do so? Not relevant, I'm afraid. We are told to give UI in the HdW school debate. > > Herman. (pot or kettle, I don't care: you're all black) > > > _______________________________________________ > blml mailing list > blml at amsterdamned.org > http://www.amsterdamned.org/mailman/listinfo/blml From john at asimere.com Thu Nov 13 14:21:56 2008 From: john at asimere.com (John (MadDog) Probst) Date: Thu, 13 Nov 2008 13:21:56 -0000 Subject: [blml] Amsterdam (2) [SEC=UNOFFICIAL] References: <491B6535.5020201@talktalk.net> <001a01c9452d$46ddddc0$0302a8c0@Mildred> Message-ID: ----- Original Message ----- From: "Grattan" To: "Bridge Laws Mailing List" Sent: Thursday, November 13, 2008 1:15 AM Subject: Re: [blml] Amsterdam (2) [SEC=UNOFFICIAL] > > > Grattan Endicott also ************************************ > "We desire truth and find within > ourselves only uncertainty." > [Pascal] > ''''''''''''''''''''''''''''''''''''''''''''''''''''''''''''''''''''''''''''''''''''''''''''''''' > > > > ----- Original Message ----- > From: "Nigel Guthrie" > To: "Bridge Laws Mailing List" > Sent: Wednesday, November 12, 2008 11:22 PM > Subject: Re: [blml] Amsterdam (2) [SEC=UNOFFICIAL] > > >> [ton] >> Imo you are contributing to this ill-coordination and futile > chatter. Somewhat below the level I expected from you. > Combining the sentences: 'Is the said Director in hiding or > something? Why are we not hearing from him as a key > person?' with: 'Not that I think there is any right to expect > that we would hear from him here' sounds like the bla bla > of a drunken sailsman. > Well, enjoy yourself >> >> [nige1] >> Grattan doesn't seem to be criticising, he appears to be > expressing our curiosity about the official version of events. > For example, it's hard to understand why the name of the > CTD is cloaked in secrecy :) >> > +=+ Nigel, the problem is that we have seen a number of > accounts of the matter, mostly rather opaque and certainly > demonstrating anomalies and divergences. As far as I can > tell the question whether the responders to the appeal were > fully and properly heard is a real one. Certainly I hear their > allegations that they were not and the anger, emotion, that > is voiced about it. Neither do we know what view was taken > of the lack of protest at the time of the alleged irregularity > and the desire latterly for a ruling (or rulings) when there was > opportunity to turn a narrow defeat into victory. > As has been observed, on blml we are not entitled to > demand to know, although there has been some defence > of what occurred. So the correspondence carries an odour > of justice not seen to be done, a lack of transparency as to > exactly how matters were handled, and what we do not know > about the Director in charge and his actions contributes to this. > One can see the scope for suspicion that the request for a > ruling may have been moved past the Director without waiting > for his appropriate action - and one can recognize the anxiety > some may feel as to the possibilities for challenging the validity > of the process if it were to appear flawed. We should not leap > to judgement, but the questions will linger. > ~ Grattan ~ +=+ > Beautifully put, Grattan. It is the apparent procedural flaws that concern me far more than the actual outcome. Bridge itself is brought into disrepute. I'm (peace Sancho et al) not concerned about angry Germans; I'm concerned about the thoughts of the pondlife. "When are WE going to suffer haphazard injustice." [The players all played at once without waiting for turns, quarrelling all the while, and fighting for the hedgehogs; and in a very short time the Queen was in a furious passion, and went stamping about, and shouting' Off with his head!' or 'Off with her head!' about once in a minute.]Alice in Wonderland; Dodgson. cheers John > > _______________________________________________ > blml mailing list > blml at amsterdamned.org > http://www.amsterdamned.org/mailman/listinfo/blml From schoderb at msn.com Thu Nov 13 14:24:39 2008 From: schoderb at msn.com (WILLIAM SCHODER) Date: Thu, 13 Nov 2008 08:24:39 -0500 Subject: [blml] Amsterdam (2) [SEC=UNOFFICIAL] In-Reply-To: <491B6535.5020201@talktalk.net> <491BEE9C.5060301@skynet.be><004501c9457a$1abb77f0$0302a8c0@Mildred> <491C18C0.7050008@skynet.be> <491C266D.7050806@skynet.be> References: <491B6535.5020201@talktalk.net> <491BEE9C.5060301@skynet.be><004501c9457a$1abb77f0$0302a8c0@Mildred> <491C18C0.7050008@skynet.be> <491C266D.7050806@skynet.be> Message-ID: No again. It is also NO to the idea of a coin toss. Come on Mr. De Wael, don't exhibit your childishness. Obviously you've read my postings on what I think should happen when players ask for a ruling. To glean from those that I would agree to a coin toss is just plain stupid. Kojak ----- Original Message ----- From: "Herman De Wael" To: "Bridge Laws Mailing List" Sent: Thursday, November 13, 2008 8:06 AM Subject: Re: [blml] Amsterdam (2) [SEC=UNOFFICIAL] > WILLIAM SCHODER wrote: > > Herman wrote: > > > > > > Surely we > >> don't want directors to give decisions that way - isn't it better then > >> to allow them to say that since the matter is already in the hands of > >> the AC, no TD decision is necessary? > >> > >> Herman. > >> > > No. Kojak > > > > You prefer the TD to give the decision by a toss of a coin then. > Well, I guess you're right at that. > > Herman. > > > _______________________________________________ > blml mailing list > blml at amsterdamned.org > http://www.amsterdamned.org/mailman/listinfo/blml > From john at asimere.com Thu Nov 13 14:39:25 2008 From: john at asimere.com (John (MadDog) Probst) Date: Thu, 13 Nov 2008 13:39:25 -0000 Subject: [blml] Amsterdam (2) [SEC=UNOFFICIAL] References: <491B6535.5020201@talktalk.net><491BEE9C.5060301@skynet.be><004501c9457a$1abb77f0$0302a8c0@Mildred><491C18C0.7050008@skynet.be> Message-ID: <257B77AED9494998B2B5321DA93AC207@JOHN> ----- Original Message ----- From: "WILLIAM SCHODER" To: "Bridge Laws Mailing List" Sent: Thursday, November 13, 2008 12:15 PM Subject: Re: [blml] Amsterdam (2) [SEC=UNOFFICIAL] > Herman wrote: > > > Surely we >> don't want directors to give decisions that way - isn't it better then to >> allow them to say that since the matter is already in the hands of the >> AC, no TD decision is necessary? >> >> Herman. >> > No. Kojak A competent TD gives a ruling. He must then advise both parties of their right to appeal. He may then advise either or both parties they should appeal. He must, if an appeal is made, advise the AC of the facts and WHY he made his judgement. Until then there is no AC at all and I doubt that its decision has any force in law, since all that an AC can do is to advise a TD to change his ruling. I buy into Kojaks's If xxx then this; If yyy then that. He has made his ruling and asked the AC to determine the facts, not asked them to make his ruling. . John > > _______________________________________________ > blml mailing list > blml at amsterdamned.org > http://www.amsterdamned.org/mailman/listinfo/blml From agot at ulb.ac.be Thu Nov 13 14:44:49 2008 From: agot at ulb.ac.be (Alain Gottcheiner) Date: Thu, 13 Nov 2008 14:44:49 +0100 Subject: [blml] A player's comment In-Reply-To: <491C27D9.4010501@skynet.be> References: <491C27D9.4010501@skynet.be> Message-ID: <491C2F51.9030904@ulb.ac.be> Herman De Wael a ?crit : > Heard yesterday from a (top-level) player in Belgium. > > She was playing in a club tournament, and the bidding went: > > 1NT - 2Di - pass - 2Sp > pass- 3NT - pass - 4Di > pass- 4Sp > (there may have been some doubles in there) > > 2Di was a multi, and the player had long hearts. It was not alerted. > 2Sp was obviously intended as natural, and the player had long spades. > In the Multi-system, 2Sp shows heart support. > 2Sp was alerted. > > Since 4Sp went lots down, there was no TD call any more. > But what she complained most about was the alert on 2Sp. > She considered this most unethical, on an equal footing with saying "2Di > was Multi". > AG : if you remember, this case is very similar to the one I told about two months ago. And with the same conclusion from the player. And I discussed with one of the best players in Belgium, and famed for his superior ethics. He, too, spontaneously used dWS (without knowing there was a HdW or that there was some online fighting about it) in a classical setting (the gadget this time was Checkback Stayman). Could there be that we Belgians are more sensible about this ? Perhaps an effect of our exposure to many systems and conventions ? Best regards Alain From nigelguthrie at talktalk.net Thu Nov 13 15:10:11 2008 From: nigelguthrie at talktalk.net (Nigel Guthrie) Date: Thu, 13 Nov 2008 14:10:11 +0000 Subject: [blml] Amsterdam (2) [SEC=UNOFFICIAL] In-Reply-To: <491C266D.7050806@skynet.be> References: <491B6535.5020201@talktalk.net> <491BEE9C.5060301@skynet.be><004501c9457a$1abb77f0$0302a8c0@Mildred> <491C18C0.7050008@skynet.be> <491C266D.7050806@skynet.be> Message-ID: <491C3543.3020100@talktalk.net> [Herman 1] Surely we don't want directors to give decisions that way - isn't it better then to allow them to say that since the matter is already in the hands of the AC, no TD decision is necessary? [Kojak] No. [Herman 2] You prefer the TD to give the decision by a toss of a coin then? [Nige1] Sad that the COC didn't allow for a radical 3rd option: the TD records the facts and his ruling after consultation with the protagionists and his colleagues. An incidental benefit of this novel procedure could be that justice be done and seen to be done :) From svenpran at online.no Thu Nov 13 16:17:30 2008 From: svenpran at online.no (Sven Pran) Date: Thu, 13 Nov 2008 16:17:30 +0100 Subject: [blml] A player's comment In-Reply-To: <491C2F51.9030904@ulb.ac.be> References: <491C27D9.4010501@skynet.be> <491C2F51.9030904@ulb.ac.be> Message-ID: <000001c945a2$f2461ef0$d6d25cd0$@no> On Behalf Of Alain Gottcheiner > Sent: 13. november 2008 14:45 > To: Bridge Laws Mailing List > Subject: Re: [blml] A player's comment > > Herman De Wael a ?crit : > > Heard yesterday from a (top-level) player in Belgium. > > > > She was playing in a club tournament, and the bidding went: > > > > 1NT - 2Di - pass - 2Sp > > pass- 3NT - pass - 4Di > > pass- 4Sp > > (there may have been some doubles in there) > > > > 2Di was a multi, and the player had long hearts. It was not alerted. > > 2Sp was obviously intended as natural, and the player had long spades. > > In the Multi-system, 2Sp shows heart support. > > 2Sp was alerted. > > > > Since 4Sp went lots down, there was no TD call any more. > > But what she complained most about was the alert on 2Sp. > > She considered this most unethical, on an equal footing with saying "2Di > > was Multi". > > > AG : if you remember, this case is very similar to the one I told about > two months ago. And with the same conclusion from the player. > > And I discussed with one of the best players in Belgium, and famed for > his superior ethics. He, too, spontaneously used dWS (without knowing > there was a HdW or that there was some online fighting about it) in a > classical setting (the gadget this time was Checkback Stayman). > > Could there be that we Belgians are more sensible about this ? Perhaps > an effect of our exposure to many systems and conventions ? Or could it be (as I have a feeling it is) that while the player who fails to alert 2D accidentally commits a misexplanation, the player that as a consequence of this does not alert 2S deliberately "fields the misexplanation"? What is the technical difference between "fielding a psyche" and "fielding a misexplanation"? Maybe the case is that the Belgians in case are unaware that instead of disclosing their understandings as they are required to do, they actually conceal their understanding all the way once a misexplanation has been committed? Regards Sven From agot at ulb.ac.be Thu Nov 13 16:48:58 2008 From: agot at ulb.ac.be (Alain Gottcheiner) Date: Thu, 13 Nov 2008 16:48:58 +0100 Subject: [blml] Amsterdam (2) [SEC=UNOFFICIAL] In-Reply-To: <257B77AED9494998B2B5321DA93AC207@JOHN> References: <491B6535.5020201@talktalk.net><491BEE9C.5060301@skynet.be><004501c9457a$1abb77f0$0302a8c0@Mildred><491C18C0.7050008@skynet.be> <257B77AED9494998B2B5321DA93AC207@JOHN> Message-ID: <491C4C6A.7030301@ulb.ac.be> John (MadDog) Probst a ?crit : > ----- Original Message ----- > From: "WILLIAM SCHODER" > To: "Bridge Laws Mailing List" > Sent: Thursday, November 13, 2008 12:15 PM > Subject: Re: [blml] Amsterdam (2) [SEC=UNOFFICIAL] > > > >> Herman wrote: >> >> >> Surely we >> >>> don't want directors to give decisions that way - isn't it better then to >>> allow them to say that since the matter is already in the hands of the >>> AC, no TD decision is necessary? >>> >>> Herman. >>> >>> >> No. Kojak >> > > AG : there are many things I don't understand with the procedure as stated. > A competent TD gives a ruling. He must then advise both parties of their > right to appeal. AG : if there is any. Decisions about facts aren't subject to appeal IIRC. And that's part of the problem in the Amsterdam case : the TD has to decide about facts, not the AC, and if we're correctly informed, the TD didn't. > He may then advise either or both parties they should > appeal. AG : may he tell one party that they'd most probably lose an appeal ? > He must, if an appeal is made, advise the AC of the facts and WHY he > made his judgement. Until then there is no AC at all and I doubt that its > decision has any force in law AG : hmm... Several Belgian tournament organizers specified there was no AC, and pretended TFL gave them that right. What does the TD do in that case ? > , since all that an AC can do is to advise a TD > to change his ruling. AG : then why do appeal books say "TD's ruling changed' , It should have been 'TD's ruling sent backl for reconsidering'. Best regards Alain From agot at ulb.ac.be Thu Nov 13 16:58:57 2008 From: agot at ulb.ac.be (Alain Gottcheiner) Date: Thu, 13 Nov 2008 16:58:57 +0100 Subject: [blml] A player's comment In-Reply-To: <000001c945a2$f2461ef0$d6d25cd0$@no> References: <491C27D9.4010501@skynet.be> <491C2F51.9030904@ulb.ac.be> <000001c945a2$f2461ef0$d6d25cd0$@no> Message-ID: <491C4EC1.9050301@ulb.ac.be> Sven Pran a ?crit : > Or could it be (as I have a feeling it is) that while the player who fails > to alert 2D accidentally commits a misexplanation, the player that as a > consequence of this does not alert 2S deliberately "fields the > misexplanation"? > > What is the technical difference between "fielding a psyche" and "fielding a > misexplanation"? > AG : the main difference is that the player doesn't allow one's bidding to be influenced by the fielding. i.e. you know there ihas been a mistake, you take this into account when explaining, but not when bidding. When you field a psyche, you don't take this into account when explaining, but you do when bidding. > Maybe the case is that the Belgians in case are unaware that instead of > disclosing their understandings as they are required to do, they actually > conceal their understanding all the way once a misexplanation has been > committed? > > AG : you misunderstood the situation. They're aware of that. And many of them think it's good for bridge, even if not lawful. If I had been introduced under a wrong name, and I felt it wouldn't be polite to correct it, I would answer to this name until the quiproquo is wiped. Now, deliberately assuming a false identity is a serious crime, but it might be good to your PR this time. Best regards Alain From Hermandw at skynet.be Thu Nov 13 17:46:20 2008 From: Hermandw at skynet.be (Herman De Wael) Date: Thu, 13 Nov 2008 17:46:20 +0100 Subject: [blml] Mr Chesterton suggested [SEC=UNOFFICIAL] In-Reply-To: <7A05D24604AF42DFAC4C29F8329E46C9@JOHN> References: <2CF77EB18250470CB513261A89089258@JOHN> <491B5AFD.8000604@skynet.be> <7A05D24604AF42DFAC4C29F8329E46C9@JOHN> Message-ID: <491C59DC.4000504@skynet.be> John (MadDog) Probst wrote: > ----- Original Message ----- > From: "Herman De Wael" >> >> Which leads me to my second question: You're the TD, and I have "broken" >> this law telling me to give UI. I have not given UI. What adjustment do >> you propose? > > I'll issue a PP. Which will see me better off than by giving UI. Which is of course not why I should do this. But my friend, who know nothing about DWS, and who simply tries to follow L20F5a, will get a PP for no reason that he can see. Or maybe you won't give him a PP, and only me? And anyway, my friend's opponents (or mine) will get a worse score than your opponents, who will have the benefit of knowing about the misunderstanding. Don't you understand that this "law" is meaningless unless you also make a correct penalty for it, and that there is no possibility for making a correct penalty? > >> And there was still my third "quiz" question. How many laws in the >> lawbook explicitely forbid a player to give a piece of UI to his >> partner? And how many oblige him to do so? > > Not relevant, I'm afraid. We are told to give UI in the HdW school debate. > Not relevant? That's another way of not answering. >> Herman. (pot or kettle, I don't care: you're all black) >> Herman. From Hermandw at skynet.be Thu Nov 13 17:52:39 2008 From: Hermandw at skynet.be (Herman De Wael) Date: Thu, 13 Nov 2008 17:52:39 +0100 Subject: [blml] Amsterdam (2) [SEC=UNOFFICIAL] In-Reply-To: References: <491B6535.5020201@talktalk.net> <491BEE9C.5060301@skynet.be><004501c9457a$1abb77f0$0302a8c0@Mildred> <491C18C0.7050008@skynet.be> <491C266D.7050806@skynet.be> Message-ID: <491C5B57.4020501@skynet.be> WILLIAM SCHODER wrote: > No again. It is also NO to the idea of a coin toss. > > Come on Mr. De Wael, don't exhibit your childishness. Obviously you've read > my postings on what I think should happen when players ask for a ruling. To > glean from those that I would agree to a coin toss is just plain stupid. > > Kojak > No Bill, I did not think you would agree - I said you would prefer? Because that is what I'll do if you insist on a ruling - I'll give a ruling. But I'm not going to spend ten minutes on my own in doing so, if there are four players waiting on my decision before going in to appeal that decision, no matter what it is. That is the situation you must compare this with (and I agree that in reality it did not go like this): - Four players, a director, and 3 appeal members are sitting in a room; - the facts are explained; - the AC Chairman says "TD, what is your decision" - both pairs say "we'll appeal that decision anyway" What is the TD to do? Go sit in a corner for 10 minutes? Or just give a ruling, no matter what? Isn't it better for all concerned if he doesn't give any ruling? Or shows that he doesn't by tossing a coin, if you insist he must? I am squarely behind the directors on this one, as a matter of common sense. And I agree that in actual fact the Germans may have been unaware that there would be just one session, not two. And that that would be wrong. >>> >> You prefer the TD to give the decision by a toss of a coin then. >> Well, I guess you're right at that. >> >> Herman. >> Herman. From Hermandw at skynet.be Thu Nov 13 17:59:13 2008 From: Hermandw at skynet.be (Herman De Wael) Date: Thu, 13 Nov 2008 17:59:13 +0100 Subject: [blml] A player's comment In-Reply-To: <000001c945a2$f2461ef0$d6d25cd0$@no> References: <491C27D9.4010501@skynet.be> <491C2F51.9030904@ulb.ac.be> <000001c945a2$f2461ef0$d6d25cd0$@no> Message-ID: <491C5CE1.4000107@skynet.be> Sven Pran wrote: > On Behalf Of Alain Gottcheiner >> >> Could there be that we Belgians are more sensible about this ? Perhaps >> an effect of our exposure to many systems and conventions ? > > Or could it be (as I have a feeling it is) that while the player who fails > to alert 2D accidentally commits a misexplanation, the player that as a > consequence of this does not alert 2S deliberately "fields the > misexplanation"? > Yes Sven, it could - but it isn't. Whatever else you may think about it, the facts are clear - West intended 2Di as Multi and has hearts. He cannot at the end of the story claim anything else. > What is the technical difference between "fielding a psyche" and "fielding a > misexplanation"? > > Maybe the case is that the Belgians in case are unaware that instead of > disclosing their understandings as they are required to do, they actually > conceal their understanding all the way once a misexplanation has been > committed? > They cannot conceal their understandings any more than any other players - their heart suit is still there to see. They will reveal the misinformation as soon as they need. The only thing they are concealing at the moment is their _mis_understanding. But that is something the opponents are not entitled to know! > Regards Sven > Sven, you have used this argument many times, and I have reacted to it every single time. I am going to stop doing so. Whenever Sven brings this same argument up again, I will remain quiet. That does not mean he is right about it - far from it. Herman. From matthias.schueller at gmx.de Thu Nov 13 18:19:51 2008 From: matthias.schueller at gmx.de (=?ISO-8859-1?Q?Matthias_Sch=FCller?=) Date: Thu, 13 Nov 2008 18:19:51 +0100 Subject: [blml] Amsterdam (2) [SEC=UNOFFICIAL] In-Reply-To: <491C5B57.4020501@skynet.be> References: <491B6535.5020201@talktalk.net> <491BEE9C.5060301@skynet.be><004501c9457a$1abb77f0$0302a8c0@Mildred> <491C18C0.7050008@skynet.be> <491C266D.7050806@skynet.be> <491C5B57.4020501@skynet.be> Message-ID: <491C61B7.8050907@gmx.de> Herman De Wael wrote: > WILLIAM SCHODER wrote: >> No again. It is also NO to the idea of a coin toss. >> >> Come on Mr. De Wael, don't exhibit your childishness. Obviously you've read >> my postings on what I think should happen when players ask for a ruling. To >> glean from those that I would agree to a coin toss is just plain stupid. >> >> Kojak >> > > No Bill, I did not think you would agree - I said you would prefer? > Because that is what I'll do if you insist on a ruling - I'll give a ruling. > But I'm not going to spend ten minutes on my own in doing so, if there > are four players waiting on my decision before going in to appeal that > decision, no matter what it is. > That is the situation you must compare this with (and I agree that in > reality it did not go like this): > - Four players, a director, and 3 appeal members are sitting in a room; > - the facts are explained; > - the AC Chairman says "TD, what is your decision" > - both pairs say "we'll appeal that decision anyway" > What is the TD to do? Go sit in a corner for 10 minutes? Or just give a > ruling, no matter what? Isn't it better for all concerned if he doesn't > give any ruling? Not at all. In fact, I am sure that it is better for all concerned if the TD *does* sit in a corner for some time (maybe even get up once or twice to consult) - and work on a ruling, just like he would in every case where judgement is involved. And this is not just a matter of "felt justice" for the players. It will also, on average, improve the AC's ruling. Yes, if ACs were absolutely perfect, then it would be a waste of 10 minutes (but so what? This is the semi finals of an EC, after all). But I always view the AC's ruling, at least partially, as a cooperative ruling between the TD and the AC. Why is that? Well, following your procedure, the AC would deliberate on a case from scratch. As soon as the TD has given a ruling (and I do not consider tossing a coin a ruling), the AC has his reasoning at its disposal and can work from there. Don't you think that this will vastly decrease the chance of the AC overlooking an aspect of the case or getting a point of law wrong? Matthias From nigelguthrie at talktalk.net Thu Nov 13 18:51:35 2008 From: nigelguthrie at talktalk.net (Nigel Guthrie) Date: Thu, 13 Nov 2008 17:51:35 +0000 Subject: [blml] Amsterdam (2) [SEC=UNOFFICIAL] In-Reply-To: <491C61B7.8050907@gmx.de> References: <491B6535.5020201@talktalk.net> <491BEE9C.5060301@skynet.be><004501c9457a$1abb77f0$0302a8c0@Mildred> <491C18C0.7050008@skynet.be> <491C266D.7050806@skynet.be> <491C5B57.4020501@skynet.be> <491C61B7.8050907@gmx.de> Message-ID: <491C6927.7090504@talktalk.net> [Matthias Sch?ller] Why is that? Well, following your procedure, the AC would deliberate on a case from scratch. As soon as the TD has given a ruling (and I do not consider tossing a coin a ruling), the AC has his reasoning at its disposal and can work from there. Don't you think that this will vastly decrease the chance of the AC overlooking an aspect of the case or getting a point of law wrong? [Nige1] I agree with Matthias, especially on his last point. Correct legal procedure avoids *illegal* decisions. The committee may be good at Bridge *judgment*, but the director is the expert on *law* and *regulation*. From wjburrows at gmail.com Thu Nov 13 19:34:46 2008 From: wjburrows at gmail.com (Wayne Burrows) Date: Fri, 14 Nov 2008 07:34:46 +1300 Subject: [blml] A player's comment In-Reply-To: <491C27D9.4010501@skynet.be> References: <491C27D9.4010501@skynet.be> Message-ID: <2a1c3a560811131034h6b48945bl9cc6d238b09159b9@mail.gmail.com> 2008/11/14 Herman De Wael : > Heard yesterday from a (top-level) player in Belgium. > > She was playing in a club tournament, and the bidding went: > > 1NT - 2Di - pass - 2Sp > pass- 3NT - pass - 4Di > pass- 4Sp > (there may have been some doubles in there) > > 2Di was a multi, and the player had long hearts. It was not alerted. > 2Sp was obviously intended as natural, and the player had long spades. > In the Multi-system, 2Sp shows heart support. > 2Sp was alerted. > > Since 4Sp went lots down, there was no TD call any more. > But what she complained most about was the alert on 2Sp. > She considered this most unethical, on an equal footing with saying "2Di > was Multi". > > I told her that I agreed with her, but that most of you would act the > same way. > I shall not repeat the words she used for all of you. > We cannot be blamed for her ignorance of the law. -- Wayne Burrows Palmerston North New Zealand From dalburn at btopenworld.com Thu Nov 13 19:53:53 2008 From: dalburn at btopenworld.com (David Burn) Date: Thu, 13 Nov 2008 18:53:53 -0000 Subject: [blml] A player's comment In-Reply-To: <2a1c3a560811131034h6b48945bl9cc6d238b09159b9@mail.gmail.com> References: <491C27D9.4010501@skynet.be> <2a1c3a560811131034h6b48945bl9cc6d238b09159b9@mail.gmail.com> Message-ID: <000601c945c1$2d3a2600$87ae7200$@com> [HdW] 1NT - 2Di - pass - 2Sp pass- 3NT - pass - 4Di pass- 4Sp 2Di was a multi, and the player had long hearts. It was not alerted. 2Sp was obviously intended as natural, and the player had long spades. In the Multi-system, 2Sp shows heart support. 2Sp was alerted. [DALB] What was the 2D bidder doing in this auction? 2S showed heart support, did it not? What were 3NT and 4S supposed to be? David Burn London, England From nigelguthrie at talktalk.net Thu Nov 13 20:09:42 2008 From: nigelguthrie at talktalk.net (Nigel Guthrie) Date: Thu, 13 Nov 2008 19:09:42 +0000 Subject: [blml] A player's comment In-Reply-To: <491C27D9.4010501@skynet.be> References: <491C27D9.4010501@skynet.be> Message-ID: <491C7B76.8040002@talktalk.net> [Herman De Wael] Heard yesterday from a (top-level) player in Belgium. She was playing in a club tournament, and the bidding went: 1NT - 2Di - pass - 2Sp pass- 3NT - pass - 4Di pass- 4Sp (there may have been some doubles in there) 2Di was a multi, and the player had long hearts. It was not alerted. 2Sp was obviously intended as natural, and the player had long spades. In the Multi-system, 2Sp shows heart support. 2Sp was alerted. Since 4Sp went lots down, there was no TD call any more. But what she complained most about was the alert on 2Sp. She considered this most unethical, on an equal footing with saying "2Di was Multi". I told her that I agreed with her, but that most of you would act the same way. I shall not repeat the words she used for all of you. [Nige1] IMO it is a pity that no conscientious international director assured her that - - the alert of 2S was correct (in common sense as wall as law) but the 3N bid may be suspect. - she should call the director to consider procedural penalties as well as further redress. From grandaeval at tiscali.co.uk Thu Nov 13 20:20:26 2008 From: grandaeval at tiscali.co.uk (Grattan) Date: Thu, 13 Nov 2008 19:20:26 -0000 Subject: [blml] Amsterdam (2) [SEC=UNOFFICIAL] References: <491B6535.5020201@talktalk.net><491BEE9C.5060301@skynet.be><004501c9457a$1abb77f0$0302a8c0@Mildred><491C18C0.7050008@skynet.be> <257B77AED9494998B2B5321DA93AC207@JOHN> Message-ID: <002401c945c4$e4c71a50$0302a8c0@Mildred> Grattan Endicott To: "Bridge Laws Mailing List" Sent: Thursday, November 13, 2008 1:39 PM Subject: Re: [blml] Amsterdam (2) [SEC=UNOFFICIAL] > A competent TD gives a ruling. He must then advise both parties of their right to appeal. He may then advise either or both parties they should appeal. > +=+ This observation by JP has put in my mind one of the subtler, and perhaps less obvious, changes that we made in the 2007 Laws. Comparison of the wording of Law 83 with the 1997 wording reveals that we have raised the height of the hurdle. The occasions that the laws call on the Director to advise a contestant of the right of appeal are fewer. It was our intention to relax the pressure on the Director to mention the subject. ~ G ~ +=+ From brian at meadows.pair.com Thu Nov 13 21:19:23 2008 From: brian at meadows.pair.com (brian) Date: Thu, 13 Nov 2008 15:19:23 -0500 Subject: [blml] Amsterdam (2) [SEC=UNOFFICIAL] In-Reply-To: <002401c945c4$e4c71a50$0302a8c0@Mildred> References: <491B6535.5020201@talktalk.net><491BEE9C.5060301@skynet.be><004501c9457a$1abb77f0$0302a8c0@Mildred><491C18C0.7050008@skynet.be> <257B77AED9494998B2B5321DA93AC207@JOHN> <002401c945c4$e4c71a50$0302a8c0@Mildred> Message-ID: <491C8BCB.5080507@meadows.pair.com> -----BEGIN PGP SIGNED MESSAGE----- Hash: SHA1 Grattan wrote: | | Grattan Endicott | To: "Bridge Laws Mailing List" | Sent: Thursday, November 13, 2008 1:39 PM | Subject: Re: [blml] Amsterdam (2) [SEC=UNOFFICIAL] | | | A competent TD gives a ruling. He must then advise both | parties of their right to appeal. He may then advise either | or both parties they should appeal. | +=+ This observation by JP has put in my mind one of the | subtler, and perhaps less obvious, changes that we made in | the 2007 Laws. Comparison of the wording of Law 83 with | the 1997 wording reveals that we have raised the height of | the hurdle. The occasions that the laws call on the Director | to advise a contestant of the right of appeal are fewer. It | was our intention to relax the pressure on the Director to | mention the subject. | ~ G ~ +=+ | Meaning that, in some circumstances, the experienced player will still know of his right to appeal, but the tournament newbie may not. Marvellous. Whatever would the game of bridge do without you people? Brian. (who is still boggling at the fact that the Chairman of the WBFLC has publicly stated that he is happy if the WBF CTD Emeritus - I think I have the titles right? - disagrees with his views. Really, folks, if two of the co-authors of TFLB regularly disagree, what chance has the poor old club TD got? IMHO, what this game is crying out for is a damn good clearout of the upper echelons.) -----BEGIN PGP SIGNATURE----- Version: GnuPG v1.4.7 (GNU/Linux) Comment: Using GnuPG with Mandriva - http://enigmail.mozdev.org iD8DBQFJHIvKgLujL0lynd0RAmxjAJwMaN1Ciwj8sBUe83U5Ji+prP8odwCfagbz R7LdgIEws/zqBuLQs5J1tyo= =wUUI -----END PGP SIGNATURE----- From rfrick at rfrick.info Thu Nov 13 21:38:43 2008 From: rfrick at rfrick.info (Robert Frick) Date: Thu, 13 Nov 2008 15:38:43 -0500 Subject: [blml] A player's comment In-Reply-To: <2a1c3a560811131034h6b48945bl9cc6d238b09159b9@mail.gmail.com> References: <491C27D9.4010501@skynet.be> <2a1c3a560811131034h6b48945bl9cc6d238b09159b9@mail.gmail.com> Message-ID: On Thu, 13 Nov 2008 13:34:46 -0500, Wayne Burrows wrote: > 2008/11/14 Herman De Wael : >> Heard yesterday from a (top-level) player in Belgium. >> >> She was playing in a club tournament, and the bidding went: >> >> 1NT - 2Di - pass - 2Sp >> pass- 3NT - pass - 4Di >> pass- 4Sp >> (there may have been some doubles in there) >> >> 2Di was a multi, and the player had long hearts. It was not alerted. >> 2Sp was obviously intended as natural, and the player had long spades. >> In the Multi-system, 2Sp shows heart support. >> 2Sp was alerted. >> >> Since 4Sp went lots down, there was no TD call any more. >> But what she complained most about was the alert on 2Sp. >> She considered this most unethical, on an equal footing with saying "2Di >> was Multi". >> >> I told her that I agreed with her, but that most of you would act the >> same way. >> I shall not repeat the words she used for all of you. >> > > We cannot be blamed for her ignorance of the law. > Actually, we do not know how "South must do nothing to correct the mistaken explanation while the auction continues" is translated into the Belgium lawbook. It may be that she knows the exact wording of the law and is just misinterpreting it. I believe many English speakers would have the same problem without guidance from above. From jfusselman at gmail.com Thu Nov 13 22:21:29 2008 From: jfusselman at gmail.com (Jerry Fusselman) Date: Thu, 13 Nov 2008 15:21:29 -0600 Subject: [blml] Amsterdam (2) [SEC=UNOFFICIAL] In-Reply-To: <491C8BCB.5080507@meadows.pair.com> References: <491B6535.5020201@talktalk.net> <491BEE9C.5060301@skynet.be> <004501c9457a$1abb77f0$0302a8c0@Mildred> <491C18C0.7050008@skynet.be> <257B77AED9494998B2B5321DA93AC207@JOHN> <002401c945c4$e4c71a50$0302a8c0@Mildred> <491C8BCB.5080507@meadows.pair.com> Message-ID: <2b1e598b0811131321q7c92d8dax71e10f88413ad626@mail.gmail.com> Brian wrote: > > Meaning that, in some circumstances, the experienced player will still > know of his right to appeal, but the tournament newbie may not. > Marvellous. Whatever would the game of bridge do without you people? My feeling is that this was taken out because it was hardly ever observed anyway, at least in the ACBL. Does any ACBL member recall being advised of his right to an appeal in as many as 1% of the cases? For me it was less than 1%, and I watched for it. > > > Brian. > > (who is still boggling at the fact that the Chairman of the WBFLC has > publicly stated that he is happy if the WBF CTD Emeritus - I think I > have the titles right? - disagrees with his views. Really, folks, if > two of the co-authors of TFLB regularly disagree, what chance has the > poor old club TD got? IMHO, what this game is crying out for is a damn > good clearout of the upper echelons.) > There is some selection bias here. WILLIAM and ton (though they disagree on the ideal number of capital letters to spell their own names) probably recommend the same procedure on the vast majority of issues that would come before a club TD, but choose not to waste time by saying so. Jerry Fusselman From brian at meadows.pair.com Thu Nov 13 22:44:42 2008 From: brian at meadows.pair.com (brian) Date: Thu, 13 Nov 2008 16:44:42 -0500 Subject: [blml] Amsterdam (2) [SEC=UNOFFICIAL] In-Reply-To: <2b1e598b0811131321q7c92d8dax71e10f88413ad626@mail.gmail.com> References: <491B6535.5020201@talktalk.net> <491BEE9C.5060301@skynet.be> <004501c9457a$1abb77f0$0302a8c0@Mildred> <491C18C0.7050008@skynet.be> <257B77AED9494998B2B5321DA93AC207@JOHN> <002401c945c4$e4c71a50$0302a8c0@Mildred> <491C8BCB.5080507@meadows.pair.com> <2b1e598b0811131321q7c92d8dax71e10f88413ad626@mail.gmail.com> Message-ID: <491C9FCA.7050205@meadows.pair.com> Jerry Fusselman wrote: > Brian wrote: >> >> (who is still boggling at the fact that the Chairman of the WBFLC has >> publicly stated that he is happy if the WBF CTD Emeritus - I think I >> have the titles right? - disagrees with his views. Really, folks, if >> two of the co-authors of TFLB regularly disagree, what chance has the >> poor old club TD got? IMHO, what this game is crying out for is a damn >> good clearout of the upper echelons.) >> > > There is some selection bias here. WILLIAM and ton (though they > disagree on the ideal number of capital letters to spell their own > names) probably recommend the same procedure on the vast majority of > issues that would come before a club TD, but choose not to waste time > by saying so. > I don't take kindly to being accused of "selection bias". Here is a cut and paste of the paragraph to which I was referring. In the past years Kojak has so often decided that I was wrong were it appeared that I wasn't, that I tend to be happy with such statement. Now, Jerry, please point out just exactly where I have misrepresented what was said, or what "selection bias" I exhibited in my quote. Just for once, CCed directly too, I want to make sure Jerry sees this post. I await his answer with interest. Brian. From jfusselman at gmail.com Fri Nov 14 00:02:05 2008 From: jfusselman at gmail.com (Jerry Fusselman) Date: Thu, 13 Nov 2008 17:02:05 -0600 Subject: [blml] Amsterdam (2) [SEC=UNOFFICIAL] In-Reply-To: <2b1e598b0811131452i49d59c3fl356dc894abcbd3e6@mail.gmail.com> References: <491B6535.5020201@talktalk.net> <004501c9457a$1abb77f0$0302a8c0@Mildred> <491C18C0.7050008@skynet.be> <257B77AED9494998B2B5321DA93AC207@JOHN> <002401c945c4$e4c71a50$0302a8c0@Mildred> <491C8BCB.5080507@meadows.pair.com> <2b1e598b0811131321q7c92d8dax71e10f88413ad626@mail.gmail.com> <491C9FCA.7050205@meadows.pair.com> <2b1e598b0811131452i49d59c3fl356dc894abcbd3e6@mail.gmail.com> Message-ID: <2b1e598b0811131502v758fe18bq426322d7548a4b90@mail.gmail.com> Brian wrote: > Jerry Fusselman wrote: >> >> There is some selection bias here. > > I don't take kindly to being accused of "selection bias". > I meant nothing nefarious on Brian's part. I was using selection bias in a technical statistical sense: We on BLML do not have a full sample of WILLIAM and ton's opinions on bridge-directing matters at our finger tips. That's the selection bias. Brian's summary of their statements this week on BLML seems an accurate one. But that sample is limited to BLML, where authorities are likely to disagree more often. Sometimes selection bias cannot be avoided, and reading statements on BLML is one of those cases. It seems obvious. So I could hardly have been criticising Brian for his method of collecting data, since it unavoidably involves selection bias. Brian's quotes were accurate and in context. But I do disagree with Brian's conclusion, apparently, that WILLIAM and ton "regularly disagree" on bridge-directing matters. Admittedly, I offered no proof. I merely guessed that they agree more often on issues they choose not to discuss on BLML. Jerry Fusselman From richard.hills at immi.gov.au Fri Nov 14 00:34:34 2008 From: richard.hills at immi.gov.au (richard.hills at immi.gov.au) Date: Fri, 14 Nov 2008 10:34:34 +1100 Subject: [blml] Mr Chesterton suggested [SEC=UNOFFICIAL] In-Reply-To: <491BEDE9.1080804@skynet.be> Message-ID: G.K. Chesterton, 25th February 1905: "Briefly, you can only find truth with logic if you have already found truth without it." Herman De Wael suggested: [words of wit and wisdom snipped] In a parallel thread, Grattan Endicott refuted: [snip] >Insofar as our knowledge goes I do have criticisms of the >apparent failure to comply with the Laws and of any >suggestion that any such failure was of little consequence. [snip] Best wishes Richard James Hills Recruitment Section, Level 3 Blue, workstation 15 (first on left) Department of Immigration and Citizenship Telephone: 02 6223 8453 Email: richard.hills at immi.gov.au -------------------------------------------------------------------- Important Notice: If you have received this email by mistake, please advise the sender and delete the message and attachments immediately. This email, including attachments, may contain confidential, sensitive, legally privileged and/or copyright information. Any review, retransmission, dissemination or other use of this information by persons or entities other than the intended recipient is prohibited. DIAC respects your privacy and has obligations under the Privacy Act 1988. The official departmental privacy policy can be viewed on the department's website at www.immi.gov.au. See: http://www.immi.gov.au/functional/privacy.htm --------------------------------------------------------------------- From richard.hills at immi.gov.au Fri Nov 14 00:58:28 2008 From: richard.hills at immi.gov.au (richard.hills at immi.gov.au) Date: Fri, 14 Nov 2008 10:58:28 +1100 Subject: [blml] Law 83 (was ...damn) [SEC=UNOFFICIAL] In-Reply-To: <491C8BCB.5080507@meadows.pair.com> Message-ID: Grattan: >>+=+ This observation by JP has put in my mind one of the >>subtler, and perhaps less obvious, changes that we made in >>the 2007 Laws. Comparison of the wording of Law 83 with >>the 1997 wording reveals that we have raised the height of >>the hurdle. The occasions that the laws call on the Director >>to advise a contestant of the right of appeal are fewer. It >>was our intention to relax the pressure on the Director to >>mention the subject. >> ~ G ~ +=+ Brian: >Meaning that, in some circumstances, the experienced player >will still know of his right to appeal, but the tournament >newbie may not. > >Marvellous. Whatever would the game of bridge do without you >people? 1997 Law 83 - Notification of the Right to Appeal: "If the Director believes that a review of his decision on a point of fact or exercise of his discretionary power **might** be in order **(as when he awards an adjusted score under Law 12)**, he shall advise a contestant of his right to appeal or may refer the matter to an appropriate committee." 1997 Law 83 - Notification of the Right to Appeal: "If the Director believes that a review of his decision on a point of fact or exercise of his discretionary power **could well** be in order, he shall advise a contestant of his right to appeal or may refer the matter to an appropriate committee." Richard Hills: Under the 1997 Law the TD had to advise a newbie of their right to appeal whenever the TD adjusted the score under Law 12. What often then happened was that the newbie did appeal, was fined 3 VPs for an appeal without merit, then complained to the TD, "Why did you tell me to appeal?" The new wording "could well" means that the Director need not suggest meritless appeals by newbies of open-and-shut rulings. (In major ABF events, such as the forthcoming Summer Festival of Bridge, this problem of meritless appeals by newbies is mitigated by having Appeals Advisors.) Best wishes Richard James Hills Recruitment Section, Level 3 Blue, workstation 15 (first on left) Department of Immigration and Citizenship Telephone: 02 6223 8453 Email: richard.hills at immi.gov.au -------------------------------------------------------------------- Important Notice: If you have received this email by mistake, please advise the sender and delete the message and attachments immediately. This email, including attachments, may contain confidential, sensitive, legally privileged and/or copyright information. Any review, retransmission, dissemination or other use of this information by persons or entities other than the intended recipient is prohibited. DIAC respects your privacy and has obligations under the Privacy Act 1988. The official departmental privacy policy can be viewed on the department's website at www.immi.gov.au. See: http://www.immi.gov.au/functional/privacy.htm --------------------------------------------------------------------- From schoderb at msn.com Fri Nov 14 01:49:51 2008 From: schoderb at msn.com (WILLIAM SCHODER) Date: Thu, 13 Nov 2008 19:49:51 -0500 Subject: [blml] Amsterdam (2) [SEC=UNOFFICIAL] In-Reply-To: <491B6535.5020201@talktalk.net><004501c9457a$1abb77f0$0302a8c0@Mildred> <491C18C0.7050008@skynet.be><257B77AED9494998B2B5321DA93AC207@JOHN><002401c945c4$e4c71a50$0302a8c0@Mildred><491C8BCB.5080507@meadows.pair.com><2b1e598b0811131321q7c92d8dax71e10f88413ad626@mail.gmail.com><491C9FCA.7050205@meadows.pair.com><2b1e598b0811131452i49d59c3fl356dc894abcbd3e6@mail.gmail.com> <2b1e598b0811131502v758fe18bq426322d7548a4b90@mail.gmail.com> References: <491B6535.5020201@talktalk.net><004501c9457a$1abb77f0$0302a8c0@Mildred> <491C18C0.7050008@skynet.be><257B77AED9494998B2B5321DA93AC207@JOHN><002401c945c4$e4c71a50$0302a8c0@Mildred><491C8BCB.5080507@meadows.pair.com><2b1e598b0811131321q7c92d8dax71e10f88413ad626@mail.gmail.com><491C9FCA.7050205@meadows.pair.com><2b1e598b0811131452i49d59c3fl356dc894abcbd3e6@mail.gmail.com> <2b1e598b0811131502v758fe18bq426322d7548a4b90@mail.gmail.com> Message-ID: My apology to those affronted by the fact the my postings are headed by capital letters. It's the way my computer decided to do it, and I never thought about it. So, to keep anyone wrongly believing that it was my spurious intent to use Capitals, I've become enough computer literate to change it to lower case (I hope) The stress, opinions, and desires to protect our game from the A.. oles who "know better" remain the same. Kojak ----- Original Message ----- From: "Jerry Fusselman" To: "Bridge Laws Mailing List" Sent: Thursday, November 13, 2008 6:02 PM Subject: [blml] Amsterdam (2) [SEC=UNOFFICIAL] > Brian wrote: > > Jerry Fusselman wrote: > >> > >> There is some selection bias here. > > > > I don't take kindly to being accused of "selection bias". > > > > I meant nothing nefarious on Brian's part. I was using selection bias > in a technical statistical sense: We on BLML do not have a full > sample of WILLIAM and ton's opinions on bridge-directing matters at > our finger tips. That's the selection bias. > > Brian's summary of their statements this week on BLML seems an > accurate one. But that sample is limited to BLML, where authorities > are likely to disagree more often. > > Sometimes selection bias cannot be avoided, and reading statements on > BLML is one of those cases. It seems obvious. So I could hardly have > been criticising Brian for his method of collecting data, since it > unavoidably involves selection bias. Brian's quotes were accurate and > in context. > > But I do disagree with Brian's conclusion, apparently, that WILLIAM > and ton "regularly disagree" on bridge-directing matters. Admittedly, > I offered no proof. I merely guessed that they agree more often on > issues they choose not to discuss on BLML. > > Jerry Fusselman > > _______________________________________________ > blml mailing list > blml at amsterdamned.org > http://www.amsterdamned.org/mailman/listinfo/blml > From grabiner at alumni.princeton.edu Fri Nov 14 03:58:49 2008 From: grabiner at alumni.princeton.edu (David Grabiner) Date: Thu, 13 Nov 2008 21:58:49 -0500 Subject: [blml] A player's comment In-Reply-To: <491C27D9.4010501@skynet.be> References: <491C27D9.4010501@skynet.be> Message-ID: How would she react if the bid was correct but the alert was missing, and necessary? P-P-1S-P 2C-P-2D!-X 2S-3D 2C was not alerted but was Drury, and opener knew that. 2D was alerted, showing a minimum, allowing the doubler to show diamond values, and allowing partner to compete to 3D. Was the alert on the 2D call, which allowed the opposition to bid properly, unethical? ----- Original Message ----- From: "Herman De Wael" To: "Bridge Laws Mailing List" Sent: Thursday, November 13, 2008 8:12 AM Subject: [blml] A player's comment > Heard yesterday from a (top-level) player in Belgium. > > She was playing in a club tournament, and the bidding went: > > 1NT - 2Di - pass - 2Sp > pass- 3NT - pass - 4Di > pass- 4Sp > (there may have been some doubles in there) > > 2Di was a multi, and the player had long hearts. It was not alerted. > 2Sp was obviously intended as natural, and the player had long spades. > In the Multi-system, 2Sp shows heart support. > 2Sp was alerted. > > Since 4Sp went lots down, there was no TD call any more. > But what she complained most about was the alert on 2Sp. > She considered this most unethical, on an equal footing with saying "2Di > was Multi". > > I told her that I agreed with her, but that most of you would act the > same way. > I shall not repeat the words she used for all of you. > > Herman. > > _______________________________________________ > blml mailing list > blml at amsterdamned.org > http://www.amsterdamned.org/mailman/listinfo/blml > > From ardelm at optusnet.com.au Fri Nov 14 05:01:40 2008 From: ardelm at optusnet.com.au (Tony Musgrove) Date: Fri, 14 Nov 2008 15:01:40 +1100 Subject: [blml] A player's comment In-Reply-To: <491C7B76.8040002@talktalk.net> References: <491C27D9.4010501@skynet.be> <491C7B76.8040002@talktalk.net> Message-ID: <6.1.0.6.2.20081114145712.01d51cd0@mail.optusnet.com.au> At 06:09 AM 14/11/2008, you wrote: >[Herman De Wael] >Heard yesterday from a (top-level) player in Belgium. She was playing in >a club tournament, and the bidding went: >1NT - 2Di - pass - 2Sp >pass- 3NT - pass - 4Di >pass- 4Sp (there may have been some doubles in there) >2Di was a multi, and the player had long hearts. It was not alerted. >2Sp was obviously intended as natural, and the player had long spades. >In the Multi-system, 2Sp shows heart support. >2Sp was alerted. > >Since 4Sp went lots down, there was no TD call any more. But what she >complained most about was the alert on 2Sp. She considered this most >unethical, on an equal footing with saying "2Di was Multi". I told her >that I agreed with her, but that most of you would act the same way. I >shall not repeat the words she used for all of you. > I am absolutely appalled. Don't try this at my clubs! Both of the culprits will have the book thrown at them for MI and UI regardless of the results. Had the 4S down 2 been a good result, perhaps even Herman would use the law book which we are forced to use. I apparently would have to apologise for taking a good result away "You should be playing in Belgium where you would be congratulated for attempting to improve the game" Cheers, Tony (Sydney) >[Nige1] >IMO it is a pity that no conscientious international director assured her >that - >- the alert of 2S was correct (in common sense as wall as law) but the 3N >bid may be suspect. >- she should call the director to consider procedural penalties as well as >further redress. > > > >_______________________________________________ >blml mailing list >blml at amsterdamned.org >http://www.amsterdamned.org/mailman/listinfo/blml From ardelm at optusnet.com.au Fri Nov 14 06:14:44 2008 From: ardelm at optusnet.com.au (Tony Musgrove) Date: Fri, 14 Nov 2008 16:14:44 +1100 Subject: [blml] Amsterdam (2) [SEC=UNOFFICIAL] In-Reply-To: <002401c945c4$e4c71a50$0302a8c0@Mildred> References: <491B6535.5020201@talktalk.net> <491BEE9C.5060301@skynet.be> <004501c9457a$1abb77f0$0302a8c0@Mildred> <491C18C0.7050008@skynet.be> <257B77AED9494998B2B5321DA93AC207@JOHN> <002401c945c4$e4c71a50$0302a8c0@Mildred> Message-ID: <6.1.0.6.2.20081114161335.01d51cd0@mail.optusnet.com.au> At 06:20 AM 14/11/2008, you wrote: >Grattan Endicottalso ************************************ >"We desire truth and find within >ourselves only uncertainty." > [Pascal] >''''''''''''''''''''''''''''''''''''''''''''''''''''''''''''''''''''''''''''''''''''''''''''''''' >----- Original Message ----- >From: "John (MadDog) Probst" >To: "Bridge Laws Mailing List" >Sent: Thursday, November 13, 2008 1:39 PM >Subject: Re: [blml] Amsterdam (2) [SEC=UNOFFICIAL] > > > > > A competent TD gives a ruling. He must then advise both >parties of their right to appeal. He may then advise either >or both parties they should appeal. > > >+=+ This observation by JP has put in my mind one of the >subtler, and perhaps less obvious, changes that we made in >the 2007 Laws. Comparison of the wording of Law 83 with >the 1997 wording reveals that we have raised the height of >the hurdle. The occasions that the laws call on the Director >to advise a contestant of the right of appeal are fewer. It >was our intention to relax the pressure on the Director to >mention the subject. > ~ G ~ +=+ > and perhaps to encourage directors not to toss a coin to make rulings? Cheers, Tony (Sydney) >_______________________________________________ >blml mailing list >blml at amsterdamned.org >http://www.amsterdamned.org/mailman/listinfo/blml From grandaeval at tiscali.co.uk Fri Nov 14 07:58:27 2008 From: grandaeval at tiscali.co.uk (Grattan) Date: Fri, 14 Nov 2008 06:58:27 -0000 Subject: [blml] Law 83 (was ...damn) [SEC=UNOFFICIAL] References: Message-ID: <002101c94626$e83cf3a0$0302a8c0@Mildred> Grattan Endicott To: "Bridge Laws Mailing List" Sent: Thursday, November 13, 2008 11:58 PM Subject: Re: [blml] Law 83 (was ...damn) [SEC=UNOFFICIAL] > > 1997 Law 83 - Notification of the Right to Appeal: > > "If the Director believes that a review of his decision on a > point of fact or exercise of his discretionary power **might** > be in order **(as when he awards an adjusted score under Law > 12)**, he shall advise a contestant of his right to appeal or > may refer the matter to an appropriate committee." > > 2007 Law 83 - Notification of the Right to Appeal: > > "If the Director believes that a review of his decision on a > point of fact or exercise of his discretionary power **could > well** be in order, he shall advise a contestant of his right > to appeal or may refer the matter to an appropriate committee." > > Richard Hills: > > Under the 1997 Law the TD had to advise a newbie of their right > to appeal whenever the TD adjusted the score under Law 12. > > What often then happened was that the newbie did appeal, was > fined 3 VPs for an appeal without merit, then complained to the > TD, "Why did you tell me to appeal?" > > The new wording "could well" means that the Director need not > suggest meritless appeals by newbies of open-and-shut rulings. > > (In major ABF events, such as the forthcoming Summer Festival > of Bridge, this problem of meritless appeals by newbies is > mitigated by having Appeals Advisors.) > +=+ Richard has more or less the flavour of it. Add to this that players at all levels are pretty well aware that they have the right of appeal, so the need to be reminded of it is reduced. ~ G ~ +=+ From grandaeval at tiscali.co.uk Fri Nov 14 08:01:29 2008 From: grandaeval at tiscali.co.uk (Grattan) Date: Fri, 14 Nov 2008 07:01:29 -0000 Subject: [blml] Amsterdam (2) [SEC=UNOFFICIAL] References: <491B6535.5020201@talktalk.net> <491BEE9C.5060301@skynet.be><004501c9457a$1abb77f0$0302a8c0@Mildred><491C18C0.7050008@skynet.be><257B77AED9494998B2B5321DA93AC207@JOHN><002401c945c4$e4c71a50$0302a8c0@Mildred> <6.1.0.6.2.20081114161335.01d51cd0@mail.optusnet.com.au> Message-ID: <002201c94626$e87fa1f0$0302a8c0@Mildred> Grattan Endicott To: "Bridge Laws Mailing List" Sent: Friday, November 14, 2008 5:14 AM Subject: Re: [blml] Amsterdam (2) [SEC=UNOFFICIAL] > At 06:20 AM 14/11/2008, you wrote: > > >>Grattan Endicott>also >************************************ >>"We desire truth and find within >>ourselves only uncertainty." >> [Pascal] >>''''''''''''''''''''''''''''''''''''''''''''''''''''''''''''''''''''''''''''''''''''''''''''''''' >>----- Original Message ----- >>From: "John (MadDog) Probst" >>To: "Bridge Laws Mailing List" >>Sent: Thursday, November 13, 2008 1:39 PM >>Subject: Re: [blml] Amsterdam (2) [SEC=UNOFFICIAL] >> >> >> > >> A competent TD gives a ruling. He must then advise both >>parties of their right to appeal. He may then advise either >>or both parties they should appeal. >> > >>+=+ This observation by JP has put in my mind one of the >>subtler, and perhaps less obvious, changes that we made in >>the 2007 Laws. Comparison of the wording of Law 83 with >>the 1997 wording reveals that we have raised the height of >>the hurdle. The occasions that the laws call on the Director >>to advise a contestant of the right of appeal are fewer. It >>was our intention to relax the pressure on the Director to >>mention the subject. >> ~ G ~ +=+ >> > and perhaps to encourage directors not to toss a > coin to make rulings? > > Cheers, > > Tony (Sydney) > +=+ When about to toss a coin it is important to consult with others as to the way in which the toss shall be done. +=+ From Hermandw at skynet.be Fri Nov 14 08:41:52 2008 From: Hermandw at skynet.be (Herman De Wael) Date: Fri, 14 Nov 2008 08:41:52 +0100 Subject: [blml] A player's comment In-Reply-To: <000601c945c1$2d3a2600$87ae7200$@com> References: <491C27D9.4010501@skynet.be> <2a1c3a560811131034h6b48945bl9cc6d238b09159b9@mail.gmail.com> <000601c945c1$2d3a2600$87ae7200$@com> Message-ID: <491D2BC0.5000306@skynet.be> David Burn wrote: > [HdW] > > 1NT - 2Di - pass - 2Sp > pass- 3NT - pass - 4Di > pass- 4Sp > > 2Di was a multi, and the player had long hearts. It was not alerted. 2Sp was > obviously intended as natural, and the player had long spades. In the > Multi-system, 2Sp shows heart support. 2Sp was alerted. > > [DALB] > > What was the 2D bidder doing in this auction? 2S showed heart support, did > it not? What were 3NT and 4S supposed to be? > Use of UI, of course! > David Burn > London, England > Herman. From Hermandw at skynet.be Fri Nov 14 08:46:01 2008 From: Hermandw at skynet.be (Herman De Wael) Date: Fri, 14 Nov 2008 08:46:01 +0100 Subject: [blml] A player's comment In-Reply-To: References: <491C27D9.4010501@skynet.be> <2a1c3a560811131034h6b48945bl9cc6d238b09159b9@mail.gmail.com> Message-ID: <491D2CB9.70504@skynet.be> Robert Frick wrote: >>> I shall not repeat the words she used for all of you. >>> >> We cannot be blamed for her ignorance of the law. >> > > Actually, we do not know how "South must do nothing to correct the > mistaken explanation while the auction continues" is translated into the > Belgium lawbook. It may be that she knows the exact wording of the law and > is just misinterpreting it. I believe many English speakers would have the > same problem without guidance from above. > > And that is precisely the point. IMO L20F5a is very clear. Many players agree that it is clear. Only you lot think that it is clear in the other direction. Now the WBFLC have issued a statement saying that you lot are right, but does that make it to the players? No it does not. And do you think it is good for the game of bridge if a very clear and important law is turned around simply by some further actions at the table? We really should get our priorities straight, and correct information (alerting 2Sp to say that it shows hearts when the whole table knows that it doesn't) should be a far less priority than not revealing to your partner that he got your previous bid wrong (even if he's too dumb to realize that's what your alert meant). Herman. From Hermandw at skynet.be Fri Nov 14 08:48:06 2008 From: Hermandw at skynet.be (Herman De Wael) Date: Fri, 14 Nov 2008 08:48:06 +0100 Subject: [blml] A player's comment In-Reply-To: <491C7B76.8040002@talktalk.net> References: <491C27D9.4010501@skynet.be> <491C7B76.8040002@talktalk.net> Message-ID: <491D2D36.9040108@skynet.be> Nigel Guthrie wrote: > > [Nige1] > IMO it is a pity that no conscientious international director assured her that - > - the alert of 2S was correct (in common sense as wall as law) but the 3N bid may be suspect. Maybe in law, certainly not in common sense. > - she should call the director to consider procedural penalties as well as further redress. > She knows quite well that 3NT and 4Sp are gross violations, and decided not to do anything about it. This was a club tournament in a club of which she is a member only in order to play in their team in the highest division - she does not wish to make waves. Besides, the culprit was a Director - I'll have a word with him at some stage. Herman. From harald.skjaran at gmail.com Fri Nov 14 08:49:00 2008 From: harald.skjaran at gmail.com (=?UTF-8?Q?Harald_Skj=C3=A6ran?=) Date: Fri, 14 Nov 2008 08:49:00 +0100 Subject: [blml] A player's comment In-Reply-To: <491C27D9.4010501@skynet.be> References: <491C27D9.4010501@skynet.be> Message-ID: On 13/11/2008, Herman De Wael wrote: > Heard yesterday from a (top-level) player in Belgium. > > She was playing in a club tournament, and the bidding went: > > 1NT - 2Di - pass - 2Sp > pass- 3NT - pass - 4Di > pass- 4Sp > (there may have been some doubles in there) > > 2Di was a multi, and the player had long hearts. It was not alerted. > 2Sp was obviously intended as natural, and the player had long spades. > In the Multi-system, 2Sp shows heart support. > 2Sp was alerted. This is one of the most obvious cases of misuse of UI I've ever seen. Where did the 3NT and 4S bids come from? Facing a partner who has, by agreement, shown good support for overcallers hearts and invited game. If overcaller had the values to bid game, surely 4H is the correct bid. I guess that would have been a better contract for the offending side here, and that's why there was no adjustment. But I'd nevertheless impose a heavy PP for this behaviour. (And of course explain why.) -- Kind regards, Harald Skj?ran > > Since 4Sp went lots down, there was no TD call any more. > But what she complained most about was the alert on 2Sp. > She considered this most unethical, on an equal footing with saying "2Di > was Multi". > > I told her that I agreed with her, but that most of you would act the > same way. > I shall not repeat the words she used for all of you. > > Herman. > > _______________________________________________ > blml mailing list > blml at amsterdamned.org > http://www.amsterdamned.org/mailman/listinfo/blml > From Hermandw at skynet.be Fri Nov 14 08:50:01 2008 From: Hermandw at skynet.be (Herman De Wael) Date: Fri, 14 Nov 2008 08:50:01 +0100 Subject: [blml] A player's comment In-Reply-To: <6.1.0.6.2.20081114145712.01d51cd0@mail.optusnet.com.au> References: <491C27D9.4010501@skynet.be> <491C7B76.8040002@talktalk.net> <6.1.0.6.2.20081114145712.01d51cd0@mail.optusnet.com.au> Message-ID: <491D2DA9.70606@skynet.be> Tony Musgrove wrote: >> > > I am absolutely appalled. Don't try this at my clubs! Both of the culprits > will have the book thrown at them for MI and UI regardless of the results. > Had the 4S down 2 been a good result, perhaps even Herman would use > the law book which we are forced to use. I apparently would have to > apologise for taking a good result away "You should be playing in > Belgium where you would be congratulated for attempting to improve > the game" > Please don't insult me by suggesting that the 3NT and 4Sp bids are anything but reprehensible. My point with this story is just that a top class player believes alerting 2Sp is reprehensible. Which it isn't, I agree, but it should be. And it is important to see that this is what players feel should be the correct way to play bridge. Herman. From Hermandw at skynet.be Fri Nov 14 08:58:30 2008 From: Hermandw at skynet.be (Herman De Wael) Date: Fri, 14 Nov 2008 08:58:30 +0100 Subject: [blml] A player's comment In-Reply-To: References: <491C27D9.4010501@skynet.be> Message-ID: <491D2FA6.6030401@skynet.be> David Grabiner wrote: > How would she react if the bid was correct but the alert was missing, and > necessary? > > P-P-1S-P > 2C-P-2D!-X > 2S-3D > > 2C was not alerted but was Drury, and opener knew that. How do we know if he didn't alert it? It's all nice to say that maybe the alert was forgotten, rather than the system, but that just postpones the problem. Let's say you see partner not alert something he should. Next you don't alert his answer. If he notices that you don't alert, he can still say "are you not alerting because I didn't? Sorry, I should alert this", and then you alert as well. No problem. Because your non-alert would not wake him if he had in fact forgotten the system, there is no UI. If he did remember the system, and forgot to alert, your non-alert only reminds him of his obligation to alert, not of the real system. So this "maybe he simply forgot to alert" business is nonsense. After all, when your partner does not alert something, you usually know why it is. Some parts of the system are so well known that you know it's the alert that's forgotten, and some parts are so often forgotten that you know that was the reason for the non-alert. > 2D was alerted, showing > a minimum, allowing the doubler to show diamond values, and allowing partner to > compete to 3D. Was the alert on the 2D call, which allowed the opposition to > bid properly, unethical? > Yes, it is unethical in the sense that it reminded partner that he had forgotten to alert (if such was the case). Don't worry about opponents, they'll get redress if they are misinformed. And of course, if the player suspects that his partner knows that 2C is Drury, and that he simply forgot to alert, then he is allowed to alert 2Di. It is not a crime to follow MS (and I have never said it was), but one has to be allowed to follow DWS too, if one chooses. Herman. From harald.skjaran at gmail.com Fri Nov 14 09:18:15 2008 From: harald.skjaran at gmail.com (=?UTF-8?Q?Harald_Skj=C3=A6ran?=) Date: Fri, 14 Nov 2008 09:18:15 +0100 Subject: [blml] Amsterdam (2) [SEC=UNOFFICIAL] In-Reply-To: <2b1e598b0811131321q7c92d8dax71e10f88413ad626@mail.gmail.com> References: <491B6535.5020201@talktalk.net> <491BEE9C.5060301@skynet.be> <004501c9457a$1abb77f0$0302a8c0@Mildred> <491C18C0.7050008@skynet.be> <257B77AED9494998B2B5321DA93AC207@JOHN> <002401c945c4$e4c71a50$0302a8c0@Mildred> <491C8BCB.5080507@meadows.pair.com> <2b1e598b0811131321q7c92d8dax71e10f88413ad626@mail.gmail.com> Message-ID: On 13/11/2008, Jerry Fusselman wrote: > Brian wrote: > > > > Meaning that, in some circumstances, the experienced player will still > > know of his right to appeal, but the tournament newbie may not. > > Marvellous. Whatever would the game of bridge do without you people? > > My feeling is that this was taken out because it was hardly ever > observed anyway, at least in the ACBL. Does any ACBL member recall > being advised of his right to an appeal in as many as 1% of the cases? > For me it was less than 1%, and I watched for it. Huh, what has the ACBL got to do with this? Who cares about the practise among ACBL TDs, the laws are there for the whold bridgeplaying world. (I'm speaking principle here, I'm not trying to insult our american friends.) I, for one, routinely inform the players about their right to appeal. I might abolish this among players I know well, and who I'll be certain know about their rights. I guess I mention the right to appeal 75+% of the times I give an appealable decision. The change in the law actually fits very well with how I've been practising for years. :-) -- Kind regards, Harald Skj?ran > > > > > > > Brian. > > > > (who is still boggling at the fact that the Chairman of the WBFLC has > > publicly stated that he is happy if the WBF CTD Emeritus - I think I > > have the titles right? - disagrees with his views. Really, folks, if > > two of the co-authors of TFLB regularly disagree, what chance has the > > poor old club TD got? IMHO, what this game is crying out for is a damn > > good clearout of the upper echelons.) > > > > There is some selection bias here. WILLIAM and ton (though they > disagree on the ideal number of capital letters to spell their own > names) probably recommend the same procedure on the vast majority of > issues that would come before a club TD, but choose not to waste time > by saying so. > > Jerry Fusselman > > _______________________________________________ > blml mailing list > blml at amsterdamned.org > http://www.amsterdamned.org/mailman/listinfo/blml > From harald.skjaran at gmail.com Fri Nov 14 09:20:49 2008 From: harald.skjaran at gmail.com (=?UTF-8?Q?Harald_Skj=C3=A6ran?=) Date: Fri, 14 Nov 2008 09:20:49 +0100 Subject: [blml] Amsterdam (2) [SEC=UNOFFICIAL] In-Reply-To: <002201c94626$e87fa1f0$0302a8c0@Mildred> References: <491B6535.5020201@talktalk.net> <491BEE9C.5060301@skynet.be> <004501c9457a$1abb77f0$0302a8c0@Mildred> <491C18C0.7050008@skynet.be> <257B77AED9494998B2B5321DA93AC207@JOHN> <002401c945c4$e4c71a50$0302a8c0@Mildred> <6.1.0.6.2.20081114161335.01d51cd0@mail.optusnet.com.au> <002201c94626$e87fa1f0$0302a8c0@Mildred> Message-ID: On 14/11/2008, Grattan wrote: > > > Grattan Endicott also ************************************ > "We desire truth and find within > ourselves only uncertainty." > [Pascal] > ''''''''''''''''''''''''''''''''''''''''''''''''''''''''''''''''''''''''''''''''''''''''''''''''' > ----- Original Message ----- > From: "Tony Musgrove" > To: "Bridge Laws Mailing List" > Sent: Friday, November 14, 2008 5:14 AM > Subject: Re: [blml] Amsterdam (2) [SEC=UNOFFICIAL] > > > > At 06:20 AM 14/11/2008, you wrote: > > > > > >>Grattan Endicott >>also >>************************************ > >>"We desire truth and find within > >>ourselves only uncertainty." > >> [Pascal] > >>''''''''''''''''''''''''''''''''''''''''''''''''''''''''''''''''''''''''''''''''''''''''''''''''' > >>----- Original Message ----- > >>From: "John (MadDog) Probst" > >>To: "Bridge Laws Mailing List" > >>Sent: Thursday, November 13, 2008 1:39 PM > >>Subject: Re: [blml] Amsterdam (2) [SEC=UNOFFICIAL] > >> > >> > >> > > >> A competent TD gives a ruling. He must then advise both > >>parties of their right to appeal. He may then advise either > >>or both parties they should appeal. > >> > > >>+=+ This observation by JP has put in my mind one of the > >>subtler, and perhaps less obvious, changes that we made in > >>the 2007 Laws. Comparison of the wording of Law 83 with > >>the 1997 wording reveals that we have raised the height of > >>the hurdle. The occasions that the laws call on the Director > >>to advise a contestant of the right of appeal are fewer. It > >>was our intention to relax the pressure on the Director to > >>mention the subject. > >> ~ G ~ +=+ > >> > > and perhaps to encourage directors not to toss a > > coin to make rulings? > > > > Cheers, > > > > Tony (Sydney) > > > +=+ When about to toss a coin it is important to consult with > others as to the way in which the toss shall be done. +=+ > Nice one, Grattan! -- Kind regards, Harald Skj?ran From ardelm at optusnet.com.au Fri Nov 14 10:40:49 2008 From: ardelm at optusnet.com.au (Tony Musgrove) Date: Fri, 14 Nov 2008 20:40:49 +1100 Subject: [blml] A player's comment In-Reply-To: <491D2DA9.70606@skynet.be> References: <491C27D9.4010501@skynet.be> <491C7B76.8040002@talktalk.net> <6.1.0.6.2.20081114145712.01d51cd0@mail.optusnet.com.au> <491D2DA9.70606@skynet.be> Message-ID: <6.1.0.6.2.20081114203527.01d51e18@mail.optusnet.com.au> At 06:50 PM 14/11/2008, you wrote: >Tony Musgrove wrote: > >> > > > > I am absolutely appalled. Don't try this at my clubs! Both of the > culprits > > will have the book thrown at them for MI and UI regardless of the results. > > Had the 4S down 2 been a good result, perhaps even Herman would use > > the law book which we are forced to use. I apparently would have to > > apologise for taking a good result away "You should be playing in > > Belgium where you would be congratulated for attempting to improve > > the game" > > >Please don't insult me by suggesting that the 3NT and 4Sp bids are >anything but reprehensible. > >My point with this story is just that a top class player believes >alerting 2Sp is reprehensible. Which it isn't, I agree, but it should >be. And it is important to see that this is what players feel should be >the correct way to play bridge. > >Herman. Perhaps the nationality of the "top class player" should not be an issue. It is hard not to draw the conclusion that Belgian players may not have access to the correct interpretation of the Law Book, Cheers, Tony (Sydney) From grandaeval at tiscali.co.uk Fri Nov 14 10:52:26 2008 From: grandaeval at tiscali.co.uk (Grattan) Date: Fri, 14 Nov 2008 09:52:26 -0000 Subject: [blml] Amsterdam (2) [SEC=UNOFFICIAL] References: <491B6535.5020201@talktalk.net><004501c9457a$1abb77f0$0302a8c0@Mildred><491C18C0.7050008@skynet.be><257B77AED9494998B2B5321DA93AC207@JOHN><002401c945c4$e4c71a50$0302a8c0@Mildred><491C8BCB.5080507@meadows.pair.com><2b1e598b0811131321q7c92d8dax71e10f88413ad626@mail.gmail.com><491C9FCA.7050205@meadows.pair.com><2b1e598b0811131452i49d59c3fl356dc894abcbd3e6@mail.gmail.com><2b1e598b0811131502v758fe18bq426322d7548a4b90@mail.gmail.com> Message-ID: <000c01c9463e$b67a39f0$0302a8c0@Mildred> Grattan Endicott To: "Bridge Laws Mailing List" Sent: Friday, November 14, 2008 12:49 AM Subject: Re: [blml] Amsterdam (2) [SEC=UNOFFICIAL] I've become enough computer literate to change it to lower case (I hope) The stress, opinions, and desires to protect our game from the A.. oles who "know better" remain the same. > Kojak > +=+ A near miss, perhaps? +=+ From agot at ulb.ac.be Fri Nov 14 11:02:39 2008 From: agot at ulb.ac.be (Alain Gottcheiner) Date: Fri, 14 Nov 2008 11:02:39 +0100 Subject: [blml] A player's comment In-Reply-To: <2a1c3a560811131034h6b48945bl9cc6d238b09159b9@mail.gmail.com> References: <491C27D9.4010501@skynet.be> <2a1c3a560811131034h6b48945bl9cc6d238b09159b9@mail.gmail.com> Message-ID: <491D4CBF.70807@ulb.ac.be> Wayne Burrows a ?crit : >> >> I told her that I agreed with her, but that most of you would act the >> same way. >> I shall not repeat the words she used for all of you. >> >> > > We cannot be blamed for her ignorance of the law. > > I think you were blamed for agreeing with a law she finds very bad. From agot at ulb.ac.be Fri Nov 14 11:06:27 2008 From: agot at ulb.ac.be (Alain Gottcheiner) Date: Fri, 14 Nov 2008 11:06:27 +0100 Subject: [blml] A player's comment In-Reply-To: <491C7B76.8040002@talktalk.net> References: <491C27D9.4010501@skynet.be> <491C7B76.8040002@talktalk.net> Message-ID: <491D4DA3.6080501@ulb.ac.be> Nigel Guthrie a ?crit : > [Herman De Wael] > Heard yesterday from a (top-level) player in Belgium. She was playing in > a club tournament, and the bidding went: > 1NT - 2Di - pass - 2Sp > pass- 3NT - pass - 4Di > pass- 4Sp (there may have been some doubles in there) > 2Di was a multi, and the player had long hearts. It was not alerted. > 2Sp was obviously intended as natural, and the player had long spades. > In the Multi-system, 2Sp shows heart support. > 2Sp was alerted. > > Since 4Sp went lots down, there was no TD call any more. But what she complained most about was the alert on 2Sp. She considered this most unethical, on an equal footing with saying "2Di was Multi". I told her that I agreed with her, but that most of you would act the same way. I shall not repeat the words she used for all of you. > > [Nige1] > IMO it is a pity that no conscientious international director assured her that - > - the alert of 2S was correct (in common sense as wall as law) but the 3N bid may be suspect. > AG : the 3NT bid is indeed very suspect. However, I've seen it made by a Belgian international, who happened to hold six solid spades and outside values. Upon hearing he was facing a misfit in spades and a probable cover of his weak spot in hearts, he punted 3NT, with success. So, the bid is suspect indeed, but it might be legitimate, and I'd rather see her hand before deciding. Best regards Alain From agot at ulb.ac.be Fri Nov 14 11:15:45 2008 From: agot at ulb.ac.be (Alain Gottcheiner) Date: Fri, 14 Nov 2008 11:15:45 +0100 Subject: [blml] A player's comment In-Reply-To: References: <491C27D9.4010501@skynet.be> Message-ID: <491D4FD1.7040104@ulb.ac.be> David Grabiner a ?crit : > How would she react if the bid was correct but the alert was missing, and > necessary? > > P-P-1S-P > 2C-P-2D!-X > 2S-3D > > 2C was not alerted but was Drury, and opener knew that. 2D was alerted, showing > a minimum, allowing the doubler to show diamond values, and allowing partner to > compete to 3D. Was the alert on the 2D call, which allowed the opposition to > bid properly, unethical? > AG : you're right. Not alerting means you're sure there was a problem with the system, and you want to keep him unknowing this. If it is more probable that partner simply omitted the alert (or thought the bid was non-alertable), rather than one of you forgetting the system, then dWS isn't for you. Sometimes, his bid will let you guess. If you play checkback Stayman (or you think you do), and he passes your 2C rebid, then you know there was a miscomprehension (of any sort), so using dWS would probably be successful, if only it were legal. If he answers 2NT, then the right explanation is that he forgot to alert and you'd rather alert his 2NT. The case you mention is compatible with both, whence you can't detect what happened. Best regards Alain From grandaeval at tiscali.co.uk Fri Nov 14 11:21:13 2008 From: grandaeval at tiscali.co.uk (Grattan) Date: Fri, 14 Nov 2008 10:21:13 -0000 Subject: [blml] Mr Chesterton suggested [SEC=UNOFFICIAL] References: <2CF77EB18250470CB513261A89089258@JOHN> <491B5AFD.8000604@skynet.be><7A05D24604AF42DFAC4C29F8329E46C9@JOHN> <491C59DC.4000504@skynet.be> Message-ID: <002301c94642$bcebb6c0$0302a8c0@Mildred> Grattan Endicott To: "Bridge Laws Mailing List" Sent: Thursday, November 13, 2008 4:46 PM Subject: Re: [blml] Mr Chesterton suggested [SEC=UNOFFICIAL] > John (MadDog) Probst wrote: >> I'll issue a PP. > <(Herman) > Which will see me better off than by giving UI. > +=+ That seems provocative. However, if the offence were aggravated - by repetition, perhaps, or because there is a knowing violation of Law 72B1, the PP could be, say, 50% or 100% of a top. Do we think John might be persuaded that Herman is aware of 72B1? ~ G ~ +=+ From Harsanyi at t-online.de Fri Nov 14 11:38:11 2008 From: Harsanyi at t-online.de (Harsanyi at t-online.de) Date: Fri, 14 Nov 2008 11:38:11 +0100 Subject: [blml] =?iso-8859-15?q?Amsterdam_=282=29_=5BSEC=3DUNOFFICIAL=5D?= In-Reply-To: <491C251C.2070807@bridgescore.de> References: <001101c944e5$84fea410$0302a8c0@Mildred> <002e01c94561$661116b0$32334410$@nl> <003501c94574$fb66f000$0302a8c0@Mildred> <491C197F.9090106@skynet.be> <491C251C.2070807@bridgescore.de> Message-ID: <1L0w43-0Ob3Eu0@fwd07.aul.t-online.de> An HTML attachment was scrubbed... URL: http://www.amsterdamned.org/pipermail/blml/attachments/20081114/53795ea5/attachment-0001.htm From agot at ulb.ac.be Fri Nov 14 11:42:22 2008 From: agot at ulb.ac.be (Alain Gottcheiner) Date: Fri, 14 Nov 2008 11:42:22 +0100 Subject: [blml] Mr Chesterton suggested [SEC=UNOFFICIAL] In-Reply-To: <002301c94642$bcebb6c0$0302a8c0@Mildred> References: <2CF77EB18250470CB513261A89089258@JOHN> <491B5AFD.8000604@skynet.be><7A05D24604AF42DFAC4C29F8329E46C9@JOHN> <491C59DC.4000504@skynet.be> <002301c94642$bcebb6c0$0302a8c0@Mildred> Message-ID: <491D560E.2050204@ulb.ac.be> Grattan a ?crit : > >> Which will see me better off than by giving UI. >> >> > +=+ That seems provocative. However, if the offence > were aggravated - by repetition, perhaps, or because > there is a knowing violation of Law 72B1, the PP could > be, say, 50% or 100% of a top. > Do we think John might be persuaded that Herman > is aware of 72B1? > AG : do you think that players would appreciate being penalized 100% of a top for having applied common sense rather than law.? Remember that Law (not Bridge Laws) consider this a non-offence. For example,. you aren't compelled to stop level with the stop sign or line (what the law says) when it is needed to move a bit farther in order to be able to see vehicles coming on the other road (what common sense says). You are encouraged to do the latter even if you know this is contrary to Law. It has even be judged that erring in good faith in deciding what common sense would require can wipe the offence, e.g. you bypassed an orange light while you could have stopped, but you thought you couldn't. Now, for many players, including strong ones, dWS is common sense. This should make us think. Best regards Alain From nigelguthrie at talktalk.net Fri Nov 14 12:41:04 2008 From: nigelguthrie at talktalk.net (Nigel Guthrie) Date: Fri, 14 Nov 2008 11:41:04 +0000 Subject: [blml] Law 83 (was ...damn) [SEC=UNOFFICIAL] In-Reply-To: References: Message-ID: <491D63D0.9080909@talktalk.net> [Richard.Hills] What often then happened was that the newbie did appeal, was fined 3 VPs for an appeal without merit, then complained to the TD, "Why did you tell me to appeal?" The new wording "could well" means that the Director need not suggest meritless appeals by newbies of open-and-shut rulings. (In major ABF events, such as the forthcoming Summer Festival of Bridge, this problem of meritless appeals by newbies is mitigated by having Appeals Advisors.) [Nigel] I hope the new 2018 rule-book says that, in any event that allows appeals - - The director should appoint at least one experienced player as appeals adviser. - The director should tell contestants that they may appeal after they have consulted an appeals adviser. - If the adviser judges the appeal to have merit, on the facts elucidated by the director, then the committee won't impose a fine or AWMW. From nigelguthrie at talktalk.net Fri Nov 14 12:50:16 2008 From: nigelguthrie at talktalk.net (Nigel Guthrie) Date: Fri, 14 Nov 2008 11:50:16 +0000 Subject: [blml] A player's comment In-Reply-To: <491D2CB9.70504@skynet.be> References: <491C27D9.4010501@skynet.be> <2a1c3a560811131034h6b48945bl9cc6d238b09159b9@mail.gmail.com> <491D2CB9.70504@skynet.be> Message-ID: <491D65F8.2010500@talktalk.net> [Herman De Wael] Now the WBFLC have issued a statement saying that you lot are right, but does that make it to the players? No it does not. [Nigel] Thank you Herman! This kind of information is never officially disseminated to mere players like me. It beggars belief that the WBF refuse to make such clarifications in place, in a web version of TFLB itself, post a list of such corrigenda, and tell players what they've done. From Hermandw at skynet.be Fri Nov 14 15:11:40 2008 From: Hermandw at skynet.be (Herman De Wael) Date: Fri, 14 Nov 2008 15:11:40 +0100 Subject: [blml] Mr Chesterton suggested [SEC=UNOFFICIAL] In-Reply-To: <002301c94642$bcebb6c0$0302a8c0@Mildred> References: <2CF77EB18250470CB513261A89089258@JOHN> <491B5AFD.8000604@skynet.be><7A05D24604AF42DFAC4C29F8329E46C9@JOHN> <491C59DC.4000504@skynet.be> <002301c94642$bcebb6c0$0302a8c0@Mildred> Message-ID: <491D871C.7010800@skynet.be> Grattan wrote: > > Grattan Endicott also ************************************ > "We desire truth and find within > ourselves only uncertainty." > [Pascal] > ''''''''''''''''''''''''''''''''''''''''''''''''''''''''''''''''''''''''''''''''''''''''''''''''' > > > > ----- Original Message ----- > From: "Herman De Wael" > To: "Bridge Laws Mailing List" > Sent: Thursday, November 13, 2008 4:46 PM > Subject: Re: [blml] Mr Chesterton suggested [SEC=UNOFFICIAL] > > >> John (MadDog) Probst wrote: >>> I'll issue a PP. > <(Herman) >> Which will see me better off than by giving UI. >> > +=+ That seems provocative. However, if the offence > were aggravated - by repetition, perhaps, or because > there is a knowing violation of Law 72B1, the PP could > be, say, 50% or 100% of a top. > Do we think John might be persuaded that Herman > is aware of 72B1? > ~ G ~ +=+ It is usually very wrong to comment on just one sentence, but this one is terribly flagrant. In the very next sentence I told you I was aware that this was not a good answer. But in the next one I told you of someone less knowledgeable than me, who might do the same, and get off more lightly than someone else who followed the rule as you put it. I am getting very angry at some of your replies, Grattan. Herman. From darkbystry at wp.pl Fri Nov 14 15:42:40 2008 From: darkbystry at wp.pl (Bystry) Date: Fri, 14 Nov 2008 15:42:40 +0100 Subject: [blml] Amsterdam (2) [SEC=UNOFFICIAL] References: <001101c944e5$84fea410$0302a8c0@Mildred> <002e01c94561$661116b0$32334410$@nl> <003501c94574$fb66f000$0302a8c0@Mildred><491C197F.9090106@skynet.be> <491C251C.2070807@bridgescore.de> <1L0w43-0Ob3Eu0@fwd07.aul.t-online.de> Message-ID: <00c901c94667$3f2c1840$15844c59@chello.pl> Hi all. Josef, [snip] > Much more about facts were not written - it means the description of the case by Grom?ller was not contested. By whom? I didn't see any Italian players nor AC members nor involved TD, who had commented at all about the facts (with an exception of Ton, but I don't know what was his exact role in this incident). That doesn't mean that they agree with your (or Michael's) recollection of what really happened, that simply means that they didn't want/couldn't comment. You must remember that no case can be judged objectivly unless two sides (and in this case probably three) can present their stance. And don't imply that I disbelieve you, I simply cannot form an opinion without hearing from the other side. > The main facts : > 1. The TD was not called during the play, only after the round was mentioned to the TD, that there was a longer hesitation on a side of the screen. The TD was still called timely. Of course such a delay could result in reducing their chances for positive ruling because of consequential difficulties with determination of important facts - mainly establishment of hesitation. But as Michael wrote, the Germans agreed that significant and unmistakeable hesitation occured. Do you really think that mere issue of delaying TD call (but still calling timely) makes it justified to reject Italian protest? Remember that during the play it is offen hard to analyze all aspects of transmition and usage of UI and the players prefer to concentrate on game, putting aside such issues till the conclusion of play, then coolly going back to them with the help from their teammates and captain. > 2. The TD did not decide in this case That is truely bad. I don't agree with Herman, who claims it was mere formality and that TD could equally have tossed a coin. In my strong opinion it is obligatory for all the officials to stick to the procedures regulated by Laws or CoC. The TD should make a thoughtful and complete ruling even if it would really be appealed anyway. That's his duty and he failed. If he didn't, we wouldn't have all this mess and going back home 30 minutes earlier wasn't worth of it. > 3. The AC dealt with this questionable claim I don't see it as questionable. TD call was timely and his (non)ruling could be appealed. > 4. The Bambergers were refused to give information about the scene of events Is it true? As I read Michael's report Bambergers were given the right to present their case, but erronously only once, not twice (before TD ruling and during an appeal). It is indeed unfortunate and shouldn't happen, but you can't claim that they couldn't state their arguments. > 5. The fact is, that the AC made a ruling which is unbelivable for the most bridge players who were asked for an opinion. Most players? Such a poll would be only valid if taken by some impartial party. Otherwise it is very frequent that such a poll produces outcome strangely matching poller's expectation. For me it is unbelievable to bid 3NT in this auction. Partner is making takeout double and you consider queen second as spade stoper? Even in the actual case, where partner had a strong hand with SK, 3NT was an awful contract that could be made only on a very lucky layout. Often Ax would not be enough (playing from the wrong side). And why we should assume that partner has three spades? Even two is quite rare for a takeout double (maybe LHO could bid 4S having three, but why should he do so with 3-4-3-3 and scattered values?). Of course hesitation changes this estimates drastically. I for one moment don't believe that stretching could be valid for such a long delay. Players at this level don't think for 30 seconds (or more) whether they should balance on 11-12 PC, they were in such situations so many times that they make such decisions quickly. So the real reason for the hesitation could be only: - an unsuitable hand for takeout double, often too long or too strong spades, short side suit - hand which lies close between bidding suit and doubling Of those two I estimate first option more probable, because it will often cause troubles in consequent bidding and can easly bypass 3NT if partner doesn't have anything in that suit (e.g. having KJx himself). So it requires more judgment and prediction of future auction. So yes, for me there was UI, 3NT was demonstrably suggested (that doesn't mean it would always be succesful, that does mean that chance for it being playable contract had risen drastically after receiving UI) and of course there were some less succesful LA's. And therefore I consider AC decision as sound and for me justice was done, although the process of reaching this decision was seriously flawed. So IMO Germans are due an apology from the organizers, but not reinstation of pre-AC score and second chance of winning the event. Having said that I must admit that top German players have some specific style of bidding which I could e.g. observe watching WMSG or playing at the table against Mr. Piekarek, Mr. Smirnov or Mr. Gotard. And I don't suggest that in this particular case or any other they didn't tried to do everything in respect for the rules and spirit of the game. Maybe 3NT was just some wild shot that happened to work and wasn't provoked by the hesitation. But TD and AC are not to read player's mind, they must rule according to uniform standard which is written in Laws. Sometimes it is sad and in particular case may seem unjust. More so if it decides who wins tournament/match/event. But it still is reasonable and justified in long terms so we must learn to live with it. [snip] > Best regards > Josef Apologize for my english and any mistakes, for a long time I was in a "read-only mode" and didn't write/speak anything in this language. Regards Maciej Bystrzejewski (from Poland, bridge player, frequent reader of BLML, interested in Bridge Laws) From darkbystry at wp.pl Fri Nov 14 15:43:48 2008 From: darkbystry at wp.pl (Bystry) Date: Fri, 14 Nov 2008 15:43:48 +0100 Subject: [blml] Amsterdam (2) [SEC=UNOFFICIAL] References: <001101c944e5$84fea410$0302a8c0@Mildred> <002e01c94561$661116b0$32334410$@nl> <003501c94574$fb66f000$0302a8c0@Mildred><491C197F.9090106@skynet.be> <491C251C.2070807@bridgescore.de> <1L0w43-0Ob3Eu0@fwd07.aul.t-online.de> Message-ID: <00de01c94667$674f5d00$15844c59@chello.pl> Hi all. Josef, [snip] > Much more about facts were not written - it means the description of the case by Grom?ller was not contested. By whom? I didn't see any Italian players nor AC members nor involved TD, who had commented at all about the facts (with an exception of Ton, but I don't know what was his exact role in this incident). That doesn't mean that they agree with your (or Michael's) recollection of what really happened, that simply means that they didn't want/couldn't comment. You must remember that no case can be judged objectivly unless two sides (and in this case probably three) can present their stance. And don't imply that I disbelieve you, I simply cannot form an opinion without hearing from the other side. > The main facts : > 1. The TD was not called during the play, only after the round was mentioned to the TD, that there was a longer hesitation on a side of the screen. The TD was still called timely. Of course such a delay could result in reducing their chances for positive ruling because of consequential difficulties with determination of important facts - mainly establishment of hesitation. But as Michael wrote, the Germans agreed that significant and unmistakeable hesitation occured. Do you really think that mere issue of delaying TD call (but still calling timely) makes it justified to reject Italian protest? Remember that during the play it is offen hard to analyze all aspects of transmition and usage of UI and the players prefer to concentrate on game, putting aside such issues till the conclusion of play, then coolly going back to them with the help from their teammates and captain. > 2. The TD did not decide in this case That is truely bad. I don't agree with Herman, who claims it was mere formality and that TD could equally have tossed a coin. In my strong opinion it is obligatory for all the officials to stick to the procedures regulated by Laws or CoC. The TD should make a thoughtful and complete ruling even if it would really be appealed anyway. That's his duty and he failed. If he didn't, we wouldn't have all this mess and going back home 30 minutes earlier wasn't worth of it. > 3. The AC dealt with this questionable claim I don't see it as questionable. TD call was timely and his (non)ruling could be appealed. > 4. The Bambergers were refused to give information about the scene of events Is it true? As I read Michael's report Bambergers were given the right to present their case, but erronously only once, not twice (before TD ruling and during an appeal). It is indeed unfortunate and shouldn't happen, but you can't claim that they couldn't state their arguments. > 5. The fact is, that the AC made a ruling which is unbelivable for the most bridge players who were asked for an opinion. Most players? Such a poll would be only valid if taken by some impartial party. Otherwise it is very frequent that such a poll produces outcome strangely matching poller's expectation. For me it is unbelievable to bid 3NT in this auction. Partner is making takeout double and you consider queen second as spade stoper? Even in the actual case, where partner had a strong hand with SK, 3NT was an awful contract that could be made only on a very lucky layout. Often Ax would not be enough (playing from the wrong side). And why we should assume that partner has three spades? Even two is quite rare for a takeout double (maybe LHO could bid 4S having three, but why should he do so with 3-4-3-3 and scattered values?). Of course hesitation changes this estimates drastically. I for one moment don't believe that stretching could be valid for such a long delay. Players at this level don't think for 30 seconds (or more) whether they should balance on 11-12 PC, they were in such situations so many times that they make such decisions quickly. So the real reason for the hesitation could be only: - an unsuitable hand for takeout double, often too long or too strong spades, short side suit - hand which lies close between bidding suit and doubling Of those two I estimate first option more probable, because it will often cause troubles in consequent bidding and can easly bypass 3NT if partner doesn't have anything in that suit (e.g. having KJx himself). So it requires more judgment and prediction of future auction. So yes, for me there was UI, 3NT was demonstrably suggested (that doesn't mean it would always be succesful, that does mean that chance for it being playable contract had risen drastically after receiving UI) and of course there were some less succesful LA's. And therefore I consider AC decision as sound and for me justice was done, although the process of reaching this decision was seriously flawed. So IMO Germans are due an apology from the organizers, but not reinstation of pre-AC score and second chance of winning the event. Having said that I must admit that top German players have some specific style of bidding which I could e.g. observe watching WMSG or playing at the table against Mr. Piekarek, Mr. Smirnov or Mr. Gotard. And I don't suggest that in this particular case or any other they didn't tried to do everything in respect for the rules and spirit of the game. Maybe 3NT was just some wild shot that happened to work and wasn't provoked by the hesitation. But TD and AC are not to read player's mind, they must rule according to uniform standard which is written in Laws. Sometimes it is sad and in particular case may seem unjust. More so if it decides who wins tournament/match/event. But it still is reasonable and justified in long terms so we must learn to live with it. [snip] > Best regards > Josef Apologize for my English and any mistakes, for a long time I was in a "read-only mode" and didn't write/speak anything in this language. Regards Maciej Bystrzejewski (from Poland, bridge player, frequent reader of BLML, interested in Bridge Laws) From ehaa at starpower.net Fri Nov 14 15:49:08 2008 From: ehaa at starpower.net (Eric Landau) Date: Fri, 14 Nov 2008 09:49:08 -0500 Subject: [blml] A player's comment In-Reply-To: <2a1c3a560811131034h6b48945bl9cc6d238b09159b9@mail.gmail.com> References: <491C27D9.4010501@skynet.be> <2a1c3a560811131034h6b48945bl9cc6d238b09159b9@mail.gmail.com> Message-ID: <9895762B-3BCE-45E6-A3A3-8DE6F689AFCC@starpower.net> On Nov 13, 2008, at 1:34 PM, Wayne Burrows wrote: > 2008/11/14 Herman De Wael : >> Heard yesterday from a (top-level) player in Belgium. >> >> She was playing in a club tournament, and the bidding went: >> >> 1NT - 2Di - pass - 2Sp >> pass- 3NT - pass - 4Di >> pass- 4Sp >> (there may have been some doubles in there) >> >> 2Di was a multi, and the player had long hearts. It was not alerted. >> 2Sp was obviously intended as natural, and the player had long >> spades. >> In the Multi-system, 2Sp shows heart support. >> 2Sp was alerted. >> >> Since 4Sp went lots down, there was no TD call any more. >> But what she complained most about was the alert on 2Sp. >> She considered this most unethical, on an equal footing with >> saying "2Di >> was Multi". >> >> I told her that I agreed with her, but that most of you would act the >> same way. >> I shall not repeat the words she used for all of you. > > We cannot be blamed for her ignorance of the law. Or her lack of logical thinking. She asserts that "saying '2D was Multi'" would be "most unethical", which means that it must be ethically preferable (or at least equivalent) to say "2D was not Multi". Given that 2D *was* in fact Multi, I don't see how she could sensibly defend that position. Eric Landau 1107 Dale Drive Silver Spring MD 20910 ehaa at starpower.net From nigelguthrie at talktalk.net Fri Nov 14 16:00:42 2008 From: nigelguthrie at talktalk.net (Nigel Guthrie) Date: Fri, 14 Nov 2008 15:00:42 +0000 Subject: [blml] A player's comment In-Reply-To: <491D2D36.9040108@skynet.be> References: <491C27D9.4010501@skynet.be> <491C7B76.8040002@talktalk.net> <491D2D36.9040108@skynet.be> Message-ID: <491D929A.4080800@talktalk.net> [Herman De Wael] My point with this story is just that a top class player believes alerting 2Sp is reprehensible. Which it isn't, I agree, but it should be. And it is important to see that this is what players feel should be the correct way to play bridge. [Nigel] Many players *criticise* some laws and regulations as ridiculous. We hope that the WBFLC will rationalise them eventually but, in the meantime, we still *comply* with them. A few directors on BLML have criticised other laws and regulations but I hope that they still rigorously obey and *enforce* them. After all, We are free to campaign to improve the rules of our game, no matter how much Kojak protests. [Herman De Wael] She knows quite well that 3NT and 4Sp are gross violations, and decided not to do anything about it. This was a club tournament in a club of which she is a member only in order to play in their team in the highest division - she does not wish to make waves. Besides, the culprit was a Director - I'll have a word with him at some stage. [Nige2] Both the expert lady and the director should learn the rules and comply with them. Unless the lady called the director to establish what really happened, she shouldn't slander other players with her version of events. Given Herman's attitude to conforming with rules of which he disapproves, his reluctance to confront the allegedly law-breaking director is understandable. From agot at ulb.ac.be Fri Nov 14 16:25:27 2008 From: agot at ulb.ac.be (Alain Gottcheiner) Date: Fri, 14 Nov 2008 16:25:27 +0100 Subject: [blml] Amsterdam (2) [SEC=UNOFFICIAL] In-Reply-To: <00c901c94667$3f2c1840$15844c59@chello.pl> References: <001101c944e5$84fea410$0302a8c0@Mildred> <002e01c94561$661116b0$32334410$@nl> <003501c94574$fb66f000$0302a8c0@Mildred><491C197F.9090106@skynet.be> <491C251C.2070807@bridgescore.de> <1L0w43-0Ob3Eu0@fwd07.aul.t-online.de> <00c901c94667$3f2c1840$15844c59@chello.pl> Message-ID: <491D9867.9050506@ulb.ac.be> Bystry a ?crit : > > The TD was still called timely. Of course such a delay could result in > reducing their chances for positive ruling because of consequential > difficulties with determination of important facts - mainly establishment of > hesitation. But as Michael wrote, the Germans agreed that significant and > unmistakeable hesitation occured. Do you really think that mere issue of > delaying TD call (but still calling timely) makes it justified to reject > Italian protest? AG : not really, but it might make their case more delicate, because facts would be more difficult to ascertain. BTW, the problem wasn't there ; the problem was in the uncautious way the AC acted. > > That is truely bad. I don't agree with Herman, who claims it was mere > formality and that TD could equally have tossed a coin. In my strong opinion > it is obligatory for all the officials to stick to the procedures regulated > by Laws or CoC. The TD should make a thoughtful and complete ruling even if > it would really be appealed anyway. AG : especially as matters of fact should be decided by the TD, not the AC, and they're essential in this case. > > For me it is unbelievable to bid 3NT in this auction. Partner is making > takeout double and you consider queen second as spade stoper? AG : remember that the score can only be changed if there is the hesitation unmistakably suggests bidding 3NT. I see you pretend it does. How do you ascertain whether the hesitator had any of those problem hands : a) Jxx - AKxx - Kxxx - AQ where he could be concerned about finding 3NT b) void - AKxx - Kxx - Axxxxx where he could be concerned about a leave-in (and where 3NT wouldn't be very succesful) c) xx - AKxxx - xx - AQxx where he could have thought about bidding 4H (and once again bidding 3NT isn't great) Contrast with an hesitation before making a *penalty* double, which can only suggest you're not sure about them going down (enough). IMHO, for a strong player, type b) is more plausible than type a), because with type a) you're able to decide what to do when 3S is passed round to you, while with type b) you don't expect that to happen (you planned to double 4S, but what do you do over 3 ?). One should ask some players what type of problem is suggested by the slow double, without knowing this hand. I suppose asking whether the AC di that is a purely rhetorical question. I guess many would cite type c) and say taking the low course in one's long suit (here 4D) was suggested. In this case, bidding 3NT wouldn't be improper at all. Best regards lain From agot at ulb.ac.be Fri Nov 14 16:51:05 2008 From: agot at ulb.ac.be (Alain Gottcheiner) Date: Fri, 14 Nov 2008 16:51:05 +0100 Subject: [blml] A player's comment In-Reply-To: <9895762B-3BCE-45E6-A3A3-8DE6F689AFCC@starpower.net> References: <491C27D9.4010501@skynet.be> <2a1c3a560811131034h6b48945bl9cc6d238b09159b9@mail.gmail.com> <9895762B-3BCE-45E6-A3A3-8DE6F689AFCC@starpower.net> Message-ID: <491D9E69.7010902@ulb.ac.be> Eric Landau a ?crit : > Or her lack of logical thinking. She asserts that "saying '2D was > Multi'" would be "most unethical", which means that it must be > ethically preferable (or at least equivalent) to say "2D was not > Multi". Given that 2D *was* in fact Multi, I don't see how she could > sensibly defend that position. > Consider the three main moral effects of using dWS : - you're lying - you avoid putting opponents into a cognitive conflict - you help te TD ensuring a fair tournament by increasing the probability that the deal won't need any assigned score Adapting your explanations to partner's is only unethical if the offence in a) isn't compensated by the positive acts in b) and c). And remember, if you've a partner that always remembers his system, using dWS means you give the right explanation. Perhaps that's why I've chosen this side. Best regards Alain From grandaeval at tiscali.co.uk Fri Nov 14 15:20:56 2008 From: grandaeval at tiscali.co.uk (Grattan) Date: Fri, 14 Nov 2008 14:20:56 -0000 Subject: [blml] Mr Chesterton suggested [SEC=UNOFFICIAL] References: <2CF77EB18250470CB513261A89089258@JOHN> <491B5AFD.8000604@skynet.be><7A05D24604AF42DFAC4C29F8329E46C9@JOHN> <491C59DC.4000504@skynet.be><002301c94642$bcebb6c0$0302a8c0@Mildred> <491D560E.2050204@ulb.ac.be> Message-ID: <003f01c9467f$f13f2cd0$0302a8c0@Mildred> Grattan Endicott To: "Bridge Laws Mailing List" Sent: Friday, November 14, 2008 10:42 AM Subject: Re: [blml] Mr Chesterton suggested [SEC=UNOFFICIAL] Grattan a ?crit : > >> Which will see me better off than by giving UI. >> >> > +=+ That seems provocative. However, if the offence > were aggravated - by repetition, perhaps, or because > there is a knowing violation of Law 72B1, the PP could > be, say, 50% or 100% of a top. > Do we think John might be persuaded that Herman > is aware of 72B1? > AG : do you think that players would appreciate being penalized 100% of a top for having applied common sense rather than law.? Remember that Law (not Bridge Laws) consider this a non-offence. For example,. you aren't compelled to stop level with the stop sign or line (what the law says) when it is needed to move a bit farther in order to be able to see vehicles coming on the other road (what common sense says). You are encouraged to do the latter even if you know this is contrary to Law. It has even be judged that erring in good faith in deciding what common sense would require can wipe the offence, e.g. you bypassed an orange light while you could have stopped, but you thought you couldn't. Now, for many players, including strong ones, dWS is common sense. This should make us think. Best regards Alain < +=+ The law having been settled my duty is to defend it. Players are not entitled to make their own decisions to go against the law. ~ Grattan ~ +=+ From grandaeval at tiscali.co.uk Fri Nov 14 18:39:18 2008 From: grandaeval at tiscali.co.uk (Grattan) Date: Fri, 14 Nov 2008 17:39:18 -0000 Subject: [blml] A player's comment References: <491C27D9.4010501@skynet.be> <2a1c3a560811131034h6b48945bl9cc6d238b09159b9@mail.gmail.com> <491D2CB9.70504@skynet.be> <491D65F8.2010500@talktalk.net> Message-ID: <004001c9467f$f1844c20$0302a8c0@Mildred> Grattan Endicott To: "Bridge Laws Mailing List" Sent: Friday, November 14, 2008 11:50 AM Subject: Re: [blml] A player's comment > [Herman De Wael] > Now the WBFLC have issued a statement saying that you lot are right, > but does that make it to the players? No it does not. > > [Nigel] > Thank you Herman! This kind of information is never officially > disseminated to mere players like me. It beggars belief that the WBF > refuse to make such clarifications in place, in a web version of TFLB > itself, post a list of such corrigenda, and tell players what they've > done. > +=+ For some years now the WBFLC minutes have been made available on the website and on the ecats website. It is not practicable to amend the law book itself year by year since this is published in a variety of places in hard copy and various languages. We could not ask the NBOs publishing it to produce a fresh lawbook after every world championship. That is pie in the sky. However, to say that the Nigels of this world have difficulty in accessing the information is unjustified. NBOs are responsible for passing information down to their members as they think fit. And this chat show also serves a purpose in giving scope to diligent subscribers to pick up decisions and air them and relevant opinions for the benefit of the esoteric community it serves. As for Herman's "you lot are right" it was ever thus and the eventual statement was only what he could have expected. That he does not like the law is immaterial; the law is what it is and comes about because of a consensus reached by the responsible parties as to what is desirable. There is no overwhelming demand to change it - merely a small group of voices here that would have it changed - if it were otherwise there would be a stream of requests from NBOs and Zones as was the case with the former Law 25B. ~ Grattan ~ +=+ From grandaeval at tiscali.co.uk Fri Nov 14 18:44:16 2008 From: grandaeval at tiscali.co.uk (Grattan) Date: Fri, 14 Nov 2008 17:44:16 -0000 Subject: [blml] Amsterdam (2) [SEC=UNOFFICIAL] References: <001101c944e5$84fea410$0302a8c0@Mildred> <002e01c94561$661116b0$32334410$@nl> <003501c94574$fb66f000$0302a8c0@Mildred><491C197F.9090106@skynet.be><491C251C.2070807@bridgescore.de><1L0w43-0Ob3Eu0@fwd07.aul.t-online.de> <00c901c94667$3f2c1840$15844c59@chello.pl> Message-ID: <004901c94680$b99ef390$0302a8c0@Mildred> Grattan Endicott To: "Bridge Laws Mailing List" Sent: Friday, November 14, 2008 2:42 PM Subject: [blml] Amsterdam (2) [SEC=UNOFFICIAL] The TD was still called timely. +=+ I thought I read that the approach was to ton, not to a Director. At least I do not think ton was a Director although he has not indicated what his function was. ~ Grattan ~ +=+ From darkbystry at wp.pl Fri Nov 14 18:48:58 2008 From: darkbystry at wp.pl (Bystry) Date: Fri, 14 Nov 2008 18:48:58 +0100 Subject: [blml] Amsterdam (2) [SEC=UNOFFICIAL] References: <001101c944e5$84fea410$0302a8c0@Mildred> <002e01c94561$661116b0$32334410$@nl> <003501c94574$fb66f000$0302a8c0@Mildred><491C197F.9090106@skynet.be> <491C251C.2070807@bridgescore.de> <1L0w43-0Ob3Eu0@fwd07.aul.t-online.de><00c901c94667$3f2c1840$15844c59@chello.pl> <491D9867.9050506@ulb.ac.be> Message-ID: <014001c94681$459e5660$15844c59@chello.pl> Hi Alain >> For me it is unbelievable to bid 3NT in this auction. Partner is making >> takeout double and you consider queen second as spade stoper? > AG : remember that the score can only be changed if there is the > hesitation unmistakably suggests bidding 3NT. Of course. But I can also take into consideration the reason why did the player make his bid. And I will ask him about it. He can state a plethora of arguments but I will require him to explain why he had made such a gambling bid. I wouldn't take his answer as a proof that his call was demonstrably suggested (one of the reasons "demonstrably" was introduced is certainly to avoid ruling against lucky coincidences) but it can be an additional evidence, for example used as sort of tiebreaker. It is obvious that experienced partnerships are more capable of catching their UI transmitions than you or me. > I see you pretend it does. How do you ascertain whether the hesitator > had any of those problem hands : > a) Jxx - AKxx - Kxxx - AQ where he could be concerned about finding 3NT This hand is what I would expect for such a pause. Maybe with some rearrangments like stronger spades, weaker short suit, less balanced hand, only 3 hearts. But surely they all fall into the same category - not knowing whether to bid 3NT or to double. > b) void - AKxx - Kxx - Axxxxx where he could be concerned about a > leave-in (and where 3NT wouldn't be very succesful) With this hand I double lightning fast. What could be wrong if my partner leaves that in? I've got my ususal amount of defense, so if I find partner with something like KW9xx of spades and a side trick they would be seriously hit. And what better contract do we have to play here? > c) xx - AKxxx - xx - AQxx where he could have thought about bidding 4H > (and once again bidding 3NT isn't great) Yes, this is possible (although this hand seems too weak to bid 4H but I can certainly construct other), I admited that, but it is IMO less frequent problem than a). We must remember that in an absence of clear-cut situations like hesitant penalty double there always will be some doubt whether UI points directly to one defined action. That is not what IMO lawmakers intented. There is always a balance of probalibities. E.g. we can say that UI suggests something at 80% and something else at 20%. Do you think we should therefore not adjust? Let's see this fabricated example: N opens 1H, E overcalls 1S, S bids 2D (NF), W jumps to 4S. Now N hesitates unmistakeably and passes. Lets assume that spade lenght hold by S (and known opponents bidding) makes it very inprobable that N was thinking about doubling. About what could N be thinking in this position? a) bidding 4NT as a multi-catering bid, e.g. searching for clubs, offering choice between 5D and 5H and so on b) bidding 5C c) bidding 5D d) bidding 5H Now you and I will probably agree that 4BA by S is demonstrably suggested because it caters to whatever N had in mind. 5C caters to a), b) and c) and may cater to d) depending on S's heart suit. In my opinion it still is demonstrably suggested although in minority of cases it would not fit into N's. 5H caters only to d) (and requires S bidding NF 2D with support for hearts, not probable). So 5H is not demonstrably suggested. So, what with 5D? It caters to a) and c), sometimes to d). Depending on his hand S can sometimes estimate the likelihood of partners problem but not always. Take your pick - is 5D demonstrably suggested or not? In my opinion it is because a) and c) are most probable (and part of d) if S has good heart doubleton). Going back to our case - I don't claim that it was a clear-cut ruling. It was a though ruling requiring knowledge about offending side methods, skill in evaluating probable reasons for hesitation for these class of players (beginner hesitating before balancing is another "animal") and an ability to resist pressure which was surely caused by knowledge that AC decision was resulting in which competitor reaches the final of a prestigious event. I can only say that it is my personal opinion that they made a correct decision. Others might disagree and they could be right. What I see is a gambling bid made by a good player who was not trailing by 50 imps. I see that most probable reason for the hesitation matches this gamble. I don't say that even with UI 3NT couldn't be wrong, it could. I'm not even claiming that the player bid it because of UI. I assume that taking given facts into consideration this decision was in accordance with enough probability of real happenings that it is just and correct. > Contrast with an hesitation before making a *penalty* double, which can > only suggest you're not sure about them going down (enough). Of course, this is easy to cope with. > IMHO, for a strong player, type b) is more plausible than type a), > because with type a) you're able to decide what to do when 3S is passed > round to you, while with type b) you don't expect that to happen (you > planned to double 4S, but what do you do over 3 ?). I don't agree with b) being at all probable but our judgment may differ. But why are you stating that hesitating player could prepare for anything? He was seated W, so he saw 3S - pass only after the board was pushed and S could pass very quickly having nothing to think about. So why should he assume 3S opening? I like to prepare earlier for incoming different bidding situations but I certainly do not speculate what would I do over every possible starting auction on the other side of the screen ;-) > One should ask some players what type of problem is suggested by the > slow double, without knowing this hand. > I suppose asking whether the AC di that is a purely rhetorical question. Yes, certainly. That's what TD should do and he was negligent in his duties. But I can assure you that I, personally, responded to this problem not knowing anything about the whole case. I saw it first time here on BLML after Josef presented only E's hand and the bidding. And I answered to this problem in vain. Of course I'm not a player comparable to real contestant so my voice could be unimportant. > I guess many would cite type c) and say taking the low course in one's > long suit (here 4D) was suggested. In this case, bidding 3NT wouldn't be > improper at all. I think that would be OS defence. But bidding strange 3NT catering to more probable a) and later claiming that it was not suggested because it didn't cater to less probable c) is not my way of complying to the letter and spirit of 73C. > Best regards > lain Regards Maciek From rfrick at rfrick.info Fri Nov 14 19:16:50 2008 From: rfrick at rfrick.info (Robert Frick) Date: Fri, 14 Nov 2008 13:16:50 -0500 Subject: [blml] A player's comment In-Reply-To: <491D9E69.7010902@ulb.ac.be> References: <491C27D9.4010501@skynet.be> <2a1c3a560811131034h6b48945bl9cc6d238b09159b9@mail.gmail.com> <9895762B-3BCE-45E6-A3A3-8DE6F689AFCC@starpower.net> <491D9E69.7010902@ulb.ac.be> Message-ID: On Fri, 14 Nov 2008 10:51:05 -0500, Alain Gottcheiner wrote: > Eric Landau a ?crit : >> Or her lack of logical thinking. She asserts that "saying '2D was >> Multi'" would be "most unethical", which means that it must be >> ethically preferable (or at least equivalent) to say "2D was not >> Multi". Given that 2D *was* in fact Multi, I don't see how she could >> sensibly defend that position. >> > Consider the three main moral effects of using dWS : > > - you're lying Actually, to describe 2S as showing heart support, when you know that isn't what your partner intended and probably doesn't have, is deliberately giving a misleading explanation. And if you are looking for an honest man, my partner's explanation in this situation was "If she took my 2 Di bid to be multi, then it shows heart support; if she didn't, then it shows spades." From darkbystry at wp.pl Fri Nov 14 19:15:33 2008 From: darkbystry at wp.pl (Bystry) Date: Fri, 14 Nov 2008 19:15:33 +0100 Subject: [blml] Amsterdam (2) [SEC=UNOFFICIAL] References: <001101c944e5$84fea410$0302a8c0@Mildred> <002e01c94561$661116b0$32334410$@nl> <003501c94574$fb66f000$0302a8c0@Mildred><491C197F.9090106@skynet.be><491C251C.2070807@bridgescore.de><1L0w43-0Ob3Eu0@fwd07.aul.t-online.de><00c901c94667$3f2c1840$15844c59@chello.pl> <004901c94680$b99ef390$0302a8c0@Mildred> Message-ID: <016801c94684$fc7f5340$15844c59@chello.pl> Grattan, >> The TD was still called timely. > > +=+ I thought I read that the approach was to ton, not to > a Director. At least I do not think ton was a Director > although he has not indicated what his function was. > ~ Grattan ~ +=+ I'm sure that you've got much bigger chance of knowing what was Ton's function than me. Not that you would always told us even if you'd known ;-) As to the merit - i responded to this: [Josef] > The main facts : > 1. The TD was not called during the play, only after the round was mentioned to the TD, that there was a longer hesitation on a side of the screen. Now I searched and found this: [Ton] > Since I was present my description of the facts could be worth something. > Yes, the Italian team reported this case late (to me) And we could now guess who is right. Or maybe both and we've got answer to our earlier riddle ;-) Regards Maciek From darkbystry at wp.pl Fri Nov 14 20:15:37 2008 From: darkbystry at wp.pl (Bystry) Date: Fri, 14 Nov 2008 20:15:37 +0100 Subject: [blml] A player's comment References: <491C27D9.4010501@skynet.be> <2a1c3a560811131034h6b48945bl9cc6d238b09159b9@mail.gmail.com><9895762B-3BCE-45E6-A3A3-8DE6F689AFCC@starpower.net> <491D9E69.7010902@ulb.ac.be> Message-ID: <017201c9468d$60cdc180$15844c59@chello.pl> Alain, Now I'm not ready to engage fully in the whole discussion about dWS. Maybe I'll do it soon. But here I'll put my two penn'orth in. > Consider the three main moral effects of using dWS : > - you're lying Certainly. You lie, deliberately mislead opponents and brake the law. That is not the way of playing Bridge which I could recommend to anyone. > - you avoid putting opponents into a cognitive conflict What conflict? Following classical school you're practically telling them that your side has a bidding misunderstanding and they could use it to their best. And I see it as an andvantage of CS, no as an advantage of dWS. > - you help te TD ensuring a fair tournament by increasing the probability that the deal won't need any assigned score Hmm. Your partner has given MI. You are now giving additional MI. You have UI available. So TD would have to rule on three aspects. Now consider CS. Your partner has given MI, but it is now corrected so only damage prior to correction will be subject to redress. You don't give additional MI. Instead you transmit UI. Using UI can be avoided if both you and partner bid as if you never have heard your explanations. Is it so much to demand this, if it was your side that commited infraction? Of course not. You would end up in terrifying contract? That's certainly not my problem nor TD problem nor anyone's. Next time you would have strong reasons to learn your system. Any damage coming from MI can't be avoided. But in CS it only is important to damage prior to correction. So rest of the deal is played and NOS knows what you are doing (even if you don't know that). Now tell me which school causes more troubles for TD? > Adapting your explanations to partner's is only unethical if the offence > in a) isn't compensated by the positive acts in b) and c). I don't see any positive aspects of a), b) and c). > And remember, if you've a partner that always remembers his system, > using dWS means you give the right explanation. Perhaps that's why I've > chosen this side. And that should be point d) cause it is an argument for dWS. I'll paraphrase your own statement: Giving always correct systemic explanations (as you understand them) is only worse than dWS if positive aspect of d) isn't at least compensated by a), b) and c) gains of CS, dodging unfortunate obligation to lie notwithstanding. But I'll be fair. dWS has one other aspect which is significantly positive for some people. To say it straight - dWS advantages OS. It reduces chances of mixup. But for me it is not and will never be any reason for such change. I'm fed up with nowadays notion to imply so called "equity" for OS. 12C3, changed revoke regulations, stupid claim rules, some other. So I oppose adding another one to the list. I can assure you that it is possible to play giving no or minimal MI and nearly never using UI (at least not consciously). That's what Bridge should look like. > Best regards > Alain Regards Maciek From ehaa at starpower.net Fri Nov 14 21:49:01 2008 From: ehaa at starpower.net (Eric Landau) Date: Fri, 14 Nov 2008 15:49:01 -0500 Subject: [blml] Amsterdam (2) In-Reply-To: <491D9867.9050506@ulb.ac.be> References: <001101c944e5$84fea410$0302a8c0@Mildred> <002e01c94561$661116b0$32334410$@nl> <003501c94574$fb66f000$0302a8c0@Mildred><491C197F.9090106@skynet.be> <491C251C.2070807@bridgescore.de> <1L0w43-0Ob3Eu0@fwd07.aul.t-online.de> <00c901c94667$3f2c1840$15844c59@chello.pl> <491D9867.9050506@ulb.ac.be> Message-ID: <24AF63A9-B8BA-48D1-A0F7-BEDF2034D94D@starpower.net> On Nov 14, 2008, at 10:25 AM, Alain Gottcheiner wrote: > Bystry a ?crit : > >> That is truely bad. I don't agree with Herman, who claims it was mere >> formality and that TD could equally have tossed a coin. In my >> strong opinion >> it is obligatory for all the officials to stick to the procedures >> regulated >> by Laws or CoC. The TD should make a thoughtful and complete >> ruling even if >> it would really be appealed anyway. > > AG : especially as matters of fact should be decided by the TD, not > the > AC, and they're essential in this case. Isn't that backwards? "...the committee may exercise all powers assigned... to the Director, except that the committee may not overrule the Director on a point of law or regulations or on exercise of his... disciplinary powers." So "matters of fact" -- whether objective "facts" (what actually happened) or subjective "facts" ("bridge judgments") -- would seem to be just about the only thing left which may legally be decided by the AC over the objection of the TD. Eric Landau 1107 Dale Drive Silver Spring MD 20910 ehaa at starpower.net From ehaa at starpower.net Fri Nov 14 22:27:30 2008 From: ehaa at starpower.net (Eric Landau) Date: Fri, 14 Nov 2008 16:27:30 -0500 Subject: [blml] A player's comment In-Reply-To: <491D9E69.7010902@ulb.ac.be> References: <491C27D9.4010501@skynet.be> <2a1c3a560811131034h6b48945bl9cc6d238b09159b9@mail.gmail.com> <9895762B-3BCE-45E6-A3A3-8DE6F689AFCC@starpower.net> <491D9E69.7010902@ulb.ac.be> Message-ID: On Nov 14, 2008, at 10:51 AM, Alain Gottcheiner wrote: > Eric Landau a ?crit : > >> Or her lack of logical thinking. She asserts that "saying '2D was >> Multi'" would be "most unethical", which means that it must be >> ethically preferable (or at least equivalent) to say "2D was not >> Multi". Given that 2D *was* in fact Multi, I don't see how she could >> sensibly defend that position. > > Consider the three main moral effects of using dWS : > > - you're lying Indeed. And in doing so, you are deliberately violating several laws, potentially to your own advantage. So you're cheating. > - you avoid putting opponents into a cognitive conflict Because your original lie has prevented them from knowing that there's anything to be conflicted about, thus insuring that they will not resolve the conflict correctly, again potentially to your own advantage. > - you help te TD ensuring a fair tournament by increasing the > probability that the deal won't need any assigned score Because your original lie prevents them even from realizing that anything has gone amiss, still potentially to your own advantage. You do indeed increase the probability that there won't be any assigned score -- even if there patently should be. > Adapting your explanations to partner's is only unethical if the > offence > in a) isn't compensated by the positive acts in b) and c). Only if you continue to cheat your way through (b) and (c), by deliberately violating L20F4 (which requires the a player offering an erroneous "dWS-type" explanation to immediately turn himself in) -- PTYOA, of course. Herman's attempt to find an "ethical dWS" approach, as he has explained, would require you to turn yourself in at the time specified by L20F5(b). I'm sure that Herman, personally, can be relied upon to resist the awful temptation to simply keep shtum and keep his illegitmately good score without any possibility of its being questioned, but that's hardly a sensible basis for encouraging him to reach the point where he could do just that. Moreover, if you take that approach, you merely exacerbate the opponents' "cognitive conflict" by giving them the information when it can only be useful for retroactive adjustment, not, any longer, for getting the conflict resolved and the board back on track. And when you do eventually 'fess up and tell your opponents that you lied to them earlier, don't you think that it will then be virtually 100% certain that the director will be needed and called at that point -- and that telling the truth is unlikely to require the director more than 100% of the time? > And remember, if you've a partner that always remembers his system, > using dWS means you give the right explanation. Perhaps that's why > I've > chosen this side. If you have a partner who always remembers your system, dWS isn't at all relelvant, nor are my comments above. The question is what you should do "given that 2D *was* in fact Multi". Eric Landau 1107 Dale Drive Silver Spring MD 20910 ehaa at starpower.net From JffEstrsn at aol.com Fri Nov 14 23:39:53 2008 From: JffEstrsn at aol.com (Jeff Easterson) Date: Fri, 14 Nov 2008 23:39:53 +0100 Subject: [blml] [Fwd: Amsterdam] Message-ID: <491DFE39.6030804@aol.com> This was my reaction to the email from Bystry. Thought the question raised at the end might be of general interest. JE -------- Original-Nachricht -------- Betreff: Amsterdam Datum: Fri, 14 Nov 2008 23:38:00 +0100 Von: Jeff Easterson An: Bystry Good analysis, it corresponds roughly to my opinion. And don't worry about your English; it is better than Harsanyi's. There is one (perhaps minor) point that bothers me. I asked about the apparent rule that only the players on the other side of the screen could call the TD after a hesitation. The explanation given was that, if the hesitation was long and obvious enough a call by a player on the side of the hesitater would be accepted. As I read the accounts of the case I gather that the TD was called some time after the session ended and the Germans had left the room. If this is really so, then the following questions: who called the TD? (Which side of the screen?) It is certainly not the intention of the rule that a player on the hesitating side of the screen tells his partner to call. And, when the TD was called, how did he establish if the hesitation was long enough to warrant further action? Surely that is not possible without talking to the Germans and apparently he didn't do this. Referring the case to the AC without talking to the Germans is surely not in conformity with the laws nor correct procedure. Czesc, JE From grandaeval at tiscali.co.uk Sat Nov 15 08:20:30 2008 From: grandaeval at tiscali.co.uk (Grattan) Date: Sat, 15 Nov 2008 07:20:30 -0000 Subject: [blml] Amsterdam (2) References: <001101c944e5$84fea410$0302a8c0@Mildred> <002e01c94561$661116b0$32334410$@nl> <003501c94574$fb66f000$0302a8c0@Mildred><491C197F.9090106@skynet.be> <491C251C.2070807@bridgescore.de> <1L0w43-0Ob3Eu0@fwd07.aul.t-online.de><00c901c94667$3f2c1840$15844c59@chello.pl><491D9867.9050506@ulb.ac.be> <24AF63A9-B8BA-48D1-A0F7-BEDF2034D94D@starpower.net> Message-ID: <001c01c946fa$715935d0$0302a8c0@Mildred> Grattan Endicott To: "Bridge Laws Mailing List" Sent: Friday, November 14, 2008 8:49 PM Subject: Re: [blml] Amsterdam (2) On Nov 14, 2008, at 10:25 AM, Alain Gottcheiner wrote: > Bystry a ?crit : > >> That is truely bad. I don't agree with Herman, who claims it was mere >> formality and that TD could equally have tossed a coin. In my >> strong opinion >> it is obligatory for all the officials to stick to the procedures >> regulated >> by Laws or CoC. The TD should make a thoughtful and complete >> ruling even if >> it would really be appealed anyway. > > AG : especially as matters of fact should be decided by the TD, not > the > AC, and they're essential in this case. Isn't that backwards? "...the committee may exercise all powers assigned... to the Director, except that the committee may not overrule the Director on a point of law or regulations or on exercise of his... disciplinary powers." So "matters of fact" -- whether objective "facts" (what actually happened) or subjective "facts" ("bridge judgments") -- would seem to be just about the only thing left which may legally be decided by the AC over the objection of the TD. +=+ I suggest that Law 83 points the way to what an appeals committee may be asked to consider: "point of fact or exercise of his discretionary power". ~ Grattan ~ +=+ From grandaeval at tiscali.co.uk Sat Nov 15 09:15:33 2008 From: grandaeval at tiscali.co.uk (Grattan) Date: Sat, 15 Nov 2008 08:15:33 -0000 Subject: [blml] [Fwd: Amsterdam] References: <491DFE39.6030804@aol.com> Message-ID: <001d01c946fa$71999a30$0302a8c0@Mildred> Grattan Endicott To: "Bridge Laws Mailing List" Sent: Friday, November 14, 2008 10:39 PM Subject: [blml] [Fwd: Amsterdam] It is certainly not the intention of the rule that a player on the hesitating side of the screen tells his partner to call. And, when the TD was called, how did he establish if the hesitation was long enough to warrant further action? Surely that is not possible without talking to the Germans and apparently he didn't do this. Referring the case to the AC without talking to the Germans is surely not in conformity with the laws nor correct procedure. Czesc, JE > +=+ When the player on the receiving side of the screen does neither what he "may "do (Law 16B2) nor what he "should" do (Law 16B3) there is an obvious presumption that at the time he saw no irregularity. If the question is raised later the Director, being appropriately sceptical, may well think that the player's partner has drawn his attention to the fact that there was a delay, something he had not particularly noticed previously. The fact that the player who paused says 'oh yes, I did take some time to reach a decision' does not add anything relevant to the facts. The question is what the players on the receiving side of the screen observed. This is the kind of thing the Director should determine, and is best placed to explore, before the question goes to the appeal committee. In the AC it is easy to get drawn into facts as worked out jointly by the two partners rather than facts as observed only on the receiving side of the screen. ~ Grattan ~ +=+ From darkbystry at wp.pl Sat Nov 15 10:30:01 2008 From: darkbystry at wp.pl (Bystry) Date: Sat, 15 Nov 2008 10:30:01 +0100 Subject: [blml] [Fw: Amsterdam] Message-ID: <003901c94704$bca15220$15844c59@chello.pl> I forward my response to Jeff > Hi Jeff, > > > Good analysis, it corresponds roughly to my opinion. And don't worry > > about your English; it is better than Harsanyi's. There is one (perhaps > > minor) point that bothers me. I asked about the apparent rule that only > > the players on the other side of the screen could call the TD after a > > hesitation. The explanation given was that, if the hesitation was long > > and obvious enough a call by a player on the side of the hesitater would > > be accepted. As I read the accounts of the case I gather that the TD > > was called some time after the session ended and the Germans had left > > the room. If this is really so, then the following questions: who > > called the TD? (Which side of the screen?) It is certainly not the > > intention of the rule that a player on the hesitating side of the screen > > tells his partner to call. And, when the TD was called, how did he > > establish if the hesitation was long enough to warrant further action? > > Surely that is not possible without talking to the Germans and > > apparently he didn't do this. Referring the case to the AC without > > talking to the Germans is surely not in conformity with the laws nor > > correct procedure. Czesc, JE > > The screens are used to minimalise transmition of UI. That's why we have > following procedure - TD should be called from the other side of the screen, > otherwise we can simply say that no UI was passed. It's of course a kind of > simplification but a rational one. So we have our current situation - the TD > is called after the match which is fully in compliance with TFLB.You may > imagine that the putative NOS will claim that UI was received on the other > side of the screen. And the putative OS can claim something different. I > agree that's up to TD to establish facts. But what can he really do? You're > certainly aware of not so infrequent situations when both sides state wholly > different set of facts. And it doesn't even mean that one of the sides is > lying, their perception could be simply anthitetic. So, the TD must decide > to believe one or the other side. In my opinion the logical procedure looks > like that - if TD is called instantly he would bend slightly to believe NOS, > but if he's called much later - quite the opposite. You want to establish > the hesitation? Call at once or get your opponents to agree with you and > call later. Therefore in our case TD should ask the Germans if they agree UI > was passed and in the absence of the other evidence (maybe Vugraph operators > or neutral kibitzers' testimony) he should be prone to believe them. And I > applaud the obvious display of integrity by the German side to confirm the > hesitation. > > What really happened in the issue of procedure is a different kettle of > fish. If the Germans are right than some officials made an astonishingly > poor work (and I don't want to use stronger words). But I'll refrain myself > from commenting this until the German appeal will be settled. > > Regards > > Maciej From nigelguthrie at talktalk.net Sat Nov 15 13:39:25 2008 From: nigelguthrie at talktalk.net (Nigel Guthrie) Date: Sat, 15 Nov 2008 12:39:25 +0000 Subject: [blml] [Fwd: Amsterdam] In-Reply-To: <001d01c946fa$71999a30$0302a8c0@Mildred> References: <491DFE39.6030804@aol.com> <001d01c946fa$71999a30$0302a8c0@Mildred> Message-ID: <491EC2FD.5060104@talktalk.net> [Grattan Endicott] +=+ When the player on the receiving side of the screen does neither what he "may "do (Law 16B2) nor what he "should" do (Law 16B3) there is an obvious presumption that at the time he saw no irregularity. If the question is raised later the Director, being appropriately sceptical, may well think that the player's partner has drawn his attention to the fact that there was a delay, something he had not particularly noticed previously. The fact that the player who paused says 'oh yes, I did take some time to reach a decision' does not add anything relevant to the facts. The question is what the players on the receiving side of the screen observed. This is the kind of thing the Director should determine, and is best placed to explore, before the question goes to the appeal committee. In the AC it is easy to get drawn into facts as worked out jointly by the two partners rather than facts as observed only on the receiving side of the screen. [Nigel] A sensible assessment on the limited information available. Officials should publish more facts about the course of events. Unnecessary censorship foments speculation. In the long term it is often futile to try to sweep things under the carpet. A more worrying case may be the Helgemo-Helnes allegation, to which the attention of WBF members has been drawn. I'm afraid this case may return to haunt the WBF later. Especially if similar public allegations are brought against a lesser pair and they cite this as precedent. From Harsanyi at t-online.de Sat Nov 15 14:10:45 2008 From: Harsanyi at t-online.de (Harsanyi at t-online.de) Date: Sat, 15 Nov 2008 14:10:45 +0100 Subject: [blml] =?iso-8859-15?q?=5BFwd=3A_Amsterdam=5D?= In-Reply-To: <491DFE39.6030804@aol.com> References: <491DFE39.6030804@aol.com> Message-ID: <1L1KvF-1yRiEa0@fwd00.aul.t-online.de> An HTML attachment was scrubbed... URL: http://www.amsterdamned.org/pipermail/blml/attachments/20081115/5ad4119f/attachment.htm From grandaeval at tiscali.co.uk Sat Nov 15 16:35:28 2008 From: grandaeval at tiscali.co.uk (Grattan) Date: Sat, 15 Nov 2008 15:35:28 -0000 Subject: [blml] [Fwd: Amsterdam] References: <491DFE39.6030804@aol.com> <001d01c946fa$71999a30$0302a8c0@Mildred> <491EC2FD.5060104@talktalk.net> Message-ID: <001801c94737$cd44a090$0302a8c0@Mildred> Grattan Endicott A more worrying case may be the Helgemo-Helnes allegation, to which the attention of WBF members has been drawn. I'm afraid this case may return to haunt the WBF later. Especially if similar public allegations are brought against a lesser pair and they cite this as precedent. << +=+ I have seen snippets about this incident but I have not seen a full account of it, nor do I know where it occurred and what regulations were applicable. ~ Grattan ~ +=+ From grabiner at alumni.princeton.edu Sat Nov 15 17:03:09 2008 From: grabiner at alumni.princeton.edu (David Grabiner) Date: Sat, 15 Nov 2008 11:03:09 -0500 Subject: [blml] A player's comment In-Reply-To: <491D2FA6.6030401@skynet.be> References: <491C27D9.4010501@skynet.be> <491D2FA6.6030401@skynet.be> Message-ID: "Herman De Wael" writes: > David Grabiner wrote: >> How would she react if the bid was correct but the alert was missing, and >> necessary? >> >> P-P-1S-P >> 2C-P-2D!-X >> 2S-3D >> >> 2C was not alerted but was Drury, and opener knew that. > > How do we know if he didn't alert it? This is a hypothetical, but we would know at the end of the hand. >> 2D was alerted, showing >> a minimum, allowing the doubler to show diamond values, and allowing partner >> to >> compete to 3D. Was the alert on the 2D call, which allowed the opposition to >> bid properly, unethical? >> > Yes, it is unethical in the sense that it reminded partner that he had > forgotten to alert (if such was the case). Don't worry about opponents, > they'll get redress if they are misinformed. You could just as easily say, "Don't worry about your own side; the opponents will get redress if you made a bid suggested by UI." And since you (regardless of which school you follow) understand the UI rules and know that they apply, while the opponents may not understand the MI rules unless you call the TD yourself every time there is a failure to alert, the UI is less likely to cause damage. > And of course, if the player suspects that his partner knows that 2C is > Drury, and that he simply forgot to alert, then he is allowed to alert > 2Di.. It is not a crime to follow MS (and I have never said it was), but > one has to be allowed to follow DWS too, if one chooses. As these paragraphs are written, you have a contradiction. I think what you are trying to say is that the alert is unethical if he believes partner forgot the convention, and ethical if he believes partner forgot to alert. While I don't agree with that policy, it is consistent. From schoderb at msn.com Sat Nov 15 18:25:10 2008 From: schoderb at msn.com (WILLIAM SCHODER) Date: Sat, 15 Nov 2008 12:25:10 -0500 Subject: [blml] [Fwd: Amsterdam] In-Reply-To: <491DFE39.6030804@aol.com> <001d01c946fa$71999a30$0302a8c0@Mildred><491EC2FD.5060104@talktalk.net> <001801c94737$cd44a090$0302a8c0@Mildred> References: <491DFE39.6030804@aol.com> <001d01c946fa$71999a30$0302a8c0@Mildred><491EC2FD.5060104@talktalk.net> <001801c94737$cd44a090$0302a8c0@Mildred> Message-ID: Ditto Kojak ----- Original Message ----- From: "Grattan" To: "Bridge Laws Mailing List" Sent: Saturday, November 15, 2008 10:35 AM Subject: Re: [blml] [Fwd: Amsterdam] > > > Grattan Endicott also ************************************ > "We desire truth and find within > ourselves only uncertainty." > [Pascal] > ''''''''''''''''''''''''''''''''''''''''''''''''''''''''''''''''''''''''''''''''''''''''''''''''' > > > > > A more worrying case may be the Helgemo-Helnes allegation, > to which the attention of WBF members has been drawn. I'm > afraid this case may return to haunt the WBF later. Especially if > similar public allegations are brought against a lesser pair and > they cite this as precedent. > << > +=+ I have seen snippets about this incident but I have not seen > a full account of it, nor do I know where it occurred and what > regulations were applicable. > ~ Grattan ~ +=+ > > > > > _______________________________________________ > blml mailing list > blml at amsterdamned.org > http://www.amsterdamned.org/mailman/listinfo/blml > From wjburrows at gmail.com Sun Nov 16 07:54:15 2008 From: wjburrows at gmail.com (Wayne Burrows) Date: Sun, 16 Nov 2008 19:54:15 +1300 Subject: [blml] [Fwd: Amsterdam] In-Reply-To: <001801c94737$cd44a090$0302a8c0@Mildred> References: <491DFE39.6030804@aol.com> <001d01c946fa$71999a30$0302a8c0@Mildred> <491EC2FD.5060104@talktalk.net> <001801c94737$cd44a090$0302a8c0@Mildred> Message-ID: <2a1c3a560811152254s74d4699bnf95ce79aac101a7b@mail.gmail.com> 2008/11/16 Grattan : > > > Grattan Endicott also ************************************ > "We desire truth and find within > ourselves only uncertainty." > [Pascal] > ''''''''''''''''''''''''''''''''''''''''''''''''''''''''''''''''''''''''''''''''''''''''''''''''' > > > >> A more worrying case may be the Helgemo-Helnes allegation, > to which the attention of WBF members has been drawn. I'm > afraid this case may return to haunt the WBF later. Especially if > similar public allegations are brought against a lesser pair and > they cite this as precedent. > << > +=+ I have seen snippets about this incident but I have not seen > a full account of it, nor do I know where it occurred and what > regulations were applicable. > ~ Grattan ~ +=+ > Beijing World Championships 2008 Open Round of 16 Third Segment Brd 14 KQJ72 East AQ None T4 AJT8 KANDEMIR T95 A84 KJ432 7 J93 AQ765 Q6 9753 HELGEMO HELNESS 63 T9865 K82 K42 KOLATA W N E S ================ P P 1H X XX P P 2S P 4S P P X P P P cascade: Helgemo opens very light spectator: its third hand, a normal approach in Norway jbgood: they do that in 3rd pos ->spectator: it seems to meet the definition of HUM if made by partnership agreement spectator: :) ->jbgood: openings on 7hcp by partnership agreement seems to be a HUM by definition spectator: I never said it was the right thing to do, just that it is common to do in Norway :) ->spectator: if that light is common i think it becomes a partnership agreement and then subject to regulation and HUMs are not allowed in this event as far as I can tell spectator: it is on their CC, so opponents do know about it spectator: even on HH cc's it tells, and even described on the first page of the CC under special notes, very light third hand openers spectator: but you could tell that Helness dident realy care, as he X'ed :) ->spectator: i cant see that on the convention card for this event spectator: hm, that was strange, have to check my self now spectator: true, must have been left out for some reason, maybe they have find out not to use them spectator: and then Helgemo just tried a psych I am cascade. jbgood is Jon SVEINDAL. spectator is a spectator who had a private conversation with me. Jon's and my comments I assume are in the public domain. I do not know the name of the spectator. I believe he was also from Norway. My understanding of the system regulations is that HUM systems were not allowed. -- Wayne Burrows Palmerston North New Zealand From grandaeval at tiscali.co.uk Sun Nov 16 11:36:21 2008 From: grandaeval at tiscali.co.uk (Grattan) Date: Sun, 16 Nov 2008 10:36:21 -0000 Subject: [blml] [Fwd: Amsterdam] References: <491DFE39.6030804@aol.com> <001d01c946fa$71999a30$0302a8c0@Mildred><491EC2FD.5060104@talktalk.net><001801c94737$cd44a090$0302a8c0@Mildred> <2a1c3a560811152254s74d4699bnf95ce79aac101a7b@mail.gmail.com> Message-ID: <001301c947d7$302631c0$0302a8c0@Mildred> Grattan Endicott To: "Bridge Laws Mailing List" Sent: Sunday, November 16, 2008 6:54 AM Subject: Re: [blml] [Fwd: Amsterdam] << > Beijing World Championships 2008 > Open > Round of 16 > Third Segment > > Brd 14 KQJ72 > East AQ > None T4 > AJT8 > KANDEMIR > T95 A84 > KJ432 7 > J93 AQ765 > Q6 9753 > HELGEMO HELNESS > 63 > T9865 > K82 > K42 > KOLATA > W N E S > ================ > P P > 1H X XX P > P 2S P 4S > P P X P > P P > > cascade: Helgemo opens very light > spectator: its third hand, a normal approach in Norway > jbgood: they do that in 3rd pos > ->spectator: it seems to meet the definition of HUM if made by > partnership agreement > spectator: :) > ->jbgood: openings on 7hcp by partnership agreement seems to be a HUM > by definition > spectator: I never said it was the right thing to do, just that it is > common to do in Norway :) > ->spectator: if that light is common i think it becomes a partnership > agreement and then subject to regulation and HUMs are not allowed in > this event as far as I can tell > spectator: it is on their CC, so opponents do know about it > spectator: even on HH cc's it tells, and even described on the first > page of the CC under special notes, very light third hand openers > spectator: but you could tell that Helness dident realy care, as he X'ed > :) > ->spectator: i cant see that on the convention card for this event > spectator: hm, that was strange, have to check my self now > spectator: true, must have been left out for some reason, maybe they > have find out not to use them > spectator: and then Helgemo just tried a psych > > I am cascade. > > jbgood is Jon SVEINDAL. > > spectator is a spectator who had a private conversation with me. > > Jon's and my comments I assume are in the public domain. I do not > know the name of the spectator. I believe he was also from Norway. > > My understanding of the system regulations is that HUM systems > were not allowed. << ...................................................................................................... +=+ HUM and BS were not allowed. However, if the result of the contract was 10 tricks there is no damage that could be adjusted. Satisfied N-S may not have referred to the Director so that, with no question raised, the matter may not have seen the light of day within the tournament. Had the question been raised, if very light third-in-hand openings were announced on the system card the question would be whether this condition in Appendix 4 to the System Policy applied: "Some partnerships of reasonably long standing develop understandings that psychic bids in certain situations will be of a certain type. These are developed partnership understandings and not conventions, and should be explained on the convention card and on the supplementary sheets. In other words the psychics should be made randomly but any understandings about them should be revealed." Third in hand would be a 'certain situation' and 'very light natural' would be 'a certain type' one might think. Alternatively, with no disclosure on the system card the question would be whether this was purely psychic, with no partnership understanding involved. The pair could have decided to dispense with this method for the tournament and this may have been an isolated deviation from agreed system genuinely unexpected by partner. The actions of Helness do not suggest any inkling that Helgemo might not have his bid. If no Director's ruling was requested that is about as far as we can go. Our knowledge lacks crucial information. The chatter among spectators is interesting but extraneous speculation and certainly not a matter for the Director. I do not see in what I know anything of which the WBF can stand accused. Thank you for the detail. ~ Grattan ~ +=+ From geller at nifty.com Sun Nov 16 13:00:44 2008 From: geller at nifty.com (Robert Geller) Date: Sun, 16 Nov 2008 21:00:44 +0900 Subject: [blml] [Fwd: Amsterdam] In-Reply-To: <001301c947d7$302631c0$0302a8c0@Mildred> References: <001301c947d7$302631c0$0302a8c0@Mildred> Message-ID: <200811161200.AA16549@geller204.nifty.com> It seems to me that those commenters on BLML are missing an important point. At this level of bridge (world championships) or at the highest level of bridge (major national championships) in any reasonably strong country, I can't imagine any pair would NOT sometimes open light in 3rd seat (especially NV), or that any pair at that level would feel put upon if their opponents opened light in 3rd hand. This is just a normal part of bridge today. If there is some silly rule that purports to prevent this ("You didn't have XX HCP. Off with your head.") the rule should be changed. But "everyone" knows that such silly rules are "really" intended to prevent FERTs and the like, and weren't meant to apply to light 3rd hand openings. Secretary-birds should find other things to carp about. And probably someday the rule-makers should redraft their rules. But it boggles my mind to think that anyone would say that a 3rd hand 1H opener (NV) on T95 KJ432 J93 Q6 is a felonious instance of unauthoried use of a HUM system. Give me a break! -Bob Grattan wrote: > > >Grattan Endicottalso ************************************ >"We desire truth and find within >ourselves only uncertainty." > [Pascal] >''''''''''''''''''''''''''''''''''''''''''''''''''''''''''''''''''''''''''''''''''''''''''''''''' >----- Original Message ----- >From: "Wayne Burrows" >To: "Bridge Laws Mailing List" >Sent: Sunday, November 16, 2008 6:54 AM >Subject: Re: [blml] [Fwd: Amsterdam] ><< >> Beijing World Championships 2008 >> Open >> Round of 16 >> Third Segment >> >> Brd 14 KQJ72 >> East AQ >> None T4 >> AJT8 >> KANDEMIR >> T95 A84 >> KJ432 7 >> J93 AQ765 >> Q6 9753 >> HELGEMO HELNESS >> 63 >> T9865 >> K82 >> K42 >> KOLATA >> W N E S >> ================ >> P P >> 1H X XX P >> P 2S P 4S >> P P X P >> P P >> >> cascade: Helgemo opens very light >> spectator: its third hand, a normal approach in Norway >> jbgood: they do that in 3rd pos >> ->spectator: it seems to meet the definition of HUM if made by >> partnership agreement >> spectator: :) >> ->jbgood: openings on 7hcp by partnership agreement seems to be a HUM >> by definition >> spectator: I never said it was the right thing to do, just that it is >> common to do in Norway :) >> ->spectator: if that light is common i think it becomes a partnership >> agreement and then subject to regulation and HUMs are not allowed in >> this event as far as I can tell >> spectator: it is on their CC, so opponents do know about it >> spectator: even on HH cc's it tells, and even described on the first >> page of the CC under special notes, very light third hand openers >> spectator: but you could tell that Helness dident realy care, as he X'ed >> :) >> ->spectator: i cant see that on the convention card for this event >> spectator: hm, that was strange, have to check my self now >> spectator: true, must have been left out for some reason, maybe they >> have find out not to use them >> spectator: and then Helgemo just tried a psych >> >> I am cascade. >> >> jbgood is Jon SVEINDAL. >> >> spectator is a spectator who had a private conversation with me. >> >> Jon's and my comments I assume are in the public domain. I do not >> know the name of the spectator. I believe he was also from Norway. >> >> My understanding of the system regulations is that HUM systems >> were not allowed. ><< >...................................................................................................... >+=+ HUM and BS were not allowed. > However, if the result of the contract was 10 tricks there is no >damage that could be adjusted. Satisfied N-S may not have referred >to the Director so that, with no question raised, the matter may not >have seen the light of day within the tournament. > Had the question been raised, if very light third-in-hand openings >were announced on the system card the question would be whether >this condition in Appendix 4 to the System Policy applied: > "Some partnerships of reasonably long standing develop > understandings that psychic bids in certain situations will > be of a certain type. These are developed partnership > understandings and not conventions, and should be explained > on the convention card and on the supplementary sheets. In > other words the psychics should be made randomly but any > understandings about them should be revealed." >Third in hand would be a 'certain situation' and 'very light natural' >would be 'a certain type' one might think. > Alternatively, with no disclosure on the system card the question >would be whether this was purely psychic, with no partnership >understanding involved. The pair could have decided to dispense with >this method for the tournament and this may have been an isolated >deviation from agreed system genuinely unexpected by partner. The >actions of Helness do not suggest any inkling that Helgemo might not >have his bid. > If no Director's ruling was requested that is about as far as we >can go. Our knowledge lacks crucial information. The chatter among >spectators is interesting but extraneous speculation and certainly not >a matter for the Director. I do not see in what I know anything of >which the WBF can stand accused. > Thank you for the detail. > ~ Grattan ~ +=+ > > >_______________________________________________ >blml mailing list >blml at amsterdamned.org >http://www.amsterdamned.org/mailman/listinfo/blml ----------------------------------------------------- Robert (Bob) Geller, Tokyo, Japan geller at nifty.com From nigelguthrie at talktalk.net Sun Nov 16 14:00:10 2008 From: nigelguthrie at talktalk.net (Nigel Guthrie) Date: Sun, 16 Nov 2008 13:00:10 +0000 Subject: [blml] [Fwd: Amsterdam] In-Reply-To: <200811161200.AA16549@geller204.nifty.com> References: <001301c947d7$302631c0$0302a8c0@Mildred> <200811161200.AA16549@geller204.nifty.com> Message-ID: <4920195A.1070509@talktalk.net> [Robert Geller] It seems to me that those commenters on BLML are missing an important point. At this level of bridge (world championships) or at the highest level of bridge (major national championships) in any reasonably strong country, I can't imagine any pair would NOT sometimes open light in 3rd seat (especially NV), or that any pair at that level would feel put upon if their opponents opened light in 3rd hand. This is just a normal part of bridge today. If there is some silly rule that purports to prevent this ("You didn't have XX HCP. Off with your head.") the rule should be changed. But "everyone" knows that such silly rules are "really" intended to prevent FERTs and the like, and weren't meant to apply to light 3rd hand openings. Secretary-birds should find other things to carp about. And probably someday the rule-makers should redraft their rules. But it boggles my mind to think that anyone would say that a 3rd hand 1H opener (NV) on T95 KJ432 J93 Q6 is a felonious instance of unauthoried use of a HUM system. Give me a break! [Nigel] IMO - - Bob Geller is right that rules that restrict systems are silly. - Such rules are among the many silly rules that should be changed. - Meanwhile some players, even at top level, masochistically handicap themselves by trying to comply with the rules. - They believe that they should obey a rule of a game, even if they disagree with it and deem it silly. - In this case, an alleged incident may or may not attract the director's interest. - But allegations of illegal patterns of behaviour should trigger official investigation of deal records. From geller at nifty.com Sun Nov 16 14:42:56 2008 From: geller at nifty.com (Robert Geller) Date: Sun, 16 Nov 2008 22:42:56 +0900 Subject: [blml] [Fwd: Amsterdam] In-Reply-To: <4920195A.1070509@talktalk.net> References: <4920195A.1070509@talktalk.net> Message-ID: <200811161342.AA16553@geller204.nifty.com> Nigel Guthrie writes: >[Nigel] >IMO - >- Bob Geller is right that rules that restrict systems are silly. >- Such rules are among the many silly rules that should be changed. >- Meanwhile some players, even at top level, masochistically handicap themselves by trying to comply with the rules. -> There are different levels of rules. TFLB, while far from perfect, have been gone over in detail by the drafters, reviewers, WBFLC, and WBF executive. Players should obey these rules. But the system regulations contain many silly provisions in terms of high card points (4-3-2-1) or the like that are widely ignored. The situation is like having a speed limit of 100 km/hr when everyone on the road is doing 110. >- They believe that they should obey a rule of a game, even if they disagree with it and deem it silly. -> Such silly system regulations are not in practice treated by the players with the same seriousness as TFLB themselves. If this is the case in an ideal world the authorities would actually change the rules to reflect reality, but this hasn't happened for whatever reasons..... I think it is reasonable for players in the World Championships to act in accord with the prevailing expert community standards. >- In this case, an alleged incident may or may not attract the director's interest. >- But allegations of illegal patterns of behaviour should trigger official investigation of deal records. -> "illegal patterns of behavior" !!?? C'mon gimme a break. If Helgemo opens a light 1H in 3rd seat at the Friday night club game against Mrs. Guggenheim and Nigel wants to hang him for that, well maybe I can go along with that. But players in the world championships can and do protect themselves against this sort of thing. It's unbecoming of BLML for members to be casually slinging around accustaions of "illegal patterns of behavior" over light 3rd seat openings (as opposed to real illegal behavior). -Bob ----------------------------------------------------- Robert (Bob) Geller, Tokyo, Japan geller at nifty.com From grandaeval at tiscali.co.uk Sun Nov 16 15:38:01 2008 From: grandaeval at tiscali.co.uk (Grattan) Date: Sun, 16 Nov 2008 14:38:01 -0000 Subject: [blml] [Fwd: Amsterdam] References: <001301c947d7$302631c0$0302a8c0@Mildred> <200811161200.AA16549@geller204.nifty.com> Message-ID: <000a01c947f8$ef0a9740$0302a8c0@Mildred> Grattan Endicott To: "Bridge Laws Mailing List" Sent: Sunday, November 16, 2008 12:00 PM Subject: Re: [blml] [Fwd: Amsterdam] > It seems to me that those commenters on BLML are missing an important > point. At this level of bridge (world championships) or at the highest > level > of bridge (major national championships) in any reasonably strong country, > I can't imagine any pair would NOT sometimes open light in 3rd seat > (especially > NV), or that any pair at that level would feel put upon if their opponents > opened > light in 3rd hand. This is just a normal part of bridge today. > > If there is some silly rule that purports to prevent this ("You didn't > have XX HCP. > Off with your head.") the rule should be changed. But "everyone" knows > that such silly rules are "really" intended to prevent FERTs and the like, > and weren't meant to apply to light 3rd hand openings. Secretary-birds > should > find other things to carp about. And probably someday the rule-makers > should > redraft their rules. But it boggles my mind to think that anyone would > say > that a 3rd hand 1H opener (NV) on > T95 > KJ432 > J93 > Q6 > is a felonious instance of unauthoried use of a HUM system. > > Give me a break! > > -Bob +=+ However, disclosure regulations have closed in a little on this in some tournaments, including the WBF events. Where disclosure is required the player should comply, and players at this level do know what they are required to disclose. ~ Grattan ~ +=+. From geller at nifty.com Sun Nov 16 15:54:28 2008 From: geller at nifty.com (Robert Geller) Date: Sun, 16 Nov 2008 23:54:28 +0900 Subject: [blml] [Fwd: Amsterdam] In-Reply-To: <000a01c947f8$ef0a9740$0302a8c0@Mildred> References: <000a01c947f8$ef0a9740$0302a8c0@Mildred> Message-ID: <200811161454.AA16558@geller204.nifty.com> Grattan wrote: >+=+ However, disclosure regulations have closed in a little >on this in some tournaments, including the WBF events. > Where disclosure is required the player should comply, >and players at this level do know what they are required to >disclose. > ~ Grattan ~ +=+. No one can disagree with this, but a level of common sense based on prevailing community standards is required. For example, 1NT openings on a good 14 (when the agreement is described as "15-17") or weak two openings on a 5 card suit, or light 3rd hand openings are so ubiquitous at this level of expert play that I wouldn't regard the failure to formally disclose such tendencies as warranting a score adjustment. It's possible my opinion is at variance with prevailing standards. Maybe someone should take an opinion poll of players at the next European championship. The canonical example is the Kantar appeal (about 18 yrs ago?) after the opponents bid Opener Responder 1NT 2C 2D 2NT 3NT P and didn't formally disclose that responder might not have a 4-card major. Amazingly Kantar won the appeal, but afterwards there was enough of a negative reaction that it became accepted practice (at least in the US) that expert players were expected to protect themselves in such cases (maybe only about 5% of North American experts would play 2NT as promising a major). Of course ideally Grattan is completely correct that all agreements ought to be formally disclosed, but I believe that in practice many are so common at a high level that they can be regarded as being "implicitly disclosed." -Bob ----------------------------------------------------- Robert (Bob) Geller, Tokyo, Japan geller at nifty.com From john at asimere.com Sun Nov 16 16:37:36 2008 From: john at asimere.com (John (MadDog) Probst) Date: Sun, 16 Nov 2008 15:37:36 -0000 Subject: [blml] [Fwd: Amsterdam] References: <001301c947d7$302631c0$0302a8c0@Mildred><200811161200.AA16549@geller204.nifty.com> <000a01c947f8$ef0a9740$0302a8c0@Mildred> Message-ID: ----- Original Message ----- From: "Grattan" To: "Bridge Laws Mailing List" Sent: Sunday, November 16, 2008 2:38 PM Subject: Re: [blml] [Fwd: Amsterdam] > > > Grattan Endicott also ************************************ > "We desire truth and find within > ourselves only uncertainty." > [Pascal] > ''''''''''''''''''''''''''''''''''''''''''''''''''''''''''''''''''''''''''''''''''''''''''''''''' > > > > ----- Original Message ----- > From: "Robert Geller" > To: "Bridge Laws Mailing List" > Sent: Sunday, November 16, 2008 12:00 PM > Subject: Re: [blml] [Fwd: Amsterdam] > > >> It seems to me that those commenters on BLML are missing an important >> point. At this level of bridge (world championships) or at the highest >> level >> of bridge (major national championships) in any reasonably strong >> country, >> I can't imagine any pair would NOT sometimes open light in 3rd seat >> (especially >> NV), or that any pair at that level would feel put upon if their >> opponents >> opened >> light in 3rd hand. This is just a normal part of bridge today. >> >> If there is some silly rule that purports to prevent this ("You didn't >> have XX HCP. >> Off with your head.") the rule should be changed. But "everyone" knows >> that such silly rules are "really" intended to prevent FERTs and the >> like, >> and weren't meant to apply to light 3rd hand openings. Secretary-birds >> should >> find other things to carp about. And probably someday the rule-makers >> should >> redraft their rules. But it boggles my mind to think that anyone would >> say >> that a 3rd hand 1H opener (NV) on >> T95 >> KJ432 >> J93 >> Q6 >> is a felonious instance of unauthoried use of a HUM system. >> >> Give me a break! >> >> -Bob > +=+ However, disclosure regulations have closed in a little > on this in some tournaments, including the WBF events. > Where disclosure is required the player should comply, > and players at this level do know what they are required to > disclose. > ~ Grattan ~ +=+. > "Of course I opened light; don't you?". Ye Gods, do we play bridge any more? It's GBK, not a HUM or a dingbat or a cormorant or any other sort of artifact. Call it a psyche if you will, but if I didn't expect it I shouldn't be at the event. This nonsense of Nigel's really does take the biscuit for complete triviality. If my opponents didn't open these some of the time I'd call the TD and ask him to check for breathing and a pulse. John > > > _______________________________________________ > blml mailing list > blml at amsterdamned.org > http://www.amsterdamned.org/mailman/listinfo/blml From grabiner at alumni.princeton.edu Sun Nov 16 16:49:26 2008 From: grabiner at alumni.princeton.edu (David Grabiner) Date: Sun, 16 Nov 2008 10:49:26 -0500 Subject: [blml] Protecting yourself (was Fwd: Amsterdam) In-Reply-To: <200811161454.AA16558@geller204.nifty.com> References: <000a01c947f8$ef0a9740$0302a8c0@Mildred> <200811161454.AA16558@geller204.nifty.com> Message-ID: <4F6FF3EE0E384A6695003768A8FA707E@erdos> "Robert Geller" writes: > The canonical example is the Kantar appeal (about 18 yrs ago?) > after the opponents bid > Opener Responder > 1NT 2C > 2D 2NT > 3NT P > and didn't formally disclose that responder might not have a 4-card major. > Amazingly Kantar won the appeal, but afterwards there was enough of > a negative reaction that it became accepted practice (at least in the > US) that expert players were expected to protect themselves in such cases > (maybe only about 5% of North American experts would play 2NT as > promising a major). The need to protect yourself here works, but it can cause problems, which are more visible if opener shows a major: 1NT 2C 2H 2NT 3NT Will opening leader always ask, "Does responder promise or deny four spades?" If he doesn't always ask, then he gives UI that his lead was not dependent on the information when he fails to ask. For example, if he doesn't ask and leads the SQ, partner could place him with QJT9 or QJT92 of spades, not QJT52 (would lead the 5 if responder has four) or QJ92 (would lead a different suit). Experienced players should protect themselves against missing alerts, but not when they will pass UI. And he has to ask, as looking at the convention card doesn't help here. Many intermediate players agree to play four-suit transfers without understanding the ramifications. I have had three different partners bid 2NT with four spades. Since I know the standard meaning of our agreement, I alerted, but if two of them had played with each other, there would have been no alert. From mfrench1 at san.rr.com Sun Nov 16 19:49:41 2008 From: mfrench1 at san.rr.com (Marvin L French) Date: Sun, 16 Nov 2008 10:49:41 -0800 Subject: [blml] Protecting yourself (was Fwd: Amsterdam) References: <000a01c947f8$ef0a9740$0302a8c0@Mildred><200811161454.AA16558@geller204.nifty.com> <4F6FF3EE0E384A6695003768A8FA707E@erdos> Message-ID: <0869218E769C489C9D562C8726AF45C8@MARVLAPTOP> > > The need to protect yourself here works, but it can cause > problems, which are > more visible if opener shows a major: > > 1NT 2C > 2H 2NT > 3NT > > Will opening leader always ask, "Does responder promise or deny > four spades?" > If he doesn't always ask, then he gives UI that his lead was not > dependent on > the information when he fails to ask. For example, if he doesn't > ask and leads > the SQ, partner could place him with QJT9 or QJT92 of spades, not > QJT52 (would > lead the 5 if responder has four) or QJ92 (would lead a different > suit). > Experienced players should protect themselves against missing > alerts, but not > when they will pass UI. > If the opening leader merely asks for an explanation of the auction, (as suggested by L20F1) there is little chance of passing useful UI. If dummy comes down with no major and that possibility wasn't mentioned, there may be redress if damage has resulted, even if the defenders are experts. Marv Marvin L French San Diego, CA www.marvinfrench.com From wjburrows at gmail.com Sun Nov 16 20:11:52 2008 From: wjburrows at gmail.com (Wayne Burrows) Date: Mon, 17 Nov 2008 08:11:52 +1300 Subject: [blml] [Fwd: Amsterdam] In-Reply-To: References: <001301c947d7$302631c0$0302a8c0@Mildred> <200811161200.AA16549@geller204.nifty.com> <000a01c947f8$ef0a9740$0302a8c0@Mildred> Message-ID: <2a1c3a560811161111u66638b40n3754e992e1a418be@mail.gmail.com> 2008/11/17 John (MadDog) Probst : > > ----- Original Message ----- > From: "Grattan" > To: "Bridge Laws Mailing List" > Sent: Sunday, November 16, 2008 2:38 PM > Subject: Re: [blml] [Fwd: Amsterdam] > > >> >> >> Grattan Endicott> also > ************************************ >> "We desire truth and find within >> ourselves only uncertainty." >> [Pascal] >> ''''''''''''''''''''''''''''''''''''''''''''''''''''''''''''''''''''''''''''''''''''''''''''''''' >> >> >> >> ----- Original Message ----- >> From: "Robert Geller" >> To: "Bridge Laws Mailing List" >> Sent: Sunday, November 16, 2008 12:00 PM >> Subject: Re: [blml] [Fwd: Amsterdam] >> >> >>> It seems to me that those commenters on BLML are missing an important >>> point. At this level of bridge (world championships) or at the highest >>> level >>> of bridge (major national championships) in any reasonably strong >>> country, >>> I can't imagine any pair would NOT sometimes open light in 3rd seat >>> (especially >>> NV), or that any pair at that level would feel put upon if their >>> opponents >>> opened >>> light in 3rd hand. This is just a normal part of bridge today. >>> >>> If there is some silly rule that purports to prevent this ("You didn't >>> have XX HCP. >>> Off with your head.") the rule should be changed. But "everyone" knows >>> that such silly rules are "really" intended to prevent FERTs and the >>> like, >>> and weren't meant to apply to light 3rd hand openings. Secretary-birds >>> should >>> find other things to carp about. And probably someday the rule-makers >>> should >>> redraft their rules. But it boggles my mind to think that anyone would >>> say >>> that a 3rd hand 1H opener (NV) on >>> T95 >>> KJ432 >>> J93 >>> Q6 >>> is a felonious instance of unauthoried use of a HUM system. >>> >>> Give me a break! >>> >>> -Bob >> +=+ However, disclosure regulations have closed in a little >> on this in some tournaments, including the WBF events. >> Where disclosure is required the player should comply, >> and players at this level do know what they are required to >> disclose. >> ~ Grattan ~ +=+. >> > "Of course I opened light; don't you?". > > Ye Gods, do we play bridge any more? It's GBK, not a HUM or a dingbat or a > cormorant or any other sort of artifact. Call it a psyche if you will, but > if I didn't expect it I shouldn't be at the event. This nonsense of Nigel's > really does take the biscuit for complete triviality. If my opponents didn't > open these some of the time I'd call the TD and ask him to check for > breathing and a pulse. John >> It may be General Bridge Knowledge that players open light in third hand. However there is also a special regulation that for the event occurred prevented the use of HUM methods (systems). The definition of a HUM included the statement "By partnership agreement an opening bid at the one level may be made with values a king or more below average strength". John there is no logic to your statement "It's GBK, not a HUM...". There is no reason or exception in the regulation that something that is General Bridge Knowledge cannot be a HUM. The regulators regulated that for this world championships HUMs would not be allowed and they defined a HUM to include any agreement that an opening at the one-level could be made with a king or more below average. When a pair open an otherwise ordinary 7 hcp is opened by a pair then it seems to me to be natural to question whether or not the method that they are employing is a HUM by definition or whether this is some unexpected deviation from their agreed method. The comments by one of the BBO commentators and a spectator suggest this may be a normal part of their methods. Of course this is not definitive. To determine whether in fact the pair's method was HUM or whether this was an unexpected deviation from their method one would need to question the pair. I don't believe that opening a 7hcp hand in third position is a psyche. I agree with John that this is "just bridge" for many partnerships so therefore it cannot be a "gross violation". However if this practice is a regular part of your method then by definition the regulators have deemed that you are playing a HUM. It is clear to me that opening 7hcp hands by partnership agreement is a HUM. Doing so deliberately when the regulations clearly prohibit such an opening to me is cheating. -- Wayne Burrows Palmerston North New Zealand From wjburrows at gmail.com Sun Nov 16 20:27:12 2008 From: wjburrows at gmail.com (Wayne Burrows) Date: Mon, 17 Nov 2008 08:27:12 +1300 Subject: [blml] [Fwd: Amsterdam] In-Reply-To: <200811161200.AA16549@geller204.nifty.com> References: <001301c947d7$302631c0$0302a8c0@Mildred> <200811161200.AA16549@geller204.nifty.com> Message-ID: <2a1c3a560811161127l74474080t2bd57c601ef4c2fc@mail.gmail.com> 2008/11/17 Robert Geller : > It seems to me that those commenters on BLML are missing an important > point. At this level of bridge (world championships) or at the highest level > of bridge (major national championships) in any reasonably strong country, > I can't imagine any pair would NOT sometimes open light in 3rd seat (especially > NV), or that any pair at that level would feel put upon if their opponents opened > light in 3rd hand. This is just a normal part of bridge today. > > If there is some silly rule that purports to prevent this ("You didn't have XX HCP. > Off with your head.") the rule should be changed. But "everyone" knows > that such silly rules are "really" intended to prevent FERTs and the like, > and weren't meant to apply to light 3rd hand openings. Secretary-birds should > find other things to carp about. And probably someday the rule-makers should > redraft their rules. But it boggles my mind to think that anyone would say > that a 3rd hand 1H opener (NV) on > T95 > KJ432 > J93 > Q6 > is a felonious instance of unauthoried use of a HUM system. > > Give me a break! > LAW 72 - GENERAL PRINCIPLES A. Observance of Laws Duplicate bridge tournaments should be played in strict accordance with the Laws. The chief object is to obtain a higher score than other contestants whilst complying with the lawful procedures and ethical standards set out in these laws. Even though the HUM rules are regulations not laws they are lawful regulations and therefore I would argue that law 72 applies. Bob's (et al) attitude seems to be in direct conflict with the law 72 requirement to play "in strict accordance with the laws". As such it is a fundamental violation of how the game should be played. -- Wayne Burrows Palmerston North New Zealand From darkbystry at wp.pl Sun Nov 16 21:36:07 2008 From: darkbystry at wp.pl (Bystry) Date: Sun, 16 Nov 2008 21:36:07 +0100 Subject: [blml] [Fwd: Amsterdam] References: <001301c947d7$302631c0$0302a8c0@Mildred> <200811161200.AA16549@geller204.nifty.com> Message-ID: <005501c9482a$f3f7cf20$15844c59@chello.pl> Bob, > It seems to me that those commenters on BLML are missing an important > point. At this level of bridge (world championships) or at the highest level > of bridge (major national championships) in any reasonably strong country, > I can't imagine any pair would NOT sometimes open light in 3rd seat (especially > NV), or that any pair at that level would feel put upon if their opponents opened > light in 3rd hand. This is just a normal part of bridge today. I agree. But there can be difference between "sometimes open light in 3rd seat" and "systematicly open light in 3rd seat". This difference lies in the means of disclosure. > If there is some silly rule that purports to prevent this ("You didn't have XX HCP. > Off with your head.") the rule should be changed. But "everyone" knows > that such silly rules are "really" intended to prevent FERTs and the like, > and weren't meant to apply to light 3rd hand openings. Secretary-birds should > find other things to carp about. And probably someday the rule-makers should > redraft their rules. Maybe. And maybe some people don't think such rules are silly. I'd advise caution in using phrases like "everyone", "it's obvious" and in assuming that if some members of BLML think smth is stupid, it is stupid. I'm personally against forbidding light 3rd seat openings on the basis of classificating them as HUM. But I'm more than worried about real-life level of disclosure of such agreements. > But it boggles my mind to think that anyone would say > that a 3rd hand 1H opener (NV) on > T95 > KJ432 > J93 > Q6 > is a felonious instance of unauthoried use of a HUM system. I would never open with this awful hand. It's 5-3-3-2, poor hearts spots, Queen second, lack of side entry even if hearts would be established, tolerance for every partner's lead. And definitely I don't want partner to save over opponents game even with 5 hearts. Now I'm not Helgemo, but I know many top Polish players (some top world eksperts) and many of them would never open with this hand. So if Helgemo-Helness are duely alerting such openings and disclose their agreements it's ok for me. But if they hide behind GBK, it's not. > Give me a break! > > -Bob Regards Maciej From nigelguthrie at talktalk.net Sun Nov 16 22:06:01 2008 From: nigelguthrie at talktalk.net (Nigel Guthrie) Date: Sun, 16 Nov 2008 21:06:01 +0000 Subject: [blml] [Fwd: Amsterdam] In-Reply-To: References: <001301c947d7$302631c0$0302a8c0@Mildred><200811161200.AA16549@geller204.nifty.com> <000a01c947f8$ef0a9740$0302a8c0@Mildred> Message-ID: <49208B39.9020001@talktalk.net> [John (MadDog) Probst] Ye Gods, do we play bridge any more? It's GBK, not a HUM or a dingbat or a cormorant or any other sort of artefact. Call it a psyche if you will, but if I didn't expect it I shouldn't be at the event. This nonsense of Nigel's really does take the biscuit for complete triviality. If my opponents didn't open these some of the time I'd call the TD and ask him to check for breathing and a pulse. [Nige1] A game is its rules. John. Tim, and other TDs often repudiate rules that restrict light opening bids. Do they appreciate how unfair this is to players who handicap themselves by sticking to the rules? Other groups criticise other rules.. Some ordinary players also rationalise the breaking of rules that they dislike. Should different groups of rule-breakers be treated differently? Bridge rules often seem trivial and nonsensical. IMO some are unnecessary and many are overly sophisticated and subjective.Some, like Rob and John.approve the breaking of daft rules. Others would rather campaign to change them. Our hope is that players will be more likely to comply with rules that they understand. To my mind, John's biscuit metaphor has some bite. Players deserve nice plain rules to replace those that are crackers and half-baked. From darkbystry at wp.pl Sun Nov 16 22:12:50 2008 From: darkbystry at wp.pl (Bystry) Date: Sun, 16 Nov 2008 22:12:50 +0100 Subject: [blml] [Fwd: Amsterdam] References: <000a01c947f8$ef0a9740$0302a8c0@Mildred> <200811161454.AA16558@geller204.nifty.com> Message-ID: <005d01c94830$1539a820$15844c59@chello.pl> Bob, > Grattan wrote: > >+=+ However, disclosure regulations have closed in a little > >on this in some tournaments, including the WBF events. > > Where disclosure is required the player should comply, > >and players at this level do know what they are required to > >disclose. > > ~ Grattan ~ +=+. > > No one can disagree with this, but a level of common sense based > on prevailing community standards is required. For example, 1NT > openings on a good 14 (when the agreement is described as "15-17") > or weak two openings on a 5 card suit, or light 3rd hand openings > are so ubiquitous at this level of expert play that I wouldn't regard > the failure to formally disclose such tendencies as warranting > a score adjustment. It's possible my opinion is at variance with > prevailing standards. Maybe someone should take an opinion poll > of players at the next European championship. If you frequently open 15-17 1NT having 14 points than IMO you should definitely disclose it to your opponents. I just write 14+ - 17 on my CC. It is just, it is fair to your opps. They are not forced to ask every time when it's important and to transmit UI or help you in "reading" the cards. I very rarely open Multi on 5-card suits, but if did it often I would certainly disclose it on my CC (and even disclose it when asked what is alerted 2D). I frequently make light 3rd suit openings. But "light" may mean something different to different people. My light openings are nothing that wouldn't be expected even by some better bunnies (and nevertheless I put it on my CC) but if was playing smth similar to Helgemo-Helness I would surely alert it and disclose. > The canonical example is the Kantar appeal (about 18 yrs ago?) > after the opponents bid > Opener Responder > 1NT 2C > 2D 2NT > 3NT P > and didn't formally disclose that responder might not have a 4-card major. > Amazingly Kantar won the appeal, but afterwards there was enough of > a negative reaction that it became accepted practice (at least in the > US) that expert players were expected to protect themselves in such cases > (maybe only about 5% of North American experts would play 2NT as > promising a major). I'm very happy you've stated this example. You will be maybe shocked but in Poland overwhelming majority plays Stayman as promising 4-card major. So our alert regulations state that you have to alert 2C if it may be bid without 4-card major. I firmly believe that SO's should be the ones to decide what has to be alerted and disclosed in their countries (zones). But some problems may appear at international competitions. Now you may state that US regulations are more important and WBF should incorporate them. And I may state the opposite. What we need is some compromise. And in my opinion the base of such compromise should be Full Disclosure. And the onus should be put on the bidding side not on their opponents. How to do it is not my concern, I'm a player not an official. But I'm equally important as every other member of WBF and it would be unjust if I was put in worse situation than US players because of the alert regulations or COC of WBF events. It was the US top player Larry Cohen who claimed that Polish Club is such a hard method to play against that it should be banned in WBF competitions and American Standard should be mandatory. I was firstly speechless when I read this and then laughed for 15 minutes or so. But one aspect of this case is not funny. I firmly believe that Polish Club players should thoroughly disclose any nuances of their system to those who are not familiar with it. And the same goes for other players using different systems, even American Standard. > Of course ideally Grattan is completely correct that all agreements ought to > be formally disclosed, but I believe that in practice many are so common at > a high level that they can be regarded as being "implicitly disclosed." I don't agree with this statement. There is nothing that is played by all the top players. Beside, not wanting to offend anybody, Mongolian players, even playing in World Chamionship, may have less knowledge about the standards known to Italian or American players. And I wholeheartedly believe they shouldn't be in worse situation because of simple lenience resulting in so-called experts not alerting or disclosing something. > -Bob Regards Maciej From richard.hills at immi.gov.au Sun Nov 16 23:05:53 2008 From: richard.hills at immi.gov.au (richard.hills at immi.gov.au) Date: Mon, 17 Nov 2008 09:05:53 +1100 Subject: [blml] Amsterdam (2) [SEC=UNOFFICIAL] Message-ID: Grattan Endicott: [snip] >My question about the timing stems from my understanding >that the Director was not summoned at the time as Law 16B3 >urges and the appeal has all the marks of an afterthought. > ~ Grattan ~ +=+ Law 16B3, first sentence: When a player has substantial reason to believe that an opponent who had a logical alternative has chosen an action that could have been suggested by such information, he should summon the Director when play ends**. Law 16B3, first sentence, footnote: ** it is not an infraction to call the Director earlier or later. Law 92B: The right to request or appeal a Director's ruling expires 30 minutes after the official score has been made available for inspection unless the Tournament Organizer has specified a different time period. Richard Hills: It seems to me that the Law 16B3 footnote permits a Law 16B3 summoning of the Director up to 30 minutes after the posting of the official score. Of course, a delay may not be in the interests of the summoning side (it may be more difficult for the Director to determine the facts), but they do have the legal right to such a delay. It also seems to me that the Law 92B wording "request or appeal" means that one must first seek and receive a ruling from a Director before choosing (or not) to appeal that Director's ruling. That is, it seems to me that Law 92B is predicated on the assumption that an appeal against nothing is a contradiction in terms. Best wishes Richard James Hills Recruitment Section, Level 3 Blue, workstation 15 (first on left) Department of Immigration and Citizenship Telephone: 02 6223 8453 Email: richard.hills at immi.gov.au -------------------------------------------------------------------- Important Notice: If you have received this email by mistake, please advise the sender and delete the message and attachments immediately. This email, including attachments, may contain confidential, sensitive, legally privileged and/or copyright information. Any review, retransmission, dissemination or other use of this information by persons or entities other than the intended recipient is prohibited. DIAC respects your privacy and has obligations under the Privacy Act 1988. The official departmental privacy policy can be viewed on the department's website at www.immi.gov.au. See: http://www.immi.gov.au/functional/privacy.htm --------------------------------------------------------------------- From geller at nifty.com Mon Nov 17 00:11:41 2008 From: geller at nifty.com (Robert Geller) Date: Mon, 17 Nov 2008 08:11:41 +0900 Subject: [blml] [Fwd: Amsterdam] In-Reply-To: <005d01c94830$1539a820$15844c59@chello.pl> References: <005d01c94830$1539a820$15844c59@chello.pl> Message-ID: <200811162311.AA16567@geller204.nifty.com> Bystry writes: >Now you may state that US regulations are more important and WBF should >incorporate them. And I may state the opposite. What we need is some >compromise. And in my opinion the base of such compromise should be Full >Disclosure. And the onus should be put on the bidding side not on their >opponents. How to do it is not my concern, I'm a player not an official. But >I'm equally important as every other member of WBF and it would be unjust if >I was put in worse situation than US players because of the alert >regulations or COC of WBF events. -> I am an American citizen but I've lived in Japan since 1984 and last played in an ACBL event in 1988. I am not advocating that US rules should become the international standard. Full disclosure is certainly desirable, and the WBF should be working on methods for promoting this. It would also be desirable to start a "GBK project", to try to codify what should be considered GBK at the world championship level. >It was the US top player Larry Cohen who claimed that Polish Club is such a >hard method to play against that it should be banned in WBF competitions and >American Standard should be mandatory. I was firstly speechless when I read >this and then laughed for 15 minutes or so. But one aspect of this case is >not funny. I firmly believe that Polish Club players should thoroughly >disclose any nuances of their system to those who are not familiar with it. >And the same goes for other players using different systems, even American >Standard. -> When I went to Poland to play in the Warsaw Grand Prix in 2003 we were given a 30 minute explanation of Polish Club, and we didn't have any problems defending it. -Bob ----------------------------------------------------- Robert (Bob) Geller, Tokyo, Japan geller at nifty.com From grandaeval at tiscali.co.uk Mon Nov 17 00:52:02 2008 From: grandaeval at tiscali.co.uk (Grattan) Date: Sun, 16 Nov 2008 23:52:02 -0000 Subject: [blml] Amsterdam (2) [SEC=UNOFFICIAL] References: Message-ID: <001401c94846$71a921b0$0302a8c0@Mildred> Grattan Endicott To: "Bridge Laws Mailing List" Sent: Sunday, November 16, 2008 10:05 PM Subject: Re: [blml] Amsterdam (2) [SEC=UNOFFICIAL] > > Law 16B3, first sentence: > > When a player has substantial reason to believe that an > opponent who had a logical alternative has chosen an action > that could have been suggested by such information, he > should summon the Director when play ends**. > > Law 16B3, first sentence, footnote: > > ** it is not an infraction to call the Director earlier or > later. > > Richard Hills: > > It seems to me that the Law 16B3 footnote permits a Law 16B3 > summoning of the Director up to 30 minutes after the posting > of the official score. Of course, a delay may not be in the > interests of the summoning side (it may be more difficult > for the Director to determine the facts), but they do have > the legal right to such a delay. > +=+ I agree. We do not want a request for a ruling to be ruled out of time because it is not made at the time the law indicates to be appropriate. However, the use of 'should' in this law does establish that the desirable time for the protest to be entered is when play ends. The Director will seek to know why he was not summoned at that time; it may indicate that the player did not feel the tray was so long delayed or perhaps that he did not think his screenmate's action was influenced. Prompting by his partner, who observed the pause on his side of the screen, may perhaps have persuaded him that perhaps the tray did come back more slowly than he had realized. The Director has questions to explore and his involvement is called for. ~ Grattan ~ +=+ From nigelguthrie at talktalk.net Mon Nov 17 01:06:54 2008 From: nigelguthrie at talktalk.net (Nigel Guthrie) Date: Mon, 17 Nov 2008 00:06:54 +0000 Subject: [blml] [Fwd: Amsterdam] In-Reply-To: <200811161342.AA16553@geller204.nifty.com> References: <4920195A.1070509@talktalk.net> <200811161342.AA16553@geller204.nifty.com> Message-ID: <4920B59E.8080202@talktalk.net> [Robert Geller] But the system regulations contain many silly provisions in terms of high card points (4-3-2-1) or the like that are widely ignored. The situation is like having a speed limit of 100 km/hr when everyone on the road is doing 110. {Nigel] I like Bob's analogy. If you see others getting away with breaking the rules, then you are tempted to catch up. If you are one of the few, who still comply with the rules, then you are left behind. In a different BLML topic, I wrote that many players break Bridge Laws that they dislike. BLMLers claimed that I was accusing players of cheating. But I hope I've never met a bridge cheat. I explained that few players understand the laws. The power of rationalisation is under-rated. With no effective deterrent, players can become careless. IMO, law-breakers would be shocked and hurt if accused of cheating. Other BLMLers claimed that law-breakers are a tiny minority. But I agree with Victor Mollo and Robert Geller: The moral climate of the Bridge microcosm reflects that of the macrocosm. A realist will agree with Bob that, in society at large, some falsify tax-returns, break marriage vows, and exceed speed-limits. IMO, Bridge would be fairer and more fun if the laws were simpler, clearer, more objective, more deterrent, and more complete. From geller at nifty.com Mon Nov 17 01:44:56 2008 From: geller at nifty.com (Robert Geller) Date: Mon, 17 Nov 2008 09:44:56 +0900 Subject: [blml] [Fwd: Amsterdam] In-Reply-To: <4920B59E.8080202@talktalk.net> References: <4920B59E.8080202@talktalk.net> Message-ID: <200811170044.AA16570@geller204.nifty.com> Nigel Guthrie writes: >[Robert Geller] >But the system regulations contain many silly provisions in terms of >high card points (4-3-2-1) or the like that are widely ignored. The >situation is like having a speed limit of 100 km/hr when everyone on the >road is doing 110. > >{Nigel] >I like Bob's analogy. If you see others getting away with breaking the rules, then you are tempted to catch up. If you are one of the few, who still comply with the rules, then you are left behind. -> The real rules are what everyone does. If everyone else on the road is going 110, someone who insists on driving 100 is holding up traffic and more likely to cause an accident than someone driving 110. >In a different BLML topic, I wrote that many players break Bridge Laws that they dislike. BLMLers claimed that I was accusing players of cheating. But I hope I've never met a bridge cheat. I explained that few players understand the laws. The power of rationalisation is under-rated. With no effective deterrent, players can become careless. IMO, law-breakers would be shocked and hurt if accused of cheating. -> Can't we reserve "lawbreakers" for heinous offenses and use a different term for less than complete and literal compliance with system regulations (which often were written by "old farts" who don't understand modern bridge. Let's not trivializing "lawbreaking" and "cheating" to apply to someone who has one or two or three fewer high card points than an old fart considers to be required under some stupid regulation ("A light 3rd hand opener is HUM. Off with Helgemo's head!). (Full disclosure: I suppose I'm an ?old fart" ?yself, inasmuch as I'm a board member of our NCBO.) >Other BLMLers claimed that law-breakers are a tiny minority. But I agree with Victor Mollo and Robert Geller: The moral climate of the Bridge microcosm reflects that of the macrocosm. A realist will agree with Bob that, in society at large, some falsify tax-returns, break marriage vows, and exceed speed-limits. -> If jaywalkers (illegally crossing the street at a red light) and speeders are "lawbreakers then 98% of the population are probably lawbreakers. But there are very few murderers at large. Let's not lump all these offenses into one category. >IMO, Bridge would be fairer and more fun if the laws were simpler, clearer, more objective, more deterrent, and more complete. Bridge is a GAME, for crying out loud. Rather than talking about deterrents let's talk about how to get better system regs. If helgemo, Hamman, Meckwell, Lauria, Ballicki, etc. (i.e. top-level players) could be cajoled into forming a copy to write the regs, then more sensible regs that would be respected and followed would result. -Bob ----------------------------------------------------- Robert (Bob) Geller, Tokyo, Japan geller at nifty.com From darkbystry at wp.pl Mon Nov 17 02:55:20 2008 From: darkbystry at wp.pl (Bystry) Date: Mon, 17 Nov 2008 02:55:20 +0100 Subject: [blml] [Fwd: Amsterdam] References: <4920195A.1070509@talktalk.net><200811161342.AA16553@geller204.nifty.com> <4920B59E.8080202@talktalk.net> Message-ID: <000701c94857$8c51de60$15844c59@chello.pl> Hi, > [Robert Geller] > But the system regulations contain many silly provisions in terms of > high card points (4-3-2-1) or the like that are widely ignored. The > situation is like having a speed limit of 100 km/hr when everyone on the > road is doing 110. > > {Nigel] > I like Bob's analogy. If you see others getting away with breaking the rules, then you are tempted to catch up. If you are one of the few, who still comply with the rules, then you are left behind. Yes, it's a clever and rational statement. But how would you try to solve such problem? To change speed limit to 110km/h? But in spite of the fact that I'm not on lawmaking side but on lawobeying side, I am far away from claiming that lawmakers were dumb or had some malicious motives when deciding which laws would be better. In Polish cities speed limit is 50km/h (was changed from 60km/h). Many drivers complain about it and don't comply to this law. They are sometimes right in a sense of pure logic. When it is 16 p.m. 50km/h looks like rational idea. When it is 2 a.m. the roads are empty and you could rationally drive at higher speed and risk less than in the first case. But to change this illogic we'd have to complicate laws. We would have to introduce many "ifs", "whens", "unlesses" and similar. We would make it hard for officials to judge. We would make it hard for drivers to understand. We would make it similar to some Bridge Laws, which deficiencies you are consequently and ineffectivly outpointing. The laws are for people. But it doesn't mean that they should be written as majority (or worse, some loud minority) would want them to be. I've heard arguments that every driver should decide himself what is his safe speed. And such arguments aren't wholly illogical. But they assume that every driver is clever, cautious and sane (it's obviously not true) and they don't take into consideration rights and fears of pedestrians or other drivers. So I like Bob's analogy too. But of course not in the way he intented it to be. I say that every driver who exceeds speed limits should be penalised and every bridge player who breaks the rules should be equally penalised. And stating some rules in terms of HCP is for me quite a good idea because it is something nearly every bridge player understands. Other methods (e.g. LP, SP, LTC, QT, PT, Zar points) are less common and it would be necessary to overcomplicate the rules to take every such method into account. > In a different BLML topic, I wrote that many players break Bridge Laws that they dislike. BLMLers claimed that I was accusing players of cheating. But I hope I've never met a bridge cheat. I explained that few players understand the laws. The power of rationalisation is under-rated. With no effective deterrent, players can become careless. IMO, law-breakers would be shocked and hurt if accused of cheating. I'm not aware of "many bridge players breaking BL that they dislike". But I saw many bridge players breaking BL because they don't know or don't understand them. And I think it's much more important to make some rules simpler and to educate bridge community about them than to satisfy a group of unhappy people who want to psych freely/open on 5 HCP/use any conventions or who don't want to pay for their mistakes (like revokes, mechanical errors). > Other BLMLers claimed that law-breakers are a tiny minority. But I agree with Victor Mollo and Robert Geller: The moral climate of the Bridge microcosm reflects that of the macrocosm. A realist will agree with Bob that, in society at large, some falsify tax-returns, break marriage vows, and exceed speed-limits. Yes, that's also my view. But it's officials' duty to catch and penalize them, not to change the laws to comfort them. > IMO, Bridge would be fairer and more fun if the laws were simpler, clearer, more objective, more deterrent, and more complete. Yes. You're writing that for some years. But it's not easy to write simple, clear and complete laws in such a complicated game as Bridge. What's even worse some influencing groups (or even nations and zones) have completely different ideas what would be simple or objective. E.g. my view (and I know many players who agree with me) is that we should adopt simple rules - a card played is a card played, a call made is a call made and so on. For revoke - automatic penalty of 2 tricks (and adjusted score if such penalty doesn't do justice for NOS with _additional_ penalty of 2 tricks). No "equity" rulings like 12C3 - bad score should be a reminder for OS to respect their obligations and NOS should be given due compensation for having been deprived of normal play on the board and chances for obtaining a very good score. But this view is completely opposed by today's authorities and majority of BLMLers. So I have to live with it and I'm trying to believe they think it will be better for Bridge. My prediction is not so optimistic, maybe such policy will minimally increase overall number of bridge players but it will drastically increase the number of law-breaking players. Why should they bother to comply to UI regulations if they get their "equity" after the infraction? PP won't be given in the whole scope of mid-level and beginner bridge. Why should they bother to comply to disclosure rules? Why should they be careful when making their calls or playing their cards? No, current official trend is to satisfy people who don't like being penalized for their inattention to game and it's rules. And people like me, who bend backwards to obey them strictly, are put in the worse position. But I won't stop playing Bridge and will obey BL as they are written - I love this game in spite of it's deficiencies. That's why our lawmakers don't care about people like me. Regards Maciej From nigelguthrie at talktalk.net Mon Nov 17 03:09:49 2008 From: nigelguthrie at talktalk.net (Nigel Guthrie) Date: Mon, 17 Nov 2008 02:09:49 +0000 Subject: [blml] [Fwd: Amsterdam] In-Reply-To: <200811170044.AA16570@geller204.nifty.com> References: <4920B59E.8080202@talktalk.net> <200811170044.AA16570@geller204.nifty.com> Message-ID: <4920D26D.1080200@talktalk.net> [Robert Geller] The real rules are what everyone does. If everyone else on the road is going 110, someone who insists on driving 100 is holding up traffic and more likely to cause an accident than someone driving 110. [Nigel] This may be the nub of our disagreement: IMO, in the event of an accident, law-enforcers should tend to blame law-breakers -- or law-makers -- rather than law-abiders. From john at asimere.com Mon Nov 17 05:30:21 2008 From: john at asimere.com (John (MadDog) Probst) Date: Mon, 17 Nov 2008 04:30:21 -0000 Subject: [blml] [Fwd: Amsterdam] References: <001301c947d7$302631c0$0302a8c0@Mildred><200811161200.AA16549@geller204.nifty.com> <000a01c947f8$ef0a9740$0302a8c0@Mildred> <49208B39.9020001@talktalk.net> Message-ID: <067FB1FC6E994CEB9CAAC274D24F7C2D@JOHN> ----- Original Message ----- From: "Nigel Guthrie" To: "Bridge Laws Mailing List" Sent: Sunday, November 16, 2008 9:06 PM Subject: Re: [blml] [Fwd: Amsterdam] > [John (MadDog) Probst] > Ye Gods, do we play bridge any more? It's GBK, not a HUM or a dingbat or > a cormorant or any other sort of artefact. Call it a psyche if you will, > but if I didn't expect it I shouldn't be at the event. This nonsense of > Nigel's really does take the biscuit for complete triviality. If my > opponents didn't open these some of the time I'd call the TD and ask him > to check for breathing and a pulse. > > [Nige1] > A game is its rules. > > John. Tim, and other TDs often repudiate rules that restrict light > opening bids. Do they appreciate how unfair this is to players who > handicap themselves by sticking to the rules? Other groups criticise > other rules.. Some ordinary players also rationalise the breaking of > rules that they dislike. Should different groups of rule-breakers be > treated differently? > > Bridge rules often seem trivial and nonsensical. IMO some are > unnecessary and many are overly sophisticated and subjective.Some, like > Rob and John.approve the breaking of daft rules. Others would rather > campaign to change them. Our hope is that players will be more likely > to comply with rules that they understand. To my mind, John's biscuit > metaphor has some bite. Players deserve nice plain rules to replace > those that are crackers and half-baked. > Nice one Nigel! I'm perfectly comfortable with booking this as a psyche (Let's assume I'd ascertained that we don't have felonious agreements and this crumby hand is not by agreement opened). As the spectator said "They do that in Norway". Quite possibly true. They do it in England too. They don't do it in America. The information content of the spectator's comment is not an indictment of the partnership method, just a comment about an ethnic style. > > _______________________________________________ > blml mailing list > blml at amsterdamned.org > http://www.amsterdamned.org/mailman/listinfo/blml From wjburrows at gmail.com Mon Nov 17 06:39:22 2008 From: wjburrows at gmail.com (Wayne Burrows) Date: Mon, 17 Nov 2008 18:39:22 +1300 Subject: [blml] [Fwd: Amsterdam] In-Reply-To: <067FB1FC6E994CEB9CAAC274D24F7C2D@JOHN> References: <001301c947d7$302631c0$0302a8c0@Mildred> <200811161200.AA16549@geller204.nifty.com> <000a01c947f8$ef0a9740$0302a8c0@Mildred> <49208B39.9020001@talktalk.net> <067FB1FC6E994CEB9CAAC274D24F7C2D@JOHN> Message-ID: <2a1c3a560811162139i75ae4c30vb312182cd88de91c@mail.gmail.com> 2008/11/17 John (MadDog) Probst : > > ----- Original Message ----- > From: "Nigel Guthrie" > To: "Bridge Laws Mailing List" > Sent: Sunday, November 16, 2008 9:06 PM > Subject: Re: [blml] [Fwd: Amsterdam] > > >> [John (MadDog) Probst] >> Ye Gods, do we play bridge any more? It's GBK, not a HUM or a dingbat or >> a cormorant or any other sort of artefact. Call it a psyche if you will, >> but if I didn't expect it I shouldn't be at the event. This nonsense of >> Nigel's really does take the biscuit for complete triviality. If my >> opponents didn't open these some of the time I'd call the TD and ask him >> to check for breathing and a pulse. >> >> [Nige1] >> A game is its rules. >> >> John. Tim, and other TDs often repudiate rules that restrict light >> opening bids. Do they appreciate how unfair this is to players who >> handicap themselves by sticking to the rules? Other groups criticise >> other rules.. Some ordinary players also rationalise the breaking of >> rules that they dislike. Should different groups of rule-breakers be >> treated differently? >> >> Bridge rules often seem trivial and nonsensical. IMO some are >> unnecessary and many are overly sophisticated and subjective.Some, like >> Rob and John.approve the breaking of daft rules. Others would rather >> campaign to change them. Our hope is that players will be more likely >> to comply with rules that they understand. To my mind, John's biscuit >> metaphor has some bite. Players deserve nice plain rules to replace >> those that are crackers and half-baked. >> > > Nice one Nigel! I'm perfectly comfortable with booking this as a psyche > (Let's assume I'd ascertained that we don't have felonious agreements and > this crumby hand is not by agreement opened). As the spectator said "They do > that in Norway". Quite possibly true. They do it in England too. They don't > do it in America. The information content of the spectator's comment is not > an indictment of the partnership method, just a comment about an ethnic > style. >> That is exactly the question. Is this opening a departure from their methods or is it part of their methods? -- Wayne Burrows Palmerston North New Zealand From darkbystry at wp.pl Mon Nov 17 06:44:04 2008 From: darkbystry at wp.pl (Bystry) Date: Mon, 17 Nov 2008 06:44:04 +0100 Subject: [blml] [Fwd: Amsterdam] References: <001301c947d7$302631c0$0302a8c0@Mildred><200811161200.AA16549@geller204.nifty.com> <000a01c947f8$ef0a9740$0302a8c0@Mildred><49208B39.9020001@talktalk.net> <067FB1FC6E994CEB9CAAC274D24F7C2D@JOHN> Message-ID: <000501c94877$80cb8760$15844c59@chello.pl> John, > Nice one Nigel! I'm perfectly comfortable with booking this as a psyche > (Let's assume I'd ascertained that we don't have felonious agreements and > this crumby hand is not by agreement opened). As the spectator said "They do > that in Norway". Quite possibly true. They do it in England too. They don't > do it in America. The information content of the spectator's comment is not > an indictment of the partnership method, just a comment about an ethnic > style. I thought this case is precisely about CPU. I don't mind if you systematicly open with 3 HCP against me. What's more, I would be delighted if you did often. I just want it to be disclosed so I could use my best judgment to count the hands when your partner passes our 3NT with 10 HCP ;-) Regards Maciej From richard.hills at immi.gov.au Mon Nov 17 07:02:46 2008 From: richard.hills at immi.gov.au (richard.hills at immi.gov.au) Date: Mon, 17 Nov 2008 17:02:46 +1100 Subject: [blml] Netiquette (was Mr Lincoln) [SEC=UNOFFICIAL] In-Reply-To: Message-ID: W.S. Gilbert, Iolanthe, Lord Chancellor's song (first verse): "The Law is the true embodiment Of everything that's excellent. It has no kind of fault or flaw, And I, my Lords, embody the Law. The constitutional guardian I Of pretty young Wards in Chancery, All very agreeable girls -- and none Are over the age of twenty-one. A pleasant occupation for A rather susceptible Chancellor!" Hans van Staveren suggested: [snip] >But apart from this, it is my strong feeling that the way >this is handled here on BLML is now way beyond what is still >acceptable in a polite society. > >Something happened in Amsterdam, there are unhappy people. >An enquiry will be made, a result will be forthcoming. It is >fitting to accept that and not make things worse. > >And no, I was not involved. W.S. Gilbert, Iolanthe, Lord Chancellor dialogue (excerpt): "The feelings of a Lord Chancellor who is in love with a Ward of Court are not to be envied. What is his position? Can he give his own consent to his own marriage with his own Ward? Can he marry his own Ward without his own consent? And if he marries his own Ward without his own consent, can he commit himself for contempt of his own Court? And if he commit himself for contempt of his own Court, can he appear by counsel before himself, to move for arrest of his own judgement? Ah, my Lords, it is indeed painful to have to sit upon a woolsack which is stuffed with such thorns as these!" Richard Hills quibbles: Normally Hans van Staveren would be 100% correct; it would be a breach of blml netiquette to discuss the merits of an unresolved appeal. But in this case the focus of blml debate is not on the actual deal, but rather untangling a Lawful thread from a cat's cradle of similar complexity to that facing Gilbert's Lord Chancellor. Nor do I think that censorship will stop things getting worse. In my opinion, the lack of blml censorship has reduced the chance of this particular error being once more repeated. (Assiduous readers of ACBL appeals casebooks will recall at least one case when the Director failed to make a ruling -- although in that case it was a Palinesque Director who did not realise that a ruling was being requested.) Best wishes Richard James Hills Recruitment Section, Level 3 Blue, workstation 15 (first on left) Department of Immigration and Citizenship Telephone: 02 6223 8453 Email: richard.hills at immi.gov.au -------------------------------------------------------------------- Important Notice: If you have received this email by mistake, please advise the sender and delete the message and attachments immediately. This email, including attachments, may contain confidential, sensitive, legally privileged and/or copyright information. Any review, retransmission, dissemination or other use of this information by persons or entities other than the intended recipient is prohibited. DIAC respects your privacy and has obligations under the Privacy Act 1988. The official departmental privacy policy can be viewed on the department's website at www.immi.gov.au. See: http://www.immi.gov.au/functional/privacy.htm --------------------------------------------------------------------- From geller at nifty.com Mon Nov 17 07:09:24 2008 From: geller at nifty.com (Robert Geller) Date: Mon, 17 Nov 2008 15:09:24 +0900 Subject: [blml] "GBK" In-Reply-To: References: Message-ID: <200811170609.AA16585@geller204.nifty.com> Dear BLMLers, John Probst and several other members sometimes use "GBK" as an abbreviation for "general bridge knowledge," but it doesn't appear in the following list. Perhaps it should be added. Any opinions? -Bob Henk Uijterwaal ????????: >(Automated, regular posting) > > Usenet Bridge Abbreviations > >ABF Australian Bridge Federation >AC Appeals committee >ACBL American Contract Bridge League >AI Authorised information >ArtAS Artificial adjusted score >AssAS Assigned adjusted score >ATF Across-the-field [matchpointing] >ATTNA Appeal to the National Authority >BBL British Bridge League [now defunct] >BGB Bridge Great Britain >BIT Break in Tempo >BLML Bridge-laws mailing list >BoD Board of directors [ACBL] >BoG Board of governors [ACBL] >BOOT Bid-Out-Of-Turn >CD Convention Disruption >C&E Conduct and ethics [often hearings] >CC Convention card >CHO Center Hand Opponent [ie partner] >CoC Conditions of contest >COOT Call-Out-Of-Turn >CoP Code of practice >CPU Concealed partnership understanding >CTD Chief Tournament director >DBF Danish Bridge Federation >DIC Director in charge >DP Disciplinary penalty >EBL European Bridge League >EBU English Bridge Union >EHAA Every Hand an Adventure [a system] >F2F Face-to-face [to distinguish from Online bridge] >FNJ Fit-Non-Jump (A non-jump bid in a new suit that implies a fit > for partner's suit). >FOLOOT Faced Opening-Lead-Out-Of-Turn >FSF Fourth Suit Forcing >GCC General Convention Chart [ACBL] >HUM Highly Unusual Method >IB Insufficient Bid >IBLF International Bridge Laws Forum >LA Logical alternative >L&EC Laws & Ethics Committee [English, Welsh or Scottish] >LHO Left hand Opponent >Lnn Law number nn >LOL Little old lady [may be of either sex] >LOOT Lead-Out-Of-Turn >MB Misbid >ME Misexplanation >MI Misinformation >MPC Major penalty card >mPC Minor penalty card >MSC Master Solvers' Club [The Bridge World] >NA National Authority >NABC ACBL North American Bridge Championships >NBB Nederlandse Bridge Bond [Dutch Bridge League] >NBO National Bridge organisation >NCBO National Contract Bridge organisation >NIBU Northern Ireland Bridge Union >NO Non-offender >NOs Non-offenders >NOS Non-offending side >OBM Old Black Magic >OBOOT Opening-Bid-Out-Of-Turn >OKB OKBridge >OLB Online bridge [to distinguish from Face-to-face bridge] >OLOOT Opening-Lead-Out-Of-Turn >OOT Out-Of-Turn >Os Offenders >OS Offending side >pd Partner >PLOOT Play-Out-Of-Turn >POOT Pass-Out-Of-Turn >PP Procedural Penalty >PH Passed Hand >RA Regulating Authority >RGB rec.games.bridge [newsgroup] >RGBO rec.games.bridge.okbridge [newsgroup] >RHO Right Hand Opponent >RLB Real Life Bridge [to distinguish from Online bridge] >RoC Rule of coincidence >RoW Rest of World [apart from North America] >RTFLB Read the [fabulous] Law book! >SAYC Standard American Yellow Card >SBU Scottish Bridge Union >SO Sponsoring organisation >TBW The Bridge World [magazine] >TD Tournament director >TDic Tournament director in charge >TFLB The [fabulous] Law book! >UI Unauthorised information >UPH UnPassed Hand >WBF World Bridge Federation >WBFLC WBF Laws Committee >WBU Welsh Bridge Union >YC Young Chelsea >ZO Zonal organisation >ZT Zero Tolerance [for unacceptable behaviour] > >Hand diagrams: > >3m 3C or 3D [minor] >3M 3H or 3S [Major] >..3H 3H after a hesitation >3H! 3H alerted > >Cards and bids: > >H3 A card (3 of hearts) >3H A bid (3 hearts. > > > The above may also be found on David Stevenson's Bridgepage at > > http://blakjak.com/usenet_br.htm > > > >_______________________________________________ >blml mailing list >blml at amsterdamned.org >http://www.amsterdamned.org/mailman/listinfo/blml ----------------------------------------------------- Robert (Bob) Geller, Tokyo, Japan geller at nifty.com From richard.hills at immi.gov.au Mon Nov 17 07:34:33 2008 From: richard.hills at immi.gov.au (richard.hills at immi.gov.au) Date: Mon, 17 Nov 2008 17:34:33 +1100 Subject: [blml] Law 83 (was ...damn) [SEC=UNOFFICIAL] In-Reply-To: <491D63D0.9080909@talktalk.net> Message-ID: Nigel Guthrie suggested: >I hope the new 2018 rule-book says that, in any event that >allows appeals - Richard Hills quibbles: Under the current Lawbook, **all** events allow appeals. WBF Laws Committee minutes, October 2008, page 5: "LAW 93 -- The power of modification given in this law is a right to modify the procedure in dealing with appeals. It does not extend to overriding the rights of contestants to appeal under Law 92A. If the Regulating Authority makes no arrangement for an appeal to be heard (see Law 80B2(k)) the Chief Director shall hear and rule upon it under Law 93A." Richard Hills asks: Or is Nigel proposing a 2018 law change to limit appeals? Nigel Guthrie suggested: >- The director should appoint at least one experienced >player as appeals adviser. [snip] Richard Hills quibbles: In a lot of small clubs, the playing Director is the only experienced player. Would Nigel permit a Director -- a la W.S. Gilbert's Pooh-Bah -- to appoint herself as both the appeals advisor and also the appeals committee (Law 93A)? W.S. Gilbert, The Mikado: KO. Why should I kill you when making an affidavit that you've been executed will do just as well? Here are plenty of witnesses -- the Lord Chief Justice, Lord High Admiral, Commander-in-Chief, Secretary of State for the Home Department, First Lord of the Treasury, and Chief Commissioner of Police. NANK. But where are they? KO. There they are. They'll all swear to it -- won't you? (To Pooh-Bah.) POOH. Am I to understand that all of us high Officers of State are required to perjure ourselves to ensure your safety? KO. Why not! You'll be grossly insulted, as usual. POOH. Will the insult be cash down, or at a date? KO. It will be a ready-money transaction. Best wishes Richard James Hills Recruitment Section, Level 3 Blue, workstation 15 (first on left) Department of Immigration and Citizenship Telephone: 02 6223 8453 Email: richard.hills at immi.gov.au -------------------------------------------------------------------- Important Notice: If you have received this email by mistake, please advise the sender and delete the message and attachments immediately. This email, including attachments, may contain confidential, sensitive, legally privileged and/or copyright information. Any review, retransmission, dissemination or other use of this information by persons or entities other than the intended recipient is prohibited. DIAC respects your privacy and has obligations under the Privacy Act 1988. The official departmental privacy policy can be viewed on the department's website at www.immi.gov.au. See: http://www.immi.gov.au/functional/privacy.htm --------------------------------------------------------------------- From geller at nifty.com Mon Nov 17 07:36:48 2008 From: geller at nifty.com (Robert Geller) Date: Mon, 17 Nov 2008 15:36:48 +0900 Subject: [blml] Netiquette (was Mr Lincoln) [SEC=UNOFFICIAL] In-Reply-To: References: Message-ID: <200811170636.AA16586@geller204.nifty.com> This whole business is very strange. The name of, say, the referee (and assistant referees) for football matches (including the finals of major Championships) is always made public. If they screw up a decision they get some flak. That's part of the job. I can't understand why no one has revealed the name of the DIC (and assistant directors) involved in the Amsterdam decision. Surely that isn't a secret. If it is, something is really wrong here. Isn't it normal these days to publicize such information on the tourney home page? -Bob richard.hills at immi.gov.au?wrote: >W.S. Gilbert, Iolanthe, Lord Chancellor's song (first verse): > >"The Law is the true embodiment >Of everything that's excellent. >It has no kind of fault or flaw, >And I, my Lords, embody the Law. >The constitutional guardian I >Of pretty young Wards in Chancery, >All very agreeable girls -- and none >Are over the age of twenty-one. > A pleasant occupation for > A rather susceptible Chancellor!" > >Hans van Staveren suggested: > >[snip] > >>But apart from this, it is my strong feeling that the way >>this is handled here on BLML is now way beyond what is still >>acceptable in a polite society. >> >>Something happened in Amsterdam, there are unhappy people. >>An enquiry will be made, a result will be forthcoming. It is >>fitting to accept that and not make things worse. >> >>And no, I was not involved. > >W.S. Gilbert, Iolanthe, Lord Chancellor dialogue (excerpt): > >"The feelings of a Lord Chancellor who is in love with a Ward >of Court are not to be envied. What is his position? Can he >give his own consent to his own marriage with his own Ward? >Can he marry his own Ward without his own consent? And if he >marries his own Ward without his own consent, can he commit >himself for contempt of his own Court? And if he commit >himself for contempt of his own Court, can he appear by >counsel before himself, to move for arrest of his own >judgement? Ah, my Lords, it is indeed painful to have to sit >upon a woolsack which is stuffed with such thorns as these!" > >Richard Hills quibbles: > >Normally Hans van Staveren would be 100% correct; it would be >a breach of blml netiquette to discuss the merits of an >unresolved appeal. > >But in this case the focus of blml debate is not on the actual >deal, but rather untangling a Lawful thread from a cat's >cradle of similar complexity to that facing Gilbert's Lord >Chancellor. > >Nor do I think that censorship will stop things getting worse. >In my opinion, the lack of blml censorship has reduced the >chance of this particular error being once more repeated. > >(Assiduous readers of ACBL appeals casebooks will recall at >least one case when the Director failed to make a ruling -- >although in that case it was a Palinesque Director who did not >realise that a ruling was being requested.) > > >Best wishes > >Richard James Hills >Recruitment Section, Level 3 Blue, workstation 15 (first on left) >Department of Immigration and Citizenship >Telephone: 02 6223 8453 >Email: richard.hills at immi.gov.au > > > >-------------------------------------------------------------------- >Important Notice: If you have received this email by mistake, please advise the sender and delete the message and attachments immediately. This email, including attachments, may contain confidential, sensitive, legally privileged and/or copyright information. Any review, retransmission, dissemination or other use of this information by persons or entities other than the intended recipient is prohibited. DIAC respects your privacy and has obligations under the Privacy Act 1988. The official departmental privacy policy can be viewed on the department's website at www.immi.gov.au. See: http://www.immi.gov.au/functional/privacy.htm > >--------------------------------------------------------------------- > > >_______________________________________________ >blml mailing list >blml at amsterdamned.org >http://www.amsterdamned.org/mailman/listinfo/blml ----------------------------------------------------- Robert (Bob) Geller, Tokyo, Japan geller at nifty.com From richard.hills at immi.gov.au Mon Nov 17 07:51:25 2008 From: richard.hills at immi.gov.au (richard.hills at immi.gov.au) Date: Mon, 17 Nov 2008 17:51:25 +1100 Subject: [blml] A player's comment [SEC=UNOFFICIAL] In-Reply-To: Message-ID: Robert Frick asserted: >Actually, to describe 2S as showing heart support, when you >know that isn't what your partner intended and probably >doesn't have, is deliberately giving a misleading >explanation. Richard Hills quibbles: Deliberately misdescribing partner's cards, but not necessarily deliberately misdescribing pre-existing mutual partnership agreement. Law 75C - Mistaken Call - second sentence: "Here there is no infraction of Law, since East-West did receive an accurate description of the North-South agreement; they have no claim to an accurate description of the North- South hands." Robert Frick asserted: >And if you are looking for an honest man, my partner's >explanation in this situation was "If she took my 2 Di bid >to be multi, then it shows heart support; if she didn't, >then it shows spades." Richard Hills quibbles: Describing 2S as simply showing heart support is not necessarily dishonest. The honesty depends upon partner's Law 40C1 frequency of forgetting the system. If partner forgets the system sufficiently often to create a two-way understanding, then a two-way description of 2S is correct. Otherwise a one-way description is honest, even -- or indeed especially -- if one has unauthorised information from a lack of alert that partner has made a very rare forget of the system. Best wishes Richard James Hills Recruitment Section, Level 3 Blue, workstation 15 (first on left) Department of Immigration and Citizenship Telephone: 02 6223 8453 Email: richard.hills at immi.gov.au -------------------------------------------------------------------- Important Notice: If you have received this email by mistake, please advise the sender and delete the message and attachments immediately. This email, including attachments, may contain confidential, sensitive, legally privileged and/or copyright information. Any review, retransmission, dissemination or other use of this information by persons or entities other than the intended recipient is prohibited. DIAC respects your privacy and has obligations under the Privacy Act 1988. The official departmental privacy policy can be viewed on the department's website at www.immi.gov.au. See: http://www.immi.gov.au/functional/privacy.htm --------------------------------------------------------------------- From richard.hills at immi.gov.au Mon Nov 17 08:06:43 2008 From: richard.hills at immi.gov.au (richard.hills at immi.gov.au) Date: Mon, 17 Nov 2008 18:06:43 +1100 Subject: [blml] "GBK" [SEC=UNOFFICIAL] In-Reply-To: <200811170609.AA16585@geller204.nifty.com> Message-ID: Robert Geller: >Dear BLMLers, > >John Probst and several other members sometimes use "GBK" as >an abbreviation for "general bridge knowledge," but it doesn't >appear in the following list. Perhaps it should be added. > >Any opinions? > >-Bob Richard Hills: In my opinion, definitely not!!! So-called "general bridge knowledge" is a misnomer -- it should be "general bridge inferences". And because it is a misnomer, it has been used to justify both misinformation and also illegal understandings. Law 40B6(a): "...but he need not disclose **inferences** drawn from his knowledge and experience of matters generally known to bridge players." Best wishes Richard James Hills Recruitment Section, Level 3 Blue, workstation 15 (first on left) Department of Immigration and Citizenship Telephone: 02 6223 8453 Email: richard.hills at immi.gov.au -------------------------------------------------------------------- Important Notice: If you have received this email by mistake, please advise the sender and delete the message and attachments immediately. This email, including attachments, may contain confidential, sensitive, legally privileged and/or copyright information. Any review, retransmission, dissemination or other use of this information by persons or entities other than the intended recipient is prohibited. DIAC respects your privacy and has obligations under the Privacy Act 1988. The official departmental privacy policy can be viewed on the department's website at www.immi.gov.au. See: http://www.immi.gov.au/functional/privacy.htm --------------------------------------------------------------------- From Hermandw at skynet.be Mon Nov 17 09:22:26 2008 From: Hermandw at skynet.be (Herman De Wael) Date: Mon, 17 Nov 2008 09:22:26 +0100 Subject: [blml] A player's comment In-Reply-To: <9895762B-3BCE-45E6-A3A3-8DE6F689AFCC@starpower.net> References: <491C27D9.4010501@skynet.be> <2a1c3a560811131034h6b48945bl9cc6d238b09159b9@mail.gmail.com> <9895762B-3BCE-45E6-A3A3-8DE6F689AFCC@starpower.net> Message-ID: <492129C2.2000107@skynet.be> Eric Landau wrote: > On Nov 13, 2008, at 1:34 PM, Wayne Burrows wrote: > >> 2008/11/14 Herman De Wael : >>> Heard yesterday from a (top-level) player in Belgium. >>> >>> She was playing in a club tournament, and the bidding went: >>> >>> 1NT - 2Di - pass - 2Sp >>> pass- 3NT - pass - 4Di >>> pass- 4Sp >>> (there may have been some doubles in there) >>> >>> 2Di was a multi, and the player had long hearts. It was not alerted. >>> 2Sp was obviously intended as natural, and the player had long >>> spades. >>> In the Multi-system, 2Sp shows heart support. >>> 2Sp was alerted. >>> >>> Since 4Sp went lots down, there was no TD call any more. >>> But what she complained most about was the alert on 2Sp. >>> She considered this most unethical, on an equal footing with >>> saying "2Di >>> was Multi". >>> >>> I told her that I agreed with her, but that most of you would act the >>> same way. >>> I shall not repeat the words she used for all of you. >> We cannot be blamed for her ignorance of the law. > > Or her lack of logical thinking. She asserts that "saying '2D was > Multi'" would be "most unethical", which means that it must be > ethically preferable (or at least equivalent) to say "2D was not > Multi". Given that 2D *was* in fact Multi, I don't see how she could > sensibly defend that position. > ehm ??? pardon ????? Law 20F5a ??? Are we about to go so far in this stup?d interpretation that the written law will also be disregarded now? Herman. > > Eric Landau From Hermandw at skynet.be Mon Nov 17 09:24:22 2008 From: Hermandw at skynet.be (Herman De Wael) Date: Mon, 17 Nov 2008 09:24:22 +0100 Subject: [blml] A player's comment [SEC=UNOFFICIAL] In-Reply-To: References: Message-ID: <49212A36.3030103@skynet.be> richard.hills at immi.gov.au wrote: > Robert Frick asserted: > >> Actually, to describe 2S as showing heart support, when you >> know that isn't what your partner intended and probably >> doesn't have, is deliberately giving a misleading >> explanation. > > Richard Hills quibbles: > Don't quibble, Richard - saying a player has hearts when you know he has spades is _misleading_. There is no doubt about that and no need to drag in laws we all know. Herman. From Hermandw at skynet.be Mon Nov 17 09:40:09 2008 From: Hermandw at skynet.be (Herman De Wael) Date: Mon, 17 Nov 2008 09:40:09 +0100 Subject: [blml] [Fwd: Amsterdam] In-Reply-To: <200811161342.AA16553@geller204.nifty.com> References: <4920195A.1070509@talktalk.net> <200811161342.AA16553@geller204.nifty.com> Message-ID: <49212DE9.3020602@skynet.be> Robert Geller wrote: > Nigel Guthrie writes: >> [Nigel] >> IMO - >> - Bob Geller is right that rules that restrict systems are silly. I agree with Bob and Nigel >> - Such rules are among the many silly rules that should be changed. >> - Meanwhile some players, even at top level, masochistically handicap themselves by > trying to comply with the rules. And that is the main point of the discussion. If some players follow silly rules, then we should not allow others to not follow then unpunished. > -> There are different levels of rules. TFLB, while far from perfect, have > been gone over in detail by the drafters, reviewers, WBFLC, and WBF executive. > Players should obey these rules. > > But the system regulations contain many silly provisions in terms of high card > points (4-3-2-1) or the like that are widely ignored. The situation is like > having a speed limit of 100 km/hr when everyone on the road is doing 110. > >> - They believe that they should obey a rule of a game, even if they disagree with it > and deem it silly. > -> Such silly system regulations are not in practice treated by the players > with the same seriousness as TFLB themselves. If this is the case in an ideal > world the authorities would actually change the rules to reflect reality, > but this hasn't happened for whatever reasons..... I think it is reasonable for > players in the World Championships to act in accord with the prevailing > expert community standards. > > >> - In this case, an alleged incident may or may not attract the director's interest. >> - But allegations of illegal patterns of behaviour should trigger official > investigation of deal records. > -> "illegal patterns of behavior" !!?? C'mon gimme a break. > > If Helgemo opens a light 1H in 3rd seat at the Friday night club game > against Mrs. Guggenheim and Nigel wants to hang him for that, well maybe > I can go along with that. But players in the world championships can and > do protect themselves against this sort of thing. It's unbecoming of > BLML for members to be casually slinging around accustaions of > "illegal patterns of behavior" over light 3rd seat openings (as opposed > to real illegal behavior). > This is just crazy. System regulations are not just for Mrs Guggenheim. If the WBF decides that some systems (BS for example) are not allowed in tournaments, then this also applies if one is playing against Italy. It is not because the Italians have encountered BS before that they are geared to dealing with BS in this partnership, in this event. > -Bob > From agot at ulb.ac.be Mon Nov 17 11:08:43 2008 From: agot at ulb.ac.be (Alain Gottcheiner) Date: Mon, 17 Nov 2008 11:08:43 +0100 Subject: [blml] Amsterdam (2) [SEC=UNOFFICIAL] In-Reply-To: <014001c94681$459e5660$15844c59@chello.pl> References: <001101c944e5$84fea410$0302a8c0@Mildred> <002e01c94561$661116b0$32334410$@nl> <003501c94574$fb66f000$0302a8c0@Mildred><491C197F.9090106@skynet.be> <491C251C.2070807@bridgescore.de> <1L0w43-0Ob3Eu0@fwd07.aul.t-online.de><00c901c94667$3f2c1840$15844c59@chello.pl> <491D9867.9050506@ulb.ac.be> <014001c94681$459e5660$15844c59@chello.pl> Message-ID: <492142AB.8070109@ulb.ac.be> Bystry a ?crit : > >> AG : remember that the score can only be changed if there is the >> hesitation unmistakably suggests bidding 3NT. >> > > Of course. But I can also take into consideration the reason why did the > player make his bid. And I will ask him about it. He can state a plethora of > arguments but I will require him to explain why he had made such a gambling > bid. I wouldn't take his answer as a proof that his call was demonstrably > suggested You are mistaken. The words "demonstrably suggested" refer to the player's peer's judgment. Interviewing the player was never defined as the right procedure to decide whether it was. >> a) Jxx - AKxx - Kxxx - AQ where he could be concerned about finding 3NT >> > > This hand is what I would expect for such a pause. AG : but you just wanted to use the player's judgment, not yours ?? Be coherent, please. > >> b) void - AKxx - Kxx - Axxxxx where he could be concerned about a >> leave-in (and where 3NT wouldn't be very succesful) >> > > With this hand I double lightning fast. AG : that's illegal, of course. > What could be wrong if my partner > leaves that in? I've got my ususal amount of defense, so if I find partner > with something like KW9xx of spades and a side trick they would be seriously > hit. And what better contract do we have to play here? > > AG : you know as well as I do that partners also leave in with Qxxx. Regards Alain From rfrick at rfrick.info Mon Nov 17 14:23:59 2008 From: rfrick at rfrick.info (Robert Frick) Date: Mon, 17 Nov 2008 08:23:59 -0500 Subject: [blml] A player's comment [SEC=UNOFFICIAL] In-Reply-To: References: Message-ID: On Mon, 17 Nov 2008 01:51:25 -0500, wrote: > Robert Frick asserted: > >> Actually, to describe 2S as showing heart support, when you >> know that isn't what your partner intended and probably >> doesn't have, is deliberately giving a misleading >> explanation. > > Richard Hills quibbles: > > Deliberately misdescribing partner's cards, but not > necessarily deliberately misdescribing pre-existing mutual > partnership agreement. Right. But the topic was honesty, not following the laws. > > Law 75C - Mistaken Call - second sentence: > > "Here there is no infraction of Law, since East-West did > receive an accurate description of the North-South agreement; > they have no claim to an accurate description of the North- > South hands." First, my phrase was misleading explanation, not mistaken call. Taken out of context. In L75, there is only one North-South agreement to report. In this context, there are potentially two -- there may be mutual partnership agreements and past history which suggest that meaning of 2S when 2 Di is taken as natural. Most people on blml, and probably in the world, will intrepret Laws 20/40/75 as saying that East-West are entitled to hear that second agreement. Or they are like Eric, who will admit that they are not entitled to that second agreement but North-South are not invited back to his club if they don't volunteer it. Of course, the WBFLC has opined differently. > > Robert Frick asserted: > >> And if you are looking for an honest man, my partner's >> explanation in this situation was "If she took my 2 Di bid >> to be multi, then it shows heart support; if she didn't, >> then it shows spades." > > Richard Hills quibbles: > > Describing 2S as simply showing heart support is not > necessarily dishonest. I find a big difference between being honest and not necessarily being dishonest. > The honesty depends upon partner's > Law 40C1 frequency of forgetting the system. If partner > forgets the system sufficiently often to create a two-way > understanding, then a two-way description of 2S is correct. But in other examples, partner has not merely failed to alert, partner has given the wrong explanation. In the given example, the player seemed confident that 2Di was misunderstood and hence that 2Sp did not have the systemic meaning. > > Otherwise a one-way description is honest, Ah, but my partner's explanation is always honest. You just have a chance of being honest. Kind of like the secretary saying the check is in the mail, when there is some chance that it really is. > even -- or indeed > especially -- if one has unauthorised information from a lack > of alert that partner has made a very rare forget of the > system. The large majority vote on blml was that you could use UI from partners alerts/explanation to provide a proper description of partner's bid. Wasn't Stephanie the only one at the end who thought that was inappropriate? [You bid 4NT intending it to be Blackwood, partner explains it as being for the minors which you realize is correct -- you have misbid. Now you are asked to explain his 5 Di call.] Bob From ehaa at starpower.net Mon Nov 17 15:00:51 2008 From: ehaa at starpower.net (Eric Landau) Date: Mon, 17 Nov 2008 09:00:51 -0500 Subject: [blml] A player's comment In-Reply-To: <004001c9467f$f1844c20$0302a8c0@Mildred> References: <491C27D9.4010501@skynet.be> <2a1c3a560811131034h6b48945bl9cc6d238b09159b9@mail.gmail.com> <491D2CB9.70504@skynet.be> <491D65F8.2010500@talktalk.net> <004001c9467f$f1844c20$0302a8c0@Mildred> Message-ID: On Nov 14, 2008, at 12:39 PM, Grattan wrote: > However, > to say that the Nigels > of this world have difficulty in accessing the information is > unjustified. > NBOs are responsible for passing information down to their > members as they think fit. Does anyone else read these two sentences as directly contradicting one another? Eric Landau 1107 Dale Drive Silver Spring MD 20910 ehaa at starpower.net From rfrick at rfrick.info Mon Nov 17 15:21:12 2008 From: rfrick at rfrick.info (Robert Frick) Date: Mon, 17 Nov 2008 09:21:12 -0500 Subject: [blml] Sunday at the club In-Reply-To: References: Message-ID: I double, meaning it as a support double. Then I realize that, systematically, it is a penalty double. Partner eventually alerts and incorrectly explains that I have made a support double. (This UI to me is irrelevant and we end up in a reasonable contract.) After the auction is over, I am surely the only person in the world to now consider if I should correct my partner's explanation. I think the answer is yes. But I don't. My opening lead is the Jack from J109x. The dummy comes down with Kx. While declarer ponders, my partner, a habitual card-detacher, detaches a card to play. That denies the ace, right? I do NOT understand why prematurely detaching a card is merely a matter of etiquette in the laws. Another example. I am dummy, the habitual card-detacher (HCD) is now on my right defending. Declarer plays a singleton diamond from dummy, the defender plays low, declarer ponders, and HCD does her thing. Declarer now plays the king. HCD puts her card back in her hand and plays the ace. That shows the queen, right? If a player detaches a card some of the time, that is UI. Then when they don't detach a card, that is also UI. You can be the director trying to untangle say 6 pieces of potential UI. Meanwhile, I do not want to be the player trying to call the director to complain about UI from a player not detaching their card. From nigelguthrie at talktalk.net Mon Nov 17 15:31:55 2008 From: nigelguthrie at talktalk.net (Nigel Guthrie) Date: Mon, 17 Nov 2008 14:31:55 +0000 Subject: [blml] Law 83 (was ...damn) [SEC=UNOFFICIAL] In-Reply-To: References: Message-ID: <4921805B.9040709@talktalk.net> [Nige1 suggested] I hope the new 2018 rule-book says that, in any event that allows appeals - - The director should appoint at least one experienced player as appeals adviser. [Richard Hills quibbles] Under the current Lawbook, **all** events allow appeals. [WBF Laws Committee minutes, October 2008, page 5] "LAW 93 -- The power of modification given in this law is a right to modify the procedure in dealing with appeals. It does not extend to overriding the rights of contestants to appeal under Law 92A. If the Regulating Authority makes no arrangement for an appeal to be heard (see Law 80B2(k)) the Chief Director shall hear and rule upon it under Law 93A." [Richard Hills asks] Or is Nigel proposing a 2018 law change to limit appeals? [snip] In a lot of small clubs, the playing Director is the only experienced player. Would Nigel permit a Director -- a la W.S. Gilbert's Pooh-Bah -- to appoint herself as both the appeals advisor and also the appeals committee (Law 93A)? [snip] [Nige2] I confess that (like nearly all players and -- I suspect -- most directors), I hadn't read this WBF minute :) At Reading Bridge Club, we appoint an ad hoc committee of *players* to decide appeals. If its impractical to assemble a local committee, it seems hard to conduct a sensible local appeal. I've misgivings about a director hearing a formal appeal against his *own* ruling. If possible it may be better to refer the matter to a higher level. In any case, IMO, would-be appellants should first consult an appeals *advisor* (by telephone if necessary). From ehaa at starpower.net Mon Nov 17 16:04:40 2008 From: ehaa at starpower.net (Eric Landau) Date: Mon, 17 Nov 2008 10:04:40 -0500 Subject: [blml] [Fwd: Amsterdam] In-Reply-To: <4920B59E.8080202@talktalk.net> References: <4920195A.1070509@talktalk.net> <200811161342.AA16553@geller204.nifty.com> <4920B59E.8080202@talktalk.net> Message-ID: <7AC9477A-5983-46F5-BF05-03284873939F@starpower.net> On Nov 16, 2008, at 7:06 PM, Nigel Guthrie wrote: > [Robert Geller] > But the system regulations contain many silly provisions in terms of > high card points (4-3-2-1) or the like that are widely ignored. The > situation is like having a speed limit of 100 km/hr when everyone > on the > road is doing 110. > > {Nigel] > I like Bob's analogy. If you see others getting away with breaking > the rules, then you are tempted to catch up. If you are one of the > few, who still comply with the rules, then you are left behind. I don't think there's a real problem with pairs who normally agree to open 7 HCP hands when the regulations require 8 HCP; they can simply modify their agreement appropriately. (When the ACBL decided to prohibit opening 1NT with less than 10 HCP, for example, EHAA 1NT openings promptly changed from 9-12 HCP to 10-12 HCP without missing a beat.) The real problem is with pairs who, for example, normally agree to open LTC 7-loser hands when the regulations require 8 HCP. For them to comply they would have to abandon their entire system and adopt an entirely different one (see the recent "Is this a HUM?" thread). If you drive 110 when the speed limit is 100 you are breaking the law and you know it, even if everyone else is driving 110. But for some pairs the situation is like having a speed limit of 100 km/hr when they have a speedometer marked in miles/hr and don't know what a km is. Eric Landau 1107 Dale Drive Silver Spring MD 20910 ehaa at starpower.net From ehaa at starpower.net Mon Nov 17 16:42:59 2008 From: ehaa at starpower.net (Eric Landau) Date: Mon, 17 Nov 2008 10:42:59 -0500 Subject: [blml] A player's comment In-Reply-To: References: Message-ID: <81C68581-7E16-4850-8930-B230C6D06D3A@starpower.net> "East-West are entitled to an accurate description of the North-South agreement" [L75B]. On Nov 17, 2008, at 8:23 AM, Robert Frick wrote: > On Mon, 17 Nov 2008 01:51:25 -0500, wrote: > >> Robert Frick asserted: >> >>> Actually, to describe 2S as showing heart support, when you >>> know that isn't what your partner intended and probably >>> doesn't have, is deliberately giving a misleading >>> explanation. I don't see how describing 2S as showing heart support according to your partnership agreement is "deliberately giving a misleading explanation" when in fact 2S shows heart support according to your partnership agreement. That it may be "not what your partner intended" doesn't matter, as you have made no assertion as to what your partner intended. That it describes something he "probably doesn't have" doesn't matter, as you have made no assertion as to what he has. >> Richard Hills quibbles: >> >> Deliberately misdescribing partner's cards, but not >> necessarily deliberately misdescribing pre-existing mutual >> partnership agreement. You cannot be "deliberately misdecribing partner's cards" when you in no way purport to be describing partner's cards. > Right. But the topic was honesty, not following the laws. Honesty requires both truth and responsiveness. It is not "honest" to reply to a question by offering what would be a legimtate answer had you been asked an entirely different question, even though it may be undeniably true. > Eric Landau 1107 Dale Drive Silver Spring MD 20910 ehaa at starpower.net From rfrick at rfrick.info Mon Nov 17 17:14:25 2008 From: rfrick at rfrick.info (Robert Frick) Date: Mon, 17 Nov 2008 11:14:25 -0500 Subject: [blml] A player's comment In-Reply-To: <81C68581-7E16-4850-8930-B230C6D06D3A@starpower.net> References: <81C68581-7E16-4850-8930-B230C6D06D3A@starpower.net> Message-ID: On Mon, 17 Nov 2008 10:42:59 -0500, Eric Landau wrote: > "East-West are entitled to an accurate description of the North-South > agreement" [L75B]. > > On Nov 17, 2008, at 8:23 AM, Robert Frick wrote: > >> On Mon, 17 Nov 2008 01:51:25 -0500, wrote: >> >>> Robert Frick asserted: >>> >>>> Actually, to describe 2S as showing heart support, when you >>>> know that isn't what your partner intended and probably >>>> doesn't have, is deliberately giving a misleading >>>> explanation. > > I don't see how describing 2S as showing heart support according to > your partnership agreement is "deliberately giving a misleading > explanation" when in fact 2S shows heart support according to your > partnership agreement. That it may be "not what your partner > intended" doesn't matter, as you have made no assertion as to what > your partner intended. That it describes something he "probably > doesn't have" doesn't matter, as you have made no assertion as to > what he has. According to Law 75, if I think my bid shows spades, but it really shows hearts and I have misbid my hand (mistaken call), partners correct statement of the partnership agreement is a misleading explanation. Unless I am reading Law 75 wrong. But it seems to clearly describe two types of misleading explanations, mistaken explanations and mistaken calls. "After a misleading explanation has been given to opponents the responsibilites of the players (and Director) are as illustrated by the consequences of this following example." > >>> Richard Hills quibbles: >>> >>> Deliberately misdescribing partner's cards, but not >>> necessarily deliberately misdescribing pre-existing mutual >>> partnership agreement. > > You cannot be "deliberately misdecribing partner's cards" when you in > no way purport to be describing partner's cards. > >> Right. But the topic was honesty, not following the laws. > > Honesty requires both truth and responsiveness. It is not "honest" > to reply to a question by offering what would be a legimtate answer > had you been asked an entirely different question, even though it may > be undeniably true. The opponents did not ask "What would 2Sp mean if 2 Di was multi?" They did not ask "What would 2Sp mean if everyone has properly understood all of the bids in the auction." The probably just asked what 2Sp meant. And you know the intended meaning (in this example). So it you want to be honest and responsive, just clarify for the opponents that you are not describing your partner's intended meaning, you are describing the sytemic meaning of partner's bid had partner understood the auction properly. Or you can follow the laws, your choice. I am not arguing that is lawful to be open and honest and responsive when answering opponent's questions. From svenpran at online.no Mon Nov 17 17:48:07 2008 From: svenpran at online.no (Sven Pran) Date: Mon, 17 Nov 2008 17:48:07 +0100 Subject: [blml] A player's comment In-Reply-To: References: <81C68581-7E16-4850-8930-B230C6D06D3A@starpower.net> Message-ID: <000a01c948d4$45372bc0$cfa58340$@no> On Behalf Of Robert Frick Frankly I have a problem understanding what you try to express, but believe you are on thin ice, therefore my comments below. ............... > According to Law 75, if I think my bid shows spades, but it really shows > hearts and I have misbid my hand (mistaken call), partners correct > statement of the partnership agreement is a misleading explanation. Unless > I am reading Law 75 wrong. But it seems to clearly describe two types of > misleading explanations, mistaken explanations and mistaken calls. "After > a misleading explanation has been given to opponents the responsibilites > of the players (and Director) are as illustrated by the consequences of > this following example." This sounds meaningless: A call can never in itself be an explanation. Law 75 explicitly requires every explanation to be correct description of the partnership understandings related to a specific call, not a description on what cards a player may (probably) hold even if the player explaining partner's call can clearly see that his partner must have misbid. ............... > > The opponents did not ask "What would 2Sp mean if 2 Di was multi?" They > did not ask "What would 2Sp mean if everyone has properly understood all > of the bids in the auction." Such questions would be improper. > > The probably just asked what 2Sp meant. And you know the intended meaning > (in this example). So it you want to be honest and responsive, just > clarify for the opponents that you are not describing your partner's > intended meaning, you are describing the sytemic meaning of partner's bid > had partner understood the auction properly. Or you can follow the laws, > your choice. I am not arguing that is lawful to be open and honest and > responsive when answering opponent's questions. Law 75 requires you to honestly explain the meaning according to your partnership understanding even when you understand that your partner has misbid. Regards Sven From rfrick at rfrick.info Mon Nov 17 18:19:40 2008 From: rfrick at rfrick.info (Robert Frick) Date: Mon, 17 Nov 2008 12:19:40 -0500 Subject: [blml] A player's comment In-Reply-To: <000a01c948d4$45372bc0$cfa58340$@no> References: <81C68581-7E16-4850-8930-B230C6D06D3A@starpower.net> <000a01c948d4$45372bc0$cfa58340$@no> Message-ID: On Mon, 17 Nov 2008 11:48:07 -0500, Sven Pran wrote: > On Behalf Of Robert Frick > > Frankly I have a problem understanding what you try to express, but > believe > you are on thin ice, therefore my comments below. > ............... >> According to Law 75, if I think my bid shows spades, but it really shows >> hearts and I have misbid my hand (mistaken call), partners correct >> statement of the partnership agreement is a misleading explanation. >> Unless >> I am reading Law 75 wrong. But it seems to clearly describe two types of >> misleading explanations, mistaken explanations and mistaken calls. >> "After >> a misleading explanation has been given to opponents the responsibilites >> of the players (and Director) are as illustrated by the consequences of >> this following example." > > This sounds meaningless: A call can never in itself be an explanation. IMO, "misleading explanation" is a clearly defined technical term in the laws. If my partner bids 4NT thinking that it is for the minors, and it really is supposed to be unusual, then my description of his bid as being for the minors is legal and appropriate. But it is still a "misleading explanation". In this situation, I can do not better than describe his bid as being for the minors; I have no clue that I am giving a misleading explanation. In the case we are discussing, the player knew that partner had misunderstood this auction. You can claim that players should be required to provide a misleading explanation in this situation, and you will have a lot of company on blml and support from the WBFLC. I will only argue if you try to claim that the laws clearly demand that or if you claim that is a wise thing for the WBF to insist upon. > > Law 75 explicitly requires every explanation to be correct description of > the partnership understandings related to a specific call, not a > description > on what cards a player may (probably) hold even if the player explaining > partner's call can clearly see that his partner must have misbid. Actually, Law 75 says nothing of the kind, that I can see. Law 75 is about the consequences of an unintended misleading explanation. THe only exception I can is "South's responsibility is to act as though North had made a strong game try opposite a weak response.." and "South must do nothing to correct the mistaken explanation while the auction continues." But this of course undermines the position you wish to argue for. > > ............... >> >> The opponents did not ask "What would 2Sp mean if 2 Di was multi?" They >> did not ask "What would 2Sp mean if everyone has properly understood all >> of the bids in the auction." > > Such questions would be improper. Can you give me the law for this? > >> >> The probably just asked what 2Sp meant. And you know the intended >> meaning >> (in this example). So it you want to be honest and responsive, just >> clarify for the opponents that you are not describing your partner's >> intended meaning, you are describing the sytemic meaning of partner's >> bid >> had partner understood the auction properly. Or you can follow the laws, >> your choice. I am not arguing that is lawful to be open and honest and >> responsive when answering opponent's questions. > > Law 75 requires you to honestly explain the meaning according to your > partnership understanding even when you understand that your partner has > misbid. Which section of the law are you referring to? Again, I cannot find it. Bob From agot at ulb.ac.be Mon Nov 17 18:45:30 2008 From: agot at ulb.ac.be (Alain Gottcheiner) Date: Mon, 17 Nov 2008 18:45:30 +0100 Subject: [blml] Sunday at the club In-Reply-To: References: Message-ID: <4921ADBA.1070708@ulb.ac.be> Robert Frick a ?crit : > I double, meaning it as a support double. Then I realize that, > systematically, it is a penalty double. Partner eventually alerts and > incorrectly explains that I have made a support double. (This UI to me is > irrelevant and we end up in a reasonable contract.) > > After the auction is over, I am surely the only person in the world to now > consider if I should correct my partner's explanation. I think the answer > is yes. But I don't. > > AG : that's many things at once. About this first one, the Laws say you have to corerect (provided you're declarer or dummy), but common sense says this will harm opponents, so you don't. I know many will say that common sense shouldn't be used in such situations, and that the Laws reign. I don't like this. The main problem, in fact, is that if you correct, you might be accused to have tried to fool them, especially in the Netherlands, where it is considered that the fact that you made a suppoert double proves that your agreement is support doubles. > > My opening lead is the Jack from J109x. The dummy comes down with Kx. > While declarer ponders, my partner, a habitual card-detacher, detaches a > card to play. > > That denies the ace, right? I do NOT understand why prematurely detaching > a card is merely a matter of etiquette in the laws. > AG : it isn't merely etiquette. Any play made with undue haste or any mannerism can be deemed to have conveyed UI. And detaching a card is a violation of normal procedure, whence an irregularity. > Another example. I am dummy, the habitual card-detacher (HCD) is now on my > right defending. Declarer plays a singleton diamond from dummy, the > defender plays low, declarer ponders, and HCD does her thing. Declarer now > plays the king. HCD puts her card back in her hand and plays the ace. That > shows the queen, right? > AG : if for any reason the fact of detaching the first card misled declarer, I will correct the score, most probably based on L72 B1. The fact that "there will usually be no penalty" doesn't mean "never". But the case is a bit strange : how could she know what declarer was going to play ? If I was the TD, after she does it twice, I'll use L81C6 and tell her that I'm going to fine her 10% of a top every time it happens again. > If a player detaches a card some of the time, that is UI. Then when they > don't detach a card, that is also UI. You can be the director trying to > untangle say 6 pieces of potential UI. Meanwhile, I do not want to be the > player trying to call the director to complain about UI from a player not > detaching their card. > > > AG : if she does it very frequently, perhaps you could. A very quick player's normal-tempo cards may contain UI. Anyway, after my intervention from previous paragraph, perhaps she'll have learned. Best regards Alain From darkbystry at wp.pl Mon Nov 17 18:41:00 2008 From: darkbystry at wp.pl (Bystry) Date: Mon, 17 Nov 2008 18:41:00 +0100 Subject: [blml] Amsterdam (2) [SEC=UNOFFICIAL] References: <001101c944e5$84fea410$0302a8c0@Mildred> <002e01c94561$661116b0$32334410$@nl> <003501c94574$fb66f000$0302a8c0@Mildred><491C197F.9090106@skynet.be> <491C251C.2070807@bridgescore.de> <1L0w43-0Ob3Eu0@fwd07.aul.t-online.de><00c901c94667$3f2c1840$15844c59@chello.pl> <491D9867.9050506@ulb.ac.be><014001c94681$459e5660$15844c59@chello.pl> <492142AB.8070109@ulb.ac.be> Message-ID: <002401c948db$a8b8f0a0$15844c59@chello.pl> Hi Alain, >> Of course. But I can also take into consideration the reason why did the >> player make his bid. And I will ask him about it. He can state a plethora of >> arguments but I will require him to explain why he had made such a gambling >> bid. I wouldn't take his answer as a proof that his call was demonstrably >> suggested > You are mistaken. The words "demonstrably suggested" refer to the > player's peer's judgment. Interviewing the player was never defined as > the right procedure to decide whether it was. Maybe my English is so bad that you're justified in completely misunderstanding my statements. Or maybe you're just deliberately searching for some minor flaws. First thing - nowhere in BL it is written that the words "demonstrably suggested" refer to the player's peer's judgement. It is only written in the definition of LA "... among the class of player...". So here you're mistaken, not me. Second thing - I'm suprised that you think it's not obligatory to ask OS players about their opinion. For me it's an obvious, correct procedure. It's quite possible they will say something (and prove it via system notes or other evidence) that _for them_ something is not demonstrably suggested or that the chosen call has no LA ("... using the methods of partnership..."). It's also possible that the offending player will admit that for him the chosen call _was_ demonstrably suggested. And than it is not correct to reject this evidence and substitute your own opinion. Third thing - I know that we often reject OS statements as self-serving. That's why it is necessary to poll player's peers. But I have never written that I would decide the case _only_ on the grounds of hearing what the OS has to say. I wrote "I wouldn't take his answer as a proof". I just noted that when the chosen call is so strange it's useful to know why the player decided to make it. >>> a) Jxx - AKxx - Kxxx - AQ where he could be concerned about finding 3NT >>> >> >> This hand is what I would expect for such a pause. > AG : but you just wanted to use the player's judgment, not yours ?? Be > coherent, please. Hard to answer such confrontional statements. I'll repeat - I don't want to decide on the basis of the offending player's judgment unless he admits that for him the chosen call was in fact "demonstrably suggested". >>> b) void - AKxx - Kxx - Axxxxx where he could be concerned about a >>> leave-in (and where 3NT wouldn't be very succesful) >> >> With this hand I double lightning fast. > AG : that's illegal, of course. In my opinion it would be better if we respected each other. Is it really necessary for me to insert some smileys or to put such words into quotation marks? >> What could be wrong if my partner >> leaves that in? I've got my ususal amount of defense, so if I find partner >> with something like KW9xx of spades and a side trick they would be seriously >> hit. And what better contract do we have to play here? > > AG : you know as well as I do that partners also leave in with Qxxx. I have no knowledge about your partners. My partners don't leave in takeout and balancing doubles with only one trick in opponents suit unless they have no other alternative. So if my partner has something like Dxxx Dxx Axx xxx he'll pass and we'll take 300 or 500 instead of struggling to make 5C. Horrible. > Regards > Alain Regards Maciej From ehaa at starpower.net Mon Nov 17 18:47:46 2008 From: ehaa at starpower.net (Eric Landau) Date: Mon, 17 Nov 2008 12:47:46 -0500 Subject: [blml] A player's comment In-Reply-To: References: <81C68581-7E16-4850-8930-B230C6D06D3A@starpower.net> Message-ID: On Nov 17, 2008, at 11:14 AM, Robert Frick wrote: > On Mon, 17 Nov 2008 10:42:59 -0500, Eric Landau > wrote: > >> "East-West are entitled to an accurate description of the North-South >> agreement" [L75B]. >> >> On Nov 17, 2008, at 8:23 AM, Robert Frick wrote: >> >>> On Mon, 17 Nov 2008 01:51:25 -0500, >>> wrote: >>> >>>> Robert Frick asserted: >>>> >>>>> Actually, to describe 2S as showing heart support, when you >>>>> know that isn't what your partner intended and probably >>>>> doesn't have, is deliberately giving a misleading >>>>> explanation. >> >> I don't see how describing 2S as showing heart support according to >> your partnership agreement is "deliberately giving a misleading >> explanation" when in fact 2S shows heart support according to your >> partnership agreement. That it may be "not what your partner >> intended" doesn't matter, as you have made no assertion as to what >> your partner intended. That it describes something he "probably >> doesn't have" doesn't matter, as you have made no assertion as to >> what he has. > > According to Law 75, if I think my bid shows spades, but it really > shows > hearts and I have misbid my hand (mistaken call), partners correct > statement of the partnership agreement is a misleading explanation. > Unless > I am reading Law 75 wrong. But it seems to clearly describe two > types of > misleading explanations, mistaken explanations and mistaken calls. > "After > a misleading explanation has been given to opponents the > responsibilites > of the players (and Director) are as illustrated by the > consequences of > this following example." That's nonsense. "Partner's correct explanation of the partnership agreement is a misleading explanation" only if partner purports to be explaining something other than the partnership agreement. And since the law requires him to offer (only) a correct explanation of the partnership agreement, why would he purport -- or be assumed -- to be offering something entirely else? >>>> Richard Hills quibbles: >>>> >>>> Deliberately misdescribing partner's cards, but not >>>> necessarily deliberately misdescribing pre-existing mutual >>>> partnership agreement. >> >> You cannot be "deliberately misdecribing partner's cards" when you in >> no way purport to be describing partner's cards. >> >>> Right. But the topic was honesty, not following the laws. >> >> Honesty requires both truth and responsiveness. It is not "honest" >> to reply to a question by offering what would be a legimtate answer >> had you been asked an entirely different question, even though it may >> be undeniably true. > > The opponents did not ask "What would 2Sp mean if 2 Di was multi?" > They > did not ask "What would 2Sp mean if everyone has properly > understood all > of the bids in the auction." > > The probably just asked what 2Sp meant. Since the law only permits them to ask what 2S meant *according to the partnership's understanding*, you are entitled to the presumption that that's what they were asking. Of course, it is ethically inappropriate to hide behind that entitlement if you suspect that your legal and correct explanantion may be misunderstood as unresponsive, so it is right to go out of your way, if necessary, to insure that your opponents understand what it is you're disclosing. But that doesn't mean you should change the content of your disclosure to suit what you think they thought they were asking. > And you know the intended meaning > (in this example). So it you want to be honest and responsive, just > clarify for the opponents that you are not describing your partner's > intended meaning, you are describing the sytemic meaning of > partner's bid > had partner understood the auction properly. Exactly. I hope I have not said anything to suggest that I would object to this. > Or you can follow the laws, > your choice. You can follow the laws by doing as Bob suggests; there is no forced choice here. It doesn't break the law to make sure your opponents know that you are following it! > I am not arguing that is lawful to be open and honest and > responsive when answering opponent's questions. It is not only lawful, it is mandatory in the ACBL, which has a published regulatory guideline telling you to do just that. But the ACBL guideline in no way contradicts or overrides the nature of the disclosure obligation specified in TFLB, which is to disclose your actual partnership understanding, regardless of what you might think your partner's differing intention might be. We should recognize that we are not discussing a realistic scenario here. One's legal obligation to disclose (only) one's actual partnership understandings only translates into unambiguous guidelines for behavior at the table only if one is entirely certain as to what one's actual partnership understandings are; what we lack is an understanding of what to do when that's not the case. When partner sees things in a manner we believe to be incorrect, we don't learn about it by utilizing hidden telepathic powers; in real life he will have said or done something to indicate an intention that differs from one's own notion of what the partnership understandings call for (e.g. describing your minor-suit-takeout 4NT as "Blackwood"), and, provided we stick to proper disclosure, the indicating discrepancy will be as obvious to the opponents as it is to us (the primary anti-dWS argument is that this is as it should be; going out of one's way to keep it from being as obvious to the opponents as it is to us is, at best, ethically questionable). It is, even so, perfectly reasonable (perhaps even rationally necessary, but we need not digress into epistomology here) to allow any such indication to spoil any presumption of certainty, and to react, as Bob would have us do, by always presuming that there is some possibility that partner knows our "true understanding" better than we do, and disclose accordingly. Which puts us back where we started, with the question of what to do when you know that your version of your systemic understanding differs from your partner's but do not know which of you is right. That is a question we have yet to answer. Eric Landau 1107 Dale Drive Silver Spring MD 20910 ehaa at starpower.net From rfrick at rfrick.info Mon Nov 17 19:42:45 2008 From: rfrick at rfrick.info (Robert Frick) Date: Mon, 17 Nov 2008 13:42:45 -0500 Subject: [blml] A player's comment In-Reply-To: References: <81C68581-7E16-4850-8930-B230C6D06D3A@starpower.net> Message-ID: I will try to be clear about this. You bid 4NT, correctly intending it to be for the minors. Your partner explains this as asking for controls and bids 5Di. You -- sooner or later -- are asked what this bid means. You presumably have partnership agreements, understandings, etc. about the meaning of 5Di as a response to 4NT for the minors. But many times you also have partnership agreements, understandings, past history, whatever, that gives you more knowledge than the opponents about what 5 Di means as a Blackwood response. (and if you don't, you can at least say no agreement.) So you have a choice: There are two different sets of agreements, etc. that you can choose to divulge. People have argued on blml that players are not required to divulge the second set of partnership agreements. This was not a popular opinion, and you did not agree with it (responding to Eric here), as I recall. The WBFLC has supported this opinion. So if the opponents, who are now defending and what to know about controls, you do not have to answer. I believe Richard will argue that you are required by law not to answer, as your chief goal is supposed to be to win. I don't see where the laws require you to provide the first set of partnership agreements and not the other. You can say it is in the laws a hundred times and I still won't see it. To me, the laws just talk about divulging relevant partnership agreements. Then, if you are going to offer just one set of agreements, which one do you do? I can see the laws very clearly saying that you offer the explanation that does not clue in partner that partner misunderstood your bid. I can't see where the laws otherwise say to prefer one partnership agreement over another. So I keep asking what law you are talking about when you claim the laws support telling the second explanation. > On Nov 17, 2008, at 11:14 AM, Robert Frick wrote: > >> On Mon, 17 Nov 2008 10:42:59 -0500, Eric Landau >> wrote: >> >>> "East-West are entitled to an accurate description of the North-South >>> agreement" [L75B]. >>> >>> On Nov 17, 2008, at 8:23 AM, Robert Frick wrote: >>> >>>> On Mon, 17 Nov 2008 01:51:25 -0500, >>>> wrote: >>>> >>>>> Robert Frick asserted: >>>>> >>>>>> Actually, to describe 2S as showing heart support, when you >>>>>> know that isn't what your partner intended and probably >>>>>> doesn't have, is deliberately giving a misleading >>>>>> explanation. >>> >>> I don't see how describing 2S as showing heart support according to >>> your partnership agreement is "deliberately giving a misleading >>> explanation" when in fact 2S shows heart support according to your >>> partnership agreement. That it may be "not what your partner >>> intended" doesn't matter, as you have made no assertion as to what >>> your partner intended. That it describes something he "probably >>> doesn't have" doesn't matter, as you have made no assertion as to >>> what he has. >> >> According to Law 75, if I think my bid shows spades, but it really >> shows >> hearts and I have misbid my hand (mistaken call), partners correct >> statement of the partnership agreement is a misleading explanation. >> Unless >> I am reading Law 75 wrong. But it seems to clearly describe two >> types of >> misleading explanations, mistaken explanations and mistaken calls. >> "After >> a misleading explanation has been given to opponents the >> responsibilites >> of the players (and Director) are as illustrated by the >> consequences of >> this following example." > > That's nonsense. "Partner's correct explanation of the partnership > agreement is a misleading explanation" only if partner purports to be > explaining something other than the partnership agreement. And since > the law requires him to offer (only) a correct explanation of the > partnership agreement, why would he purport -- or be assumed -- to be > offering something entirely else? I think you are confusing "misleading explanation" with "mistaken explanation". You can provide a perfectly correct explanation of partner's bid according to the partnership agreement. But if partner has made a mistaken call, you have made a misleading explanation. Misleading explanation is a superset, containing both mistaken calls and mistaken explanations." From ehaa at starpower.net Mon Nov 17 19:47:41 2008 From: ehaa at starpower.net (Eric Landau) Date: Mon, 17 Nov 2008 13:47:41 -0500 Subject: [blml] Sunday at the club In-Reply-To: <4921ADBA.1070708@ulb.ac.be> References: <4921ADBA.1070708@ulb.ac.be> Message-ID: <6E89CACF-EC40-4554-90E5-6ABDEABF23EC@starpower.net> Ah, semantics... On Nov 17, 2008, at 12:45 PM, Alain Gottcheiner wrote: > Robert Frick a ?crit : > >> I double, meaning it as a support double. Then I realize that, >> systematically, it is a penalty double. Partner eventually alerts and >> incorrectly explains that I have made a support double. (This UI >> to me is >> irrelevant and we end up in a reasonable contract.) >> >> After the auction is over, I am surely the only person in the >> world to now >> consider if I should correct my partner's explanation. I think the >> answer >> is yes. But I don't. What is the semantic force of, "Then I realize that, systematically, it is a penalty double"? You didn't mean it as a support double when you made it only because some outside agency took over your mind and implanted a perverse momentary hypnotic suggestion. You thought, for some reason, at the time, that it was a support double. Your partner explained it as a support double. How does that justify "incorrectly"? What you in fact "realized" was that your implicit agreement by which it was a support double has come to supplant an earlier (perhaps only now recalled) explicit agreement by which it would have been something else. That may be so, but it doesn't affect either your legal or your ethical obligations to disclose what it is by your actual agreement, of whatever sort. > AG : that's many things at once. About this first one, the Laws say > you > have to corerect (provided you're declarer or dummy), but common sense > says this will harm opponents, so you don't. I know many will say that > common sense shouldn't be used in such situations, and that the Laws > reign. I don't like this. > The main problem, in fact, is that if you correct, you might be > accused > to have tried to fool them, especially in the Netherlands, where it is > considered that the fact that you made a suppoert double proves that > your agreement is support doubles. If in the Netherlands "it is considered" by force of regulation "that the fact that you made a support double proves that your agreement is support doubles", then -- guess what? -- it was a support double. (IMO that would be a silly regulation, but one that based such a presumption on both your making a support double and partner's exlaining it as such would be indisputably sensible). It is nonsense to assert that "the Laws say you have to correct". You have made a statement which the regulations explicitly define as true. You cannot "correct" a statement that was not incorrect to begin with. ISTM that Robert and Alan have quite clearly established that the law requiring you to disclose your actual partnership agreement does not actually require you to disclose your actual partnership agreement if your actual partnership agreement is not actually your partnership agreement. I'm not sure what we're supposed to take away from that. Eric Landau 1107 Dale Drive Silver Spring MD 20910 ehaa at starpower.net From nigelguthrie at talktalk.net Mon Nov 17 20:29:25 2008 From: nigelguthrie at talktalk.net (Nigel Guthrie) Date: Mon, 17 Nov 2008 19:29:25 +0000 Subject: [blml] [Fwd: Amsterdam] In-Reply-To: <000701c94857$8c51de60$15844c59@chello.pl> References: <4920195A.1070509@talktalk.net><200811161342.AA16553@geller204.nifty.com> <4920B59E.8080202@talktalk.net> <000701c94857$8c51de60$15844c59@chello.pl> Message-ID: <4921C615.7030501@talktalk.net> [Nige1] I won't quote Maciej's entire post about the laws but he echoes the views of most players with whom I've talked and I agree with almost all he wrote. Some comments ... [Maciej] .. So I like Bob's analogy too. But of course not in the way he intended it to be. I say that every driver who exceeds speed limits should be penalised and every bridge player who breaks the rules should be equally penalised. And stating some rules in terms of HCP is for me quite a good idea because it is something nearly every bridge player understands. Other methods (e.g. LP, SP, LTC, QT, PT, Zar points) are less common and it would be necessary to overcomplicate the rules to take every such method into account. [Nige1] I think there are too many system restrictions; but I agree with Maciej that it is better to phrase rules in crude objective terms that players can understand (eg Milton Work HCP) rather than rely on more sophisticated and nebulous measures tempered by subjective director judgement. [Maciej] .. But it's not easy to write simple, clear and complete laws in such a complicated game as Bridge. What's even worse some influencing groups (or even nations and zones) have completely different ideas what would be simple or objective. [Nige1] I agree that it would be a hard task but I reckon it would be worth the effort. I also think that the rules should be complete but allow local legislators to substitute local variants. [Maciej] .. E.g. my view (and I know many players who agree with me) is that we should adopt simple rules - a card played is a card played, a call made is a call made and so on. For revoke - automatic penalty of 2 tricks (and adjusted score if such penalty doesn't do justice for NOS with _additional_ penalty of 2 tricks). No "equity" rulings like 12C3 - bad score should be a reminder for OS to respect their obligations and NOS should be given due compensation for having been deprived of normal play on the board and chances for obtaining a very good score. [Nigel] And so say all of us :) [Maciej] .. But this view is completely opposed by today's authorities and majority of BLMLers. So I have to live with it and I'm trying to believe they think it will be better for Bridge. My prediction is not so optimistic, maybe such policy will minimally increase overall number of bridge players but it will drastically increase the number of law-breaking players. Why should they bother to comply to UI regulations if they get their "equity" after the infraction? PP won't be given in the whole scope of mid-level and beginner bridge. Why should they bother to comply to disclosure rules? Why should they be careful when making their calls or playing their cards? [Nige1] Most players of my acquaintance agree with Maciej. But directors point out that they are players themselves. They, too, claim that their views reflect those of ordinary players. I suspect, however, that law-makers and directors have a subtly different agenda form other players. Complex sophisticated rules increase director involvement. Equity rules also require more subjective assessments. Directors may have a bias towards such rules that rely on director judgement and give directors more power to decide events. This is speculation. Of course it would be easy to canvass players to find out what their views really are, if the WBF were interested :) [Maciej] .. No, current official trend is to satisfy people who don't like being penalized for their inattention to game and it's rules. And people like me, who bend backwards to obey them strictly, are put in the worse position. But I won't stop playing Bridge and will obey BL as they are written - I love this game in spite of it's deficiencies. That's why our lawmakers don't care about people like me. [Nigel] As Maciej says, "Equity" rules encourage the law-breaker to break the law and discourage his victim from reporting an infraction. "Equity" rulings usually provide inadequate redress: often an average result; instead of the better result that the victim might hope to achieve without the infraction; and might expect as a reward for putting up with the hassle of conscientiously calling the director. The victim may even suffer the ultimate indignity of being unfairly deprived of any redress because the director judges that he "failed to protect himself" or that a subsequent action by him was "wild and gambling". Just two examples of unfair and unnecessary rules. Such rules are biased in favour of experts. They add zero enjoyment to the game. I suppose that "Equity" rules may become more popular as more and more players benefit from breaking them. IMO, separate procedural and disciplinary penalties have their place but appropriate redress and deterrence should be built into most laws and regulations. From sater at xs4all.nl Mon Nov 17 21:40:16 2008 From: sater at xs4all.nl (Hans van Staveren) Date: Mon, 17 Nov 2008 21:40:16 +0100 Subject: [blml] Sunday at the club In-Reply-To: <4921ADBA.1070708@ulb.ac.be> References: <4921ADBA.1070708@ulb.ac.be> Message-ID: <003e01c948f4$b54d9370$1fe8ba50$@nl> [AG] The main problem, in fact, is that if you correct, you might be accused to have tried to fool them, especially in the Netherlands, where it is considered that the fact that you made a suppoert double proves that your agreement is support doubles. [HvS] By now I almost feel as I have to defend the Dutch National AC. This is contrary to my normal role, which is to shout worldwide that they are total idiots without any sense whatsoever. But really, could people from outside NL refrain from explaining how we do things, because: 1) We might just have stopped doing it. 2) We might not have done it in the first place. Hans From richard.hills at immi.gov.au Mon Nov 17 22:03:10 2008 From: richard.hills at immi.gov.au (richard.hills at immi.gov.au) Date: Tue, 18 Nov 2008 08:03:10 +1100 Subject: [blml] A player's comment [SEC=UNOFFICIAL] In-Reply-To: Message-ID: Eric Landau asserted: [big snip] >Which puts us back where we started, with the question of >what to do when you know that your version of your systemic >understanding differs from your partner's but do not know >which of you is right. > >That is a question we have yet to answer. Law 20F5(b): "The player must call the Director and inform his opponents that, **in his opinion**, his partner's explanation was erroneous (see Law 75) but only at his first legal opportunity..." What's the problem? Best wishes Richard James Hills Recruitment Section, Level 3 Blue, workstation 15 (first on left) Department of Immigration and Citizenship Telephone: 02 6223 8453 Email: richard.hills at immi.gov.au -------------------------------------------------------------------- Important Notice: If you have received this email by mistake, please advise the sender and delete the message and attachments immediately. This email, including attachments, may contain confidential, sensitive, legally privileged and/or copyright information. Any review, retransmission, dissemination or other use of this information by persons or entities other than the intended recipient is prohibited. DIAC respects your privacy and has obligations under the Privacy Act 1988. The official departmental privacy policy can be viewed on the department's website at www.immi.gov.au. See: http://www.immi.gov.au/functional/privacy.htm --------------------------------------------------------------------- From richard.hills at immi.gov.au Mon Nov 17 22:33:05 2008 From: richard.hills at immi.gov.au (richard.hills at immi.gov.au) Date: Tue, 18 Nov 2008 08:33:05 +1100 Subject: [blml] Should (was ...damn) [SEC=UNOFFICIAL] In-Reply-To: <001401c94846$71a921b0$0302a8c0@Mildred> Message-ID: Law 16B3, first sentence, final phrase: "...he should summon the Director when play ends**. Law 16B3, first sentence, footnote: "** it is not an infraction to call the Director earlier or later." Introduction: "...'should' do (failure to do it is an infraction jeopardizing the infractor's rights but not often penalized)..." Grattan Endicott: ...We do not want a request for a ruling to be ruled out of time because it is not made at the time the law indicates to be appropriate. However, the use of 'should' in this law does establish that the desirable time for the protest to be entered is when play ends... Richard Hills: According to the Introduction, the default meaning of the word "should", when used in a Law, is "an infraction jeopardizing the infractor's rights". The specific Law 16B3 footnote over-rides the default meaning of "infraction", but does not over-ride the default meaning of "jeopardizing rights". Hence Grattan's concept of a "desirable" but not required time for summoning the Director under Law 16B3 seems to me to be fully consistent with a careful parsing of the Lawbook. Best wishes Richard James Hills Recruitment Section, Level 3 Blue, workstation 15 (first on left) Department of Immigration and Citizenship Telephone: 02 6223 8453 Email: richard.hills at immi.gov.au -------------------------------------------------------------------- Important Notice: If you have received this email by mistake, please advise the sender and delete the message and attachments immediately. This email, including attachments, may contain confidential, sensitive, legally privileged and/or copyright information. Any review, retransmission, dissemination or other use of this information by persons or entities other than the intended recipient is prohibited. DIAC respects your privacy and has obligations under the Privacy Act 1988. The official departmental privacy policy can be viewed on the department's website at www.immi.gov.au. See: http://www.immi.gov.au/functional/privacy.htm --------------------------------------------------------------------- From grandaeval at tiscali.co.uk Mon Nov 17 21:37:04 2008 From: grandaeval at tiscali.co.uk (Grattan) Date: Mon, 17 Nov 2008 20:37:04 -0000 Subject: [blml] Diverse cultures Message-ID: <009c01c948fe$6c0575c0$0302a8c0@Mildred> Grattan Endicott Grattan Endicott<200811161342.AA16553@geller204.nifty.com> <4920B59E.8080202@talktalk.net><000701c94857$8c51de60$15844c59@chello.pl> <4921C615.7030501@talktalk.net> Message-ID: <000701c94901$2ed86ba0$15844c59@chello.pl> Hi Nigel > [Nige1] > I think there are too many system restrictions; but I agree with Maciej that it is better to phrase rules in crude objective terms that players can understand (eg > Milton Work HCP) rather than rely on more sophisticated and nebulous measures tempered by subjective director judgement. The discussion about system restrictions could be very long and I don't think it's ever possible to reach common agreement. That's because of different expectations from different people. And it doesn't even have always to be dependent on the level of players. E.g. some low-level players want only to have fun playing something they know but some want to experiment and some want to play against strange systems to learn. I personally don't have any strong opinion about system (or psychic) restrictions. I know from my personal experience that when I was at the low end of mid-level I had troubles with unexcpected systems because my skills were lacking. Now I've read about many systems, learned (invented) some defences and my skills are better so I could rearrange my thinking even during 2-board round. So I don't mind if my opps play something atypical and even volunteered to play against opponents using HUM in league games. But the real problem is that most of our tournaments are open and players of different levels are participating. And because money prizes are given even top players wish to have more bunnies participating to raise the pool. So it is for them logical to have rules which encourage those bunnies to come and play and therefore it is so for signifant majority of players. So our system policy reflects those views and allows different systems for different levels and kinds of competitions. Of course it is full of errors and holes, but main idea seems right. There are some who strongly oppose this and I understand their grievance. But there is no perfect solution to satisfy everyone. > [Maciej] > .. But it's not easy to write simple, clear and complete laws in such a complicated game as Bridge. What's even > worse some influencing groups (or even nations and zones) have completely different ideas what would be simple or objective. > > [Nige1] > I agree that it would be a hard task but I reckon it would be worth the effort. I also think that the rules should be complete but allow local legislators to substitute > local variants. Yes, it is worth of trying. But the main problem lies in logistics. There is a significant group of players who wish the rules were better. But to get this view to the officials they would have to organise, think about it solidly and state an official petition. It is not enough for you or me to represent such opinions here on BLML. We don't have an official position to represent anyone but us. So David, Grattan, Kojak, Ton can rightfully claim that in their opinion players like present Laws and their voice has equal (for them higher) significance than ours. So I must admit - it is inter alia my personal culpability. I could write my position, get many votes to sign under it (it would be no problem to get many players to agree, only to collect those voices) and give it to our NO so that they could present it to WBFLC (or drafting commision, forgive me my ignorance). I didn't do this. And I think nearly no group of players did. Therefore it is clear that what we now have in TFLB is mostly a compilation of opinions from the officials. And I can't blame them for that, the players' are to blame. I try to be objective and look at things from TD' and other officials' point of view. They have been dealing with Laws and their application for so many years that it's hard for them to imagine that what the players mostly want is _no director at the table_, not _the director at the table in any possible happenings_. And instead of trying their best to _eliminate or vastly reduce_ the number of infractions and other obstacles that cause the boards to be normally unplayable they are mainly considering _how to most thoroughly deal with existing infractions_. I don't claim that this second target is unimportant. I strongly claim that the first one is much more important. And I'm in deep sadness they don't realize that their present attitude to concentrate on the second one and use a bit strange concept of "equity" (which is not present in this form in any other law system of which I know) is causing the first target to diminish or worse - to work in the opposite way. I have a question for our prominent TD's. Which situation is better for bridge: - most players are educated about current Laws. They try their best to comply to them, e.g. they bend themselves backward and try to not use UI or get their system agreements better. They know that breaking the Laws will very often result in poor score and may get an additional penalty. They know that when they are deprived of the possibility to play the board normally they would get due redress which means their best possible result. Cheaters have hard times because most of their misdeeds are penalised harshly. - many players are not educated about Laws or don't understand them because of their complexity. Even if they know some portion of Laws they have no incentive to obey them because the TD's will play God and restore the board to the point before an infraction. Often they can be prone to disobey the Law if they judge that an "equity" ruling will be better for them than the bad score at the table. Cheaters profit from their misdeeds because they will not be distinguished from unintentional lawbreakers and the worse they can get is a rare PP from the TD who don't put himself into our modern trend of "don't penalize". For me an answer is obvious. And for me it's also obvious that the current trend will result unavoidably in the second situation. If you want I can show you some real-life examples and of course I'm able to invent some too. I am aware of the fact that you will propably ignore my whining. But it would be much better if you at least considered it even if then you would reject it. > [Nige1] > Most players of my acquaintance agree with Maciej. But directors point out that they are players themselves. They, too, claim that their views reflect those of > ordinary players. They have every right to present their views. They must however be aware that their view could be tainted by their job. > I suspect, however, that law-makers and directors have a subtly different agenda form other players. Complex sophisticated rules increase director involvement. > Equity rules also require more subjective assessments. Directors may have a bias towards such rules that rely on director judgement and give directors more > power to decide events. I agree wholeheartedly. > This is speculation. Of course it would be easy to canvass players to find out what their views really are, if the WBF were interested :) As I understand WBF prefers the players to state their ideas to NO's and then NO's should state this ideas to WBF. I understand such position although it inevitably results in _no imput_ from players. > [Nigel] > As Maciej says, "Equity" rules encourage the law-breaker to break the law and discourage his victim from reporting an infraction. "Equity" rulings usually provide > inadequate redress: often an average result; instead of the better result that the victim might hope to achieve without the infraction; and might expect as a reward > for putting up with the hassle of conscientiously calling the director. Yes. I have an undiscussable right to play the game of Bridge and my result has to be a summary of my actions. I don't want to be denied winning because of any TD imaginating what would have happened. I equally don't want to be denied losing (!) because of that. So I want to personally decide how would I solve my two-way finesse problem. If I'm deprived of this decision by my opponents infraction I should be given automatically the best possible result. I asked 5 random people who haven't ever played Bridge what do they think about such situation. They all stated it is incomprehensible for them not to give the best of the best to someone who is fauled and therefore unable to do something himself. In Bridge we have some borders - we shouldn't give unrealistic results to anyone but that should be all. What's more, "equity" as understood by our TD's have nothing in common with "equity" on one given board. It is only close to real equity if we take into consideration all similar, virtual boards. It's making an average. And it's sometimes not fair. When my tournament score is 40% after many rounds I don't care if I miss two-way finesse. But if I have 60% so far my decision can decide the winner of the tournament. And I have a good day, play lucky and read all what is to read. Now my chance to get it right is higher than 50%. Maybe even higher than 60%. Maybe opponents will make some defensive blunder. And the TD won't take that into consideration. So I'm effectively deprived of my present "equity" despite the fact that in a long run it will get back to an average. Many players have only one chance in their lives to win a major event. Please, don't deprive them of that. Give them the best they can get because they are "the right side", not their law-infracting opponents. > The victim may even suffer the ultimate indignity of being unfairly deprived of any redress because the director judges that he "failed to protect himself" or that a > subsequent action by him was "wild and gambling". Just two examples of unfair and unnecessary rules. Such rules are biased in favour of experts. They add zero > enjoyment to the game. To the first I agree. To the second not necessarily. I dislike double shots and wild actions based on the presumptions that the TD will give them their normal result. No, IMO NOS should still play normal bridge. It can be poor, inferior, but not gambling or wild. Otherwise they could be equally viewed as second "OS" and rightly denied redress for such selfinflicted result. > I suppose that "Equity" rules may become more popular as more and more players benefit from breaking them. Yes, I'm afraid of that too. > IMO, separate procedural and disciplinary penalties have their place but appropriate redress and deterrence should be built into most laws and regulations. Those penalties are not given. At least in Poland. Some TD's want to give them, but our TD gurus discourage this. And those gurus decide if you are eligible to direct best and most profitable events. The choice is yours ;-) Regards Maciej From svenpran at online.no Mon Nov 17 23:20:02 2008 From: svenpran at online.no (Sven Pran) Date: Mon, 17 Nov 2008 23:20:02 +0100 Subject: [blml] A player's comment In-Reply-To: References: <81C68581-7E16-4850-8930-B230C6D06D3A@starpower.net> <000a01c948d4$45372bc0$cfa58340$@no> Message-ID: <000001c94902$a2f02860$e8d07920$@no> On Behalf Of Robert Frick .............. > IMO, "misleading explanation" is a clearly defined technical term in the > laws. If my partner bids 4NT thinking that it is for the minors, and it > really is supposed to be unusual, then my description of his bid as being > for the minors is legal and appropriate. But it is still a "misleading > explanation". I'm sorry but the only place the word "misleading" occurs in the laws is in the beginning of Law 75, and from the context it is perfectly clear that "misleading explanation" is an explanation that does not reflect the partnership understanding. So an explanation that misleads opponents as to what cards the partner actually holds (only) because that partner has not called according to his partnership understanding is NOT a "misleading explanation". > > In this situation, I can do not better than describe his bid as being for > the minors; I have no clue that I am giving a misleading explanation. And you are not! > > In the case we are discussing, the player knew that partner had > misunderstood this auction. You can claim that players should be required > to provide a misleading explanation in this situation, and you will have a > lot of company on blml and support from the WBFLC. The requirement by the laws is still that the explanation shall reflect the partnership understanding and nothing else. > > I will only argue if you try to claim that the laws clearly demand that or > if you claim that is a wise thing for the WBF to insist upon. They certainly do demand that. > > > > > > Law 75 explicitly requires every explanation to be correct description of > > the partnership understandings related to a specific call, not a > > description > > on what cards a player may (probably) hold even if the player explaining > > partner's call can clearly see that his partner must have misbid. > > Actually, Law 75 says nothing of the kind, that I can see. Law 75 is about > the consequences of an unintended misleading explanation. THe only > exception I can is "South's responsibility is to act as though North had > made a strong game try opposite a weak response.." and "South must do > nothing to correct the mistaken explanation while the auction continues." > But this of course undermines the position you wish to argue for. >From Law 75B: "since East-West are entitled to an accurate description of the North-South agreement" >From Law 75C: "since East-West did receive an accurate description of the North-South agreement; they have no claim to an accurate description of the North-South hands." > > > > > ............... > >> > >> The opponents did not ask "What would 2Sp mean if 2 Di was multi?" They > >> did not ask "What would 2Sp mean if everyone has properly understood all > >> of the bids in the auction." > > > > Such questions would be improper. > > Can you give me the law for this? >From Law 20F1: "He is entitled to know about calls actually made, about relevant alternative calls available that were not made, and about relevant inferences from the choice of action where these are matters of partnership understanding." A call that is not included in the partnership understandings is certainly not "relevant". Every explanation shall be given under the assumption that there is no misunderstanding in the auction so specifying this in a question comes very close to causing "annoyance or embarrassment" that is prohibited in Law 74A2. > > > > >> > >> The probably just asked what 2Sp meant. And you know the intended > >> meaning > >> (in this example). So it you want to be honest and responsive, just > >> clarify for the opponents that you are not describing your partner's > >> intended meaning, you are describing the sytemic meaning of partner's > >> bid > >> had partner understood the auction properly. Or you can follow the laws, > >> your choice. I am not arguing that is lawful to be open and honest and > >> responsive when answering opponent's questions. > > > > Law 75 requires you to honestly explain the meaning according to your > > partnership understanding even when you understand that your partner has > > misbid. > > Which section of the law are you referring to? Again, I cannot find it. Carefully read and understand Laws 20 and 75 and I trust you will find it. Sven From svenpran at online.no Mon Nov 17 23:30:13 2008 From: svenpran at online.no (Sven Pran) Date: Mon, 17 Nov 2008 23:30:13 +0100 Subject: [blml] A player's comment In-Reply-To: References: <81C68581-7E16-4850-8930-B230C6D06D3A@starpower.net> Message-ID: <000101c94904$0f137500$2d3a5f00$@no> On Behalf Of Robert Frick > I will try to be clear about this. You bid 4NT, correctly intending it to > be for the minors. Your partner explains this as asking for controls and > bids 5Di. You -- sooner or later -- are asked what this bid means. > > You presumably have partnership agreements, understandings, etc. about the > meaning of 5Di as a response to 4NT for the minors. > > But many times you also have partnership agreements, understandings, past > history, whatever, that gives you more knowledge than the opponents about > what 5 Di means as a Blackwood response. (and if you don't, you can at > least say no agreement.) > > So you have a choice: There are two different sets of agreements, etc. > that you can choose to divulge. You have no choice: You are required to explain that the 5D bid shows preference for diamonds and nothing else. Furthermore you are explicitly prohibited from taking any action whatsoever that "could have been suggested" by your possible understanding that partner has misbid and given an answer to a Blackwood 4NT bid. > > People have argued on blml that players are not required to divulge the > second set of partnership agreements. This was not a popular opinion, and > you did not agree with it (responding to Eric here), as I recall. The > WBFLC has supported this opinion. So if the opponents, who are now > defending and what to know about controls, you do not have to answer. I > believe Richard will argue that you are required by law not to answer, as > your chief goal is supposed to be to win. > > I don't see where the laws require you to provide the first set of > partnership agreements and not the other. You can say it is in the laws a > hundred times and I still won't see it. To me, the laws just talk about > divulging relevant partnership agreements. > > Then, if you are going to offer just one set of agreements, which one do > you do? I can see the laws very clearly saying that you offer the > explanation that does not clue in partner that partner misunderstood your > bid. I can't see where the laws otherwise say to prefer one partnership > agreement over another. So I keep asking what law you are talking about > when you claim the laws support telling the second explanation. Laws 20 and 75 clearly tells you that opponents shall be given explanation on your agreements, not on what you suspect or know to the effect that partner has misbid unless your suspicion or knowledge comes from legal calls and/or play. However, if at the end of the play suspicion has arisen that you may have used such unauthorized information to your advantage then the Director is obliged to adjust the result on the board and take away from you whatever you may have gained from this irregularity. This should be clear enough? sven From grandaeval at tiscali.co.uk Tue Nov 18 00:39:14 2008 From: grandaeval at tiscali.co.uk (Grattan) Date: Mon, 17 Nov 2008 23:39:14 -0000 Subject: [blml] [Fwd: Amsterdam] References: <4920195A.1070509@talktalk.net><200811161342.AA16553@geller204.nifty.com> <4920B59E.8080202@talktalk.net><000701c94857$8c51de60$15844c59@chello.pl><4921C615.7030501@talktalk.net> <000701c94901$2ed86ba0$15844c59@chello.pl> Message-ID: <002201c9490d$b63d6850$0302a8c0@Mildred> Grattan Endicott To: "Bridge Laws Mailing List" Sent: Monday, November 17, 2008 10:09 PM Subject: [blml] [Fwd: Amsterdam] >> [Nige1] So David, Grattan, Kojak, Ton can rightfully claim that in their opinion players like present Laws and their voice has equal (for them higher) significance than ours. > +=+ Misses the target. The key would be widespread requests from NBOs and Zones. They are the members of the WBF and have the voting powers. We had this re Law 25B and to a lesser extent re claims procedure and the Maastricht case. We have also been made aware that tournament organizers want to limit permitted systems etc. at times. ~ Grattan ~ +=+ From gro at bridgepro.de Tue Nov 18 00:58:20 2008 From: gro at bridgepro.de (Gro) Date: Tue, 18 Nov 2008 00:58:20 +0100 Subject: [blml] Appeals procedures In-Reply-To: <009d01c948fe$6c45da20$0302a8c0@Mildred> References: <009d01c948fe$6c45da20$0302a8c0@Mildred> Message-ID: <4922051C.9060805@bridgepro.de> Hi, Grattan schrieb: > Grattan Endicott also ************************************ > "We desire truth and find within > ourselves only uncertainty." > [Pascal] > ''''''''''''''''''''''''''''''''''''''''''''''''''''''''''''''''''''''''''''''''''''''''''''''''' > +=+ Many moons ago ton kooijman rightly observed > that there is no restriction in the laws on the manner in > which an AC may deal with three simultaneous appeals > from the same match. The laws only set the basics; it is > left to regulations to specify detail. > The EBL has two and a half pages of regulations > on the subject. > G.1.1 deals with preliminaries and in particular the > involvement of the Director. The Director must be given > notification of the appeal and then has certain duties to > perform. At the appeal the appealing side must be > represented. The responding side has a right to attend > and be heard. If the responders are not present their > case may be damaged by the inability of the committee > to ask questions. > G.1.2 deals with committee procedure. It does not > restrict the discretion of the committee as to the way in > which it will handle a number of appeals from the same > match. It will take into account the representations or > statement of any person, and any documentary evidence. > It may interview anyone it wishes. > G21,2,3 deal with the structure of the committee, its > power to investigate, its powers and delivery of judgment. > ........................................................................... > In my experience the common way to deal with multiple > appeals involving the same contestants is to hear the > evidence of each successively, then retire and deliberate > upon them one by one. Subsequently the several decisions > are conveyed to the Director simultaneously. But there is > nothing sacrosanct about it. > On the other hand, there is a > worry in such cases when a series of attempts are made > to overturn the result of a narrowly fought match. In this > area the evidence of the Director is important; when he > was called, the basis of his ruling. When the Director was > not called at the time and it appears the losing side is > digging up anything it can in the situation the Director and > the AC is expected to exercise the strongest scepticism. > ~ Grattan ~ +=+ > why do I so strongly think about 8 days ago in Amsterdam....:) thank you for this statement- I liked it very much! Michael From darkbystry at wp.pl Tue Nov 18 01:37:33 2008 From: darkbystry at wp.pl (Bystry) Date: Tue, 18 Nov 2008 01:37:33 +0100 Subject: [blml] [Fwd: Amsterdam] References: <4920195A.1070509@talktalk.net><200811161342.AA16553@geller204.nifty.com> <4920B59E.8080202@talktalk.net><000701c94857$8c51de60$15844c59@chello.pl><4921C615.7030501@talktalk.net><000701c94901$2ed86ba0$15844c59@chello.pl> <002201c9490d$b63d6850$0302a8c0@Mildred> Message-ID: <000501c94915$d9653800$15844c59@chello.pl> Grattan, > Grattan Endicott also ************************************ > "We desire truth and find within > ourselves only uncertainty." > [Pascal] > '''''''''''''''''''''''''''''''''''''''''''''''''''''''''''''''''''''''''''' ''''''''''''''''''''' > > > > ----- Original Message ----- > From: "Bystry" > To: "Bridge Laws Mailing List" > Sent: Monday, November 17, 2008 10:09 PM > Subject: [blml] [Fwd: Amsterdam] > > > So David, Grattan, Kojak, Ton can rightfully claim that > in their opinion players like present Laws and their voice > has equal (for them higher) significance than ours. > > > +=+ Misses the target. The key would be widespread > requests from NBOs and Zones. They are the members > of the WBF and have the voting powers. > We had this re Law 25B and to a lesser extent re > claims procedure and the Maastricht case. We have also > been made aware that tournament organizers want to > limit permitted systems etc. at times. > ~ Grattan ~ +=+ a) I think you misunderstood about what I was writing. I simply refered to the statements from some prominent TD's. I'm not fully sure that you did it, but I can easily find the quotations from David, Ton and Kojak which stated something similar to: "players like equity laws or old 12C3, now 12C1 (if i'm not mistaken)". And I think people don't like them or more precise - frequent offenders like them, the others not. So my statement was made in this context. b) I accept that WBF doesn't want to engage in any player polls or in the other methods of gathering opinions. But in reality NBO's and Zones are represented by the officials, mainly top TD's. So de facto _only TD's_ decide what is good for players. I'm not aware of any poll made by any NBO among their members. If somebody have heard about such poll please tell me. Therefore my chance of representing my opinion is close to nil, what counts is an opinion of Mr. Latala and some small group of our top TD. Now, we have wideworld forum for expressing opinions, namely the Internet. So why couldn't WBF (or its Laws Comission) conduct some polls on its page? Wouldn't it be useful source of information what is players' opinion about TFLB? I repeat, for me and many other players it is virtually impossible to get to attention of our NBO's because it would require collecting thousands of votes agreeing with official petition. Regards Maciej From jfusselman at gmail.com Tue Nov 18 04:11:53 2008 From: jfusselman at gmail.com (Jerry Fusselman) Date: Mon, 17 Nov 2008 21:11:53 -0600 Subject: [blml] A player's comment [SEC=UNOFFICIAL] In-Reply-To: References: Message-ID: <2b1e598b0811171911w608a097bx3654e7ee5e51c53c@mail.gmail.com> On Mon, Nov 17, 2008 at 3:03 PM, wrote: > Eric Landau asserted: > > [big snip] > >>Which puts us back where we started, with the question of >>what to do when you know that your version of your systemic >>understanding differs from your partner's but do not know >>which of you is right. >> >>That is a question we have yet to answer. > > Law 20F5(b): > > "The player must call the Director and inform his opponents > that, **in his opinion**, his partner's explanation was > erroneous (see Law 75) but only at his first legal > opportunity..." > > What's the problem? > The problem is that it ignores probabilities, as I have stressed several times before. What do you say if your probability of error in your original statement is 40%? 60%? What if you think your probability of error is 40%, but you believe that the director is 60% likely to rule that you are the one who erred? The wording in the law gives guidance when you are almost certain one way or the other. It is unclear in other cases. It also ignores the simple truth the director is the one who rules what your agreements were. I think Eric's final sentence above is correct, and I wonder how MS can be so clear, obvious, and perfect when Eric's question is eternally unanswered. In fairness, there have been a few brief stabs at answering it, but there is no consensus among MS endorsers. Jerry Fusselman From richard.hills at immi.gov.au Tue Nov 18 06:52:13 2008 From: richard.hills at immi.gov.au (richard.hills at immi.gov.au) Date: Tue, 18 Nov 2008 16:52:13 +1100 Subject: [blml] A Bruce's comment [SEC=UNOFFICIAL] In-Reply-To: <2b1e598b0811171911w608a097bx3654e7ee5e51c53c@mail.gmail.com> Message-ID: Eric Idle, Bruces' Philosophers Song (verse eight): Aristotle, Aristotle was a bugger for the bottle. Hobbes was fond of his dram, And Ren? Descartes was a drunken fart. "I drink, therefore I am." Jerry Fusselman asserted: >The problem is that it ignores probabilities, as I have >stressed several times before. What do you say if your >probability of error in your original statement is 40%? >60%? Law 45C2: "Declarer must play a card from his hand if it is (a) held face up, touching or nearly touching the table; or (b) maintained in such a position as to indicate that it has been played." Richard Hills: Aristotle's Law of the Excluded Middle -- "Either A must be true, or not-A must be true, but no third option can be true" -- applies to a Director determining whether or not declarer's card has been played. The card cannot be "probably" played or "partially" played. Likewise, the Law of the Excluded Middle applies to a Director's decision on whether misinformation has occurred. The Director may not rule that a misinformation infraction "probably" or "partially" occurred. Jerry Fusselman asserted: >What if you think your probability of error is 40%, but >you believe that the director is 60% likely to rule that >you are the one who erred? > >The wording in the law gives guidance when you are almost >certain one way or the other. It is unclear in other >cases. It also ignores the simple truth the director is >the one who rules what your agreements were. [snip] Richard Hills: Jerry Fusselman is ignoring the simple truth that there are a number of Laws which require me to act in accordance with my own knowledge and belief. For example, "I think, therefore I am" electing to "knowingly" return an unearned trick under Law 79A2. There is no Law which requires me to lie to the opponents, in order to save time, because I have a very well-founded suspicion that the Leviathan Director would arbitrarily assess my truth-telling as falsehood, due to the Director's bias under his Hobbesian philosophy. Like another arbitrary ruler, Charles I, it is the arbitrary Director's career as a Director which will be nasty, brutish and short. Best wishes Richard James Hills Recruitment Section, Level 3 Blue, workstation 15 (first on left) Department of Immigration and Citizenship Telephone: 02 6223 8453 Email: richard.hills at immi.gov.au -------------------------------------------------------------------- Important Notice: If you have received this email by mistake, please advise the sender and delete the message and attachments immediately. This email, including attachments, may contain confidential, sensitive, legally privileged and/or copyright information. Any review, retransmission, dissemination or other use of this information by persons or entities other than the intended recipient is prohibited. DIAC respects your privacy and has obligations under the Privacy Act 1988. The official departmental privacy policy can be viewed on the department's website at www.immi.gov.au. See: http://www.immi.gov.au/functional/privacy.htm --------------------------------------------------------------------- From rfrick at rfrick.info Tue Nov 18 07:04:40 2008 From: rfrick at rfrick.info (Robert Frick) Date: Tue, 18 Nov 2008 01:04:40 -0500 Subject: [blml] A player's comment In-Reply-To: <000001c94902$a2f02860$e8d07920$@no> References: <81C68581-7E16-4850-8930-B230C6D06D3A@starpower.net> <000a01c948d4$45372bc0$cfa58340$@no> <000001c94902$a2f02860$e8d07920$@no> Message-ID: On Mon, 17 Nov 2008 17:20:02 -0500, Sven Pran wrote: > On Behalf Of Robert Frick > .............. >> IMO, "misleading explanation" is a clearly defined technical term in the >> laws. If my partner bids 4NT thinking that it is for the minors, and it >> really is supposed to be unusual, then my description of his bid as >> being >> for the minors is legal and appropriate. But it is still a "misleading >> explanation". > > I'm sorry but the only place the word "misleading" occurs in the laws is > in > the beginning of Law 75, and from the context it is perfectly clear that > "misleading explanation" is an explanation that does not reflect the > partnership understanding. It is very difficult for me to believe that anyone could come to this conclusion from reading the new laws. Perhaps that is my failure of imagination. The 1997 laws state "Two examples may clarify responsibilities of the players (and the director) after a misleading explanation has been given to the opponents." The two examples are titled "Mistaken Explanation" and "Mistaken Bid". As in the new laws, there is no hint that a mistaken bid/call is not a misleading explanation. There is no hint that the laws inexplicably change terminology and use mistaken explanation as synonymous with misleading explanation. There is no use of any other terminology to describe the discrepancy between an explanation and a bid's actual intended meaning, which is the commonality between the two examples. > > So an explanation that misleads opponents as to what cards the partner > actually holds (only) because that partner has not called according to > his > partnership understanding is NOT a "misleading explanation". > >> >> In this situation, I can do not better than describe his bid as being >> for >> the minors; I have no clue that I am giving a misleading explanation. > > And you are not! > >> >> In the case we are discussing, the player knew that partner had >> misunderstood this auction. You can claim that players should be >> required >> to provide a misleading explanation in this situation, and you will >> have a >> lot of company on blml and support from the WBFLC. > > The requirement by the laws is still that the explanation shall reflect > the > partnership understanding and nothing else. I don't think you understand the issue as I have described it. Or you just deny it exists, I can't tell. Bob From grandaeval at tiscali.co.uk Tue Nov 18 08:50:59 2008 From: grandaeval at tiscali.co.uk (Grattan) Date: Tue, 18 Nov 2008 07:50:59 -0000 Subject: [blml] Appeals procedures References: <009d01c948fe$6c45da20$0302a8c0@Mildred> <4922051C.9060805@bridgepro.de> Message-ID: <001401c94952$84db24b0$0302a8c0@Mildred> Grattan Endicott To: "Bridge Laws Mailing List" Sent: Monday, November 17, 2008 11:58 PM Subject: Re: [blml] Appeals procedures > Hi, > > > Grattan schrieb: >> Grattan Endicott> also > ************************************ >> "We desire truth and find within >> ourselves only uncertainty." >> [Pascal] >> ''''''''''''''''''''''''''''''''''''''''''''''''''''''''''''''''''''''''''''''''''''''''''''''''' >> +=+ Many moons ago ton kooijman rightly observed >> that there is no restriction in the laws on the manner in >> which an AC may deal with three simultaneous appeals >> from the same match. The laws only set the basics; it is >> left to regulations to specify detail. >> The EBL has two and a half pages of regulations >> on the subject. >> G.1.1 deals with preliminaries and in particular the >> involvement of the Director. The Director must be given >> notification of the appeal and then has certain duties to >> perform. At the appeal the appealing side must be >> represented. The responding side has a right to attend >> and be heard. If the responders are not present their >> case may be damaged by the inability of the committee >> to ask questions. >> G.1.2 deals with committee procedure. It does not >> restrict the discretion of the committee as to the way in >> which it will handle a number of appeals from the same >> match. It will take into account the representations or >> statement of any person, and any documentary evidence. >> It may interview anyone it wishes. >> G21,2,3 deal with the structure of the committee, its >> power to investigate, its powers and delivery of judgment. >> ........................................................................... >> In my experience the common way to deal with multiple >> appeals involving the same contestants is to hear the >> evidence of each successively, then retire and deliberate >> upon them one by one. Subsequently the several decisions >> are conveyed to the Director simultaneously. But there is >> nothing sacrosanct about it. > > >> On the other hand, there is a >> worry in such cases when a series of attempts are made >> to overturn the result of a narrowly fought match. In this >> area the evidence of the Director is important; when he >> was called, the basis of his ruling. When the Director was >> not called at the time and it appears the losing side is >> digging up anything it can in the situation the Director and >> the AC is expected to exercise the strongest scepticism. >> ~ Grattan ~ +=+ >> > why do I so strongly think about 8 days ago in Amsterdam....:) > > thank you for this statement- I liked it very much! > > Michael > +=+ Timeo Danaos et dona ferentes. I do not suggest that the 3NT bid can be justified. Returned to due process I would expect the Director to disallow it and for a diamond contract, 4 or 5, to be substituted. I believe only an opening club lead beats 5D because of the location of the C9 and of the HK. To me that suggests a 12C1(c) adjustment. ~ Grattan ~ +=+ From Hermandw at skynet.be Tue Nov 18 09:16:36 2008 From: Hermandw at skynet.be (Herman De Wael) Date: Tue, 18 Nov 2008 09:16:36 +0100 Subject: [blml] A player's comment In-Reply-To: <81C68581-7E16-4850-8930-B230C6D06D3A@starpower.net> References: <81C68581-7E16-4850-8930-B230C6D06D3A@starpower.net> Message-ID: <492279E4.5010801@skynet.be> Eric Landau wrote: > > You cannot be "deliberately misdecribing partner's cards" when you in > no way purport to be describing partner's cards. > >> Right. But the topic was honesty, not following the laws. > > Honesty requires both truth and responsiveness. It is not "honest" > to reply to a question by offering what would be a legimtate answer > had you been asked an entirely different question, even though it may > be undeniably true. > And what was the question. Usually "what does he have?" or even "what does he show?", not "what does that bid mean in the system you think you are playing?". As an answer to that third question, "heart support" is certainly true, but in answer to the first and second question "spades" is true. Now we were not talking about legal issues, simply about misleading. Saying "hearts" when you know the answer to the question they are asking is "spades" is misleading. And of course it's deliberate. > Eric Landau Herman. From Hermandw at skynet.be Tue Nov 18 09:21:16 2008 From: Hermandw at skynet.be (Herman De Wael) Date: Tue, 18 Nov 2008 09:21:16 +0100 Subject: [blml] A player's comment In-Reply-To: <000101c94904$0f137500$2d3a5f00$@no> References: <81C68581-7E16-4850-8930-B230C6D06D3A@starpower.net> <000101c94904$0f137500$2d3a5f00$@no> Message-ID: <49227AFC.3050208@skynet.be> Sven - again! Sven Pran wrote: > > Furthermore you are explicitly prohibited from taking any action whatsoever > that "could have been suggested" by your possible understanding that partner > has misbid and given an answer to a Blackwood 4NT bid. > Said 27 times and answered 28 times: L16 does not forbid you from taking any other actions than calls and plays. Drop this argument now Sven! It is WRONG. Herman. From Hermandw at skynet.be Tue Nov 18 09:26:54 2008 From: Hermandw at skynet.be (Herman De Wael) Date: Tue, 18 Nov 2008 09:26:54 +0100 Subject: [blml] A Bruce's comment [SEC=UNOFFICIAL] In-Reply-To: References: Message-ID: <49227C4E.3050007@skynet.be> richard.hills at immi.gov.au wrote: > > There is no Law which requires me to lie to the opponents, > in order to save time, because I have a very well-founded > suspicion that the Leviathan Director would arbitrarily > assess my truth-telling as falsehood, due to the Director's > bias under his Hobbesian philosophy. > Yes there is. L20F5a. This law requires you to keep quiet over an untruth spoken by partner. If you don't call that lying then you exhibit a level of hypocrisy which I associate with the Catholic Church, not with people thinking from reason. And no, this is not the end of the discussion. But I don't like seeing half-truths given away in defence of a set of actions which is, at best, acceptable because such a large majority believes they are acceptable. I think your actions are comtemptible. You deliberately give UI, you give opponents deliberate misleading information, and you create a game in which a totally different outcome is reached depending on a totally unimportant piece of action by opponents (an action, moreover, that we do not wish to encourage but that you give a huge advantage to). But I am not entering this discussion. Herman. From harald.skjaran at gmail.com Tue Nov 18 09:27:09 2008 From: harald.skjaran at gmail.com (=?UTF-8?Q?Harald_Skj=C3=A6ran?=) Date: Tue, 18 Nov 2008 09:27:09 +0100 Subject: [blml] [Fwd: Amsterdam] In-Reply-To: <2a1c3a560811162139i75ae4c30vb312182cd88de91c@mail.gmail.com> References: <001301c947d7$302631c0$0302a8c0@Mildred> <200811161200.AA16549@geller204.nifty.com> <000a01c947f8$ef0a9740$0302a8c0@Mildred> <49208B39.9020001@talktalk.net> <067FB1FC6E994CEB9CAAC274D24F7C2D@JOHN> <2a1c3a560811162139i75ae4c30vb312182cd88de91c@mail.gmail.com> Message-ID: On 17/11/2008, Wayne Burrows wrote: > 2008/11/17 John (MadDog) Probst : > > > > ----- Original Message ----- > > From: "Nigel Guthrie" > > To: "Bridge Laws Mailing List" > > Sent: Sunday, November 16, 2008 9:06 PM > > Subject: Re: [blml] [Fwd: Amsterdam] > > > > > >> [John (MadDog) Probst] > >> Ye Gods, do we play bridge any more? It's GBK, not a HUM or a dingbat or > >> a cormorant or any other sort of artefact. Call it a psyche if you will, > >> but if I didn't expect it I shouldn't be at the event. This nonsense of > >> Nigel's really does take the biscuit for complete triviality. If my > >> opponents didn't open these some of the time I'd call the TD and ask him > >> to check for breathing and a pulse. > >> > >> [Nige1] > >> A game is its rules. > >> > >> John. Tim, and other TDs often repudiate rules that restrict light > >> opening bids. Do they appreciate how unfair this is to players who > >> handicap themselves by sticking to the rules? Other groups criticise > >> other rules.. Some ordinary players also rationalise the breaking of > >> rules that they dislike. Should different groups of rule-breakers be > >> treated differently? > >> > >> Bridge rules often seem trivial and nonsensical. IMO some are > >> unnecessary and many are overly sophisticated and subjective.Some, like > >> Rob and John.approve the breaking of daft rules. Others would rather > >> campaign to change them. Our hope is that players will be more likely > >> to comply with rules that they understand. To my mind, John's biscuit > >> metaphor has some bite. Players deserve nice plain rules to replace > >> those that are crackers and half-baked. > >> > > > > Nice one Nigel! I'm perfectly comfortable with booking this as a psyche > > (Let's assume I'd ascertained that we don't have felonious agreements and > > this crumby hand is not by agreement opened). As the spectator said "They do > > that in Norway". Quite possibly true. They do it in England too. They don't > > do it in America. The information content of the spectator's comment is not > > an indictment of the partnership method, just a comment about an ethnic > > style. > >> > > That is exactly the question. > > Is this opening a departure from their methods or is it part of their methods? It's a departure from their methods. They DO make some light 3rd seat openings, Helgemo a tad lighter then Helness. But this hand is outside their range. And it's far from standard to open the actual hand in Norway, very, very few would. And if they were aware of how our LC looks at this, nobody would open the actual hand in Norway - since it's deemed to be a HUM here. It's OK to open light, but just traversing the line to what's considered HUM is NOT a psyche, according to our LC. > > -- > Wayne Burrows > Palmerston North > New Zealand > > _______________________________________________ > blml mailing list > blml at amsterdamned.org > http://www.amsterdamned.org/mailman/listinfo/blml > -- Kind regards, Harald Skj?ran From Hermandw at skynet.be Tue Nov 18 09:34:13 2008 From: Hermandw at skynet.be (Herman De Wael) Date: Tue, 18 Nov 2008 09:34:13 +0100 Subject: [blml] [Fwd: Amsterdam] In-Reply-To: <7AC9477A-5983-46F5-BF05-03284873939F@starpower.net> References: <4920195A.1070509@talktalk.net> <200811161342.AA16553@geller204.nifty.com> <4920B59E.8080202@talktalk.net> <7AC9477A-5983-46F5-BF05-03284873939F@starpower.net> Message-ID: <49227E05.9030408@skynet.be> Eric Landau wrote: > > If you drive 110 when the speed limit is 100 you are breaking the law > and you know it, even if everyone else is driving 110. But for some > pairs the situation is like having a speed limit of 100 km/hr when > they have a speedometer marked in miles/hr and don't know what a km is. > But that is their problem. If they want to know what the speed limit is in m/hr, they can make the change. And if that is too difficult (because bridge valuation is not an exact correspondence), then they just have to add a second speedometer to their car and check it at some times. Anyway, if they drive 105, they are breaking the law. Similarly, if the limit is 8HCP, then all GH needs to do if he wants to know if he is breaking the limit is count his HCP. There is no excuse for breaking HUM regulations in this matter, IMO. If only the HUM regulations were followed by players and directors alike. If there is never any speed control, then all the cars will be driving 130 or more. Except for me, of course, and I don't like all those cars whizzing past me! Herman. > > Eric Landau From Hermandw at skynet.be Tue Nov 18 09:35:26 2008 From: Hermandw at skynet.be (Herman De Wael) Date: Tue, 18 Nov 2008 09:35:26 +0100 Subject: [blml] [Fwd: Amsterdam] In-Reply-To: References: <001301c947d7$302631c0$0302a8c0@Mildred> <200811161200.AA16549@geller204.nifty.com> <000a01c947f8$ef0a9740$0302a8c0@Mildred> <49208B39.9020001@talktalk.net> <067FB1FC6E994CEB9CAAC274D24F7C2D@JOHN> <2a1c3a560811162139i75ae4c30vb312182cd88de91c@mail.gmail.com> Message-ID: <49227E4E.5000502@skynet.be> Harald Skj?ran wrote: >> >> Is this opening a departure from their methods or is it part of their methods? > > It's a departure from their methods. They DO make some light 3rd seat > openings, Helgemo a tad lighter then Helness. But this hand is outside > their range. > > And it's far from standard to open the actual hand in Norway, very, > very few would. And if they were aware of how our LC looks at this, > nobody would open the actual hand in Norway - since it's deemed to be > a HUM here. It's OK to open light, but just traversing the line to > what's considered HUM is NOT a psyche, according to our LC. Sensible answer from Norway. So if Norwegian players know the laws are like this, why don't WBF directors apply them as well? Herman. From darkbystry at wp.pl Tue Nov 18 09:46:48 2008 From: darkbystry at wp.pl (Bystry) Date: Tue, 18 Nov 2008 09:46:48 +0100 Subject: [blml] A player's comment References: <81C68581-7E16-4850-8930-B230C6D06D3A@starpower.net> <492279E4.5010801@skynet.be> Message-ID: <000d01c9495a$32a92540$15844c59@chello.pl> Herman, > > Honesty requires both truth and responsiveness. It is not "honest" > > to reply to a question by offering what would be a legimtate answer > > had you been asked an entirely different question, even though it may > > be undeniably true. > > > > And what was the question. Usually "what does he have?" or even "what > does he show?", not "what does that bid mean in the system you think you > are playing?". Not so. The Laws order you to explain your calls in accordance with your partnership agreements. And the Laws allow you to ask only about the meanings of those calls in accordance with your opponents' partnership agreements. Therefore questions like "what does he have?" are merely an inappropriate form of correct question "what does your partner's bidding mean in accordance with your partnership agreements?". Clear? > As an answer to that third question, "heart support" is certainly true, > but in answer to the first and second question "spades" is true. No. The only correct and truthful answer is "hearts". > Now we were not talking about legal issues, simply about misleading. > Saying "hearts" when you know the answer to the question they are asking > is "spades" is misleading. And of course it's deliberate. No. You don't mislead them. Only their poor stated questions mislead them. You could equally deny to answer inappropriate questions like "what does he have?" but it's easier and more pleasant to omit this unnecessary stage and answer directly to correctly and lawfully stated question. > Herman. Regards Maciej From darkbystry at wp.pl Tue Nov 18 10:17:19 2008 From: darkbystry at wp.pl (Bystry) Date: Tue, 18 Nov 2008 10:17:19 +0100 Subject: [blml] A Bruce's comment [SEC=UNOFFICIAL] References: <49227C4E.3050007@skynet.be> Message-ID: <001501c9495e$75116ec0$15844c59@chello.pl> Herman, > > There is no Law which requires me to lie to the opponents, > > in order to save time, because I have a very well-founded > > suspicion that the Leviathan Director would arbitrarily > > assess my truth-telling as falsehood, due to the Director's > > bias under his Hobbesian philosophy. > > > > Yes there is. L20F5a. > > This law requires you to keep quiet over an untruth spoken by partner. > > If you don't call that lying then you exhibit a level of hypocrisy which > I associate with the Catholic Church, not with people thinking from reason. It would be better if you didn't offend the feelings of many Catholics. I'm not a member of CC and could even agree with you but I don't consider public forums unrelated to this topic to be proper places to state such opinions. Second thing - there is a distinct difference between lying and keeping quiet over an unintented untruth. If you don't understand it I'm not the one who will try to explain this to you. > And no, this is not the end of the discussion. But I don't like seeing > half-truths given away in defence of a set of actions which is, at best, > acceptable because such a large majority believes they are acceptable. I don't pretend to speak for the majority. I comply with the CS because of it's merits and because the Laws order me to do it, not because of the effect of flock. > I think your actions are comtemptible. Arguable statement. > You deliberately give UI, Yes. And what is wrong with giving additional UI? It can't hurt the NOS. At least you don't deliberately give MI which is illegal and may be misrepresented as cheating by your opponents. > you give opponents deliberate misleading information, No. You tell them only that to what they are entitled - your partnership agreements. And you definitely don't mislead them. In effect you inform them that your side is having a mixup. And that is true and that can only help them. > and you create a game in which a totally different outcome > is reached depending on a totally unimportant piece of action by > opponents (an action, moreover, that we do not wish to encourage but > that you give a huge advantage to). Herman, but Bridge works this way. Asking about alerted calls is necessary to play this game. Imagine an auction: 1C - 1D - 1H(A) - ? If you don't ask about alert, bid 2S, get doubled and later you realize that 1H was a transfer to spades whom do you want to blame? Questioning _is not_ unimportant, it's the most frequently used way of getting information. And you need this information to play real Bridge. > But I am not entering this discussion. > Herman. Regards Maciej From darkbystry at wp.pl Tue Nov 18 10:42:41 2008 From: darkbystry at wp.pl (Bystry) Date: Tue, 18 Nov 2008 10:42:41 +0100 Subject: [blml] Appeals procedures References: <009d01c948fe$6c45da20$0302a8c0@Mildred><4922051C.9060805@bridgepro.de> <001401c94952$84db24b0$0302a8c0@Mildred> Message-ID: <002701c94962$0024bc80$15844c59@chello.pl> Grattan, ----- Original Message ----- From: "Grattan" To: "Bridge Laws Mailing List" Sent: Tuesday, November 18, 2008 8:50 AM Subject: Re: [blml] Appeals procedures > +=+ Timeo Danaos et dona ferentes. > I do not suggest that the 3NT bid can be justified. Returned > to due process I would expect the Director to disallow it and > for a diamond contract, 4 or 5, to be substituted. I believe only an > opening club lead beats 5D because of the location of the C9 and > of the HK. To me that suggests a 12C1(c) adjustment. > ~ Grattan ~ +=+ Yes, only an opening club lead (which is certainly the best one anyway) definitively beats 5D but it nevertheless requires declarer to guess DQ which is improbable. So even if we use 12C1(c) the weight of 5D= should be very small. Regards Maciej From svenpran at online.no Tue Nov 18 11:32:31 2008 From: svenpran at online.no (Sven Pran) Date: Tue, 18 Nov 2008 11:32:31 +0100 Subject: [blml] A player's comment In-Reply-To: References: <81C68581-7E16-4850-8930-B230C6D06D3A@starpower.net> <000a01c948d4$45372bc0$cfa58340$@no> <000001c94902$a2f02860$e8d07920$@no> Message-ID: <001201c94968$f6785f00$e3691d00$@no> On Behalf Of Robert Frick ............. > As in the new laws, there is no hint that a mistaken bid/call is not a > misleading explanation. There is no hint that the laws inexplicably change > terminology and use mistaken explanation as synonymous with misleading > explanation. There is no use of any other terminology to describe the > discrepancy between an explanation and a bid's actual intended meaning, > which is the commonality between the two examples. .............. > I don't think you understand the issue as I have described it. Or you just > deny it exists, I can't tell. Quite possibly I don't understand you, but please just consider: A call is an action in which you make a pass, a Double, a redouble or a bid. An explanation is a description of the information available from this call. The laws require all explanations to reflect partnership understandings and nothing else. An explanation is mistaken (or misleading) if it does not convey all the information related to this call according to the partnership understandings. A call not in accordance with the partnership understanding does not make the corresponding correct explanation misleading even though this explanation no longer describes the actual hand. A call can also be misleading; the best example is a psychic call. But calls are never "explanations" of anything? So "misleading call" cannot be synonymous with "misleading explanation". Regards Sven From svenpran at online.no Tue Nov 18 11:42:17 2008 From: svenpran at online.no (Sven Pran) Date: Tue, 18 Nov 2008 11:42:17 +0100 Subject: [blml] A player's comment In-Reply-To: <492279E4.5010801@skynet.be> References: <81C68581-7E16-4850-8930-B230C6D06D3A@starpower.net> <492279E4.5010801@skynet.be> Message-ID: <001301c9496a$53feac50$fbfc04f0$@no> On Behalf Of Herman De Wael .................. > And what was the question. Usually "what does he have?" "What does he have" is an improper question and should never be answered literally. > "What does he show?", not "what does that bid mean in the system you think you > are playing?". These two questions are equivalent, they both literally ask for the partnership understanding related to the auction, not for a description of the actual hand. > > As an answer to that third question, "heart support" is certainly true, > but in answer to the first and second question "spades" is true. "Spades" is a false answer to the second question; he "shows" heart support. Sven From grandaeval at tiscali.co.uk Tue Nov 18 11:57:57 2008 From: grandaeval at tiscali.co.uk (Grattan) Date: Tue, 18 Nov 2008 10:57:57 -0000 Subject: [blml] Appeals procedures References: <009d01c948fe$6c45da20$0302a8c0@Mildred><4922051C.9060805@bridgepro.de><001401c94952$84db24b0$0302a8c0@Mildred> <002701c94962$0024bc80$15844c59@chello.pl> Message-ID: <004001c9496c$88eba510$0302a8c0@Mildred> Grattan Endicott To: "Bridge Laws Mailing List" Sent: Tuesday, November 18, 2008 9:42 AM Subject: [blml] Appeals procedures > Grattan, > > ----- Original Message ----- > From: "Grattan" > To: "Bridge Laws Mailing List" > Sent: Tuesday, November 18, 2008 8:50 AM > Subject: Re: [blml] Appeals procedures > > >> +=+ Timeo Danaos et dona ferentes. >> I do not suggest that the 3NT bid can be justified. Returned >> to due process I would expect the Director to disallow it and >> for a diamond contract, 4 or 5, to be substituted. I believe only an >> opening club lead beats 5D because of the location of the C9 and >> of the HK. To me that suggests a 12C1(c) adjustment. >> ~ Grattan ~ +=+ > > Yes, only an opening club lead (which is certainly the best one anyway) > definitively beats 5D but it nevertheless requires declarer to guess DQ > which is improbable. So even if we use 12C1(c) the weight of 5D= should be > very small. > +=+ I doubt you are right about the diamond finesse. It looks as though I will play for North to have three. Then you might look further. Even putting up the Ace on a low heart lead and subsequently misguessing on a second low heart, it seems that ruffing and running diamonds will squeeze the eleventh trick. Spades must be eliminated early of course. If a Spade is led, after drawing trumps a club to dummy reveals the condition of the suit; declarer can then exit with a Spade if they have not been cleared or with a club to his own hand. The Heart finesse then finds the eleventh trick. A heart opening lead is interesting. Suppose I take the Ace, clear trumps. lead a club to dummy. clear Spades and return a club to my own hand? Do I make? Richard Hills says: "if I was declarer I would find South, after taking the ace of clubs, leading another low heart. Undecided whether to put in the ten or the queen of hearts, I accidentally play the six of hearts. North's eight of hearts beats dummy's six of hearts, forcing me to ruff. Not to worry, I run all my diamonds to positionally squeeze South in clubs and hearts, making 5D the hard way." ~ Grattan ~ +=+ (And what is this about CA being the best lead? You take off control in an outside suit when you may need to establish a trick in Spades?) From nigelguthrie at talktalk.net Tue Nov 18 12:17:22 2008 From: nigelguthrie at talktalk.net (Nigel Guthrie) Date: Tue, 18 Nov 2008 11:17:22 +0000 Subject: [blml] A Bruce's comment [SEC=UNOFFICIAL] In-Reply-To: <49227C4E.3050007@skynet.be> References: <49227C4E.3050007@skynet.be> Message-ID: <4922A442.4000406@talktalk.net> [Herman De Wael] If you don't call that lying then you exhibit a level of hypocrisy which I associate with the Catholic Church, not with people thinking from reason. [Nigel] IMO, BLML is an inappropriate medium for religious bigotry (or political rants). From darkbystry at wp.pl Tue Nov 18 13:28:33 2008 From: darkbystry at wp.pl (Bystry) Date: Tue, 18 Nov 2008 13:28:33 +0100 Subject: [blml] Appeals procedures References: <009d01c948fe$6c45da20$0302a8c0@Mildred><4922051C.9060805@bridgepro.de><001401c94952$84db24b0$0302a8c0@Mildred><002701c94962$0024bc80$15844c59@chello.pl> <004001c9496c$88eba510$0302a8c0@Mildred> Message-ID: <000b01c94979$2cc135e0$15844c59@chello.pl> Grattan, ----- Original Message ----- From: "Grattan" To: "Bridge Laws Mailing List" Sent: Tuesday, November 18, 2008 11:57 AM Subject: Re: [blml] Appeals procedures > +=+ I doubt you are right about the diamond finesse. It looks as > though I will play for North to have three. Sorry Grattan, but maybe your view is flawed by the knowledge of the layout of the whole deal. N showed seven spades and S two. Should I continue? > Then you might look further. Even putting up the Ace on a > low heart lead and subsequently misguessing on a second low heart, > it seems that ruffing and running diamonds will squeeze the eleventh > trick. Spades must be eliminated early of course. > If a Spade is led, after drawing trumps a club to dummy reveals > the condition of the suit; declarer can then exit with a Spade if they > have not been cleared or with a club to his own hand. The Heart > finesse then finds the eleventh trick. > A heart opening lead is interesting. Suppose I take the Ace, > clear trumps. lead a club to dummy. clear Spades and return a club > to my own hand? Do I make? > Richard Hills says: "if I was declarer I would > find South, after taking the ace of clubs, leading another > low heart. Undecided whether to put in the ten or the > queen of hearts, I accidentally play the six of hearts. > North's eight of hearts beats dummy's six of hearts, forcing > me to ruff. Not to worry, I run all my diamonds to positionally > squeeze South in clubs and hearts, making 5D the hard way." Yes, after guessing the DQ there is no real chance to go down on this layout. And there is even 100% line assuming 7 spades at N (maybe there are such lines for 6-7 spades, I don't want to search for them). > ~ Grattan ~ +=+ > (And what is this about CA being the best lead? You take > off control in an outside suit when you may need to establish a > trick in Spades?) What real chances do you have to defeat the contract? You can count on CA as one trick. Partner's assumed spade trick makes it two. And what now? Hearts are probably under AQ. Diamond trick is improbable. Of course, there are layouts when the spade lead is needed but they require specific spade position, namely Ax xx (where x can be even a Q). But even in such positions it may be required to give partner his ruff instantly. CA may be wrong but for me it's better choice than to count on preempting partner having two defensive tricks. Finally, my ruling wouldn't depend on the choice of opening lead. I'd assume as an AC member that declarer would misguess diamonds nearly always so giving the benefit of doubt to the NOS (and taking into consideration the possibility of CA lead) it results in an assigned score of 5D-1. Regards Maciej From Hermandw at skynet.be Tue Nov 18 13:51:47 2008 From: Hermandw at skynet.be (Herman De Wael) Date: Tue, 18 Nov 2008 13:51:47 +0100 Subject: [blml] A Bruce's comment [SEC=UNOFFICIAL] In-Reply-To: <001501c9495e$75116ec0$15844c59@chello.pl> References: <49227C4E.3050007@skynet.be> <001501c9495e$75116ec0$15844c59@chello.pl> Message-ID: <4922BA63.30707@skynet.be> Bystry wrote: > Herman, > >>> There is no Law which requires me to lie to the opponents, >>> in order to save time, because I have a very well-founded >>> suspicion that the Leviathan Director would arbitrarily >>> assess my truth-telling as falsehood, due to the Director's >>> bias under his Hobbesian philosophy. >>> >> Yes there is. L20F5a. >> >> This law requires you to keep quiet over an untruth spoken by partner. >> >> If you don't call that lying then you exhibit a level of hypocrisy which >> I associate with the Catholic Church, not with people thinking from > reason. > > It would be better if you didn't offend the feelings of many Catholics. I'm > not a member of CC and could even agree with you but I don't consider public > forums unrelated to this topic to be proper places to state such opinions. > I was not talking about Catholics or believers of any faith, but about a church whc?ich throughout history has shown many signs of hypocrisy. Anyway, apologies to any who feel offended. > Second thing - there is a distinct difference between lying and keeping > quiet over an unintented untruth. If you don't understand it I'm not the one > who will try to explain this to you. > And if you don't see they are one and the same thing I'm not the one who will try to explain this to you. Just answer me this: Is the information you kept quiet about MI? Is it being punished? Could you have corrected it? If you believe this is acceptable, then you can just as well accept any other form of MI as acceptable. >> And no, this is not the end of the discussion. But I don't like seeing >> half-truths given away in defence of a set of actions which is, at best, >> acceptable because such a large majority believes they are acceptable. > > I don't pretend to speak for the majority. I comply with the CS because of > it's merits and because the Laws order me to do it, not because of the > effect of flock. > What merits? Have you ever been able to follow my full analysis of this problem? I have many arguments about merits. Many more than I have ever heard about MS. Don't talk to me about merits. >> I think your actions are comtemptible. > > Arguable statement. > >> You deliberately give UI, > > Yes. And what is wrong with giving additional UI? It can't hurt the NOS. At > least you don't deliberately give MI which is illegal and may be > misrepresented as cheating by your opponents. > NO - not additional - new one. DWS creates additional MI; MS creates new UI. Precisely the merits I'm talking about. DWS creates no new work for the TD - the MI was already there and can be said to have increased (I'm not even adding a qualifier "a little" here - not needed for my argumentation). MS creates an entire new problem for the TD. >> you give opponents deliberate misleading information, > > No. You tell them only that to what they are entitled - your partnership > agreements. And you definitely don't mislead them. In effect you inform them > that your side is having a mixup. And that is true and that can only help > them. > Of course you mislead them. If they don't understand that there was a Misunderstanding and so don't hear that they are deliberately being mislead, they have received a piece of information that they think describes the hand that bid it when it does nothing of the sort. And you knew that! >> and you create a game in which a totally different outcome >> is reached depending on a totally unimportant piece of action by >> opponents (an action, moreover, that we do not wish to encourage but >> that you give a huge advantage to). > > Herman, but Bridge works this way. Asking about alerted calls is necessary > to play this game. Imagine an auction: > > 1C - 1D - 1H(A) - ? > > If you don't ask about alert, bid 2S, get doubled and later you realize that > 1H was a transfer to spades whom do you want to blame? Questioning _is not_ > unimportant, it's the most frequently used way of getting information. And > you need this information to play real Bridge. > But that is not what I was talking about - questioning is important, yes, but you have created a situation where questioning makes one reach a different situation. Allow me three paragraphs to explain: If 4NT is explained as Blackwood and 5Di is bid next, the next player is normally interested in knowing how many aces there are in that hand. If he looks at the convention card or simply remembers the type of Blackwood these people are playing, play continues normally. But if he asks "how many aces?" he will get from a MS adept th reply "diamond preference". This leads to extra UI, and to interesting information for the question-asker. OTOH, from a DWS adept, he will receive the answer "1 ace", giving him precisely the information he wanted. Now many of you compare these situtions and think the DWS does something wrong, since he gets a better result than the MS player. But that is only the case if the question is asked. If no question is asked, the MS player will happily go on having his opponent believe 1 ace was shown. The DWS supporter gets the exact same result as the MS player without a question. It is the MS player with the question who is treated worse. Turn it around, and you will see that the question-asker has an advantage, when playing against MS supporters. This may lead to an explosion of questions, asked for no other reason that to discover misunderstandings. That is not how I want bridge to be played, and if you think this is a better situation then you are just trying to defend a non-defendable system. This is a real merit for the DWS, and a demerit for the MS. You may think this is unimportant, but you cannot insist that it is not true. What merits do you have to b?put in its place? >> But I am not entering this discussion. >> Herman. > > Regards > > Maciej > Herman. From gro at bridgepro.de Tue Nov 18 14:33:01 2008 From: gro at bridgepro.de (Gro) Date: Tue, 18 Nov 2008 14:33:01 +0100 Subject: [blml] Appeals procedures In-Reply-To: <002701c94962$0024bc80$15844c59@chello.pl> References: <009d01c948fe$6c45da20$0302a8c0@Mildred><4922051C.9060805@bridgepro.de> <001401c94952$84db24b0$0302a8c0@Mildred> <002701c94962$0024bc80$15844c59@chello.pl> Message-ID: <4922C40D.1050700@bridgepro.de> Hi, the question is which alternatives should be for that hand ( for those who dont know the hand: Qx, x, AJ109xx, Kxxx- 3S pass pass X pass ? ) 3NT could be a desaster- could be the right contract - you bid this, because of hesitation of partner? pass should lead to 500-800, sometimes only 200 - you bid this, because of hesitation of partner? 4D underbid, but partner is 4th hand X, so... - you bid this, because of hesitation of partner? 5D if partner has 2 small spades, you need all other high cards, so then you can also pass the X if partner has only singleton you have good chances - you bid this, because of hesitation of partner? when you in in 5D, you 75% probably more - dont make the contract which bid is the "normal" bid without any hesitation? unfortunatly no player could be asked there, cause no TD was called at the table. which of all this alternatives was given as a choice after the hesitation? difficult or? so a weighted score would be ok?...only problem: how to weight! but 5D minus 1 as the only final result seems not really justified...or? but my points have nothing to do with appeal procedures - but anyway wanted to say it Michael Bystry schrieb: > Grattan, > > ----- Original Message ----- > From: "Grattan" > To: "Bridge Laws Mailing List" > Sent: Tuesday, November 18, 2008 8:50 AM > Subject: Re: [blml] Appeals procedures > > > >> +=+ Timeo Danaos et dona ferentes. >> I do not suggest that the 3NT bid can be justified. Returned >> to due process I would expect the Director to disallow it and >> for a diamond contract, 4 or 5, to be substituted. I believe only an >> opening club lead beats 5D because of the location of the C9 and >> of the HK. To me that suggests a 12C1(c) adjustment. >> ~ Grattan ~ +=+ >> > > Yes, only an opening club lead (which is certainly the best one anyway) > definitively beats 5D but it nevertheless requires declarer to guess DQ > which is improbable. So even if we use 12C1(c) the weight of 5D= should be > very small. > > Regards > > Maciej > > > _______________________________________________ > blml mailing list > blml at amsterdamned.org > http://www.amsterdamned.org/mailman/listinfo/blml > > > From nigelguthrie at talktalk.net Tue Nov 18 14:41:50 2008 From: nigelguthrie at talktalk.net (Nigel Guthrie) Date: Tue, 18 Nov 2008 13:41:50 +0000 Subject: [blml] A player's comment In-Reply-To: References: <81C68581-7E16-4850-8930-B230C6D06D3A@starpower.net> Message-ID: <4922C61E.1000409@talktalk.net> [Robert Frick] The probably just asked what 2Sp meant. And you know the intended meaning (in this example). So it you want to be honest and responsive, just clarify for the opponents that you are not describing your partner's intended meaning, you are describing the sytemic meaning of partner's bid had partner understood the auction properly. Or you can follow the laws, your choice. I am not arguing that is lawful to be open and honest and responsive when answering opponent's questions. [Nigel] The truth in this case is the systemic meaning. Compliance with this rule isn't dishonest. But if it was, so what? IMO it's bizarre for Stefanie, Herman, and Robert to express revulsion at telling a lie to comply with a bridge rule. The rules of a game *redefine* local morals. Games suspend social mores in specific contexts. Have these BLMLers never played games like *Postman's knock* or *Hangman* or *Liar dice*? From ehaa at starpower.net Tue Nov 18 14:56:21 2008 From: ehaa at starpower.net (Eric Landau) Date: Tue, 18 Nov 2008 08:56:21 -0500 Subject: [blml] A player's comment In-Reply-To: References: Message-ID: On Nov 17, 2008, at 4:03 PM, richard.hills at immi.gov.au wrote: > Eric Landau asserted: > > [big snip] > >> Which puts us back where we started, with the question of >> what to do when you know that your version of your systemic >> understanding differs from your partner's but do not know >> which of you is right. >> >> That is a question we have yet to answer. > > Law 20F5(b): > > "The player must call the Director and inform his opponents > that, **in his opinion**, his partner's explanation was > erroneous (see Law 75) but only at his first legal > opportunity..." > > What's the problem? The problem is that this begs the question. Although the psychologists tell us that "In my opinion X is true" and "I know that X is true" are very different statements in the mind of the speaker, the epistomologists tell us that there is no difference between them as far as anyone else is concerned. Eric Landau 1107 Dale Drive Silver Spring MD 20910 ehaa at starpower.net From karel at esatclear.ie Tue Nov 18 12:18:21 2008 From: karel at esatclear.ie (Karel) Date: Tue, 18 Nov 2008 11:18:21 +0000 Subject: [blml] How should this have been handled ? Message-ID: Hi Havent posted in a fair while .... must be the new laws are working !! Played in a tourney on BBO last night and came up against a somewhat unusal situation. The 3 hands are attached. Hand one it goes 2H Pass (by me) 4D P P double all pass Not sure what the standard meaning for 4D is, probably a splinter. This sailed of for a big number. At this point the 2H opener called the TD and said that 4D was possibly the most ridiculous bid he had ever seen (there was also an implication that this wasnt the first poor bid he had witnessed). In any event this one must have broke the camels back because ..... Hand two it goes 1D(me) 2C dbl 6NT dbl rdbl all pass Now if re wanted his pd to suffer he should have bid 7C but in any event this was his effort at trying to get even with his pd for ridiculous bridge. The rdbl was clearly showing pd he wasnt going to be out done. Another telephone number. Hand 3 it goes P P 1D dbl 3D 3H P P 4D P P dbl rdbl all pass Now this hand is note worthy for 2 points. (a) His old partner had left, excused himself with the TD on the grounds of sanity (b) 4D-1 was the contract at 2 tables -2 for a zero. The td now adjusts boards 1 and 3 to average MINUS for partner and I !!?. Board 2 gets adjusted to average plus. First off while I realise results 1 + 2 are totally skewed, we surely should get at least average plus for them all. Result 3 though I dont think is unreasonable and should be left alone. There were 2 4D-1 which means this is not completely of the wall unlike the other 2 and redouble is not completely daft. So what should have happened ? Karel ps. I was in a local club once where on the last hand of the night it went 1NT P 7NT. Dummy said thank you very much for the game pd (acid and sarcasm oozing) and left. -------------- next part -------------- A non-text attachment was scrubbed... Name: blm hand 1.png Type: image/png Size: 20853 bytes Desc: not available Url : http://www.amsterdamned.org/pipermail/blml/attachments/20081118/669021ac/attachment-0003.png -------------- next part -------------- A non-text attachment was scrubbed... Name: blm hand 2.png Type: image/png Size: 22893 bytes Desc: not available Url : http://www.amsterdamned.org/pipermail/blml/attachments/20081118/669021ac/attachment-0004.png -------------- next part -------------- A non-text attachment was scrubbed... Name: blm hand 3.png Type: image/png Size: 21941 bytes Desc: not available Url : http://www.amsterdamned.org/pipermail/blml/attachments/20081118/669021ac/attachment-0005.png From nigelguthrie at talktalk.net Tue Nov 18 15:29:52 2008 From: nigelguthrie at talktalk.net (Nigel Guthrie) Date: Tue, 18 Nov 2008 14:29:52 +0000 Subject: [blml] A player's comment In-Reply-To: References: <81C68581-7E16-4850-8930-B230C6D06D3A@starpower.net> <000a01c948d4$45372bc0$cfa58340$@no> <000001c94902$a2f02860$e8d07920$@no> Message-ID: <4922D160.2030401@talktalk.net> [Robert Frick] As in the new laws, there is no hint that a mistaken bid/call is not a misleading explanation. [Nigel] It isn't necessary for the law-book to distinguish *bid* from *explanation* explicitly :) From darkbystry at wp.pl Tue Nov 18 15:33:28 2008 From: darkbystry at wp.pl (Bystry) Date: Tue, 18 Nov 2008 15:33:28 +0100 Subject: [blml] A Bruce's comment [SEC=UNOFFICIAL] References: <49227C4E.3050007@skynet.be><001501c9495e$75116ec0$15844c59@chello.pl> <4922BA63.30707@skynet.be> Message-ID: <000701c9498a$9f949880$15844c59@chello.pl> Herman, [snip] >> Second thing - there is a distinct difference between lying and keeping >> quiet over an unintented untruth. If you don't understand it I'm not the one >> who will try to explain this to you. > And if you don't see they are one and the same thing I'm not the one who > will try to explain this to you. So let us remain at our positions. > Just answer me this: Is the information you kept quiet about MI? Is it > being punished? Could you have corrected it? If you believe this is > acceptable, then you can just as well accept any other form of MI as > acceptable. I'm not sure I understand correctly. You'd want the Laws to allow for instant during-the-auction MI corrections? I fully support this. It would render any MI moot (from this point) and the OS would only have to deal with UI. But the lawmakers stated otherwise and I have no choice but to obey the Laws. >> I don't pretend to speak for the majority. I comply with the CS because of >> it's merits and because the Laws order me to do it, not because of the >> effect of flock. > What merits? a) nobody has to lie b) the NOS know that the OS has a bidding mixup and can use it to their advantage c) much frequently the TD won't be even bothered to come to the table (the OS result will be sufficiently bad so the NOS won't need any improvements) d) the player is more often sure what is his understanding of the partnership agreements but he cannot be sure what is in his partner's mind (not all cases are so simple as your 4NT example) > Have you ever been able to follow my full analysis of this problem? Actuallly, yes. I read many BLML discussions in the archives about this topic. > I have many arguments about merits. Yes, you have. They are quite logical. No, I don't agree with them. Arguments supporting CS are more logical. > Many more than I have ever heard about MS. Don't talk to me about merits. In my humble opinion it is very hard for someone, who is so much devoted to one idea, than to listen to and appreciate cogent arguments of his opponents. You see - I've made my mind on this subject only _after_ reading of your numerous discussions on BLML. I didn't have any such problems in practice (I play quite simple and easy-to-remember system). So you may agree with me or not but please accept that I've decided to follow CS on it's merits, not because of lack of information about dWS. >> Yes. And what is wrong with giving additional UI? It can't hurt the NOS. At >> least you don't deliberately give MI which is illegal and may be >> misrepresented as cheating by your opponents. > NO - not additional - new one. Ok, in a sense. I stand corrected. > DWS creates additional MI; MS creates new UI. You see. Creating UI doesn't hurt the NOS. Only using it can hurt them. So your side should not use it, otherwise you will get your deserved poor score from the TD. Period. MI is harder to manage. The NOS has to prove it hurted them. Sometimes their subtle decisions will miss a point, but the TD won't give them redress. And there are those nebulous "equity" laws. No, thanks. The Laws shouldn't be used as a rescue for offenders, denying the NOS their due good scores because of your mixups. > Precisely the merits I'm talking about. > DWS creates no new work for the TD - the MI was already there and can be > said to have increased (I'm not even adding a qualifier "a little" here > - not needed for my argumentation). MS creates an entire new problem for > the TD. You seem to forget that UI is already in too. Just for you, not for your partner. So _if_ the ruling is needed the TD would have to cope with one "portion" of UI and two "portions" of MI in dWS and in CS with "two portions of UI" and one "portion" of MI (or none because your misunderstanding will be clear for the NOS and it's very improbable it'll hurt them). And I consider UI much less harmful because it may be unused and is easy to adjust with the profit for the NOS. >> No. You tell them only that to what they are entitled - your partnership >> agreements. And you definitely don't mislead them. In effect you inform them >> that your side is having a mixup. And that is true and that can only help >> them. > Of course you mislead them. If they don't understand that there was a > Misunderstanding and so don't hear that they are deliberately being > mislead, they have received a piece of information that they think > describes the hand that bid it when it does nothing of the sort. And you > knew that! Sorry, but they are _not_ entitled to know what is in your or your partner's hand. Herman, you're a TD, you know the Laws. I'll tell you more - I wouldn't even want to know what is in their hands. Bridge would cease to have any sense for me. I could just as well solve 4-hand quizes. If the followers of your school don't understand that they should not be told what is in their opponents' hands they have to be instantly reeducated as to the correct reading of TFLB and more - they should be instantly told what is the game of Bridge about. > But that is not what I was talking about - questioning is important, > yes, but you have created a situation where questioning makes one reach > a different situation. And so what? If you're so troubled with this than think about other, better means of disclosure but don't deny to anyone what is for him lawfully available. Questioning often leads to reaching different situations - for your side and for the opponents. It can create UI regardless which school you follow. > Allow me three paragraphs to explain: > 4NT is explained as Blackwood and 5Di is bid next, the next player is > normally interested in knowing how many aces there are in that hand. If > he looks at the convention card or simply remembers the type of > Blackwood these people are playing, play continues normally. But if he > asks "how many aces?" he will get from a MS adept th reply "diamond > preference". This leads to extra UI, and to interesting information for > the question-asker. I've read this argument 100 times. And I don't agree with it. Firstly - the player is _not_ interested how many aces your partner has. He is interested in knowing what does 5D mean in your system. That's what the Laws say he is entitled to. His other "wantings" are irrelevant. He can play pseudobridge at home with all the hands exposed before the bidding. Secondly - what is better for him? To know that you have a bidding mixup or to know how many aces does your partner have? Thirdly - there is a golden rule - when alerted ask, do not assume. So he asked and got rewarded. He showed his interest in the auction what I see as an important part of Bridge. And don't impute any bad motives on his side. He couldn't have foreseen the result. It was your partnership who caused this mess, why blame him? > OTOH, from a DWS adept, he will receive the answer "1 ace", giving him > precisely the information he wanted. > Now many of you compare these situtions and think the DWS does something > wrong, since he gets a better result than the MS player. But that is > only the case if the question is asked. If no question is asked, the MS > player will happily go on having his opponent believe 1 ace was shown. > The DWS supporter gets the exact same result as the MS player without a > question. It is the MS player with the question who is treated worse. And rightfully so. You know, I can understand being sorry for the poor player whose opponents guess 5 two-way finesses during one session. He is just plainly unlucky. I can clap him on his back, go for a beer with him. But the player who breaks the Laws cannot expect even a little dose of compassion from me. > Turn it around, and you will see that the question-asker has an > advantage, when playing against MS supporters. This may lead to an > explosion of questions, asked for no other reason that to discover > misunderstandings. That is not how I want bridge to be played, and if > you think this is a better situation then you are just trying to defend > a non-defendable system. Sorry Herman, but Bridge _is_ played like that. And where do you see such an explosion of questions? I see none. Most people are not devious. Besides don't forget that questions generate UI and your argument is statistically at least feeble. Do you really think that such method would result in more profits from nailing the opponents than losses because of adverse UI rulings? Let's get real. > This is a real merit for the DWS, and a demerit for the MS. You may > think this is unimportant, but you cannot insist that it is not true. Read thoroughly what I have written above and think about it once more. > What merits do you have to be put in its place? To that question I have already responded. > Herman. Regards Maciej From nigelguthrie at talktalk.net Tue Nov 18 15:50:33 2008 From: nigelguthrie at talktalk.net (Nigel Guthrie) Date: Tue, 18 Nov 2008 14:50:33 +0000 Subject: [blml] A player's comment In-Reply-To: References: <81C68581-7E16-4850-8930-B230C6D06D3A@starpower.net> Message-ID: <4922D639.7000103@talktalk.net> [Eric Landau] Which puts us back where we started, with the question of what to do when you know that your version of your systemic understanding differs from your partner's but do not know which of you is right. That is a question we have yet to answer. [Nigel] Another illustration that it's virtually impossible to be certain of a partnership understanding. You are rarely sure but, IMO, the law should mandate that you make your best guess, rather than join in with the popular "No understanding" mantra. From darkbystry at wp.pl Tue Nov 18 15:53:29 2008 From: darkbystry at wp.pl (Bystry) Date: Tue, 18 Nov 2008 15:53:29 +0100 Subject: [blml] Appeals procedures References: <009d01c948fe$6c45da20$0302a8c0@Mildred><4922051C.9060805@bridgepro.de> <001401c94952$84db24b0$0302a8c0@Mildred><002701c94962$0024bc80$15844c59@chello.pl> <4922C40D.1050700@bridgepro.de> Message-ID: <001a01c9498d$6b34ab40$15844c59@chello.pl> Hi, > the question is which alternatives should be for that hand > ( for those who dont know the hand: Qx, x, AJ109xx, Kxxx- 3S pass pass X > pass ? ) > > 3NT could be a desaster- could be the right contract - you bid this, > because of hesitation of partner? What is a standart balancing double? 1-4-4-4, 1-(5-4-3), less frequently 2-(4-4-3). With any of this distributions 3NT is most probably a disaster. Now the hesitation suggest that your partner doesn't have a typical hand. So the chances for 3NT being the right contract rise. That alone isn't enough, but you've got an idea? > pass should lead to 500-800, sometimes only 200 - you bid this, > because of hesitation of partner? Yes, you can apply nearly the same conclusion as above. > 4D underbid, but partner is 4th hand X, so... - you bid this, > because of hesitation of partner? No. 4D is quite a normal bid. Maybe it is in some way suggested over 5D (because if your partner's hand is not typical it builds up chances for D doubleton). > 5D if partner has 2 small spades, you need all other high cards, so > then you can also pass the X > if partner has only singleton you have good chances - you bid > this, because of hesitation of partner? No. Normal balancing double (something like 1-4-4-4) should give you at least fair chances of making 5D. But with a hesitation partner may have D doubleton, so 5D is not suggested, it's even anti-suggested. > which bid is the "normal" bid without any hesitation? unfortunatly no > player could be asked there, cause no TD was called at the table. Yes, that is bad. But an AC consists of players and their judgment applied. > which of all this alternatives was given as a choice after the hesitation? > difficult or? so a weighted score would be ok?...only problem: how to > weight! I might be convinced to rule something weighted between 4D= and 5D-1. It is not an easy judgment. > but 5D minus 1 as the only final result seems not really justified...or? > but my points have nothing to do with appeal procedures - but anyway > wanted to say it > > Michael Regards Maciej From Hermandw at skynet.be Tue Nov 18 17:35:30 2008 From: Hermandw at skynet.be (Herman De Wael) Date: Tue, 18 Nov 2008 17:35:30 +0100 Subject: [blml] A Bruce's comment [SEC=UNOFFICIAL] In-Reply-To: <000701c9498a$9f949880$15844c59@chello.pl> References: <49227C4E.3050007@skynet.be><001501c9495e$75116ec0$15844c59@chello.pl> <4922BA63.30707@skynet.be> <000701c9498a$9f949880$15844c59@chello.pl> Message-ID: <4922EED2.6080806@skynet.be> Bystry wrote: > Herman, > > > I'm not sure I understand correctly. You'd want the Laws to allow for > instant during-the-auction MI corrections? I fully support this. It would > render any MI moot (from this point) and the OS would only have to deal with > UI. But the lawmakers stated otherwise and I have no choice but to obey the > Laws. > Indeed the lawmakers stated otherwise. For many good reasons, which I will go into detail on if needed. But not now. The point is, the lawmakers have decided that it is better for the game of bridge for MI to continue to exist, rather than for UI to be created. And we all accept that. Now we are one second later and there is a situation where again a choice has to be made between MI (small increment) and UI (same huge amount), Why is it so hard to accept that the choice should be the same one? That MI is to be preferred over UI? And I would not mind so much if this change happens all the time; but it only happens at those tables where a follow-up question is asked. That is what I am most concerned about. Herman. From Hermandw at skynet.be Tue Nov 18 18:05:16 2008 From: Hermandw at skynet.be (Herman De Wael) Date: Tue, 18 Nov 2008 18:05:16 +0100 Subject: [blml] A Bruce's comment [SEC=UNOFFICIAL] In-Reply-To: <000701c9498a$9f949880$15844c59@chello.pl> References: <49227C4E.3050007@skynet.be><001501c9495e$75116ec0$15844c59@chello.pl> <4922BA63.30707@skynet.be> <000701c9498a$9f949880$15844c59@chello.pl> Message-ID: <4922F5CC.1000702@skynet.be> Bystry wrote: > > >> What merits? > > a) nobody has to lie That is not a merit - the statement uttered is not a lie under one definition - it is precisely what bidder intended. > b) the NOS know that the OS has a bidding mixup and can use it to their > advantage And why is this a merit? The NOS gets something extra, the OS gets something worse, not a merit in my book. Besides, the OS are getting something that in other places is denied them - they do not have any entitlement to this information. And again, this information is only received after asking a follow-up question. This may well lead to the asking of many more questions, not really a merit in my opinion. > c) much frequently the TD won't be even bothered to come to the table (the > OS result will be sufficiently bad so the NOS won't need any improvements) Only if the OS are masochistically staying in the worst contract possible. The combination of UI in two directions may well lead to even worse possible contracts. Which the TD has to sort out. But just compare the two cases, if you think the TD has less work. Under DWS, the TD has one important piece of MI to deal with, one less important one (since it describes the hand there will usually be no damage), and one piece of UI. Under MS, the TD still has the important piece of UI (the NOS will always claim their one bid in the meantime was influenced by MI), and two pieces of UI. The first MI and the first UI are unavoidable. The second MI and second UI are avoidable. Guess which one I prefer? You may prefer the other one, but it's not a clear merit for MS. > d) the player is more often sure what is his understanding of the > partnership agreements but he cannot be sure what is in his partner's mind > (not all cases are so simple as your 4NT example) > Granted, but if we can settle the easy case first, will we not have a better understanding of the more difficult one? >> Have you ever been able to follow my full analysis of this problem? > > Actuallly, yes. I read many BLML discussions in the archives about this > topic. > I have not ever been allowed to work this out till the end. >> I have many arguments about merits. > > Yes, you have. They are quite logical. No, I don't agree with them. > Arguments supporting CS are more logical. > Such as the reason why you accept two widely different results at two tables, just because a second question is asked at one of them? Can you give any logical argument why this is good for bridge? >> Many more than I have ever heard about MS. Don't talk to me about merits. > > In my humble opinion it is very hard for someone, who is so much devoted to > one idea, than to listen to and appreciate cogent arguments of his > opponents. In my equally humble opinion this argument works both ways. You are equally devoted to your idea (and you find consolation in being in a majority) that you too fail to listen and appreciate to cogent arguments. You have given four arguments above. I have patiently read them and given answers. Why does that not help you change your opinion? You ask me to change mine, but you have no intention of changing yours. Menawhile, I have given cogent arguments of my own, and I don't get reasoned replies. Is it any wonder that I believe this situation is not strictly symmetric? > You see - I've made my mind on this subject only _after_ reading > of your numerous discussions on BLML. I didn't have any such problems in > practice (I play quite simple and easy-to-remember system). So you may agree > with me or not but please accept that I've decided to follow CS on it's > merits, not because of lack of information about dWS. > >>> Yes. And what is wrong with giving additional UI? It can't hurt the NOS. > At >>> least you don't deliberately give MI which is illegal and may be >>> misrepresented as cheating by your opponents. > >> NO - not additional - new one. > > Ok, in a sense. I stand corrected. > >> DWS creates additional MI; MS creates new UI. > > You see. Creating UI doesn't hurt the NOS. Only using it can hurt them. So > your side should not use it, otherwise you will get your deserved poor score > from the TD. Period. > And here you are simply wrong. Creating UI DOES hurt the OS (I assume the N above was a typo). Partner is not allowed to choose the "correct" action. And so I want to avoid giving UI. But here you are, advocating a law change in which you OBLIGE me to give UI! Have you ever thought about what the penalty ought to be for refusing to give obligatory UI? Do you really think this is not a merit for the DWS? > MI is harder to manage. The NOS has to prove it hurted them. Sometimes their > subtle decisions will miss a point, but the TD won't give them redress. And > there are those nebulous "equity" laws. No, thanks. The Laws shouldn't be > used as a rescue for offenders, denying the NOS their due good scores > because of your mixups. > And this is exactly where you have the wrong comparison. DWS does not end up with a better score than MS, as you seem to think. If no follow-up question is asked, DWS gets you exactly the same result as MS. it is only after a follow-up question, that MS lands you in worse shit than DWS. I fail to see why I should suffer such a fate. My neighbour who has done exactly the same thing wrong (forgetting his system) will get exactly the same bad score as I do - if his opponent does not ask the question. Why should I suffer because my opponent is more lazy than his? >> Precisely the merits I'm talking about. >> DWS creates no new work for the TD - the MI was already there and can be >> said to have increased (I'm not even adding a qualifier "a little" here >> - not needed for my argumentation). MS creates an entire new problem for >> the TD. > > You seem to forget that UI is already in too. Just for you, not for your > partner. So _if_ the ruling is needed the TD would have to cope with one > "portion" of UI and two "portions" of MI in dWS and in CS with "two portions > of UI" and one "portion" of MI (or none because your misunderstanding will > be clear for the NOS and it's very improbable it'll hurt them). And I > consider UI much less harmful because it may be unused and is easy to adjust > with the profit for the NOS. > Indeed, and the first piece of UI is unavoidable, but the second one isn't. I want to be allowed to avoid giving that UI. >>> No. You tell them only that to what they are entitled - your partnership >>> agreements. And you definitely don't mislead them. In effect you inform > them >>> that your side is having a mixup. And that is true and that can only help >>> them. > >> Of course you mislead them. If they don't understand that there was a >> Misunderstanding and so don't hear that they are deliberately being >> mislead, they have received a piece of information that they think >> describes the hand that bid it when it does nothing of the sort. And you >> knew that! > > Sorry, but they are _not_ entitled to know what is in your or your partner's > hand. Herman, you're a TD, you know the Laws. Indeed. They are not entitled that. But it is what they want to know. So if you tell them something else, you are misleading them. Simple language, not laws at all. > I'll tell you more - I > wouldn't even want to know what is in their hands. ????????? > Bridge would cease to > have any sense for me. I could just as well solve 4-hand quizes. If the > followers of your school don't understand that they should not be told what > is in their opponents' hands they have to be instantly reeducated as to the > correct reading of TFLB and more - they should be instantly told what is the > game of Bridge about. > >> But that is not what I was talking about - questioning is important, >> yes, but you have created a situation where questioning makes one reach >> a different situation. > > And so what? If you're so troubled with this than think about other, better > means of disclosure but don't deny to anyone what is for him lawfully > available. Questioning often leads to reaching different situations - for > your side and for the opponents. It can create UI regardless which school > you follow. > No, it doesn't create UI in DWS. >> Allow me three paragraphs to explain: >> 4NT is explained as Blackwood and 5Di is bid next, the next player is >> normally interested in knowing how many aces there are in that hand. If >> he looks at the convention card or simply remembers the type of >> Blackwood these people are playing, play continues normally. But if he >> asks "how many aces?" he will get from a MS adept th reply "diamond >> preference". This leads to extra UI, and to interesting information for >> the question-asker. > > I've read this argument 100 times. And I don't agree with it. Aha. I'll listen. > Firstly - the > player is _not_ interested how many aces your partner has. Of course he is! > He is interested > in knowing what does 5D mean in your system. That's what the Laws say he is > entitled to. His other "wantings" are irrelevant. He wants to know how to defend. Knowing that 5Di showed diamonds is useless to him. Knowing that it showed 1 ace is useful. If you don't agree with that you are deliberately being obtuse. > He can play pseudobridge > at home with all the hands exposed before the bidding. Secondly - what is > better for him? To know that you have a bidding mixup or to know how many > aces does your partner have? Of course it is better for him to know we have a bidding misunderstanding. But he is not entitled to that knowledge. You cannot have it both ways - first saying he is not interested in knowing there is one ace, and then that he is is interested in knowing about a misunderstanding. > Thirdly - there is a golden rule - when alerted > ask, do not assume. Who said anything about assuming? I'm talking "knowing", like in remembering what the answers system was previously, or having studied opponents system, or reading the CC. All of these are to be preferred (in a general bridge sense) to asking, and yet you award an advantage to "asking". And this is far worse than you seem to imagine. Turn the example around with 4NT being described as "minors". Who in heaven's name would dream of asking what 5Di means in this sequence? Yet if you do ask, you are rewarded with a windfall when the response by a MS adept is "1 ace". > So he asked and got rewarded. He showed his interest in > the auction what I see as an important part of Bridge. And don't impute any > bad motives on his side. He couldn't have foreseen the result. It was your > partnership who caused this mess, why blame him? > At this time in history, he cannot foresee the result. But when the laws are rewritten with the change in L20F5a clearly in view, he can foresee a result. Expect 100% of questions from some opponents. See if you like that? >> OTOH, from a DWS adept, he will receive the answer "1 ace", giving him >> precisely the information he wanted. >> Now many of you compare these situtions and think the DWS does something >> wrong, since he gets a better result than the MS player. But that is >> only the case if the question is asked. If no question is asked, the MS >> player will happily go on having his opponent believe 1 ace was shown. >> The DWS supporter gets the exact same result as the MS player without a >> question. It is the MS player with the question who is treated worse. > > And rightfully so. You know, I can understand being sorry for the poor > player whose opponents guess 5 two-way finesses during one session. He is > just plainly unlucky. I can clap him on his back, go for a beer with him. > But the player who breaks the Laws cannot expect even a little dose of > compassion from me. > We are not talking of breaking laws here - we are talking about what the law should be. And sorry, but I have a Gandhi-like approach to unjust laws - I won't follow them. >> Turn it around, and you will see that the question-asker has an >> advantage, when playing against MS supporters. This may lead to an >> explosion of questions, asked for no other reason that to discover >> misunderstandings. That is not how I want bridge to be played, and if >> you think this is a better situation then you are just trying to defend >> a non-defendable system. > > Sorry Herman, but Bridge _is_ played like that. And where do you see such an > explosion of questions? I see none. Not yet, there are none. > Most people are not devious. This is not being devious - it is a legitimate way of enhancing my scores. Rather than look something up, I shall always ask a question. > Besides > don't forget that questions generate UI and your argument is statistically > at least feeble. Oh no they don't, because I will ask about the previous bid at every single turn. Just a "what is 5Di?" will do. And only about one call of course - my partner will have asked about the other one. > Do you really think that such method would result in more > profits from nailing the opponents than losses because of adverse UI > rulings? Let's get real. > Since there is no UI in asking all the time - I am real. Anyway, there is just as much UI in looking at a CC. >> This is a real merit for the DWS, and a demerit for the MS. You may >> think this is unimportant, but you cannot insist that it is not true. > > Read thoroughly what I have written above and think about it once more. > I have, and patiently answered all your arguments. Please think about it once again. >> What merits do you have to be put in its place? > > To that question I have already responded. > To be fair, yes you have. Your arguments are sadly unconvincing (to me). >> Herman. > > Regards > > Maciej > Herman. From wjburrows at gmail.com Tue Nov 18 19:54:25 2008 From: wjburrows at gmail.com (Wayne Burrows) Date: Wed, 19 Nov 2008 07:54:25 +1300 Subject: [blml] Appeals procedures In-Reply-To: <001a01c9498d$6b34ab40$15844c59@chello.pl> References: <009d01c948fe$6c45da20$0302a8c0@Mildred> <4922051C.9060805@bridgepro.de> <001401c94952$84db24b0$0302a8c0@Mildred> <002701c94962$0024bc80$15844c59@chello.pl> <4922C40D.1050700@bridgepro.de> <001a01c9498d$6b34ab40$15844c59@chello.pl> Message-ID: <2a1c3a560811181054u665d9e7ej9d6816fca8dc49b0@mail.gmail.com> 2008/11/19 Bystry : > Hi, > >> the question is which alternatives should be for that hand >> ( for those who dont know the hand: Qx, x, AJ109xx, Kxxx- 3S pass pass X >> pass ? ) >> >> 3NT could be a desaster- could be the right contract - you bid this, >> because of hesitation of partner? > > What is a standart balancing double? 1-4-4-4, 1-(5-4-3), less frequently > 2-(4-4-3). With any of this distributions 3NT is most probably a disaster. > Now the hesitation suggest that your partner doesn't have a typical hand. So > the chances for 3NT being the right contract rise. That alone isn't enough, > but you've got an idea? > >> pass should lead to 500-800, sometimes only 200 - you bid this, >> because of hesitation of partner? > > Yes, you can apply nearly the same conclusion as above. > >> 4D underbid, but partner is 4th hand X, so... - you bid this, >> because of hesitation of partner? > > No. 4D is quite a normal bid. Maybe it is in some way suggested over 5D > (because if your partner's hand is not typical it builds up chances for D > doubleton). > >> 5D if partner has 2 small spades, you need all other high cards, so >> then you can also pass the X >> if partner has only singleton you have good chances - you bid >> this, because of hesitation of partner? > > No. Normal balancing double (something like 1-4-4-4) should give you at > least fair chances of making 5D. But with a hesitation partner may have D > doubleton, so 5D is not suggested, it's even anti-suggested. > >> which bid is the "normal" bid without any hesitation? unfortunatly no >> player could be asked there, cause no TD was called at the table. > > Yes, that is bad. But an AC consists of players and their judgment applied. > >> which of all this alternatives was given as a choice after the hesitation? >> difficult or? so a weighted score would be ok?...only problem: how to >> weight! > > I might be convinced to rule something weighted between 4D= and 5D-1. It is > not an easy judgment. > >> but 5D minus 1 as the only final result seems not really justified...or? >> but my points have nothing to do with appeal procedures - but anyway >> wanted to say it >> If you are arguing that 4D is not an allowed bid then the only adjustment you can make is between 5D -1 and 5D=. I believe your reasoning is flawed in that 1=4=4=4 is not a normal takeout double. It is a perfect one. In responding to a takeout double one doesn't normally assume partner has the perfect hand. On this occasion there is addition information that suggests a stiff spade is unlikely - Partner of the 3S bidder didn't raise which would often occur with three or four spades. Disallowing 3NT specifically requires that you can reason that the slow double demonstably suggests that partner has a spade honour rather than no spade honour. To me that is a wild conclusion. Even knowing that partner has two spades then the odds are (2 x 9 + 1) / 11C2 = 19/55 = 0.34 which is well against the odds. Given that there are many reasons for a slow double, a slow double hardly suggests that he has a spade honour. Has it yet been determined that the slowness of the double transmitted through the screen. This needs more than an admission that the double was slow. -- Wayne Burrows Palmerston North New Zealand From mfrench1 at san.rr.com Tue Nov 18 21:29:18 2008 From: mfrench1 at san.rr.com (Marvin L French) Date: Tue, 18 Nov 2008 12:29:18 -0800 Subject: [blml] L12C1(b) Message-ID: <9B89FCA6048C4FA489EEDC499121DB34@MARVLAPTOP> WBFLC minutes, Lille 1998: ...advantage gained by an offender, provided it is related to the infraction,...shall be construed as an advantage in the table score whether subsequent or consequent to the infraction. Damage to a non-offending side shall be a consequence of the infraction if redress is to be given in an adjusted score. ...the right to redress for a non-offending side is not annulled by a normal error or misjudgment in the subsequent action, but only by an action that is evidently irrational, wild or gambling. L12C1(b), first paragraph, substitutes "serious error" for "irrational" error. That was welcome, because Grattan didn't think a revoke is irrational. After such error, "the non-offending side...does not receive relief for such part of the damage as is self-inflicted." Evidently that means a serious error that lets a contract make is included for them when the score is adjusted. I maintain that a "serious error" (like a revoke) that lets a contract make means that *all* damage is self-inflicted. When a revoke leads merely to an overtrick, only the revoke is self-inflicted damage and the non-offenders are entitled to a score adjustment (keeping the revoke) L12C1(b), second paragraph, says: The offending side should be awarded the score that it would have been allotted as the consequence of its infraction only. I am not sure what this high-falutin language means. Let me guess. A revoke by the non-offenders is not included in the offenders' score adjustment. If that applies to all "serious errors," as it must, adjusting could get very complicated, especially since "serious" now depends on the calibre of player (WBFLC, 10 Oct). Did the non-offenders do something really stupid in the play? Yes? Then don't include that effect in the score adjustment for the offenders. Put another way, adjusted scores reflect play by the non-offenders that includes no serious error (not just a revoke) that they may have committed. That doesn't make sense. What makes sense is that the play of the cards, assuming the level of contract is irrelevant, should not be changed, and that includes serious errors like revoking. Offenders should only have to give up a gain related to the offense and nothing more. If this be error, and upon me proved, I never writ, nor no man ever played. Marv Marvin L French San Diego, CA www.marvinfrench.com . From rfrick at rfrick.info Tue Nov 18 22:02:24 2008 From: rfrick at rfrick.info (Robert Frick) Date: Tue, 18 Nov 2008 16:02:24 -0500 Subject: [blml] A player's comment In-Reply-To: <001201c94968$f6785f00$e3691d00$@no> References: <81C68581-7E16-4850-8930-B230C6D06D3A@starpower.net> <000a01c948d4$45372bc0$cfa58340$@no> <000001c94902$a2f02860$e8d07920$@no> <001201c94968$f6785f00$e3691d00$@no> Message-ID: On Tue, 18 Nov 2008 05:32:31 -0500, Sven Pran wrote: > On Behalf Of Robert Frick > ............. >> As in the new laws, there is no hint that a mistaken bid/call is not a >> misleading explanation. There is no hint that the laws inexplicably >> change >> terminology and use mistaken explanation as synonymous with misleading >> explanation. There is no use of any other terminology to describe the >> discrepancy between an explanation and a bid's actual intended meaning, >> which is the commonality between the two examples. > .............. >> I don't think you understand the issue as I have described it. Or you >> just >> deny it exists, I can't tell. > > Quite possibly I don't understand you, but please just consider: > > A call is an action in which you make a pass, a Double, a redouble or a > bid. Right > > An explanation is a description of the information available from this > call. Right > > The laws require all explanations to reflect partnership understandings > and > nothing else. Right > > An explanation is mistaken (or misleading) if it does not convey all the > information related to this call according to the partnership > understandings. A call not in accordance with the partnership > understanding > does not make the corresponding correct explanation misleading even > though > this explanation no longer describes the actual hand. You are equating misleading explanations with mistaken explanations. I don't see the point of doing that in this discussion. I am claiming that Law 75 says the opposite -- that a misleading explanation occurs for mistaken explanation and mistaken call. > > A call can also be misleading; the best example is a psychic call. But > calls > are never "explanations" of anything? Right > > So "misleading call" cannot be synonymous with "misleading explanation". Right, but irrelevant. The two terms you are claiming have the same meaning are misleading explanation and mistaken explanation. From harsanyi at t-online.de Tue Nov 18 23:06:30 2008 From: harsanyi at t-online.de (Josef Harsanyi) Date: Tue, 18 Nov 2008 23:06:30 +0100 Subject: [blml] Appeals procedures In-Reply-To: <4922C40D.1050700@bridgepro.de> References: <009d01c948fe$6c45da20$0302a8c0@Mildred><4922051C.9060805@bridgepro.de> <001401c94952$84db24b0$0302a8c0@Mildred> <002701c94962$0024bc80$15844c59@chello.pl> <4922C40D.1050700@bridgepro.de> Message-ID: <005001c949c9$e99aa4d0$bccfee70$@de> Hi, has somebody any idea about the defense system of E/W? Do they play leaping Michaels or any other convention after 3s from opponent? If a 4c and 4d bid by west shows 5-5 in the bid minor and hearth, than the set of hands for a double will be much greater, than 1444. Consequently the set of LA-s can vary with different biding methods. Also if a world class pair is at the table, their methods should be also understand by the TD. This is possible only by asking them and other players, who know them. But nobody asked E/W. Best regards Josef -----Urspr?ngliche Nachricht----- Von: blml-bounces at amsterdamned.org [mailto:blml-bounces at amsterdamned.org] Im Auftrag von Gro Gesendet: Dienstag, 18. November 2008 14:33 An: Bridge Laws Mailing List Betreff: Re: [blml] Appeals procedures Hi, the question is which alternatives should be for that hand ( for those who dont know the hand: Qx, x, AJ109xx, Kxxx- 3S pass pass X pass ? ) 3NT could be a desaster- could be the right contract - you bid this, because of hesitation of partner? pass should lead to 500-800, sometimes only 200 - you bid this, because of hesitation of partner? 4D underbid, but partner is 4th hand X, so... - you bid this, because of hesitation of partner? 5D if partner has 2 small spades, you need all other high cards, so then you can also pass the X if partner has only singleton you have good chances - you bid this, because of hesitation of partner? when you in in 5D, you 75% probably more - dont make the contract which bid is the "normal" bid without any hesitation? unfortunatly no player could be asked there, cause no TD was called at the table. which of all this alternatives was given as a choice after the hesitation? difficult or? so a weighted score would be ok?...only problem: how to weight! but 5D minus 1 as the only final result seems not really justified...or? but my points have nothing to do with appeal procedures - but anyway wanted to say it Michael Bystry schrieb: > Grattan, > > ----- Original Message ----- > From: "Grattan" > To: "Bridge Laws Mailing List" > Sent: Tuesday, November 18, 2008 8:50 AM > Subject: Re: [blml] Appeals procedures > > > >> +=+ Timeo Danaos et dona ferentes. >> I do not suggest that the 3NT bid can be justified. Returned >> to due process I would expect the Director to disallow it and >> for a diamond contract, 4 or 5, to be substituted. I believe only an >> opening club lead beats 5D because of the location of the C9 and >> of the HK. To me that suggests a 12C1(c) adjustment. >> ~ Grattan ~ +=+ >> > > Yes, only an opening club lead (which is certainly the best one anyway) > definitively beats 5D but it nevertheless requires declarer to guess DQ > which is improbable. So even if we use 12C1(c) the weight of 5D= should be > very small. > > Regards > > Maciej > > > _______________________________________________ > blml mailing list > blml at amsterdamned.org > http://www.amsterdamned.org/mailman/listinfo/blml > > > _______________________________________________ blml mailing list blml at amsterdamned.org http://www.amsterdamned.org/mailman/listinfo/blml From richard.hills at immi.gov.au Tue Nov 18 23:16:54 2008 From: richard.hills at immi.gov.au (richard.hills at immi.gov.au) Date: Wed, 19 Nov 2008 09:16:54 +1100 Subject: [blml] A Bruce's comment [SEC=UNOFFICIAL] In-Reply-To: <4922F5CC.1000702@skynet.be> Message-ID: Eric Idle, Bruces' Philosophers Song "Immanuel Kant was a real pissant." Herman De Wael: >I think your actions are comtemptible. Richard Hills: I think Herman's spelling is contemptible. :-) But seriously, the WBF Code of Practice has defined as non-contemptible (page 6): "A player who has conformed to the laws and regulations is not subject to criticism." Herman De Wael: >I was not talking about Catholics or believers of >any faith, but about a church which throughout >history has shown many signs of hypocrisy. >Anyway, apologies to any who feel offended. Richard Hills: Apology accepted. Herman De Wael: >And sorry, but I have a Gandhi-like approach to >unjust laws - I won't follow them. Richard Hills: Consistent with Immanuel Kant's categorical imperative, Mahatma Gandhi was willing to take the consequences of breaking unjust British laws -- being jailed by the British. Likewise a Director who repeatedly chooses many intentional infractions of the Laws in general, and of Law 72B1 in particular, should either: (a) follow Kant's categorical imperative by taking the Gandhian consequential decision to resign his position as a Director, or (b) switch to merely advocating changed Laws in 2018, but choose to abide by the official 2007 Lawbook and its official WBF LC Interpretations in the interim. Apologies to that blmler who may be upset by this home truth: "An International Director who chooses to remain so while intentionally infracting the Laws is a hypocrite." Best wishes Richard James Hills Recruitment Section, Level 3 Blue, workstation 15 (first on left) Department of Immigration and Citizenship Telephone: 02 6223 8453 Email: richard.hills at immi.gov.au -------------------------------------------------------------------- Important Notice: If you have received this email by mistake, please advise the sender and delete the message and attachments immediately. This email, including attachments, may contain confidential, sensitive, legally privileged and/or copyright information. Any review, retransmission, dissemination or other use of this information by persons or entities other than the intended recipient is prohibited. DIAC respects your privacy and has obligations under the Privacy Act 1988. The official departmental privacy policy can be viewed on the department's website at www.immi.gov.au. See: http://www.immi.gov.au/functional/privacy.htm --------------------------------------------------------------------- From rfrick at rfrick.info Wed Nov 19 00:55:35 2008 From: rfrick at rfrick.info (Robert Frick) Date: Tue, 18 Nov 2008 18:55:35 -0500 Subject: [blml] A player's comment In-Reply-To: <4922C61E.1000409@talktalk.net> References: <81C68581-7E16-4850-8930-B230C6D06D3A@starpower.net> <4922C61E.1000409@talktalk.net> Message-ID: On Tue, 18 Nov 2008 08:41:50 -0500, Nigel Guthrie wrote: > [Robert Frick] > The probably just asked what 2Sp meant. And you know the intended meaning > (in this example). So it you want to be honest and responsive, just > clarify for the opponents that you are not describing your partner's > intended meaning, you are describing the sytemic meaning of partner's bid > had partner understood the auction properly. Or you can follow the laws, > your choice. I am not arguing that is lawful to be open and honest and > responsive when answering opponent's questions. > > [Nigel] > The truth in this case is the systemic meaning. Compliance with this > rule isn't dishonest. But if it was, so what? > > IMO it's bizarre for Stefanie, Herman, and Robert to express revulsion > at telling a lie to comply with a bridge rule. > > The rules of a game *redefine* local morals. Games suspend social mores > in specific contexts. Have these BLMLers never played games like > *Postman's knock* or *Hangman* or *Liar dice*? Isn't this a possibility? My partner incorrectly describes my bid. Partner then bids 2Sp. Partner intends this to be artificial, not showing spades, and in fact partner is short in spades. I know this from our past agreements -- we used to play that way and we switched recently and my partner forgot. But in the "correct" auction (according to our partnershp agreement), the "correct" meaning of my partner's bid is natural. So I don't alert (waking up partner to his error, with the opponents still suspecting nothing; but I digress). Then we go through proper procedures, ending up with partner playing 3NT. No matter how the opponents ask their question, I describe the 2S bid as being natural, showing spades. As Maciej says (without dissent), "only their poorly stated questions mislead them." So they "defend correctly" (according to our partnershp agreement), never attack spades, and we make 3NT. From darkbystry at wp.pl Wed Nov 19 01:24:49 2008 From: darkbystry at wp.pl (Bystry) Date: Wed, 19 Nov 2008 01:24:49 +0100 Subject: [blml] A Bruce's comment [SEC=UNOFFICIAL] References: <49227C4E.3050007@skynet.be><001501c9495e$75116ec0$15844c59@chello.pl> <4922BA63.30707@skynet.be><000701c9498a$9f949880$15844c59@chello.pl> <4922EED2.6080806@skynet.be> Message-ID: <003101c949dd$3c10bde0$15844c59@chello.pl> Herman, > Indeed the lawmakers stated otherwise. For many good reasons, which I > will go into detail on if needed. But not now. > The point is, the lawmakers have decided that it is better for the game > of bridge for MI to continue to exist, rather than for UI to be created. > And we all accept that. It's not so simple. The lawmakers were aware that there has to be balance between both priorities. They sometimes allow for the creation of UI, sometimes not. I agree with you that there is a strange inconsequence here - the dWS would be much more complementary to the general approach of correcting MI during the auction. But because I don't agree with this approach, I don't agree with dWS either. > Now we are one second later and there is a situation where again a > choice has to be made between MI (small increment) and UI (same huge > amount), > Why is it so hard to accept that the choice should be the same one? That > MI is to be preferred over UI? I don't know why the lawmakers thought so. Ask them, not me. In my opinion UI is much less harmful. Look - the board is played normally. The NOS has full information. If the OS didn't use UI there is no reason for any adjustment. If UI was used it is often easy to adjust (demonstrably suggested, LA's). The NOS isn't bothered at all. Now think about MI. Sometimes it will be inconsequential, I agree. But much more often it will have at least indirect influence on the NOS play and now the TD has to analyze all possible options, he has to harass the NOS, believe them or not, arbitrarly allow or deny their arguments etc. The OS is left in peace. So, what is better for Bridge? > And I would not mind so much if this change happens all the time; but it > only happens at those tables where a follow-up question is asked. That > is what I am most concerned about. I understand and respect your concerns. But I simply have a different opinion about questioning. For me getting the essential information is an important part of bridge skill. Remember, that even if it sometimes hurts the OS it is only so because they _earlier_ commited an infraction. > Herman. Regards Maciej From ardelm at optusnet.com.au Wed Nov 19 04:39:08 2008 From: ardelm at optusnet.com.au (Tony Musgrove) Date: Wed, 19 Nov 2008 14:39:08 +1100 Subject: [blml] How should this have been handled ? In-Reply-To: References: Message-ID: <6.1.0.6.2.20081119143708.01d6cec0@mail.optusnet.com.au> At 10:18 PM 18/11/2008, you wrote: >Hi > > >Havent posted in a fair while .... must be the new laws are working !! > Played in a tourney on BBO last night and came up against a somewhat >unusal situation. > >The 3 hands are attached. > >Hand one it goes > >2H Pass (by me) 4D P >P double all pass > >Not sure what the standard meaning for 4D is, probably a splinter. >This sailed of for a big number. > >At this point the 2H opener called the TD and said that 4D was >possibly the most ridiculous bid he had ever seen (there was also an >implication that this wasnt the first poor bid he had witnessed). In >any event this one must have broke the camels back because ..... > > >Hand two it goes > >1D(me) 2C dbl 6NT >dbl rdbl all pass > >Now if re wanted his pd to suffer he should have bid 7C but in any >event this was his effort at trying to get even with his pd for >ridiculous bridge. The rdbl was clearly showing pd he wasnt going to >be out done. Another telephone number. > > >Hand 3 it goes > >P P 1D dbl >3D 3H P P >4D P P dbl >rdbl all pass > >Now this hand is note worthy for 2 points. >(a) His old partner had left, excused himself with the TD on the >grounds of sanity >(b) 4D-1 was the contract at 2 tables > >-2 for a zero. > >The td now adjusts boards 1 and 3 to average MINUS for partner and I >!!?. Board 2 gets adjusted to average plus. > >First off while I realise results 1 + 2 are totally skewed, we surely >should get at least average plus for them all. Result 3 though I dont >think is unreasonable and should be left alone. There were 2 4D-1 >which means this is not completely of the wall unlike the other 2 and >redouble is not completely daft. > >So what should have happened ? As we now know, the director must be named and shamed. He obviously got the info mixed up on board 1, board 2 perhaps OK, and obviously did not have the full info on board 3. Cheers, Tony (Sydney) From ardelm at optusnet.com.au Wed Nov 19 04:42:24 2008 From: ardelm at optusnet.com.au (Tony Musgrove) Date: Wed, 19 Nov 2008 14:42:24 +1100 Subject: [blml] How should this have been handled ? In-Reply-To: References: Message-ID: <6.1.0.6.2.20081119144156.01e45640@mail.optusnet.com.au> Sorry I meant score stands for case 1, Tony (Sydney) At 10:18 PM 18/11/2008, you wrote: >Hi > > >Havent posted in a fair while .... must be the new laws are working !! > Played in a tourney on BBO last night and came up against a somewhat >unusal situation. > >The 3 hands are attached. > >Hand one it goes > >2H Pass (by me) 4D P >P double all pass > >Not sure what the standard meaning for 4D is, probably a splinter. >This sailed of for a big number. > >At this point the 2H opener called the TD and said that 4D was >possibly the most ridiculous bid he had ever seen (there was also an >implication that this wasnt the first poor bid he had witnessed). In >any event this one must have broke the camels back because ..... > > >Hand two it goes > >1D(me) 2C dbl 6NT >dbl rdbl all pass > >Now if re wanted his pd to suffer he should have bid 7C but in any >event this was his effort at trying to get even with his pd for >ridiculous bridge. The rdbl was clearly showing pd he wasnt going to >be out done. Another telephone number. > > >Hand 3 it goes > >P P 1D dbl >3D 3H P P >4D P P dbl >rdbl all pass > >Now this hand is note worthy for 2 points. >(a) His old partner had left, excused himself with the TD on the >grounds of sanity >(b) 4D-1 was the contract at 2 tables > >-2 for a zero. > >The td now adjusts boards 1 and 3 to average MINUS for partner and I >!!?. Board 2 gets adjusted to average plus. > >First off while I realise results 1 + 2 are totally skewed, we surely >should get at least average plus for them all. Result 3 though I dont >think is unreasonable and should be left alone. There were 2 4D-1 >which means this is not completely of the wall unlike the other 2 and >redouble is not completely daft. > >So what should have happened ? > >Karel > >ps. I was in a local club once where on the last hand of the night it >went 1NT P 7NT. Dummy said thank you very much for the game pd (acid >and sarcasm oozing) and left. >Content-Type: image/png; name="blm hand 1.png" >X-Attachment-Id: f_fnofa09n >Content-Disposition: attachment; filename="blm hand 1.png" > >Content-Type: image/png; name="blm hand 2.png" >X-Attachment-Id: f_fnofahbv >Content-Disposition: attachment; filename="blm hand 2.png" > >Content-Type: image/png; name="blm hand 3.png" >X-Attachment-Id: f_fnofak40 >Content-Disposition: attachment; filename="blm hand 3.png" > >_______________________________________________ >blml mailing list >blml at amsterdamned.org >http://www.amsterdamned.org/mailman/listinfo/blml From darkbystry at wp.pl Wed Nov 19 04:49:01 2008 From: darkbystry at wp.pl (Bystry) Date: Wed, 19 Nov 2008 04:49:01 +0100 Subject: [blml] A Bruce's comment [SEC=UNOFFICIAL] References: <49227C4E.3050007@skynet.be><001501c9495e$75116ec0$15844c59@chello.pl> <4922BA63.30707@skynet.be><000701c9498a$9f949880$15844c59@chello.pl> <4922F5CC.1000702@skynet.be> Message-ID: <003f01c949f9$c2d51a80$15844c59@chello.pl> Herman, > >> What merits? > > > > a) nobody has to lie > > That is not a merit - the statement uttered is not a lie under one > definition - it is precisely what bidder intended. You are still under the same misconception. The Laws of Bridge _do not allow_ for questioning about the contents of anybody's hand. Why it is so hard to understand? There are _no_ such questions as "what does he have?". They are improper and unlawful. Look, somebody could misbid or psyche. Do you now claim that his partner has lied because his response do not match his partner's hand? No, all that is available to your opponents is fully disclosed set of your partnership agreements (of course here I include habits, style etc). Nothing more. We don't want to be overformalistic. We answer such badly formed questions with correct answers instead of refusing to respond and waiting for the correct form. There is no need for unnecessary aggravating the opponents. But if they later feel mislead it is only _their_ fault. They don't understand Bridge and its Laws if they expect to obtain knowledge about the contents of the hands, not about the agreements. If you still don't understand I give up. Assume we're playing different game and forget about it. > > b) the NOS know that the OS has a bidding mixup and can use it to their > > advantage > > And why is this a merit? The NOS gets something extra, the OS gets > something worse, not a merit in my book. And unfortunately not in the books of today's lawmakers. But allow me to disagree. I'm doing my best to obey the Laws. If I rarely break them I humbly accept any severe penalties I get. I just want to play Bridge - a game of skill, not a game of lawyering or coping with irreguralities. Is it so much from me to expect the officials to do their best to meet my expectations? Is it fair that after the opponents' infraction I can't concentrate on the game and instead I have to answer TD's questions, think about x variants of what would be if smth else, swallow the humiliation when the TD doesn't accept my statements and claims to know better than me what I would have done? I did _nothing_ wrong to be a subject of such things. And all I get is my "pseudo-equity" - what would have happened if there was no infraction. So be it, but I expect my opponents to be _severely_ punished, so if they come next time to my table I can expect them to obey the rules and don't take my pleasure away from me. Why are you so attentive to those "poor" offenders, why are you clapping their backs telling them "you've done nothing wrong" instead of caring for the _poor_ NOS who have to suffer many discomforts because of somebody else's misdeeds? Sorry for my emotions but I'm sick of it all. Bridge is the only game known to me where the lawmakers care more for the law-breakers than for the law-obeyers. All this to entice more bunnies and to get more income from their membership fees. Sorry, this is a point where we'll never agree. > Besides, the OS are getting something that in other places is denied > them - they do not have any entitlement to this information. > And again, this information is only received after asking a follow-up > question. This may well lead to the asking of many more questions, not > really a merit in my opinion. I think you've got the NOS in mind. Nevermind, this argument was responded by me many times and I don't want to repeat myself. > > c) much frequently the TD won't be even bothered to come to the table (the > > OS result will be sufficiently bad so the NOS won't need any improvements) > > Only if the OS are masochistically staying in the worst contract > possible. The combination of UI in two directions may well lead to even > worse possible contracts. Which the TD has to sort out. Yes, but it it often very easy to sort out. And the NOS isn't bothered, they can calmly play their game and only the OS is the subject of TD's investigations. > But just compare the two cases, if you think the TD has less work. Under > DWS, the TD has one important piece of MI to deal with, one less > important one (since it describes the hand there will usually be no > damage), and one piece of UI. Under MS, the TD still has the important > piece of UI (the NOS will always claim their one bid in the meantime was > influenced by MI), and two pieces of UI. The first MI and the first UI > are unavoidable. The second MI and second UI are avoidable. Guess which > one I prefer? > You may prefer the other one, but it's not a clear merit for MS. Ok. But this is something relative, not absolute. So for you and for me it may differ what we see as a merit. That's normal and we don't have to ponder about it. > > d) the player is more often sure what is his understanding of the > > partnership agreements but he cannot be sure what is in his partner's mind > > (not all cases are so simple as your 4NT example) > > > > Granted, but if we can settle the easy case first, will we not have a > better understanding of the more difficult one? We will have. But the player at the table not. He can be easily wrong and create much more MI than you can imagine. And as easy he can create the same UI as in CS because he will "miss" his guess. Remember, it is easy to explain something which you remember perfectly. If you are guessing it is hard to make it with sure voice, without any breaks and other signs of uncertainty. And all of this creates UI. So your school has merit for it's experienced veterans but may be hard to implement for the players who rarely are put in such position. > >> Have you ever been able to follow my full analysis of this problem? > > > > Actuallly, yes. I read many BLML discussions in the archives about this > > topic. > > > > I have not ever been allowed to work this out till the end. O, Herman, head's up. I know it's hard to defend someone's own ideas when all are against. But you've got some followers (au contraire to your claim ideas) so don't give up. But believe me - I perfectly understand your opinion. We simply differ fundamentally in our ideas what is important and more required in Bridge. > >> I have many arguments about merits. > > > > Yes, you have. They are quite logical. No, I don't agree with them. > > Arguments supporting CS are more logical. > > > > Such as the reason why you accept two widely different results at two > tables, just because a second question is asked at one of them? Can you > give any logical argument why this is good for bridge? But the same goes for the first question. It is sometimes unavoidable. It is neither good nor bad for Bridge. If you think otherwise put your efforts into inventing and applying better means of disclosure. There are some at present - screens, pre-alerts. And remember that questioning is not a no-lose strategy. Much more often questioners will fall into trouble because of UI than the opponents. > >> Many more than I have ever heard about MS. Don't talk to me about merits. > > > > In my humble opinion it is very hard for someone, who is so much devoted to > > one idea, than to listen to and appreciate cogent arguments of his > > opponents. > > In my equally humble opinion this argument works both ways. You are > equally devoted to your idea (and you find consolation in being in a > majority) that you too fail to listen and appreciate to cogent arguments. No. Apparently you didn't understand my position. I've just stated it clear above, at point b). In fact I'm not a follower of the CS. I'd rather see every MI corrected as soon as possible. But if I'm left with the choice between the CS and the dWS my choice is obvious (really I've got no choice because dWS in unlawful). > You have given four arguments above. I have patiently read them and > given answers. Why does that not help you change your opinion? You ask > me to change mine, but you have no intention of changing yours. No, I'm not asking you to do anything. I've stated my arguments and if they're unconvincing for you it's fine. I won't change my position because it is fundamentally embedded in my perception what is good for Bridge. Your perception may be different and it is not bad at all. If everyone agreed with everyone there would be no development. > Menawhile, I have given cogent arguments of my own, and I don't get > reasoned replies. Is it any wonder that I believe this situation is not > strictly symmetric? It's because you are caring for the OS (to decrease their bad results) and for the TD's (to give them less work). I'm caring for the NOS (to make up for their discomfort because of infraction). > > You see. Creating UI doesn't hurt the NOS. Only using it can hurt them. So > > your side should not use it, otherwise you will get your deserved poor score > > from the TD. Period. > > > > And here you are simply wrong. Creating UI DOES hurt the OS (I assume > the N above was a typo). Partner is not allowed to choose the "correct" > action. And so I want to avoid giving UI. But here you are, advocating a > law change in which you OBLIGE me to give UI! Have you ever thought > about what the penalty ought to be for refusing to give obligatory UI? > Do you really think this is not a merit for the DWS? No, I'm not wrong. N was _not_ a typo. You must forgive me, I'm back to using written English after about 4-year pause and I'm not always clear in my statements. It should have been: "Creating UI [by the OS] doesn't hurt the NOS. Only using it [by the OS] can hurt them [the NOS]". Sorry for troubles. > > MI is harder to manage. The NOS has to prove it hurted them. Sometimes their > > subtle decisions will miss a point, but the TD won't give them redress. And > > there are those nebulous "equity" laws. No, thanks. The Laws shouldn't be > > used as a rescue for offenders, denying the NOS their due good scores > > because of your mixups. > > > > And this is exactly where you have the wrong comparison. DWS does not > end up with a better score than MS, as you seem to think. If no > follow-up question is asked, DWS gets you exactly the same result as MS. If no follow-up questions are asked, the dWS is identical to the CS. That has nothing to do with our discussion. > it is only after a follow-up question, that MS lands you in worse shit > than DWS. I fail to see why I should suffer such a fate. My neighbour > who has done exactly the same thing wrong (forgetting his system) will > get exactly the same bad score as I do - if his opponent does not ask > the question. Why should I suffer because my opponent is more lazy than his? No, you suffer because your opponent is _active_. Bad that doesn't matter. It is just a rub-of-the-green. As you would suffer when your opponent guesses two-way finesse, bids good slam, psyches effectivly, misbids and fixes you. And please, don't start again that this will result in deluge of questioning. We're operating under the CS for some years and nothing like that has happened. > Indeed, and the first piece of UI is unavoidable, but the second one > isn't. I want to be allowed to avoid giving that UI. Selfish of you. And I want to avoid misinforming my opponents. If it is at cost of giving some UI to my partner (who will correctly bend backwards not to use it), then so be it. Our bad score will be a reminder to remember our agreements. And sometimes even we will land on our feet and get a good score. In a lawful way. Some words of clearance - I'm not saint. But I'd want my opponents to behave in the describes ways, so I'm obliged to do it first myself. > > Sorry, but they are _not_ entitled to know what is in your or your partner's > > hand. Herman, you're a TD, you know the Laws. > > Indeed. They are not entitled that. But it is what they want to know. > So if you tell them something else, you are misleading them. Simple > language, not laws at all. That is not my problem. They mislead themselves. But I have a proposal. Please tell your followers to wear some buttons or labels so that I could identify them. Then I won't respond to their badly phrased questions so that they won't feel mislead. > > I'll tell you more - I > > wouldn't even want to know what is in their hands. > > ????????? Yes, Herman. Bridge is much less interesting if you know the contents of opponents' hands. There is no guessing, no hand counting, no cooperation with partner on defense. Just 4-hand quiz solving. Thanks, I'll play my way. > > And so what? If you're so troubled with this than think about other, better > > means of disclosure but don't deny to anyone what is for him lawfully > > available. Questioning often leads to reaching different situations - for > > your side and for the opponents. It can create UI regardless which school > > you follow. > > > > No, it doesn't create UI in DWS. It does. First question creates UI, just not the second one. > > Firstly - the > > player is _not_ interested how many aces your partner has. > > Of course he is! You're still under the same misconception? Ok, I'll leave it as it is. > He wants to know how to defend. Knowing that 5Di showed diamonds is > useless to him. Knowing that it showed 1 ace is useful. If you don't > agree with that you are deliberately being obtuse. But he is not entitled to know how many Aces does your partner have. How many times do I have to write it? He is entitled to know that your partner prefers diamonds to clubs. Now if this MI causes damage the score will be adjusted. But a number of Aces is just a red herring. It is totally irrevelant. What's more, the opponents can read your CC and find this answer themselves. Now they are simply in better position knowing that your side went astray and still having the possibility of redress for MI connected to "better diamonds". Cool and just. Now we could talk about other topic - should it be allowed to ask about any partnership agreements, even unrelated to the current auction (that would "level" all the cases, because Blackwood will be described on CC, but some bids not). But let's leave it for another time. > Of course it is better for him to know we have a bidding > misunderstanding. But he is not entitled to that knowledge. You cannot > have it both ways - first saying he is not interested in knowing there > is one ace, and then that he is is interested in knowing about a > misunderstanding. He is entitled to this knowledge. The action of your partner's misexplanation caused that. Of course they have to wind up this themselves. And they still risk getting poor result when they misread the situation. They can double such 5D, but if your partner accidentally _has_ a good diamond fit and you make 5D they won't get redress because they were fully and correctly informed (provided that an earlier auction was free of MI). > > Thirdly - there is a golden rule - when alerted > > ask, do not assume. > > Who said anything about assuming? I'm talking "knowing", like in > remembering what the answers system was previously, or having studied > opponents system, or reading the CC. All of these are to be preferred > (in a general bridge sense) to asking, and yet you award an advantage to > "asking". Yes, sometimes (but rarely) asking would be rewarded. So what? I still maintain that such asking much often will result in troubles for the questioners than will reward them with establishing your mixup. The odds are so bad that nobody will risk numerous adverse adjustments just to get their one-time top. > And this is far worse than you seem to imagine. Turn the example around > with 4NT being described as "minors". Who in heaven's name would dream > of asking what 5Di means in this sequence? Yet if you do ask, you are > rewarded with a windfall when the response by a MS adept is "1 ace". So I am. So I will get 5 poor scores because of UI restraining my partner's choices in other rounds just to catch Herman and his partner having a mixup! ;-) > > So he asked and got rewarded. He showed his interest in > > the auction what I see as an important part of Bridge. And don't impute any > > bad motives on his side. He couldn't have foreseen the result. It was your > > partnership who caused this mess, why blame him? > > > At this time in history, he cannot foresee the result. But when the laws > are rewritten with the change in L20F5a clearly in view, he can foresee > a result. Expect 100% of questions from some opponents. See if you like > that? Change in L20F5a? Herman, nearly nobody knows that such law exists and what it states. Most Bridge players learn the rules at the table - from their friends, local TD's, opponents. They are taught they have the right to ask questions. Then they learn about UI, that questions aren't risk-free. They learn they should disclose partnership agreements (here there is some small minority in Poland who are under the misconception - you could recruit them to your school, but they don't even know that somebody like Herman de Wael exists). But all this players are not aware of what we're talking about. And they still won't be after L20F5a will be changed. > > And rightfully so. You know, I can understand being sorry for the poor > > player whose opponents guess 5 two-way finesses during one session. He is > > just plainly unlucky. I can clap him on his back, go for a beer with him. > > But the player who breaks the Laws cannot expect even a little dose of > > compassion from me. > > > > We are not talking of breaking laws here - we are talking about what the > law should be. > And sorry, but I have a Gandhi-like approach to unjust laws - I won't > follow them. That's your choice. I can only tell you that I dislike some Laws very much (claims, corrections of mistaken bids and plays, new revoke law and especially new 12C1). But nevertheless I still play Bridge and obey the rules. > > Besides > > don't forget that questions generate UI and your argument is statistically > > at least feeble. > > Oh no they don't, because I will ask about the previous bid at every > single turn. Just a "what is 5Di?" will do. And only about one call of > course - my partner will have asked about the other one. And how much UI will you create for _your_ partnership? Besides questioning about single bids is not generally allowed. > Since there is no UI in asking all the time - I am real. > Anyway, there is just as much UI in looking at a CC. You won't prove that you always ask. The TD won't believe you. And he'll rule against you. > > To that question I have already responded. > > > To be fair, yes you have. Your arguments are sadly unconvincing (to me). As I've already written it is nothing bad. My position is clearly stated and it is your choice what to think about it. > Herman. Regards Maciej From richard.hills at immi.gov.au Wed Nov 19 05:18:15 2008 From: richard.hills at immi.gov.au (richard.hills at immi.gov.au) Date: Wed, 19 Nov 2008 15:18:15 +1100 Subject: [blml] A Bruce's comment [SEC=UNOFFICIAL] In-Reply-To: <49227C4E.3050007@skynet.be> Message-ID: . Richard Hills noted: >>.....There is no Law which requires me to lie to the >>opponents..... Herman De Wael asserted: >Yes there is. L20F5a. > >This law requires you to keep quiet over an untruth >spoken by partner. > >If you don't call that lying then you exhibit a level of >hypocrisy..... Richard Hills refutes: Qui tacet consentire videtire ubi loqui debuit ac potuit. Best wishes Richard James Hills Recruitment Section, Level 3 Blue, workstation 15 (first on left) Department of Immigration and Citizenship Telephone: 02 6223 8453 Email: richard.hills at immi.gov.au -------------------------------------------------------------------- Important Notice: If you have received this email by mistake, please advise the sender and delete the message and attachments immediately. This email, including attachments, may contain confidential, sensitive, legally privileged and/or copyright information. Any review, retransmission, dissemination or other use of this information by persons or entities other than the intended recipient is prohibited. DIAC respects your privacy and has obligations under the Privacy Act 1988. The official departmental privacy policy can be viewed on the department's website at www.immi.gov.au. See: http://www.immi.gov.au/functional/privacy.htm --------------------------------------------------------------------- From grabiner at alumni.princeton.edu Wed Nov 19 05:32:39 2008 From: grabiner at alumni.princeton.edu (David Grabiner) Date: Tue, 18 Nov 2008 23:32:39 -0500 Subject: [blml] How should this have been handled ? In-Reply-To: References: Message-ID: Karel writes: > Hand one it goes > > 2H Pass (by me) 4D P > P double all pass > > Not sure what the standard meaning for 4D is, probably a splinter. > This sailed of for a big number. > > At this point the 2H opener called the TD and said that 4D was > possibly the most ridiculous bid he had ever seen (there was also an > implication that this wasnt the first poor bid he had witnessed). In > any event this one must have broke the camels back because ..... > > > Hand two it goes > > 1D(me) 2C dbl 6NT > dbl rdbl all pass > > Now if re wanted his pd to suffer he should have bid 7C but in any > event this was his effort at trying to get even with his pd for > ridiculous bridge. The rdbl was clearly showing pd he wasnt going to > be out done. Another telephone number. > > > Hand 3 it goes > > P P 1D dbl > 3D 3H P P > 4D P P dbl > rdbl all pass > > Now this hand is note worthy for 2 points. > (a) His old partner had left, excused himself with the TD on the > grounds of sanity > (b) 4D-1 was the contract at 2 tables > > -2 for a zero. I would interpret this as the opponents getting a bad score on board 1 and then effectively withdrawing from the tournament and trying to spoil it for everyone else. The ruling should be result stands on board 1, average-plus on boards 2 and 3, and both opponents suspended. I would throw out Board 3 because of the redouble. The redoubler has AQT 63 KJT2 J985, and partner opened in third seat, so there is no reason to expect 4D to make, particularly once it gets doubled. And there is no point in the redouble; 4Dx will be a top if it makes, but the redouble could turn -200 which is duplicated at other tables into -400 which isn't. > The td now adjusts boards 1 and 3 to average MINUS for partner and I > !!?. Board 2 gets adjusted to average plus. The average-minus is obviously wrong, and I would ask the TD for a reconsideration. You can only get average-minus on an unplayable board if your side was at fault. From darkbystry at wp.pl Wed Nov 19 05:50:11 2008 From: darkbystry at wp.pl (Bystry) Date: Wed, 19 Nov 2008 05:50:11 +0100 Subject: [blml] Appeals procedures References: <009d01c948fe$6c45da20$0302a8c0@Mildred><4922051C.9060805@bridgepro.de><001401c94952$84db24b0$0302a8c0@Mildred><002701c94962$0024bc80$15844c59@chello.pl><4922C40D.1050700@bridgepro.de><001a01c9498d$6b34ab40$15844c59@chello.pl> <2a1c3a560811181054u665d9e7ej9d6816fca8dc49b0@mail.gmail.com> Message-ID: <004b01c94a02$4e2976a0$15844c59@chello.pl> Wayne, > If you are arguing that 4D is not an allowed bid then the only > adjustment you can make is between 5D -1 and 5D=. I'm not arguing that 4D is not allowed. I'm not sure if it is enough suggested over 5D. That's because of an unambiguity in the definition. What is "demonstrably suggested"? I saw many discussions on this topic and I still don't know of any clear conclusion. Is 30% enough? 50% 75%? It's easy to demonstrate that the hesitation makes 5D less attractive. normally partner has 3-4 diamonds for a balancing double. But here it is probable that he was afraid of our diamond call and he was thinking about bidding 3NT, 4C, 4H. But _how much less_ attractive? It's sometimes very hard to estimate. So we can apply three different standarts: a) if it is not nearly 100% then no adjustment. Such situation is for me not acceptable because we would be only able to catch abusers in clear-cut situations like hesitant penalty doubles. But we know that experienced partnership (especially high-level) are much more attuned to their behaviors and it would result in great amounts of used UI to go unpunished. b) if it is more than 0% then we adjust. It is equally bad, because sometimes the player would be put in situation when all what he does will be adjusted. c) we adjust if it is probable. Here the best method is to poll players and if e.g. x% of them sees the connection then we satisfy the condition. But what number should we substitute for x? Does anybody has a clear, well thought proposal? > I believe your reasoning is flawed in that 1=4=4=4 is not a normal > takeout double. It is a perfect one. In responding to a takeout > double one doesn't normally assume partner has the perfect hand. On > this occasion there is addition information that suggests a stiff > spade is unlikely - Partner of the 3S bidder didn't raise which would > often occur with three or four spades. A perfect one? I don't think so. But there is a class of "normal" balancing doubles. We can include: 0-(4-4-5), 0-(3-4-6), 0-(3-5-5), 1-4-4-4, 1-(3-4-5), 1-(5-2-5), 1-(2-4-6), 2-(3-4-4), 2-(2-4-5), 3-(2-4-4 with hearts), 3-(2-2-5 with hearts). I've put more than I consider normal to avoid any corrections. Now, when will it be a difficult choice to double? In all cases where one side suit is short. And in cases of semibalanced or balanced hands, which require more general strength. In all this cases 5D (and even 4D) becomes less attractive. But the chances for 3NT and pass to be succesful rise. I know that 3NT is not directly pointed. But if the player thinks along my lines and follows them, then in your opinion we should not adjust? Because sometimes he would get it wrong? I saw many cases when the UI-suggested action was in fact a losing one, but appeared succesful after poor or unlucky defense (play). That doesn't change the fact, that player _used_ UI. And that is unlawful. As to the chances for spade singleton - both sides were vulnerable. I don't think that the player of Nunes class would raise with something like 3-(3-3-4) and scattered 7-9 HCP. Opponents have 22-26 HCP, no sure fit, no bidding space. And 4S will go for 500-800. > Disallowing 3NT specifically requires that you can reason that the > slow double demonstably suggests that partner has a spade honour > rather than no spade honour. Why? You can as well prove that all the other LA's are strongly anti-suggested. Here it is not so, but I can imagine such an example. Going further, in many cases you don't have to prove that player's hesitation suggested some specific honours. It is enough to show that the hesitation showed better hand, shorter opponents suit and many other things. And sometimes those extras won't fit to player's hand but it is not a reason not to adjust. > To me that is a wild conclusion. Even > knowing that partner has two spades then the odds are (2 x 9 + 1) / > 11C2 = 19/55 = 0.34 which is well against the odds. It is against the odds. 3NT is a wild bid. But do you really believe that any player of sufficient skill would risk going 3 down vulnerable having +600 in 5D available without any clou? Let's think about it again. Partner's hesitation suggest atypical distribution. Most probably he is too balanced. If it is so, then we can expect him to have more spades and more HCP. So the chances for a spade honour rise. And now, having taken into consideration all the points we're back at our initial question - what does "demonstrably suggested" mean. For me it is enough to reason along this lines: initially 5D has a chance of, say, 60%, 4D 30%, pass and 3NT 5% both. Now, after the hesitation, 5D has 15%, 4D 25%, pass and 3NT 30% both. If you don't agree with my percentages substitute yours. For me it is enough to disallow 3NT but for you it may not be. And I don't claim I'm right, that's how I would have decided knowing only given facts and being unable to poll players. > Has it yet been determined that the slowness of the double transmitted > through the screen. This needs more than an admission that the double > was slow. I'm unable to establish that. If there was no UI received, no adjustment. > > -- > Wayne Burrows > Palmerston North > New Zealand Regards Maciej From dalburn at btopenworld.com Wed Nov 19 06:53:13 2008 From: dalburn at btopenworld.com (David Burn) Date: Wed, 19 Nov 2008 05:53:13 -0000 Subject: [blml] A Bruce's comment [SEC=UNOFFICIAL] In-Reply-To: References: <49227C4E.3050007@skynet.be> Message-ID: <000001c94a0b$1cc70330$56550990$@com> [RH] Qui tacet consentire videtire ubi loqui debuit ac potuit. [DALB] That's "videtur", not "videtire". But as they used to say, si tacuisses philosophus mansisses. For the benefit of any laymen who may have wandered in, the Latin phrase "qui tacet consentire videtur" means "he who keeps silent is seen to consent". The rest means "when [literally: where] he ought to have, and could have, spoken". It is not clear to me what all this has to do with the dWS any more. Herman has argued his case as eloquently as anyone could, has succeeded in bringing it to the attention of the Lawmakers, and has been directly rebuffed when those Lawmakers said in effect: "when faced with an obligation to explain your system (under Law 40 and others) that conflicts with an obligation not to give UI to partner (under Law 20), the former obligation prevails". As Mr Chesterton said: "You who have snarled through the ages, take your answer and go." David Burn London, England From darkbystry at wp.pl Wed Nov 19 07:51:01 2008 From: darkbystry at wp.pl (Bystry) Date: Wed, 19 Nov 2008 07:51:01 +0100 Subject: [blml] A Bruce's comment [SEC=UNOFFICIAL] References: <49227C4E.3050007@skynet.be> <000001c94a0b$1cc70330$56550990$@com> Message-ID: <009f01c94a13$2f7faba0$15844c59@chello.pl> > [RH] > > Qui tacet consentire videtire ubi loqui debuit ac potuit. > > [DALB] > > That's "videtur", not "videtire". But as they used to say, si tacuisses > philosophus mansisses. > > For the benefit of any laymen who may have wandered in, the Latin phrase > "qui tacet consentire videtur" means "he who keeps silent is seen to > consent". The rest means "when [literally: where] he ought to have, and > could have, spoken". > > It is not clear to me what all this has to do with the dWS any more. Actually Richard has pointed to Herman that his statement [Herman] > This law requires you to keep quiet over an untruth > spoken by partner. > If you don't call that lying then you exhibit a level of > hypocrisy..... [/Herman] would be right if you take into consideration only the first part of this clever Latinian phrase. But as the following proviso is added, Herman's argument falls a bit short of being cogent. > Herman > has argued his case as eloquently as anyone could, has succeeded in bringing > it to the attention of the Lawmakers, and has been directly rebuffed when > those Lawmakers said in effect: "when faced with an obligation to explain > your system (under Law 40 and others) that conflicts with an obligation not > to give UI to partner (under Law 20), the former obligation prevails". As Mr > Chesterton said: "You who have snarled through the ages, take your answer > and go." "Never take away somebody's hope, it may be all they have." > David Burn > London, England Regards Maciej From richard.hills at immi.gov.au Wed Nov 19 07:56:05 2008 From: richard.hills at immi.gov.au (richard.hills at immi.gov.au) Date: Wed, 19 Nov 2008 17:56:05 +1100 Subject: [blml] The Nebraskan issue [SEC=UNOFFICIAL] In-Reply-To: <000801c9407b$b54c2070$0302a8c0@Mildred> Message-ID: Grattan Endicott: >In the Southern Hemisphere water swirls in antipodean >fashion. David Moren et al: On a merry-go-round in the night Coriolis was shaken with fright Despite how he walked 'Twas like he was stalked By some fiend always pushing him right Grattan Endicott: >In Nebraska blue donkeys walk to the left of red >elephants, Richard Hills: Omaha Obama! Nigel Guthrie: >>IMO, BLML is an inappropriate medium for ... political >>rants. Richard Hills: Omaha Obama! In 1960 Alabama split its Electoral College votes between Senators Kennedy and Byrd. Now in 2008 Byrd is still a Senator, and Nebraska has split its Electoral College votes between Senators McCain and Obama. Grattan Endicott: >>while in England the red political spectrum is found >>to the left of the blue. All perception. Richard Hills: No, the trans-Atlantic differences between colour labels of political parties of the left and right is due to "path dependence". An initial semi-random setting of a standard (British party colours defined decades ago, American party colours standardised as recently as at the 2000 election) followed by a lock-in of that standard. For example, changing from the standard QWERTY keyboard to the slightly more effective Dvorak keyboard is more trouble than it is worth. Likewise, many aspects of Bridge Law were fixed by semi- arbitrary "path dependence" choices up to three-quarters of a century ago. For example: (a) minors are worth 20 per trick, instead of the slightly more effective 30 per trick, and (b) most players do give full disclosure of their mutual partnership understandings, instead of the slightly more effective strategy of disclosing only that which will not restrict partner's choice amongst logical alternatives. Best wishes Richard James Hills Recruitment Section, Level 3 Blue, workstation 15 (first on left) Department of Immigration and Citizenship Telephone: 02 6223 8453 Email: richard.hills at immi.gov.au -------------------------------------------------------------------- Important Notice: If you have received this email by mistake, please advise the sender and delete the message and attachments immediately. This email, including attachments, may contain confidential, sensitive, legally privileged and/or copyright information. Any review, retransmission, dissemination or other use of this information by persons or entities other than the intended recipient is prohibited. DIAC respects your privacy and has obligations under the Privacy Act 1988. The official departmental privacy policy can be viewed on the department's website at www.immi.gov.au. See: http://www.immi.gov.au/functional/privacy.htm --------------------------------------------------------------------- From Hermandw at skynet.be Wed Nov 19 08:50:27 2008 From: Hermandw at skynet.be (Herman De Wael) Date: Wed, 19 Nov 2008 08:50:27 +0100 Subject: [blml] A Bruce's comment [SEC=UNOFFICIAL] In-Reply-To: <003101c949dd$3c10bde0$15844c59@chello.pl> References: <49227C4E.3050007@skynet.be><001501c9495e$75116ec0$15844c59@chello.pl> <4922BA63.30707@skynet.be><000701c9498a$9f949880$15844c59@chello.pl> <4922EED2.6080806@skynet.be> <003101c949dd$3c10bde0$15844c59@chello.pl> Message-ID: <4923C543.4040408@skynet.be> Hello Maciej, Bystry wrote: > Herman, > >> Indeed the lawmakers stated otherwise. For many good reasons, which I >> will go into detail on if needed. But not now. >> The point is, the lawmakers have decided that it is better for the game >> of bridge for MI to continue to exist, rather than for UI to be created. >> And we all accept that. > > It's not so simple. The lawmakers were aware that there has to be balance > between both priorities. They sometimes allow for the creation of UI, > sometimes not. You are talking about two different things. The lawmakers sometimes allow for the creation of UI, yes; example: thinking is permitted; sometimes not, also yes; example: dummy may not ask a defender if he has revoked. That sentence is true. But your first sentence is not true when coupled with the second one: The lawmakers were aware that there has to be balance between both priorities (of course true) but then saying that they sometimes allow UI and not, is NOT true. They wrote two laws about cases where UI and MI are in possible conflict: L20F1 (the partner should reply) and L20F5a (the bidder shall not correct). Both times the lawmakers have made a clear choice: UI is to be avoided, even at the price of MI. If you believe that the DWS is a third case and the lawmakers have ruled in the other direction, then this discussion is futile. I am still hoping the lawmakers see the error of their ways. > I agree with you that there is a strange inconsequence here - > the dWS would be much more complementary to the general approach of > correcting MI during the auction. But because I don't agree with this > approach, I don't agree with dWS either. > How can you not agree with an apporach which is the same as two other cases that you are so familiar with that you don't even recognize them for what they are: a clear choice by the WBF. Herman. From Hermandw at skynet.be Wed Nov 19 09:04:05 2008 From: Hermandw at skynet.be (Herman De Wael) Date: Wed, 19 Nov 2008 09:04:05 +0100 Subject: [blml] A Bruce's comment [SEC=UNOFFICIAL] In-Reply-To: <003101c949dd$3c10bde0$15844c59@chello.pl> References: <49227C4E.3050007@skynet.be><001501c9495e$75116ec0$15844c59@chello.pl> <4922BA63.30707@skynet.be><000701c9498a$9f949880$15844c59@chello.pl> <4922EED2.6080806@skynet.be> <003101c949dd$3c10bde0$15844c59@chello.pl> Message-ID: <4923C875.30407@skynet.be> Bystry wrote: >> Why is it so hard to accept that the choice should be the same one? That >> MI is to be preferred over UI? > > I don't know why the lawmakers thought so. Ask them, not me. You have played under these laws for your whole life. You have been told not to correct MI given by partner. You have probably told off newcomers for doing so. It's the way bridge is played. If you don't recognize this as a good thing, then why don't you ask to have it changed? > In my opinion > UI is much less harmful. A wrong opinion, when you start thinking about it - please do so. > Look - the board is played normally. The NOS has > full information. If the OS didn't use UI there is no reason for any > adjustment. If UI was used it is often easy to adjust (demonstrably > suggested, LA's). The NOS isn't bothered at all. Now think about MI. > Sometimes it will be inconsequential, I agree. But much more often it will > have at least indirect influence on the NOS play and now the TD has to > analyze all possible options, he has to harass the NOS, believe them or not, > arbitrarly allow or deny their arguments etc. The OS is left in peace. So, > what is better for Bridge? > Maciej, when MI occurs, the board is always in trouble. If you think UI is less important than MI, then why don't you start correcting partner's MI immediately from now on? You may well be right (no, I don't think so, I've thought about that, but I grant you the right to think otherwise), but then why not take this to its logical conclusion and start correcting MI straight away? Your arguments are just as valid there. The WBF have decided that it is better for the game of bridge if only one partner has UI, not both. They acknowledge that some MI is created and left in place. We are all, as TD's, qualified to handle MI. Bridge has been played like this for generations. How can you say that the opposite would be better for bridge? Why don't you start a campaign to change L20F5a then? I suggested this to Grattan last year as well - nothing has come of it. It would be a totally different game! >> And I would not mind so much if this change happens all the time; but it >> only happens at those tables where a follow-up question is asked. That >> is what I am most concerned about. > > I understand and respect your concerns. But I simply have a different > opinion about questioning. For me getting the essential information is an > important part of bridge skill. Remember, that even if it sometimes hurts > the OS it is only so because they _earlier_ commited an infraction. > No Maciej, it is not a question about getting essential information. it is a question of _asking_ for that information. Opponents can get the information is half a dozen ways, but only one way triggers this extraordinary bonus for them. The five other opponents get the information they are interested (1 ace), the sixth get something totally different (we were having a misunderstanding and now we have UI both so we can't escape anymore). None of the six get the information they are entitled to (diamond preference) but none of the six are damaged by that lack, because no one, not even partner, can say anything sensible about the diamond content in bidder's hand. Those are the two options you should be looking at. Look at an opponent who takes the CC and checks that 5Di shows 1 ace. Do you feel he's been damaged by not knowing there was a misunderstanding? Then why do you insist that if he asks rather than looks up, he gets that information? And if you feel the looker-upper is in fact damaged, then compare his situation with the one of the opponent who doesn't care about the number of aces? Why does he not also get the information that there was a misunderstanding? >> Herman. > > Regards > > Maciej > Herman. From Hermandw at skynet.be Wed Nov 19 09:10:03 2008 From: Hermandw at skynet.be (Herman De Wael) Date: Wed, 19 Nov 2008 09:10:03 +0100 Subject: [blml] A Bruce's comment [SEC=UNOFFICIAL] In-Reply-To: <003f01c949f9$c2d51a80$15844c59@chello.pl> References: <49227C4E.3050007@skynet.be><001501c9495e$75116ec0$15844c59@chello.pl> <4922BA63.30707@skynet.be><000701c9498a$9f949880$15844c59@chello.pl> <4922F5CC.1000702@skynet.be> <003f01c949f9$c2d51a80$15844c59@chello.pl> Message-ID: <4923C9DB.5080009@skynet.be> Bystry wrote: > Herman, > >>>> What merits? >>> a) nobody has to lie >> That is not a merit - the statement uttered is not a lie under one >> definition - it is precisely what bidder intended. > > You are still under the same misconception. The Laws of Bridge _do not > allow_ for questioning about the contents of anybody's hand. Why it is so > hard to understand? There are _no_ such questions as "what does he have?". > They are improper and unlawful. Look, somebody could misbid or psyche. Do > you now claim that his partner has lied because his response do not match > his partner's hand? No, all that is available to your opponents is fully > disclosed set of your partnership agreements (of course here I include > habits, style etc). Nothing more. > > We don't want to be overformalistic. We answer such badly formed questions > with correct answers instead of refusing to respond and waiting for the > correct form. There is no need for unnecessary aggravating the opponents. > But if they later feel mislead it is only _their_ fault. They don't > understand Bridge and its Laws if they expect to obtain knowledge about the > contents of the hands, not about the agreements. > > If you still don't understand I give up. Assume we're playing different game > and forget about it. > I know all the above. But it is YOU who is using the argument that a player needs to lie about PU in order to follow DWS principles. You, not me. In my defence, I say that I don't feel that this is lying. I am not considering myself in breach of any moral obligations by saying that my partner is showing 1 ace. I know that I am legally wrong, that by PU 5Di shows diamond preference, and I accept that this may lead to an additional score adjustment, but I don't feel bad about that. Especially since I have followed another law (L20F5a) which I consider more important. By using as an argument a) that nobody needs to lie, you bring into bridge a moral argument which is very weak, while actually intending a legal argument which is easily countered. If your argument had been a) you break L40, then my response is even easier: but not L20F5a. It is your argument which is flawed, not my understanding of the laws or anything. Herman. From Hermandw at skynet.be Wed Nov 19 09:13:41 2008 From: Hermandw at skynet.be (Herman De Wael) Date: Wed, 19 Nov 2008 09:13:41 +0100 Subject: [blml] A Bruce's comment [SEC=UNOFFICIAL] In-Reply-To: <003f01c949f9$c2d51a80$15844c59@chello.pl> References: <49227C4E.3050007@skynet.be><001501c9495e$75116ec0$15844c59@chello.pl> <4922BA63.30707@skynet.be><000701c9498a$9f949880$15844c59@chello.pl> <4922F5CC.1000702@skynet.be> <003f01c949f9$c2d51a80$15844c59@chello.pl> Message-ID: <4923CAB5.5020203@skynet.be> Bystry wrote: > >>> c) much frequently the TD won't be even bothered to come to the table > (the >>> OS result will be sufficiently bad so the NOS won't need any > improvements) >> Only if the OS are masochistically staying in the worst contract >> possible. The combination of UI in two directions may well lead to even >> worse possible contracts. Which the TD has to sort out. > > Yes, but it it often very easy to sort out. And the NOS isn't bothered, they > can calmly play their game and only the OS is the subject of TD's > investigations. > But that is also true at the DWS table - the TD sorts out the MI and the UI in one direction and the NOS isn't bothered! >> But just compare the two cases, if you think the TD has less work. Under >> DWS, the TD has one important piece of MI to deal with, one less >> important one (since it describes the hand there will usually be no >> damage), and one piece of UI. Under MS, the TD still has the important >> piece of UI (the NOS will always claim their one bid in the meantime was >> influenced by MI), and two pieces of UI. The first MI and the first UI >> are unavoidable. The second MI and second UI are avoidable. Guess which >> one I prefer? >> You may prefer the other one, but it's not a clear merit for MS. > > Ok. But this is something relative, not absolute. So for you and for me it > may differ what we see as a merit. That's normal and we don't have to ponder > about it. > OK, but then you don't have a point. Maybe I don't either, but this was your list of merits. A and C have already gone. Herman. From Hermandw at skynet.be Wed Nov 19 09:27:21 2008 From: Hermandw at skynet.be (Herman De Wael) Date: Wed, 19 Nov 2008 09:27:21 +0100 Subject: [blml] A Bruce's comment [SEC=UNOFFICIAL] In-Reply-To: <003f01c949f9$c2d51a80$15844c59@chello.pl> References: <49227C4E.3050007@skynet.be><001501c9495e$75116ec0$15844c59@chello.pl> <4922BA63.30707@skynet.be><000701c9498a$9f949880$15844c59@chello.pl> <4922F5CC.1000702@skynet.be> <003f01c949f9$c2d51a80$15844c59@chello.pl> Message-ID: <4923CDE9.3020707@skynet.be> Bystry wrote: >>> >> In my equally humble opinion this argument works both ways. You are >> equally devoted to your idea (and you find consolation in being in a >> majority) that you too fail to listen and appreciate to cogent arguments. > > No. Apparently you didn't understand my position. I've just stated it clear > above, at point b). In fact I'm not a follower of the CS. I'd rather see > every MI corrected as soon as possible. But if I'm left with the choice > between the CS and the dWS my choice is obvious (really I've got no choice > because dWS in unlawful). > So in fact you want L20F5a changed, but since that cannot be arranged, you are going for the second best thing? Well, in that case, there is no need talking to you about equality of OS against askers and non-askers. >> You have given four arguments above. I have patiently read them and >> given answers. Why does that not help you change your opinion? You ask >> me to change mine, but you have no intention of changing yours. > > No, I'm not asking you to do anything. I've stated my arguments and if > they're unconvincing for you it's fine. I won't change my position because > it is fundamentally embedded in my perception what is good for Bridge. Your > perception may be different and it is not bad at all. If everyone agreed > with everyone there would be no development. > OK, so let's just agree to disagree - but meanwhile you're in the majority and are happy there? Nice one! >> Menawhile, I have given cogent arguments of my own, and I don't get >> reasoned replies. Is it any wonder that I believe this situation is not >> strictly symmetric? > > It's because you are caring for the OS (to decrease their bad results) and > for the TD's (to give them less work). I'm caring for the NOS (to make up > for their discomfort because of infraction). > Yes, and you are of the super-masochistic kind who would even like to see L20F5a removed. Tell me Maciej, what do you do when your partner misexplains? If you are at all consistent, you would correct it! >>> You see. Creating UI doesn't hurt the NOS. Only using it can hurt them. > So >>> your side should not use it, otherwise you will get your deserved poor > score >>> from the TD. Period. >>> >> And here you are simply wrong. Creating UI DOES hurt the OS (I assume >> the N above was a typo). Partner is not allowed to choose the "correct" >> action. And so I want to avoid giving UI. But here you are, advocating a >> law change in which you OBLIGE me to give UI! Have you ever thought >> about what the penalty ought to be for refusing to give obligatory UI? >> Do you really think this is not a merit for the DWS? > > No, I'm not wrong. N was _not_ a typo. You must forgive me, I'm back to > using written English after about 4-year pause and I'm not always clear in > my statements. It should have been: "Creating UI [by the OS] doesn't hurt > the NOS. Only using it [by the OS] can hurt them [the NOS]". Sorry for > troubles. > OK granted. So you happily create UI, do you? Well, that's a masochistic way of playing bridge but your opponents will certainly be happy. >>> MI is harder to manage. The NOS has to prove it hurted them. Sometimes > their >>> subtle decisions will miss a point, but the TD won't give them redress. > And >>> there are those nebulous "equity" laws. No, thanks. The Laws shouldn't > be >>> used as a rescue for offenders, denying the NOS their due good scores >>> because of your mixups. >>> >> And this is exactly where you have the wrong comparison. DWS does not >> end up with a better score than MS, as you seem to think. If no >> follow-up question is asked, DWS gets you exactly the same result as MS. > > If no follow-up questions are asked, the dWS is identical to the CS. That > has nothing to do with our discussion. > No, you still have it wrong. If a follow-up question is asked, the DWS ends up in the same position as the MS without follow-up question. >> it is only after a follow-up question, that MS lands you in worse shit >> than DWS. I fail to see why I should suffer such a fate. My neighbour >> who has done exactly the same thing wrong (forgetting his system) will >> get exactly the same bad score as I do - if his opponent does not ask >> the question. Why should I suffer because my opponent is more lazy than > his? > > No, you suffer because your opponent is _active_. Bad that doesn't matter. > It is just a rub-of-the-green. As you would suffer when your opponent > guesses two-way finesse, bids good slam, psyches effectivly, misbids and > fixes you. And please, don't start again that this will result in deluge of > questioning. We're operating under the CS for some years and nothing like > that has happened. > Because the players don't realize yet that they can get a bonus by asking! >> Indeed, and the first piece of UI is unavoidable, but the second one >> isn't. I want to be allowed to avoid giving that UI. > > Selfish of you. And I want to avoid misinforming my opponents. If it is at > cost of giving some UI to my partner (who will correctly bend backwards not > to use it), then so be it. Our bad score will be a reminder to remember our > agreements. And sometimes even we will land on our feet and get a good > score. In a lawful way. > I'm pretty certain the bad score will arrive even before we get to the DWS problem. > Some words of clearance - I'm not saint. But I'd want my opponents to behave > in the describes ways, so I'm obliged to do it first myself. > I am not a saint either. But if my opponents behave as in DWS, I feel I have received all I am entitled to: I will get correct information (by the TD if not from the table), and the UI in one direction will be handled. I don't go asking for UI to be given the other direction as well, and I won't complain that I wasn't informed they were having a misunderstanding, since I am not entitled to that. > >> Of course it is better for him to know we have a bidding >> misunderstanding. But he is not entitled to that knowledge. You cannot >> have it both ways - first saying he is not interested in knowing there >> is one ace, and then that he is is interested in knowing about a >> misunderstanding. > > He is entitled to this knowledge. No he's not. He's entitled to correct information, but not to the existence of a misunderstanding. If you believe this, we have a more fundamental gap between us. And I'm not in a minority of one on this one, Maciej. Herman. From Hermandw at skynet.be Wed Nov 19 09:31:06 2008 From: Hermandw at skynet.be (Herman De Wael) Date: Wed, 19 Nov 2008 09:31:06 +0100 Subject: [blml] A Bruce's comment [SEC=UNOFFICIAL] In-Reply-To: References: Message-ID: <4923CECA.4000708@skynet.be> richard.hills at immi.gov.au wrote: > . > > Richard Hills noted: > >>> .....There is no Law which requires me to lie to the >>> opponents..... > > Herman De Wael asserted: > >> Yes there is. L20F5a. >> >> This law requires you to keep quiet over an untruth >> spoken by partner. >> >> If you don't call that lying then you exhibit a level of >> hypocrisy..... > > Richard Hills refutes: > > Qui tacet consentire videtire ubi loqui debuit ac potuit. > I have very little latin but I have since read David's translation and it seems to say: He who keeps silent when he could have spoken is presumed to consent. Isn't that what I said? What is the difference between lying by omission and lying by speaking out? L20F5a obliges a player to "lie by omission". How then can lying to the next question be any worse? Herman. From grandaeval at tiscali.co.uk Wed Nov 19 10:42:28 2008 From: grandaeval at tiscali.co.uk (Grattan) Date: Wed, 19 Nov 2008 09:42:28 -0000 Subject: [blml] A Bruce's comment [SEC=UNOFFICIAL] References: <49227C4E.3050007@skynet.be><001501c9495e$75116ec0$15844c59@chello.pl> <4922BA63.30707@skynet.be><000701c9498a$9f949880$15844c59@chello.pl> <4922F5CC.1000702@skynet.be><003f01c949f9$c2d51a80$15844c59@chello.pl> <4923CDE9.3020707@skynet.be> Message-ID: <009001c94a2d$783e91c0$0302a8c0@Mildred> Grattan Endicott To: "Bridge Laws Mailing List" Sent: Wednesday, November 19, 2008 8:27 AM Subject: Re: [blml] A Bruce's comment [SEC=UNOFFICIAL] Meanwhile, I have given cogent arguments of my own, and I don't get reasoned replies. Is it any wonder that I believe this situation is not strictly symmetric? > +=+ Since we are not in a period of law review these lengthy theoretical debates are diverting but inconsequential. 'Reasoned replies' are for those who are interested in idle discussion; in the meantime the practical consideration is conformity with Law 72A at the table. ~ Grattan ~ +=+ From grandaeval at tiscali.co.uk Wed Nov 19 10:58:12 2008 From: grandaeval at tiscali.co.uk (Grattan) Date: Wed, 19 Nov 2008 09:58:12 -0000 Subject: [blml] Appeals procedures References: <009d01c948fe$6c45da20$0302a8c0@Mildred><4922051C.9060805@bridgepro.de><001401c94952$84db24b0$0302a8c0@Mildred><002701c94962$0024bc80$15844c59@chello.pl><4922C40D.1050700@bridgepro.de><001a01c9498d$6b34ab40$15844c59@chello.pl><2a1c3a560811181054u665d9e7ej9d6816fca8dc49b0@mail.gmail.com> <004b01c94a02$4e2976a0$15844c59@chello.pl> Message-ID: <009101c94a2d$787ef620$0302a8c0@Mildred> Grattan Endicott To: "Bridge Laws Mailing List" Sent: Wednesday, November 19, 2008 4:50 AM Subject: [blml] Appeals procedures > Wayne, > > >> Has it yet been determined that the slowness of the double transmitted >> through the screen. This needs more than an admission that the double >> was slow. > > I'm unable to establish that. If there was no UI received, no adjustment. > +=+ Of course. And let us remind ourselves of the exact wording of Law 16B3: "......substantial reason to believe that an opponent who had a logical alternative has chosen an action that could have been suggested by..." ~ G ~ +=+ From Hermandw at skynet.be Wed Nov 19 11:07:27 2008 From: Hermandw at skynet.be (Herman De Wael) Date: Wed, 19 Nov 2008 11:07:27 +0100 Subject: [blml] Belgian Appeal Message-ID: <4923E55F.5090108@skynet.be> We've recently had an interesting case in the Belgian Honour division. After three passes, East opens 1NT. South bids 2Di, intending to show a single suit major (Multi). However, in a passed hand they play Dont, not Multi. This is confirmed by the SC. South explains his bid to West as Multi. West doubles. West will later tell the TD that he would also double 2Di if explained as Dont - so he is not damaged at this moment. North explains 2Di as Dont (diamonds and another), and passes. East bids 2He. When the tray returns to South, he remembers that his 2Di was in fact Dont, and he tells West. West now calls the TD, who tells them to continue play. West bids 3Cl and this ends the auction. He makes 11 tricks. After the hand, East/West complain that North has 5 diamonds and should not have passed over 3Cl. The TD and AC award no adjustment. North has no UI, so is not under any restrains. One of the players conducts a poll. None of the polled players would have passed over 3Cl. However, the player in question points out that he has also already passed over 2DiX, something most people would not have done. He presents some arguments in favour of his decision, also based on his partner being a passed hand (excluding 5-5's and so on). The AC ruled that there was no UI. Some people point to the director call, which must have been observed on the other side of the screen. Given that it's never easy to point to an action being suggested by this, should a TD call by the opponent at the other side of the screen (exceedingly loud as well according to the TD) be considered UI, and if so, is it UI from partner or extraneous UI? Herman. From Hermandw at skynet.be Wed Nov 19 11:18:49 2008 From: Hermandw at skynet.be (Herman De Wael) Date: Wed, 19 Nov 2008 11:18:49 +0100 Subject: [blml] Belgian Ruling Message-ID: <4923E809.7090801@skynet.be> Another one from the Honour Division (this time I was the TD). During this competition, matches of 16 boards are played in 4 matches. Every board is duplicated twice, and the boards circle in the open and closed room respectively. During its second outing in the open room, board 10 gets fouled. At the next two tables it is played with the strong hands NS, not EW. These two tables play 3NT+1 and 4S=. For this fourth match the comparison is 620 to both pairs of the same team. Board cancelled, and the Belgian regulations do not award +3 IMPs to both teams. However, in the third match, the result at the other table was 4S-1, due to a combination of an inferior selection of game plan and inspired defence. All the other tables make 4S=. Now here we have a non-offending side, who have scored a good score at the other table, with the result at the other table being fouled. Applying L86D, I felt they deserved 11 IMPs on the board (7/8th of a 12 IMP swing, rounded up), and I awarded them that. However, the other side was non-offending as well. Yet I felt no law compelled me to adjust for them downwards. So in the end the match score was 22-10 (reduced to 21-9 for time penalties, but that's another matter). Do you agree? What sort of regulations should be written to deal with things like this? For that matter, let's suppose that the fouled board is played at more tables than just the two - why should we disregard the score of +630? I did investigate and give the appropriate penalties to the table that fouled the board, of course. Herman. From grandaeval at tiscali.co.uk Wed Nov 19 11:26:07 2008 From: grandaeval at tiscali.co.uk (Grattan) Date: Wed, 19 Nov 2008 10:26:07 -0000 Subject: [blml] Appeals procedures References: <009d01c948fe$6c45da20$0302a8c0@Mildred><4922051C.9060805@bridgepro.de> <001401c94952$84db24b0$0302a8c0@Mildred> <002701c94962$0024bc80$15844c59@chello.pl><4922C40D.1050700@bridgepro.de> <005001c949c9$e99aa4d0$bccfee70$@de> Message-ID: <00a401c94a31$3ed61440$0302a8c0@Mildred> Grattan Endicott >> +=+ Timeo Danaos et dona ferentes. >> I do not suggest that the 3NT bid can be justified. Returned >> to due process I would expect the Director to disallow it and >> for a diamond contract, 4 or 5, to be substituted. I believe only an >> opening club lead beats 5D because of the location of the C9 and >> of the HK. To me that suggests a 12C1(c) adjustment. >> ~ Grattan ~ +=+ >> > > Yes, only an opening club lead (which is certainly the best one anyway) > definitively beats 5D but it nevertheless requires declarer to guess DQ > which is improbable. So even if we use 12C1(c) the weight of 5D= should be > very small. > > Regards > > Maciej > +=+ Each will judge his own relevancies and percentages. Ultimately the decisions may go down quite a different path. This is merely where my off-page thoughts have led. ~ Grattan ~ +=+ From agot at ulb.ac.be Wed Nov 19 11:35:51 2008 From: agot at ulb.ac.be (Alain Gottcheiner) Date: Wed, 19 Nov 2008 11:35:51 +0100 Subject: [blml] Sunday at the club In-Reply-To: <6E89CACF-EC40-4554-90E5-6ABDEABF23EC@starpower.net> References: <4921ADBA.1070708@ulb.ac.be> <6E89CACF-EC40-4554-90E5-6ABDEABF23EC@starpower.net> Message-ID: <4923EC07.2050308@ulb.ac.be> Eric Landau a ?crit : > Ah, semantics... > > You cannot "correct" a statement that was not incorrect to begin with. > > AG : that more or less settles it. But there is another case, which goes the other way. Say your partner explained your 1NT-2S as showing clubs, but it was either minor. Perhaps he was influenced by the fact that your ensuing bidding (or passing, perhaps) showed that you did hold clubs. Now, opponents are rightly informed of what you have in hand. In that case, however, you have to correct partner's explanation and say your 2S bid could have been either minor, because that has a bearing on his rebids and that's information they're entitled to. The logic of a 2NT bid is different. Best regards Alain From nigelguthrie at talktalk.net Wed Nov 19 12:10:41 2008 From: nigelguthrie at talktalk.net (Nigel Guthrie) Date: Wed, 19 Nov 2008 11:10:41 +0000 Subject: [blml] A Bruce's comment [SEC=UNOFFICIAL] In-Reply-To: <4923CECA.4000708@skynet.be> References: <4923CECA.4000708@skynet.be> Message-ID: <4923F431.2030707@talktalk.net> [Richard Hills] Qui tacet consentire videtire ubi loqui debuit ac potuit. [David Burn] For the benefit of any laymen who may have wandered in, the Latin phrase "qui tacet consentire videtur" means "he who keeps silent is seen to consent". The rest means "when [literally: where] he ought to have, and could have, spoken". [Herman De Wael] I have very little latin but I have since read David's translation and it seems to say: "He who keeps silent when he could have spoken is presumed to consent." Isn't that what I said? What is the difference between lying by omission and lying by speaking out? L20F5a obliges a player to "lie by omission". How then can lying to the next question be any worse? [Maciej] .. would be right if you take into consideration only the first part of this clever Latinian phrase. But as the following proviso is added, Herman's argument falls a bit short of being cogent. [Nige1] Maciej is right: The rules of Bridge tell you not to speak until later. Anyway the rules of Bridge encourage specific "lies". For example, players happily psyche and false-card. From agot at ulb.ac.be Wed Nov 19 12:55:51 2008 From: agot at ulb.ac.be (Alain Gottcheiner) Date: Wed, 19 Nov 2008 12:55:51 +0100 Subject: [blml] The Nebraskan issue [SEC=UNOFFICIAL] In-Reply-To: References: Message-ID: <4923FEC7.8040102@ulb.ac.be> richard.hills at immi.gov.au a ?crit : > > No, the trans-Atlantic differences between colour labels > of political parties of the left and right is due to > "path dependence". An initial semi-random setting of a > standard (British party colours defined decades ago, > American party colours standardised as recently as at > the 2000 election) followed by a lock-in of that standard. > AG : not quite random. Red is the color of left-wingers since the Commune, all around the World, as can be seen on umpteen flags, from China to Angola. US seem to diverge alone. And 'reds under the bed' doesn't meazn phobia of Republicans. > > (b) most players do give full disclosure of their mutual > partnership understandings, instead of the slightly more > effective strategy of disclosing only that which will not > restrict partner's choice amongst logical alternatives. > > AG : do you mean that those who act Lawfully do it only by chance ? From agot at ulb.ac.be Wed Nov 19 13:02:29 2008 From: agot at ulb.ac.be (Alain Gottcheiner) Date: Wed, 19 Nov 2008 13:02:29 +0100 Subject: [blml] Belgian Appeal In-Reply-To: <4923E55F.5090108@skynet.be> References: <4923E55F.5090108@skynet.be> Message-ID: <49240055.3040705@ulb.ac.be> Herman De Wael a ?crit : > We've recently had an interesting case in the Belgian Honour division. > > After three passes, East opens 1NT. > South bids 2Di, intending to show a single suit major (Multi). > However, in a passed hand they play Dont, not Multi. This is confirmed > by the SC. > South explains his bid to West as Multi. AG : from what follows, it means "any long suit " ? Not the classical Multi from "Multi-Landy". > West doubles. West will later > tell the TD that he would also double 2Di if explained as Dont - so he > is not damaged at this moment. > North explains 2Di as Dont (diamonds and another), and passes. East bids > 2He. > When the tray returns to South, he remembers that his 2Di was in fact > Dont, and he tells West. > West now calls the TD, who tells them to continue play. > West bids 3Cl and this ends the auction. He makes 11 tricks. > > After the hand, East/West complain that North has 5 diamonds and should > not have passed over 3Cl. > > The TD and AC award no adjustment. North has no UI, so is not under any > restrains. > > One of the players conducts a poll. None of the polled players would > have passed over 3Cl. However, the player in question points out that he > has also already passed over 2DiX, something most people would not have > done. He presents some arguments in favour of his decision, also based > on his partner being a passed hand (excluding 5-5's and so on). > > The AC ruled that there was no UI. > AG : there was AI that a wheel had come loose, since 2D denies clubs under their agreements (DONT). In such circumstances, many players have a tendency to pass before things go really bad, so I'm a bit surprised that nobody passed 3C. In fact, they would probably have done so at the table, and this shows that even polling may give wrong answers. Let's try to remember this. > Some people point to the director call, which must have been observed on > the other side of the screen. Given that it's never easy to point to an > action being suggested by this, should a TD call by the opponent at the > other side of the screen (exceedingly loud as well according to the TD) > be considered UI, and if so, is it UI from partner or extraneous UI? > AG : it is UI more or less equal to AI, so I'd let the score stand. Best regards Alain From karel at esatclear.ie Wed Nov 19 14:38:09 2008 From: karel at esatclear.ie (Karel) Date: Wed, 19 Nov 2008 13:38:09 +0000 Subject: [blml] How should this have been handled ? In-Reply-To: References: Message-ID: > > I would throw out Board 3 because of the redouble. The redoubler has AQT 63 > KJT2 J985, and partner opened in third seat, so there is no reason to expect 4D > to make, particularly once it gets doubled. And there is no point in the > redouble; 4Dx will be a top if it makes, but the redouble could turn -200 which > is duplicated at other tables into -400 which isn't. > +++ wouldnt be convinced here. Pairs is abit wierd at times. I've been doubled in 2C which I know is making but which is useless even with 2 overtricks if game is on. So redoubling is necessary. Similarly, if we bid to 3H - the assumption is we are making 3H which is 140. So if the opps bid 4D - you are almost forced to double regardless of whether you think its making or not as -100 is a zero anyway. So if you expect most of the field to bid 4D over 3H and that most of the field is forced to double, then redouble is not so strange or illogical. As for 3rd seat opener, so what - lots of pairs dont open light 3rd seat - I dont think you can automatically say the opps have their values and we arent making this. We have the balance of points, an extra diamond, an extra spade, a spade less a heart more and 4D makes. Redouble going for the 100% is not imo in the same category as the 2 previous telephone numbers. K. From Hermandw at skynet.be Wed Nov 19 15:52:15 2008 From: Hermandw at skynet.be (Herman De Wael) Date: Wed, 19 Nov 2008 15:52:15 +0100 Subject: [blml] A Bruce's comment [SEC=UNOFFICIAL] In-Reply-To: <4923F431.2030707@talktalk.net> References: <4923CECA.4000708@skynet.be> <4923F431.2030707@talktalk.net> Message-ID: <4924281F.6000700@skynet.be> Nigel Guthrie wrote: > [Richard Hills] Qui tacet consentire videtire ubi loqui debuit ac > potuit. > > [David Burn] For the benefit of any laymen who may have wandered in, > the Latin phrase "qui tacet consentire videtur" means "he who keeps > silent is seen to consent". The rest means "when [literally: where] > he ought to have, and could have, spoken". > > [Herman De Wael] I have very little latin but I have since read > David's translation and it seems to say: "He who keeps silent when he > could have spoken is presumed to consent." Isn't that what I said? > What is the difference between lying by omission and lying by > speaking out? L20F5a obliges a player to "lie by omission". How then > can lying to the next question be any worse? > > [Maciej] .. would be right if you take into consideration only the > first part of this clever Latinian phrase. But as the following > proviso is added, Herman's argument falls a bit short of being > cogent. > > [Nige1] Maciej is right: The rules of Bridge tell you not to speak > until later. Anyway the rules of Bridge encourage specific "lies". > For example, players happily psyche and false-card. > Yet Maciej was one of those who uses the DWS are lying argument. I would like to see that argument being dropped. There is nothing valid in it. Herman. From Hermandw at skynet.be Wed Nov 19 15:53:43 2008 From: Hermandw at skynet.be (Herman De Wael) Date: Wed, 19 Nov 2008 15:53:43 +0100 Subject: [blml] Belgian Appeal In-Reply-To: <49240055.3040705@ulb.ac.be> References: <4923E55F.5090108@skynet.be> <49240055.3040705@ulb.ac.be> Message-ID: <49242877.5060301@skynet.be> Alain Gottcheiner wrote: >> South explains his bid to West as Multi. > AG : from what follows, it means "any long suit " ? Not the classical > Multi from "Multi-Landy". > No it did mean "any major" as in Multi-Landy. Herman. From darkbystry at wp.pl Thu Nov 20 00:18:49 2008 From: darkbystry at wp.pl (Bystry) Date: Thu, 20 Nov 2008 00:18:49 +0100 Subject: [blml] A Bruce's comment [SEC=UNOFFICIAL] References: <49227C4E.3050007@skynet.be><001501c9495e$75116ec0$15844c59@chello.pl> <4922BA63.30707@skynet.be><000701c9498a$9f949880$15844c59@chello.pl> <4922F5CC.1000702@skynet.be><003f01c949f9$c2d51a80$15844c59@chello.pl> <4923CDE9.3020707@skynet.be> Message-ID: <006b01c94a9d$2e40f660$15844c59@chello.pl> Herman, allow me to respond to your all messages here. > So in fact you want L20F5a changed, but since that cannot be arranged, > you are going for the second best thing? Well, in that case, there is no > need talking to you about equality of OS against askers and non-askers. Yes, exactly. And so, if you've realised this, you should also perfectly understand why the dWS is not for me. > OK, so let's just agree to disagree - but meanwhile you're in the > majority and are happy there? Nice one! I don't care whether I'm in the majority or not. I didn't care anytime in my whole life. > Yes, and you are of the super-masochistic kind who would even like to > see L20F5a removed. Tell me Maciej, what do you do when your partner > misexplains? If you are at all consistent, you would correct it! I cannot. I won't deliberately break the Laws, even when I think they suck. > OK granted. So you happily create UI, do you? Well, that's a masochistic > way of playing bridge but your opponents will certainly be happy. Not happily. But it is my negligence not to know or remember my system. And I'm causing troubles for everyone - me, the opponents, the TDs. So I should be the only responsible one. And all what results should fall on my back. Here I'm in the minority and I still don't care. Yes, that's the way of playing Bridge I would hope everybody chooses. [snip about questions] > Because the players don't realize yet that they can get a bonus by asking! That's my last answer on this topic. Imagine mid-level partnership. You play against them 100 boards. How many real misunderstandings will they have? Let's assume 5. Now you propose to ask constantly about every call to try to catch them? And how many times will your questions result in UI rulings against your side? 15? 20? Herman, you are ignoring the statistics, the logic and the sanity of all the players. > I am not a saint either. But if my opponents behave as in DWS, I feel I > have received all I am entitled to: I will get correct information (by > the TD if not from the table), and the UI in one direction will be > handled. I don't go asking for UI to be given the other direction as > well, and I won't complain that I wasn't informed they were having a > misunderstanding, since I am not entitled to that. I won't complain either. And I'm not opting for instant correction of MI only to punish the OS. In that case I could easier opt for mandatory heavy PP for the OS. That's dumb. No, my reasons are as follow: a) that gives the best chance to play the board normally and with full information b) that is the best choice for the NOS and troubles them in the smallest way c) that gives the best chance to avoid TD intervention And all that reasons are fundamental in my view of Bridge. Now to your other mails. The lawmakers have told you that your view is flawed. That they intented "...in any manner..." as "...in any manner except obeying other Law...". And you perfectly know that was their intention. But you choose to ignore this and deliberately break the Laws. My position is different. I think that the MI should be corrected instantly. But I know that the Laws state the opposite. So I'm bound to obey them. Nothing more is to be said. You've asked why don't I try to change the Laws. It's simple - I don't have a power to do it. Nowadays the authorities' priority is to draw new players to the game. I applaud this goal. But not the way of reaching it. And this way is unfortunately to change the Laws to comfort the players who don't want to pay for their mistakes. The beginners do not like the TD's to take the tricks away from them. They dislike being penalised. So the Laws are changed to do the "equity", not to deter the law-breakers from infracting. The advanced players won't leave Bridge because they are addicted to it. So they are less important. Sad but true. And the UI/MI dilemma. UI is not per se harmful. Thinking is necessary and it transmits UI. Questioning is necessary and it transmits UI. But as long as the recipient doesn't use it, all is ok, board is played normally. If he actually uses it, then the result will be often adjusted but the NOS aren't at all bothered. They can go to the other table, forget about the whole thing and get the result later from the TD. MI is different. Yes, sometimes it isn't harmful. But it always causes the board to be played in some strange way. And it's sometimes hard to tell what would have happened in the different scenario. The non-offenders have to be interrogated about their methods, about what would they have done without the MI. They are often disbelieved, called "litigators", the TD's are playing the boards instead of them. They get only their statistical outcome for all this mess. Isn't it right to direct all the troubles to one side, the offending one? For me it's so obvious, so fair, so just, that I have hard times to understand the adverse opinions. Therefore don't be suprised I can't agree with you, our philosophical approach differs too much. > Herman. Regards Maciej From rfrick at rfrick.info Thu Nov 20 00:27:06 2008 From: rfrick at rfrick.info (Robert Frick) Date: Wed, 19 Nov 2008 18:27:06 -0500 Subject: [blml] Belgian Appeal In-Reply-To: <4923E55F.5090108@skynet.be> References: <4923E55F.5090108@skynet.be> Message-ID: On Wed, 19 Nov 2008 05:07:27 -0500, Herman De Wael wrote: > We've recently had an interesting case in the Belgian Honour division. > > After three passes, East opens 1NT. > South bids 2Di, intending to show a single suit major (Multi). > However, in a passed hand they play Dont, not Multi. This is confirmed > by the SC. > South explains his bid to West as Multi. West doubles. West will later > tell the TD that he would also double 2Di if explained as Dont - so he > is not damaged at this moment. > North explains 2Di as Dont (diamonds and another), and passes. East bids > 2He. > When the tray returns to South, he remembers that his 2Di was in fact > Dont, and he tells West. > West now calls the TD, who tells them to continue play. > West bids 3Cl and this ends the auction. He makes 11 tricks. > > After the hand, East/West complain that North has 5 diamonds and should > not have passed over 3Cl. > > The TD and AC award no adjustment. North has no UI, so is not under any > restrains. > > One of the players conducts a poll. None of the polled players would > have passed over 3Cl. However, the player in question points out that he > has also already passed over 2DiX, something most people would not have > done. He presents some arguments in favour of his decision, also based > on his partner being a passed hand (excluding 5-5's and so on). > > The AC ruled that there was no UI. > Some people point to the director call, which must have been observed on > the other side of the screen. Given that it's never easy to point to an > action being suggested by this, should a TD call by the opponent at the > other side of the screen (exceedingly loud as well according to the TD) > be considered UI, and if so, is it UI from partner or extraneous UI? I think the director call is part of the proper procedures of the game, hence AI -- L16A1(c). So North can use it to infer that partner might have misunderstood a bid. From richard.hills at immi.gov.au Thu Nov 20 00:48:23 2008 From: richard.hills at immi.gov.au (richard.hills at immi.gov.au) Date: Thu, 20 Nov 2008 10:48:23 +1100 Subject: [blml] Belgian Ruling [SEC=UNOFFICIAL] In-Reply-To: <4923E809.7090801@skynet.be> Message-ID: Herman De Wael: >Every board is duplicated twice, and the boards circle in the >open and closed room respectively. Richard Hills; In my opinion, poor logistics, since that means a fouled board is unrecoverable. The Aussie Interstate Teams has an open room and a closed room, but boards circulate by row, instead of by room. Opposing teams sit in the same row in the open and closed rooms. This means that if a board is fouled in the Seniors match between NSW and VIC, a result on the board might still be obtained in the Seniors match between WA and ACT because both the Closed and Open tables of the WA-ACT match played the board after it was fouled, hence in Law 87A "identical form". Herman De Wael: >.....Applying L86D..... >.....However, the other side was non-offending as well..... >.....Do you agree?..... Law 86D - Result Obtained at Other Table In team play when the Director awards an adjusted score (excluding any award that ensues from application of Law 6D2), and a result has been obtained* between the same contestants at another table, the Director may assign an adjusted score in IMPs or total points (and should do so when that result appears favourable to the non-offending side). * if commenced between the same two contestants at another table the board may be completed Richard Hills: A legal Director's ruling -- Law 86D says "the Director may assign" -- but not a compulsory Director's ruling. Law 86D refers to "should do so" only when the non-offending side did well at the other table, but here one of the two non-offending sides did poorly at the other table. One must remember the presumed reason Law 86D was added to the 2007 Lawbook. A number of players (including, Bobby Wolff alleged, a former ACBL President) had, after getting a bad teams result, an incentive to foul that board before it reached the other table so that it would get thrown out. As with the Law 23 "could have been aware" it was simpler to remove such incentives. But here there was no incentive, since the offending side was playing in another match. Ergo, if I had been the Director and had been notified of the fouled board before score comparison I would have ordered the affected teams to play a substitute board. Law 86C. Otherwise, in accordance with Belgian regulations, the Director's discretionary "may" power under Law 86D, and Law 87, as Director I would choose to cancel the fouled boards. What's the problem? Best wishes Richard James Hills Recruitment Section, Level 3 Blue, workstation 15 (first on left) Department of Immigration and Citizenship Telephone: 02 6223 8453 Email: richard.hills at immi.gov.au -------------------------------------------------------------------- Important Notice: If you have received this email by mistake, please advise the sender and delete the message and attachments immediately. This email, including attachments, may contain confidential, sensitive, legally privileged and/or copyright information. Any review, retransmission, dissemination or other use of this information by persons or entities other than the intended recipient is prohibited. DIAC respects your privacy and has obligations under the Privacy Act 1988. The official departmental privacy policy can be viewed on the department's website at www.immi.gov.au. See: http://www.immi.gov.au/functional/privacy.htm --------------------------------------------------------------------- From mfrench1 at san.rr.com Thu Nov 20 01:03:53 2008 From: mfrench1 at san.rr.com (Marvin L French) Date: Wed, 19 Nov 2008 16:03:53 -0800 Subject: [blml] Unsubscribing Message-ID: <0283F2CBF17C484AA60BB082EBD00F3E@MARVLAPTOP> Leaving town for the Boston NABC, followed by a trip to Hawaii. I'm unsubscribing because I can't face the backlog when I return. If anyone has something to say to me, my e-mail address is available. I shall return. Marv Marvin L French San Diego, CA www.marvinfrench.com From rfrick at rfrick.info Thu Nov 20 04:49:16 2008 From: rfrick at rfrick.info (Robert Frick) Date: Wed, 19 Nov 2008 22:49:16 -0500 Subject: [blml] laissez faire In-Reply-To: <000001c94a0b$1cc70330$56550990$@com> References: <49227C4E.3050007@skynet.be> <000001c94a0b$1cc70330$56550990$@com> Message-ID: wherein I attempt to be practical. 1. Reprise A player learns that his partner has misunderstood his bid. Partner is wrong. Partner then makes a bid. The intended meaning of this bid does not correspond to what it should mean if partner had correctly understood the player's bid. Option 1: The player can explain the bid as partner intended. Option 2: The player can explain the bid as if had partner understood the auction. Both involve telling the opponents the relevant partnership agreements, understandings, etc. Alerts have the same problem as explanations. 2. Current Affairs Players do both. Presumably....some are trying to follow laws, and which they do depends on which law they focus on; some are trying to be nice to the opponents; some are trying to do whatever advantages them the most. Some probably choose whichever choice involves not alerting (so that opponents do not ask what partner's bid means and the worst of the dilemma is avoided). Directors do not punish either choice. I would guess that there has never been a procedural penalty for this choice. I would be even more confident that no director has ever rectified for the noncreation of UI when a player choose Option 1. 3. What would be Wise Leadership? You can imagine that the laws are the laws, the WBFLC is in charge, and the players should follow the laws and the WBFLC no matter what. You can imagine that it does not matter whether the players respect the laws or the WBFLC, or that trust does not matter. You can imagine that it does not matter how often the player has had to not follow a law in the past because the law was impractical or whatever. I hope you can see that it does matter. IMO, it is mistake of leadership to try to lead if people are not going to follow you. (Unless that direction is so important as to require desperate measures. But the current state of bridge is okay.) I understand that it would be anathematic to the Laws people to leave the situation with players receiving no guidance. But I also think that attempting to promulgate/force players to choose Option 2 is a poor idea. Option 2 can be characerized as deliberately giving a misleading explanation. It can also be described as deliberately creating UI. The WBFLC will lose respect and trust if it tries to force players to deliberately give misleading explanations which deliberately create UI. It doesn't sound-bite well. And it won't feel right to many players. If there is no penalty for choosing option 1, then Probst cheats (and most unaware players) can choose Option 1 when it seems to be to their advantage. But the WBFLC will lose if it tries to encourage directors to give procedural penalties for choosing option 1. They will lose if they encourage directors to rectify for the noncreation of UI. In fact, directors won't like Option 2 simply because it creates twice as much UI for them to deal with. A recent WBFLC opinion addressed this situation. They opined that the partnership is not required at any time or under any circumstances or in response to any question to divulge the partnership agreements, understandings, and experience that would lead to an understanding of the player's intended meaning. I believe that was a step in the wrong direction. (Note that they probably did this in response to my poll on blml, and there were very few respondents there who liked playing bridge this way and the majority of respondents did not interpret the laws this way.) Just trying to help! Bob From darkbystry at wp.pl Thu Nov 20 06:41:59 2008 From: darkbystry at wp.pl (Bystry) Date: Thu, 20 Nov 2008 06:41:59 +0100 Subject: [blml] laissez faire References: <49227C4E.3050007@skynet.be><000001c94a0b$1cc70330$56550990$@com> Message-ID: <000d01c94ad2$b54bc880$15844c59@chello.pl> Bob, [snip] > Option 1: The player can explain the bid as partner intended. > > Option 2: The player can explain the bid as if had partner understood the > auction. > > Both involve telling the opponents the relevant partnership agreements, > understandings, etc. Alerts have the same problem as explanations. You see, it's not always true. You are under the false impression caused by specifically selected examples like 4NT Blackwood/minors. Here you have knowledge what would partner's bids mean according to your system. But imagine you've never played Multi and don't know following sequences. Now your partner alerts your 2D as Multi (because he plays like that with someone else) and what should you answer if the opponents question his 2NT bid? You can say "I don't know" or "undiscussed" but this will in effect wake partner up and give him UI, so you're back at Option 2. And there are many situations where bidding positions are less clear and you have no idea what does your partner intend. But to follow Option 2 is quite easy as long as you remember your agreements. Option 2 works nearly always, Option 1 requires specific conditions. > 2. Current Affairs > Players do both. Presumably....some are trying to follow laws, and which > they do depends on which law they focus on; That's not true. Excuse me, I don't know what is your nationality, but ask 10 random mid-level players in your country do they have any idea about Law 20F1 and 20F5(a) (and as well other points in Law 20). Ask them if they know anything about dWS or CS. You'll see that the players have no idea about this Law. In Poland only small minority read and understood TFLB. Most of the players follow only some virtual concept I would call "practice". They know that after the hesitations their bids can be cancelled. They know they are obliged to describe their bids according to their system. Some may know what should be done after the LOOT. But that is all. And in most cases it is enough. But were we to go into details things would start to be worse. Here's something I send privately to Jeff Easterton earlier: "There was a famous case in Poland - one of the top women players bid something after her partner's hesitation. It was a good bid, I'd bid it myself. But now the TD was called, polled some players, established that pass was LA (we had an informal 25% border) and imposed it on her. The AC has supported his verdict and now the madness began. Articles in bridge newspapers, accusations, endless discussions. The player was suspended for publicly offending the members of the AC. And so on. And all this was caused by the general ignorance of Laws. But is it only players' guilt? I don't think so. There is a need for education and simplification." I can only add that this player is former World Mixed Pairs Champion. > some are trying to be nice to > the opponents; some are trying to do whatever advantages them the most. > Some probably choose whichever choice involves not alerting (so that > opponents do not ask what partner's bid means and the worst of the dilemma > is avoided). Yes, such behaviors happen. But most often at the low level. > Directors do not punish either choice. I would guess that there has never > been a procedural penalty for this choice. I would be even more confident > that no director has ever rectified for the noncreation of UI when a > player choose Option 1. The directors generally do not give PPs. And I don't think it would be appropriate to give one to the player who was simply ignorant of the correct procedure. But I wouldn't stop for one second before giving e.g. Herman a huge PP, because he is aware of his obligations and deliberately breaks the Laws. > 3. What would be Wise Leadership? [snip] > I hope you can see that it does matter. IMO, it is mistake of leadership > to try to lead if people are not going to follow you. (Unless that > direction is so important as to require desperate measures. But the > current state of bridge is okay.) Agreed. > I understand that it would be anathematic to the Laws people to leave the > situation with players receiving no guidance. But I also think that > attempting to promulgate/force players to choose Option 2 is a poor idea. > > Option 2 can be characerized as deliberately giving a misleading > explanation. No. You are under the same misconception as Herman. The solution is easy - promote the correct notion, that _all_, to which the opponents are entitled are your partnership's agreements. And from my experience I can tell you that most mid-level plus players in Poland understand this. Only low-level players have similar ideas that the contents of the hand should be told. Most of the NBO's have their bulletins (and Net sites) send to every registrated member. Is there a problem to use one page every time and to print most important Laws players should know? Education is necessary, otherwise the players won't even know they're breaking the Laws. > It can also be described as deliberately creating UI. The > WBFLC will lose respect and trust if it tries to force players to > deliberately give misleading explanations which deliberately create UI. It > doesn't sound-bite well. And it won't feel right to many players. First look above. Then think about this procedure: a) you teach people what the essence of Bridge is. That they shouldn't expect to get their opponent's hands' contents because that is something their opponents don't know either. They're entitled only to this information, which is available from the systemic meanings of chosen calls - that's the same information their opponents have. And that is written in TFLB. b) now, starting from a), you can tell them they shouldn't ask questions like "what does he have?" and that they shouldn't expect answers to such questions. You should explain to them that the only correct form is "what, according to your partnership's methods, does your partner's bidding mean?". You can now make things simpler and point out that other questions may be used (to save time etc) but the answer will always be directed to this proper question. And that is written in TFLB. c) having b) the next step is easy - you tell them they should explain in a way described in b). Only answer to proper question, not to the meaning of badly formed questions. And that is written in TFLB. d) and only now you should go further to MI, UI etc Second procedure: a') you teach them to answer the questions not precising what exactly they should do b') UI,MI etc Procedure 2 results in all the things you've described. Procedure 1 will be easy to understand even for beginners and they will follow it. So the problem is in inadequate education, not in the choice between dWS and CS. > If there is no penalty for choosing option 1, then Probst cheats (and most > unaware players) can choose Option 1 when it seems to be to their > advantage. But the WBFLC will lose if it tries to encourage directors to > give procedural penalties for choosing option 1. Why? If somebody repeatedly and deliberately breaks the Laws he is certainly aware he will be penalised for that. I do not advocate giving PPs to first- or second-time offenders. But repeated infractions should be dealt harshly with. > They will lose if they > encourage directors to rectify for the noncreation of UI. In fact, > directors won't like Option 2 simply because it creates twice as much UI > for them to deal with. Yes, that is unfortunately true. But the WBFLC _makes_ some Laws more complicated and therefore gives more work to the TDs. Think about all those claim, correction, revoke Laws. Some of them require nearly mindreading, complicated judgment and so on. It would be much easier for the TDs to give e.g 2 tricks to the NOS or not to allow for the corrections. The WBFLC does so because the Laws are for players and for the game. The players should have a fine game not TDs less work. So, if the WBFLC can make some Laws "nice" for the OS, why shouldn't they analogously make some Laws "nice" for the NOS? And Option 2 is better for the NOS which I've proven in my discussion with Herman. > A recent WBFLC opinion addressed this situation. They opined that the > partnership is not required at any time or under any circumstances or in > response to any question to divulge the partnership agreements, > understandings, and experience that would lead to an understanding of the > player's intended meaning. I believe that was a step in the wrong > direction. (Note that they probably did this in response to my poll on > blml, and there were very few respondents there who liked playing bridge > this way and the majority of respondents did not interpret the laws this > way.) Ok, your opinion is as well as mine. But isn't it better to try to educate players than to change the Laws to comfort those who are lazy and obstructive? Remember, almost every change will result in satisfying a group of people and doing the opposite to the other group. Which group is more worth caring for - the misexplainers or their innocent opponents? > Just trying to help! > > Bob Regards Maciej From Hermandw at skynet.be Thu Nov 20 09:28:23 2008 From: Hermandw at skynet.be (Herman De Wael) Date: Thu, 20 Nov 2008 09:28:23 +0100 Subject: [blml] Belgian Ruling [SEC=UNOFFICIAL] In-Reply-To: References: Message-ID: <49251FA7.8000908@skynet.be> richard.hills at immi.gov.au wrote: > Herman De Wael: > >> Every board is duplicated twice, and the boards circle in the >> open and closed room respectively. > > Richard Hills; > > In my opinion, poor logistics, since that means a fouled board > is unrecoverable. > I know this, but other reasons mean this way is chosen. And fouled boards are very rare anyway... Herman. From sater at xs4all.nl Thu Nov 20 09:35:08 2008 From: sater at xs4all.nl (Hans van Staveren) Date: Thu, 20 Nov 2008 09:35:08 +0100 Subject: [blml] Belgian Ruling [SEC=UNOFFICIAL] In-Reply-To: <49251FA7.8000908@skynet.be> References: <49251FA7.8000908@skynet.be> Message-ID: <00e401c94aea$e76aab20$b6400160$@nl> Actually in NL we play the same. Of course you run the risk of fouled boards, but as Herman says, it is rare. I have not read Herman's post very clearly, but I think he let (part of) the result count for one party because of a good result other table. As Richard said, that rule is not for this case, although you could legally use it. We used to have a rule in NL(not any more) that if a board was cancelled at one table in the match for such a reason, we allowed the result on the other table to be scored against the Butler datum score, which is supposed to roughly give the value of the board. I actually came up with this rule when I was the head of the national competition(AKL) but in retrospect I doubt I should have done it. Hans van Staveren From john at asimere.com Thu Nov 20 16:23:58 2008 From: john at asimere.com (John (MadDog) Probst) Date: Thu, 20 Nov 2008 15:23:58 -0000 Subject: [blml] Belgian Ruling [SEC=UNOFFICIAL] References: <49251FA7.8000908@skynet.be> <00e401c94aea$e76aab20$b6400160$@nl> Message-ID: <5C9E46AF1154485EA1CF97F3A8BFCB10@JOHN> ----- Original Message ----- From: "Hans van Staveren" To: "'Bridge Laws Mailing List'" Sent: Thursday, November 20, 2008 8:35 AM Subject: Re: [blml] Belgian Ruling [SEC=UNOFFICIAL] > Actually in NL we play the same. Of course you run the risk of fouled > boards, but as Herman says, it is rare. At brighton we have 250 tables and 65 sets of boards. It's not a problem. John > > I have not read Herman's post very clearly, but I think he let (part of) > the > result count for one party because of a good result other table. > As Richard said, that rule is not for this case, although you could > legally > use it. > > We used to have a rule in NL(not any more) that if a board was cancelled > at > one table in the match for such a reason, we allowed the result on the > other > table to be scored against the Butler datum score, which is supposed to > roughly give the value of the board. I actually came up with this rule > when > I was the head of the national competition(AKL) but in retrospect I doubt > I > should have done it. > > Hans van Staveren > > > > _______________________________________________ > blml mailing list > blml at amsterdamned.org > http://www.amsterdamned.org/mailman/listinfo/blml From nigelguthrie at talktalk.net Thu Nov 20 20:58:51 2008 From: nigelguthrie at talktalk.net (Nigel Guthrie) Date: Thu, 20 Nov 2008 19:58:51 +0000 Subject: [blml] laissez faire In-Reply-To: <000d01c94ad2$b54bc880$15844c59@chello.pl> References: <49227C4E.3050007@skynet.be><000001c94a0b$1cc70330$56550990$@com> <000d01c94ad2$b54bc880$15844c59@chello.pl> Message-ID: <4925C17B.1010706@talktalk.net> [Maciej] The directors generally do not give PPs. And I don't think it would be appropriate to give one to the player who was simply ignorant of the correct procedure. But I wouldn't stop for one second before giving e.g. Herman a huge PP, because he is aware of his obligations and deliberately breaks the Laws. [Nige1] Herman is an *international director*, who publicly advertises that he breaks Bridge law, deliberately and repeatedly :) Cruel Maciej would impose a PP on Herman :( That seems way over the top when neither his NBO nor the WBF take any action :) From richard.hills at immi.gov.au Thu Nov 20 22:54:12 2008 From: richard.hills at immi.gov.au (richard.hills at immi.gov.au) Date: Fri, 21 Nov 2008 08:54:12 +1100 Subject: [blml] Belgian Ruling [SEC=UNOFFICIAL] In-Reply-To: <5C9E46AF1154485EA1CF97F3A8BFCB10@JOHN> Message-ID: Hans van Staveren: >>Actually in NL we play the same. Of course you >>run the risk of fouled boards, but as Herman >>says, it is rare. John Probst: >At brighton we have 250 tables and 65 sets of >boards. It's not a problem. Richard Hills: The qualifying Swiss of the Aussie National Open Teams also has circa 250 tables playing 14 matches of 20 boards each. Although each round's boards are duplicated across the field, every six teams (who are drawn against each other) share only one set of 20 boards. Thus if the Tournament Organizer erred in duplicating a particular board, an "identical form" result is automatically obtained in the three affected matches. Likewise, a board fouled by a player has a chance of being partially recoverable by being played post-foul in an "identical form" in one or two other matches. Sure fouled boards are rare, But why not arrange logistics to minimise rarely occurring adverse outcomes? What's the problem? Best wishes Richard Hills, Aqua 5, workstation 51 Telephone: 02 6223 8453 Email: richard.hills at immi.gov.au Recruitment Section & DIAC Social Club movie tickets -------------------------------------------------------------------- Important Notice: If you have received this email by mistake, please advise the sender and delete the message and attachments immediately. This email, including attachments, may contain confidential, sensitive, legally privileged and/or copyright information. Any review, retransmission, dissemination or other use of this information by persons or entities other than the intended recipient is prohibited. DIAC respects your privacy and has obligations under the Privacy Act 1988. The official departmental privacy policy can be viewed on the department's website at www.immi.gov.au. See: http://www.immi.gov.au/functional/privacy.htm --------------------------------------------------------------------- From nigelguthrie at talktalk.net Thu Nov 20 23:32:48 2008 From: nigelguthrie at talktalk.net (Nigel Guthrie) Date: Thu, 20 Nov 2008 22:32:48 +0000 Subject: [blml] laissez faire In-Reply-To: <000d01c94ad2$b54bc880$15844c59@chello.pl> References: <49227C4E.3050007@skynet.be><000001c94a0b$1cc70330$56550990$@com> <000d01c94ad2$b54bc880$15844c59@chello.pl> Message-ID: <4925E590.6090609@talktalk.net> [Maciej] No. You are under the same misconception as Herman. The solution is easy - promote the correct notion, that _all_, to which the opponents are entitled are your partnership's agreements. And from my experience I can tell you that most mid-level plus players in Poland understand this. Only low-level players have similar ideas that the contents of the hand should be told. [Nigel] Until I read BLML, I hadn't encountered DWS. One reason is that, in the UK, partnerships are required to complete two identical, standard, comprehensive system-cards, for perusal by opponents. An opponent would be puzzled if your spoken explanation differed from what you wrote on your card. By 2018, I hope that the WBFLC will consider reducing confusion elsewhere by - - clarifying existing disclosure laws. - completing the law-book with, for example, default regulations on all aspects of disclosure, such as system-cards. - including belated corrections, in-place, in the law-book, rather than in obscure minutes. From richard.hills at immi.gov.au Fri Nov 21 02:57:27 2008 From: richard.hills at immi.gov.au (richard.hills at immi.gov.au) Date: Fri, 21 Nov 2008 12:57:27 +1100 Subject: [blml] The Nebraskan issue [SEC=UNOFFICIAL] In-Reply-To: <4923FEC7.8040102@ulb.ac.be> Message-ID: Monsignor Ronald Knox (1888-1957): There once was a man who said "God Must think it exceedingly odd If he finds that this tree Continues to be When there's no one about in the Quad." Richard Hills: >> ..... due to "path dependence". An initial semi-random >>setting of a standard ..... followed by a lock-in of that >>standard. Alain Gottcheiner: >AG : do you mean that those who act Lawfully do it only by >chance ? Richard Hills: No, I am saying that the exceedingly odd rules of bridge arose by Darwinian chance, not by Intelligent Design. The classic case was the introduction of the concept of vulnerability. As Harold Vanderbilt was beta-testing contract bridge in 1925, a fleeting acquaintance (so fleeting an acquaintance that Vanderbilt forgot her name) suggested that vulnerability be added to the rules. Had she not been there, contract bridge would have evolved from a different starting point. Nigel Guthrie frequently suggests that the Lawbook should be simplified. Abolition of vulnerability in 2018 would simplify Laws 2, 77 and 87. It would also simplify the game of Duplicate Bridge itself. But lock-in means that such a major simplifying change will not happen. Especially when one considers that many like complicated assessments of how vulnerability affects their matchpoint and imp decisions in somewhat different ways. Best wishes Richard Hills, Aqua 5, workstation 51 Telephone: 02 6223 8453 Email: richard.hills at immi.gov.au Recruitment Section & DIAC Social Club movie tickets -------------------------------------------------------------------- Important Notice: If you have received this email by mistake, please advise the sender and delete the message and attachments immediately. This email, including attachments, may contain confidential, sensitive, legally privileged and/or copyright information. Any review, retransmission, dissemination or other use of this information by persons or entities other than the intended recipient is prohibited. DIAC respects your privacy and has obligations under the Privacy Act 1988. The official departmental privacy policy can be viewed on the department's website at www.immi.gov.au. See: http://www.immi.gov.au/functional/privacy.htm --------------------------------------------------------------------- From darkbystry at wp.pl Fri Nov 21 03:25:33 2008 From: darkbystry at wp.pl (Bystry) Date: Fri, 21 Nov 2008 03:25:33 +0100 Subject: [blml] laissez faire References: <49227C4E.3050007@skynet.be><000001c94a0b$1cc70330$56550990$@com> <000d01c94ad2$b54bc880$15844c59@chello.pl> <4925E590.6090609@talktalk.net> Message-ID: <002d01c94b80$6eca0440$15844c59@chello.pl> Hi, > [Maciej] > No. You are under the same misconception as Herman. The solution is easy - promote the correct notion, that _all_, to which the opponents are entitled are your partnership's agreements. And from my experience I can tell you that most mid-level plus players in Poland understand this. Only low-level players have similar ideas that the contents of the hand should be told. > > [Nigel] > Until I read BLML, I hadn't encountered DWS. > > One reason is that, in the UK, partnerships are required to complete two identical, standard, comprehensive system-cards, for perusal by opponents. An opponent would be puzzled if your spoken explanation differed from what you wrote on your card. In Poland the CCs are even involuntary. Or being more precise - in our System Policy it is stated in one place they are mandatory, and in the second place that the pair having no CC can play only Green Systems (which in Poland contain natural, Polish Club and Strefa [mix of Polish Club and natural], all the most popular systems). The only real reason for having the CC is a regulation that without it all your mistakes and psyches would be treated automatically as MI. But to satisfy this point a Minicard is enough (it contains only general approach, opening bids, simple overcalls and leads/discards, all without following sequences). But despite that, the dWS is nearly unknown. The systems are quite uniform in Poland (most are using PC or Strefa) and widely known. And if sometimes there is a mixup most players can't resist themselves and make it known to the whole table. Very often they break 20F5 unintentionally. So they are de facto closer to my position of changing this Law to allow for instant MI correction. Regards Maciej From darkbystry at wp.pl Fri Nov 21 03:33:37 2008 From: darkbystry at wp.pl (Bystry) Date: Fri, 21 Nov 2008 03:33:37 +0100 Subject: [blml] laissez faire References: <49227C4E.3050007@skynet.be><000001c94a0b$1cc70330$56550990$@com> <000d01c94ad2$b54bc880$15844c59@chello.pl> <4925C17B.1010706@talktalk.net> Message-ID: <000701c94b81$8f468e40$15844c59@chello.pl> Hi, > [Nige1] > Herman is an *international director*, who publicly advertises that he breaks Bridge law, deliberately and repeatedly :) > Cruel Maciej would impose a PP on Herman :( > That seems way over the top when neither his NBO nor the WBF take any action :) You know, Herman pretends to be a leader, a guru of all the dWS rebels. So what could be better for him than to become a martyr, PPed by an opressive tyrant from the CS? ;-) Regards Maciej From wjburrows at gmail.com Fri Nov 21 08:49:20 2008 From: wjburrows at gmail.com (Wayne Burrows) Date: Fri, 21 Nov 2008 20:49:20 +1300 Subject: [blml] What is demonstably suggested by this hesitation? Message-ID: <2a1c3a560811202349w557f1237gc36fec9b1879fde9@mail.gmail.com> This situation was conveyed to me by email. I don't know the identities of anyone involved except the emailer and I assume his role was on the appeal committee. It involves an unusual raise to 5S by a limited hand after partner made a slow jump raise to game. Is this a case of if it hesitates shoot or could a slow 4S suggest something other than you have extra values. The uncontested auction was: 1C 1S 2S 4S! 5S 6S I was not given the vulnerability. Without being given the hands I have these questions: 1. Does a slow 4S necessarily suggest a strong hand rather than a hand that might have stretched in a 3S/4S decision (or some other game try)? 2. What sort of hand would you expect for this 5S or any other move after this 4S sign-off? In other words how good would opener's hand need to be to justify a move over 4S? -- Wayne Burrows Palmerston North New Zealand From Hermandw at skynet.be Fri Nov 21 09:03:03 2008 From: Hermandw at skynet.be (Herman De Wael) Date: Fri, 21 Nov 2008 09:03:03 +0100 Subject: [blml] laissez faire In-Reply-To: <4925C17B.1010706@talktalk.net> References: <49227C4E.3050007@skynet.be><000001c94a0b$1cc70330$56550990$@com> <000d01c94ad2$b54bc880$15844c59@chello.pl> <4925C17B.1010706@talktalk.net> Message-ID: <49266B37.90306@skynet.be> Nigel Guthrie wrote: > [Maciej] > The directors generally do not give PPs. And I don't think it would be > appropriate to give one to the player who was simply ignorant of the correct > procedure. But I wouldn't stop for one second before giving e.g. Herman a > huge PP, because he is aware of his obligations and deliberately breaks the > Laws. > > [Nige1] > Herman is an *international director*, who publicly advertises that he breaks Bridge law, deliberately and repeatedly :) > Cruel Maciej would impose a PP on Herman :( > That seems way over the top when neither his NBO nor the WBF take any action :) > It is certainly over the top if this is the only penalty about this law. It means that others get away with beneficial actions, whereas I can't. That must strike even Richard as unfair. This is one of my strongest reasons why this system? cannot work. You cannot have a law which obliges someone to give UI to his partner, because you cannot have any penalty that correctly handles the situations where he forgets to do so. Herman. From Hermandw at skynet.be Fri Nov 21 09:07:43 2008 From: Hermandw at skynet.be (Herman De Wael) Date: Fri, 21 Nov 2008 09:07:43 +0100 Subject: [blml] Belgian Ruling [SEC=UNOFFICIAL] In-Reply-To: References: Message-ID: <49266C4F.8000009@skynet.be> richard.hills at immi.gov.au wrote: > Hans van Staveren: > > > The qualifying Swiss of the Aussie National Open > Teams also has circa 250 tables playing 14 matches > of 20 boards each. Although each round's boards > are duplicated across the field, every six teams > (who are drawn against each other) share only one > set of 20 boards. > > Thus if the Tournament Organizer erred in > duplicating a particular board, an "identical > form" result is automatically obtained in the > three affected matches. > > Likewise, a board fouled by a player has a chance > of being partially recoverable by being played > post-foul in an "identical form" in one or two > other matches. > > Sure fouled boards are rare, But why not arrange > logistics to minimise rarely occurring adverse > outcomes? What's the problem? > The problem is that your system requires more boards (20 for 12 tables) than ours (16 for 4 tables) thus requiring more board moving. And while duplicating errors are very rare, you cannot eliminate handling errors - so your system will have more chance of a problem than ours (as the boards are handled more times). But did we ever say that _was_ a problem? Herman. From darkbystry at wp.pl Fri Nov 21 10:24:15 2008 From: darkbystry at wp.pl (Bystry) Date: Fri, 21 Nov 2008 10:24:15 +0100 Subject: [blml] laissez faire References: <49227C4E.3050007@skynet.be><000001c94a0b$1cc70330$56550990$@com> <000d01c94ad2$b54bc880$15844c59@chello.pl><4925C17B.1010706@talktalk.net> <49266B37.90306@skynet.be> Message-ID: <000f01c94bba$ecc985c0$15844c59@chello.pl> >> [Nige1] >> Herman is an *international director*, who publicly advertises that he breaks Bridge law, deliberately and repeatedly :) >> Cruel Maciej would impose a PP on Herman :( >> That seems way over the top when neither his NBO nor the WBF take any action :) > It is certainly over the top if this is the only penalty about this law. > It means that others get away with beneficial actions, whereas I can't. > That must strike even Richard as unfair. Why? You are the Guru. You should be proud to suffer for your followers :-) > This is one of my strongest reasons why this system? cannot work. You > cannot have a law which obliges someone to give UI to his partner, > because you cannot have any penalty that correctly handles the > situations where he forgets to do so. I'm shocked that an experienced, international TD claims that there is no penalty for deliberately breaking the Laws. Small hint: 72B1, 90A or even 91A. Or this experienced, international TD claims that any player can "accidentally", but having full knowledge about TFLB, apply the dWS? > Herman. Regards Maciej From darkbystry at wp.pl Fri Nov 21 10:34:03 2008 From: darkbystry at wp.pl (Bystry) Date: Fri, 21 Nov 2008 10:34:03 +0100 Subject: [blml] What is demonstably suggested by this hesitation? References: <2a1c3a560811202349w557f1237gc36fec9b1879fde9@mail.gmail.com> Message-ID: <001501c94bbc$4ab9df80$15844c59@chello.pl> Hi, > This situation was conveyed to me by email. I don't know the > identities of anyone involved except the emailer and I assume his role > was on the appeal committee. It involves an unusual raise to 5S by a > limited hand after partner made a slow jump raise to game. > > Is this a case of if it hesitates shoot or could a slow 4S suggest > something other than you have extra values. For me slow 4S doesn't suggest anything but it's quite possible that bidder's partner tried to use UI (risking misguessing or knowing something about his CHO tendencies). > The uncontested auction was: > > 1C 1S > 2S 4S! > 5S 6S > > I was not given the vulnerability. > > Without being given the hands I have these questions: > > 1. Does a slow 4S necessarily suggest a strong hand rather than a > hand that might have stretched in a 3S/4S decision (or some other game > try)? I was always taught that such situations do not fall under "demonstrably suggested". It's like 1NT-2NT(invitional, hesitation). > 2. What sort of hand would you expect for this 5S or any other move > after this 4S sign-off? In the place of the 4S bidder I would assume that 2S was a mistake, otherwise I absolutely should instantly call for an ambulance with psychiatrists. > In other words how good would opener's hand need to be to justify a > move over 4S? Too good to bid 2S. > Wayne Burrows > Palmerston North > New Zealand Regards Maciej From nigelguthrie at talktalk.net Fri Nov 21 11:44:20 2008 From: nigelguthrie at talktalk.net (Nigel Guthrie) Date: Fri, 21 Nov 2008 10:44:20 +0000 Subject: [blml] The Nebraskan issue [SEC=UNOFFICIAL] In-Reply-To: References: Message-ID: <49269104.6010904@talktalk.net> [richard.hills] Nigel Guthrie frequently suggests that the Lawbook should be simplified. Abolition of vulnerability in 2018 would simplify Laws 2, 77 and 87. It would also simplify the game of Duplicate Bridge itself. But lock-in means that such a major simplifying change will not happen. Especially when one considers that many like complicated assessments of how vulnerability affects their matchpoint and imp decisions in somewhat different ways. [Nige1] I don't like the kind of simplification that Richard suggests because it would radically change the nature of the game. Thus, I objected to the recent change in the scoring for doubled non-vulnerable undertricks. This made the game less skilful, interesting and exciting by discouraging sacrificing. It reduced the scope for judgement, especially at the slam-level, when the likes of Meckstroth would often sacrifice profitably. Hence poor Walruses who picked up enormous hands would be deprived of what they imagined to be their God-given right, No. The rule simplifications that I advocate keep the *spirit^ of the game. From grandaeval at tiscali.co.uk Fri Nov 21 11:43:12 2008 From: grandaeval at tiscali.co.uk (Grattan) Date: Fri, 21 Nov 2008 10:43:12 -0000 Subject: [blml] laissez faire References: <49227C4E.3050007@skynet.be><000001c94a0b$1cc70330$56550990$@com> <000d01c94ad2$b54bc880$15844c59@chello.pl><4925C17B.1010706@talktalk.net> <000701c94b81$8f468e40$15844c59@chello.pl> Message-ID: <001601c94bc6$94165870$0302a8c0@Mildred> Grattan Endicott To: "Bridge Laws Mailing List" Sent: Friday, November 21, 2008 2:33 AM Subject: [blml] laissez faire > Hi, > >> [Nige1] >> Herman is an *international director*, who publicly advertises that he > breaks Bridge law, deliberately and repeatedly :) >> Cruel Maciej would impose a PP on Herman :( >> That seems way over the top when neither his NBO nor the WBF take any > action :) > > You know, Herman pretends to be a leader, a guru of all the dWS rebels. So > what could be better for him than to become a martyr, PPed by an opressive > tyrant from the CS? > +=+ *international director*. Please amplify. I have not seen Herman as a director. ~ G ~ +=+ From nigelguthrie at talktalk.net Fri Nov 21 12:19:05 2008 From: nigelguthrie at talktalk.net (Nigel Guthrie) Date: Fri, 21 Nov 2008 11:19:05 +0000 Subject: [blml] laissez faire In-Reply-To: <001601c94bc6$94165870$0302a8c0@Mildred> References: <49227C4E.3050007@skynet.be><000001c94a0b$1cc70330$56550990$@com> <000d01c94ad2$b54bc880$15844c59@chello.pl><4925C17B.1010706@talktalk.net> <000701c94b81$8f468e40$15844c59@chello.pl> <001601c94bc6$94165870$0302a8c0@Mildred> Message-ID: <49269929.5080900@talktalk.net> [Grattan Endicott] +=+ *international director*. Please amplify. I have not seen Herman as a director. ~ G ~ +=+ [Nige1] Grattan is in a better position to be aware of Herman's credentials. Many of us ordinary players haven't yet enjoyed the privilege of meeting Herman; whereas WBF members serve on Appeals committees, for which Herman acts as secretary and editor; and they meet Herman socially at WBF events. From Hermandw at skynet.be Fri Nov 21 15:13:27 2008 From: Hermandw at skynet.be (Herman De Wael) Date: Fri, 21 Nov 2008 15:13:27 +0100 Subject: [blml] laissez faire In-Reply-To: <001601c94bc6$94165870$0302a8c0@Mildred> References: <49227C4E.3050007@skynet.be><000001c94a0b$1cc70330$56550990$@com> <000d01c94ad2$b54bc880$15844c59@chello.pl><4925C17B.1010706@talktalk.net> <000701c94b81$8f468e40$15844c59@chello.pl> <001601c94bc6$94165870$0302a8c0@Mildred> Message-ID: <4926C207.6020208@skynet.be> Grattan wrote: > >> > +=+ *international director*. Please amplify. I have not seen Herman > as a director. ~ G ~ +=+ > I have the official qualification of NBO TD issued by the EBL. You were there in Torino when I got that title - twice in the past few years. I have directed internationally, and even in EBL events. True, the most recent of those was "only" a youth event, Prague 2004. As you know, I usually have other functions in EBL events. Herman. From Hermandw at skynet.be Fri Nov 21 15:24:22 2008 From: Hermandw at skynet.be (Herman De Wael) Date: Fri, 21 Nov 2008 15:24:22 +0100 Subject: [blml] laissez faire In-Reply-To: <000f01c94bba$ecc985c0$15844c59@chello.pl> References: <49227C4E.3050007@skynet.be><000001c94a0b$1cc70330$56550990$@com> <000d01c94ad2$b54bc880$15844c59@chello.pl><4925C17B.1010706@talktalk.net> <49266B37.90306@skynet.be> <000f01c94bba$ecc985c0$15844c59@chello.pl> Message-ID: <4926C496.6080904@skynet.be> Bystry wrote: >>> [Nige1] >>> Herman is an *international director*, who publicly advertises that he > breaks Bridge law, deliberately and repeatedly :) >>> Cruel Maciej would impose a PP on Herman :( >>> That seems way over the top when neither his NBO nor the WBF take any > action :) > >> It is certainly over the top if this is the only penalty about this law. >> It means that others get away with beneficial actions, whereas I can't. >> That must strike even Richard as unfair. > > Why? You are the Guru. You should be proud to suffer for your followers :-) > Indeed - make me a martyr. I would enjoy that. >> This is one of my strongest reasons why this system? cannot work. You >> cannot have a law which obliges someone to give UI to his partner, >> because you cannot have any penalty that correctly handles the >> situations where he forgets to do so. > > I'm shocked that an experienced, international TD claims that there is no > penalty for deliberately breaking the Laws. Small hint: 72B1, 90A or even > 91A. > But that is not my point. My point is that if I make the action that is beneficial to me, I will get punished by a PP, yet I get to keep my benefit. Whereas some other poor guy, who acts in the same manner, will not get a PP, since he doesn't know better. And he too gets to keep his benefit. Which means that we ARE treated differently, but also that your rule is no good. What will Probst cheat do? Take the action and the benefit. What will John Probst then do? Say "I don't know that you did not know better, so I'll judge as if you did" and issue the same penalty he would to me. End solution: both me and the Probst cheat have the benefit of our actions. If you really want a law that obliges people to give UI to their partner, you must also be prepared to have penalties that mean people stick to that law; And that will mean creating a sort of construction that no TD will be able to pronounce. "You have broken the law which requires you to give UI to your partner, so now we will award you a score as if you had given such UI, which means that your partner will not be allowed to take that particular action." With that particular action one taken by this player in the absence of UI. Try explaining that there are logical alternatives to an action that a player actually took without it being suggested to him! > Or this experienced, international TD claims that any player can > "accidentally", but having full knowledge about TFLB, apply the dWS? > My aim is to have the DWS actions being accepted, and dealt with with the laws at hand, being the UI laws (in one direction only) and the MI laws (both on the first and second MI). There is (IMO) nothing unfair about this construction. All the infractions that I do will be dealt with - but not the silly sounding infractions of "failure to give UI" and "failure to tell opponents something they are not entitled to". >> Herman. > > Regards > > Maciej > Herman. From jfusselman at gmail.com Fri Nov 21 16:18:30 2008 From: jfusselman at gmail.com (Jerry Fusselman) Date: Fri, 21 Nov 2008 09:18:30 -0600 Subject: [blml] laissez faire In-Reply-To: References: <49227C4E.3050007@skynet.be> <000001c94a0b$1cc70330$56550990$@com> Message-ID: <2b1e598b0811210718w4425c26fv4747eac0dca478fe@mail.gmail.com> Robert Frick wrote: > > A recent WBFLC opinion addressed this situation. They opined that the > partnership is not required at any time or under any circumstances or in > response to any question to divulge the partnership agreements, > understandings, and experience that would lead to an understanding of the > player's intended meaning. I believe that was a step in the wrong > direction. > I don't see why you call it a step in the wrong direction, exactly. To me it sounds a little more like vacationing on the wrong planet. I seem to have missed this statement by the WBFLC. I wonder if someone would kindly repeat it here. Jerry Fusselman From rfrick at rfrick.info Fri Nov 21 16:31:59 2008 From: rfrick at rfrick.info (Robert Frick) Date: Fri, 21 Nov 2008 10:31:59 -0500 Subject: [blml] The Nebraskan issue (really about the methods of change) In-Reply-To: References: Message-ID: > No, I am saying that the exceedingly odd rules of bridge > arose by Darwinian chance, not by Intelligent Design. > > The classic case was the introduction of the concept of > vulnerability. As Harold Vanderbilt was beta-testing > contract bridge in 1925, a fleeting acquaintance (so > fleeting an acquaintance that Vanderbilt forgot her name) > suggested that vulnerability be added to the rules. Had she > not been there, contract bridge would have evolved from a > different starting point. To tie together threads and make a general point. Maciej suggested that we allow instant correction of MI. To me, this is a great idea. I would love to try playing bridge this way and see if it works. Because it might be a better way to play bridge. Or it might not. Or, third possibility, it might not work as is but it might be changeable into something that does work. Or it might just suggest a way to change the current system that I otherwise wouldn't have thought of. This might not be exactly Darwinian evolution, but it would be close. There would be change, people would adopt the things that made bridge most enjoyable, and bridge would become more enjoyable. But in bridge, we try to adopt the locked-in-place approach that Richard has been describing. No one can do anything different. Instead, everyone has to follow the rules. Then when it comes time to make new rules, the rule makers are never going to put into the rules something new and untested that might completely backfire. (Sometimes they make very tiny changes, and of course sometimes those changes do backfire.) It is not quite that bleak. The ACBL doesn't consider appeals from clubs. So clubs can do whatever they want. There is one club here that doesn't accept director calls. You all probably think they are a bunch of hooligans. I think they are making an interesting and potentially useful experiment, in the best Darwinian sense. Also, the rules allow some flexibility. Usually the WBF reaction to any discovered ambiguity in the laws is to stamp it out. (Is it legal to leave out the final contract when using bidding cards?) But they have given directors flexibility on when to give procedure penalities (almost no one does that here in ACBL-land). (And I can ask a player to leave the table and have his partner explain his bid! Thank you new laws, even though I think you were just catering to on-line play, which is a source of change in bridge rules.) From agot at ulb.ac.be Fri Nov 21 16:41:38 2008 From: agot at ulb.ac.be (Alain Gottcheiner) Date: Fri, 21 Nov 2008 16:41:38 +0100 Subject: [blml] laissez faire In-Reply-To: <4926C496.6080904@skynet.be> References: <49227C4E.3050007@skynet.be><000001c94a0b$1cc70330$56550990$@com> <000d01c94ad2$b54bc880$15844c59@chello.pl><4925C17B.1010706@talktalk.net> <49266B37.90306@skynet.be> <000f01c94bba$ecc985c0$15844c59@chello.pl> <4926C496.6080904@skynet.be> Message-ID: <4926D6B2.4000101@ulb.ac.be> Herman De Wael a ?crit : > My aim is to have the DWS actions being accepted, and dealt with with > the laws at hand AG : I don't think we can achieve this. What we need is a new law telling that, if circumsatnces force a player to shoot himself in the foot (whatever he will do, he'll risk a penalty), he may choose the weapon. Similar cases already occur : if you drop two cards, one of which a honor, you're allowed to play the honor, in order to suffer less severe penalties (mpc in lieu of Mpc) ; however, you always can choose to play the low card if you think that penalties from the Mpc will be less tragic than having to play a honor on the present trick. In the same way, a player should, in dWS-sensitive cases, be allowed to choose between limitations due to UI, and possible score rectification due to MI. It won't usually be aggravated by the second MI, but the bad cases are when you misinterpret partner's action (e.g. you create new UI because you think he misunderstood the sequence, but he merely forgot to alert, or you didn't see him alerting). So it's "take your risk", and if you lose you lose. But, once again, this will need to be accepted as a general principle before dWS has any hope. What worries me is that if a player says "following partner's explanation, I thought in all good faith that I was wrong, so I explained the ensuing bidding according to this new interpretation", this attempt to explain your true agreements will be undistinguishable from conscious dWS use in case partner -and you- happen to be wrong. (of course, there won't be any problem in case you're right). Best regards Alain From grandaeval at tiscali.co.uk Fri Nov 21 16:41:39 2008 From: grandaeval at tiscali.co.uk (Grattan) Date: Fri, 21 Nov 2008 15:41:39 -0000 Subject: [blml] laissez faire References: <49227C4E.3050007@skynet.be><000001c94a0b$1cc70330$56550990$@com> <000d01c94ad2$b54bc880$15844c59@chello.pl><4925C17B.1010706@talktalk.net> <000701c94b81$8f468e40$15844c59@chello.pl><001601c94bc6$94165870$0302a8c0@Mildred> <4926C207.6020208@skynet.be> Message-ID: <001401c94bef$a9241170$0302a8c0@Mildred> Grattan Endicott To: "Bridge Laws Mailing List" Sent: Friday, November 21, 2008 2:13 PM Subject: Re: [blml] laissez faire > I have the official qualification of NBO TD issued by the EBL. You were > there in Torino when I got that title - twice in the past few years. > I have directed internationally, and even in EBL events. > True, the most recent of those was "only" a youth event, Prague 2004. > As you know, I usually have other functions in EBL events. > +=+ What I can attest to is that Herman does not raise his idiosyncratic ideas in the course of his functions at EBL tournaments. ~ Grattan ~ +=+ From rfrick at rfrick.info Fri Nov 21 16:50:07 2008 From: rfrick at rfrick.info (Robert Frick) Date: Fri, 21 Nov 2008 10:50:07 -0500 Subject: [blml] laissez faire In-Reply-To: <2b1e598b0811210718w4425c26fv4747eac0dca478fe@mail.gmail.com> References: <49227C4E.3050007@skynet.be> <000001c94a0b$1cc70330$56550990$@com> <2b1e598b0811210718w4425c26fv4747eac0dca478fe@mail.gmail.com> Message-ID: On Fri, 21 Nov 2008 10:18:30 -0500, Jerry Fusselman wrote: > Robert Frick wrote: >> >> A recent WBFLC opinion addressed this situation. They opined that the >> partnership is not required at any time or under any circumstances or in >> response to any question to divulge the partnership agreements, >> understandings, and experience that would lead to an understanding of >> the >> player's intended meaning. I believe that was a step in the wrong >> direction. >> > > I don't see why you call it a step in the wrong direction, exactly. > To me it sounds a little more like vacationing on the wrong planet. I > seem to have missed this statement by the WBFLC. I wonder if someone > would kindly repeat it here I may have paraphrased. (But context reveals that the above quote was talking about the situation, for example, where you correctly bid 4NT for the minors, partner explains it as being Blackwood and responds accordingly, and there are existing partnership agreements which would explain the meaning of his Blackwood call.) "20F1 defines the manner in which, during the auction and play, a player may request and receive an explanation of the opponents? prior auction. At this time he is entitled to an explanation only of calls actually made, relevant available alternative calls not made, and any partnership understanding as to inferences from the choice of action among the foregoing. (An ?alternative? call is not the same call with another meaning ? for example, if the reply to an opponent is that ?5D shows diamonds preference?, any reply to a further question ?what would it mean if 4NT were Blackwood ?? is given voluntarily and not as a requirement of Law 20F1.)" From Hermandw at skynet.be Fri Nov 21 19:17:31 2008 From: Hermandw at skynet.be (Herman De Wael) Date: Fri, 21 Nov 2008 19:17:31 +0100 Subject: [blml] The Nebraskan issue (really about the methods of change) In-Reply-To: References: Message-ID: <4926FB3B.7060408@skynet.be> Robert Frick wrote: > > > To tie together threads and make a general point. > > Maciej suggested that we allow instant correction of MI. To me, this is a > great idea. I would love to try playing bridge this way and see if it > works. > > Because it might be a better way to play bridge. Or it might not. Or, > third possibility, it might not work as is but it might be changeable into > something that does work. Or it might just suggest a way to change the > current system that I otherwise wouldn't have thought of. > I have also already thought of this, as it is one way of solving the inconsistency of the MS. I even suggested this to Grattan in a private conversation in Antalya. However, I don't think it would work. Just off the top of my head, I have four objections: a) it would be difficult to re-educate the players (although that should not be too big an objection) b) it would create a huge difference between F2F and screens bridge, since this would mean that in F2F, all misunderstandings would become known to opponents, something which is impossible with screens. Also the on-line version would drift further away from F2F, since there it is even possible to have wrong explanations without partner even realizing, creating even less UI than in F2F at present. c) it would have the same problem as with the MS interpretation: what do we do with a player who refuses (or forgets) to correct the MI? He will create for himself an advantage (partner has no UI - opponents no knowledge of the misunderstanding) which is very difficult to quantify and punish. I do realize that this is a lesser problem if it is always doen, lesser than in current MS, but still. d) you still don't tell us what a player should do who is very/somewhat/mildly certain that his partner has given a correct explanation - should he reveal that he misbid too? Herman. From darkbystry at wp.pl Sat Nov 22 03:11:28 2008 From: darkbystry at wp.pl (Bystry) Date: Sat, 22 Nov 2008 03:11:28 +0100 Subject: [blml] laissez faire References: <49227C4E.3050007@skynet.be><000001c94a0b$1cc70330$56550990$@com> <000d01c94ad2$b54bc880$15844c59@chello.pl><4925C17B.1010706@talktalk.net> <49266B37.90306@skynet.be><000f01c94bba$ecc985c0$15844c59@chello.pl> <4926C496.6080904@skynet.be> Message-ID: <003901c94c47$a1541140$15844c59@chello.pl> Herman, >> I'm shocked that an experienced, international TD claims that there is no >> penalty for deliberately breaking the Laws. Small hint: 72B1, 90A or even >> 91A. > But that is not my point. > My point is that if I make the action that is beneficial to me, I will > get punished by a PP, yet I get to keep my benefit. "Benefit" should be judged globally. I can choose a size of PP, so you will always be in worse position than by obeying the Laws. Remember, deliberately breaking the Laws to hurt the other contestants and to try to profit from that is one of the gravest possible offences. It can easily end in front of L&E Comittee. > Whereas some other poor guy, who acts in the same manner, will not get a > PP, since he doesn't know better. And he too gets to keep his benefit. Yes, in a sense he'll keep his benefit. But this will happen only one time, after then he'll fall into the second category. And of course if his result is better than A- I'll assign an artificial adjusted score. I don't like such things, but here is a perfect situation to apply them. Remember, there is no problem with the OS because they can get a PP and it makes it a losing strategy to cheat. The problem is with the NOS who sometimes will be hurt. So I'll give them A+ or I can even use 12A1. > Which means that we ARE treated differently, but also that your rule is > no good. In many situations people are treated differently. The PPs are left to the TD discretion. And everybody accepts that they are sometimes given to the experienced players who should have known better but not to the novices. And of course repeated serious infractions are different than one-time infraction caused by lack of knowledge. > What will Probst cheat do? Take the action and the benefit. > What will John Probst then do? Say "I don't know that you did not know > better, so I'll judge as if you did" and issue the same penalty he would > to me. End solution: both me and the Probst cheat have the benefit of > our actions. Why do you think that it is something terrible if Probst cheat escapes once with A- instead of worse score? Next time he'll get a 200% of top PP and this will show him that breaking the Laws _is not_ a profitable strategy. > If you really want a law that obliges people to give UI to their > partner, you must also be prepared to have penalties that mean people > stick to that law; And that will mean creating a sort of construction > that no TD will be able to pronounce. "You have broken the law which > requires you to give UI to your partner, so now we will award you a > score as if you had given such UI, which means that your partner will > not be allowed to take that particular action." With that particular > action one taken by this player in the absence of UI. Try explaining > that there are logical alternatives to an action that a player actually > took without it being suggested to him! Of course you are right. Such a method would be hard to implement. So I simply state that there is no way to obtain a real table result and I rule under 12A2 and 12A1 adding an appropriate PP for the OS. And I can guarantee you that after some of such rulings the news will spread and nobody would want to risk severe sanctions. >> Or this experienced, international TD claims that any player can >> "accidentally", but having full knowledge about TFLB, apply the dWS? > My aim is to have the DWS actions being accepted, and dealt with with > the laws at hand, being the UI laws (in one direction only) and the MI > laws (both on the first and second MI). I'm aware of it. > There is (IMO) nothing unfair about this construction. All the > infractions that I do will be dealt with - but not the silly sounding > infractions of "failure to give UI" and "failure to tell opponents > something they are not entitled to". Herman, your approach may seem to be logical in theory, but in practice it is unworkable. a) very often you'll be unable to read partner's mind. So you'll end giving him UI. b) very often you'll be so suprised by your partner's wrong explanantion that it'll be practically impossible not to get it known for the whole table. Look, partner's wrong explanation, 3-5 seconds, and the opponents ask you what does partner's call mean. Do you really believe that such a player will be able to keep his face straight and instantly give an explanation which fits his partner's delusion? Absurd. I'm looking at such behaviors every day. Your approach would be profitable for veteran, hardskin liars, only they can really fit into the dWS. c) imagine the feelings of the NOS when after the auction you correct all the explanations (yours and partner's). They were building an image of the deal during the auction and now all perishes and they're forced to do it again. What would you tell them as TD when they claim damage because this confussion resulted in their misdefence? Are you ready to give them redress only for their state of mind? And leave that nonsensical notion of "failure to give UI". In Bridge UI is transmitted all the time. Sometimes innocent people end up being disadvantaged because they simply needed to know the meaning of the auction or had a difficult decision to make and hesitated. Why don't you worry about them? Why all your efforts concetrate on the law-infractors who can avoid all the troubles by simply remembering their agreements? The dWS would show to people that it is profitable to lie in order to save one's own skin. Go and tell that to those Bridge teachers who encourage young people to come to our game. Continue you efforts to induce people to think it's good for Bridge. I'm definitely out of it. > Herman. Regards Maciej From dalburn at btopenworld.com Sat Nov 22 05:26:13 2008 From: dalburn at btopenworld.com (David Burn) Date: Sat, 22 Nov 2008 04:26:13 -0000 Subject: [blml] laissez faire In-Reply-To: <003901c94c47$a1541140$15844c59@chello.pl> References: <49227C4E.3050007@skynet.be><000001c94a0b$1cc70330$56550990$@com> <000d01c94ad2$b54bc880$15844c59@chello.pl><4925C17B.1010706@talktalk.net> <49266B37.90306@skynet.be><000f01c94bba$ecc985c0$15844c59@chello.pl> <4926C496.6080904@skynet.be> <003901c94c47$a1541140$15844c59@chello.pl> Message-ID: <005201c94c5a$746f5230$5d4df690$@com> [Bystry] Remember, deliberately breaking the Laws to hurt the other contestants and to try to profit from that is one of the gravest possible offences. [DALB] This is grossly unfair. Herman's position has always been that where partner has misexplained your call and you now have to explain his, you are forced to break one Law or another. If you explain correctly, you break Law 20 (because you indicate prematurely that a mistake has been made). If you misexplain, you break Law 40 (because you do not disclose your actual agreements). In a position where you must break one of the Laws, it is not a grave offence to do so. Of course you could avoid being in such a position by remembering your system, but as Eric and others have cogently argued, it is not a grave offence not to know your system - if it were, bridge would cease to be played in clubs the world over. Herman has consistently asserted that if you do have to break some Law, it is preferable to break Law 40 rather than Law 20. Others have asserted the contrary position, and the WBFLC has now made a decision that to Nigel Guthrie's disgust was not the main item on the BBC news bulletins for the next several days. I have no doubt that in his role as a Director, Herman will abide by the WBFLC's decision as conscientiously and as meticulously as he does everything else. That he has argued a perfectly tenable legal point up to the stage where the WBFLC has had to make a final determination adds to rather than detracting from his credentials both as a Director and as a human being (these categories may not always overlap, but in Herman's case they do). Herman may continue to believe that the WBFLC has made the wrong decision, and may continue so to argue. From time to time others may weary of such arguments, but please let us have no more talk of lying or cheating, and please let us have no more attempts to discredit Herman's considerable and well-merited standing in the bridge world. David Burn London, England From darkbystry at wp.pl Sat Nov 22 06:39:27 2008 From: darkbystry at wp.pl (Bystry) Date: Sat, 22 Nov 2008 06:39:27 +0100 Subject: [blml] laissez faire References: <49227C4E.3050007@skynet.be><000001c94a0b$1cc70330$56550990$@com> <000d01c94ad2$b54bc880$15844c59@chello.pl><4925C17B.1010706@talktalk.net> <49266B37.90306@skynet.be><000f01c94bba$ecc985c0$15844c59@chello.pl> <4926C496.6080904@skynet.be><003901c94c47$a1541140$15844c59@chello.pl> <005201c94c5a$746f5230$5d4df690$@com> Message-ID: <001d01c94c64$af88f9c0$15844c59@chello.pl> Hi David, > [Bystry] > > Remember, deliberately breaking the Laws to hurt the other contestants and > to try to profit from that is one of the gravest possible offences. > > [DALB] > > This is grossly unfair. > > Herman's position has always been that where partner has misexplained your > call and you now have to explain his, you are forced to break one Law or > another. If you explain correctly, you break Law 20 (because you indicate > prematurely that a mistake has been made). If you misexplain, you break Law > 40 (because you do not disclose your actual agreements). > > In a position where you must break one of the Laws, it is not a grave > offence to do so. Of course you could avoid being in such a position by > remembering your system, but as Eric and others have cogently argued, it is > not a grave offence not to know your system - if it were, bridge would cease > to be played in clubs the world over. > > Herman has consistently asserted that if you do have to break some Law, it > is preferable to break Law 40 rather than Law 20. Others have asserted the > contrary position, and the WBFLC has now made a decision that to Nigel > Guthrie's disgust was not the main item on the BBC news bulletins for the > next several days. > > I have no doubt that in his role as a Director, Herman will abide by the > WBFLC's decision as conscientiously and as meticulously as he does > everything else. That he has argued a perfectly tenable legal point up to > the stage where the WBFLC has had to make a final determination adds to > rather than detracting from his credentials both as a Director and as a > human being (these categories may not always overlap, but in Herman's case > they do). > > Herman may continue to believe that the WBFLC has made the wrong decision, > and may continue so to argue. From time to time others may weary of such > arguments, but please let us have no more talk of lying or cheating, and > please let us have no more attempts to discredit Herman's considerable and > well-merited standing in the bridge world. > > David Burn > London, England Thank you for your cogent and well-stated contribution. But let me explain my position and some points in which you've misunderstood it. First - I've never accused Herman of lying and cheating. Mere fact of promoting his dWS here on BLML is nothing bad per se (although it can be harmful). Only his one's own, Herman's words caused me to write this statement you are referring to. Here is one quotation: [Herman De Wael] >And sorry, but I have a Gandhi-like approach to >unjust laws - I won't follow them. and there were more I can find for you if you wish so. Herman perfectly knows how should Law 20F5 be understood. He was told it many, many times. So if he really still wants to disobey Law 40 as a player, I have every right to call that lying and cheating. But if those are only his fantasies and he won't break the Laws, then he won't be a liar and cheat. It is all conditional and theoretical. Next point. I may be wrong but it's hard for me to swallow that any average player would understand words from Law 20F5 "...in any manner..." in any other way than "...in any manner except when another Law forces you to do it". For me it's an obvious and general rule. And the lawmakers though so too, remember Grattan's statements, and as you can see, those words *were not changed* in current Laws, despite all the efforts from Herman. And finally, nearly all arguments put forward by Herman are concentrating around one thing - how can the offenders save their skins. He doesn't think about the non-offenders and the game as a whole, this unfortunate loophole in the Laws is only a tool he uses to promote his ideas. So I have every right to criticize his stated motivations and ideas. But it still doesn't mean that I downgrade Herman as a person. I'm never evaulating anybody until I get to know him personally. But I'm in full right to evaulate his ideas, credos, words. Please distinguish between those two things and you will see I was not grossly unfair. Thanks again, Regards Maciej From Hermandw at skynet.be Sat Nov 22 12:06:52 2008 From: Hermandw at skynet.be (Herman De Wael) Date: Sat, 22 Nov 2008 12:06:52 +0100 Subject: [blml] laissez faire In-Reply-To: <003901c94c47$a1541140$15844c59@chello.pl> References: <49227C4E.3050007@skynet.be><000001c94a0b$1cc70330$56550990$@com> <000d01c94ad2$b54bc880$15844c59@chello.pl><4925C17B.1010706@talktalk.net> <49266B37.90306@skynet.be><000f01c94bba$ecc985c0$15844c59@chello.pl> <4926C496.6080904@skynet.be> <003901c94c47$a1541140$15844c59@chello.pl> Message-ID: <4927E7CC.4030303@skynet.be> Bystry wrote: > Herman, > >>> I'm shocked that an experienced, international TD claims that there is no >>> penalty for deliberately breaking the Laws. Small hint: 72B1, 90A or even >>> 91A. > >> But that is not my point. >> My point is that if I make the action that is beneficial to me, I will >> get punished by a PP, yet I get to keep my benefit. > > "Benefit" should be judged globally. I can choose a size of PP, so you will > always be in worse position than by obeying the Laws. Remember, deliberately > breaking the Laws to hurt the other contestants and to try to profit from > that is one of the gravest possible offences. It can easily end in front of > L&E Comittee. > quite true - but if you make the PP big enough, then the effects I am talking about are just going to be bigger - if you don't also give the PP to every other poor guy who just does this out of ignorance. And if you decide to give the PP to everyone, then the poor ignorant guy is going to cause havoc - I don't even know what I did wrong and they gave me a 200% fine! >> Whereas some other poor guy, who acts in the same manner, will not get a >> PP, since he doesn't know better. And he too gets to keep his benefit. > > Yes, in a sense he'll keep his benefit. But this will happen only one time, > after then he'll fall into the second category. And of course if his result > is better than A- I'll assign an artificial adjusted score. I don't like > such things, but here is a perfect situation to apply them. Remember, there > is no problem with the OS because they can get a PP and it makes it a losing > strategy to cheat. The problem is with the NOS who sometimes will be hurt. > So I'll give them A+ or I can even use 12A1. > But then again you'll be creating a difference. The guy who follows the law gets a very bad score, just from following the law. His opponents get a very good score. But the guy who does not follow your law gets a PP, maybe a very bad one, OK; but HIS opponents only score AV+? Again, unfair. If you decide to follow this route, then you need to take away the damage. And that is a very difficult thing to do. >> Which means that we ARE treated differently, but also that your rule is >> no good. > > In many situations people are treated differently. The PPs are left to the > TD discretion. And everybody accepts that they are sometimes given to the > experienced players who should have known better but not to the novices. And > of course repeated serious infractions are different than one-time > infraction caused by lack of knowledge. > I agree with all that - but it does not change a thing to the fact that the act itself was beneficial, and you have not taken away the benefit. Compare it to a dummy asking opponent if he has revoked. He should not do this, and if he's experienced, a PP may be in order. But apart from that we'll check for UI given to declarer - and if we think opponents were damaged, we'll also adjust the score. We don't need to bother with the PP just to keep dummy from trying to warn his partner, we have the UI laws for that. Your law is an anti-UI law. A player is obliged to give UI. If he doesn't, you should need a correction of the damage he's caused. Which means a law other than a PP, since the PP will not stop him if it's not big enough. And a PP will not compensate for the damage to opponents. >> What will Probst cheat do? Take the action and the benefit. >> What will John Probst then do? Say "I don't know that you did not know >> better, so I'll judge as if you did" and issue the same penalty he would >> to me. End solution: both me and the Probst cheat have the benefit of >> our actions. > > Why do you think that it is something terrible if Probst cheat escapes once > with A- instead of worse score? Next time he'll get a 200% of top PP and > this will show him that breaking the Laws _is not_ a profitable strategy. > As I said above - a 200% PP seems very strange to everyone. >> If you really want a law that obliges people to give UI to their >> partner, you must also be prepared to have penalties that mean people >> stick to that law; And that will mean creating a sort of construction >> that no TD will be able to pronounce. "You have broken the law which >> requires you to give UI to your partner, so now we will award you a >> score as if you had given such UI, which means that your partner will >> not be allowed to take that particular action." With that particular >> action one taken by this player in the absence of UI. Try explaining >> that there are logical alternatives to an action that a player actually >> took without it being suggested to him! > > Of course you are right. Such a method would be hard to implement. So I > simply state that there is no way to obtain a real table result and I rule > under 12A2 and 12A1 adding an appropriate PP for the OS. And I can guarantee > you that after some of such rulings the news will spread and nobody would > want to risk severe sanctions. > I can guarantee you that it won't. These cases are too far between in any event. >>> Or this experienced, international TD claims that any player can >>> "accidentally", but having full knowledge about TFLB, apply the dWS? > >> My aim is to have the DWS actions being accepted, and dealt with with >> the laws at hand, being the UI laws (in one direction only) and the MI >> laws (both on the first and second MI). > > I'm aware of it. > >> There is (IMO) nothing unfair about this construction. All the >> infractions that I do will be dealt with - but not the silly sounding >> infractions of "failure to give UI" and "failure to tell opponents >> something they are not entitled to". > > Herman, your approach may seem to be logical in theory, but in practice it > is unworkable. > Why? It's very simple - even the lowest bridgers can understand the principles. And the TDs need to learn nothing more than the laws they already know. Including the MI law on the second MI that I give. > a) very often you'll be unable to read partner's mind. So you'll end giving > him UI. very often you know exactly what is in partner's mind, and you know how to NOT give him UI. I wanted to say "more often" but I won't. > b) very often you'll be so suprised by your partner's wrong explanantion > that it'll be practically impossible not to get it known for the whole > table. Look, partner's wrong explanation, 3-5 seconds, and the opponents ask > you what does partner's call mean. Do you really believe that such a player > will be able to keep his face straight and instantly give an explanation > which fits his partner's delusion? Absurd. I'm looking at such behaviors > every day. Your approach would be profitable for veteran, hardskin liars, > only they can really fit into the dWS. So you mean that people break L20F5a all the time? No poker faces out there? You have a very strange perception. I have exactly the opposite one. > c) imagine the feelings of the NOS when after the auction you correct all > the explanations (yours and partner's). They were building an image of the > deal during the auction and now all perishes and they're forced to do it > again. What would you tell them as TD when they claim damage because this > confussion resulted in their misdefence? Are you ready to give them redress > only for their state of mind? > But that is present in any MI case! And it's up to the TD to solve. And, just to be precise, the DWS is very much more easy to follow for opponents. 4NT Blackwood, 5Di one ace. What's the problem? They can easily play on, and the TD will solve the problems later. OTOH 4NT Blackwood, 5Di diamond preference sees the TD at the table immediately and the mess needs to be sorted out now. NOW the opponents are in a mess. And you make another mistake - misdefence is not on the cards, since if they are defenders the mess is cleared up in the clarification period. Why don't you just set yourself at a table and act the story out with the two different explanations, and see which one leads to the least problems for TD. > And leave that nonsensical notion of "failure to give UI". In Bridge UI is > transmitted all the time. Sometimes innocent people end up being > disadvantaged because they simply needed to know the meaning of the auction > or had a difficult decision to make and hesitated. Why don't you worry about > them? Why all your efforts concetrate on the law-infractors who can avoid > all the troubles by simply remembering their agreements? > Yes, UI is given all the time. But there are NO laws which oblige a player to give UI. I don't give UI, and you can deal with the case very easily. You give UI, and I can deal with that case as well. But when you write a law obliging me to give UI, and I don't, you are in a mess, as TD. > The dWS would show to people that it is profitable to lie in order to save > one's own skin. Go and tell that to those Bridge teachers who encourage > young people to come to our game. Continue you efforts to induce people to > think it's good for Bridge. I'm definitely out of it. > But it is not profitable to lie! yes, my score will be better than yours. But the opponents will not be damaged. Not in the sense that they have received everything they are entitled to. Not in the sense that my score is exactly the same as that of a table where the question is not asked. >> Herman. > > Regards > > Maciej > Herman. From Hermandw at skynet.be Sat Nov 22 12:08:15 2008 From: Hermandw at skynet.be (Herman De Wael) Date: Sat, 22 Nov 2008 12:08:15 +0100 Subject: [blml] laissez faire In-Reply-To: <005201c94c5a$746f5230$5d4df690$@com> References: <49227C4E.3050007@skynet.be><000001c94a0b$1cc70330$56550990$@com> <000d01c94ad2$b54bc880$15844c59@chello.pl><4925C17B.1010706@talktalk.net> <49266B37.90306@skynet.be><000f01c94bba$ecc985c0$15844c59@chello.pl> <4926C496.6080904@skynet.be> <003901c94c47$a1541140$15844c59@chello.pl> <005201c94c5a$746f5230$5d4df690$@com> Message-ID: <4927E81F.5050907@skynet.be> Thank you David for your kind words. You describe my position clearly. David Burn wrote: > [Bystry] > > Remember, deliberately breaking the Laws to hurt the other contestants and > to try to profit from that is one of the gravest possible offences. > > [DALB] > > This is grossly unfair. > > Herman's position has always been that where partner has misexplained your > call and you now have to explain his, you are forced to break one Law or > another. If you explain correctly, you break Law 20 (because you indicate > prematurely that a mistake has been made). If you misexplain, you break Law > 40 (because you do not disclose your actual agreements). > > In a position where you must break one of the Laws, it is not a grave > offence to do so. Of course you could avoid being in such a position by > remembering your system, but as Eric and others have cogently argued, it is > not a grave offence not to know your system - if it were, bridge would cease > to be played in clubs the world over. > > Herman has consistently asserted that if you do have to break some Law, it > is preferable to break Law 40 rather than Law 20. Others have asserted the > contrary position, and the WBFLC has now made a decision that to Nigel > Guthrie's disgust was not the main item on the BBC news bulletins for the > next several days. > > I have no doubt that in his role as a Director, Herman will abide by the > WBFLC's decision as conscientiously and as meticulously as he does > everything else. That he has argued a perfectly tenable legal point up to > the stage where the WBFLC has had to make a final determination adds to > rather than detracting from his credentials both as a Director and as a > human being (these categories may not always overlap, but in Herman's case > they do). > > Herman may continue to believe that the WBFLC has made the wrong decision, > and may continue so to argue. From time to time others may weary of such > arguments, but please let us have no more talk of lying or cheating, and > please let us have no more attempts to discredit Herman's considerable and > well-merited standing in the bridge world. > > David Burn > London, England > > > _______________________________________________ > blml mailing list > blml at amsterdamned.org > http://www.amsterdamned.org/mailman/listinfo/blml > From Hermandw at skynet.be Sat Nov 22 12:10:55 2008 From: Hermandw at skynet.be (Herman De Wael) Date: Sat, 22 Nov 2008 12:10:55 +0100 Subject: [blml] laissez faire In-Reply-To: <001d01c94c64$af88f9c0$15844c59@chello.pl> References: <49227C4E.3050007@skynet.be><000001c94a0b$1cc70330$56550990$@com> <000d01c94ad2$b54bc880$15844c59@chello.pl><4925C17B.1010706@talktalk.net> <49266B37.90306@skynet.be><000f01c94bba$ecc985c0$15844c59@chello.pl> <4926C496.6080904@skynet.be><003901c94c47$a1541140$15844c59@chello.pl> <005201c94c5a$746f5230$5d4df690$@com> <001d01c94c64$af88f9c0$15844c59@chello.pl> Message-ID: <4927E8BF.8010705@skynet.be> Bystry wrote: > > Next point. I may be wrong but it's hard for me to swallow that any average > player would understand words from Law 20F5 "...in any manner..." in any > other way than "...in any manner except when another Law forces you to do > it". It is very hard for me to swallow that anyone would understand the latter. Only a director who thinks he has a clear grasp of the bridge laws can feel that it is obvious that other laws overwrite something which is so clearly put. Just another proof of people who turn themselves into impossible curbs just to prove to themselves that the thing they believed in from gut feeling must be right. Herman. From nigelguthrie at talktalk.net Sat Nov 22 12:59:01 2008 From: nigelguthrie at talktalk.net (Nigel Guthrie) Date: Sat, 22 Nov 2008 11:59:01 +0000 Subject: [blml] laissez faire In-Reply-To: <005201c94c5a$746f5230$5d4df690$@com> References: <49227C4E.3050007@skynet.be><000001c94a0b$1cc70330$56550990$@com> <000d01c94ad2$b54bc880$15844c59@chello.pl><4925C17B.1010706@talktalk.net> <49266B37.90306@skynet.be><000f01c94bba$ecc985c0$15844c59@chello.pl> <4926C496.6080904@skynet.be> <003901c94c47$a1541140$15844c59@chello.pl> <005201c94c5a$746f5230$5d4df690$@com> Message-ID: <4927F405.20307@talktalk.net> [DALB] This is grossly unfair. Herman's position has always been that where partner has misexplained your call and you now have to explain his, you are forced to break one Law or another. If you explain correctly, you break Law 20 (because you indicate prematurely that a mistake has been made). If you misexplain, you break Law 40 (because you do not disclose your actual agreements). In a position where you must break one of the Laws, it is not a grave offence to do so. Of course you could avoid being in such a position by remembering your system, but as Eric and others have cogently argued, it is not a grave offence not to know your system - if it were, bridge would cease to be played in clubs the world over. Herman has consistently asserted that if you do have to break some Law, it is preferable to break Law 40 rather than Law 20. Others have asserted the contrary position, and the WBFLC has now made a decision that to Nigel Guthrie's disgust was not the main item on the BBC news bulletins for the next several days. I have no doubt that in his role as a Director, Herman will abide by the WBFLC's decision as conscientiously and as meticulously as he does everything else. That he has argued a perfectly tenable legal point up to the stage where the WBFLC has had to make a final determination adds to rather than detracting from his credentials both as a Director and as a human being (these categories may not always overlap, but in Herman's case they do). Herman may continue to believe that the WBFLC has made the wrong decision, and may continue so to argue. From time to time others may weary of such arguments, but please let us have no more talk of lying or cheating, and please let us have no more attempts to discredit Herman's considerable and well-merited standing in the bridge world. [David] Herman writes that he breaks law 40 rather than law 20. He says that he did this in the past and he continues to do so. Although the law-book seems to be ambiguous, the WBF has always claimed that law 40 supervenes. Belatedly, we are told, the WBF has clarified this position, in some obscure minute. If the WBF published such clarifications in place, in the law-book, itself, the ordinary player would have a better chance of compliance. His NBO and the WBF both continue to turn a blind eye to the self-publicised practices of an international director. Despite ridicule from Grattan Endicott and David Burn, it is quite fair for us to discuss these issues. From darkbystry at wp.pl Sat Nov 22 14:30:04 2008 From: darkbystry at wp.pl (Bystry) Date: Sat, 22 Nov 2008 14:30:04 +0100 Subject: [blml] laissez faire References: <49227C4E.3050007@skynet.be><000001c94a0b$1cc70330$56550990$@com> <000d01c94ad2$b54bc880$15844c59@chello.pl><4925C17B.1010706@talktalk.net> <49266B37.90306@skynet.be><000f01c94bba$ecc985c0$15844c59@chello.pl> <4926C496.6080904@skynet.be><003901c94c47$a1541140$15844c59@chello.pl> <005201c94c5a$746f5230$5d4df690$@com><001d01c94c64$af88f9c0$15844c59@chello.pl> <4927E8BF.8010705@skynet.be> Message-ID: <001501c94ca6$6e71be80$15844c59@chello.pl> Herman, > > Next point. I may be wrong but it's hard for me to swallow that any average > > player would understand words from Law 20F5 "...in any manner..." in any > > other way than "...in any manner except when another Law forces you to do > > it". > > It is very hard for me to swallow that anyone would understand the > latter. Only a director who thinks he has a clear grasp of the bridge > laws can feel that it is obvious that other laws overwrite something > which is so clearly put. > > Just another proof of people who turn themselves into impossible curbs > just to prove to themselves that the thing they believed in from gut > feeling must be right. Following your ideas every bylaw of field firing range should include such worlds: "it is allowed to shoot at the targets unless the shooting instructor is there to change the target". Or every bylaw of public toilets should include such worlds: "it is allowed to use the urinal unless somebody else is just using it". Get real. There is something like common sense. I agree with you that after your whole crusade it would be better to change the wording of 20F5. But the lawmakers apparently thought it's obvious how to read it. Grattan stated something similar a few years ago. So acknowledge it and try to put your efforts in something more important. You may believe me or not, but I like your devotion and consequence in fighting for something you believe to be better. I admire your commitment and I regret you may sometimes be taken as someone too obstinate or hilarious. I too found myself in my life in your position and I know how it is. But there always is a border where me must stop and reconsider. Your approach is logical, true. It is better in some circumstances, true. But you are so owly in your crusade that you no longer accept any arguments given by your opponents. You simply reject them, claiming they are bad, but never proving that. And it makes it hard to reason with you. Just to give an example - your worries that the CS will induce a deluge of questioning. I must have repeated myself several times to explain it to you that such a statement is illogical and simply wrong. Because we're under the CS over many years and there is no such thing. Because people are no cheats-in-nature and most of them won't do something like it. Because it is a losing strategy to ask - the risk of reducing partner's options via UI is overwhelmingly greater than the outcome from rare catching the opponents in the middle of the mixup (it's like having 10 doors to choose, in 9 you are killed and in 1 you get 10000 Euro). Now that doesn't mean I'm always right. You gave some cogent arguments. The best one is certainly how to adjust when somebody disobeys the Law. Yes, the OS can be penalised but it's hard to redress the damage made to the NOS. You're right. It is practically impossible although 12A1 gives us some possibility. So I admitted, sometimes we have to choose lesser evil. We can harshly penalise repeated offenders (to make it unprofitable for them). Therefore we can highly reduce such cases. We can try to adjust as long as it is possible given the MI and one-side UI. Sometimes we can assign an artificial score. And sometimes there will come a case, when the NOS will be really hurted and we'll be helpless. But look, think about reality. After the proper education, after catching those single players unaware of the Laws (remember that most people follow the CS even now), after eliminating very poor scores obtained even under the dWS, with what we were left? A small group of obstinate cheaters? Who could only realise their plans in different places, otherwise they will get a whopping PP and can land in front of L&E Comittee? How many cases are left, per mille? I think even less. And the entire number of disadvantaged non-offenders will be significally smaller than the entire number of those who get unfair rulings or who are unaware they were fouled. We are unable to achieve perfection, we can only try. > Herman. Regards Maciej From nigelguthrie at talktalk.net Sat Nov 22 14:45:50 2008 From: nigelguthrie at talktalk.net (Nigel Guthrie) Date: Sat, 22 Nov 2008 13:45:50 +0000 Subject: [blml] laissez faire In-Reply-To: <4927F405.20307@talktalk.net> References: <49227C4E.3050007@skynet.be><000001c94a0b$1cc70330$56550990$@com> <000d01c94ad2$b54bc880$15844c59@chello.pl><4925C17B.1010706@talktalk.net> <49266B37.90306@skynet.be><000f01c94bba$ecc985c0$15844c59@chello.pl> <4926C496.6080904@skynet.be> <003901c94c47$a1541140$15844c59@chello.pl> <005201c94c5a$746f5230$5d4df690$@com> <4927F405.20307@talktalk.net> Message-ID: <49280D0E.4050304@talktalk.net> [Nigel] Herman writes that he breaks law 40 rather than law 20. He says that he did this in the past and he continues to do so. Although the law-book seems to be ambiguous, the WBF has always claimed that law 40 supervenes. Belatedly, we are told, the WBF has clarified this position, in some obscure minute. If the WBF published such clarifications in place, in the law-book, itself, the ordinary player would have a better chance of compliance. His NBO and the WBF both continue to turn a blind eye to the self-publicised practices of an international director. Despite ridicule from Grattan Endicott and David Burn, it is quite fair for us to discuss these issues. From darkbystry at wp.pl Sat Nov 22 15:19:13 2008 From: darkbystry at wp.pl (Bystry) Date: Sat, 22 Nov 2008 15:19:13 +0100 Subject: [blml] laissez faire References: <49227C4E.3050007@skynet.be><000001c94a0b$1cc70330$56550990$@com> <000d01c94ad2$b54bc880$15844c59@chello.pl><4925C17B.1010706@talktalk.net> <49266B37.90306@skynet.be><000f01c94bba$ecc985c0$15844c59@chello.pl> <4926C496.6080904@skynet.be><003901c94c47$a1541140$15844c59@chello.pl> <4927E7CC.4030303@skynet.be> Message-ID: <001f01c94cad$4bf57340$15844c59@chello.pl> Herman, [snip, answered in another email] > > Herman, your approach may seem to be logical in theory, but in practice it > > is unworkable. > > > > Why? It's very simple - even the lowest bridgers can understand the > principles. And the TDs need to learn nothing more than the laws they > already know. Including the MI law on the second MI that I give. It's not wise to answer before you read the whole statement. > > a) very often you'll be unable to read partner's mind. So you'll end giving > > him UI. > > very often you know exactly what is in partner's mind, and you know how > to NOT give him UI. I wanted to say "more often" but I won't. Even if it was true, it does not make my point invalid. Following the CS you always know what to do. Following the dWS you sometimes are unable to do anything clever. And I'm sure the answers will take you so much time that everybody at the table will know what is going on. Telling the truth is easy, lying requires practice and cool blood. > > b) very often you'll be so suprised by your partner's wrong explanantion > > that it'll be practically impossible not to get it known for the whole > > table. Look, partner's wrong explanation, 3-5 seconds, and the opponents ask > > you what does partner's call mean. Do you really believe that such a player > > will be able to keep his face straight and instantly give an explanation > > which fits his partner's delusion? Absurd. I'm looking at such behaviors > > every day. Your approach would be profitable for veteran, hardskin liars, > > only they can really fit into the dWS. > > So you mean that people break L20F5a all the time? No poker faces out > there? You have a very strange perception. I have exactly the opposite one. Yes, that's exactly what I mean. Most people cannot keep poker faces. That doesn't mean their partners are aware of it. Very often they don't look and nothing is wrong. But I think that good observers will usually feel such things. Only the veterans, who were in such positions many times, can remain cool. But it's not important. Most of such things remain undetected. We are talking about 5 seconds in which you must decide what to do. And in my opinion it is not enough. First reaction is quite natural - who is wrong. It is not necessary for dWS but it will come to mind nevertheless. Than you must realise what does partner's bid mean in his mind. And it can be quite hard. You deliberately give simple examples like 4NT but sometimes it won't be obvious for you. Imagine opening 2D as a major-minor two suiter. Your partner explains it as Multi and bids 3C which is being questioned. You've played Multi two years ago and now you're trying to remember what did 3C mean. Was it natural or invitational with hearts? And the things get worse when you've never played Multi in this partnership, but in two different partnerships with different agreements. What to answer (and time passes)? Look, I'm not claiming that your method won't always work. But the CS is simple and clear. No need to invent anything. No mindreading. No confusion. > > c) imagine the feelings of the NOS when after the auction you correct all > > the explanations (yours and partner's). They were building an image of the > > deal during the auction and now all perishes and they're forced to do it > > again. What would you tell them as TD when they claim damage because this > > confussion resulted in their misdefence? Are you ready to give them redress > > only for their state of mind? > > > > But that is present in any MI case! And it's up to the TD to solve. The TD is not a God. Acknowledge it. The players want to play their game, not to have the TD playing it for them. And MI rulings are usually worst for the NOS. So exchanging MI ruling for UI ruling is almost always good for them. > And, just to be precise, the DWS is very much more easy to follow for > opponents. 4NT Blackwood, 5Di one ace. What's the problem? They can > easily play on, and the TD will solve the problems later. Again. They want to make their own decisions. They want to know what is going on. They don't want some other people to decide what they would have done. And remember, they are following something completely false. > OTOH 4NT Blackwood, 5Di diamond preference sees the TD at the table > immediately and the mess needs to be sorted out now. NOW the opponents > are in a mess. You can't be real. I'll take is as a delusion, otherwise I'll have to assume you are deliberately giving false arguments. The NOS are in the *great position*, not in a mess. They perfectly know you are in a mixup. Since that moment they can do many things knowing you are bound by UI. > And you make another mistake - misdefence is not on the cards, since if > they are defenders the mess is cleared up in the clarification period. It is. But it's very hard to change your whole view in such a short time. Put yourself in their position. They were given clear explanations during the auction which resulted in building an image of the deal. They've taken some inferences, they're ready for the battle. And now you ruin all their assumptions correcting *both* parts of explanations. This can easily result in a confusion, which would be hard to overcome. Maybe not for Meckwell, but for most ordinary players. And now there is no MI. So if they misdefend they *won't be given any redress*. There is no Law which allows the TD to rule an adjusted score on the basis of their own misconception. And the OS has done everything to confuse them and to induce any possible error. That is for me plainly unfair and I won't change my mind. > Why don't you just set yourself at a table and act the story out with > the two different explanations, and see which one leads to the least > problems for TD. The TD is paid to do his duty. That is not of my concern if it takes him 1 hour or 5 minutes to give correct ruling. I only care for the NOS, they are disadvantaged by the infraction. If the TD is angry because he's got more work he can unload his frustration and give a PP to the OS. But I'd rather he didn't. > Yes, UI is given all the time. But there are NO laws which oblige a > player to give UI. I don't give UI, and you can deal with the case very > easily. You give UI, and I can deal with that case as well. But when you > write a law obliging me to give UI, and I don't, you are in a mess, as TD. Why are you constantly worrying about the OS and the TD? Should I assume that those are the only roles you have been at as a member of Bridge society? Sorry, I know this is certainly unjust, but there is none, even the smallest thing, that you use as an argument for the dWS which is good for the NOS. > But it is not profitable to lie! yes, my score will be better than > yours. But the opponents will not be damaged. Not in the sense that they > have received everything they are entitled to. Not in the sense that my > score is exactly the same as that of a table where the question is not > asked. Your definition of "damage" is of the highest quality. Yes, I wasn't not damaged. Herman came to my table, misexplained all he could, then he corrected it, I was never more sure what is going on, misdefended, then after the hard fought battle with the TD I received my "equity" ruling giving me half of what I would get playing against opponents who don't infract. Yes, I wasn't damaged. My feelings, my wasted time, my nerves, that is all irrelevant. All that counts is Herman's right to escape from his self-induced disaster and the comfort of the TD's work. Thank you, no other questions. Bye. > Herman. Regards Maciej From rfrick at rfrick.info Sat Nov 22 19:15:59 2008 From: rfrick at rfrick.info (Robert Frick) Date: Sat, 22 Nov 2008 13:15:59 -0500 Subject: [blml] laissez faire In-Reply-To: <001f01c94cad$4bf57340$15844c59@chello.pl> References: <49227C4E.3050007@skynet.be> <000001c94a0b$1cc70330$56550990$@com> <000d01c94ad2$b54bc880$15844c59@chello.pl> <4925C17B.1010706@talktalk.net> <49266B37.90306@skynet.be> <000f01c94bba$ecc985c0$15844c59@chello.pl> <4926C496.6080904@skynet.be> <003901c94c47$a1541140$15844c59@chello.pl> <4927E7CC.4030303@skynet.be> <001f01c94cad$4bf57340$15844c59@chello.pl> Message-ID: Hi Maciej. I am trying to figure out your position. But I am interested in how anyone would answer this question. Is there a consensus? You make a bid which partner incorrectly explains. For example, your agreement is to play 2Di as weak in first seat and multi otherwise. You open 2Di in first seat and partner explains it as multi. Your partner now bids 3S. In your system, which the opponents do not know, partnership agreement is that this shows a strong hand with a spade suit over a weak 2Di. Over multi, in your system which the opponents do not know, that asks for a spade stopper. So you know your partner is asking for a spade stopper, but you have to bid like he is showing a spade stopper. You have a great hand for 3NT, except no spade stopper. So you are forced to bid 3NT, awaiting disaster. That ends the auction. You now explain that your partner incorrectly described your bid, and in fact your 2 Di bid was weak. The opponents now ask you to explain your other bids. What are you (and/or your partner) required to explain? Bob, who, unless he screwed something up, which is common, is asking a question he thinks people won't like From svenpran at online.no Sat Nov 22 21:14:18 2008 From: svenpran at online.no (Sven Pran) Date: Sat, 22 Nov 2008 21:14:18 +0100 Subject: [blml] laissez faire In-Reply-To: References: <49227C4E.3050007@skynet.be> <000001c94a0b$1cc70330$56550990$@com> <000d01c94ad2$b54bc880$15844c59@chello.pl> <4925C17B.1010706@talktalk.net> <49266B37.90306@skynet.be> <000f01c94bba$ecc985c0$15844c59@chello.pl> <4926C496.6080904@skynet.be> <003901c94c47$a1541140$15844c59@chello.pl> <4927E7CC.4030303@skynet.be> <001f01c94cad$4bf57340$15844c59@chello.pl> Message-ID: <000001c94cde$e66eedd0$b34cc970$@no> On Behalf Of Robert Frick > Hi Maciej. I am trying to figure out your position. But I am interested in > how anyone would answer this question. Is there a consensus? > > You make a bid which partner incorrectly explains. For example, your > agreement is to play 2Di as weak in first seat and multi otherwise. You > open 2Di in first seat and partner explains it as multi. > > Your partner now bids 3S. In your system, which the opponents do not know, > partnership agreement is that this shows a strong hand with a spade suit > over a weak 2Di. Over multi, in your system which the opponents do not > know, that asks for a spade stopper. So you know your partner is asking > for a spade stopper, but you have to bid like he is showing a spade > stopper. > > You have a great hand for 3NT, except no spade stopper. So you are forced > to bid 3NT, awaiting disaster. That ends the auction. You now explain that > your partner incorrectly described your bid, and in fact your 2 Di bid was > weak. > > The opponents now ask you to explain your other bids. What are you (and/or > your partner) required to explain? You are required to explain according to your partnership understanding after an opening bid of weak 2D, not the understanding that would be applicable after a multi 2D. Your partner will (hopefully) now realize that he had forgotten your partnership understanding and must also give his part of the explanation according to your opening bid being a weak 2D. That he as a consequence of his mistake has not called according to your partnership understandings is irrelevant and need not be emphasized. Is there really any problem here? Regards Sven From grandaeval at tiscali.co.uk Sun Nov 23 00:24:12 2008 From: grandaeval at tiscali.co.uk (Grattan) Date: Sat, 22 Nov 2008 23:24:12 -0000 Subject: [blml] laissez faire References: <49227C4E.3050007@skynet.be><000001c94a0b$1cc70330$56550990$@com> <000d01c94ad2$b54bc880$15844c59@chello.pl><4925C17B.1010706@talktalk.net> <49266B37.90306@skynet.be><000f01c94bba$ecc985c0$15844c59@chello.pl> <4926C496.6080904@skynet.be> <003901c94c47$a1541140$15844c59@chello.pl> <005201c94c5a$746f5230$5d4df690$@com><4927F405.20307@talktalk.net> <49280D0E.4050304@talktalk.net> Message-ID: <001601c94cf9$81daaf10$0302a8c0@Mildred> Grattan Endicott To: "Bridge Laws Mailing List" Sent: Saturday, November 22, 2008 1:45 PM Subject: Re: [blml] laissez faire > Despite ridicule from Grattan Endicott > +=+ What 'ridicule'? I simply say what the law is, as determined and publicly announced by the WBFLC. ~ Grattan ~ +=+ From geller at nifty.com Sun Nov 23 01:26:05 2008 From: geller at nifty.com (Robert Geller) Date: Sun, 23 Nov 2008 09:26:05 +0900 Subject: [blml] laissez faire In-Reply-To: <001601c94cf9$81daaf10$0302a8c0@Mildred> References: <001601c94cf9$81daaf10$0302a8c0@Mildred> Message-ID: <200811230026.AA16699@geller204.nifty.com> The WBFLC minutes are all available on the WEB so technically Nigel's criticisms on this point are incorrect, but the site is far from user-friendly. For example, the minutes could be cross-indexed by Law number as well as date, with hyperlinks available to each law number. This would be helpful. -Bob Grattan ????????: > > >Grattan Endicottalso ************************************ >"We desire truth and find within >ourselves only uncertainty." > [Pascal] >''''''''''''''''''''''''''''''''''''''''''''''''''''''''''''''''''''''''''''''''''''''''''''''''' >----- Original Message ----- >From: "Nigel Guthrie" >To: "Bridge Laws Mailing List" >Sent: Saturday, November 22, 2008 1:45 PM >Subject: Re: [blml] laissez faire > > >> Despite ridicule from Grattan Endicott >> >+=+ What 'ridicule'? I simply say what the law is, as determined >and publicly announced by the WBFLC. > ~ Grattan ~ +=+ > > >_______________________________________________ >blml mailing list >blml at amsterdamned.org >http://www.amsterdamned.org/mailman/listinfo/blml ----------------------------------------------------- Robert (Bob) Geller, Tokyo, Japan geller at nifty.com From Hermandw at skynet.be Sun Nov 23 10:54:15 2008 From: Hermandw at skynet.be (Herman De Wael) Date: Sun, 23 Nov 2008 10:54:15 +0100 Subject: [blml] laissez faire In-Reply-To: <001f01c94cad$4bf57340$15844c59@chello.pl> References: <49227C4E.3050007@skynet.be><000001c94a0b$1cc70330$56550990$@com> <000d01c94ad2$b54bc880$15844c59@chello.pl><4925C17B.1010706@talktalk.net> <49266B37.90306@skynet.be><000f01c94bba$ecc985c0$15844c59@chello.pl> <4926C496.6080904@skynet.be><003901c94c47$a1541140$15844c59@chello.pl> <4927E7CC.4030303@skynet.be> <001f01c94cad$4bf57340$15844c59@chello.pl> Message-ID: <49292847.9010802@skynet.be> Bystry wrote: > >>> a) very often you'll be unable to read partner's mind. So you'll end > giving >>> him UI. >> very often you know exactly what is in partner's mind, and you know how >> to NOT give him UI. I wanted to say "more often" but I won't. > > Even if it was true, it does not make my point invalid. Following the CS you > always know what to do. Following the dWS you sometimes are unable to do > anything clever. And I'm sure the answers will take you so much time that > everybody at the table will know what is going on. Telling the truth is > easy, lying requires practice and cool blood. > Sorry Maciej, but this is simply not true. Following the MS (I don't know where you get the C from - M is for Majority), you DONT always know what to do. Since you are sometimes unsure of who has the correct explanation of system. OTOH, following the DWS, you always know what to do, provided you know the system your partner thinks you are playing. Now of course there can be circumstances where your partner starts explaining a system that you have no idea about. But do you really think this is so common an occurance? And even if it sometimes happens, what then about the DWSer who tries to explain his partner's system and fails? Well, his partner will just think that he has forgotten the system, not that he was wrong in the first place. Of course the player should not say "I have no idea what it means, I have never played that system before"; but saying "I think that's XX" does not convey the UI that the previous explanation was wrong. Herman. From nigelguthrie at talktalk.net Sun Nov 23 14:41:35 2008 From: nigelguthrie at talktalk.net (Nigel Guthrie) Date: Sun, 23 Nov 2008 13:41:35 +0000 Subject: [blml] Web Rules In-Reply-To: <200811230026.AA16699@geller204.nifty.com> References: <001601c94cf9$81daaf10$0302a8c0@Mildred> <200811230026.AA16699@geller204.nifty.com> Message-ID: <49295D8F.9020500@talktalk.net> [Robert Geller] The WBFLC minutes are all available on the WEB so technically Nigel's criticisms on this point are incorrect, but the site is far from user-friendly. For example, the minutes could be cross-indexed by Law number as well as date, with hyperlinks available to each law number. This would be helpful. [Nigel] Long, fragmented minutes are obscure (when compared with a few words of correction, in place, in the law-book). A brief look confirms that information is still in slow bulky PDF rather than web-friendly HTML. This would be slightly less frustrating were I prepared to print it all but I'm not. In the not-too-distant future, directors will consult an intelligent interactive on-line law-book that presents all rules relevant to a case. I hope the WBF won't hold back progress, indefinitely. The first step is a set of complete up-to-date rules. It would still be useful to have a separate list of dated corrections. From rfrick at rfrick.info Sun Nov 23 16:19:51 2008 From: rfrick at rfrick.info (Robert Frick) Date: Sun, 23 Nov 2008 10:19:51 -0500 Subject: [blml] laissez faire In-Reply-To: <000001c94cde$e66eedd0$b34cc970$@no> References: <49227C4E.3050007@skynet.be> <000001c94a0b$1cc70330$56550990$@com> <000d01c94ad2$b54bc880$15844c59@chello.pl> <4925C17B.1010706@talktalk.net> <49266B37.90306@skynet.be> <000f01c94bba$ecc985c0$15844c59@chello.pl> <4926C496.6080904@skynet.be> <003901c94c47$a1541140$15844c59@chello.pl> <4927E7CC.4030303@skynet.be> <001f01c94cad$4bf57340$15844c59@chello.pl> <000001c94cde$e66eedd0$b34cc970$@no> Message-ID: On Sat, 22 Nov 2008 15:14:18 -0500, Sven Pran wrote: > On Behalf Of Robert Frick >> Hi Maciej. I am trying to figure out your position. But I am interested >> in >> how anyone would answer this question. Is there a consensus? >> >> You make a bid which partner incorrectly explains. For example, your >> agreement is to play 2Di as weak in first seat and multi otherwise. You >> open 2Di in first seat and partner explains it as multi. >> >> Your partner now bids 3S. In your system, which the opponents do not >> know, >> partnership agreement is that this shows a strong hand with a spade suit >> over a weak 2Di. Over multi, in your system which the opponents do not >> know, that asks for a spade stopper. So you know your partner is asking >> for a spade stopper, but you have to bid like he is showing a spade >> stopper. >> >> You have a great hand for 3NT, except no spade stopper. So you are >> forced >> to bid 3NT, awaiting disaster. That ends the auction. You now explain >> that >> your partner incorrectly described your bid, and in fact your 2 Di bid >> was >> weak. >> >> The opponents now ask you to explain your other bids. What are you >> (and/or >> your partner) required to explain? > > You are required to explain according to your partnership understanding > after an opening bid of weak 2D, not the understanding that would be > applicable after a multi 2D. > > Your partner will (hopefully) now realize that he had forgotten your > partnership understanding and must also give his part of the explanation > according to your opening bid being a weak 2D. That he as a consequence > of > his mistake has not called according to your partnership understandings > is > irrelevant and need not be emphasized. > > Is there really any problem here? I don't know, that's why I am asking. Everyone agrees that after the auction is over, 3 Sp can be described as showing spades, even though both players know it was meant to ask for a spade stopper? (Doing so should substantially increase your chances of making 3NT.) From nigelguthrie at talktalk.net Sun Nov 23 18:12:59 2008 From: nigelguthrie at talktalk.net (Nigel Guthrie) Date: Sun, 23 Nov 2008 17:12:59 +0000 Subject: [blml] laissez faire In-Reply-To: <001601c94cf9$81daaf10$0302a8c0@Mildred> References: <49227C4E.3050007@skynet.be><000001c94a0b$1cc70330$56550990$@com> <000d01c94ad2$b54bc880$15844c59@chello.pl><4925C17B.1010706@talktalk.net> <49266B37.90306@skynet.be><000f01c94bba$ecc985c0$15844c59@chello.pl> <4926C496.6080904@skynet.be> <003901c94c47$a1541140$15844c59@chello.pl> <005201c94c5a$746f5230$5d4df690$@com><4927F405.20307@talktalk.net> <49280D0E.4050304@talktalk.net> <001601c94cf9$81daaf10$0302a8c0@Mildred> Message-ID: <49298F1B.5080604@talktalk.net> [Grattan Endicott] +=+ What 'ridicule'? I simply say what the law is, as determined and publicly announced by the WBFLC. [Nigel] I apologise to Grattan who simply claims that I'm wrong. It was David Burn who wrote .. [David Burn] ... the WBFLC has now made a decision that to Nigel Guthrie's disgust was not the main item on the BBC news bulletins for the next several days. From svenpran at online.no Sun Nov 23 18:13:08 2008 From: svenpran at online.no (Sven Pran) Date: Sun, 23 Nov 2008 18:13:08 +0100 Subject: [blml] laissez faire In-Reply-To: References: <49227C4E.3050007@skynet.be> <000001c94a0b$1cc70330$56550990$@com> <000d01c94ad2$b54bc880$15844c59@chello.pl> <4925C17B.1010706@talktalk.net> <49266B37.90306@skynet.be> <000f01c94bba$ecc985c0$15844c59@chello.pl> <4926C496.6080904@skynet.be> <003901c94c47$a1541140$15844c59@chello.pl> <4927E7CC.4030303@skynet.be> <001f01c94cad$4bf57340$15844c59@chello.pl> <000001c94cde$e66eedd0$b34cc970$@no> Message-ID: <000301c94d8e$c1c90000$455b0000$@no> On Behalf Of Robert Frick ............... > >> The opponents now ask you to explain your other bids. What are you > >> (and/or your partner) required to explain? > > > > You are required to explain according to your partnership understanding > > after an opening bid of weak 2D, not the understanding that would be > > applicable after a multi 2D. > > > > Your partner will (hopefully) now realize that he had forgotten your > > partnership understanding and must also give his part of the explanation > > according to your opening bid being a weak 2D. That he as a consequence > > of his mistake has not called according to your partnership understandings > > is irrelevant and need not be emphasized. > > > > Is there really any problem here? > > I don't know, that's why I am asking. Everyone agrees that after the > auction is over, 3 Sp can be described as showing spades, even though both > players know it was meant to ask for a spade stopper? (Doing so should > substantially increase your chances of making 3NT.) If both players (presumed declarer and his partner) "know" that 3 Sp can be described as showing spades even though both of them "know" that it was meant to ask for a spade stopper then either they must have a concealed partnership understanding (e.g. from experience with that partner) or one of them must have received UI during the auction (in the form of noticing some explanation been given by his partner to opponents). As the latter is the case here I shall not comment on CPU. But as presumed declarer (you) is aware that opponents have received incorrect explanation during the auction you are responsible for correcting this incorrect explanation during the clarification period without being asked any further questions. In this case opponents should be informed that the 2D bid was incorrectly explained as multi and that over 2D multi a 3Sp bid asks for spade stopper. Doing so should effectively take away any chances of making 3NT that was increased by the incorrect information. Do you still see any problem (for the defenders)? Sven From svenpran at online.no Sun Nov 23 19:05:42 2008 From: svenpran at online.no (Sven Pran) Date: Sun, 23 Nov 2008 19:05:42 +0100 Subject: [blml] laissez faire In-Reply-To: <000301c94d8e$c1c90000$455b0000$@no> References: <49227C4E.3050007@skynet.be> <000001c94a0b$1cc70330$56550990$@com> <000d01c94ad2$b54bc880$15844c59@chello.pl> <4925C17B.1010706@talktalk.net> <49266B37.90306@skynet.be> <000f01c94bba$ecc985c0$15844c59@chello.pl> <4926C496.6080904@skynet.be> <003901c94c47$a1541140$15844c59@chello.pl> <4927E7CC.4030303@skynet.be> <001f01c94cad$4bf57340$15844c59@chello.pl> <000001c94cde$e66eedd0$b34cc970$@no> <000301c94d8e$c1c90000$455b0000$@no> Message-ID: <000401c94d96$19f416f0$4ddc44d0$@no> On Behalf Of Sven Pran .................. > If both players (presumed declarer and his partner) "know" that 3 Sp can be > described as showing spades even though both of them "know" that it was > meant to ask for a spade stopper then either they must have a concealed > partnership understanding (e.g. from experience with that partner) or one of > them must have received UI during the auction (in the form of noticing some > explanation been given by his partner to opponents). > > As the latter is the case here I shall not comment on CPU. But as presumed > declarer (you) is aware that opponents have received incorrect explanation > during the auction you are responsible for correcting this incorrect > explanation during the clarification period without being asked any further > questions. In this case opponents should be informed that the 2D bid was > incorrectly explained as multi and that over 2D multi a 3Sp bid asks for > spade stopper. Correction, because multi was not relevant in this position you are not supposed to explain your responses to a multi 2D opening bid here. (But opponents must anyway be alerted that the explanation was incorrect!) Mea culpa. Sven From rfrick at rfrick.info Sun Nov 23 23:49:07 2008 From: rfrick at rfrick.info (Robert Frick) Date: Sun, 23 Nov 2008 17:49:07 -0500 Subject: [blml] laissez faire In-Reply-To: <000401c94d96$19f416f0$4ddc44d0$@no> References: <49227C4E.3050007@skynet.be> <000001c94a0b$1cc70330$56550990$@com> <000d01c94ad2$b54bc880$15844c59@chello.pl> <4925C17B.1010706@talktalk.net> <49266B37.90306@skynet.be> <000f01c94bba$ecc985c0$15844c59@chello.pl> <4926C496.6080904@skynet.be> <003901c94c47$a1541140$15844c59@chello.pl> <4927E7CC.4030303@skynet.be> <001f01c94cad$4bf57340$15844c59@chello.pl> <000001c94cde$e66eedd0$b34cc970$@no> <000301c94d8e$c1c90000$455b0000$@no> <000401c94d96$19f416f0$4ddc44d0$@no> Message-ID: On Sun, 23 Nov 2008 13:05:42 -0500, Sven Pran wrote: > On Behalf Of Sven Pran > .................. >> If both players (presumed declarer and his partner) "know" that 3 Sp can > be >> described as showing spades even though both of them "know" that it was >> meant to ask for a spade stopper then either they must have a concealed >> partnership understanding (e.g. from experience with that partner) or >> one > of >> them must have received UI during the auction (in the form of noticing > some >> explanation been given by his partner to opponents). >> >> As the latter is the case here I shall not comment on CPU. But as >> presumed >> declarer (you) is aware that opponents have received incorrect >> explanation >> during the auction you are responsible for correcting this incorrect >> explanation during the clarification period without being asked any > further >> questions. In this case opponents should be informed that the 2D bid was >> incorrectly explained as multi and that over 2D multi a 3Sp bid asks for >> spade stopper. > > Correction, because multi was not relevant in this position you are not > supposed to explain your responses to a multi 2D opening bid here. (But > opponents must anyway be alerted that the explanation was incorrect!) > > Mea culpa. > > Sven Okay. Just to make sure. You of course tell them that the explanation of 2 Di was wrong. But you are saying that you tell them only that 3 Sp showed spades. And you have clarified that there are two sets of partnership agreements here. Are they entitled to the partnership agreement about the meaning of 3 Sp over a multi 2Di, or can you just tell them the meaning of 3 Sp over a weak 2 Di. From dalburn at btopenworld.com Mon Nov 24 00:39:57 2008 From: dalburn at btopenworld.com (David Burn) Date: Sun, 23 Nov 2008 23:39:57 -0000 Subject: [blml] laissez faire In-Reply-To: References: <49227C4E.3050007@skynet.be> <000001c94a0b$1cc70330$56550990$@com> <000d01c94ad2$b54bc880$15844c59@chello.pl> <4925C17B.1010706@talktalk.net> <49266B37.90306@skynet.be> <000f01c94bba$ecc985c0$15844c59@chello.pl> <4926C496.6080904@skynet.be> <003901c94c47$a1541140$15844c59@chello.pl> <4927E7CC.4030303@skynet.be> <001f01c94cad$4bf57340$15844c59@chello.pl> <000001c94cde$e66eedd0$b34cc970$@no> <000301c94d8e$c1c90000$455b0000$@no> <000401c94d96$19f416f0$4ddc44d0$@no> Message-ID: <000001c94dc4$cbb14150$6313c3f0$@com> [RF] And you have clarified that there are two sets of partnership agreements here. [DALB] I am not sure that this is what Sven has said, but of course he can answer for himself. For myself, there is only one "set of partnership agreements" - we do not "agree" to forget our system. [RF] Are they entitled to the partnership agreement about the meaning of 3 Sp over a multi 2Di, or can you just tell them the meaning of 3 Sp over a weak 2 Di? [DALB] Others have said that in the canonical example of 4NT being minors but taken as Blackwood, the opponents are entitled to know what a 5D response would have meant if 4NT had been Blackwood. To me, this makes no sense. Suppose I alert a 2C response to 1NT and tell the opponents that it is a puppet to 2D, which I must now bid. Are they entitled to know what 2D would have meant if 2C had been Stayman? I can see no reason why they should be. Grattan has said that the opponents are allowed (a different thing altogether from "entitled") to know what a 5D response to Blackwood would mean if I choose to tell them. Moreover, if they can somehow glean that the 5D bidder was responding to Blackwood and not giving preference between the minors, they are allowed - and perhaps even entitled - to consult our pre-event disclosure as to the meanings of responses to Blackwood (if the Regulating Authority mandates such disclosure). That seems a reasonable approach. Of course, it may lead to arbitrariness, but I remain of the opinion that if I tell the opponents "5D is my partner's preferred minor" and they ask "yes, but how many aces has he shown?" I don't have to tell them. After all, I don't know how many aces he's shown; or if I do, I must proceed as though I don't. Why should my opponents not have to proceed similarly? David Burn From svenpran at online.no Mon Nov 24 01:54:10 2008 From: svenpran at online.no (Sven Pran) Date: Mon, 24 Nov 2008 01:54:10 +0100 Subject: [blml] laissez faire In-Reply-To: References: <49227C4E.3050007@skynet.be> <000001c94a0b$1cc70330$56550990$@com> <000d01c94ad2$b54bc880$15844c59@chello.pl> <4925C17B.1010706@talktalk.net> <49266B37.90306@skynet.be> <000f01c94bba$ecc985c0$15844c59@chello.pl> <4926C496.6080904@skynet.be> <003901c94c47$a1541140$15844c59@chello.pl> <4927E7CC.4030303@skynet.be> <001f01c94cad$4bf57340$15844c59@chello.pl> <000001c94cde$e66eedd0$b34cc970$@no> <000301c94d8e$c1c90000$455b0000$@no> <000401c94d96$19f416f0$4ddc44d0$@no> Message-ID: <000901c94dcf$29472ff0$7bd58fd0$@no> On Behalf Of Robert Frick .......... > Okay. Just to make sure. You of course tell them that the explanation of 2 > Di was wrong. But you are saying that you tell them only that 3 Sp showed > spades. > > And you have clarified that there are two sets of partnership agreements > here. Are they entitled to the partnership agreement about the meaning of > 3 Sp over a multi 2Di, or can you just tell them the meaning of 3 Sp over > a weak 2 Di. One thing is certain: A partnership has absolutely no obligation to explain what 3S over multi 2D shows unless multi 2D is included in their partnership understanding. If not then 2D multi can never be a "relevant call" even when partner by mistake understood it that way. Here we have a partnership that uses the multi 2D opening bid except in first hand. Law 40A1(b) requires a partnership "to make available its partnership understandings to opponents before commencing play against them". How this is to be done (including the extent of such information) is a matter of regulation, but I feel that at least first round answers to multi 2D should be included in the information. (I shall accept that my understanding here can be questioned.) Multi 2D is not a relevant call in the situation we discuss so opponents have under Law 20F1 no right to explanation of multi 2D and answers to multi. However, they most certainly have such right under Law 40A1(b), and if the partnership has failed to make this information available before play started they must be ready to give such information no later than at the time partner's mistake of the opening bid is revealed, i.e. during the clarification period. (BTW, I notice with interest that this probably also answers the question on what information opponents is entitled to after revealing that a 4NT bid was request for minor preference but understood as Blackwood.) Regards Sven From richard.hills at immi.gov.au Mon Nov 24 03:17:52 2008 From: richard.hills at immi.gov.au (richard.hills at immi.gov.au) Date: Mon, 24 Nov 2008 13:17:52 +1100 Subject: [blml] The Nebraskan issue [SEC=UNOFFICIAL] In-Reply-To: <49269104.6010904@talktalk.net> Message-ID: F.E. Smith, Lord Birkenhead (1872-1930): "Nature has no cure for this sort of madness [Bolshevism], though I have known a legacy from a rich relative work wonders." Nigel Guthrie: >I don't like the kind of simplification that Richard >suggests because it would radically change the nature of >the game. Richard Hills: I, too, oppose radical changes to the nature of the game. My suggestion to abolish vulnerability was merely that of an advocatus diaboli. Nigel Guthrie: >Thus, I objected to the recent change in the scoring for >doubled non-vulnerable undertricks. Richard Hills: Is 1987 recent? A year when Reagan was President, The Simpsons appeared for the first time on television, Kylie Minogue sang The Loco-Motion, and Louis-Victor de Broglie (formulator of wave mechanics, hence winner of the 1929 Nobel Prize for Physics) passed away. Nigel Guthrie: >This made the game less skilful, interesting and exciting >by discouraging sacrificing. It reduced the scope for >judgement, especially at the slam-level..... Richard Hills: This made the game more skilful, interesting and exciting by discouraging sacrificing. It increased the scope for homework on bidding systems, especially at the slam-level. (Copies of my version of Professor Roy Kerr's Symmetric Relay system emailed on request.) Best wishes Richard Hills, Aqua 5, workstation W551 Telephone: 02 6223 8453 Email: richard.hills at immi.gov.au Recruitment Section & DIAC Social Club movie tickets -------------------------------------------------------------------- Important Notice: If you have received this email by mistake, please advise the sender and delete the message and attachments immediately. This email, including attachments, may contain confidential, sensitive, legally privileged and/or copyright information. Any review, retransmission, dissemination or other use of this information by persons or entities other than the intended recipient is prohibited. DIAC respects your privacy and has obligations under the Privacy Act 1988. The official departmental privacy policy can be viewed on the department's website at www.immi.gov.au. See: http://www.immi.gov.au/functional/privacy.htm --------------------------------------------------------------------- From Hermandw at skynet.be Mon Nov 24 09:20:55 2008 From: Hermandw at skynet.be (Herman De Wael) Date: Mon, 24 Nov 2008 09:20:55 +0100 Subject: [blml] laissez faire In-Reply-To: <000001c94dc4$cbb14150$6313c3f0$@com> References: <49227C4E.3050007@skynet.be> <000001c94a0b$1cc70330$56550990$@com> <000d01c94ad2$b54bc880$15844c59@chello.pl> <4925C17B.1010706@talktalk.net> <49266B37.90306@skynet.be> <000f01c94bba$ecc985c0$15844c59@chello.pl> <4926C496.6080904@skynet.be> <003901c94c47$a1541140$15844c59@chello.pl> <4927E7CC.4030303@skynet.be> <001f01c94cad$4bf57340$15844c59@chello.pl> <000001c94cde$e66eedd0$b34cc970$@no> <000301c94d8e$c1c90000$455b0000$@no> <000401c94d96$19f416f0$4ddc44d0$@no> <000001c94dc4$cbb14150$6313c3f0$@com> Message-ID: <492A63E7.3070500@skynet.be> Very bad argument, David: David Burn wrote: > > That seems a reasonable approach. Of course, it may lead to arbitrariness, > but I remain of the opinion that if I tell the opponents "5D is my partner's > preferred minor" and they ask "yes, but how many aces has he shown?" I don't > have to tell them. After all, I don't know how many aces he's shown; or if I > do, I must proceed as though I don't. Why should my opponents not have to > proceed similarly? > David, you do know how many aces he has shown! And while you are indeed not allowed to bid or play as though you know it, when you write "proceed" you enlarge the meaning of L16 into incorrect territory. So while you may be right in the sense that Grattan has allowed the WBFLC to write down this silly rule, you are absolutely incorrect if you think this rule has any other basis in law than that WBFLC decision. > David Burn > Herman. From nigelguthrie at talktalk.net Mon Nov 24 10:34:11 2008 From: nigelguthrie at talktalk.net (Nigel Guthrie) Date: Mon, 24 Nov 2008 09:34:11 +0000 Subject: [blml] laissez faire In-Reply-To: References: <49227C4E.3050007@skynet.be> <000001c94a0b$1cc70330$56550990$@com> <000d01c94ad2$b54bc880$15844c59@chello.pl> <4925C17B.1010706@talktalk.net> <49266B37.90306@skynet.be> <000f01c94bba$ecc985c0$15844c59@chello.pl> <4926C496.6080904@skynet.be> <003901c94c47$a1541140$15844c59@chello.pl> <4927E7CC.4030303@skynet.be> <001f01c94cad$4bf57340$15844c59@chello.pl> <000001c94cde$e66eedd0$b34cc970$@no> <000301c94d8e$c1c90000$455b0000$@no> <000401c94d96$19f416f0$4ddc44d0$@no> Message-ID: <492A7513.6010700@talktalk.net> [Robert Frick] Okay. Just to make sure. You of course tell them that the explanation of 2 Di was wrong. But you are saying that you tell them only that 3 Sp showed spades. And you have clarified that there are two sets of partnership agreements here. Are they entitled to the partnership agreement about the meaning of 3 Sp over a multi 2Di, or can you just tell them the meaning of 3 Sp over a weak 2 Di. [Nigel] IMO: Partner says your 2D meant something else. This is unauthorised information to you. It makes it likely that 3S also has an abnormal meaning. This is more UI to you. If you may be in a better position than opponents to interpret this or any subsequent UI, then you should divulge such inferences to opponents (or the director, if you end up defending). From rfrick at rfrick.info Mon Nov 24 22:15:08 2008 From: rfrick at rfrick.info (Robert Frick) Date: Mon, 24 Nov 2008 16:15:08 -0500 Subject: [blml] laissez faire In-Reply-To: <000001c94dc4$cbb14150$6313c3f0$@com> References: <49227C4E.3050007@skynet.be> <000001c94a0b$1cc70330$56550990$@com> <000d01c94ad2$b54bc880$15844c59@chello.pl> <4925C17B.1010706@talktalk.net> <49266B37.90306@skynet.be> <000f01c94bba$ecc985c0$15844c59@chello.pl> <4926C496.6080904@skynet.be> <003901c94c47$a1541140$15844c59@chello.pl> <4927E7CC.4030303@skynet.be> <001f01c94cad$4bf57340$15844c59@chello.pl> <000001c94cde$e66eedd0$b34cc970$@no> <000301c94d8e$c1c90000$455b0000$@no> <000401c94d96$19f416f0$4ddc44d0$@no> <000001c94dc4$cbb14150$6313c3f0$@com> Message-ID: Thanks for answering. Because you also offered arguments.... > [RF] > > And you have clarified that there are two sets of partnership agreements > here. > > [DALB] > > I am not sure that this is what Sven has said, but of course he can > answer > for himself. For myself, there is only one "set of partnership > agreements" - > we do not "agree" to forget our system. There is a set of partnership agreements about bidding over the multi-2Di, including the meaning of 3S. To me, they don't just disappear on the auctions they aren't used. > > [RF] > > Are they entitled to the partnership agreement about the meaning of 3 Sp > over a multi 2Di, or can you just tell them the meaning of 3 Sp over a > weak > 2 Di? > > [DALB] > > Others have said that in the canonical example of 4NT being minors but > taken > as Blackwood, the opponents are entitled to know what a 5D response would > have meant if 4NT had been Blackwood. To me, this makes no sense. > Suppose I > alert a 2C response to 1NT and tell the opponents that it is a puppet to > 2D, > which I must now bid. Are they entitled to know what 2D would have meant > if > 2C had been Stayman? I can see no reason why they should be. It is irrelvant what 2D would mean if 2C had been Stayman. Law 20F1 mentions "relevant" twice. It doesn't seem fair to choose a situation in which the information the opponents request is obviously irrelevant, and try to generalize that answer to a situation where many people would think the information is relevant. > > Grattan has said that the opponents are allowed (a different thing > altogether from "entitled") to know what a 5D response to Blackwood would > mean if I choose to tell them. Moreover, if they can somehow glean that > the > 5D bidder was responding to Blackwood and not giving preference between > the > minors, they are allowed - and perhaps even entitled - to consult our > pre-event disclosure as to the meanings of responses to Blackwood (if the > Regulating Authority mandates such disclosure). Most people here (ACBL-land) just write "Gerber" or "RKC" on their card. They are very unlikely to write specialized or even nonspecialized responses to conventional bids. > > That seems a reasonable approach. Of course, it may lead to > arbitrariness, > but I remain of the opinion that if I tell the opponents "5D is my > partner's > preferred minor" and they ask "yes, but how many aces has he shown?" I > don't > have to tell them. After all, I don't know how many aces he's shown; or > if I > do, I must proceed as though I don't. Why should my opponents not have to > proceed similarly? This argument does not work. If you had bid 4NT erroneously thinking it was for the minors, and partner correctly described it as being Smithtown Blackwood, everyone agrees that the opponents are entitled to know the meaning of your partner's 5 Di bid even though you must proceed as though you do not. Bob > > David Burn > > > _______________________________________________ > blml mailing list > blml at amsterdamned.org > http://www.amsterdamned.org/mailman/listinfo/blml From richard.hills at immi.gov.au Tue Nov 25 01:56:47 2008 From: richard.hills at immi.gov.au (richard.hills at immi.gov.au) Date: Tue, 25 Nov 2008 11:56:47 +1100 Subject: [blml] Lazy fair [SEC=UNOFFICIAL] In-Reply-To: Message-ID: Marcus Tullius Cicero (106-43 BCE): "Salus populi suprema est lex." Robert Frick wrote: >>wherein I attempt to be practical. [big snip of sweeping statements] >>Directors do not punish either choice. Richard Hills responds: Correct, Directors do no longer punish. One "practical" change in the 2007 Lawbook is that Directors now rectify instead. Robert Frick wrote: >>I would guess that there has never been a procedural >>penalty for this choice. Richard Hills responds: Incorrect. Read the EBU Appeals Casebooks. http://www.ebu.co.uk/lawsandethics/misc/publications.htm Robert Frick wrote: >>I would be even more confident that no director has >>ever rectified for the noncreation of UI when a >>player choose Option 1. Herman De Wael concurred: >And that will mean creating a sort of construction >that no TD will be able to pronounce. "You have broken >the law which requires you to give UI to your partner, >so now we will award you a score as if you had given >such UI, which means that your partner will not be >allowed to take that particular action." With that >particular action one taken by this player in the >absence of UI. Try explaining that there are logical >alternatives to an action that a player actually took >without it being suggested to him! Law 75A (first two sentences): "Whether or not North's explanation is a correct statement of partnership agreement, South, having heard North's explanation, knows that his own 2D bid has been misinterpreted. This knowledge is 'unauthorised information' (see Law 16A), so South must be careful to avoid taking any advantage from that unauthorized information (see Law 73C)." Richard Hills: If: (a) North deliberately infracts Law (or unintentionally misunderstands Law) and chooses to misexplain the pre- existing mutual partnership understanding, so that (b) South without Law 75A UI constraint triumphantly bids to 7S for +2210, but (c) South with Law 75A UI constraint (after a hypothetically accurate explanation by North) would have ingloriously bid to 7NTxx for -7600, then Law 40B4: "A side that is damaged as a consequence of its opponents' failure to provide disclosure of the meaning of a call or play as these laws require, is entitled to rectification through the award of an adjusted score." So as Director I would provide the non-offending side with their **entitled** rectification by changing the score from +2210 to -7600. There is not any technical difficulty in assessing logical alternatives, since LAs after UI which _should_ have occurred are no more (and no less) difficult to assess than LAs after UI which _did_ occur. As TD I would apply the mildest PP of education against an inexperienced North who misunderstood Law. As TD I would refer a highly experienced recidivist North to the local Conduct and Ethics Committee under Law 81C7. David Burn: >>>...I have no doubt that in his role as a Director, >>>Herman will abide by the WBFLC's decision as >>>conscientiously and as meticulously as he does >>>everything else... Richard Hills: I fully agree with David in acknowledging Herman De Wael's integrity as a Director. But... I have major doubts (derived from Herman De Wael's own statements) that in his role as a **player**, Herman will abide by the WBFLC's decision as conscientiously and as meticulously as he does everything else. Off-topic: Some years ago the Queensland Bridge Association suspended a player who was also a leading Director. In theory that player could have continued Directing while banned from playing. In practice expensive legal action was undertaken instead at the Queensland Supreme Court -- a warning to players, Directors and bridge bureaucrats to conscientiously and meticulously abide by rules and procedures. Best wishes Richard Hills, Aqua 5, workstation W550 Telephone: 02 6223 8453 Email: richard.hills at immi.gov.au Recruitment Section & DIAC Social Club movie tickets -------------------------------------------------------------------- Important Notice: If you have received this email by mistake, please advise the sender and delete the message and attachments immediately. This email, including attachments, may contain confidential, sensitive, legally privileged and/or copyright information. Any review, retransmission, dissemination or other use of this information by persons or entities other than the intended recipient is prohibited. DIAC respects your privacy and has obligations under the Privacy Act 1988. The official departmental privacy policy can be viewed on the department's website at www.immi.gov.au. See: http://www.immi.gov.au/functional/privacy.htm --------------------------------------------------------------------- From jfusselman at gmail.com Tue Nov 25 02:56:05 2008 From: jfusselman at gmail.com (Jerry Fusselman) Date: Mon, 24 Nov 2008 19:56:05 -0600 Subject: [blml] Lazy fair [SEC=UNOFFICIAL] In-Reply-To: References: Message-ID: <2b1e598b0811241756g6eb40a22m6b8264ef06da1dd9@mail.gmail.com> > > Incorrect. Read the EBU Appeals Casebooks. > http://www.ebu.co.uk/lawsandethics/misc/publications.htm > This is no refutation unless and until someone supplies a specific case. From richard.hills at immi.gov.au Tue Nov 25 04:06:30 2008 From: richard.hills at immi.gov.au (richard.hills at immi.gov.au) Date: Tue, 25 Nov 2008 14:06:30 +1100 Subject: [blml] Lazy fair [SEC=UNOFFICIAL] In-Reply-To: <2b1e598b0811241756g6eb40a22m6b8264ef06da1dd9@mail.gmail.com> Message-ID: Historian Barbara Tuchman (1912-1989), "August 1914": "No more distressing moment can ever face a British government than that which requires it to come to a hard, fast and specific decision." Jerry Fusselman: >This is no refutation unless and until someone supplies a >specific case. Swiss Pairs (matchpoint pairs converted to victory points) Dlr: East Vul: East-West The bidding has gone: WEST NORTH EAST SOUTH --- --- Pass Pass 2D (1) 2S 3C 3H 4C Pass 4D Pass 5C ? (1) Strong, either single-suited or strong balanced You, North, hold: AQ7652 QT832 76 --- What call do you make? What call didn't you make last round? Best wishes Richard Hills, Aqua 5, workstation W550 Telephone: 02 6223 8453 Email: richard.hills at immi.gov.au Recruitment Section & DIAC Social Club movie tickets -------------------------------------------------------------------- Important Notice: If you have received this email by mistake, please advise the sender and delete the message and attachments immediately. This email, including attachments, may contain confidential, sensitive, legally privileged and/or copyright information. Any review, retransmission, dissemination or other use of this information by persons or entities other than the intended recipient is prohibited. DIAC respects your privacy and has obligations under the Privacy Act 1988. The official departmental privacy policy can be viewed on the department's website at www.immi.gov.au. See: http://www.immi.gov.au/functional/privacy.htm --------------------------------------------------------------------- From daisy_duck at btopenworld.com Tue Nov 25 04:34:46 2008 From: daisy_duck at btopenworld.com (Stefanie Rohan) Date: Tue, 25 Nov 2008 03:34:46 -0000 Subject: [blml] laissez faire References: <49227C4E.3050007@skynet.be> <000001c94a0b$1cc70330$56550990$@com> <000d01c94ad2$b54bc880$15844c59@chello.pl> <4925C17B.1010706@talktalk.net><49266B37.90306@skynet.be> <000f01c94bba$ecc985c0$15844c59@chello.pl> <4926C496.6080904@skynet.be> <003901c94c47$a1541140$15844c59@chello.pl> <4927E7CC.4030303@skynet.be> <001f01c94cad$4bf57340$15844c59@chello.pl> <000001c94cde$e66eedd0$b34cc970$@no> <000301c94d8e$c1c90000$455b0000$@no> <000401c94d96$19f416f0$4ddc44d0$@no> <000001c94dc4$cbb14150$6313c3f0$@com> Message-ID: <7FBFC8D22B2248D580B17051DEA72D04@stefanie> > Others have said that in the canonical example of 4NT being minors but > taken > as Blackwood, the opponents are entitled to know what a 5D response would > have meant if 4NT had been Blackwood. To me, this makes no sense. Suppose > I > alert a 2C response to 1NT and tell the opponents that it is a puppet to > 2D, > which I must now bid. Are they entitled to know what 2D would have meant > if > 2C had been Stayman? I can see no reason why they should be. This is incorrect. The opponents are entitled to know the meanings of bids you have not made -- see 20F1. > That seems a reasonable approach. Of course, it may lead to arbitrariness, > but I remain of the opinion that if I tell the opponents "5D is my > partner's > preferred minor" and they ask "yes, but how many aces has he shown?" I > don't > have to tell them. No, but if they want to know your Blackwood responses you must tell them. > After all, I don't know how many aces he's shown; or if I > do, I must proceed as though I don't. Why should my opponents not have to > proceed similarly? Because they are not constrained by UI. Stefanie Rohan London, England From jfusselman at gmail.com Tue Nov 25 04:44:18 2008 From: jfusselman at gmail.com (Jerry Fusselman) Date: Mon, 24 Nov 2008 21:44:18 -0600 Subject: [blml] Lazy fair [SEC=UNOFFICIAL] In-Reply-To: References: <2b1e598b0811241756g6eb40a22m6b8264ef06da1dd9@mail.gmail.com> Message-ID: <2b1e598b0811241944s2f4ff7dfl95d1093cb7ddb162@mail.gmail.com> > > What call do you make? > What call didn't you make last round? > Golly, Richard, if you wish to support your statement that Robert is "incorrect" in his guess that X has never occurred, all you need to do is supply one example where X has occurred. You initially pretended to have evidence, and maybe you do, but if you have none, instead of sending us through some labyrinth, would you consider apologizing to Robert? Jerry Fusselman From richard.hills at immi.gov.au Tue Nov 25 06:49:48 2008 From: richard.hills at immi.gov.au (richard.hills at immi.gov.au) Date: Tue, 25 Nov 2008 16:49:48 +1100 Subject: [blml] Lazy fair [SEC=UNOFFICIAL] In-Reply-To: <2b1e598b0811241944s2f4ff7dfl95d1093cb7ddb162@mail.gmail.com> Message-ID: Lord Halifax, "the Trimmer" (1633-1695): "Most men make little use of their speech than to give evidence against their own understanding." Jerry Fusselman gollied: >Golly, Richard, if you wish to support your statement that Robert >is "incorrect" in his guess that X has never occurred, all you >need to do is supply one example where X has occurred. You >initially pretended to have evidence, and maybe you do, but if >you have none, instead of sending us through some labyrinth, >would you consider apologizing to Robert? Grattan Endicott, non-labyrinthine evidence, November 2002: http://www.amsterdamned.org/pipermail/blml/2002-November/002114.html Best wishes Richard Hills, Aqua 5, workstation W550 Telephone: 02 6223 8453 Email: richard.hills at immi.gov.au Recruitment Section & DIAC Social Club movie tickets -------------------------------------------------------------------- Important Notice: If you have received this email by mistake, please advise the sender and delete the message and attachments immediately. This email, including attachments, may contain confidential, sensitive, legally privileged and/or copyright information. Any review, retransmission, dissemination or other use of this information by persons or entities other than the intended recipient is prohibited. DIAC respects your privacy and has obligations under the Privacy Act 1988. The official departmental privacy policy can be viewed on the department's website at www.immi.gov.au. See: http://www.immi.gov.au/functional/privacy.htm --------------------------------------------------------------------- From harald.skjaran at gmail.com Tue Nov 25 08:28:54 2008 From: harald.skjaran at gmail.com (=?UTF-8?Q?Harald_Skj=C3=A6ran?=) Date: Tue, 25 Nov 2008 08:28:54 +0100 Subject: [blml] laissez faire In-Reply-To: <7FBFC8D22B2248D580B17051DEA72D04@stefanie> References: <49227C4E.3050007@skynet.be> <001f01c94cad$4bf57340$15844c59@chello.pl> <000001c94cde$e66eedd0$b34cc970$@no> <000301c94d8e$c1c90000$455b0000$@no> <000401c94d96$19f416f0$4ddc44d0$@no> <000001c94dc4$cbb14150$6313c3f0$@com> <7FBFC8D22B2248D580B17051DEA72D04@stefanie> Message-ID: On 25/12/2008, Stefanie Rohan wrote: > > Others have said that in the canonical example of 4NT being minors but > > taken > > as Blackwood, the opponents are entitled to know what a 5D response would > > have meant if 4NT had been Blackwood. To me, this makes no sense. Suppose > > I > > alert a 2C response to 1NT and tell the opponents that it is a puppet to > > 2D, > > which I must now bid. Are they entitled to know what 2D would have meant > > if > > 2C had been Stayman? I can see no reason why they should be. > > This is incorrect. The opponents are entitled to know the meanings of bids > you have not made -- see 20F1. 20F1 says "he is entitled to know about calls actually made, about relevant alternative calls available that were not made, and ....". For a pair NOT applying the Stayman convention, replies to 2C Stayman are neither relevant nor available. Thus they're not entitled to any information regarding Stayman responses. In fact, not using Stayman, the bidding pair could be unaware of the convention (unlikely!). Opponents are only entitled to information about the calls actually made and alternative calls in opponents methods. > > > Stefanie Rohan > London, England > > > _______________________________________________ > blml mailing list > blml at amsterdamned.org > http://www.amsterdamned.org/mailman/listinfo/blml > -- Kind regards, Harald Skj?ran From rfrick at rfrick.info Tue Nov 25 10:14:37 2008 From: rfrick at rfrick.info (Robert Frick) Date: Tue, 25 Nov 2008 04:14:37 -0500 Subject: [blml] laissez faire In-Reply-To: References: <49227C4E.3050007@skynet.be> <001f01c94cad$4bf57340$15844c59@chello.pl> <000001c94cde$e66eedd0$b34cc970$@no> <000301c94d8e$c1c90000$455b0000$@no> <000401c94d96$19f416f0$4ddc44d0$@no> <000001c94dc4$cbb14150$6313c3f0$@com> <7FBFC8D22B2248D580B17051DEA72D04@stefanie> Message-ID: On Tue, 25 Nov 2008 02:28:54 -0500, Harald Skj?ran wrote: > On 25/12/2008, Stefanie Rohan wrote: >> > Others have said that in the canonical example of 4NT being minors but >> > taken >> > as Blackwood, the opponents are entitled to know what a 5D response >> would >> > have meant if 4NT had been Blackwood. To me, this makes no sense. >> Suppose >> > I >> > alert a 2C response to 1NT and tell the opponents that it is a puppet >> to >> > 2D, >> > which I must now bid. Are they entitled to know what 2D would have >> meant >> > if >> > 2C had been Stayman? I can see no reason why they should be. >> >> This is incorrect. The opponents are entitled to know the meanings of >> bids >> you have not made -- see 20F1. > > 20F1 says "he is entitled to know about calls actually made, about > relevant alternative calls available that were not made, and ....". ...relevant inferences from the choice of action where these are matters of partnership understanding." > For a pair NOT applying the Stayman convention, replies to 2C Stayman > are neither relevant nor available. Thus they're not entitled to any > information regarding Stayman responses. In fact, not using Stayman, > the bidding pair could be unaware of the convention (unlikely!). I don't think you can talk about a situation where you have no partnership agreement and generalize to a situation where you do have a partnership agreement. In my example, there was a partnership understanding for the meaning of 3 Sp over the multi 2Di openener and the meaning of 3 Sp over the weak 2 Di opener. > > Opponents are only entitled to information about the calls actually > made and alternative calls in opponents methods. > >> >> >> Stefanie Rohan >> London, England >> >> >> _______________________________________________ >> blml mailing list >> blml at amsterdamned.org >> http://www.amsterdamned.org/mailman/listinfo/blml >> > > From rfrick at rfrick.info Tue Nov 25 11:09:46 2008 From: rfrick at rfrick.info (Robert Frick) Date: Tue, 25 Nov 2008 05:09:46 -0500 Subject: [blml] can't think of a title In-Reply-To: References: Message-ID: > > http://www.amsterdamned.org/pipermail/blml/2002-November/002114.html "That reasoning may be common sense but it ignores bridge law." "Common morality may require declarer to reveal, without deceit, what she holds, but bridge law requires something quite different." "thus, the absurd requirement that she give an accurate explanation of an agreement she has honestly forgotten." Am I the only one who sees a problem here? If a law doesn't fit common sense, doesn't fit common morality, and if it leads to absurd requirements, aren't you supposed to fix it? From rfrick at rfrick.info Tue Nov 25 11:44:23 2008 From: rfrick at rfrick.info (Robert Frick) Date: Tue, 25 Nov 2008 05:44:23 -0500 Subject: [blml] Lazy fair [SEC=UNOFFICIAL] In-Reply-To: References: Message-ID: On Mon, 24 Nov 2008 19:56:47 -0500, wrote: > Marcus Tullius Cicero (106-43 BCE): > > "Salus populi suprema est lex." > > Robert Frick wrote: > >>> wherein I attempt to be practical. > > [big snip of sweeping statements] > >>> Directors do not punish either choice. > > Richard Hills responds: > > Correct, Directors do no longer punish. One "practical" > change in the 2007 Lawbook is that Directors now > rectify instead. > > Robert Frick wrote: > >>> I would guess that there has never been a procedural >>> penalty for this choice. > > Richard Hills responds: > > Incorrect. Read the EBU Appeals Casebooks. > http://www.ebu.co.uk/lawsandethics/misc/publications.htm Richard later put this as presumably the full link http://www.amsterdamned.org/pipermail/blml/2002-November/002114.html This doesn't count as an example. It wasn't a procedural penalty. She was rectified for misleading information. Directors don't give procedural penalties for forgetting one's convention. Everyone agrees that it is okay to give a misleading explanation if you don't know any better. Then you suffer the consequences. The problem occurs when a WBFLC opinion says that you are required to provide a misleading explanation which you know is misleading, and you are not required you to divulge the partnership agreements your partner was using when he made his bid. > > Robert Frick wrote: > >>> I would be even more confident that no director has >>> ever rectified for the noncreation of UI when a >>> player choose Option 1. > > Herman De Wael concurred: > >> And that will mean creating a sort of construction >> that no TD will be able to pronounce. "You have broken >> the law which requires you to give UI to your partner, >> so now we will award you a score as if you had given >> such UI, which means that your partner will not be >> allowed to take that particular action." With that >> particular action one taken by this player in the >> absence of UI. Try explaining that there are logical >> alternatives to an action that a player actually took >> without it being suggested to him! > > Law 75A (first two sentences): > > "Whether or not North's explanation is a correct > statement of partnership agreement, South, having heard > North's explanation, knows that his own 2D bid has been > misinterpreted. This knowledge is 'unauthorised > information' (see Law 16A), so South must be careful to > avoid taking any advantage from that unauthorized > information (see Law 73C)." Actually, it seems to be blml majority opinion that a player may use UI to provide the proper explanation of partner's bid to opponents. I would be happy to discuss that point. (You bid 4NT for the minors, partner explains it as Blackwood, you realize that partner is correct and it is Blackwood. Now the opps ask you to explain his 5 Di bid.) > > Richard Hills: > > If: > > (a) North deliberately infracts Law (or unintentionally > misunderstands Law) and chooses to misexplain the pre- > existing mutual partnership understanding, so that > > (b) South without Law 75A UI constraint triumphantly > bids to 7S for +2210, but > > (c) South with Law 75A UI constraint (after a > hypothetically accurate explanation by North) would > have ingloriously bid to 7NTxx for -7600, then > > Law 40B4: > > "A side that is damaged as a consequence of its > opponents' failure to provide disclosure of the meaning > of a call or play as these laws require, is entitled to > rectification through the award of an adjusted score." > > So as Director I would provide the non-offending side > with their **entitled** rectification by changing the > score from +2210 to -7600. There is not any technical > difficulty in assessing logical alternatives, since LAs > after UI which _should_ have occurred are no more (and > no less) difficult to assess than LAs after UI which > _did_ occur. > > As TD I would apply the mildest PP of education against > an inexperienced North who misunderstood Law. > > As TD I would refer a highly experienced recidivist > North to the local Conduct and Ethics Committee under > Law 81C7. I was guessing that no one had ever done this, and it sounds like you have not. I would have also guessed that no one would do this. I don't want to dare you to do this just to prove me wrong, because I don't think it is a wise thing to do. I will concede that someone might do this. The fact remains that right now this is not common director practice, so much so that no one has found an example. > > David Burn: > >>>> ...I have no doubt that in his role as a Director, >>>> Herman will abide by the WBFLC's decision as >>>> conscientiously and as meticulously as he does >>>> everything else... > > Richard Hills: > > I fully agree with David in acknowledging Herman De > Wael's integrity as a Director. > > But... > > I have major doubts (derived from Herman De Wael's own > statements) that in his role as a **player**, Herman > will abide by the WBFLC's decision as conscientiously > and as meticulously as he does everything else. More Herman bashing? My impression is that dWS involves attempting to minimize the creation of UI. In my examples, the request for explanation occurs after the auction is over and partner is either declarer or dummy. So the creation of UI is not an issue. Bob From agot at ulb.ac.be Tue Nov 25 12:41:22 2008 From: agot at ulb.ac.be (Alain Gottcheiner) Date: Tue, 25 Nov 2008 12:41:22 +0100 Subject: [blml] Lazy fair [SEC=UNOFFICIAL] In-Reply-To: References: Message-ID: <492BE462.6040307@ulb.ac.be> Robert Frick a ?crit : > > Everyone agrees that it is okay to give a misleading explanation if you > don't know any better. Then you suffer the consequences. > AG : be careful ! This might legitimate the dWS procedure. Your partner exlpains so?mething else than what you thought you were playing. You're in doubt. You are compelled to explain your agreements, but you're not sure anymore what they are. Eventually, you say to yourself "nah, he must be right" and you explain his way. Too bad if you're wrong. Doe you say everyone agree this is allowed ? Yet it's pure hardcore dWS. Best regards Alain From Hermandw at skynet.be Tue Nov 25 13:43:58 2008 From: Hermandw at skynet.be (Herman De Wael) Date: Tue, 25 Nov 2008 13:43:58 +0100 Subject: [blml] Two alternatives for the WBF? No, three Message-ID: <492BF30E.8020808@skynet.be> Over the past few days, I've come to realise why there is such an animosity over the DWS. I believe that many of us (me included) do not realise that the WBF actually has three possible choices. Most of you probably think I am in favour of one choice, while I am actually in favour of another one. While I may believe that you are in favour of a third one, when perhaps that second choice might be good enough for you as well. Allow me to explain. First of all, let's get rid of those silly acronyms DWS and MS. Let's focus on the choice the player at the table must make, and give sensible names to the options available. Basically there are two options: - a player can explain his partner's call in the same system as partner has previously explained his own call. Let me call that the CE, the Consistent Explanation; - or the player can explain his partner's call in the system that he himself believes the pair are playing, let's call that the SE, the Systemic Explanation. There is a third option, which is the one Stephanie proposed, and which explains according to the own system, even when the player has meanwhile realised that his partner's explanation was the correct one after all. That option basically gives the same explanation as the SE, and I shall not use it furthermore. These two options are mutually exclusive. Anything that is added to the CE which reveals that the previous explanation was not correct, triggers in opponents a series of questions which will ultimately lead in both the CE and the SE being known to them, and UI being given to partner. So the player has two choices. But the WBF has twice a choice to make: The WBF can outlaw or accept the CE. The WBF can outlaw or accept the SE. Remember that since neither explanation conforms to all the laws in the book, there is no option to declare the explanation "legal". There is merely an option to declare it "acceptable". The opposite of accepting is outlawing. Combined, these options make for four alternatives, one of which is clearly nonsensical. To outlaw both explanations means a player has no more options left. He will have to choose one option anyway, and to call either option unacceptable is ridiculous. So there are three alternatives: A) to outlaw the CE B) to outlaw the SE C) to call both the CE and SE acceptable I believe many of you think I advocate option B. I have often stated what is wrong with the SE, and this may make you believe I would like to see it outlawed. I don't. I don't think there is any reason to outlaw the SE. Just apply the MI and UI laws and everyone is happy. I may have believed that most of you advocate option A. You have often told us why you believe the CE is unethical and wrong. But maybe you don't think it should be outlawed per se. After all, maybe some of you have meanwhile seen that apart from applying the UI and MI laws on the CE, nothing else can be done. Which is why I think we should work to see the WBF accept alternative C, which is to allow players to make their own choices, standing ready to apply MI and UI laws over the choices made. Comments? From ziffbridge at t-online.de Tue Nov 25 14:49:22 2008 From: ziffbridge at t-online.de (Matthias Berghaus) Date: Tue, 25 Nov 2008 14:49:22 +0100 Subject: [blml] can't think of a title In-Reply-To: References: Message-ID: <492C0262.7060204@t-online.de> Robert Frick schrieb: >> http://www.amsterdamned.org/pipermail/blml/2002-November/002114.html > > > > "That reasoning may be common sense but it ignores bridge law." > > "Common morality may require declarer to reveal, without deceit, what she > holds, but bridge law requires something quite different." > > "thus, the absurd requirement that she give an accurate explanation of an > agreement she has honestly forgotten." > > > Am I the only one who sees a problem here? I have no idea. What I can say is that I do not see any problem here. > If a law doesn't fit common > sense, doesn't fit common morality, and if it leads to absurd > requirements, aren't you supposed to fix it? Bridge knows the "double shot", where - after an infraction by the opponents - you try some odds-against scheme, and if it fails you try to get it back from the TD. American Football knows a ploy where the quarterback tries some high-risk play in a situation where he is sure that a penalty is about to be called against the opposing team. If his play succeeds he can waive the penalty, if it doesn't work he accepts the penalty. This is a risk-free double-shot (under the assumption that the penalty goes against the opposing team). This is strictly legitimate in AF, but not in Bridge. Leaving aside the question which point of view (if any) fits "common morality" here, it is clear that at least one of this views has to be in conflict with common morality, as they are absolutely contrary to each other. So which one is to be fixed? Or could it be that different sets of rules generate different "moralities"? The rules of Bridge say that someone who conforms to the rules of the game does no wrong. So where is the problem? In another game called "Bluff" or "Liar's dice" the point and only strategy is to lie convincingly and confidently. Now what? Do you want to ban the game because it is against "common morality", whatever that may be? Don't get me wrong, I do not want to argue that there is no such thing as morality, just that it is not "common". Sense is never common, either. We have to part with the idea that it is possible to construct a set of rules that will be applauded by every player on this planet. This will not happen. The number of people who will ever be unreservedly happy with every detail of a set of rules as complex as those of Bridge will never exceed 1, and it is pretty unlikely to reach 1 as long as it is done by commitee. To return to your question: What is more "moral"? To tell your agreements, even if you think partner is off the rails, or to try to describe his hand, even if you know that this is not your agreement? I do not know, and I do not care. What I care about is what the rules ask me to do. It is a game, it has rules, it is a universe in itself. Morality is a concept from a different universe. Bridge has its own morality. You can go on and state your point, you can try to convince others that your way would be a better way of playing Bridge, and if you succed I will play the game your way, but you should realize that it is a game, and "common morality" has no place there, because that is based on the points of view of all people on this planet (else it wouldn't be common, would it?), most of which never heard of Bridge in their life. > > > _______________________________________________ > blml mailing list > blml at amsterdamned.org > http://www.amsterdamned.org/mailman/listinfo/blml > > From agot at ulb.ac.be Tue Nov 25 15:32:17 2008 From: agot at ulb.ac.be (Alain Gottcheiner) Date: Tue, 25 Nov 2008 15:32:17 +0100 Subject: [blml] can't think of a title In-Reply-To: <492C0262.7060204@t-online.de> References: <492C0262.7060204@t-online.de> Message-ID: <492C0C71.10207@ulb.ac.be> Matthias Berghaus a ?crit : > Bridge knows the "double shot", where - after an infraction by the > opponents - you try some odds-against scheme, and if it fails you try > to get it back from the TD. > American Football knows a ploy where the quarterback tries some > high-risk play in a situation where he is sure that a penalty is about > to be called against the opposing team. If his play succeeds he can > waive the penalty, if it doesn't work he accepts the penalty. This is a > risk-free double-shot (under the assumption that the penalty goes > against the opposing team). This is strictly legitimate in AF, but not > in Bridge. Leaving aside the question which point of view (if any) fits > "common morality" here, it is clear that at least one of this views has > to be in conflict with common morality, as they are absolutely contrary > to each other. So which one is to be fixed? > AG : none. Contexts are different. In Rugby, letting play continue and, if it doesn't help the NOS, walking the cat back to the point of the infraction is the norm. This encourages such high risk behaviour. I guess this is the same at US Football. This isn't the case at bridge, not because of different morality, but simply because it's a game of gaining information, so that one can't simply go back to the point of the infraction and let play continue from there as if further play had never occurred. It's not morality, but efficiency, that governs. Disallowing double shots is merely a way to say 'if you compel us to use an adjusted score while you could have done otherwise, it won't be adjusted in your favor', in the hope of minimizing the number of adjusted scores needed, which is (rightly) seen as good for the game. > > I do not know, and I do not care. What I care about is what the rules > ask me to do. It is a game, it has rules, it is a universe in itself. > AG : so said Levi-Strauss. But then you void of any sense the words 'fair play', which often mean 'not trying to take advantage from the rules'. Many items of rule were added /a posteriori/ to make rues of such-and-such game or sport more in adequation with a 'fair' view of what it should be, e.g. disallowance of injure-prone plays (I don't know the technical English name) in Soccer or of bodyline bowling in Cricket. The 'universe' you mention is made by humans, and humans might wish to change it according to the idea they have of their game. Many do want to see their game as 'fair', and changes are often introduced in this direction, succeding or not. The disallowance of psyches followed the same idea ; only, it was seen to be too impractical Best regards Alain From ziffbridge at t-online.de Tue Nov 25 15:35:26 2008 From: ziffbridge at t-online.de (Matthias Berghaus) Date: Tue, 25 Nov 2008 15:35:26 +0100 Subject: [blml] Two alternatives for the WBF? No, three In-Reply-To: <492BF30E.8020808@skynet.be> References: <492BF30E.8020808@skynet.be> Message-ID: <492C0D2E.4080607@t-online.de> Snipped, not because I disagree with the snipped parts, but to get to the part I want to comment on. Herman De Wael schrieb: > Remember that since neither explanation conforms to all the laws in the > book, there is no option to declare the explanation "legal". There is > merely an option to declare it "acceptable". I have to disagree. Of course I can define some action as legal, even if a rule is broken. Let us assume that I am about to invent some game. It is a simple game, so I call it "Bonk". It has a number of rules, which I shall call "Laws". The precise content of this laws shall be of no concern, the game is not ready to be released to the market. Now Bonk has a law with the number 20, which says that it is forbidden to plurk. It is an infraction to plurk, and may lead to negative consequences to my score. Bonk has another law, numbered 40. Law 40 says that I may plurk, it may even force me to plurk, but only under circumstances laid down in law 40. I make and interpret the rules, and by my definition it is perfectly legal to plurk, but only when law 40 calls for it. The laws say that it is legal, so it is legal. Now you may disagree with my concept of the game. You may (as long as I am willing to listen, or even to discuss) try to convince me that it is a bad idea to legalize plurking, even in the situations defined in law 40. What you cannot do is make plurking illegal in situations where law 40 says it is legal. It is legal, because I say so.It is my game. It may be a good idea to allow plurking, it may be a bad idea, but as long as it is my game the rules remain how I make them. You may not like my game, and try to change my mind, but it remains legal because I defined it as legal, until I change it. It so happens that my game has some parallels to Bridge, insofar as in both games someone makes the rules and defines what is legal and what is not. But in Bonk as in Bridge it is a case of definition. It may be a good or a bad definition, but it remains one all the same. As long as the WBF says that it is legal - in certain circumstances - to describe the answer to 4NT as minor-suit preference (even if partner has explained 4NT as Blackwood) it remains legal, because the WBF makes the rules. You make think this unwise and tell them so, you may try to make them change their minds, but as long as they hold certain rights it is their opinion which counts, not yours or mine. We are lucky enough to live in countries where we are free to speak our minds about plurking, about WBF definitions and lots of other things, and we remain free to change your mind, my mind, even the minds of the WBF Laws Committee. As David Burn pointed out i is a good thing that these laws are being discussed, that there are people with different opinions, to get some discussion going. We need discussion. But what we need, too, is the realization that someone has the authority to tell us what the rules presently are, and that they remain that way until someone convinces the lawgivers (by argument, or by political campaign to change the people in power), or they change their mind on their own. Even Chess and Go had rule changes. Best regards Matthias From ziffbridge at t-online.de Tue Nov 25 15:44:38 2008 From: ziffbridge at t-online.de (Matthias Berghaus) Date: Tue, 25 Nov 2008 15:44:38 +0100 Subject: [blml] can't think of a title In-Reply-To: <492C0C71.10207@ulb.ac.be> References: <492C0262.7060204@t-online.de> <492C0C71.10207@ulb.ac.be> Message-ID: <492C0F56.1080007@t-online.de> Alain Gottcheiner schrieb: > Matthias Berghaus a ?crit : >> I do not know, and I do not care. What I care about is what the rules >> ask me to do. It is a game, it has rules, it is a universe in itself. >> > AG : so said Levi-Strauss. But then you void of any sense the words > 'fair play', which often mean 'not trying to take advantage from the > rules'. No. I just maintain that "fair play" can only be defined in the context of the game. Different games, different concepts of "fair". There is no sense in taking concepts from game "A" into game "B", or vice versa. > Many items of rule were added /a posteriori/ to make rues of > such-and-such game or sport more in adequation with a 'fair' view of > what it should be, e.g. disallowance of injure-prone plays (I don't know > the technical English name) in Soccer or of bodyline bowling in Cricket. > The 'universe' you mention is made by humans, and humans might wish to > change it according to the idea they have of their game. Sure. This is what people do. Why not? But the rules cannot be changed by ignoring existing rules, only by convincing the lawmakers. Best regards Matthias > Many do want to > see their game as 'fair', and changes are often introduced in this > direction, succeding or not. > The disallowance of psyches followed the same idea ; only, it was seen > to be too impractical > > > Best regards > > Alain > > _______________________________________________ > blml mailing list > blml at amsterdamned.org > http://www.amsterdamned.org/mailman/listinfo/blml > > From nigelguthrie at talktalk.net Tue Nov 25 16:08:51 2008 From: nigelguthrie at talktalk.net (Nigel Guthrie) Date: Tue, 25 Nov 2008 15:08:51 +0000 Subject: [blml] can't think of a title In-Reply-To: <492C0C71.10207@ulb.ac.be> References: <492C0262.7060204@t-online.de> <492C0C71.10207@ulb.ac.be> Message-ID: <492C1503.90301@talktalk.net> [Alain Gottcheiner] Contexts are different. In Rugby, letting play continue and, if it doesn't help the NOS, walking the cat back to the point of the infraction is the norm. This encourages such high risk behaviour. I guess this is the same at US Football. This isn't the case at bridge, not because of different morality, but simply because it's a game of gaining information, so that one can't simply go back to the point of the infraction and let play continue from there as if further play had never occurred. It's not morality, but efficiency, that governs. Disallowing double shots is merely a way to say 'if you compel us to use an adjusted score while you could have done otherwise, it won't be adjusted in your favor', in the hope of minimizing the number of adjusted scores needed, which is (rightly) seen as good for the game. [Nigel] I don't accept Alain's argument. I don't know the reason for the "Double-shot" rule, but we can all appreciate its main effect: it protects experts against less experienced players. A director is unlikely to successfully accuse an expert of a wild and gambling action. Whereas directors often so classify actions that an ordinary player regards as quite normal. To preserve his chance of redress, the ordinary player may try to avoid profitable but risky actions only to discover later that there was no infraction after all. Finally, this rule (and the daft "Protect yourself" rule) reduce the incentive for ordinary players to report infractions. From agot at ulb.ac.be Tue Nov 25 16:20:33 2008 From: agot at ulb.ac.be (Alain Gottcheiner) Date: Tue, 25 Nov 2008 16:20:33 +0100 Subject: [blml] Two alternatives for the WBF? No, three In-Reply-To: <492C0D2E.4080607@t-online.de> References: <492BF30E.8020808@skynet.be> <492C0D2E.4080607@t-online.de> Message-ID: <492C17C1.20605@ulb.ac.be> Matthias Berghaus a ?crit : > Snipped, not because I disagree with the snipped parts, but to get to > the part I want to comment on. > > Herman De Wael schrieb: > > >> Remember that since neither explanation conforms to all the laws in the >> book, there is no option to declare the explanation "legal". There is >> merely an option to declare it "acceptable". >> > > I have to disagree. Of course I can define some action as legal, even if > a rule is broken. > > Let us assume that I am about to invent some game. It is a simple game, > so I call it "Bonk". It has a number of rules, which I shall call > "Laws". The precise content of this laws shall be of no concern, the > game is not ready to be released to the market. Now Bonk has a law with > the number 20, which says that it is forbidden to plurk. It is an > infraction to plurk, and may lead to negative consequences to my score. > Bonk has another law, numbered 40. Law 40 says that I may plurk, it may > even force me to plurk, but only under circumstances laid down in law > 40. I make and interpret the rules, and by my definition it is perfectly > legal to plurk, but only when law 40 calls for it. The laws say that it > is legal, so it is legal. > AG : the description you give is similar to TFLB, but it shouldn't work so. If L40 says that sometimes plurking is allowed after all, then L20 shouldn't read 'it is forbidden to plurk', but rather 'it is forbidden to plurk except when L40 says otherwise'. And of course, it would be better if L20 was in fact L41 : 'apart from cases mentioned above in L40, it is forbidden to plurk.' For example, the definition of a goalkeeper comes before the law about handling the ball, and the law about handling says 'except by the goalkeeper within his 18 yards box'. Then (only) does the apparent contradiction disappear. > the WBF makes the > rules. You make think this unwise and tell them so, you may try to make > them change their minds, but as long as they hold certain rights it is > their opinion which counts, not yours or mine. > > AG : and surely I can explain them that their rules should be rewritten, even if unchanged, because it is so difficult to interpret them in their current form. As an active wargame- and board game player, I've always considered clarity of the rules one of the most important virtues of a game, especially as there isn't a board of officials to give their most authorized opinion, nor a TD to split hairs and opinions when two wargamers disagree. And somehow, many game conceptors take the players' efforts to signal ambiguities in the rules as help rather than criticism. Websites like BGG are full of fora asking or giving particulars about board game laws. Best regards Alain From agot at ulb.ac.be Tue Nov 25 16:35:05 2008 From: agot at ulb.ac.be (Alain Gottcheiner) Date: Tue, 25 Nov 2008 16:35:05 +0100 Subject: [blml] can't think of a title In-Reply-To: <492C0F56.1080007@t-online.de> References: <492C0262.7060204@t-online.de> <492C0C71.10207@ulb.ac.be> <492C0F56.1080007@t-online.de> Message-ID: <492C1B29.6010109@ulb.ac.be> Matthias Berghaus a ?crit : > I just maintain that "fair play" can only be defined in the context > of the game. Different games, different concepts of "fair". There is no > sense in taking concepts from game "A" into game "B", or vice versa. > AG : right, but taking them from social life into games A and B is somehow needed. A big chunk of the properties section, most notably L74, are concerned with this. The laws of the board game /Illuminati/ says that you're allowed to steal money from the pot, as long as you aren't caught (i.e. if they realize afterwards that your amount of money somehow doesn't fit with the progress of the game, they just can keep mum). But in 99% of boardgames, this is disallowed, and the important part is that *nobody felt necessary to state this in the rules*. Why is it so ? Because there exist something like 'default values' about what is allowed or not. You may use the spot where your oppnent intended to play, if this fits your strategy, but you may not legerdemain his markers out of the board. These commonly admitted limits are what's caller /fair play/. Of course, there are borderline actions, so that some players will consider them as foul play, and others not. For example, I've always felt coalizing against one player is foul play unless 'diplomatic actions' are explicitly allowed by the game, but many think otherwise. Discarding 'natural law' fair play is the best way to make your game a game of crooks and lawyers ... well, perhaps that's what happened to bridge. Best regards Alain From agot at ulb.ac.be Tue Nov 25 16:42:50 2008 From: agot at ulb.ac.be (Alain Gottcheiner) Date: Tue, 25 Nov 2008 16:42:50 +0100 Subject: [blml] can't think of a title In-Reply-To: <492C1503.90301@talktalk.net> References: <492C0262.7060204@t-online.de> <492C0C71.10207@ulb.ac.be> <492C1503.90301@talktalk.net> Message-ID: <492C1CFA.5060701@ulb.ac.be> Nigel Guthrie a ?crit : > [Alain Gottcheiner] > Disallowing double shots is merely a way to say 'if you compel us to use an adjusted score while you could have done otherwise, it won't be adjusted in your favor', in the hope of minimizing the number of adjusted scores needed, which is (rightly) seen as good for the game. > > [Nigel] > I don't accept Alain's argument. I don't know the reason for the "Double-shot" rule, but we can all appreciate its main effect: it protects experts against less experienced players. A director is unlikely to successfully accuse an expert of a wild and gambling action. Whereas directors often so classify actions that an ordinary player regards as quite normal. To preserve his chance of redress, the ordinary player may try to avoid profitable but risky actions only to discover later that there was no infraction after all. Finally, this rule (and the daft "Protect yourself" rule) reduce the incentive for ordinary players to report infractions. > AG : I'm saying there was a goal to this rule. You say you disagree, because its effect is perverse. So what ? The fact that the rule backfired doesn't mean it had no goal. This isn't the first time that a regulation's effects aren't what was intended (ever heard about banking regulation ?). BTW, I don't like the way this 'double shot' prohibition is enforced either. From brian at meadows.pair.com Tue Nov 25 16:50:45 2008 From: brian at meadows.pair.com (brian) Date: Tue, 25 Nov 2008 10:50:45 -0500 Subject: [blml] Two alternatives for the WBF? No, three In-Reply-To: <492C17C1.20605@ulb.ac.be> References: <492BF30E.8020808@skynet.be> <492C0D2E.4080607@t-online.de> <492C17C1.20605@ulb.ac.be> Message-ID: <492C1ED5.90500@meadows.pair.com> Alain Gottcheiner wrote: > AG : and surely I can explain them that their rules should be rewritten, > even if unchanged, because it is so difficult to interpret them in their > current form. > You can, but for as long as the person who (according to Grattan) did the final proofread was actively involved, let alone the chairman, of the subcommittee responsible for producing the new Laws, problems and/or inconsistencies will be less likely to be picked up. I know absolutely sod all about writing bridge laws. I do, however, have a reasonable amount of experience in helping produce other technical documentation. The vast majority of people who have been involved with an extended drafting process will have too great a tendency to see what they THINK has been written, rather than what has ACTUALLY been written. Bridge is in no way special in this regard. Maybe for the next FLB, the WBFLC will have the sense, when they set up their DSC, to nominate someone NOT in any way involved with the drafting to do the final proofread. In the worst case, they could reserve one of their own members for that task. And maybe pigs will fly. Brian. From darkbystry at wp.pl Tue Nov 25 17:00:29 2008 From: darkbystry at wp.pl (Bystry) Date: Tue, 25 Nov 2008 17:00:29 +0100 Subject: [blml] The Nebraskan issue (really about the methods of change) References: <4926FB3B.7060408@skynet.be> Message-ID: <003901c94f16$f0976be0$15844c59@chello.pl> Herman, > > Maciej suggested that we allow instant correction of MI. To me, this is a > > great idea. I would love to try playing bridge this way and see if it > > works. > > > > Because it might be a better way to play bridge. Or it might not. Or, > > third possibility, it might not work as is but it might be changeable into > > something that does work. Or it might just suggest a way to change the > > current system that I otherwise wouldn't have thought of. > > > > I have also already thought of this, as it is one way of solving the > inconsistency of the MS. I even suggested this to Grattan in a private > conversation in Antalya. > > However, I don't think it would work. Just off the top of my head, I > have four objections: > a) it would be difficult to re-educate the players (although that should > not be too big an objection) I don't think so. It's quite natural to object to your partner's misexplanation. You can imagine the player just wanting to shout: "What????". Instant correction of MI would be IMO the easiest for the players to understand and to apply. > b) it would create a huge difference between F2F and screens bridge, > since this would mean that in F2F, all misunderstandings would become > known to opponents, something which is impossible with screens. Also the > on-line version would drift further away from F2F, since there it is > even possible to have wrong explanations without partner even realizing, > creating even less UI than in F2F at present. Playing Bridge with screens is different in many aspects. MI is usually given only to one player and therefore we have some unique cases when redress is given because of inluence that MI-only-to-one-player has on the NOS further understandings of their own bidding. There are generally no such cases in F2F Bridge. And there are more things that differ. So it's not a big deal. In online Bridge you're usually better placed as the NOS (as compared with F2F), because you know about the mixups. So in this aspect it would actually cause both forms to come closer. And an UI aspect is not present in online Bridge (at least not caused by explanations) anyway. Having written that, I don't think it is at all necessary to compare different forms of Bridge. They are different and at least for the close future they'll remain to be different. > c) it would have the same problem as with the MS interpretation: what do > we do with a player who refuses (or forgets) to correct the MI? He will > create for himself an advantage (partner has no UI - opponents no > knowledge of the misunderstanding) which is very difficult to quantify > and punish. I do realize that this is a lesser problem if it is always > doen, lesser than in current MS, but still. And what do you do with a player who refuses (or forgets) to correct MI after the auction? The sanctions will only have to be strengthened. Ok, it would be hard to redress damage coming from noncreation of UI and I already admitted it is a strong argument to which I have no simple solution. But big PPs to deliberate abusers should suffice in a long run. Nobody will risk getting much less than 0% from the board if he can try for 20% or 30% legitimately. > d) you still don't tell us what a player should do who is > very/somewhat/mildly certain that his partner has given a correct > explanation - should he reveal that he misbid too? My solution is simple - if you are sure that your partner has given a correct explanation, than you should correct your explanation. In all other cases you should stick to your version. And I think it is easy for the players to comply with. Regardless, the same problem actually exists, when after the auction you are obliged to correct the misexplanations. > Herman. Regards Maciej From darkbystry at wp.pl Tue Nov 25 18:23:49 2008 From: darkbystry at wp.pl (Bystry) Date: Tue, 25 Nov 2008 18:23:49 +0100 Subject: [blml] laissez faire References: <49227C4E.3050007@skynet.be><000001c94a0b$1cc70330$56550990$@com><000d01c94ad2$b54bc880$15844c59@chello.pl><4925C17B.1010706@talktalk.net> <49266B37.90306@skynet.be><000f01c94bba$ecc985c0$15844c59@chello.pl><4926C496.6080904@skynet.be><003901c94c47$a1541140$15844c59@chello.pl><4927E7CC.4030303@skynet.be><001f01c94cad$4bf57340$15844c59@chello.pl> Message-ID: <008701c94f22$94d773c0$15844c59@chello.pl> Hi, > Hi Maciej. I am trying to figure out your position. But I am interested in > how anyone would answer this question. Is there a consensus? > > You make a bid which partner incorrectly explains. For example, your > agreement is to play 2Di as weak in first seat and multi otherwise. You > open 2Di in first seat and partner explains it as multi. > > Your partner now bids 3S. In your system, which the opponents do not know, > partnership agreement is that this shows a strong hand with a spade suit > over a weak 2Di. Over multi, in your system which the opponents do not > know, that asks for a spade stopper. So you know your partner is asking > for a spade stopper, but you have to bid like he is showing a spade > stopper. > > You have a great hand for 3NT, except no spade stopper. So you are forced > to bid 3NT, awaiting disaster. That ends the auction. You now explain that > your partner incorrectly described your bid, and in fact your 2 Di bid was > weak. > > The opponents now ask you to explain your other bids. What are you (and/or > your partner) required to explain? You are obliged to give correct explanation of partnership agreements. So you explain to them that 2D is natural, that 3S shows spades (but you should underline it is so after 2D natural) and that 3NT is natural (without spades in this case). > Bob, who, unless he screwed something up, which is common, is asking a > question he thinks people won't like Regards Maciej From darkbystry at wp.pl Tue Nov 25 18:46:45 2008 From: darkbystry at wp.pl (Bystry) Date: Tue, 25 Nov 2008 18:46:45 +0100 Subject: [blml] laissez faire References: <49227C4E.3050007@skynet.be><000001c94a0b$1cc70330$56550990$@com> <000d01c94ad2$b54bc880$15844c59@chello.pl><4925C17B.1010706@talktalk.net> <49266B37.90306@skynet.be><000f01c94bba$ecc985c0$15844c59@chello.pl> <4926C496.6080904@skynet.be><003901c94c47$a1541140$15844c59@chello.pl> <4927E7CC.4030303@skynet.be><001f01c94cad$4bf57340$15844c59@chello.pl> <49292847.9010802@skynet.be> Message-ID: <009901c94f25$c89286c0$15844c59@chello.pl> Herman, > >>> a) very often you'll be unable to read partner's mind. So you'll end > > giving > >>> him UI. > >> very often you know exactly what is in partner's mind, and you know how > >> to NOT give him UI. I wanted to say "more often" but I won't. > > > > Even if it was true, it does not make my point invalid. Following the CS you > > always know what to do. Following the dWS you sometimes are unable to do > > anything clever. And I'm sure the answers will take you so much time that > > everybody at the table will know what is going on. Telling the truth is > > easy, lying requires practice and cool blood. > > Sorry Maciej, but this is simply not true. Following the MS (I don't > know where you get the C from - M is for Majority), I don't care about majority. I call it CS - the classical school ;-) > you DONT always know > what to do. Since you are sometimes unsure of who has the correct > explanation of system. OTOH, following the DWS, you always know what to > do, provided you know the system your partner thinks you are playing. Tell me Herman, what is more probable - that you know the system *you* are playing and *you* have chosen to put into use or that you know system that *partner* "plays" and *he* has chosen to put into use? And what problem do you have with uncertainty? Just keep explaining your version unless you are pretty sure that you were wrong. > Now of course there can be circumstances where your partner starts > explaining a system that you have no idea about. But do you really think > this is so common an occurance? And even if it sometimes happens, what > then about the DWSer who tries to explain his partner's system and > fails? Well, his partner will just think that he has forgotten the > system, not that he was wrong in the first place. Of course the player > should not say "I have no idea what it means, I have never played that > system before"; but saying "I think that's XX" does not convey the UI > that the previous explanation was wrong. Sorry Herman, but what is the purpose for all this maneuvering, lying, inventing non-existent things, creating confusion? All this to evade UI and save your skin? I told you strongly enough what do I think about such an attitude. Learn your system. Remember it. Accept poor scores when you can't. And by all means don't try to save yourself at the cost of innocent opponents. > Herman. Regards Maciej From nigelguthrie at talktalk.net Tue Nov 25 19:59:23 2008 From: nigelguthrie at talktalk.net (Nigel Guthrie) Date: Tue, 25 Nov 2008 18:59:23 +0000 Subject: [blml] Two alternatives for the WBF? No, three In-Reply-To: <492C1ED5.90500@meadows.pair.com> References: <492BF30E.8020808@skynet.be> <492C0D2E.4080607@t-online.de> <492C17C1.20605@ulb.ac.be> <492C1ED5.90500@meadows.pair.com> Message-ID: <492C4B0B.8020706@talktalk.net> [Brian Meadows] I know absolutely sod all about writing bridge laws. I do, however, have a reasonable amount of experience in helping produce other technical documentation. The vast majority of people who have been involved with an extended drafting process will have too great a tendency to see what they THINK has been written, rather than what has ACTUALLY been written. Bridge is in no way special in this regard. Maybe for the next FLB, the WBFLC will have the sense, when they set up their DSC, to nominate someone NOT in any way involved with the drafting to do the final proofread. In the worst case, they could reserve one of their own members for that task. And maybe pigs will fly. [Nigel] I agree with Brian: the actions of the WBF on DWS appear considered and deliberate. - For years, in BLML, Herman argued his interpretation with members of WBFLC. - Bored BLMLers often asked for a change to the law-book to clarify this, once and for all. - When the new law-book was about to be published, Herman attracted more BLMLers to his cause. - Law-committee members still refused to change the law because they were loth to pander to Herman. - When the issue threatened to take over BLML, the WBF decided to act, belatedly. - The WBF decided not to insert a few words directly into the law-book. - Instead they kept controversy alive, by posting a separate minute. From darkbystry at wp.pl Tue Nov 25 20:15:07 2008 From: darkbystry at wp.pl (Bystry) Date: Tue, 25 Nov 2008 20:15:07 +0100 Subject: [blml] Lazy fair [SEC=UNOFFICIAL] References: Message-ID: <016401c94f32$21142fe0$15844c59@chello.pl> Hi Bob, > Everyone agrees that it is okay to give a misleading explanation if you > don't know any better. Then you suffer the consequences. > > The problem occurs when a WBFLC opinion says that you are required to > provide a misleading explanation which you know is misleading, and you are > not required you to divulge the partnership agreements your partner was > using when he made his bid. That is not a case of what is an opinion of WBFLC. It's more like a case of what is Bridge. As long as any player bases his calls only on AI, he can bid everything he wants. The opponents aren't entitled to know his motives and certainly not to the contents of his hand. And if you look at it closely you'll see that this rule is reflected in the cases of misunderstandings. Let's take your example auction 2D-3S-3NT. According to your system this sequence means - 2D natural, 3S - natural, 3NT - natural. And that is all to which the opponents are entitled. Whether you actually have in your hands what is pointed by this auction is not important. Now we've got MI because 2D was misexplained. You are obliged to correct it after the auction and the redress will be given if the MI hurted the NOS during the auction. As you can see up to this point all seems to be clear. Now your objections come into the light - 3S was bid because of bidder's misconception, so it doesn't show what it should show. True, but imagine that 3S was bid as a psyche or real misbid. Wouldn't it be exactly the same? It would. Because 3S *is* in fact a misbid. You know about it but you can't use that knowledge (worse, you should bend backwards not to use it). And the opponents don't have any right to know it was a misbid. But here they are in the better position anyway, beacuse they know 3S was bid under the misconception. It can show the same thing or not. They can consult your CC if they wish so. And that is all they are entitled to. Now you could say that it's better for the OS to have a deficient CC. But it is illusionary because without the proof the TD can rule that your explanation of 3S was MI. To sum it up - we've got two similar cases - 2D-3S-3NT where 3S is a misbid and 2D-3S-3NT where 2D was misexplained and therefore 3S is probably a misbid. In the first case the NOS are simply fixed. In the second they've got much more information and they can infer that probably spades are your weak point. They can't be sure but it's just Bridge. > > So as Director I would provide the non-offending side > > with their **entitled** rectification by changing the > > score from +2210 to -7600. There is not any technical > > difficulty in assessing logical alternatives, since LAs > > after UI which _should_ have occurred are no more (and > > no less) difficult to assess than LAs after UI which > > _did_ occur. > > > > As TD I would apply the mildest PP of education against > > an inexperienced North who misunderstood Law. > > > > As TD I would refer a highly experienced recidivist > > North to the local Conduct and Ethics Committee under > > Law 81C7. > > I was guessing that no one had ever done this, and it sounds like you have > not. I would have also guessed that no one would do this. I don't want to > dare you to do this just to prove me wrong, because I don't think it is a > wise thing to do. I will concede that someone might do this. > > The fact remains that right now this is not common director practice, so > much so that no one has found an example. I can't speak for the other countries but in Poland I have only once (as I remember) met with Herman's approach to explanations and it was because the player was convinced that his partner is right. The first time I've ever heard of dWS was here, on BLML. So it's impossible for me to know about any cases in which such approach was hit with PP. But if it the dWS was to spread I'm sure the PPs would be given to the frequent offenders. At least to the point they are already given for different infractions (sadly too rarely). > More Herman bashing? My impression is that dWS involves attempting to > minimize the creation of UI. In my examples, the request for explanation > occurs after the auction is over and partner is either declarer or dummy. > So the creation of UI is not an issue. Yes, there is a big difference between your approach and the dWS. You're just trying to be over-helpful to the NOS (this approach is for me recomendable), Herman is trying to find a way for the OS to improve their own score at the cost of confusion for the NOS (and this I detest strongly). > Bob Regards Maciej From rfrick at rfrick.info Tue Nov 25 22:08:44 2008 From: rfrick at rfrick.info (Robert Frick) Date: Tue, 25 Nov 2008 16:08:44 -0500 Subject: [blml] laissez faire In-Reply-To: <008701c94f22$94d773c0$15844c59@chello.pl> References: <49227C4E.3050007@skynet.be> <000001c94a0b$1cc70330$56550990$@com> <000d01c94ad2$b54bc880$15844c59@chello.pl> <4925C17B.1010706@talktalk.net> <49266B37.90306@skynet.be> <000f01c94bba$ecc985c0$15844c59@chello.pl> <4926C496.6080904@skynet.be> <003901c94c47$a1541140$15844c59@chello.pl> <4927E7CC.4030303@skynet.be> <001f01c94cad$4bf57340$15844c59@chello.pl> <008701c94f22$94d773c0$15844c59@chello.pl> Message-ID: On Tue, 25 Nov 2008 12:23:49 -0500, Bystry wrote: > Hi, > >> Hi Maciej. I am trying to figure out your position. But I am interested >> in >> how anyone would answer this question. Is there a consensus? >> >> You make a bid which partner incorrectly explains. For example, your >> agreement is to play 2Di as weak in first seat and multi otherwise. You >> open 2Di in first seat and partner explains it as multi. >> >> Your partner now bids 3S. In your system, which the opponents do not >> know, >> partnership agreement is that this shows a strong hand with a spade suit >> over a weak 2Di. Over multi, in your system which the opponents do not >> know, that asks for a spade stopper. So you know your partner is asking >> for a spade stopper, but you have to bid like he is showing a spade >> stopper. >> >> You have a great hand for 3NT, except no spade stopper. So you are >> forced >> to bid 3NT, awaiting disaster. That ends the auction. You now explain >> that >> your partner incorrectly described your bid, and in fact your 2 Di bid >> was >> weak. >> >> The opponents now ask you to explain your other bids. What are you >> (and/or >> your partner) required to explain? > > You are obliged to give correct explanation of partnership agreements. So > you explain to them that 2D is natural, that 3S shows spades (but you > should > underline it is so after 2D natural) and that 3NT is natural (without > spades > in this case). Hi Maciej. Thanks for answering. I did want your opinion. I don't understand the part where you say "underline it is so after 2D natural". Are you doing this just to be nice because you really don't want to play bridge according to the law as you perceive it? I find that many people who think that you don't have to divulge the meaning of 3S still want to do it. Or do you think it is required? I almost never do this when I explain partner's bids in a normal auction. Would you still do the underlining is 3S was natural in both sequences? Now it is to your advantage to do so, because you will increase the chances of a spade lead even though partner really does have a spade suit. These might seem like small points to a player. But if I am called to a table to rule, I need to know if the underlining is required in the first situation and permitted in the second. Bob From richard.hills at immi.gov.au Tue Nov 25 23:56:09 2008 From: richard.hills at immi.gov.au (richard.hills at immi.gov.au) Date: Wed, 26 Nov 2008 09:56:09 +1100 Subject: [blml] What Is the Name of This Thread? [SEC=UNOFFICIAL] In-Reply-To: Message-ID: Melvin Fitting: "I now introduce Professor Smullyan, who will prove to you that either he doesn't exist or you don't exist, but you won't know which." The philosopher and stage magician Raymond Smullyan (1919- ) is best known for his books of logic puzzles, such as "What Is the Name of This Book?", but he also wrote two book of retrograde analysis chess problems, "The Chess Mysteries of Sherlock Holmes" and "The Chess Mysteries of the Arabian Knights". In 1957 Smullyan created what is, in my opinion, the greatest retrograde analysis chess problem of all time. The simple but deceptive layout is: White Bishop on a4, Black Rook on b5, Black Bishop on d5, Black King on d1. The White King has been accidentally knocked off the board. What square is the White King's location? Why is that square the White King's location? Edgar Kaplan: "That reasoning may be common sense but it ignores bridge law." "Common morality may require declarer to reveal, without deceit, what she holds, but bridge law requires something quite different." "thus, the absurd requirement that she give an accurate explanation of an agreement she has honestly forgotten." Robert Frick: Am I the only one who sees a problem here? If a law doesn't fit common sense, doesn't fit common morality, and if it leads to absurd requirements, aren't you supposed to fix it? Richard Hills: The point of the rules of the game "Diplomacy" is that one is rewarded for lying with a straight face, then stabbing one's blood brother ally in the back, contrary to common morality. And the point of the darkly humorous card game "Nuclear War" is the satirical fun one has on the way. So "Nuclear War" has rules which do not fit common sense, definitely do not fit common morality, and almost always lead to the absurd (but realistic) outcome of everybody losing. Joshua: "A strange game. The only winning move is not to play. How about a nice game of chess?" Best wishes Richard Hills, Aqua 5, workstation W550 Telephone: 02 6223 8453 Email: richard.hills at immi.gov.au Recruitment Section & DIAC Social Club movie tickets -------------------------------------------------------------------- Important Notice: If you have received this email by mistake, please advise the sender and delete the message and attachments immediately. This email, including attachments, may contain confidential, sensitive, legally privileged and/or copyright information. Any review, retransmission, dissemination or other use of this information by persons or entities other than the intended recipient is prohibited. DIAC respects your privacy and has obligations under the Privacy Act 1988. The official departmental privacy policy can be viewed on the department's website at www.immi.gov.au. See: http://www.immi.gov.au/functional/privacy.htm --------------------------------------------------------------------- From darkbystry at wp.pl Wed Nov 26 00:55:48 2008 From: darkbystry at wp.pl (Bystry) Date: Wed, 26 Nov 2008 00:55:48 +0100 Subject: [blml] laissez faire References: <49227C4E.3050007@skynet.be><000001c94a0b$1cc70330$56550990$@com><000d01c94ad2$b54bc880$15844c59@chello.pl><4925C17B.1010706@talktalk.net> <49266B37.90306@skynet.be><000f01c94bba$ecc985c0$15844c59@chello.pl><4926C496.6080904@skynet.be><003901c94c47$a1541140$15844c59@chello.pl><4927E7CC.4030303@skynet.be><001f01c94cad$4bf57340$15844c59@chello.pl><008701c94f22$94d773c0$15844c59@chello.pl> Message-ID: <001701c94f59$56f5c200$15844c59@chello.pl> Hi, > Hi Maciej. Thanks for answering. I did want your opinion. > > I don't understand the part where you say "underline it is so after 2D > natural". Are you doing this just to be nice because you really don't want > to play bridge according to the law as you perceive it? I find that many > people who think that you don't have to divulge the meaning of 3S still > want to do it. By "underlining" I mean refraining from creating an illusion that 3S clearly shows spades. Normally, when explaining anything, people use to say something like "he's got 4 spades, 3 diamonds...", "he showed 5-5". Here it would be better to say something like "my 2D was misexplained, according to our system it is natural. 3S shows spades after the natural 2D and 3NT is to play without the spade support". I don't think it would be necessary against strong players, but against beginners such form is much more clear. But it's of course not an obligation, merely my idea of courtesy. > Or do you think it is required? I almost never do this when I explain > partner's bids in a normal auction. Neither do I. It would be a waste of time. > Would you still do the underlining is 3S was natural in both sequences? > Now it is to your advantage to do so, because you will increase the > chances of a spade lead even though partner really does have a spade suit. See above. > These might seem like small points to a player. But if I am called to a > table to rule, I need to know if the underlining is required in the first > situation and permitted in the second. "Required" by Laws it is not. And I don't think that we should now be debating what to do with cheats who would say something like "but you know, 3S was bid because partner misunderstood, you know that? But do you *really" understand what I'm talking about? Yes, 3S, it shoooowsss, ekhhmm, shhows spades according to our system but remember that my partner has just forgotten it". In my whole life I met only one player whom I consider to be capable of doing something like that. Statistical error, nothing to worry about. > Bob Regards Maciej From richard.hills at immi.gov.au Wed Nov 26 01:17:47 2008 From: richard.hills at immi.gov.au (richard.hills at immi.gov.au) Date: Wed, 26 Nov 2008 11:17:47 +1100 Subject: [blml] Code of Practice [SEC=UNOFFICIAL] In-Reply-To: <001401c932a5$761833a0$0202a8c0@Mildred> Message-ID: Grattan Endicott, 20th October 2008: >An updated Code of Practice will be posted very soon. >Regulating authorities are urged to adopt it. Those which >adopt only parts of it are invited to say which parts they >do not apply. Richard Hills: Now available at http://www.ecatsbridge.com/ Some small but significant improvements to both format and substance. And, of course, the CoP is now fully aligned with the 2007 Lawbook. Best wishes Richard Hills, Aqua 5, workstation W550 Telephone: 02 6223 8453 Email: richard.hills at immi.gov.au Recruitment Section & DIAC Social Club movie tickets -------------------------------------------------------------------- Important Notice: If you have received this email by mistake, please advise the sender and delete the message and attachments immediately. This email, including attachments, may contain confidential, sensitive, legally privileged and/or copyright information. Any review, retransmission, dissemination or other use of this information by persons or entities other than the intended recipient is prohibited. DIAC respects your privacy and has obligations under the Privacy Act 1988. The official departmental privacy policy can be viewed on the department's website at www.immi.gov.au. See: http://www.immi.gov.au/functional/privacy.htm --------------------------------------------------------------------- From grandaeval at tiscali.co.uk Wed Nov 26 02:12:55 2008 From: grandaeval at tiscali.co.uk (Grattan) Date: Wed, 26 Nov 2008 01:12:55 -0000 Subject: [blml] can't think of a title References: Message-ID: <002a01c94f64$21f3a7b0$0302a8c0@Mildred> Grattan Endicott To: "Bridge Laws Mailing List" Sent: Tuesday, November 25, 2008 10:09 AM Subject: [blml] can't think of a title > Am I the only one who sees a problem here? If a law doesn't fit common sense, doesn't fit common morality, and if it leads to absurd requirements, aren't you supposed to fix it? > +=+ The element that it is suggested be changed was fundamental to the game when the international laws were first devised (and for long before that). A change would be a change at root of the concept of the game. The opposition to changing the nature of the game is powerful and deep seated. Since lesser changes under recent consideration evoked responses that were insurmountable I see no prospect of the change envisaged above for decades. ~ Grattan ~ +=+ From rfrick at rfrick.info Wed Nov 26 02:26:33 2008 From: rfrick at rfrick.info (Robert Frick) Date: Tue, 25 Nov 2008 20:26:33 -0500 Subject: [blml] laissez faire In-Reply-To: <001701c94f59$56f5c200$15844c59@chello.pl> References: <49227C4E.3050007@skynet.be> <000001c94a0b$1cc70330$56550990$@com> <000d01c94ad2$b54bc880$15844c59@chello.pl> <4925C17B.1010706@talktalk.net> <49266B37.90306@skynet.be> <000f01c94bba$ecc985c0$15844c59@chello.pl> <4926C496.6080904@skynet.be> <003901c94c47$a1541140$15844c59@chello.pl> <4927E7CC.4030303@skynet.be> <001f01c94cad$4bf57340$15844c59@chello.pl> <008701c94f22$94d773c0$15844c59@chello.pl> <001701c94f59$56f5c200$15844c59@chello.pl> Message-ID: On Tue, 25 Nov 2008 18:55:48 -0500, Bystry wrote: > Hi, > >> Hi Maciej. Thanks for answering. I did want your opinion. >> >> I don't understand the part where you say "underline it is so after 2D >> natural". Are you doing this just to be nice because you really don't >> want >> to play bridge according to the law as you perceive it? I find that many >> people who think that you don't have to divulge the meaning of 3S still >> want to do it. > > By "underlining" I mean refraining from creating an illusion that 3S > clearly > shows spades. Illusion? You have spent several emails lengthily arguing that 3S does show spades, according to partnership agreement. (Which we agree is all you need to explain.) > Normally, when explaining anything, people use to say > something like "he's got 4 spades, 3 diamonds...", "he showed 5-5". Here > it > would be better to say something like "my 2D was misexplained, according > to > our system it is natural. 3S shows spades after the natural 2D and 3NT > is to > play without the spade support". I don't think it would be necessary > against > strong players, but against beginners such form is much more clear. But > it's > of course not an obligation, merely my idea of courtesy. What I here is one vote that the rules say you can just tell the opponents that 3S shows spades, and another vote that you don't want to play bridge that way. In my experience, most people who say this is the rule don't particular like it and try one way or another to get the "non-required" information to the opponents. Bob From nigelguthrie at talktalk.net Wed Nov 26 02:42:48 2008 From: nigelguthrie at talktalk.net (Nigel Guthrie) Date: Wed, 26 Nov 2008 01:42:48 +0000 Subject: [blml] Prior agreement Message-ID: <492CA998.6010307@talktalk.net> [Edgar Kaplan quoted by Grattan Endicott quoted by Richard Hills] ... you know that your partner has made a mistaken bid. Suppose he opens four clubs, which is supposed to show a strong heart opening with at least a semi-solid suit, when you hold S. A x x H. K Q J x x D. J 10 x.x.x C void. It is obvious from your cards that he has forgotten the agreement, so you intend to pass him right there. First though, your right-hand opponent asks about the four clubs bid. Your explanation must be "Strong four hearts opening with a very good heart suit". That is, your obligation under bridge law is to describe your partnership agreement, not your partner's hand. That legal obligation remains the same when, behind screens, you must explain your own action. [Nigel] Edgar assumes that your default agreement is 4C = natural pre-empt. So he would *pass* 4C. But suppose that for the last 10 years partner insisted that 4C is always *Gerber*. Yesterday you managed to persuade him that it was better if it showed something else but you suspect he has forgotten. In this case, almost all players would bid *4H* (to show one ace). This shows that when you guess that a wheel has fallen off, then your *previous* agreement not only affects your diagnosis of the likely mistake; it may also influence your subsequent action. Is your *previous* agreement *authorised* information to you? In any case, should you disclose it to opponents? IMO this poses problems, in general, not just with DWS. From richard.hills at immi.gov.au Wed Nov 26 03:15:20 2008 From: richard.hills at immi.gov.au (richard.hills at immi.gov.au) Date: Wed, 26 Nov 2008 13:15:20 +1100 Subject: [blml] Lazy fair [SEC=UNOFFICIAL] In-Reply-To: Message-ID: Adolf Hitler, during the Munich crisis: "Czechoslovakia is a dagger pointed at the heart of Germany." Henry Kissinger, assessment of Kiwi strategic importance: "New Zealand is a dagger pointed at the heart of Antarctica." Swiss Pairs (matchpoint pairs converted to victory points) Dlr: East Vul: East-West The bidding has gone: WEST NORTH EAST SOUTH --- --- Pass Pass 2D (1) 2S 3C 3H (2) 4C Pass(3) 4D Pass 5C Pass(3) Pass Pass (1) Strong, either single-suited or strong balanced (2) Not alerted. Described during the play as natural with spade tolerance. (One out of two not bad?) (3) Evidence suggestive of an implicit pre-existing mutual partnership understanding that South bids a short suit to confuse the other side after an opposing artificial and strong opening bid. The complete deal: AQ7652 QT832 76 --- KJ T3 KJ4 A96 AKQ4 9852 AT93 KJ72 984 75 JT3 Q8654 Result: 5C by East, North-South +300, S8 opening lead The TD awarded Ave+ to East-West, Ave- to North-South, plus a standard procedural penalty was applied to North- South, under the EBU's so-called "Red Psyche" regulation. [At Swiss Pairs the standard PP is defined as 0.5 vp.] Robert Frick asserted: >Directors don't give procedural penalties for forgetting >one's convention. Richard Hills quibbles: Yes and no. It would be a Director's Error to apply a PP for the non-infraction of a misbid. But if a player (such as North above) just happens to repeatedly forget to correctly explain partner's convention, then Law 40C3(b) encourages the Director to apply a PP. Robert Frick asserted: >Everyone agrees that it is okay to give a misleading >explanation if you don't know any better. Because North-South did not know any better, they elected to appeal. Basis of appeal: "N/S do not believe the psychic call and subsequent action by North to be a red psychic bid." Comments by North-South: "North is happy to play in 4C or 5C based on the bidding, intending to sacrifice over 6C." Appeals Committee's comments: "The committee believe and find the fielding by North to be blatant and are not convinced by N/S's appeal nor the basis for the appeal. On any view, at the vulnerability, 5H, 6H or 7H would be likely to be beneficial if 3H was genuine and taken as so by North. The committee increases the fine to 1VP..." Best wishes Richard Hills, Aqua 5, workstation W550 Telephone: 02 6223 8453 Email: richard.hills at immi.gov.au Recruitment Section & DIAC Social Club movie tickets -------------------------------------------------------------------- Important Notice: If you have received this email by mistake, please advise the sender and delete the message and attachments immediately. This email, including attachments, may contain confidential, sensitive, legally privileged and/or copyright information. Any review, retransmission, dissemination or other use of this information by persons or entities other than the intended recipient is prohibited. DIAC respects your privacy and has obligations under the Privacy Act 1988. The official departmental privacy policy can be viewed on the department's website at www.immi.gov.au. See: http://www.immi.gov.au/functional/privacy.htm --------------------------------------------------------------------- From darkbystry at wp.pl Wed Nov 26 03:54:47 2008 From: darkbystry at wp.pl (Bystry) Date: Wed, 26 Nov 2008 03:54:47 +0100 Subject: [blml] laissez faire References: <49227C4E.3050007@skynet.be><000001c94a0b$1cc70330$56550990$@com><000d01c94ad2$b54bc880$15844c59@chello.pl><4925C17B.1010706@talktalk.net> <49266B37.90306@skynet.be><000f01c94bba$ecc985c0$15844c59@chello.pl><4926C496.6080904@skynet.be><003901c94c47$a1541140$15844c59@chello.pl><4927E7CC.4030303@skynet.be><001f01c94cad$4bf57340$15844c59@chello.pl><008701c94f22$94d773c0$15844c59@chello.pl><001701c94f59$56f5c200$15844c59@chello.pl> Message-ID: <001101c94f72$583f9dc0$15844c59@chello.pl> > > By "underlining" I mean refraining from creating an illusion that 3S > > clearly > > shows spades. > > Illusion? > > You have spent several emails lengthily arguing that 3S does show spades, > according to partnership agreement. (Which we agree is all you need to > explain.) Listen, I'm trying my best but I'm not a native English speaker. You can continue to nitpick every word I wrote but does it have any sense? Correct, full explanation should be: "according to partnership agreement 3S shows spades". Now in practice we often use shorcuts: "he has spades", "he's shown spades". My personal opinion (not any Law) is that in our discussed example it would be better to use this correct, full explanation. Maybe not necessarily against good players but surely against weak players. It can help them to understand the situation. Against complete novices I could even tell them straightly that although 3S systematically shows spades, here it isn't so for sure. That's my personal approach which has nothing in common with what the Laws oblige me to do. The Laws are clear - you have to explain your partnership understandings relevant to current auction. > What I here is one vote that the rules say you can just tell the opponents > that 3S shows spades, and another vote that you don't want to play bridge > that way. In my experience, most people who say this is the rule don't > particular like it and try one way or another to get the "non-required" > information to the opponents. Your assumptions about my position are completely wrong. I won't explain to anybody that 3S doesn't show spades. To anybody. I can only mildly try to put novices on the right track. I can abstain from creating an illusion for weak players. The rest are as much suitable as me in drawing logical conclusions. > Bob Regards Maciej From grabiner at alumni.princeton.edu Wed Nov 26 04:39:34 2008 From: grabiner at alumni.princeton.edu (David Grabiner) Date: Tue, 25 Nov 2008 22:39:34 -0500 Subject: [blml] What Is the Name of This Thread? [SEC=UNOFFICIAL] In-Reply-To: References: Message-ID: <8198AE2800654F75AA03C0453A6A1625@erdos> Caution: spoiler ahead. > In 1957 Smullyan created what is, in my opinion, the greatest > retrograde analysis chess problem of all time. The simple > but deceptive layout is: White Bishop on a4, Black Rook on > b5, Black Bishop on d5, Black King on d1. The White King has > been accidentally knocked off the board. > > What square is the White King's location? > Why is that square the White King's location? This is a great, simple problem. If it is white's move, then black cannot be in check from the bishop at a4, so the white king must be in the way. However, the white king cannot be on c2, which is next to the black king. Nor can it be on b3, as white would be in double check, and neither the black rook nor the black bishop could have moved or captured to produce check while also unmasking the other piece. If it is black's move, then black might be in check this move but could not have been in check before white's last move. If the bishop made that last move, it could only have come from b3 or c2, and on either space, there is nowhere that the white king could have blocked the check. If the white king made the last move, it could have blocked the bishop if it was on b3. It would be in double check, and as before, neither the black rook or bishop could have moved to produce that double check. However, there is a third possibility of double check; a black pawn on b4 could have unmasked both pieces at once by capturing a pawn at c4 en passant. Since that black pawn is no longer on the board, the white king must have captured it at c3 with the last move. Thus the white king is on c3. From richard.hills at immi.gov.au Wed Nov 26 04:48:58 2008 From: richard.hills at immi.gov.au (richard.hills at immi.gov.au) Date: Wed, 26 Nov 2008 14:48:58 +1100 Subject: [blml] What Is the Name of This Thread? [SEC=UNOFFICIAL] In-Reply-To: Message-ID: Brian: [snip] >I know absolutely sod all about writing bridge laws. I do, however, >have a reasonable amount of experience in helping produce other >technical documentation. The vast majority of people who have been >involved with an extended drafting process will have too great a >tendency to see what they THINK has been written, rather than what >has ACTUALLY been written. Bridge is in no way special in this >regard. > >Maybe for the next FLB, the WBFLC will have the sense, when they set >up their DSC, to nominate someone NOT in any way involved with the >drafting to do the final proofread. Richard: For _this_ 2007 Lawbook the WBF LC used "the wisdom of crowds" to do the final proofread, using the bridge community in general and blml in particular to assist in fixing several minor errors and one major (Law 27) error. See: http://www.ecatsbridge.com/Documents/2007laws.asp Admittedly the final proofread was slightly unorthodox, occurring post-publication rather than the normal pre-publication. Brian: [snip] >And maybe pigs will fly. Winston Churchill, attributed: "I am fond of pigs. Dogs look up to us. Cats look down on us. Pigs treat us as equals." Best wishes Richard Hills, Aqua 5, workstation W550 Telephone: 02 6223 8453 Email: richard.hills at immi.gov.au Recruitment Section & DIAC Social Club movie tickets -------------------------------------------------------------------- Important Notice: If you have received this email by mistake, please advise the sender and delete the message and attachments immediately. This email, including attachments, may contain confidential, sensitive, legally privileged and/or copyright information. Any review, retransmission, dissemination or other use of this information by persons or entities other than the intended recipient is prohibited. DIAC respects your privacy and has obligations under the Privacy Act 1988. The official departmental privacy policy can be viewed on the department's website at www.immi.gov.au. See: http://www.immi.gov.au/functional/privacy.htm --------------------------------------------------------------------- From darkbystry at wp.pl Wed Nov 26 05:17:23 2008 From: darkbystry at wp.pl (Bystry) Date: Wed, 26 Nov 2008 05:17:23 +0100 Subject: [blml] Prior agreement References: <492CA998.6010307@talktalk.net> Message-ID: <002b01c94f7d$e1ea3a20$15844c59@chello.pl> Hi Nigel, > [Edgar Kaplan quoted by Grattan Endicott quoted by Richard Hills] > ... you know that your partner has made a mistaken bid. Suppose he opens > four clubs, which is supposed to show a strong heart opening with at > least a semi-solid suit, when you hold > S. A x x H. K Q J x x D. J 10 x.x.x C void. > It is obvious from your cards that he has forgotten the agreement, so > you intend to pass him right there. First though, your right-hand > opponent asks about the four clubs bid. Your explanation must be "Strong > four hearts opening with a very good heart suit". That is, your > obligation under bridge law is to describe your partnership agreement, > not your partner's hand. That legal obligation remains the same when, > behind screens, you must explain your own action. > > [Nigel] > Edgar assumes that your default agreement is 4C = natural pre-empt. So > he would *pass* 4C. > > But suppose that for the last 10 years partner insisted that 4C is > always *Gerber*. Yesterday you managed to persuade him that it was > better if it showed something else but you suspect he has forgotten. In > this case, almost all players would bid *4H* (to show one ace). > > This shows that when you guess that a wheel has fallen off, then your > *previous* agreement not only affects your diagnosis of the likely > mistake; it may also influence your subsequent action. > > Is your *previous* agreement *authorised* information to you? In any > case, should you disclose it to opponents? > > IMO this poses problems, in general, not just with DWS. I don't see any problem here. Your hand is certainly AI for you. So you know that 4C is not Namyats. And anyway, without UI you are completely free to make any bid you wish. So normally you would pass. But now you have knowledge that partner probably bid Gerber (but you can't be 100% sure, maybe he confused your change as Gerber->Natural instead of Gerber->Namyats). Of course your previous agreements are AI for you (I can't even imagine how could you think otherwise). So you have no UI and still can bid all that you want. But know you have to disclose to your opponents the history of 4C in your partnership. And you have to do it whether you want to bid 4H, pass or 7NT. As a side issue - you are always obliged to include in your explanations all the things that you learn from your partnership experience. So if you play 2D as Multi and partner sometimes (IMO it should be reasonable frequency, not once a year, but once a month may be adequate) forgets and bids it as natural you should tell that to your opponents even if you don't base your calls on this knowledge. Regards Maciej From ardelm at optusnet.com.au Wed Nov 26 05:27:16 2008 From: ardelm at optusnet.com.au (Tony Musgrove) Date: Wed, 26 Nov 2008 15:27:16 +1100 Subject: [blml] Helgemo Smelgemo Message-ID: <6.1.0.6.2.20081126152018.01c3a3a0@mail.optusnet.com.au> From the latest ABF Newsletter: Dealer South, Vul EW J 8 4 Q J 7 6 Q 9 7 10 7 3 6 3 2 A K Q 5 10 9 8 5 A K 10 5 2 A K 6 J 6 2 A K Q 4 10 9 7 4 3 2 J 8 4 3 9 8 5 Bidding West North East South pass pass 1H X pass 1S pass 6S all pass The only comment by the 6S bidder was that the 1H was clearly short of the normal values for an opening bid. Perhaps normal in Norway, but I take it, much too strong for the infamous H1H. Cheers, Tony (Sydney) From Hermandw at skynet.be Wed Nov 26 09:58:15 2008 From: Hermandw at skynet.be (Herman De Wael) Date: Wed, 26 Nov 2008 09:58:15 +0100 Subject: [blml] The Nebraskan issue (really about the methods of change) In-Reply-To: <003901c94f16$f0976be0$15844c59@chello.pl> References: <4926FB3B.7060408@skynet.be> <003901c94f16$f0976be0$15844c59@chello.pl> Message-ID: <492D0FA7.1000407@skynet.be> Maciej, while your first two objections to my remarks may be valid, your third isn't: Bystry wrote: > Herman, > > >> c) it would have the same problem as with the MS interpretation: what do >> we do with a player who refuses (or forgets) to correct the MI? He will >> create for himself an advantage (partner has no UI - opponents no >> knowledge of the misunderstanding) which is very difficult to quantify >> and punish. I do realize that this is a lesser problem if it is always >> doen, lesser than in current MS, but still. > > And what do you do with a player who refuses (or forgets) to correct MI > after the auction? But we know what to do with that! It happens all the time, and we deal with it. We apply the MI laws during the play as well. Absolutely no comparison with the problem I'm talking about higher - what is the correction for _not_ having given UI? > The sanctions will only have to be strengthened. Ok, it > would be hard to redress damage coming from noncreation of UI and I already > admitted it is a strong argument to which I have no simple solution. But big > PPs to deliberate abusers should suffice in a long run. Nobody will risk > getting much less than 0% from the board if he can try for 20% or 30% > legitimately. > >> d) you still don't tell us what a player should do who is >> very/somewhat/mildly certain that his partner has given a correct >> explanation - should he reveal that he misbid too? > > My solution is simple - if you are sure that your partner has given a > correct explanation, than you should correct your explanation. In all other > cases you should stick to your version. And I think it is easy for the > players to comply with. Regardless, the same problem actually exists, when > after the auction you are obliged to correct the misexplanations. > And how do you, as TD, confirm that the player was indeed certain? After all, a player cannot go and check - so you must accept that a player is certain and still turns out to be wrong. But how do you know that he was really certain when he tells you so? This is an unworkable law, and the sooner you realise it, the sooner you'll be able to look deeper into DWS principles and check that they are not so bad after all. >> Herman. > > Regards > > Maciej > Herman. From Hermandw at skynet.be Wed Nov 26 10:01:16 2008 From: Hermandw at skynet.be (Herman De Wael) Date: Wed, 26 Nov 2008 10:01:16 +0100 Subject: [blml] laissez faire In-Reply-To: <008701c94f22$94d773c0$15844c59@chello.pl> References: <49227C4E.3050007@skynet.be><000001c94a0b$1cc70330$56550990$@com><000d01c94ad2$b54bc880$15844c59@chello.pl><4925C17B.1010706@talktalk.net> <49266B37.90306@skynet.be><000f01c94bba$ecc985c0$15844c59@chello.pl><4926C496.6080904@skynet.be><003901c94c47$a1541140$15844c59@chello.pl><4927E7CC.4030303@skynet.be><001f01c94cad$4bf57340$15844c59@chello.pl> <008701c94f22$94d773c0$15844c59@chello.pl> Message-ID: <492D105C.8010805@skynet.be> Maciej, most of what you think comes from an erroneous way of saying one thing: Bystry wrote: >> The opponents now ask you to explain your other bids. What are you (and/or >> your partner) required to explain? > > You are obliged to give correct explanation of partnership agreements. So > you explain to them that 2D is natural, that 3S shows spades (but you should > underline it is so after 2D natural) and that 3NT is natural (without spades > in this case). > The wrong thing is your first sentence: It should read: Your opponents are entitled to a correct explanation of partnership agreements. Since you are unable to provide them with that (L20F5a forbids you to say it), they will get an adjustment later. If you think about this like that, the dilemma disappears, and you can play bridge again. And yes, opponents are misinformed, and they will get an adjustment for that. But play continues in a reasonable way. >> Bob, who, unless he screwed something up, which is common, is asking a >> question he thinks people won't like > > Regards > > Maciej > Herman. From harald.skjaran at gmail.com Wed Nov 26 10:02:19 2008 From: harald.skjaran at gmail.com (=?UTF-8?Q?Harald_Skj=C3=A6ran?=) Date: Wed, 26 Nov 2008 10:02:19 +0100 Subject: [blml] laissez faire In-Reply-To: References: <49227C4E.3050007@skynet.be> <000001c94cde$e66eedd0$b34cc970$@no> <000301c94d8e$c1c90000$455b0000$@no> <000401c94d96$19f416f0$4ddc44d0$@no> <000001c94dc4$cbb14150$6313c3f0$@com> <7FBFC8D22B2248D580B17051DEA72D04@stefanie> Message-ID: 2008/11/25 Robert Frick : > On Tue, 25 Nov 2008 02:28:54 -0500, Harald Skj?ran > wrote: > >> On 25/12/2008, Stefanie Rohan wrote: >>> > Others have said that in the canonical example of 4NT being minors but >>> > taken >>> > as Blackwood, the opponents are entitled to know what a 5D response >>> would >>> > have meant if 4NT had been Blackwood. To me, this makes no sense. >>> Suppose >>> > I >>> > alert a 2C response to 1NT and tell the opponents that it is a puppet >>> to >>> > 2D, >>> > which I must now bid. Are they entitled to know what 2D would have >>> meant >>> > if >>> > 2C had been Stayman? I can see no reason why they should be. >>> >>> This is incorrect. The opponents are entitled to know the meanings of >>> bids >>> you have not made -- see 20F1. >> >> 20F1 says "he is entitled to know about calls actually made, about >> relevant alternative calls available that were not made, and ....". > > ...relevant inferences from the choice of action where these are matters > of partnership understanding." > > >> For a pair NOT applying the Stayman convention, replies to 2C Stayman >> are neither relevant nor available. Thus they're not entitled to any >> information regarding Stayman responses. In fact, not using Stayman, >> the bidding pair could be unaware of the convention (unlikely!). > > I don't think you can talk about a situation where you have no partnership > agreement and generalize to a situation where you do have a partnership > agreement. In my example, there was a partnership understanding for the > meaning of 3 Sp over the multi 2Di openener and the meaning of 3 Sp over > the weak 2 Di opener. > I don't think you can extrapolate my comments about the 2C puppet to 2D and question about Stayman responses to your 2D-3S case. I've not commented about that at all. -- Kind regards, Harald Skj?ran > > > > >> >> Opponents are only entitled to information about the calls actually >> made and alternative calls in opponents methods. >> >>> >>> >>> Stefanie Rohan >>> London, England >>> >>> >>> _______________________________________________ >>> blml mailing list >>> blml at amsterdamned.org >>> http://www.amsterdamned.org/mailman/listinfo/blml >>> >> >> > > > > _______________________________________________ > blml mailing list > blml at amsterdamned.org > http://www.amsterdamned.org/mailman/listinfo/blml > From Hermandw at skynet.be Wed Nov 26 10:13:58 2008 From: Hermandw at skynet.be (Herman De Wael) Date: Wed, 26 Nov 2008 10:13:58 +0100 Subject: [blml] laissez faire In-Reply-To: <009901c94f25$c89286c0$15844c59@chello.pl> References: <49227C4E.3050007@skynet.be><000001c94a0b$1cc70330$56550990$@com> <000d01c94ad2$b54bc880$15844c59@chello.pl><4925C17B.1010706@talktalk.net> <49266B37.90306@skynet.be><000f01c94bba$ecc985c0$15844c59@chello.pl> <4926C496.6080904@skynet.be><003901c94c47$a1541140$15844c59@chello.pl> <4927E7CC.4030303@skynet.be><001f01c94cad$4bf57340$15844c59@chello.pl> <49292847.9010802@skynet.be> <009901c94f25$c89286c0$15844c59@chello.pl> Message-ID: <492D1356.5040104@skynet.be> Bystry wrote: > Herman, > >>>>> a) very often you'll be unable to read partner's mind. So you'll end >>> giving >>>>> him UI. >>>> very often you know exactly what is in partner's mind, and you know how >>>> to NOT give him UI. I wanted to say "more often" but I won't. >>> Even if it was true, it does not make my point invalid. Following the CS > you >>> always know what to do. Following the dWS you sometimes are unable to do >>> anything clever. And I'm sure the answers will take you so much time > that >>> everybody at the table will know what is going on. Telling the truth is >>> easy, lying requires practice and cool blood. >> Sorry Maciej, but this is simply not true. Following the MS (I don't >> know where you get the C from - M is for Majority), > > I don't care about majority. I call it CS - the classical school ;-) > >> you DONT always know >> what to do. Since you are sometimes unsure of who has the correct >> explanation of system. OTOH, following the DWS, you always know what to >> do, provided you know the system your partner thinks you are playing. > > Tell me Herman, what is more probable - that you know the system *you* are > playing and *you* have chosen to put into use or that you know system that > *partner* "plays" and *he* has chosen to put into use? And what problem do > you have with uncertainty? Just keep explaining your version unless you are > pretty sure that you were wrong. > Maciej, you forget one thing. Even if you are not certain of the particular version of the system your partner thinks you are playing, explaining it consistently, even if wrong, gives him no UI about the previous explanation. If he explains 4NT as Blackwood, and you explain his 5Di as one ace, then even wheh he has 0 key-cards, he is still none the wiser that your 4NT was actually for the minors. >> Now of course there can be circumstances where your partner starts >> explaining a system that you have no idea about. But do you really think >> this is so common an occurance? And even if it sometimes happens, what >> then about the DWSer who tries to explain his partner's system and >> fails? Well, his partner will just think that he has forgotten the >> system, not that he was wrong in the first place. Of course the player >> should not say "I have no idea what it means, I have never played that >> system before"; but saying "I think that's XX" does not convey the UI >> that the previous explanation was wrong. > > Sorry Herman, but what is the purpose for all this maneuvering, lying, > inventing non-existent things, creating confusion? All this to evade UI and > save your skin? I told you strongly enough what do I think about such an > attitude. > Maciej, I don't like you calling this by all these bad words. They create the impression that it is indeed a form of cheating. It is not. There is no maneuvering. If I explain 5Di as one ace, that is precisely the extension of you not saying "what?" when he explains 4NT as Blackwood. Not maneuvering, just being consistent. There is no lying. When I explain 5Di as showing 1 ace, he really meant it as such, and he has one ace. There is no inventing non-existent things. 5Di did show one ace. And there is far less confusion than when you explain 5Di as diamond preference after he's just told them 4NT was Blackwood. If you still don't see this, then please compare two tables: At one table, the opponent checks the SC and sees that 5Di shows one ace, at the other, he asks me and I tell him it shows one ace. Any difference? NO. Now add a third table, where the opponent asks you and you say "diamond preference". Difference? HUGE. > Learn your system. > Remember it. > Accept poor scores when you can't. > And by all means don't try to save yourself at the cost of innocent > opponents. > As to your "you take your medicine", that would be a valid argument if I were also to have to say "sorry, no" when he explains "Blackwood". But the laws are not like that. You may not like it, but that is no reason to start inventing new laws when you are stopped from being restricted. It is the WBF who chose L20F5a in the direction it is. You should not use the argument you are using. It does not fit with the current law. The Lawmakers are quite happy with MI being present and continueing to be present. You should not be unhappy with it. And if you are unhappy with it, try to get it changed. But don't use your unhappyness as an argument in some other field. Herman. >> Herman. > > Regards > > Maciej > Herman. From Hermandw at skynet.be Wed Nov 26 10:23:31 2008 From: Hermandw at skynet.be (Herman De Wael) Date: Wed, 26 Nov 2008 10:23:31 +0100 Subject: [blml] Two alternatives for the WBF? No, three In-Reply-To: <492C0D2E.4080607@t-online.de> References: <492BF30E.8020808@skynet.be> <492C0D2E.4080607@t-online.de> Message-ID: <492D1593.1040006@skynet.be> Matthias Berghaus wrote: > Snipped, not because I disagree with the snipped parts, but to get to > the part I want to comment on. > > Herman De Wael schrieb: > >> Remember that since neither explanation conforms to all the laws in the >> book, there is no option to declare the explanation "legal". There is >> merely an option to declare it "acceptable". > > I have to disagree. Of course I can define some action as legal, even if > a rule is broken. > > Let us assume that I am about to invent some game. It is a simple game, > so I call it "Bonk". It has a number of rules, which I shall call > "Laws". The precise content of this laws shall be of no concern, the > game is not ready to be released to the market. Now Bonk has a law with > the number 20, which says that it is forbidden to plurk. It is an > infraction to plurk, and may lead to negative consequences to my score. > Bonk has another law, numbered 40. Law 40 says that I may plurk, it may > even force me to plurk, but only under circumstances laid down in law > 40. I make and interpret the rules, and by my definition it is perfectly > legal to plurk, but only when law 40 calls for it. The laws say that it > is legal, so it is legal. > Matthias, I understand what you are trying to do, but that was not the point of the exercise. You make it sound as if the laws have only one possible meaning. They haven't. It is up to the WBF to decide where they want to go. And they have three alternatives, not two. Writing this as if this has been settled, long ago, and for all time, does not make it so. Herman. From agot at ulb.ac.be Wed Nov 26 10:45:21 2008 From: agot at ulb.ac.be (Alain Gottcheiner) Date: Wed, 26 Nov 2008 10:45:21 +0100 Subject: [blml] Prior agreement In-Reply-To: <492CA998.6010307@talktalk.net> References: <492CA998.6010307@talktalk.net> Message-ID: <492D1AB1.3000501@ulb.ac.be> Nigel Guthrie a ?crit : > [Edgar Kaplan quoted by Grattan Endicott quoted by Richard Hills] > ... you know that your partner has made a mistaken bid. Suppose he opens > four clubs, which is supposed to show a strong heart opening with at > least a semi-solid suit, when you hold > S. A x x H. K Q J x x D. J 10 x.x.x C void. > It is obvious from your cards that he has forgotten the agreement, so > you intend to pass him right there. First though, your right-hand > opponent asks about the four clubs bid. Your explanation must be "Strong > four hearts opening with a very good heart suit". That is, your > obligation under bridge law is to describe your partnership agreement, > not your partner's hand. That legal obligation remains the same when, > behind screens, you must explain your own action. > > [Nigel] > Edgar assumes that your default agreement is 4C = natural pre-empt. So > he would *pass* 4C. > > But suppose that for the last 10 years partner insisted that 4C is > always *Gerber*. Yesterday you managed to persuade him that it was > better if it showed something else but you suspect he has forgotten. In > this case, almost all players would bid *4H* (to show one ace). > AG : I think this is the wrong problem. If faced with this hand and problem, I'd assume 4C is natural, because : - even diehard Gerberists don't play a 4C Gerber opening - my club holding and fairly strong hand are AI pointing to a natural 4C even after considering partner's tendencies - "when the bid can't be understood by other means, it's to play" And any sane bridge player would too, except perhaps that some would conclude partner has mixed up the meanings and showed spades, if that fits with partener's "pattern of forgetting". Make the hand something like xxx - KQJxx - x - xxxx and you'll see more spread votes. Best regards Alain From nigelguthrie at talktalk.net Wed Nov 26 11:10:06 2008 From: nigelguthrie at talktalk.net (Nigel Guthrie) Date: Wed, 26 Nov 2008 10:10:06 +0000 Subject: [blml] Prior agreement In-Reply-To: <002b01c94f7d$e1ea3a20$15844c59@chello.pl> References: <492CA998.6010307@talktalk.net> <002b01c94f7d$e1ea3a20$15844c59@chello.pl> Message-ID: <492D207E.4030501@talktalk.net> [Maciej] I don't see any problem here. Your hand is certainly AI for you. So you know that 4C is not Namyats. And anyway, without UI you are completely free to make any bid you wish. So normally you would pass. But now you have knowledge that partner probably bid Gerber (but you can't be 100% sure, maybe he confused your change as Gerber->Natural instead of Gerber->Namyats). Of course your previous agreements are AI for you (I can't even imagine how could you think otherwise). So you have no UI and still can bid all that you want. But know you have to disclose to your opponents the history of 4C in your partnership. And you have to do it whether you want to bid 4H, pass or 7NT. As a side issue - you are always obliged to include in your explanations all the things that you learn from your partnership experience. So if you play 2D as Multi and partner sometimes (IMO it should be reasonable frequency, not once a year, but once a month may be adequate) forgets and bids it as natural you should tell that to your opponents even if you don't base your calls on this knowledge. [Nigel] I fear that some BLMLers may say that you should not tell opponents about *previous* agreements because doing so may tip off that you know partner has misbid. In Edgar's scenario, an opponent *asks* what 4C means. But suppose your opponent doesn't ask. Your knowledge of your *previous* agreement still confers on you the same advantage. From brian at meadows.pair.com Wed Nov 26 11:15:47 2008 From: brian at meadows.pair.com (brian) Date: Wed, 26 Nov 2008 05:15:47 -0500 Subject: [blml] What Is the Name of This Thread? [SEC=UNOFFICIAL] In-Reply-To: References: Message-ID: <492D21D3.8040803@meadows.pair.com> richard.hills at immi.gov.au wrote: > Brian: > > [snip] > >> I know absolutely sod all about writing bridge laws. I do, however, >> have a reasonable amount of experience in helping produce other >> technical documentation. The vast majority of people who have been >> involved with an extended drafting process will have too great a >> tendency to see what they THINK has been written, rather than what >> has ACTUALLY been written. Bridge is in no way special in this >> regard. >> >> Maybe for the next FLB, the WBFLC will have the sense, when they set >> up their DSC, to nominate someone NOT in any way involved with the >> drafting to do the final proofread. > > Richard: > > For _this_ 2007 Lawbook the WBF LC used "the wisdom of crowds" to do > the final proofread, using the bridge community in general and blml > in particular to assist in fixing several minor errors and one major > (Law 27) error. See: > > http://www.ecatsbridge.com/Documents/2007laws.asp > > Admittedly the final proofread was slightly unorthodox, occurring > post-publication rather than the normal pre-publication. > I'm glad to hear it was done that way, even if belatedly. I was relying on Grattan's (much) earlier statement that John Wignall had taken the task of the final proof read. When I said that I thought that was a bad idea, I was asked my grounds for doubting John Wignall's qualifications. Maybe lessons are being learned, and porcine aviation is more likely than I thought. Brian. From nigelguthrie at talktalk.net Wed Nov 26 11:30:56 2008 From: nigelguthrie at talktalk.net (Nigel Guthrie) Date: Wed, 26 Nov 2008 10:30:56 +0000 Subject: [blml] Prior agreement In-Reply-To: <492D1AB1.3000501@ulb.ac.be> References: <492CA998.6010307@talktalk.net> <492D1AB1.3000501@ulb.ac.be> Message-ID: <492D2560.8000904@talktalk.net> [Alain Gottcheiner] I'd assume 4C is natural, because : - even diehard Gerberists don't play a 4C Gerber opening - my club holding and fairly strong hand are AI pointing to a natural 4C even after considering partner's tendencies - "when the bid can't be understood by other means, it's to play" [Nigel] The *stipulation* was that for the past 10 years, partner had insisted you play 4C as Gerber. But, if it suits you better, assume that you open 4N and partner replies 4C. (Gerber until yesterday, now solid hearts). You know a wheel has fallen off. Do you masochistically treat it as natural? From agot at ulb.ac.be Wed Nov 26 11:41:56 2008 From: agot at ulb.ac.be (Alain Gottcheiner) Date: Wed, 26 Nov 2008 11:41:56 +0100 Subject: [blml] Prior agreement In-Reply-To: <492D207E.4030501@talktalk.net> References: <492CA998.6010307@talktalk.net> <002b01c94f7d$e1ea3a20$15844c59@chello.pl> <492D207E.4030501@talktalk.net> Message-ID: <492D27F4.6070106@ulb.ac.be> Nigel Guthrie a ?crit : > [Maciej] > I don't see any problem here. Your hand is certainly AI for you. So you know > that 4C is not Namyats. And anyway, without UI you are completely free to > make any bid you wish. So normally you would pass. But now you have > knowledge that partner probably bid Gerber (but you can't be 100% sure, > maybe he confused your change as Gerber->Natural instead of > Gerber->Namyats). Of course your previous agreements are AI for you (I can't > even imagine how could you think otherwise). So you have no UI and still can > bid all that you want. But know you have to disclose to your opponents the > history of 4C in your partnership. And you have to do it whether you want to > bid 4H, pass or 7NT. > > AG : when do you explain ? Directly when they ask ? Not only there is the 'tip off' problem, as you mentioned yourself, but you only would think of explaining history if something straznge had happened. I don't think the laws support the fact that you explain differently according to your hand or to oppoentns' reactions. Now you could surely give a guess after they see your incredible pass, but they don't need it ; they understand what happened as well as you do. Also notice that, if I had to exlpain the history of the 1C-1NT response in my partnership, that would take up the next three or four rounds. > As a side issue - you are always obliged to include in your explanations all > the things that you learn from your partnership experience. So if you play > 2D as Multi and partner sometimes (IMO it should be reasonable frequency, > not once a year, but once a month may be adequate) forgets and bids it as > natural you should tell that to your opponents even if you don't base your > calls on this knowledge. > > AG : if your partner forgets some convention once a month, I'd disallow its use. Best regards Alain From agot at ulb.ac.be Wed Nov 26 11:47:50 2008 From: agot at ulb.ac.be (Alain Gottcheiner) Date: Wed, 26 Nov 2008 11:47:50 +0100 Subject: [blml] Prior agreement In-Reply-To: <492D2560.8000904@talktalk.net> References: <492CA998.6010307@talktalk.net> <492D1AB1.3000501@ulb.ac.be> <492D2560.8000904@talktalk.net> Message-ID: <492D2956.4060608@ulb.ac.be> Nigel Guthrie a ?crit : > [Alain Gottcheiner] > I'd assume 4C is natural, because : > > - even diehard Gerberists don't play a 4C Gerber opening > - my club holding and fairly strong hand are AI pointing to a natural 4C > even after considering partner's tendencies > - "when the bid can't be understood by other means, it's to play" > > > [Nigel] > The *stipulation* was that for the past 10 years, partner had insisted you play 4C as Gerber. > But, if it suits you better, assume that you open 4N and partner replies 4C. > (Gerber until yesterday, now solid hearts). You know a wheel has fallen off. Do you masochistically treat it as natural? > AG : not a good example either. How could you be sure, even holding Axx - KQJx - KQJx - xx that partner doesn't hold a perfectly normal transfer hand, like xx - A10xxxx - xx - KQx ? How do you dare assume it's Gerber ? Why doiesn't he simply differ on what constitutes a transfer hand ? To be honest, finding a case where you could be sure a wheeel had come loose and identify it and not tell them, by your mere bid (or pass) ths whole story, proves truly difficult. You needed to pretend 1NT-4C shows *solid* hearts, which not one pair in the world play. From dalburn at btopenworld.com Wed Nov 26 13:54:25 2008 From: dalburn at btopenworld.com (David Burn) Date: Wed, 26 Nov 2008 12:54:25 -0000 Subject: [blml] Prior agreement In-Reply-To: <492D2560.8000904@talktalk.net> References: <492CA998.6010307@talktalk.net> <492D1AB1.3000501@ulb.ac.be> <492D2560.8000904@talktalk.net> Message-ID: <002d01c94fc6$1c8ecf90$55ac6eb0$@com> [NG] But, if it suits you better, assume that you open 4N and partner replies 4C. [DALB] I suppose that if I want to know how many aces partner has (which is why I opened Blackwood) it is only reasonable that he wants to know how many aces I have (which is why he responded Gerber). David Burn London, England From nigelguthrie at talktalk.net Wed Nov 26 14:50:54 2008 From: nigelguthrie at talktalk.net (Nigel Guthrie) Date: Wed, 26 Nov 2008 13:50:54 +0000 Subject: [blml] Prior agreement In-Reply-To: <002d01c94fc6$1c8ecf90$55ac6eb0$@com> References: <492CA998.6010307@talktalk.net> <492D1AB1.3000501@ulb.ac.be> <492D2560.8000904@talktalk.net> <002d01c94fc6$1c8ecf90$55ac6eb0$@com> Message-ID: <492D543E.9060701@talktalk.net> [NG] But, if it suits you better, assume that you open 4N and partner replies 4C. [DALB] I suppose that if I want to know how many aces partner has (which is why I opened Blackwood) it is only reasonable that he wants to know how many aces I have (which is why he responded Gerber). [Nigel] Good point David :) I meant 1N-4C. But the exact example is irrelevant. >From the auction and your own hand, you are sure that partner's bid was a mistake. You know your previous agreement and the kind of mistake to which partner is prone. To me (if to nobody else) it seems that you are often better placed than opponents - to diagnose partner's likely mistake, - and to take appropriate compensating action. IMO ... - *Either* such information is *unauthorised* to you - *or* you should *disclose* it to opponents. Furthermore, this isn't just of theoretical interest. It's obviously relevant to psyching and DWS. But it's also relevant to other auctions with and without alerts and explanations. From darkbystry at wp.pl Wed Nov 26 18:05:06 2008 From: darkbystry at wp.pl (Bystry) Date: Wed, 26 Nov 2008 18:05:06 +0100 Subject: [blml] The Nebraskan issue (really about the methods of change) References: <4926FB3B.7060408@skynet.be><003901c94f16$f0976be0$15844c59@chello.pl> <492D0FA7.1000407@skynet.be> Message-ID: <001d01c94fe9$22069200$15844c59@chello.pl> Hi, > > And what do you do with a player who refuses (or forgets) to correct MI > > after the auction? > > But we know what to do with that! It happens all the time, and we deal > with it. We apply the MI laws during the play as well. > Absolutely no comparison with the problem I'm talking about higher - > what is the correction for _not_ having given UI? I'm not obliged to repeat myself. Reread my past emails regarding this issue. > > My solution is simple - if you are sure that your partner has given a > > correct explanation, than you should correct your explanation. In all other > > cases you should stick to your version. And I think it is easy for the > > players to comply with. Regardless, the same problem actually exists, when > > after the auction you are obliged to correct the misexplanations. > > > > And how do you, as TD, confirm that the player was indeed certain? > After all, a player cannot go and check - so you must accept that a > player is certain and still turns out to be wrong. But how do you know > that he was really certain when he tells you so? I thought that you are the fan of mindreading as a TD. Show your power. Or simply use classical "could have known" phrase and punish him whether he's real villain or not. > This is an unworkable law, and the sooner you realise it, the sooner > you'll be able to look deeper into DWS principles and check that they > are not so bad after all. I don't care about your fantasies. You're not a preceptor of mine. I'm used to consider thoroughly any opinion I later submit to the serious discussion. I made so this time. > Herman. Regards Maciej From darkbystry at wp.pl Wed Nov 26 18:33:44 2008 From: darkbystry at wp.pl (Bystry) Date: Wed, 26 Nov 2008 18:33:44 +0100 Subject: [blml] laissez faire References: <49227C4E.3050007@skynet.be><000001c94a0b$1cc70330$56550990$@com><000d01c94ad2$b54bc880$15844c59@chello.pl><4925C17B.1010706@talktalk.net> <49266B37.90306@skynet.be><000f01c94bba$ecc985c0$15844c59@chello.pl><4926C496.6080904@skynet.be><003901c94c47$a1541140$15844c59@chello.pl><4927E7CC.4030303@skynet.be><001f01c94cad$4bf57340$15844c59@chello.pl> <008701c94f22$94d773c0$15844c59@chello.pl> <492D105C.8010805@skynet.be> Message-ID: <002501c94fed$21e0e4c0$15844c59@chello.pl> Hi, > Maciej, most of what you think comes from an erroneous way of saying one > thing: I'd advise you to search for errors in your statements first, only than to hunt for them in those of the other people. > >> The opponents now ask you to explain your other bids. What are you (and/or > >> your partner) required to explain? > > > > You are obliged to give correct explanation of partnership agreements. So > > you explain to them that 2D is natural, that 3S shows spades (but you should > > underline it is so after 2D natural) and that 3NT is natural (without spades > > in this case). > > > > The wrong thing is your first sentence: > > It should read: > > Your opponents are entitled to a correct explanation of partnership > agreements. Since you are unable to provide them with that (L20F5a > forbids you to say it), they will get an adjustment later. Moonshine. Maybe you'd bother first to check what was I responding to? [Bob Frick] > You have a great hand for 3NT, except no spade stopper. So you are forced > to bid 3NT, awaiting disaster. ***That ends the auction.*** You now explain > that your partner incorrectly described your bid, and in fact your 2 Di bid was > weak. > The opponents now ask you to explain your other bids. What are you (and/or > your partner) required to explain? [/Bob Frick] Moonshine 2. L20F5a was clearly interpreted. [WBFLC Minutes, Beijing, 2008 (October 10)] Law 20 There is no infraction when a correct explanation discloses that partner's prior explanation was mistaken. The words "nor may he indicate in any manner that a mistake has been made" (in Law 20F5(a)) do not refer to compliance with the overriding requirement of the laws always to respond to enquiries under Law 20F with correct explanations of the partnership understandings. [/WBFLC Minutes, Beijing, 2008 (October 10)] > If you think about this like that, the dilemma disappears, and you can > play bridge again. And yes, opponents are misinformed, and they will get > an adjustment for that. But play continues in a reasonable way. Moonshine 3. I have no trouble playing bridge. In *my reasonable way". Without unnecessary TD interventions. Without misinforming the opponents in order to avoid self responsibility. What is *your reasonable way* I could notice reading your numerous contributions to BLML (e.g. glorious idea that it is normal to claim because one *feels* he's got the rest of the tricks not bothering to actually count them). Thanks, I'll continue by my way. > Herman. Regards Maciej From darkbystry at wp.pl Wed Nov 26 19:39:41 2008 From: darkbystry at wp.pl (Bystry) Date: Wed, 26 Nov 2008 19:39:41 +0100 Subject: [blml] laissez faire References: <49227C4E.3050007@skynet.be><000001c94a0b$1cc70330$56550990$@com> <000d01c94ad2$b54bc880$15844c59@chello.pl><4925C17B.1010706@talktalk.net> <49266B37.90306@skynet.be><000f01c94bba$ecc985c0$15844c59@chello.pl> <4926C496.6080904@skynet.be><003901c94c47$a1541140$15844c59@chello.pl> <4927E7CC.4030303@skynet.be><001f01c94cad$4bf57340$15844c59@chello.pl> <49292847.9010802@skynet.be><009901c94f25$c89286c0$15844c59@chello.pl> <492D1356.5040104@skynet.be> Message-ID: <002d01c94ff6$5876ce60$15844c59@chello.pl> Hi, > > Tell me Herman, what is more probable - that you know the system *you* are > > playing and *you* have chosen to put into use or that you know system that > > *partner* "plays" and *he* has chosen to put into use? And what problem do > > you have with uncertainty? Just keep explaining your version unless you are > > pretty sure that you were wrong. > > > > Maciej, you forget one thing. Even if you are not certain of the > particular version of the system your partner thinks you are playing, > explaining it consistently, even if wrong, gives him no UI about the > previous explanation. I have only one version of my system. If my partner or me forget it temporarily, there is no "substitute" system. There is only a delusion, fantasy. I don't have even a bit of will to participate in this delusion. And I don't care about UI. I'm creating it many times - by questioning, by explaining, by hesitating, by unwillful mimic etc. My partners do not use it. If yours do, change them. > If he explains 4NT as Blackwood, and you explain his 5Di as one ace, > then even wheh he has 0 key-cards, he is still none the wiser that your > 4NT was actually for the minors. We were broaching another topic. What has UI to do with it? Anyway, I care only about not giving MI, not about not giving UI. > > Sorry Herman, but what is the purpose for all this maneuvering, lying, > > inventing non-existent things, creating confusion? All this to evade UI and > > save your skin? I told you strongly enough what do I think about such an > > attitude. > > > > Maciej, I don't like you calling this by all these bad words. They > create the impression that it is indeed a form of cheating. It is not. I've chosen these words consciously. In my opinion they describe the situation correctly. I don't equate them with cheating although it is close. Because for me deliberately giving MI only to avoid self-damaging UI is very close to cheating. I have a right to such judgment. > There is no maneuvering. If I explain 5Di as one ace, that is precisely > the extension of you not saying "what?" when he explains 4NT as > Blackwood. Not maneuvering, just being consistent. Saying "what?" (although unlawful) *do* *not* *misinforms* your opponents. You're always concetrating only on the UI side on the problem. > There is no lying. When I explain 5Di as showing 1 ace, he really meant > it as such, and he has one ace. If I ask you how many apples did the procudent put into the box (correctly 10) and you answer me 8 (because you've eaten 2) are you lying or not? > There is no inventing non-existent things. 5Di did show one ace. > And there is far less confusion than when you explain 5Di as diamond > preference after he's just told them 4NT was Blackwood. Not every given word applies to every situation. Leave Blackwood alone. Open 2D (Multi) and try to explain your partner's 2S response after he's just explained your 2D as "Burkina Faso Convention - strong two-suiter in diamonds and spades or weak preempt in hearts or 12-15 balanced with clubs at least as long as spades or GF with the sum of length in black suits not higher than sum of length in red suits divided by 2". Sweet? Yes, I know. This example is crazy. But there are many normal situations in which you don't know an answer. You can know the convention but not the set of follow-ups. You can know different follow-ups but not which would apply (you've never covered it with your partner). But in dWS you have to give an answer so you have to invent something non-existent in your partnership. > If you still don't see this, then please compare two tables: At one > table, the opponent checks the SC and sees that 5Di shows one ace, at > the other, he asks me and I tell him it shows one ace. > Any difference? NO. > Now add a third table, where the opponent asks you and you say "diamond > preference". Difference? HUGE. And so what? First question (before we apply dWS or CS) also creates UI and also can create different outcomes. First question for the first opponent and second question for the second opponent. Isn't it beautiful symmetry? :-) > > Learn your system. > > Remember it. > > Accept poor scores when you can't. > > And by all means don't try to save yourself at the cost of innocent > > opponents. > > > > As to your "you take your medicine", that would be a valid argument if I > were also to have to say "sorry, no" when he explains "Blackwood". But > the laws are not like that. You may not like it, but that is no reason > to start inventing new laws when you are stopped from being restricted. I'm not inventing any new Laws. Yes, I'd wish MI was corected instantly. But it's not allowed and I obey this rule. I'm obeying 20F1 too. *You* want to break it and invent *your* new Laws. > It is the WBF who chose L20F5a in the direction it is. You should not > use the argument you are using. It does not fit with the current law. And? I don't break the Laws. That doesn't mean I have to agree with them. > The Lawmakers are quite happy with MI being present and continueing to > be present. I think that this statement is far from true. The lawmakers had to reach difficult compromise between restricting MI and UI. They've made their choice. But they certainly aren't happy with MI being present. > You should not be unhappy with it. Because Herman said so? I can say: "You should not be unhappy with CS because the lawmakers support it". Both statements are equally stupid and unfounded. > And if you are unhappy > with it, try to get it changed. But don't use your unhappyness as an > argument in some other field. Moonshine. + there exists slavery in Ruritania $ human-rights activist wants it to be completely banished # he's got some arguments why freedom is better ! he'd wish to convince Ruritarian government to banish slavery * Ruritarian government won't banish slavery entirely but can be persuaded to give freedom to a half of slaves Now your argument is equal to such: - the activist shouldn't use (#) because he can only achieve (*) and not ($) My argument is different - (#) applies to ($) as well as to (*) which is a subset of ($). And because of (!), (*) is better than (+). > Herman. Regards Maciej From darkbystry at wp.pl Wed Nov 26 19:54:33 2008 From: darkbystry at wp.pl (Bystry) Date: Wed, 26 Nov 2008 19:54:33 +0100 Subject: [blml] Prior agreement References: <492CA998.6010307@talktalk.net><002b01c94f7d$e1ea3a20$15844c59@chello.pl> <492D207E.4030501@talktalk.net> Message-ID: <004701c94ff8$6bd56280$15844c59@chello.pl> Hi Nigel, > [Maciej] > I don't see any problem here. Your hand is certainly AI for you. So you know > that 4C is not Namyats. And anyway, without UI you are completely free to > make any bid you wish. So normally you would pass. But now you have > knowledge that partner probably bid Gerber (but you can't be 100% sure, > maybe he confused your change as Gerber->Natural instead of > Gerber->Namyats). Of course your previous agreements are AI for you (I can't > even imagine how could you think otherwise). So you have no UI and still can > bid all that you want. But know you have to disclose to your opponents the > history of 4C in your partnership. And you have to do it whether you want to > bid 4H, pass or 7NT. > > As a side issue - you are always obliged to include in your explanations all > the things that you learn from your partnership experience. So if you play > 2D as Multi and partner sometimes (IMO it should be reasonable frequency, > not once a year, but once a month may be adequate) forgets and bids it as > natural you should tell that to your opponents even if you don't base your > calls on this knowledge. > [Nigel] > I fear that some BLMLers may say that you should not tell opponents about *previous* agreements because doing so may tip off that you know partner has misbid. I don't care what they might say. I know what the Laws oblige me to do. My explanation is UI for partner and he can't use it in his bidding. > In Edgar's scenario, an opponent *asks* what 4C means. But suppose your opponent > doesn't ask. Your knowledge of your *previous* agreement still confers on you > the same advantage. In case of alerted bids there is no problem (the opponents should ask if they are interested). The only interesting case would be if there was a natural (non-alertable)bid which partner forgets often. But here we come into the areas of CPU etc. I'm not knowledgable on this topic, in my opinion the best solution is to alert this non-alertable call and explain on question that it's natural but partner is prone to forget. But that is only my opinion and don't take it as something correct. Regards Maciej From darkbystry at wp.pl Wed Nov 26 20:06:41 2008 From: darkbystry at wp.pl (Bystry) Date: Wed, 26 Nov 2008 20:06:41 +0100 Subject: [blml] Prior agreement References: <492CA998.6010307@talktalk.net> <002b01c94f7d$e1ea3a20$15844c59@chello.pl><492D207E.4030501@talktalk.net> <492D27F4.6070106@ulb.ac.be> Message-ID: <005301c94ffa$1dbfd7e0$15844c59@chello.pl> Hi Alain, >> [Maciej] >> I don't see any problem here. Your hand is certainly AI for you. So you know >> that 4C is not Namyats. And anyway, without UI you are completely free to >> make any bid you wish. So normally you would pass. But now you have >> knowledge that partner probably bid Gerber (but you can't be 100% sure, >> maybe he confused your change as Gerber->Natural instead of >> Gerber->Namyats). Of course your previous agreements are AI for you (I can't >> even imagine how could you think otherwise). So you have no UI and still can >> bid all that you want. But know you have to disclose to your opponents the >> history of 4C in your partnership. And you have to do it whether you want to >> bid 4H, pass or 7NT. > AG : when do you explain ? Directly when they ask ? Not only there is > the 'tip off' problem, as you mentioned yourself, What "tip off" problem? I didn't mention anything like that. My explanation would be UI for partner but so what? > AG: but you only would think of explaining history if something straznge had happened. It would be nice if you didn't tried to read my mind. I didn't write anything like that. I alert 4C. On question I would *always* explain it as "Namyats, but we played Gerber for 10 years and changed only yesterday", even having void in hearts. > AG: I don't think the laws support the fact that you explain differently according > to your hand or to oppoentns' reactions. Of course the Laws don't support this. But opponents have to ask to receive the answer. I can only alert and can't force them to receive the explanation. If 4C wasn't alertable I would alert it anyway and explain as given earlier. > AG: Also notice that, if I had to exlpain the history of the 1C-1NT response > in my partnership, that would take up the next three or four rounds. Reductio ad absurdum. I won't bother responding to it. >> As a side issue - you are always obliged to include in your explanations all >> the things that you learn from your partnership experience. So if you play >> 2D as Multi and partner sometimes (IMO it should be reasonable frequency, >> not once a year, but once a month may be adequate) forgets and bids it as >> natural you should tell that to your opponents even if you don't base your >> calls on this knowledge. > AG : if your partner forgets some convention once a month, I'd disallow > its use. On what basis? According to which Law? Good luck. > Best regards > Alain Regards Maciej From richard.hills at immi.gov.au Thu Nov 27 04:34:50 2008 From: richard.hills at immi.gov.au (richard.hills at immi.gov.au) Date: Thu, 27 Nov 2008 14:34:50 +1100 Subject: [blml] pugio tuus est interfector tuus [SEC=UNOFFICIAL] In-Reply-To: Message-ID: "Aristotle and an Aardvark go to Washington", Thomas Cathcart and Daniel Klein, pages 163 and 164: (A delegate to the Utah Republican Convention was arguing that a fence should be constructed along the entire U.S.-Mexican border, to prevent illegal immigrants from entering.) GOP Official: What happens when they [illegals] climb the fence? Delegate: You electrify it. Then they won't touch it. GOP Official: But what if they touch it? You would let them die? Delegate: It would be their choice. GOP Official: What about a mother with a baby strapped to her back? You would let the mother and the baby die? Delegate: It would be the mother's choice to kill that baby. GOP Official: Then you're in favour of abortion? [Dead silence] -- As reported by Paul Rolly in the Salt Lake Tribune Imps Dlr: West Vul: North-South You, West, hold: 952 KT3 43 T9864 The bidding has gone: WEST NORTH EAST SOUTH Pass Pass 1H 2S (1) 2H (2) Pass Pass Pass(3) (1) Weak, circa 5-9 hcp and a 6-card suit (2) Director summoned, North declines his Law 27A option (3) Mandatory pass by East under Law 27B2 What opening lead do you make? What other opening lead do you consider making? Best wishes Richard Hills, Aqua 5, workstation W550 Telephone: 02 6223 8453 Email: richard.hills at immi.gov.au Recruitment Section & DIAC Social Club movie tickets -------------------------------------------------------------------- Important Notice: If you have received this email by mistake, please advise the sender and delete the message and attachments immediately. This email, including attachments, may contain confidential, sensitive, legally privileged and/or copyright information. Any review, retransmission, dissemination or other use of this information by persons or entities other than the intended recipient is prohibited. DIAC respects your privacy and has obligations under the Privacy Act 1988. The official departmental privacy policy can be viewed on the department's website at www.immi.gov.au. See: http://www.immi.gov.au/functional/privacy.htm --------------------------------------------------------------------- From grabiner at alumni.princeton.edu Thu Nov 27 05:08:58 2008 From: grabiner at alumni.princeton.edu (David Grabiner) Date: Wed, 26 Nov 2008 23:08:58 -0500 Subject: [blml] pugio tuus est interfector tuus [SEC=UNOFFICIAL] In-Reply-To: References: Message-ID: <0068AB896DF44D9BA54BE8E6AF44CD11@erdos> Richard Hills writes: > Imps > Dlr: West > Vul: North-South > > You, West, hold: > > 952 > KT3 > 43 > T9864 > > The bidding has gone: > > WEST NORTH EAST SOUTH > Pass Pass 1H 2S (1) > 2H (2) > Pass Pass Pass(3) > > (1) Weak, circa 5-9 hcp and a 6-card suit > (2) Director summoned, North declines his Law 27A option > (3) Mandatory pass by East under Law 27B2 > > What opening lead do you make? > What other opening lead do you consider making? I consider both the H3 and the HK, and ultimately lead the H3. The HK would be attractive if partner knew I had three hearts, but he doesn't know that because the 2H bid is UI. I don't think it is urgent to get the right card through dummy, and if partner has six hearts and sees two hearts in dummy and only low trumps, he might try to give me a third-round ruff after the HK lead and give up a ruff and sluff instead. Or if he has AQJxxx of hearts, he might overtake and lead trumps unnecessarily to stop heart ruffs. From darkbystry at wp.pl Thu Nov 27 05:24:17 2008 From: darkbystry at wp.pl (Bystry) Date: Thu, 27 Nov 2008 05:24:17 +0100 Subject: [blml] pugio tuus est interfector tuus [SEC=UNOFFICIAL] References: Message-ID: <000701c95048$03777e80$15844c59@chello.pl> [snip] > Imps > Dlr: West > Vul: North-South > > You, West, hold: > > 952 > KT3 > 43 > T9864 > > The bidding has gone: > > WEST NORTH EAST SOUTH > Pass Pass 1H 2S (1) > 2H (2) > Pass Pass Pass(3) > > (1) Weak, circa 5-9 hcp and a 6-card suit > (2) Director summoned, North declines his Law 27A option > (3) Mandatory pass by East under Law 27B2 I would have changed 2H to 3H. > What opening lead do you make? D3 > What other opening lead do you consider making? none > Best wishes > > Richard Hills, Aqua 5, workstation W550 > Telephone: 02 6223 8453 > Email: richard.hills at immi.gov.au > Recruitment Section & DIAC Social Club movie tickets Regards Maciej From richard.hills at immi.gov.au Thu Nov 27 06:20:17 2008 From: richard.hills at immi.gov.au (richard.hills at immi.gov.au) Date: Thu, 27 Nov 2008 16:20:17 +1100 Subject: [blml] Prior argument [SEC=UNOFFICIAL] In-Reply-To: <492D207E.4030501@talktalk.net> Message-ID: William Browne (1692-1774): The King to Oxford sent a troop of horse, For Tories own no argument but force: With equal skill to Cambridge books he sent, For Whigs admit no force but argument. Nigel Guthrie >I fear that some BLMLers may say that you should not tell >opponents about *previous* agreements because doing so may >tip off that you know partner has misbid. Richard Hills: The opponents are entitled to know your pre-existing mutual explicit or implicit partnership understandings. It is entirely possible that a pre-existing mutual implicit partnership understanding about the meaning of partner's 4C opening bid could be two-way, either weak with clubs or a solid heart suit, due to partner frequently forgetting that weak with clubs is theoretically supposed to be an obsolete *previous* agreement. But..... If there is no _pre-existing_ mutual implicit partnership understanding created by frequent forgetting, a rare forget is an entirely legal misbid. If without looking at one's cards one would expect partner's 4C opening bid to promise solely a solid heart suit, then "solely a solid heart suit" is the only answer one gives to an opponent's enquiry, even if one sights the ace of hearts nestling in one's own hand. Nigel Guthrie: >you are often better placed than opponents >- to diagnose partner's likely mistake, >- and to take appropriate compensating action. Richard Hills: So what? In the majority of auctions in which a wheel comes off it is the opponents who diagnose the debacle with a penalty double. And as Herman De Wael notes, if both members of an unwheeled partnership act Lawfully, then one or both are likely to be subject to Law 16 and/or Law 73C and/or Law 75A constraints, restricting any attempts at rewheeling. But sometimes bad bidding gets good results. C'est la vie. Nigel Guthrie: >IMO ... > >- *Either* such information is *unauthorised* to you Richard Hills: Information is authorised if it comes from a licit source. Sighting the ace of hearts in one's own legal hand is licit. Law 7B2 and Law 16A1(c). Nigel Guthrie: >- *or* you should *disclose* it to opponents. Law 75C, second sentence: "Here there is no infraction of Law, since East-West did receive an accurate description of the North-South agreement; they have no claim to an accurate description of the North- South hands." Best wishes Richard Hills, Aqua 5, workstation W550 Telephone: 02 6223 8453 Email: richard.hills at immi.gov.au Recruitment Section & DIAC Social Club movie tickets -------------------------------------------------------------------- Important Notice: If you have received this email by mistake, please advise the sender and delete the message and attachments immediately. This email, including attachments, may contain confidential, sensitive, legally privileged and/or copyright information. Any review, retransmission, dissemination or other use of this information by persons or entities other than the intended recipient is prohibited. DIAC respects your privacy and has obligations under the Privacy Act 1988. The official departmental privacy policy can be viewed on the department's website at www.immi.gov.au. See: http://www.immi.gov.au/functional/privacy.htm --------------------------------------------------------------------- From richard.hills at immi.gov.au Thu Nov 27 07:37:46 2008 From: richard.hills at immi.gov.au (richard.hills at immi.gov.au) Date: Thu, 27 Nov 2008 17:37:46 +1100 Subject: [blml] Lazy fair [SEC=UNOFFICIAL] In-Reply-To: <005201c94c5a$746f5230$5d4df690$@com> Message-ID: Edward Gibbon (1737-1794): "The various modes of worship, which prevailed in the Roman world, were all considered by the people as equally true; by the philosopher, as equally false; and by the magistrate, as equally useful. And thus toleration produced not only mutual indulgence, but even religious concord." David Burn asserted: >Herman's position has always been that where partner has >misexplained your call and you now have to explain his, you >are forced to break one Law or another. [snip] >In a position where you must break one of the Laws, it is not >a grave offence to do so. [snip] >Herman has consistently asserted that if you do have to break >some Law, it is preferable to break Law 40 rather than Law 20. >Others have asserted the contrary position, and the WBFLC has >now made a decision [snip] Richard Hills quibbles: No, the point of the pre-October 2008 debate on the De Wael School is that most others (David Burn perhaps excepted) did _not_ take the contrary position to Herman De Wael, but instead took the non-contrary position (3) below. (1) Herman De Wael's position was break Law 40, not Law 20, (2) Straw Man contrary position was break Law 20, not Law 40, (3) October 2008 WBF LC position is no Law breaking required, as truthfully answering questions deemed to be an over-riding "but" exception to Law 20F5(a). David Burn: >Herman may continue to believe that the WBFLC has made the >wrong decision, and may continue so to argue. From time to >time others may weary of such arguments, Richard Hills: In a similar fashion Nigel Guthrie and I have argued the merits of the 1987 scoring changes, Nigel liking the doughnut but me liking the hole. David Burn: >but please let us have no more talk of lying or cheating, and >please let us have no more attempts to discredit Herman's >considerable and well-merited standing in the bridge world. Richard Hills: Yes, it is a funny sort of bridge liar and bridge infractor who publicly announces his intention to lie and infract to a world- wide bridge laws mailing list, because he wishes to emulate Mahatma Gandhi at the bridge table -- and then hardly spends any time infracting as a bridge player due to his constant directing. Plus Herman's considerable merits in applied logic have meant that his blml posts have side-stepped the pugio tuus est interfector tuus error (at least so far). Best wishes Richard Hills, Aqua 5, workstation W550 Telephone: 02 6223 8453 Email: richard.hills at immi.gov.au Recruitment Section & DIAC Social Club movie tickets -------------------------------------------------------------------- Important Notice: If you have received this email by mistake, please advise the sender and delete the message and attachments immediately. This email, including attachments, may contain confidential, sensitive, legally privileged and/or copyright information. Any review, retransmission, dissemination or other use of this information by persons or entities other than the intended recipient is prohibited. DIAC respects your privacy and has obligations under the Privacy Act 1988. The official departmental privacy policy can be viewed on the department's website at www.immi.gov.au. See: http://www.immi.gov.au/functional/privacy.htm --------------------------------------------------------------------- From grandaeval at tiscali.co.uk Thu Nov 27 08:36:56 2008 From: grandaeval at tiscali.co.uk (Grattan) Date: Thu, 27 Nov 2008 07:36:56 -0000 Subject: [blml] Two alternatives for the WBF? No, three References: <492BF30E.8020808@skynet.be> <492C0D2E.4080607@t-online.de> <492D1593.1040006@skynet.be> Message-ID: <000e01c95062$f148d8b0$0302a8c0@Mildred> Grattan Endicott To: "Bridge Laws Mailing List" Sent: Wednesday, November 26, 2008 9:23 AM Subject: Re: [blml] Two alternatives for the WBF? No, three >> > Matthias, I understand what you are trying to do, but that was not the point of the exercise. You make it sound as if the laws have only one possible meaning. They haven't. It is up to the WBF to decide where they want to go. And they have three alternatives, not two. Writing this as if this has been settled, long ago, and for all time, does not make it so. < +=+ The WBF Laws Committee is stated in the By-Laws of the WBF to have "the function and duty .. to consider and take account of all matters relating to the international laws of bridge. It shall "interpret the laws". NBOs in membership of the WBF have subscribed to (signed their acceptance of) its by-laws. A number of matters of law were interpreted with the above authority in the WBF LC minutes of 10th October 2008. Is there an issue in this correspondence that is not decided in those (or any) WBFLC minutes? ~ Grattan ~ +=+ . From Hermandw at skynet.be Thu Nov 27 09:12:35 2008 From: Hermandw at skynet.be (Herman De Wael) Date: Thu, 27 Nov 2008 09:12:35 +0100 Subject: [blml] The Nebraskan issue (really about the methods of change) In-Reply-To: <001d01c94fe9$22069200$15844c59@chello.pl> References: <4926FB3B.7060408@skynet.be><003901c94f16$f0976be0$15844c59@chello.pl> <492D0FA7.1000407@skynet.be> <001d01c94fe9$22069200$15844c59@chello.pl> Message-ID: <492E5673.9020401@skynet.be> Hello Maciej, Bystry wrote: > Hi, > >>> And what do you do with a player who refuses (or forgets) to correct MI >>> after the auction? >> But we know what to do with that! It happens all the time, and we deal >> with it. We apply the MI laws during the play as well. >> Absolutely no comparison with the problem I'm talking about higher - >> what is the correction for _not_ having given UI? > > I'm not obliged to repeat myself. Reread my past emails regarding this > issue. > Well, I don't recall any definite answer to that one. Are you in favour of PPs here? >>> My solution is simple - if you are sure that your partner has given a >>> correct explanation, than you should correct your explanation. In all > other >>> cases you should stick to your version. And I think it is easy for the >>> players to comply with. Regardless, the same problem actually exists, > when >>> after the auction you are obliged to correct the misexplanations. >>> >> And how do you, as TD, confirm that the player was indeed certain? >> After all, a player cannot go and check - so you must accept that a >> player is certain and still turns out to be wrong. But how do you know >> that he was really certain when he tells you so? > > I thought that you are the fan of mindreading as a TD. Show your power. Or > simply use classical "could have known" phrase and punish him whether he's > real villain or not. > Thereby rendering your previous answer totally useless. Maciej, you really should think about two steps at the same time. I ask yuou a question - you give an answer - and I tell you that answer won't work. Then please go back to the question and think about it some more. What are you going to do about punishing players who act like myself and give consistent answers, claiming that they were certain that their partner was right all along? Do you not see that this is a system which cannot work in real life? You will be unable to enforce this law from all but Herman De Wael, of whom you know that he would break this law willingly. Yet there will be thousands of players out there who will say exactly the same thing as I could be saying (I won't say it because you'd know it won't be true although you could not prove that beyond the fact that I would say that anyway). Many of those thousands will be completely innocent, telling the truth and not knowing that they are doing anything wrong. Some of those thousands will be lying through their teeth, knowing that this is the best way to get away with a better score. There will be no way for you to know who is who (except in the case of HDW). You might want to punish them all, just to be sure that there are no Probst cheats left, and you might want to punish them severely, for the same reason. That will mean giving huge penalties to people who are totally innocent. Unworkable. >> This is an unworkable law, and the sooner you realise it, the sooner >> you'll be able to look deeper into DWS principles and check that they >> are not so bad after all. > > I don't care about your fantasies. You're not a preceptor of mine. I'm used > to consider thoroughly any opinion I later submit to the serious discussion. > I made so this time. > You considered all I wrote above - and maintain your position? Remarkable - for two reasons. >> Herman. > > Regards > > Maciej > Herman. From Hermandw at skynet.be Thu Nov 27 09:16:04 2008 From: Hermandw at skynet.be (Herman De Wael) Date: Thu, 27 Nov 2008 09:16:04 +0100 Subject: [blml] Lazy fair [SEC=UNOFFICIAL] In-Reply-To: References: Message-ID: <492E5744.1070701@skynet.be> richard.hills at immi.gov.au wrote: > > Richard Hills quibbles: > > No, the point of the pre-October 2008 debate on the De Wael > School is that most others (David Burn perhaps excepted) did > _not_ take the contrary position to Herman De Wael, but > instead took the non-contrary position (3) below. > > (1) Herman De Wael's position was break Law 40, not Law 20, > > (2) Straw Man contrary position was break Law 20, not Law 40, > > (3) October 2008 WBF LC position is no Law breaking required, > as truthfully answering questions deemed to be an over-riding > "but" exception to Law 20F5(a). > But Richard: (3) IS the contrary position to (1)! In (1) and (2) you are speaking of accepting CE and SE. In (3) you are speaking of outlawing CE. That is the contrary position! And this one is the worst choice the WBF could be making. Herman. From Hermandw at skynet.be Thu Nov 27 09:23:28 2008 From: Hermandw at skynet.be (Herman De Wael) Date: Thu, 27 Nov 2008 09:23:28 +0100 Subject: [blml] The real text. In-Reply-To: <002501c94fed$21e0e4c0$15844c59@chello.pl> References: <49227C4E.3050007@skynet.be><000001c94a0b$1cc70330$56550990$@com><000d01c94ad2$b54bc880$15844c59@chello.pl><4925C17B.1010706@talktalk.net> <49266B37.90306@skynet.be><000f01c94bba$ecc985c0$15844c59@chello.pl><4926C496.6080904@skynet.be><003901c94c47$a1541140$15844c59@chello.pl><4927E7CC.4030303@skynet.be><001f01c94cad$4bf57340$15844c59@chello.pl> <008701c94f22$94d773c0$15844c59@chello.pl> <492D105C.8010805@skynet.be> <002501c94fed$21e0e4c0$15844c59@chello.pl> Message-ID: <492E5900.1060401@skynet.be> Maciej posted the real text once again: > [WBFLC Minutes, Beijing, 2008 (October 10)] > > Law 20 > > There is no infraction when a correct explanation discloses > that partner's prior explanation was mistaken. The words "nor > may he indicate in any manner that a mistake has been made" > (in Law 20F5(a)) do not refer to compliance with the > overriding requirement of the laws always to respond to > enquiries under Law 20F with correct explanations of the > partnership understandings. > > [/WBFLC Minutes, Beijing, 2008 (October 10)] > What does this mean? does it mean: A) SEs are accepted, or: B) CEs are outlawed Of course you'll say that it's B), but I only read A). I don't mind that. Of course you'll say that it was written to mean B), but since B) is not in the lawbook, and this text is meant to say B), should it not say B) more clearly than this? And if CEs are being outlawed, why don't I find any penalty in here? I would really like to see a prosecuting attorney at work with this against my actions. From Hermandw at skynet.be Thu Nov 27 09:31:42 2008 From: Hermandw at skynet.be (Herman De Wael) Date: Thu, 27 Nov 2008 09:31:42 +0100 Subject: [blml] laissez faire In-Reply-To: <002d01c94ff6$5876ce60$15844c59@chello.pl> References: <49227C4E.3050007@skynet.be><000001c94a0b$1cc70330$56550990$@com> <000d01c94ad2$b54bc880$15844c59@chello.pl><4925C17B.1010706@talktalk.net> <49266B37.90306@skynet.be><000f01c94bba$ecc985c0$15844c59@chello.pl> <4926C496.6080904@skynet.be><003901c94c47$a1541140$15844c59@chello.pl> <4927E7CC.4030303@skynet.be><001f01c94cad$4bf57340$15844c59@chello.pl> <49292847.9010802@skynet.be><009901c94f25$c89286c0$15844c59@chello.pl> <492D1356.5040104@skynet.be> <002d01c94ff6$5876ce60$15844c59@chello.pl> Message-ID: <492E5AEE.1070707@skynet.be> Bystry wrote: > > Not every given word applies to every situation. Leave Blackwood alone. Open > 2D (Multi) and try to explain your partner's 2S response after he's just > explained your 2D as "Burkina Faso Convention - strong two-suiter in > diamonds and spades or weak preempt in hearts or 12-15 balanced with clubs > at least as long as spades or GF with the sum of length in black suits not > higher than sum of length in red suits divided by 2". > > Sweet? Yes, I know. This example is crazy. But there are many normal > situations in which you don't know an answer. You can know the convention > but not the set of follow-ups. You can know different follow-ups but not > which would apply (you've never covered it with your partner). But in dWS > you have to give an answer so you have to invent something non-existent in > your partnership. > I have no problem whatsoever in describing 2Sp in the Burkina Faso system. Well, of course I have, but then your example is really too silly, and you realise that as well. The point I am making is that if I answer "2Sp in the Burkina Faso system means that he prefers spades to diamonds and hearts to clubs" then he can only imagine that I am playing the Ouagadougou variant while he is more familiar with the Banfora system (I had to look up the name of a second city, sorry). But if I tell him 2Sp shows heart preference then he may well understand that I am telling him that 2Di was not Burkina Faso but Multi. (of course this meaning is also the Koudougou variant of the BF system, but that's just me being lucky that my explanation has two possible conclusions) Herman. From darkbystry at wp.pl Thu Nov 27 10:57:07 2008 From: darkbystry at wp.pl (Bystry) Date: Thu, 27 Nov 2008 10:57:07 +0100 Subject: [blml] The Nebraskan issue (really about the methods of change) References: <4926FB3B.7060408@skynet.be><003901c94f16$f0976be0$15844c59@chello.pl> <492D0FA7.1000407@skynet.be><001d01c94fe9$22069200$15844c59@chello.pl> <492E5673.9020401@skynet.be> Message-ID: <001301c95076$846f00c0$15844c59@chello.pl> Hi, > Hello Maciej, > >>> And what do you do with a player who refuses (or forgets) to correct MI > >>> after the auction? > >> But we know what to do with that! It happens all the time, and we deal > >> with it. We apply the MI laws during the play as well. > >> Absolutely no comparison with the problem I'm talking about higher - > >> what is the correction for _not_ having given UI? > > > > I'm not obliged to repeat myself. Reread my past emails regarding this > > issue. > > > > Well, I don't recall any definite answer to that one. Are you in favour > of PPs here? Yes, mainly. > >> And how do you, as TD, confirm that the player was indeed certain? > >> After all, a player cannot go and check - so you must accept that a > >> player is certain and still turns out to be wrong. But how do you know > >> that he was really certain when he tells you so? > > > > I thought that you are the fan of mindreading as a TD. Show your power. Or > > simply use classical "could have known" phrase and punish him whether he's > > real villain or not. > > > > Thereby rendering your previous answer totally useless. > > Maciej, you really should think about two steps at the same time. I ask > yuou a question - you give an answer - and I tell you that answer won't > work. Then please go back to the question and think about it some more. Your logic is nearly impeccable. With one minor flaw. Your claim that my answer won't work is just your claim, not a definite truth. I have not even a slightest idea what people in Belgium do. But in Poland there are nearly no problems. The players just explain their system and more often they break the Law correcting MI instantly than adopting the dWS. So forgive me, but my experience shows that your fears are completely unsupported. > What are you going to do about punishing players who act like myself and > give consistent answers, claiming that they were certain that their > partner was right all along? As a TD I'm obliged to collect evidence and make a judgment. If I find that e.g. CC states that you were correct, than it is first and strong evidence that you can be lying. There will be usually more evidence available. If it is first time I've caught you doing something like that you will be educated and advised not to do that in future. For the second time I may believe or not but for the for third time you will get PP. Do you really think that anybody will believe you when you state that you remember your system making your bid, than forget it hearing your partner's explanation and this happens regularly? Second thing - I was just yesterday reading some past BLML contributions and I found your own statement (regarding 12C3 as I remember but that's irrelevant) in which you strongly alleged that the cheaters and liars are very small minority and we shouldn't really care about them. I wholeheartedly agree with you. They can be sometimes caught and punished severely, they are often feeling public pressure and so on. The behavior you are now describing is exactly that - deliberately breaking the Laws, lying, cheating. How much pairs do you think will really do this? And some of them will be caught regardless. And one more argument - you are afraid that some, so called, "innocents" will be punished. Sorry, they are no innocents. They break the Laws. First time they can do this because of ignorance and I accept that. Next time they do that on purpose and they are cheating. And don't try to persuade my to believe that people so easily change their minds regarding the identification of misexplainer. They make a call which is correct. They believe it is correct. They would need really strong suspicion that they are wrong. It could be so in novice-expert partnerships but than it is nearly always the novice who forgets, not an expert. Try to look at the things from the statistical point of view, Herman. How many cases will not be punished? First of all the OS will very often get a bad board caused by one-side UI and MI. Than, even if the score is not bad, the TD can adjust - by an assigned score (rarely), or if it is too hard, by an artificial score. Practically A+/A- will be the worst score for the NOS (and the best for the OS). And only those cases can be unfair. Yes, the justice can't be served in all cases. And it isn't in normal UI, MI, claim and other rulings. But if you add the PPs for frequent infractors there will remain only a small handful of such cases. That is an adequate price to pay for the obtained gain. [rest sniped, answered above] > > I don't care about your fantasies. You're not a preceptor of mine. I'm used > > to consider thoroughly any opinion I later submit to the serious discussion. > > I made so this time. > > > > You considered all I wrote above - and maintain your position? > Remarkable - for two reasons. Herman, allow it to get to your mind, only for one short moment, that maybe you are not right after all. That maybe all those people who disagree with you have stronger arguments. I know that it's hard when somebody is so fanatical in trying to force his ideas. But just try. The difference between us is that I'm not claiming that you are wrong. I'm not sure that I'm right, too. I just have different goals than you. M: MI is the worst for game H: UI is the worst for game M: all the efforts should be put into making things better for the NOS H: TD's amount of work and the possible life belts for the OS are at least as much important M: the primary obligation of every player is to obey the Laws and to do everything he can not to damage his opponents H: the primary obligation of every player is to try as much as he can to improve his position, even at the cost of the opponents M: the amount of infractions should be drastically decreased - by education, by restrictive Laws, by consequent harsh ruling H: "equity" should be most important, frequent offenders are as much valuable for game as those who try everything to conform to the rules M: as much boards as possible should be played normally by the players, not in TD's consultations and judgment, not in appeals rooms, TD at the table should be as a last resort H: TDs at the table are nothing bad, they make good rulings and people are delighted having been given a ruling instead of their own play There may be some more but I'll stop in this point. Of course I might have misjudged some of your beliefs. I've read many your mails from the last three or four years and I base my opinions on them. Feel free to correct me (but using solid arguments not mere denial) and I'll change my mind and even apologize. > Herman. Regards Maciej From agot at ulb.ac.be Thu Nov 27 10:59:15 2008 From: agot at ulb.ac.be (Alain Gottcheiner) Date: Thu, 27 Nov 2008 10:59:15 +0100 Subject: [blml] pugio tuus est interfector tuus [SEC=UNOFFICIAL] In-Reply-To: References: Message-ID: <492E6F73.6060305@ulb.ac.be> richard.hills at immi.gov.au a ?crit : > > Imps > Dlr: West > Vul: North-South > > You, West, hold: > > 952 > KT3 > 43 > T9864 > > The bidding has gone: > > WEST NORTH EAST SOUTH > Pass Pass 1H 2S (1) > 2H (2) > Pass Pass Pass(3) > > (1) Weak, circa 5-9 hcp and a 6-card suit > (2) Director summoned, North declines his Law 27A option > (3) Mandatory pass by East under Law 27B2 > > What opening lead do you make? > What other opening lead do you consider making? > AG : I lead the three of hearts. This 'odd number' lead will make UI from my 2H bid (I've got three cards in that suit) AI. But I fear L23, with such a weak hand. Best regards Alain From darkbystry at wp.pl Thu Nov 27 11:03:02 2008 From: darkbystry at wp.pl (Bystry) Date: Thu, 27 Nov 2008 11:03:02 +0100 Subject: [blml] laissez faire References: <49227C4E.3050007@skynet.be><000001c94a0b$1cc70330$56550990$@com> <000d01c94ad2$b54bc880$15844c59@chello.pl><4925C17B.1010706@talktalk.net> <49266B37.90306@skynet.be><000f01c94bba$ecc985c0$15844c59@chello.pl> <4926C496.6080904@skynet.be><003901c94c47$a1541140$15844c59@chello.pl> <4927E7CC.4030303@skynet.be><001f01c94cad$4bf57340$15844c59@chello.pl> <49292847.9010802@skynet.be><009901c94f25$c89286c0$15844c59@chello.pl> <492D1356.5040104@skynet.be><002d01c94ff6$5876ce60$15844c59@chello.pl> <492E5AEE.1070707@skynet.be> Message-ID: <001701c95077$5637d640$15844c59@chello.pl> > I have no problem whatsoever in describing 2Sp in the Burkina Faso > system. Well, of course I have, but then your example is really too > silly, and you realise that as well. > The point I am making is that if I answer "2Sp in the Burkina Faso > system means that he prefers spades to diamonds and hearts to clubs" > then he can only imagine that I am playing the Ouagadougou variant while > he is more familiar with the Banfora system (I had to look up the name > of a second city, sorry). > But if I tell him 2Sp shows heart preference then he may well understand > that I am telling him that 2Di was not Burkina Faso but Multi. (of > course this meaning is also the Koudougou variant of the BF system, but > that's just me being lucky that my explanation has two possible conclusions) This example was given to show you that sometimes you have to invent something non-existent in your partnership. Nothing more. There is no need for Burkina Faso, if you've never played Wilkosz you will still have to explain partner's responses not knowing them. So you will have to invent something. Bah, even if 2D is explained as Multi and you perfectly know this convention, still you are inventing something non-existing in your partnership (because Multi doesn't exist in your system). That is only a side issue of problems with dWS but it can be nevertheless important. > Herman. Regards Maciej From darkbystry at wp.pl Thu Nov 27 11:20:15 2008 From: darkbystry at wp.pl (Bystry) Date: Thu, 27 Nov 2008 11:20:15 +0100 Subject: [blml] The real text. References: <49227C4E.3050007@skynet.be><000001c94a0b$1cc70330$56550990$@com><000d01c94ad2$b54bc880$15844c59@chello.pl><4925C17B.1010706@talktalk.net> <49266B37.90306@skynet.be><000f01c94bba$ecc985c0$15844c59@chello.pl><4926C496.6080904@skynet.be><003901c94c47$a1541140$15844c59@chello.pl><4927E7CC.4030303@skynet.be><001f01c94cad$4bf57340$15844c59@chello.pl> <008701c94f22$94d773c0$15844c59@chello.pl> <492D105C.8010805@skynet.be><002501c94fed$21e0e4c0$15844c59@chello.pl> <492E5900.1060401@skynet.be> Message-ID: <001d01c95079$bd5e2b60$15844c59@chello.pl> > Maciej posted the real text once again: > > > [WBFLC Minutes, Beijing, 2008 (October 10)] > > > > Law 20 > > > > There is no infraction when a correct explanation discloses > > that partner's prior explanation was mistaken. The words "nor > > may he indicate in any manner that a mistake has been made" > > (in Law 20F5(a)) do not refer to compliance with the > > overriding requirement of the laws always to respond to > > enquiries under Law 20F with correct explanations of the > > partnership understandings. > > > > [/WBFLC Minutes, Beijing, 2008 (October 10)] > > > > What does this mean? > > does it mean: > > A) SEs are accepted, or: > > B) CEs are outlawed > > Of course you'll say that it's B), but I only read A). I don't mind that. CE were always outlawed. The only weak point was in possible conflict between L20F1 and L20F5a. So you could claim (although in my opinion unreasonably that you were forced to break one of these Laws and therefore could decide for yourself which one. Now the only way to obey the Laws is to give SE. > Of course you'll say that it was written to mean B), but since B) is not > in the lawbook, and this text is meant to say B), should it not say B) > more clearly than this? No, this minute was to ensure that A) is legal. B) was always illegal. > And if CEs are being outlawed, why don't I find any penalty in here? [L40C3] (b) Repeated violations of requirements to disclose partnership understandings may be penalized. [/L40C3] > I would really like to see a prosecuting attorney at work with this > against my actions. Come to Poland, get your due PPs and than sue me. It will be fun :-) Regards Maciej From nigelguthrie at talktalk.net Thu Nov 27 11:25:41 2008 From: nigelguthrie at talktalk.net (Nigel Guthrie) Date: Thu, 27 Nov 2008 10:25:41 +0000 Subject: [blml] Prior argument [SEC=UNOFFICIAL] In-Reply-To: References: Message-ID: <492E75A5.4040407@talktalk.net> [Richard Hillls] Information is authorised if it comes from a licit source. Sighting the ace of hearts in one's own legal hand is licit. Law 7B2 and Law 16A1(c). [Nigel] Obviously, information from the auction and your own hand is authorised to you; so it is OK for you to guess that a wheel has come off; but what about information from *previous agreements* and your knowledge of partner's likely mistakes? Richard says you should divulge this to opponents. Should you also divulge such information when partner's bid is - not alerted? - not explained? - not suspicious to you (an opponent may deem it suspicious)? I have never known an opponent divulge a previous agreement, in any circumstances. IMO a simpler better rule would be that such information is unauthorised to you. It would also reduce the problem with psychs and DWS. From Hermandw at skynet.be Thu Nov 27 11:40:41 2008 From: Hermandw at skynet.be (Herman De Wael) Date: Thu, 27 Nov 2008 11:40:41 +0100 Subject: [blml] The real text. In-Reply-To: <001d01c95079$bd5e2b60$15844c59@chello.pl> References: <49227C4E.3050007@skynet.be><000001c94a0b$1cc70330$56550990$@com><000d01c94ad2$b54bc880$15844c59@chello.pl><4925C17B.1010706@talktalk.net> <49266B37.90306@skynet.be><000f01c94bba$ecc985c0$15844c59@chello.pl><4926C496.6080904@skynet.be><003901c94c47$a1541140$15844c59@chello.pl><4927E7CC.4030303@skynet.be><001f01c94cad$4bf57340$15844c59@chello.pl> <008701c94f22$94d773c0$15844c59@chello.pl> <492D105C.8010805@skynet.be><002501c94fed$21e0e4c0$15844c59@chello.pl> <492E5900.1060401@skynet.be> <001d01c95079$bd5e2b60$15844c59@chello.pl> Message-ID: <492E7929.7020405@skynet.be> Bystry wrote: >> Maciej posted the real text once again: >> >>> [WBFLC Minutes, Beijing, 2008 (October 10)] >>> >>> Law 20 >>> >>> There is no infraction when a correct explanation discloses >>> that partner's prior explanation was mistaken. The words "nor >>> may he indicate in any manner that a mistake has been made" >>> (in Law 20F5(a)) do not refer to compliance with the >>> overriding requirement of the laws always to respond to >>> enquiries under Law 20F with correct explanations of the >>> partnership understandings. >>> >>> [/WBFLC Minutes, Beijing, 2008 (October 10)] >>> >> What does this mean? >> >> does it mean: >> >> A) SEs are accepted, or: >> >> B) CEs are outlawed >> >> Of course you'll say that it's B), but I only read A). I don't mind that. > > CE were always outlawed. No they are not. And if you say they are, then SE are also outlawed, and we're in a dilemma situation. > The only weak point was in possible conflict > between L20F1 and L20F5a. So you could claim (although in my opinion > unreasonably that you were forced to break one of these Laws and therefore > could decide for yourself which one. Now the only way to obey the Laws is to > give SE. > Which is precisely what I am saying in this thread to be not true. Simply saying that it is does not bring us anything. >> Of course you'll say that it was written to mean B), but since B) is not >> in the lawbook, and this text is meant to say B), should it not say B) >> more clearly than this? > > No, this minute was to ensure that A) is legal. B) was always illegal. > How can that be? We just saw that it wasn't. Saying that SE is legal does not make CE illegal. Not any more than it already was. >> And if CEs are being outlawed, why don't I find any penalty in here? > > [L40C3] > > (b) Repeated violations of requirements to disclose partnership > understandings may be penalized. > > [/L40C3] > HAHAHAHAHAHAHAHA. May be penalized? By what? You have a lawbook, and some people say it's incomplete. So you write an addition, and the same people still say it's incomplete. So you refer to the lawbook itself? Surely even you must see that this won't work. And why do you point to "repeated violations"? Despite 10 years of talking about this, I have never even once encountered a real case like this at the table - not as player and not even as director. How can there be repeated violations then? And even if there were - what is the penalty? >> I would really like to see a prosecuting attorney at work with this >> against my actions. > > Come to Poland, get your due PPs and than sue me. It will be fun :-) > Indeed it will. What else are you going to use than "Herman always says he'd break this law"? And what shall you use against some other Belgian you have never seen before? > Regards > > Maciej > Herman. From agot at ulb.ac.be Thu Nov 27 11:44:41 2008 From: agot at ulb.ac.be (Alain Gottcheiner) Date: Thu, 27 Nov 2008 11:44:41 +0100 Subject: [blml] laissez faire In-Reply-To: <001701c95077$5637d640$15844c59@chello.pl> References: <49227C4E.3050007@skynet.be><000001c94a0b$1cc70330$56550990$@com> <000d01c94ad2$b54bc880$15844c59@chello.pl><4925C17B.1010706@talktalk.net> <49266B37.90306@skynet.be><000f01c94bba$ecc985c0$15844c59@chello.pl> <4926C496.6080904@skynet.be><003901c94c47$a1541140$15844c59@chello.pl> <4927E7CC.4030303@skynet.be><001f01c94cad$4bf57340$15844c59@chello.pl> <49292847.9010802@skynet.be><009901c94f25$c89286c0$15844c59@chello.pl> <492D1356.5040104@skynet.be><002d01c94ff6$5876ce60$15844c59@chello.pl> <492E5AEE.1070707@skynet.be> <001701c95077$5637d640$15844c59@chello.pl> Message-ID: <492E7A19.5020000@ulb.ac.be> Bystry a ?crit : > if you've never played Wilkosz you will still have to > explain partner's responses not knowing them. So you will have to invent > something. AG : I don't see how this could be possible. There are three possible cases : 1. You play W, you know what responses are and there is no need to invent. 2. You agreed to play complete W, but you forgot to ask partner what it meant. Or you don't remember what the response means. You tell them you don't know (or don't remember), and perhaps the TD will send you away from the table and have your partner explain his bid, which will satisfy the requests of L20. Or partner could explain it in written form without letting you look. There is UI. Too bad. 3. You didn't agree anything about that opening, so the answer is "no agreement". But inventing an explanation because you don't know which it could be will cost you both UI, MI and bad reputation. In short, it goes against the Law. Best regards Alain PS : don't think the first sentence of # 2 is absurd ; there are umpteen pairs who agreed to play Landy but don't know what everyday bidding sequences (like passing over 1NT 2C X, or responding 2NT or 3C to 2C) would mean. From agot at ulb.ac.be Thu Nov 27 12:46:39 2008 From: agot at ulb.ac.be (Alain Gottcheiner) Date: Thu, 27 Nov 2008 12:46:39 +0100 Subject: [blml] Prior argument [SEC=UNOFFICIAL] In-Reply-To: <492E75A5.4040407@talktalk.net> References: <492E75A5.4040407@talktalk.net> Message-ID: <492E889F.9030303@ulb.ac.be> Nigel Guthrie a ?crit : > > I have never known an opponent divulge a previous agreement, in any circumstances. IMO a simpler better rule would be that such information is unauthorised to you. AG : perhaps it's the best theoretical way to solve the problem. But totally unenforceable laws should be avoided. How are you to deduce that there was indeed use of information from an agreement which isn't written anywhere anymore ? Especially as such use might be unconscious. I think that cases where the guess of which wheel fell off will be helped by the knowledge of previous (or other) agreements are quite uncommon and we shouldn't bother about this. If partner asked me always to correct his transfers, and I bid 1NT-2H-3H, he'll have to guess whether : - I took 2H as natural (I play this in one partnership) ; - I'm showing a spade raise, contrary to his demands ; and in this case, why ; and the difference between 3H and 3C/D ; - I pulled 3H in lieu of 2S ; - I realized my 1NT opening was wrong ; That's the ordinary guess situation. The case of the pair formaerly playing Gerber and partner using this knowledge is so uncommon that one had to invent very artificial cases, and not convincing ones. I can't remember of one single case when this knowledge was used. What will happen more frequently is something like this deal from our last match : We play 2D (NV) as Wagner, and quite wild. But 4-th-in-hand (or vulnerable) 2D is a transfer for 2H, usually showing a sound weak 2-bid. We also play (at all vulnerabilities) a 2C opening that includes a major two-suiter with weak-2-bid values and few contra-indications. I passed ; partner opened 2D, NV and 3rd-in-hand. I let you guess what he could have had for such a bid. RHO doubled. I answered 4C, showing a raise to game in both majors and asking partner to transfer. Now partner could see something had gone wrong. I can't have a value raise (I didn't open), nor a distributional raise in both majors (I didn't open 2C) Perhaps many of you would now "guess" that I saw him open 4th-in-hand a very constructive weak 2H, and made a fit-jump. That's using 'other information from the system, past or present', and taking unfair advantage, so they would have felt compelled to tell the opponents their interpretation of the facts. They would have given MI. The explanation was that I held an opening bid and didn't make it for obscure reasons. BTW, partner always takes my (however strange) bids at their systemic value, so he went right. Which shows that, even when there are very strong reasons for considering partner used such-and-such other (or old) part of the system, you can hardly be sure of your guess. Whence the prescript of 'same information to partner and opponents' is abided by, or nearly so. Best regards Alain From agot at ulb.ac.be Thu Nov 27 12:49:19 2008 From: agot at ulb.ac.be (Alain Gottcheiner) Date: Thu, 27 Nov 2008 12:49:19 +0100 Subject: [blml] laissez faire In-Reply-To: <001701c95077$5637d640$15844c59@chello.pl> References: <49227C4E.3050007@skynet.be><000001c94a0b$1cc70330$56550990$@com> <000d01c94ad2$b54bc880$15844c59@chello.pl><4925C17B.1010706@talktalk.net> <49266B37.90306@skynet.be><000f01c94bba$ecc985c0$15844c59@chello.pl> <4926C496.6080904@skynet.be><003901c94c47$a1541140$15844c59@chello.pl> <4927E7CC.4030303@skynet.be><001f01c94cad$4bf57340$15844c59@chello.pl> <49292847.9010802@skynet.be><009901c94f25$c89286c0$15844c59@chello.pl> <492D1356.5040104@skynet.be><002d01c94ff6$5876ce60$15844c59@chello.pl> <492E5AEE.1070707@skynet.be> <001701c95077$5637d640$15844c59@chello.pl> Message-ID: <492E893F.6060503@ulb.ac.be> Bystry a ?crit : > if you've never played Wilkosz you will still have to > explain partner's responses not knowing them. So you will have to invent > something. AG : I don't see how this could be possible. There are three possible cases : 1. You play W, you know what responses are and there is no need to invent. 2. You agreed to play W, but don't know (or don't remember) about that response. You tell them you don't know (or don't remember), and perhaps the TD will send you away from the table and have your partner explain his bid, which will satisfy the requests of L20. Too bad, there is UI. 3. You didn't agree anything about that opening, so the answer is "no agreement". But inventing an explanation because you don't know which it could be will cost you both UI, MI and bad reputation. Best regards Alain From jfusselman at gmail.com Thu Nov 27 14:36:48 2008 From: jfusselman at gmail.com (Jerry Fusselman) Date: Thu, 27 Nov 2008 07:36:48 -0600 Subject: [blml] Two alternatives for the WBF? No, three In-Reply-To: <000e01c95062$f148d8b0$0302a8c0@Mildred> References: <492BF30E.8020808@skynet.be> <492C0D2E.4080607@t-online.de> <492D1593.1040006@skynet.be> <000e01c95062$f148d8b0$0302a8c0@Mildred> Message-ID: <2b1e598b0811270536j3a6224c5o80bbde10422a9111@mail.gmail.com> Grattan wrote: > Is > there an issue in this correspondence that is not decided in > those (or any) WBFLC minutes? Yes. I'll list four. 1. Suppose that your partner's alert or explanation changes your mind as to what the correct explanation of your understanding. When is it correct to use partner's explanation to remind yourself what the understanding is. One case is when you were thinking that 2N is natural until partner's alert reminded you that we almost certainly play Lebensohl here. Now it is time to alert or not alert 3C. 2. What do you explain if you are unsure which is your understanding? One case is 50/50. 3. When choosing what to explain, when is it best to express doubt, and when is it best to keep any doubt you might have to yourself? Once case is 95/5. 4. Do you take into account the possibility that the director may rule that your understanding is different from what you believe it is. For example, there might be two possible understandings, X and Y. Suppose that one or the other is the proper alert. You are 40% sure that your understanding is X, but you think that there is a 60% chance that the director will rule that your understanding is Y. To me, these are crystal-clear questions, but if something needs to be clarified, I would gladly endeavor to do so. It seems to me that the WBF addresses the case where all probabilities in the mind of the explainer are 0 and 1, but the other cases are obviously more realistic. After all, when there is a disagreement, who is ever 100% sure that his own thinking is perfect? And even for those who do believe in their own perfection, the director is the one who decides what the agreements are, and who is 100% sure what a director will rule? Jerry Fusselman From ziffbridge at t-online.de Thu Nov 27 16:25:30 2008 From: ziffbridge at t-online.de (Matthias Berghaus) Date: Thu, 27 Nov 2008 16:25:30 +0100 Subject: [blml] Two alternatives for the WBF? No, three In-Reply-To: <492C17C1.20605@ulb.ac.be> References: <492BF30E.8020808@skynet.be> <492C0D2E.4080607@t-online.de> <492C17C1.20605@ulb.ac.be> Message-ID: <492EBBE9.5030207@t-online.de> Alain Gottcheiner schrieb: > > As an active wargame- and board game player, I've always considered > clarity of the rules one of the most important virtues of a game, Couldn't agree more. I agree with your remarks about L20 and L40, too. Best regards Matthias From Hermandw at skynet.be Thu Nov 27 16:27:44 2008 From: Hermandw at skynet.be (Herman De Wael) Date: Thu, 27 Nov 2008 16:27:44 +0100 Subject: [blml] The Nebraskan issue (really about the methods of change) In-Reply-To: <001301c95076$846f00c0$15844c59@chello.pl> References: <4926FB3B.7060408@skynet.be><003901c94f16$f0976be0$15844c59@chello.pl> <492D0FA7.1000407@skynet.be><001d01c94fe9$22069200$15844c59@chello.pl> <492E5673.9020401@skynet.be> <001301c95076$846f00c0$15844c59@chello.pl> Message-ID: <492EBC70.30801@skynet.be> Hello Maciej, Bystry wrote: > Hi, > >> Hello Maciej, > >>>>> And what do you do with a player who refuses (or forgets) to correct > MI >>>>> after the auction? >>>> But we know what to do with that! It happens all the time, and we deal >>>> with it. We apply the MI laws during the play as well. >>>> Absolutely no comparison with the problem I'm talking about higher - >>>> what is the correction for _not_ having given UI? >>> I'm not obliged to repeat myself. Reread my past emails regarding this >>> issue. >>> >> Well, I don't recall any definite answer to that one. Are you in favour >> of PPs here? > > Yes, mainly. > OK. Have you understood why I believe this is a bad idea? No need to agree, just tell me if you understood. >>>> And how do you, as TD, confirm that the player was indeed certain? >>>> After all, a player cannot go and check - so you must accept that a >>>> player is certain and still turns out to be wrong. But how do you know >>>> that he was really certain when he tells you so? >>> I thought that you are the fan of mindreading as a TD. Show your power. > Or >>> simply use classical "could have known" phrase and punish him whether > he's >>> real villain or not. >>> >> Thereby rendering your previous answer totally useless. >> >> Maciej, you really should think about two steps at the same time. I ask >> yuou a question - you give an answer - and I tell you that answer won't >> work. Then please go back to the question and think about it some more. > > Your logic is nearly impeccable. With one minor flaw. Your claim that my > answer won't work is just your claim, not a definite truth. I have not even > a slightest idea what people in Belgium do. But in Poland there are nearly > no problems. The players just explain their system and more often they break > the Law correcting MI instantly than adopting the dWS. So forgive me, but my > experience shows that your fears are completely unsupported. > There are no problems in Belgium either. Players just play on. But my fears are not about what players do - but about what you would do, as TD, when a player acts in DWS manner. If all you are going to do is give a PP, and then only if a player has done this before (or is called HDW), then your actions (as TD) are exactly the same as mine (except for that lone PP). So why are you then so adamant that DWS is unacceptable, if you accept it on the field? >> What are you going to do about punishing players who act like myself and >> give consistent answers, claiming that they were certain that their >> partner was right all along? > > As a TD I'm obliged to collect evidence and make a judgment. If I find that > e.g. CC states that you were correct, than it is first and strong evidence > that you can be lying. OK. But the fact remains that a player can be certain and still wrong. If you are going to rule on him being wrong, then you are not ruling on him being certain. > There will be usually more evidence available. If it > is first time I've caught you doing something like that you will be educated > and advised not to do that in future. For the second time I may believe or > not but for the for third time you will get PP. Do you really think that > anybody will believe you when you state that you remember your system making > your bid, than forget it hearing your partner's explanation and this happens > regularly? > The problem with that view is that I don't come accross cases like this every week, not even once a year. How then are you going to catch someone doing it for a third time? > Second thing - I was just yesterday reading some past BLML contributions and > I found your own statement (regarding 12C3 as I remember but that's > irrelevant) in which you strongly alleged that the cheaters and liars are > very small minority and we shouldn't really care about them. I > wholeheartedly agree with you. They can be sometimes caught and punished > severely, they are often feeling public pressure and so on. The behavior you > are now describing is exactly that - deliberately breaking the Laws, lying, > cheating. How much pairs do you think will really do this? And some of them > will be caught regardless. > I agree - if the law becomes as you suggest, then it will probably be enough to educate. But it will mean that innocent people will be ruled against, and this for a law change that is not needed! > And one more argument - you are afraid that some, so called, "innocents" > will be punished. Sorry, they are no innocents. Yes there are. I am talking about People who honestly believe their partner is right. And that is just because I was following your rule that only if you are certain partner is right, you are allowed to give a CE. IMO, many more people should be allowed to give CE - I would include those that think their partner is right, even if not 100% certain. If that is the rule (and no-one but you is suggesting that yours is the only possible correct rule), then there are many more innocents. > They break the Laws. First > time they can do this because of ignorance and I accept that. Next time they > do that on purpose and they are cheating. And don't try to persuade my to > believe that people so easily change their minds regarding the > identification of misexplainer. They make a call which is correct. They > believe it is correct. They would need really strong suspicion that they are > wrong. It could be so in novice-expert partnerships but than it is nearly > always the novice who forgets, not an expert. > > Try to look at the things from the statistical point of view, Herman. How > many cases will not be punished? Who says punishment is necessary? > First of all the OS will very often get a > bad board caused by one-side UI and MI. > Than, even if the score is not bad, > the TD can adjust - by an assigned score (rarely), or if it is too hard, by > an artificial score. Practically A+/A- will be the worst score for the NOS > (and the best for the OS). And only those cases can be unfair. Yes, the > justice can't be served in all cases. And it isn't in normal UI, MI, claim > and other rulings. But if you add the PPs for frequent infractors there will > remain only a small handful of such cases. That is an adequate price to pay > for the obtained gain. > The problem with that reasoning is that you talk of an obtained gain. I say there is no such gain. If the CE is allowed, the final result is just the same as if no question is asked. Look at that as the basis. What you are doing, is inventing another law, and then comparing my result to the result under that law. it is you who is acting masochistically, not me who acting illegally. All this of course in a universe in which this law is not yet fixed by the WBF (which I believe it isn't - but that is not the discussion). Our discussion is about the correctness of one law or another. It is not fair of you to dish my law because yours is harsher on OS. Mine is harsh enough, IMO. > [rest sniped, answered above] > >>> I don't care about your fantasies. You're not a preceptor of mine. I'm > used >>> to consider thoroughly any opinion I later submit to the serious > discussion. >>> I made so this time. >>> >> You considered all I wrote above - and maintain your position? >> Remarkable - for two reasons. > > Herman, allow it to get to your mind, only for one short moment, that maybe > you are not right after all. That maybe all those people who disagree with > you have stronger arguments. I know that it's hard when somebody is so > fanatical in trying to force his ideas. But just try. The difference between > us is that I'm not claiming that you are wrong. I'm not sure that I'm right, > too. I just have different goals than you. > > M: MI is the worst for game > H: UI is the worst for game > > M: all the efforts should be put into making things better for the NOS > H: TD's amount of work and the possible life belts for the OS are at least > as much important > > M: the primary obligation of every player is to obey the Laws and to do > everything he can not to damage his opponents > H: the primary obligation of every player is to try as much as he can to > improve his position, even at the cost of the opponents > Unfair. I believe players should act within the laws. But if they act within those laws and try to get the best result, that is allowed. > M: the amount of infractions should be drastically decreased - by education, > by restrictive Laws, by consequent harsh ruling > H: "equity" should be most important, frequent offenders are as much > valuable for game as those who try everything to conform to the rules > There are no frequent offenders in my system - they are only offenders in yours! > M: as much boards as possible should be played normally by the players, not > in TD's consultations and judgment, not in appeals rooms, TD at the table > should be as a last resort > H: TDs at the table are nothing bad, they make good rulings and people are > delighted having been given a ruling instead of their own play > Totally wrong. In your cases the TD has more work than in mine. The MI has not totally gone away, and there are 2 pieces of UI to deal with. In mine, there is only one piece of UI and the second piece of MI is usually harmless (especially when compared or added to the first one). > There may be some more but I'll stop in this point. Of course I might have > misjudged some of your beliefs. I've read many your mails from the last > three or four years and I base my opinions on them. Feel free to correct me > (but using solid arguments not mere denial) and I'll change my mind and even > apologize. > >> Herman. > > Regards > > Maciej > Herman. From ziffbridge at t-online.de Thu Nov 27 16:51:05 2008 From: ziffbridge at t-online.de (Matthias Berghaus) Date: Thu, 27 Nov 2008 16:51:05 +0100 Subject: [blml] Two alternatives for the WBF? No, three In-Reply-To: <492D1593.1040006@skynet.be> References: <492BF30E.8020808@skynet.be> <492C0D2E.4080607@t-online.de> <492D1593.1040006@skynet.be> Message-ID: <492EC1E9.5000707@t-online.de> Herman De Wael schrieb: > > Matthias, I understand what you are trying to do, but that was not the > point of the exercise. You make it sound as if the laws have only one > possible meaning. They haven't. It is up to the WBF to decide where they > want to go. And they have three alternatives, not two. > Writing this as if this has been settled, long ago, and for all time, > does not make it so. > > Herman. Herman, I believe that a set of rules or laws has only one _meaning_ (or, to put it differently, that the meaning of a text is inextricably bound to the intent of the writer. I know that lots of people agree and at least as much disagree with me), but I agree that it can be read in different ways if they writer is not careful enough. What we all want is a text which is as reading-resistant as possible. Next chance in a couple of years, I guess. Best regards Matthias > > _______________________________________________ > blml mailing list > blml at amsterdamned.org > http://www.amsterdamned.org/mailman/listinfo/blml > > From darkbystry at wp.pl Thu Nov 27 16:48:09 2008 From: darkbystry at wp.pl (Bystry) Date: Thu, 27 Nov 2008 16:48:09 +0100 Subject: [blml] laissez faire References: <49227C4E.3050007@skynet.be><000001c94a0b$1cc70330$56550990$@com> <000d01c94ad2$b54bc880$15844c59@chello.pl><4925C17B.1010706@talktalk.net> <49266B37.90306@skynet.be><000f01c94bba$ecc985c0$15844c59@chello.pl> <4926C496.6080904@skynet.be><003901c94c47$a1541140$15844c59@chello.pl> <4927E7CC.4030303@skynet.be><001f01c94cad$4bf57340$15844c59@chello.pl> <49292847.9010802@skynet.be><009901c94f25$c89286c0$15844c59@chello.pl> <492D1356.5040104@skynet.be><002d01c94ff6$5876ce60$15844c59@chello.pl> <492E5AEE.1070707@skynet.be><001701c95077$5637d640$15844c59@chello.pl> <492E7A19.5020000@ulb.ac.be> Message-ID: <002501c950a7$8c88b5e0$15844c59@chello.pl> Hi Alain, > Bystry a ?crit : > if you've never played Wilkosz you will still have to > explain partner's responses not knowing them. So you will have to invent > something. AG : I don't see how this could be possible. There are three possible cases : 1. You play W, you know what responses are and there is no need to invent. MB: No, no. Sorry, I wasn't clear and you mistunderstood me. You are playing Multi. Partner explains it as Wilkosz. That was the situation I was writing about. 2. You agreed to play complete W, but you forgot to ask partner what it meant. Or you don't remember what the response means. You tell them you don't know (or don't remember), and perhaps the TD will send you away from the table and have your partner explain his bid, which will satisfy the requests of L20. Or partner could explain it in written form without letting you look. There is UI. Too bad. MB: You're right. But look above. 3. You didn't agree anything about that opening, so the answer is "no agreement". MB: The same as above. But inventing an explanation because you don't know which it could be will cost you both UI, MI and bad reputation. In short, it goes against the Law. MB: Yes, and that's one reason I don't like dWS. Because you play Multi, have knowledge about 2D natural but never have played Wilkosz. And you're in difficult position when partner misexplains 2D as Wilkosz (he plays it with his other partner) and the opponents question you about his follow-up. Herman claims you should invent something according to dWS. And I see no reason to do it. Best regards Alain PS : don't think the first sentence of # 2 is absurd ; there are umpteen pairs who agreed to play Landy but don't know what everyday bidding sequences (like passing over 1NT 2C X, or responding 2NT or 3C to 2C) would mean. MB: I'm in no doubt you're right. In Poland that is a rarirty because our systems are quite uniform and standard set of conventions is widely known. But I've heard that in the USA it's quite common. Regards Maciej From grandaeval at tiscali.co.uk Thu Nov 27 17:25:52 2008 From: grandaeval at tiscali.co.uk (Grattan) Date: Thu, 27 Nov 2008 16:25:52 -0000 Subject: [blml] Two alternatives for the WBF? No, three References: <492BF30E.8020808@skynet.be> <492C0D2E.4080607@t-online.de> <492C17C1.20605@ulb.ac.be> Message-ID: <00e501c950af$99d0d9f0$0302a8c0@Mildred> Grattan Endicott To: "Bridge Laws Mailing List" Sent: Tuesday, November 25, 2008 3:20 PM Subject: Re: [blml] Two alternatives for the WBF? No, three > > the WBF makes the > rules. You make think this unwise and tell them so, you > may try to make them change their minds, but as long as > they hold certain rights it is their opinion which counts, > not yours or mine. > AG : and surely I can explain them that their rules should be rewritten, even if unchanged, because it is so difficult to interpret them in their current form. << +=+ 1. Yes, certainly you may. 2. About six years ago I would have agreed with this.. I did make a serious attempt to persuade friends that the task should be attempted for 2007. I failed to persuade. There were strong opinions that the previous code had worked well enough. 3. I am not prepared to argue that those who opposed a complete rewrite were wrong. I have no exclusive seal of superior judgement. 4. With refreshed laws instituted in 2007, incorporating such revisions as were agreed, I do not now think it realistic to be arguing for further change at an early date. The strain upon the resources of NBOs etc. inhibits it. So does the strong probability that agreement would not be reached in the places that have the power. ~ Grattan ~ +=+ From darkbystry at wp.pl Thu Nov 27 17:44:52 2008 From: darkbystry at wp.pl (Bystry) Date: Thu, 27 Nov 2008 17:44:52 +0100 Subject: [blml] The real text. References: <49227C4E.3050007@skynet.be><000001c94a0b$1cc70330$56550990$@com><000d01c94ad2$b54bc880$15844c59@chello.pl><4925C17B.1010706@talktalk.net> <49266B37.90306@skynet.be><000f01c94bba$ecc985c0$15844c59@chello.pl><4926C496.6080904@skynet.be><003901c94c47$a1541140$15844c59@chello.pl><4927E7CC.4030303@skynet.be><001f01c94cad$4bf57340$15844c59@chello.pl> <008701c94f22$94d773c0$15844c59@chello.pl> <492D105C.8010805@skynet.be><002501c94fed$21e0e4c0$15844c59@chello.pl> <492E5900.1060401@skynet.be><001d01c95079$bd5e2b60$15844c59@chello.pl> <492E7929.7020405@skynet.be> Message-ID: <002b01c950af$78a8d840$15844c59@chello.pl> Hi Herman, > > CE were always outlawed. > > No they are not. And if you say they are, then SE are also outlawed, and > we're in a dilemma situation. Of course CE are outlawed. I'm sure I don't have to cite the relevant Laws? Or maybe should I cite your own statements? You've always claimed that the dWS is acceptable because the player has to brake one of two Laws and therefore he's free to decide which one. Now it's clear (for me it always was) that the player doesn't brake L20F5a when he uses SE. I'll leave the conclusion to you. And please, Herman. Don't try to wriggle and hedge. I like to discuss with you but not when you're speaking with a forked tongue. > >> And if CEs are being outlawed, why don't I find any penalty in here? > > > > [L40C3] > > > > (b) Repeated violations of requirements to disclose partnership > > understandings may be penalized. > > > > [/L40C3] > > > > HAHAHAHAHAHAHAHA. I've tried many dictionaries - English, Belgian, even Suahili. And heck, I couldn't find it. Maybe it's some local Antwerpian dialect? ;-) > May be penalized? By what? By flagellating. Or by tickling. Or maybe by PPs? > You have a lawbook, and some people say it's incomplete. So you write an > addition, and the same people still say it's incomplete. So you refer to > the lawbook itself? Surely even you must see that this won't work. Hmm. I must admit to something terrible. Forgive me, but I respect TFLB as a source of Laws. I respect official minutes of WBFLC as a source of Laws. But I can't force myself to respect some people's strange ideas as a source of Laws. > And why do you point to "repeated violations"? Despite 10 years of > talking about this, I have never even once encountered a real case like > this at the table - not as player and not even as director. How can > there be repeated violations then? > And even if there were - what is the penalty? Ooo, I love this statement. Thank you, Herman. So you were bellyaching in 10 (or more) emails about the great injustice that would happen to poor PPed innocents. You were trying to put me to shame because of my harshness and bloody desire to hurt those poor souls. You were worrying about the deluge of cases and a myriad of additional work for the TDs. And now you say that you've never encountered such case in 10 years? Despite the fact that (in your opinion) the dWS was not clearly outlawed? Thanks again. That definitely ends our discussions on this topic. :-) > >> I would really like to see a prosecuting attorney at work with this > >> against my actions. > > > > Come to Poland, get your due PPs and than sue me. It will be fun :-) > > > > Indeed it will. What else are you going to use than "Herman always says > he'd break this law"? Creative TD is always able to find a cute reason to PP somebody. > And what shall you use against some other Belgian you have never seen > before? Fleming or Wallon (is it correct form?)? If he were Fleming than I'd say my cousin is a Wallon. If he were Wallon than I'd say my cousin is Fleming. WTP? > Herman. Regards Maciej PS Don't take my statements as ribbing. There is never too much humour. The real discussion should end by the end of first passage. From Hermandw at skynet.be Thu Nov 27 18:06:31 2008 From: Hermandw at skynet.be (Herman De Wael) Date: Thu, 27 Nov 2008 18:06:31 +0100 Subject: [blml] laissez faire In-Reply-To: <002501c950a7$8c88b5e0$15844c59@chello.pl> References: <49227C4E.3050007@skynet.be><000001c94a0b$1cc70330$56550990$@com> <000d01c94ad2$b54bc880$15844c59@chello.pl><4925C17B.1010706@talktalk.net> <49266B37.90306@skynet.be><000f01c94bba$ecc985c0$15844c59@chello.pl> <4926C496.6080904@skynet.be><003901c94c47$a1541140$15844c59@chello.pl> <4927E7CC.4030303@skynet.be><001f01c94cad$4bf57340$15844c59@chello.pl> <49292847.9010802@skynet.be><009901c94f25$c89286c0$15844c59@chello.pl> <492D1356.5040104@skynet.be><002d01c94ff6$5876ce60$15844c59@chello.pl> <492E5AEE.1070707@skynet.be><001701c95077$5637d640$15844c59@chello.pl> <492E7A19.5020000@ulb.ac.be> <002501c950a7$8c88b5e0$15844c59@chello.pl> Message-ID: <492ED397.5000205@skynet.be> Bystry wrote: > > But inventing an explanation because you don't know which it could be > will cost you both UI, MI and bad reputation. In short, it goes against > the Law. > > MB: Yes, and that's one reason I don't like dWS. Because you play Multi, > have knowledge about 2D natural but never have played Wilkosz. And you're in > difficult position when partner misexplains 2D as Wilkosz (he plays it with > his other partner) and the opponents question you about his follow-up. > Herman claims you should invent something according to dWS. And I see no > reason to do it. > Maciej, you are inventing a case just to find an argument. When one partner misexplains something from another, do you really believe he is going to explain something the other has never heard of? So this is a very minor occurence. More frequent is of course that the partner knows the system, but cannot be certain of the particular version. As I said, it is then quite possible to give a CE (consistent explanation) of one version. No invention is necessary. And no UI (of the nature of revealing the MI) is given. Most frequently however, the player will know exactly what version his partner is thinking about, because they do play the convention in some other situation. I don't see why an unimportant negative thing, which happens infrequently, can be a deterrent from using a system. > Best regards > > Alain > > > Maciej > Herman. From Hermandw at skynet.be Thu Nov 27 18:08:13 2008 From: Hermandw at skynet.be (Herman De Wael) Date: Thu, 27 Nov 2008 18:08:13 +0100 Subject: [blml] Two alternatives for the WBF? No, three In-Reply-To: <492EC1E9.5000707@t-online.de> References: <492BF30E.8020808@skynet.be> <492C0D2E.4080607@t-online.de> <492D1593.1040006@skynet.be> <492EC1E9.5000707@t-online.de> Message-ID: <492ED3FD.8060805@skynet.be> Matthias Berghaus wrote: > Herman De Wael schrieb: >> Matthias, I understand what you are trying to do, but that was not the >> point of the exercise. You make it sound as if the laws have only one >> possible meaning. They haven't. It is up to the WBF to decide where they >> want to go. And they have three alternatives, not two. >> Writing this as if this has been settled, long ago, and for all time, >> does not make it so. >> >> Herman. > > Herman, > > I believe that a set of rules or laws has only one _meaning_ (or, to put > it differently, that the meaning of a text is inextricably bound to the > intent of the writer. I know that lots of people agree and at least as > much disagree with me), but I agree that it can be read in different > ways if they writer is not careful enough. > > What we all want is a text which is as reading-resistant as possible. > Next chance in a couple of years, I guess. > And what do we do in the meantime? Discuss another ten years? OK, I'm up for it. > Best regards > Matthias > Herman. From Hermandw at skynet.be Thu Nov 27 18:13:17 2008 From: Hermandw at skynet.be (Herman De Wael) Date: Thu, 27 Nov 2008 18:13:17 +0100 Subject: [blml] The real text. In-Reply-To: <002b01c950af$78a8d840$15844c59@chello.pl> References: <49227C4E.3050007@skynet.be><000001c94a0b$1cc70330$56550990$@com><000d01c94ad2$b54bc880$15844c59@chello.pl><4925C17B.1010706@talktalk.net> <49266B37.90306@skynet.be><000f01c94bba$ecc985c0$15844c59@chello.pl><4926C496.6080904@skynet.be><003901c94c47$a1541140$15844c59@chello.pl><4927E7CC.4030303@skynet.be><001f01c94cad$4bf57340$15844c59@chello.pl> <008701c94f22$94d773c0$15844c59@chello.pl> <492D105C.8010805@skynet.be><002501c94fed$21e0e4c0$15844c59@chello.pl> <492E5900.1060401@skynet.be><001d01c95079$bd5e2b60$15844c59@chello.pl> <492E7929.7020405@skynet.be> <002b01c950af$78a8d840$15844c59@chello.pl> Message-ID: <492ED52D.40209@skynet.be> Bystry wrote: > Hi Herman, > >>> CE were always outlawed. >> No they are not. And if you say they are, then SE are also outlawed, and >> we're in a dilemma situation. > > Of course CE are outlawed. I'm sure I don't have to cite the relevant Laws? No, you don't need to cite the laws. But if that is your definition of outlawing, then the SE are outlawed as well. I'm sure I don't have to cite the relevant laws? So let's use the word outlaw for doing something stronger than the current laws say. As it stands, the CE are not outlawed. > Or maybe should I cite your own statements? You've always claimed that the > dWS is acceptable because the player has to brake one of two Laws and > therefore he's free to decide which one. Now it's clear (for me it always > was) that the player doesn't brake L20F5a when he uses SE. I'll leave the > conclusion to you. > And I still maintain that. The words acceptable and outlawed are antonyms. > And please, Herman. Don't try to wriggle and hedge. I like to discuss with > you but not when you're speaking with a forked tongue. > What forked tongue? >>>> And if CEs are being outlawed, why don't I find any penalty in here? >>> [L40C3] >>> >>> (b) Repeated violations of requirements to disclose partnership >>> understandings may be penalized. >>> >>> [/L40C3] >>> >> HAHAHAHAHAHAHAHA. > > I've tried many dictionaries - English, Belgian, even Suahili. And heck, I > couldn't find it. Maybe it's some local Antwerpian dialect? ;-) > >> May be penalized? By what? > > By flagellating. Or by tickling. Or maybe by PPs? > >> You have a lawbook, and some people say it's incomplete. So you write an >> addition, and the same people still say it's incomplete. So you refer to >> the lawbook itself? Surely even you must see that this won't work. > > Hmm. I must admit to something terrible. Forgive me, but I respect TFLB as a > source of Laws. I respect official minutes of WBFLC as a source of Laws. But > I can't force myself to respect some people's strange ideas as a source of > Laws. > I respect the lawbook as much as you do. It shows me no penalty for giving CEs. Yes, it gives me a penalty in the form of the MI I give, of course. But it does not tell me when, or to whom, or how much, of a PP to give to someone who gives a CE. Herman. From grandaeval at tiscali.co.uk Thu Nov 27 18:48:56 2008 From: grandaeval at tiscali.co.uk (Grattan) Date: Thu, 27 Nov 2008 17:48:56 -0000 Subject: [blml] Two alternatives for the WBF? No, three References: <492BF30E.8020808@skynet.be> <492C0D2E.4080607@t-online.de><492D1593.1040006@skynet.be> <000e01c95062$f148d8b0$0302a8c0@Mildred> <2b1e598b0811270536j3a6224c5o80bbde10422a9111@mail.gmail.com> Message-ID: <010401c950b8$6eb68630$0302a8c0@Mildred> Grattan Endicott To: "Bridge Laws Mailing List" Sent: Thursday, November 27, 2008 1:36 PM Subject: Re: [blml] Two alternatives for the WBF? No, three > Grattan wrote: >> Is >> there an issue in this correspondence that is not decided in >> those (or any) WBFLC minutes? > > Yes. I'll list four. > > 1. Suppose that your partner's alert or explanation changes your mind > as to what the correct explanation of your understanding. When is it > correct to use partner's explanation to remind yourself what the > understanding is. One case is when you were thinking that 2N is > natural until partner's alert reminded you that we almost certainly > play Lebensohl here. Now it is time to alert or not alert 3C. > > 2. What do you explain if you are unsure which is your understanding? > One case is 50/50. > > 3. When choosing what to explain, when is it best to express doubt, > and when is it best to keep any doubt you might have to yourself? > Once case is 95/5. > > 4. Do you take into account the possibility that the director may > rule that your understanding is different from what you believe it is. > For example, there might be two possible understandings, X and Y. > Suppose that one or the other is the proper alert. You are 40% sure > that your understanding is X, but you think that there is a 60% chance > that the director will rule that your understanding is Y. > > To me, these are crystal-clear questions, but if something needs to be > clarified, I would gladly endeavor to do so. > > It seems to me that the WBF addresses the case where all probabilities > in the mind of the explainer are 0 and 1, but the other cases are > obviously more realistic. After all, when there is a disagreement, > who is ever 100% sure that his own thinking is perfect? And even for > those who do believe in their own perfection, the director is the one > who decides what the agreements are, and who is 100% sure what a > director will rule? > > Jerry Fusselman > +=+ 1. My first thought is that here are prime examples of situations in which the Director should be called upon to advise players of their responsibilities (see Law 81C2). So ask the Director. 2. If for some reason the Director cannot be summoned then the law requires that the player shall give correct information as to his partnership agreement. The player must decide what he believes it to be and respond to enquiry accordingly. Both in this and under 1 above he may well be required subsequently to justify his explanation to the Director. 3. Players worldwide who are gifted with a measure of cynicism will have little difficulty in accepting that, if the explainer has changed his mind in the course of the matter and the change appears to favour his side's cause, the Director will be highly sceptical in hearing the player's account and will require some convincing of what he is told. ~ Grattan ~ +=+ From grandaeval at tiscali.co.uk Thu Nov 27 19:01:29 2008 From: grandaeval at tiscali.co.uk (Grattan) Date: Thu, 27 Nov 2008 18:01:29 -0000 Subject: [blml] The real text. References: <49227C4E.3050007@skynet.be><000001c94a0b$1cc70330$56550990$@com><000d01c94ad2$b54bc880$15844c59@chello.pl><4925C17B.1010706@talktalk.net> <49266B37.90306@skynet.be><000f01c94bba$ecc985c0$15844c59@chello.pl><4926C496.6080904@skynet.be><003901c94c47$a1541140$15844c59@chello.pl><4927E7CC.4030303@skynet.be><001f01c94cad$4bf57340$15844c59@chello.pl> <008701c94f22$94d773c0$15844c59@chello.pl> <492D105C.8010805@skynet.be><002501c94fed$21e0e4c0$15844c59@chello.pl> <492E5900.1060401@skynet.be><001d01c95079$bd5e2b60$15844c59@chello.pl> <492E7929.7020405@skynet.be><002b01c950af$78a8d840$15844c59@chello.pl> <492ED52D.40209@skynet.be> Message-ID: <011101c950ba$3163c110$0302a8c0@Mildred> Grattan Endicott To: "Bridge Laws Mailing List" Sent: Thursday, November 27, 2008 5:13 PM Subject: Re: [blml] The real text. >>the player has to break one of two Laws and >> therefore he's free to decide which one. << +=+ This is a false statement. He must act as authority directs, not as he chooses himself. ~ Grattan ~ +=+ From darkbystry at wp.pl Thu Nov 27 19:06:51 2008 From: darkbystry at wp.pl (Bystry) Date: Thu, 27 Nov 2008 19:06:51 +0100 Subject: [blml] The Nebraskan issue (really about the methods of change) References: <4926FB3B.7060408@skynet.be><003901c94f16$f0976be0$15844c59@chello.pl> <492D0FA7.1000407@skynet.be><001d01c94fe9$22069200$15844c59@chello.pl> <492E5673.9020401@skynet.be><001301c95076$846f00c0$15844c59@chello.pl> <492EBC70.30801@skynet.be> Message-ID: <005201c950ba$ec9fbb00$15844c59@chello.pl> Hi Herman, > >> Well, I don't recall any definite answer to that one. Are you in favour > >> of PPs here? > > > > Yes, mainly. > > > > OK. Have you understood why I believe this is a bad idea? No need to > agree, just tell me if you understood. Yes, I have. [much sniped, resolved in other email] > Yes there are. I am talking about People who honestly believe their > partner is right. And that is just because I was following your rule > that only if you are certain partner is right, you are allowed to give a > CE. IMO, many more people should be allowed to give CE - I would include > those that think their partner is right, even if not 100% certain. If > that is the rule (and no-one but you is suggesting that yours is the > only possible correct rule), then there are many more innocents. The Laws don't state what should you do when you're uncertain. So here your view is as valuable as mine. My opinion (that it's better to keep to one's own version unless certain it's incorrect) is based on those arguments: - it's easier to keep to one's own version because that's what you clearly know and you are not forced to guess - if you are consitent, it looks ok in the eyes of the opponents and the TD. If you change your version and later it turns out you were initially right, it may look as you were trying something unfair - there is no need for a TD during the auction (his arrival frequently engenders emotions and the future auction may be influenced by this) Of course I may easily accept other, contradictory arguments. For me it just looks fine. > The problem with that reasoning is that you talk of an obtained gain. I > say there is no such gain. If the CE is allowed, the final result is > just the same as if no question is asked. Look at that as the basis. And so what? Maybe you should demand total banishment of questions? That would be consinsent because the first question already creates UI for your side and only without it the final result would be good enough to satisfy you. Herman, accept that it is your side who is responsible for your supposed poor score. Not the opponents, who have every right to get full disclosure. You forget your system - you usually get bad scores. I don't see any reason why to debate in thousands of emails, why to reject the official decisions of authorities, why to try to put your arguments over and over. Instead of this, use one percent of wasted time to learn your system and you won't have any problems. > What you are doing, is inventing another law, and then comparing my > result to the result under that law. it is you who is acting > masochistically, not me who acting illegally. Herman, I'll be brutal but I'm sick of it all. I'm not inventing any Law. *You* are doing it. I'm just reading the actual Laws and it's interpretations made by authorised body, WBFLC. *They* are the authorities ,*they* have necessary knowledge and skills, *they* are accredited to make decisions, not *you*, ordinary TD with some ideas. These are my last words regarding the topic of what the Laws are at present. If you wish so, continue to live in your glass sphere. > > M: the primary obligation of every player is to obey the Laws and to do > > everything he can not to damage his opponents > > H: the primary obligation of every player is to try as much as he can to > > improve his position, even at the cost of the opponents > > > > Unfair. I believe players should act within the laws. But if they act > within those laws and try to get the best result, that is allowed. I didn't suggest that you approve of law-breaking. I'm sure you don't. I'll explain in a more clear way. M: if the player stays within the Laws he should certainly do all he can to defeat his opponents H: I think that you agree M: if the player breaks any Law he should do all he can to ensure that his infraction will have the least possible effect on the innocent side, even if it costs him more than it could H: such player should accept the penalty (present or future) but he can do all that is possible to lessen the effects of this penalty on himself, even if it harms the innocent side I emphasise that it is only my own idea of how the game of Bridge should work. > > M: the amount of infractions should be drastically decreased - by education, > > by restrictive Laws, by consequent harsh ruling > > H: "equity" should be most important, frequent offenders are as much > > valuable for game as those who try everything to conform to the rules > > > > There are no frequent offenders in my system - they are only offenders > in yours! I was using "offenders" as a global term, not just restricted to dWS. > > M: as much boards as possible should be played normally by the players, not > > in TD's consultations and judgment, not in appeals rooms, TD at the table > > should be as a last resort > > H: TDs at the table are nothing bad, they make good rulings and people are > > delighted having been given a ruling instead of their own play > > > > Totally wrong. In your cases the TD has more work than in mine. The MI > has not totally gone away, and there are 2 pieces of UI to deal with. In > mine, there is only one piece of UI and the second piece of MI is > usually harmless (especially when compared or added to the first one). Yes, the TD has more work but the board is played normally (the score will rarely be adjusted) and only the offenders have to explain their actions (and are ruled against UI), the NOS can calmly concentrate on the following boards and they don't have to waste their time persuading the TD (AC) that they would do something or not. > Herman. Regards Maciej From darkbystry at wp.pl Thu Nov 27 19:15:19 2008 From: darkbystry at wp.pl (Bystry) Date: Thu, 27 Nov 2008 19:15:19 +0100 Subject: [blml] The real text. References: <49227C4E.3050007@skynet.be><000001c94a0b$1cc70330$56550990$@com><000d01c94ad2$b54bc880$15844c59@chello.pl><4925C17B.1010706@talktalk.net> <49266B37.90306@skynet.be><000f01c94bba$ecc985c0$15844c59@chello.pl><4926C496.6080904@skynet.be><003901c94c47$a1541140$15844c59@chello.pl><4927E7CC.4030303@skynet.be><001f01c94cad$4bf57340$15844c59@chello.pl> <008701c94f22$94d773c0$15844c59@chello.pl> <492D105C.8010805@skynet.be><002501c94fed$21e0e4c0$15844c59@chello.pl> <492E5900.1060401@skynet.be><001d01c95079$bd5e2b60$15844c59@chello.pl> <492E7929.7020405@skynet.be><002b01c950af$78a8d840$15844c59@chello.pl> <492ED52D.40209@skynet.be> Message-ID: <006801c950bc$1b7cb080$15844c59@chello.pl> [sniped, as promised] > I respect the lawbook as much as you do. It shows me no penalty for > giving CEs. Yes, it gives me a penalty in the form of the MI I give, of > course. But it does not tell me when, or to whom, or how much, of a PP > to give to someone who gives a CE. L40C3 states that the TD can penalise CE. It doesn't measure the size of penalty. But show me *any* Law that states how big a PP should be for anything. Do it or next time rethink your arguments before you send an email. > Herman. Regards Maciej From nigelguthrie at talktalk.net Thu Nov 27 20:08:26 2008 From: nigelguthrie at talktalk.net (Nigel Guthrie) Date: Thu, 27 Nov 2008 19:08:26 +0000 Subject: [blml] Prior argument [SEC=UNOFFICIAL] In-Reply-To: <492E889F.9030303@ulb.ac.be> References: <492E75A5.4040407@talktalk.net> <492E889F.9030303@ulb.ac.be> Message-ID: <492EF02A.60700@talktalk.net> [Alain Gottcheiner] I think that cases where the guess of which wheel fell off will be helped by the knowledge of previous (or other) agreements are quite uncommon and we shouldn't bother about this ... [Nigel] Uncommon but such cases cause controversy - - A DWS explanation is usually an explanation of a default agreement. - When you suspect partner of psyching, it helps to know his habits. For example suppose LHO opens 1H, partner overcalls 1N, and RHO doubles. A dodgy 1N overcall may be the kind of psych that partner has used in the past. Your knowledge of partner's habits may indicate that it is he who is more likely to be messing around than either opponent. Furthermore, you may know the kind of hand that partner prefers for this kind of gambit (for instance, values for a weak two). If authorised, this information will help you, in both the bidding and the play. - Other contexts, with and without alerts and explanations, when you suspect partner may have made a bidding mistake. For example, partner may be prone to sorting his diamonds into his hearts. If information from prior agreements and partner's idiosyncrasies were *unauthorised* to you, then these situations would be legally simpler. From nigelguthrie at talktalk.net Thu Nov 27 20:10:29 2008 From: nigelguthrie at talktalk.net (Nigel Guthrie) Date: Thu, 27 Nov 2008 19:10:29 +0000 Subject: [blml] Prior argument [SEC=UNOFFICIAL] In-Reply-To: <492E889F.9030303@ulb.ac.be> References: <492E75A5.4040407@talktalk.net> <492E889F.9030303@ulb.ac.be> Message-ID: <492EF0A5.9050905@talktalk.net> [Alain Gottcheiner] ... Perhaps many of you would now "guess" that I saw him open 4th-in-hand a very constructive weak 2H, and made a fit-jump. That's using 'other information from the system, past or present', and taking unfair advantage, so they would have felt compelled to tell the opponents their interpretation of the facts. They would have given MI. The explanation was that I held an opening bid and didn't make it for obscure reasons. BTW, partner always takes my (however strange) bids at their systemic value, so he went right. Which shows that, even when there are very strong reasons for considering partner used such-and-such other (or old) part of the system, you can hardly be sure of your guess. Whence the prescript of 'same information to partner and opponents' is abided by, or nearly so. [Nigel] I agree with Alain, that you are rarely certain of *prior understandings*. And, just because partner has a distinct style and *idiosyncrasies*, that does not mean that you can predict all his behaviour. Nevertheless, IMO, if such knowledge gives you a statistical advantage over opponents, then either - you should disclose the information. - or it should not be authorised to you. From darkbystry at wp.pl Thu Nov 27 21:28:41 2008 From: darkbystry at wp.pl (Bystry) Date: Thu, 27 Nov 2008 21:28:41 +0100 Subject: [blml] Prior argument [SEC=UNOFFICIAL] References: <492E75A5.4040407@talktalk.net><492E889F.9030303@ulb.ac.be> <492EF02A.60700@talktalk.net> Message-ID: <000901c950ce$bcae62c0$15844c59@chello.pl> Hi, > If information from prior agreements and partner's idiosyncrasies were > *unauthorised* to you, then these situations would be legally simpler. Nigel, I admire your efforts because you have your heart on the right side. But try to refrain yourself from such strange ideas. How on earth would this situation be legally simpler? Only in a way of *never* ruling against breaking such rule. Most of the UI we're currently dealing with is the result of the table actions - hesitations, behavior, explanations, withdrawn calls etc. And in most cases this UI can be recognized by the opponents (or TD). Now how do you imagine UI from your prior agreements may come to light? Do you expect the opponents to know your past bidding agreements? Or your partner's idiosyncrasies? The only thing to achieve this way is having the bidders to continue using those interferences but denying the opponents' access to them. Regards Maciej From richard.hills at immi.gov.au Thu Nov 27 23:23:11 2008 From: richard.hills at immi.gov.au (richard.hills at immi.gov.au) Date: Fri, 28 Nov 2008 09:23:11 +1100 Subject: [blml] pugio tuus est interfector tuus [SEC=UNOFFICIAL] In-Reply-To: Message-ID: "...the curious incident of the dog in the night-time." "The dog did nothing in the night-time." "That was the curious incident." Imps Dlr: West Vul: North-South You, West, hold: 952 KT3 43 T9864 The bidding has gone: WEST NORTH EAST SOUTH Pass Pass 1H 2S (1) 2H (2) Pass Pass Pass(3) (1) Weak, circa 5-9 hcp and a 6-card suit (2) Director summoned, North declines his Law 27A option (3) Mandatory pass by East under Law 27B2 What opening lead do you make? What other opening lead do you consider making? * * * None of the blmlers responding to this problem mentioned the curious incident of footnote two-and-a-half (which I deliberately omitted as a test of acuity). At the table the acute Director did not omit mentioning Law 26A2 to the table in general, and to West in particular. West was advised that unless he made a call promising hearts during the legal auction, then if East-West became defenders East would have a lead restriction when East first gained the lead (including the opening lead). And that restriction would be that declarer could either: (a) demand that East lead a heart, or (b) prohibit East from leading a heart **for as long as East retained the lead**. Dummy held Qx of hearts, declarer held Jx of hearts. At the table West foolishly led the normal three of hearts. After East won the ace of hearts, declarer (moi) elected to prevent East from again leading hearts while East retained the lead. East postponed the inevitable by cashing a few winners, but was unable to tunnel over to the West hand in a non-heart suit so that West could cash the king of hearts. So eventually I was able to discard my heart loser on a winner in dummy and score up +110. If West had carefully considered the authorised information from the Director's explanation of Law 26A2, West might have chosen as his opening lead the one card that defeats 2S -- the king of hearts! Best wishes Richard Hills, Aqua 5, workstation W550 Telephone: 02 6223 8453 Email: richard.hills at immi.gov.au Recruitment Section & DIAC Social Club movie tickets -------------------------------------------------------------------- Important Notice: If you have received this email by mistake, please advise the sender and delete the message and attachments immediately. This email, including attachments, may contain confidential, sensitive, legally privileged and/or copyright information. Any review, retransmission, dissemination or other use of this information by persons or entities other than the intended recipient is prohibited. DIAC respects your privacy and has obligations under the Privacy Act 1988. The official departmental privacy policy can be viewed on the department's website at www.immi.gov.au. See: http://www.immi.gov.au/functional/privacy.htm --------------------------------------------------------------------- From richard.hills at immi.gov.au Fri Nov 28 00:27:04 2008 From: richard.hills at immi.gov.au (richard.hills at immi.gov.au) Date: Fri, 28 Nov 2008 10:27:04 +1100 Subject: [blml] One alternative for the WBF [SEC=UNOFFICIAL] In-Reply-To: <00e501c950af$99d0d9f0$0302a8c0@Mildred> Message-ID: Grattan Endicott: [snip] > 4. With refreshed laws instituted in 2007, incorporating >such revisions as were agreed, I do not now think it realistic >to be arguing for further change at an early date. The strain >upon the resources of NBOs etc. inhibits it. So does the >strong probability that agreement would not be reached in >the places that have the power. > ~ Grattan ~ +=+ Richard Hills: As a resourceful etc, I note that assisting Grattan with the version control of the drafts, redrafts and re-redrafts of the 2007 Lawbook was entertaining. But the strain of so doing would inhibit me from volunteering again if new editions of the Lawbook were accelerated from decennial to annual events. Best wishes Richard Hills, Aqua 5, workstation W550 Telephone: 02 6223 8453 Email: richard.hills at immi.gov.au Recruitment Section & DIAC Social Club movie tickets -------------------------------------------------------------------- Important Notice: If you have received this email by mistake, please advise the sender and delete the message and attachments immediately. This email, including attachments, may contain confidential, sensitive, legally privileged and/or copyright information. Any review, retransmission, dissemination or other use of this information by persons or entities other than the intended recipient is prohibited. DIAC respects your privacy and has obligations under the Privacy Act 1988. The official departmental privacy policy can be viewed on the department's website at www.immi.gov.au. See: http://www.immi.gov.au/functional/privacy.htm --------------------------------------------------------------------- From richard.hills at immi.gov.au Fri Nov 28 03:49:54 2008 From: richard.hills at immi.gov.au (richard.hills at immi.gov.au) Date: Fri, 28 Nov 2008 13:49:54 +1100 Subject: [blml] pugio tuus est interfector tuus [SEC=UNOFFICIAL] In-Reply-To: Message-ID: >Dummy held Qx of hearts, declarer held Jx of hearts. At >the table West foolishly led the normal three of hearts. >After East won the ace of hearts, declarer (moi) elected >to prevent East from again leading hearts while East >retained the lead. East postponed the inevitable by >cashing a few winners, but was unable to tunnel over to >the West hand in a non-heart suit so that West could >cash the king of hearts. So eventually I was able to >discard my heart loser on a winner in dummy and score >up +110. > >If West had carefully considered the authorised >information from the Director's explanation of Law 26A2, >West might have chosen as his opening lead the one card >that defeats 2S -- the king of hearts! Addendum: West choosing the opening lead of the king of hearts is necessary but not sufficient to defeat 2S. West must be careful not to lead a heart to East's ace at trick two, since then I could exercise my other Law 26A2 option and demand that East give me a ruff-and-sluff by playing a third round of hearts. Best wishes Richard Hills, Aqua 5, workstation W550 Telephone: 02 6223 8453 Email: richard.hills at immi.gov.au Recruitment Section & DIAC Social Club movie tickets -------------------------------------------------------------------- Important Notice: If you have received this email by mistake, please advise the sender and delete the message and attachments immediately. This email, including attachments, may contain confidential, sensitive, legally privileged and/or copyright information. Any review, retransmission, dissemination or other use of this information by persons or entities other than the intended recipient is prohibited. DIAC respects your privacy and has obligations under the Privacy Act 1988. The official departmental privacy policy can be viewed on the department's website at www.immi.gov.au. See: http://www.immi.gov.au/functional/privacy.htm --------------------------------------------------------------------- From richard.hills at immi.gov.au Fri Nov 28 05:05:48 2008 From: richard.hills at immi.gov.au (richard.hills at immi.gov.au) Date: Fri, 28 Nov 2008 15:05:48 +1100 Subject: [blml] pugio tuus est interfector tuus [SEC=UNOFFICIAL] In-Reply-To: Message-ID: >>If West had carefully considered the authorised >>information from the Director's explanation of Law 26A2, >>West might have chosen as his opening lead the one card >>that defeats 2S -- the king of hearts! >Addendum: West choosing the opening lead of the king of >hearts is necessary but not sufficient to defeat 2S. > >West must be careful not to lead a heart to East's ace at >trick two, since then I could exercise my other Law 26A2 >option and demand that East give me a ruff-and-sluff by >playing a third round of hearts. Addendum to addendum: If East-West must immediately cash two hearts in order to prevent a discard of a heart loser, and if East-West cashing three rounds of hearts allows a ruff-sluff discard of a diamond or club loser, then Law 26A2 and Morton's Fork mean that 2S is cold. The truth was somewhat more prosaic; West leading the three of hearts rather than the king of hearts made it easier for East to misdefend (since East thought it might be possible that South held KJ doubleton of hearts). The complete deal: 54 Q4 AQT985 J75 952 A3 KT3 A98752 43 KJ T9864 AK2 KQJT76 J6 762 Q3 After West's opening lead of the three of hearts was won by East's ace, and I used Law 26A2 to prohibit East from leading another heart while East retained the lead, then East's 100% best line of defence is to immediately stop retaining the lead by switching to the trey of trumps. Instead East opted for the 0% defensive line of ace, king and deuce of clubs. Note that Maciej Bystry, who as West preferred changing the insufficient 2H to 3H, would have been a big winner at the table, since 4H does not need any Law 26A2 rectification to make. Best wishes Richard Hills, Aqua 5, workstation W550 Telephone: 02 6223 8453 Email: richard.hills at immi.gov.au Recruitment Section & DIAC Social Club movie tickets -------------------------------------------------------------------- Important Notice: If you have received this email by mistake, please advise the sender and delete the message and attachments immediately. This email, including attachments, may contain confidential, sensitive, legally privileged and/or copyright information. Any review, retransmission, dissemination or other use of this information by persons or entities other than the intended recipient is prohibited. DIAC respects your privacy and has obligations under the Privacy Act 1988. The official departmental privacy policy can be viewed on the department's website at www.immi.gov.au. See: http://www.immi.gov.au/functional/privacy.htm --------------------------------------------------------------------- From rfrick at rfrick.info Fri Nov 28 06:25:48 2008 From: rfrick at rfrick.info (Robert Frick) Date: Fri, 28 Nov 2008 00:25:48 -0500 Subject: [blml] One alternative for the WBF [SEC=UNOFFICIAL] In-Reply-To: References: Message-ID: On Thu, 27 Nov 2008 18:27:04 -0500, wrote: > Grattan Endicott: > > [snip] > >> 4. With refreshed laws instituted in 2007, incorporating >> such revisions as were agreed, I do not now think it realistic >> to be arguing for further change at an early date. The strain >> upon the resources of NBOs etc. inhibits it. So does the >> strong probability that agreement would not be reached in >> the places that have the power. >> ~ Grattan ~ +=+ > > Richard Hills: > > As a resourceful etc, I note that assisting Grattan with the > version control of the drafts, redrafts and re-redrafts of the > 2007 Lawbook was entertaining. But the strain of so doing > would inhibit me from volunteering again if new editions of > the Lawbook were accelerated from decennial to annual events. Hi Richard. Are you proposing to stop this argument and leave the laws alone? That would be great for me. Herman asked the WBFLC to sanctify the current practices of players and directors, but I just suggested leaving it alone and I am guessing Herman would accept that too. For example, the laws say regarding mistaken explanations "nor may he indicate in any manner that a mistake has been made." So you can try to change "manner" to "mannerisms" in 2018, or you can add "...except in giving explanations of partner's bids." in 2018. But you agree to leave it alone until then? Are you authorized to make that offer? If I agree to this, can I fight against changes that have been made since the 2008 laws come out? I still think the don't-have-tell-them-responses-to-Blackwood opinion isn't going to work. There is nothing wrong with it except most players think it isn't right and don't want to play that way. As near as I can tell. But I won't be asking the WBFLC to make a different opinion. From richard.hills at immi.gov.au Fri Nov 28 07:38:02 2008 From: richard.hills at immi.gov.au (richard.hills at immi.gov.au) Date: Fri, 28 Nov 2008 17:38:02 +1100 Subject: [blml] One alternative for the WBF [SEC=UNOFFICIAL] In-Reply-To: Message-ID: Richard Hills, typist: >>As a resourceful etc, I note that assisting Grattan with the >>version control of the drafts, redrafts and re-redrafts of the >>2007 Lawbook was entertaining. But the strain of so doing >>would inhibit me from volunteering again if new editions of >>the Lawbook were accelerated from decennial to annual events. Robert Frick, misconstruer: >Hi Richard. Are you proposing to stop this argument and leave >the laws alone? > >Are you authorized to make that offer? Richard Hills, typist: I have zero bureaucratic bridge authority at the moment. Even when my authority was at its peak, it was merely that of President of the Bridge Federation of the Australian Capital Territory (and even then I often got outvoted in committee meetings seven votes to one). As a proof-reader and indexer of the drafts of the Lawbook I had zero decision-making power, as that rested with the Drafting Committee and eventually with the WBF Executive. Robert Frick, misconstruer: >For example, the laws say regarding mistaken explanations "nor >may he indicate in any manner that a mistake has been made." >..... >or you can add "...except in giving explanations of partner's >bids." in 2018. But you agree to leave it alone until then? Richard Hills, typist: I have zero bureaucratic bridge authority at the moment. But the WBF Laws Committee does have bureaucratic bridge authority under the WBF Regulations to interpret the Laws. The 1987 and 1997 Law 75D2 (now the 2007 Law 20F5(a)) was in antediluvian times (prior to the great flood of emails which commenced ten years ago) assumed by almost all Directors to contain the implied statement "...except in giving explanations of partner's bids," since otherwise the Lawbook would be self-contradictory. An ever-increasing number of players across several countries had no objection to interpreting the Lawbook in an inconsistent and self-contradictory way, such as ACBLer Steve Willner and Polish player Konrad Ciborowski. This may have been the reason (as a mere former typist I do not know for sure) that the common assumption about Law 20F5(a) -- and other common assumptions about MI and UI -- became part of the official October minute of the WBF Laws Committee. This minute is legally equivalent to the WBF inserting an erratum in the Laws. See attached. Best wishes Richard Hills, Aqua 5, workstation W550 Telephone: 02 6223 8453 Email: richard.hills at immi.gov.au Recruitment Section & DIAC Social Club movie tickets * * * LAW 16 and others - concerning information rights 16A1(d) allows the player use of his memory of information in the laws and regulations. It does not authorize him to look during the auction and play at the printed regulations, the law book, or anyone's scorecard or the backs of bidding cards etc. as (Law 40C3(a)) an aid to memory. For system card and notes see Law 20G2. Neither does 78D authorize players to consult during the auction and play printed copies of the information given them under this law. 20F1 defines the manner in which, during the auction and play, a player may request and receive an explanation of the opponents' prior auction. At this time he is entitled to an explanation only of calls actually made, relevant available alternative calls not made, and any partnership understanding as to inferences from the choice of action among the foregoing. (An "alternative" call is not the same call with another meaning - for example, if the reply to an opponent is that "5D shows diamonds preference", any reply to a further question "what would it mean if 4NT were Blackwood ?" is given voluntarily and not as a requirement of Law 20F1.) 81C2 requires the Director to advise players of their rights and responsibilities under the laws. He confines such information to rights and responsibilities that are relevant to the situation he is dealing with. * * * LAW 20 There is no infraction when a correct explanation discloses that partner's prior explanation was mistaken. The words "nor may he indicate in any manner that a mistake has been made" (in Law 20F5(a)) do not refer to compliance with the overriding requirement of the laws always to respond to enquiries under Law 20F with correct explanations of the partnership understandings. * * * LAW 75C The phrase "they have no claim to an accurate description of the N-S hands" first appeared in the 1975 laws of the game. It was accompanied then as now by the injunction forbidding the Director to alter the table result. It was entered primarily to establish beyond doubt that that the partnership agreement must be described accurately in response to lawful enquiry and that the explanation given must not aim to describe what the explainer believes as to the contents of either hand. It was continued in those terms in the 1987 law book, while for 2007 NBOs were invited to say whether the example or the wording should be updated. Among replies received there was a general consensus for retaining them as they had been previously, whilst moving the statements from a footnote into the body of the Law. -------------------------------------------------------------------- Important Notice: If you have received this email by mistake, please advise the sender and delete the message and attachments immediately. This email, including attachments, may contain confidential, sensitive, legally privileged and/or copyright information. Any review, retransmission, dissemination or other use of this information by persons or entities other than the intended recipient is prohibited. DIAC respects your privacy and has obligations under the Privacy Act 1988. The official departmental privacy policy can be viewed on the department's website at www.immi.gov.au. See: http://www.immi.gov.au/functional/privacy.htm --------------------------------------------------------------------- From rfrick at rfrick.info Fri Nov 28 08:15:46 2008 From: rfrick at rfrick.info (Robert Frick) Date: Fri, 28 Nov 2008 02:15:46 -0500 Subject: [blml] Two alternatives for the WBF? No, three In-Reply-To: <492BF30E.8020808@skynet.be> References: <492BF30E.8020808@skynet.be> Message-ID: Hi Herman. Players can give the SE (what partner's bid should have meant), the CE (what partner thought his bid meant), or both. You didn't mention the Both option. Both is probably impractical. But read on. I did not want to complicate your argument needlessly. But now I think this actually helps it. SE is mandated by any obvious reading of Law 20F5(a). As of course you note. Actually, this laws mandates not CE, leaving SE by default. The MS has argued for CE. Law 75 seems to be representative and/or their best case, though I do not understand their position well enough to actually characterize it. I think their understanding is that this law also clearly states that CE is not required. Most bridge players seem to think that after the auction is over, the opponents are entitled to both SE and CE. Most of the exceptions don't particular want to play bridge with the opponents entitled to one. I will argue that once partner misunderstands the auction, and say is treating 2Di as multi, the partership agreements, etc. (thanks Nigel!) concerning responses to 2Di become relevant. Hence by Law 20F1, both must be divulged. But at least in ACBL-land, Conditions of Contest is, roughly, complete divulgence following any question. Complete divulgence isn't practical in some cases, but I think you just do your best and get rectified when that isn't good enough. That means that as most people interpret Law 20F1, both expanations are also required during the auction in ACBL-land. So players in ACBL-land, or anywhere with similar divulgence regulations, are faced with three laws all saying different things. Then it becomes pretty reasonable that Law 72B1 doesn't apply. Bob, who would prefer that his position not be called the Bob School On Tue, 25 Nov 2008 07:43:58 -0500, Herman De Wael wrote: > Over the past few days, I've come to realise why there is such an > animosity over the DWS. I believe that many of us (me included) do not > realise that the WBF actually has three possible choices. Most of you > probably think I am in favour of one choice, while I am actually in > favour of another one. While I may believe that you are in favour of a > third one, when perhaps that second choice might be good enough for you > as well. > > Allow me to explain. > > First of all, let's get rid of those silly acronyms DWS and MS. Let's > focus on the choice the player at the table must make, and give sensible > names to the options available. > > Basically there are two options: > > - a player can explain his partner's call in the same system as partner > has previously explained his own call. Let me call that the CE, the > Consistent Explanation; > - or the player can explain his partner's call in the system that he > himself believes the pair are playing, let's call that the SE, the > Systemic Explanation. > > There is a third option, which is the one Stephanie proposed, and which > explains according to the own system, even when the player has meanwhile > realised that his partner's explanation was the correct one after all. > That option basically gives the same explanation as the SE, and I shall > not use it furthermore. > > These two options are mutually exclusive. Anything that is added to the > CE which reveals that the previous explanation was not correct, triggers > in opponents a series of questions which will ultimately lead in both > the CE and the SE being known to them, and UI being given to partner. > > So the player has two choices. > > But the WBF has twice a choice to make: > > The WBF can outlaw or accept the CE. > The WBF can outlaw or accept the SE. > > Remember that since neither explanation conforms to all the laws in the > book, there is no option to declare the explanation "legal". There is > merely an option to declare it "acceptable". The opposite of accepting > is outlawing. > > Combined, these options make for four alternatives, one of which is > clearly nonsensical. To outlaw both explanations means a player has no > more options left. He will have to choose one option anyway, and to call > either option unacceptable is ridiculous. > > So there are three alternatives: > > A) to outlaw the CE > B) to outlaw the SE > C) to call both the CE and SE acceptable > > I believe many of you think I advocate option B. I have often stated > what is wrong with the SE, and this may make you believe I would like to > see it outlawed. I don't. I don't think there is any reason to outlaw > the SE. Just apply the MI and UI laws and everyone is happy. > > I may have believed that most of you advocate option A. You have often > told us why you believe the CE is unethical and wrong. But maybe you > don't think it should be outlawed per se. After all, maybe some of you > have meanwhile seen that apart from applying the UI and MI laws on the > CE, nothing else can be done. > > Which is why I think we should work to see the WBF accept alternative C, > which is to allow players to make their own choices, standing ready to > apply MI and UI laws over the choices made. > > Comments? > > _______________________________________________ > blml mailing list > blml at amsterdamned.org > http://www.amsterdamned.org/mailman/listinfo/blml From rfrick at rfrick.info Fri Nov 28 08:38:16 2008 From: rfrick at rfrick.info (Robert Frick) Date: Fri, 28 Nov 2008 02:38:16 -0500 Subject: [blml] One alternative for the WBF [SEC=UNOFFICIAL] In-Reply-To: References: Message-ID: On Fri, 28 Nov 2008 01:38:02 -0500, wrote: > Richard Hills, typist: > >>> As a resourceful etc, I note that assisting Grattan with the >>> version control of the drafts, redrafts and re-redrafts of the >>> 2007 Lawbook was entertaining. But the strain of so doing >>> would inhibit me from volunteering again if new editions of >>> the Lawbook were accelerated from decennial to annual events. > > Robert Frick, misconstruer: > >> Hi Richard. Are you proposing to stop this argument and leave >> the laws alone? >> >> Are you authorized to make that offer? > > Richard Hills, typist: > > I have zero bureaucratic bridge authority at the moment. Even > when my authority was at its peak, it was merely that of > President of the Bridge Federation of the Australian Capital > Territory (and even then I often got outvoted in committee > meetings seven votes to one). As a proof-reader and indexer of > the drafts of the Lawbook I had zero decision-making power, as > that rested with the Drafting Committee and eventually with the > WBF Executive. > > Robert Frick, misconstruer: > >> For example, the laws say regarding mistaken explanations "nor >> may he indicate in any manner that a mistake has been made." >> ..... >> or you can add "...except in giving explanations of partner's >> bids." in 2018. But you agree to leave it alone until then? > > Richard Hills, typist: > > I have zero bureaucratic bridge authority at the moment. But > the WBF Laws Committee does have bureaucratic bridge authority > under the WBF Regulations to interpret the Laws. The 1987 and > 1997 Law 75D2 (now the 2007 Law 20F5(a)) was in antediluvian > times (prior to the great flood of emails which commenced ten > years ago) assumed by almost all Directors to contain the > implied statement "...except in giving explanations of partner's > bids," since otherwise the Lawbook would be self-contradictory. > > An ever-increasing number of players across several countries > had no objection to interpreting the Lawbook in an inconsistent > and self-contradictory way, such as ACBLer Steve Willner and > Polish player Konrad Ciborowski. This may have been the reason > (as a mere former typist I do not know for sure) that the > common assumption about Law 20F5(a) -- and other common > assumptions about MI and UI -- became part of the official > October minute of the WBF Laws Committee. This minute is > legally equivalent to the WBF inserting an erratum in the Laws. > > See attached. > > > Best wishes > > Richard Hills, Aqua 5, workstation W550 > Telephone: 02 6223 8453 > Email: richard.hills at immi.gov.au > Recruitment Section & DIAC Social Club movie tickets > > * * * > > LAW 16 and others - concerning information rights > > 16A1(d) allows the player use of his memory of information in > the laws and regulations. It does not authorize him to look > during the auction and play at the printed regulations, the law > book, or anyone's scorecard or the backs of bidding cards etc. > as (Law 40C3(a)) an aid to memory. For system card and notes > see Law 20G2. > > Neither does 78D authorize players to consult during the auction > and play printed copies of the information given them under this > law. > > 20F1 defines the manner in which, during the auction and play, a > player may request and receive an explanation of the opponents' > prior auction. At this time he is entitled to an explanation > only of calls actually made, relevant available alternative > calls not made, and any partnership understanding as to > inferences from the choice of action among the foregoing. (An > "alternative" call is not the same call with another meaning - > for example, if the reply to an opponent is that "5D shows > diamonds preference", any reply to a further question "what > would it mean if 4NT were Blackwood ?" is given voluntarily and > not as a requirement of Law 20F1.) > > 81C2 requires the Director to advise players of their rights and > responsibilities under the laws. He confines such information to > rights and responsibilities that are relevant to the situation he > is dealing with. > > * * * > > LAW 20 > > There is no infraction when a correct explanation discloses that > partner's prior explanation was mistaken. The words "nor may he > indicate in any manner that a mistake has been made" (in Law > 20F5(a)) do not refer to compliance with the overriding > requirement of the laws always to respond to enquiries under Law > 20F with correct explanations of the partnership understandings. > > * * * > > LAW 75C > > The phrase "they have no claim to an accurate description of the > N-S hands" first appeared in the 1975 laws of the game. It was > accompanied then as now by the injunction forbidding the Director > to alter the table result. It was entered primarily to establish > beyond doubt that that the partnership agreement must be > described accurately in response to lawful enquiry and that the > explanation given must not aim to describe what the explainer > believes as to the contents of either hand. It was continued in > those terms in the 1987 law book, while for 2007 NBOs were > invited to say whether the example or the wording should be > updated. Among replies received there was a general consensus for > retaining them as they had been previously, whilst moving the > statements from a footnote into the body of the Law. QED. I was responding to the claim that the laws cannot be changed until 2018. Either the minutes of the WFBLC are not law, or the laws can be changed. I think the WBFLC does more than correct typos. And its committee minutes are not inserted into the laws. I have always assumed that the minutes are not the laws. The practical question is if people treat those minutes as law or not and if the NBOs or whatever disburse them to players. > The 1987 and > 1997 Law 75D2 (now the 2007 Law 20F5(a)) was in antediluvian > times (prior to the great flood of emails which commenced ten > years ago) assumed by almost all Directors to contain the > implied statement "...except in giving explanations of partner's > bids," since otherwise the Lawbook would be self-contradictory. This is an interpretation, not a reading of that Law. The lawbook is self-contradictory. That's probably unavoidable. Try "Any insufficient bid may be accepted (treated as legal) at the option of offender's LHO." That is inconsistent with a plain reading of Law 25, and attempts to resolve the inconsistency AFAIK just produce new ones. Resolving that contradiction is pretty simple, but that doesn't make it disppear from the laws. From Hermandw at skynet.be Fri Nov 28 09:40:10 2008 From: Hermandw at skynet.be (Herman De Wael) Date: Fri, 28 Nov 2008 09:40:10 +0100 Subject: [blml] The Nebraskan issue (really about the methods of change) In-Reply-To: <005201c950ba$ec9fbb00$15844c59@chello.pl> References: <4926FB3B.7060408@skynet.be><003901c94f16$f0976be0$15844c59@chello.pl> <492D0FA7.1000407@skynet.be><001d01c94fe9$22069200$15844c59@chello.pl> <492E5673.9020401@skynet.be><001301c95076$846f00c0$15844c59@chello.pl> <492EBC70.30801@skynet.be> <005201c950ba$ec9fbb00$15844c59@chello.pl> Message-ID: <492FAE6A.7070808@skynet.be> Bystry wrote: > Hi Herman, > >>>> Well, I don't recall any definite answer to that one. Are you in favour >>>> of PPs here? >>> Yes, mainly. >>> >> OK. Have you understood why I believe this is a bad idea? No need to >> agree, just tell me if you understood. > > Yes, I have. > OK, thanks. > [much sniped, resolved in other email] > >> Yes there are. I am talking about People who honestly believe their >> partner is right. And that is just because I was following your rule >> that only if you are certain partner is right, you are allowed to give a >> CE. IMO, many more people should be allowed to give CE - I would include >> those that think their partner is right, even if not 100% certain. If >> that is the rule (and no-one but you is suggesting that yours is the >> only possible correct rule), then there are many more innocents. > > The Laws don't state what should you do when you're uncertain. So here your > view is as valuable as mine. My opinion (that it's better to keep to one's > own version unless certain it's incorrect) is based on those arguments: > > - it's easier to keep to one's own version because that's what you clearly > know and you are not forced to guess That's just plain silly. When you are certain partner is correct, you also know what the next bid means. When you are partly certain, that's still the case. Besides, you can always have an exception for the times when you are uncertain of the version partner is playing. In all other cases, your argument does not hold. > - if you are consitent, it looks ok in the eyes of the opponents and the TD. > If you change your version and later it turns out you were initially right, > it may look as you were trying something unfair Ehm, please use the word consistent in one direction only. Consistent is the word I use for explanations which are consistent with the explanation partner has given. I do not change my version, since I have not explained my first version (partner has done that, differently). I do change the system I "know" we are playing, yes, but I cannot hide the first system, since it corresponds to the cards I hold. You may think that is "trying something unfair", I just call it "I made a misbid". > - there is no need for a TD during the auction (his arrival frequently > engenders emotions and the future auction may be influenced by this) > Ehm, again, get your facts straight. In my cases, there is no TD during the auction. When a call is explained consistently, no-one (except the first bidder) knows there is anything wrong. Only in _your_ system does the TD arrive, after such strange happenings as "4NT?" "Blackwood" "5Di" "diamond preference". > Of course I may easily accept other, contradictory arguments. For me it just > looks fine. > Well, try it at the table sometime and see which of the two "looks fine". >> The problem with that reasoning is that you talk of an obtained gain. I >> say there is no such gain. If the CE is allowed, the final result is >> just the same as if no question is asked. Look at that as the basis. > > And so what? Maybe you should demand total banishment of questions? That > would be consinsent because the first question already creates UI for your > side and only without it the final result would be good enough to satisfy > you. > No Maciej, you miscomprehend completely. When my partner misexplains my bid, I am in for a bad score already. So are you. Then, when the next bid comes, and they don't ask, we are still in the same situation. Still bad, still as bad as each other. But when they ask us what the next bid means, our situations diverge. My result will still be the same, bad one, as before. Your result will deteriorate immensily. Now I don't mind for you to be masochistic, but I do mind it for you to blame me for not being masochistic. That just means you're sadistic as well. My point is that my result is not affected by the extra question. Yours is. That means that my system is a correct one, while yours has a terrible flaw: the final result at the table is function of something that has no bearing on the bridge itself: the asking of a question. Your opponents can be just as interested in the meaning of your partner's call as mine, but depending on whether they check your SC or ask you, your result will be the same as mine or much worse. You may want to play bridge like that, but I don't like it. And I don't like a set of laws which forces me to play like that. Certainly not when there is a set of laws that does not force me to play like that, and which still treats my opponents correctly. You do agree that your opponents are treated correctly, the ones that look at your SC, I mean? > Herman, accept that it is your side who is responsible for your supposed > poor score. Not the opponents, who have every right to get full disclosure. But I don't touch that right - they do get full disclosure - at the end. Which is what your opponents get as well - on the first misexplanation! Bridge has always been played that way - MI is allowed to exist, and is corrected at the end. What you are giving your opponents, masochistically, is not just correct information about the second bid, but also correct information about the first bid, something which L20F5a tells us they are not -yet- entitled to, and also the information that you are having a misunderstanding, something case law tells us they are not entitled to, ever. > You forget your system - you usually get bad scores. I don't see any reason > why to debate in thousands of emails, why to reject the official decisions > of authorities, why to try to put your arguments over and over. Instead of > this, use one percent of wasted time to learn your system and you won't have > any problems. > Sorry, it's my partner who forgot the system :) But anyway, you are comparing me with a situation which only exists in your masochistic mind. If you compare me to the situation without a question, I do have a bad score. I feel my bad score is bad enough and, as TD, I do not need to punish people more severely than that. >> What you are doing, is inventing another law, and then comparing my >> result to the result under that law. it is you who is acting >> masochistically, not me who acting illegally. > > Herman, I'll be brutal but I'm sick of it all. I'm not inventing any Law. > *You* are doing it. I'm just reading the actual Laws and it's > interpretations made by authorised body, WBFLC. *They* are the authorities > ,*they* have necessary knowledge and skills, *they* are accredited to make > decisions, not *you*, ordinary TD with some ideas. These are my last words > regarding the topic of what the Laws are at present. If you wish so, > continue to live in your glass sphere. > Well Maciej, let me tell you a little bit of history about when this thing started. It's some 10 years ago now and when I first wrote something like this, a few people jumped on it and gave it a name. I doubt if they had ever thought about this thing before then. I consider that they "invented" the MS and its consequences then. I doubt if you ever realised that the bad score you give is the one the laws (according to you) warrant. It is perfectly clear to me that the 1997 laws were silent about this issue, and do not impose one law or another. It is equally clear that the 2007 do nothing either. Maybe the 2008 interpretation is a step in the direction you want, but if it is, it is very badly written. You (and Grattan, and everyone else) are still "inventing" this new law, which obliges people to give UI. You still have not put your noses in the same direction as to how certain someone should be before he's allowed to give a consistent explanation. You still haven't figured out what the penalty is for breaking this new law. You talk of PP's, but how many, and for whom? This is not fixed, at all. Until all that is done I shall continue to argue against this proposed new law. >>> M: the primary obligation of every player is to obey the Laws and to do >>> everything he can not to damage his opponents >>> H: the primary obligation of every player is to try as much as he can to >>> improve his position, even at the cost of the opponents >>> >> Unfair. I believe players should act within the laws. But if they act >> within those laws and try to get the best result, that is allowed. > > I didn't suggest that you approve of law-breaking. I'm sure you don't. I'll > explain in a more clear way. > > M: if the player stays within the Laws he should certainly do all he can to > defeat his opponents > H: I think that you agree > > M: if the player breaks any Law he should do all he can to ensure that his > infraction will have the least possible effect on the innocent side, even if > it costs him more than it could > H: such player should accept the penalty (present or future) but he can do > all that is possible to lessen the effects of this penalty on himself, even > if it harms the innocent side > And I think that principle is in the laws: L10C3 and L10C4. > I emphasise that it is only my own idea of how the game of Bridge should > work. > >>> M: the amount of infractions should be drastically decreased - by > education, >>> by restrictive Laws, by consequent harsh ruling >>> H: "equity" should be most important, frequent offenders are as much >>> valuable for game as those who try everything to conform to the rules >>> >> There are no frequent offenders in my system - they are only offenders >> in yours! > > I was using "offenders" as a global term, not just restricted to dWS. > Yes, but you were calling people offenders in my system when they only offended yours. >>> M: as much boards as possible should be played normally by the players, > not >>> in TD's consultations and judgment, not in appeals rooms, TD at the > table >>> should be as a last resort >>> H: TDs at the table are nothing bad, they make good rulings and people > are >>> delighted having been given a ruling instead of their own play >>> >> Totally wrong. In your cases the TD has more work than in mine. The MI >> has not totally gone away, and there are 2 pieces of UI to deal with. In >> mine, there is only one piece of UI and the second piece of MI is >> usually harmless (especially when compared or added to the first one). > > Yes, the TD has more work but the board is played normally (the score will > rarely be adjusted) and only the offenders have to explain their actions > (and are ruled against UI), the NOS can calmly concentrate on the following > boards and they don't have to waste their time persuading the TD (AC) that > they would do something or not. > This only depends on what you define by "normally". When a player is bound by UI, and follows those bounds, I don't call that "normally". And it is not because the player has thought about following the bounds of UI that the TD has no work: he still needs to check if the player has conformed correctly. So the TD has just as much work. Maybe, in your system, there are less score adjustments. But is that really such an advantage? If you would make a law that says that every board affected by DWS is simply ruled 0% or -15 IMPs, that will make TD life even more easy. But is it a good law? >> Herman. > > Regards > > Maciej > Herman. From Hermandw at skynet.be Fri Nov 28 09:47:16 2008 From: Hermandw at skynet.be (Herman De Wael) Date: Fri, 28 Nov 2008 09:47:16 +0100 Subject: [blml] The real text. In-Reply-To: <006801c950bc$1b7cb080$15844c59@chello.pl> References: <49227C4E.3050007@skynet.be><000001c94a0b$1cc70330$56550990$@com><000d01c94ad2$b54bc880$15844c59@chello.pl><4925C17B.1010706@talktalk.net> <49266B37.90306@skynet.be><000f01c94bba$ecc985c0$15844c59@chello.pl><4926C496.6080904@skynet.be><003901c94c47$a1541140$15844c59@chello.pl><4927E7CC.4030303@skynet.be><001f01c94cad$4bf57340$15844c59@chello.pl> <008701c94f22$94d773c0$15844c59@chello.pl> <492D105C.8010805@skynet.be><002501c94fed$21e0e4c0$15844c59@chello.pl> <492E5900.1060401@skynet.be><001d01c95079$bd5e2b60$15844c59@chello.pl> <492E7929.7020405@skynet.be><002b01c950af$78a8d840$15844c59@chello.pl> <492ED52D.40209@skynet.be> <006801c950bc$1b7cb080$15844c59@chello.pl> Message-ID: <492FB014.9060308@skynet.be> Bystry wrote: > [sniped, as promised] > >> I respect the lawbook as much as you do. It shows me no penalty for >> giving CEs. Yes, it gives me a penalty in the form of the MI I give, of >> course. But it does not tell me when, or to whom, or how much, of a PP >> to give to someone who gives a CE. > > L40C3 states that the TD can penalise CE. Only in repeated violations! Apart from HDW, no players have yet been warned for giving CEs - so how can you punish them? And the point remains: L40C3 is an addendum to L40. I always say that the MI will be punished, just like any other. > It doesn't measure the size of > penalty. But show me *any* Law that states how big a PP should be for > anything. Do it or next time rethink your arguments before you send an > email. > True, but if you don't give a measure - how can you be certain that the "benefit" of breaking your proposed law is countered by the PP. That is what I am bothered about: YOU create a situation in which the breaking of a law is potentially beneficial, and yet you don't take away that benefit. PPs are to stop people from doing things, over and above the taking away of any benefit they may get from it. >> Herman. > > Regards > > Maciej > Herman. From darkbystry at wp.pl Fri Nov 28 10:10:06 2008 From: darkbystry at wp.pl (Bystry) Date: Fri, 28 Nov 2008 10:10:06 +0100 Subject: [blml] Two alternatives for the WBF? No, three References: <492BF30E.8020808@skynet.be> Message-ID: <002101c95139$1b17cc60$15844c59@chello.pl> Hi, > Most bridge players seem to think that after the auction is over, the > opponents are entitled to both SE and CE. Most of the exceptions don't > particular want to play bridge with the opponents entitled to one. Did you make any official polls? You're using the phrase "most bridge players" but what are your grounds to make such assumptions? In Poland we have less than 7000 of PBU members (Bridge is reportedly played by 2 milion but mainly privately, at homes, the players are not bound by any Laws). From these 7000 of players the majority (I estimate it to be circa 6500) plays recreationary Bridge in lower leauges and some local tournaments. They have no idea what do the Laws state, they play in a way learned many years ago (e.g popular conscious "body play") or were only taught the basics of bridge by their friends, frequently their games are played without any qualified TDs. From the remaining 500 of players who play in open tournaments and have at least vague ideas about the Laws, I saw, say, 350-400 "in action". They simply exlain what does their system state. Of course there are some who may do it according to dWS or Bob's School (not every of those players had a bidding misunderstanding playing against me) but I'm certain they are in minority. So I'm entitled to say that most *Polish* bridge players choose SE (counting only those who, being asked, could give any reasonable answer). But I have no idea what do the players from other countries choose. You're from the ACBL which has many more members. Against how many did you happen to play? How many did you see explaining? Do you have any knowledge about the other countries? Regards Maciej From Hermandw at skynet.be Fri Nov 28 10:15:45 2008 From: Hermandw at skynet.be (Herman De Wael) Date: Fri, 28 Nov 2008 10:15:45 +0100 Subject: [blml] Two alternatives for the WBF? No, three In-Reply-To: References: <492BF30E.8020808@skynet.be> Message-ID: <492FB6C1.8040903@skynet.be> Hello Robert. You have confused CE and SE CE = consistent explanation = DWS SE = systemic explanation = MS Robert Frick wrote: > Hi Herman. Players can give the SE (what partner's bid should have meant), > the CE (what partner thought his bid meant), or both. You didn't mention > the Both option. Both is probably impractical. But read on. I did not want > to complicate your argument needlessly. But now I think this actually > helps it. > I think that when a player gives SE, the opponents will also ask him to tell them the CE. But of course a silly WBFLC interpretation now says he doesn't have to give them that. Denying opponents something they need to know and the player knows - incredible! > SE is mandated by any obvious reading of Law 20F5(a). As of course you > note. Actually, this laws mandates not CE, leaving SE by default. > CE. L20F5a prohibits SE. > The MS has argued for CE. Law 75 seems to be representative and/or their > best case, though I do not understand their position well enough to > actually characterize it. I think their understanding is that this law > also clearly states that CE is not required. > > Most bridge players seem to think that after the auction is over, the > opponents are entitled to both SE and CE. Most of the exceptions don't > particular want to play bridge with the opponents entitled to one. I will > argue that once partner misunderstands the auction, and say is treating > 2Di as multi, the partership agreements, etc. (thanks Nigel!) concerning > responses to 2Di become relevant. Hence by Law 20F1, both must be divulged. > > But at least in ACBL-land, Conditions of Contest is, roughly, complete > divulgence following any question. Complete divulgence isn't practical in > some cases, but I think you just do your best and get rectified when that > isn't good enough. > > That means that as most people interpret Law 20F1, both expanations are > also required during the auction in ACBL-land. > > So players in ACBL-land, or anywhere with similar divulgence regulations, > are faced with three laws all saying different things. > > Then it becomes pretty reasonable that Law 72B1 doesn't apply. > > Bob, who would prefer that his position not be called the Bob School > I agree with you Bob, so maybe no need for yet another school? You are allowed to choose another name for the school, Bob. Herman. From Hermandw at skynet.be Fri Nov 28 10:24:18 2008 From: Hermandw at skynet.be (Herman De Wael) Date: Fri, 28 Nov 2008 10:24:18 +0100 Subject: [blml] Prior argument [SEC=UNOFFICIAL] In-Reply-To: <492EF02A.60700@talktalk.net> References: <492E75A5.4040407@talktalk.net> <492E889F.9030303@ulb.ac.be> <492EF02A.60700@talktalk.net> Message-ID: <492FB8C2.2080506@skynet.be> Nigel Guthrie wrote: > [Alain Gottcheiner] > I think that cases where the guess of which wheel fell off will be > helped by the knowledge of previous (or other) agreements are quite > uncommon and we shouldn't bother about this ... > > [Nigel] > Uncommon but such cases cause controversy - > - A DWS explanation is usually an explanation of a default agreement. > - When you suspect partner of psyching, it helps to know his habits. > For example suppose LHO opens 1H, partner overcalls 1N, and RHO doubles. > A dodgy 1N overcall may be the kind of psych that partner has used in > the past. Your knowledge of partner's habits may indicate that it is he > who is more likely to be messing around than either opponent. > Furthermore, you may know the kind of hand that partner prefers for this > kind of gambit (for instance, values for a weak two). If authorised, > this information will help you, in both the bidding and the play. > - Other contexts, with and without alerts and explanations, when you > suspect partner may have made a bidding mistake. > For example, partner may be prone to sorting his diamonds into his hearts. > > If information from prior agreements and partner's idiosyncrasies were > *unauthorised* to you, then these situations would be legally simpler. > But that would create many more problems than it would solve. IMO. The answer to things like this, Nigel, is that you must detach the notions of "things the opponents are entitled to know" from "things you must tell them". There are always a number of things that you know about your partner. Those things, the opponents should know them too. But it is impractical, illegal even, to tell them these things. So what you must do is wait until the hand is over (or until the correction period, if of application) and tell them those things at that time. Then ask them if they feel damaged and call the TD if they do (or call the TD anyway). The TD can then correct if he feels that they would in fact have acted differently if they had known about the possibility of the psyche. Take you example. You know that your partner sometimes psyches, and you know how. You hear them bidding, and you have some points yourself. You conclude that he does not have his bid, and you pass. Now they play all the points in his hand, and you score a few queens. If they had known he is a more regular psycher, they could have played the hand differently and the TD may adjust for that. I have no problems with not telling them "my partner is a regular psycher" and I will try to get that message accross in some other way (mentioning on the SC seems OK), but if I fail, I'll accept my AS and not feel bad about it. Some people object to my not feeling bad about it. OK, I do feel bad about it, I accept my AS and then my bad feelings dissappear. Herman. From darkbystry at wp.pl Fri Nov 28 10:32:25 2008 From: darkbystry at wp.pl (Bystry) Date: Fri, 28 Nov 2008 10:32:25 +0100 Subject: [blml] The real text. References: <49227C4E.3050007@skynet.be><000001c94a0b$1cc70330$56550990$@com><000d01c94ad2$b54bc880$15844c59@chello.pl><4925C17B.1010706@talktalk.net> <49266B37.90306@skynet.be><000f01c94bba$ecc985c0$15844c59@chello.pl><4926C496.6080904@skynet.be><003901c94c47$a1541140$15844c59@chello.pl><4927E7CC.4030303@skynet.be><001f01c94cad$4bf57340$15844c59@chello.pl> <008701c94f22$94d773c0$15844c59@chello.pl> <492D105C.8010805@skynet.be><002501c94fed$21e0e4c0$15844c59@chello.pl> <492E5900.1060401@skynet.be><001d01c95079$bd5e2b60$15844c59@chello.pl> <492E7929.7020405@skynet.be><002b01c950af$78a8d840$15844c59@chello.pl> <492ED52D.40209@skynet.be><006801c950bc$1b7cb080$15844c59@chello.pl> <492FB014.9060308@skynet.be> Message-ID: <002b01c9513c$391d9f20$15844c59@chello.pl> Hi, > > L40C3 states that the TD can penalise CE. > > Only in repeated violations! Apart from HDW, no players have yet been > warned for giving CEs - so how can you punish them? And so what? Herman, are you 5 years old? Do I have to treat you as such? Even if every player in the world will choose the dWS two times in his whole career and remains unpunished, how does it compare to the number of situations in which classic UI usage goes unpunished? We're talking about *the statistical deviation*, nothing more. And in those cases it usually costs the NOS to have 70% instead of 100%, big deal. > True, but if you don't give a measure - how can you be certain that the > "benefit" of breaking your proposed law is countered by the PP. Now you're treating the TD as small children? There are many serious infractions which deserve to be punished. The TDs are to judge how severe this punishment should be. They use their judgment in the other cases, they will use it here. > That is what I am bothered about: YOU create a situation in which the > breaking of a law is potentially beneficial, and yet you don't take away > that benefit. PPs are to stop people from doing things, over and above > the taking away of any benefit they may get from it. I don't create anything. I'm not a masked man behind the curtain who pulls the strings of the lawmakers' puppets. If you want to unleash you anger write to WBFLC. I happen to (partially) agree with them but I'm not their representantive and I don't have any will to stand the gaff of your constant needling. > Herman. Regards Maciej From Hermandw at skynet.be Fri Nov 28 10:39:42 2008 From: Hermandw at skynet.be (Herman De Wael) Date: Fri, 28 Nov 2008 10:39:42 +0100 Subject: [blml] Two alternatives for the WBF? No, three In-Reply-To: <002101c95139$1b17cc60$15844c59@chello.pl> References: <492BF30E.8020808@skynet.be> <002101c95139$1b17cc60$15844c59@chello.pl> Message-ID: <492FBC5E.4020705@skynet.be> Maciej, think about it. Bystry wrote: > Hi, > >> Most bridge players seem to think that after the auction is over, the >> opponents are entitled to both SE and CE. Most of the exceptions don't >> particular want to play bridge with the opponents entitled to one. > > Did you make any official polls? You're using the phrase "most bridge > players" but what are your grounds to make such assumptions? > > In Poland we have less than 7000 of PBU members (Bridge is reportedly played > by 2 milion but mainly privately, at homes, the players are not bound by any > Laws). From these 7000 of players the majority (I estimate it to be circa > 6500) plays recreationary Bridge in lower leauges and some local > tournaments. They have no idea what do the Laws state, they play in a way > learned many years ago (e.g popular conscious "body play") or were only > taught the basics of bridge by their friends, frequently their games are > played without any qualified TDs. From the remaining 500 of players who play > in open tournaments and have at least vague ideas about the Laws, I saw, > say, 350-400 "in action". They simply exlain what does their system state. > Of course there are some who may do it according to dWS or Bob's School (not > every of those players had a bidding misunderstanding playing against me) > but I'm certain they are in minority. > These bridge players will be very surprised to hear the following conversation: "4NT?" "Blackwood" "5Di?" "diamond preference". "OK, but how many aces does he have" "I don't have to tell you". Do you seriously intend to say that bridge players will accept that answer? That they will believe you when you say that indeed he does not have to say that? That they will have any other reaction to your showing them the 2008 LC interpretation than "crazy people". Do you seriously believe that any player out there will not simply answer "1 ace" to the second question (if he doesn't do it to the first one immediately?) Full disclosure is so engrained in the philosophy of bridge that I find it totally crazy that there are even people thinking otherwise. Furthermore, I feel it so terribly strange that people who believe full disclosure is so much more important than UI, to the point that they answer "diamond preference" to a question about aces, would feel that the same full disclosure is so unimportant in this way. Too crazy for words. > So I'm entitled to say that most *Polish* bridge players choose SE (counting > only those who, being asked, could give any reasonable answer). But I have > no idea what do the players from other countries choose. > > You're from the ACBL which has many more members. Against how many did you > happen to play? How many did you see explaining? Do you have any knowledge > about the other countries? > > Regards > > Maciej > Herman. From Hermandw at skynet.be Fri Nov 28 11:16:14 2008 From: Hermandw at skynet.be (Herman De Wael) Date: Fri, 28 Nov 2008 11:16:14 +0100 Subject: [blml] The real text. In-Reply-To: <002b01c9513c$391d9f20$15844c59@chello.pl> References: <49227C4E.3050007@skynet.be><000001c94a0b$1cc70330$56550990$@com><000d01c94ad2$b54bc880$15844c59@chello.pl><4925C17B.1010706@talktalk.net> <49266B37.90306@skynet.be><000f01c94bba$ecc985c0$15844c59@chello.pl><4926C496.6080904@skynet.be><003901c94c47$a1541140$15844c59@chello.pl><4927E7CC.4030303@skynet.be><001f01c94cad$4bf57340$15844c59@chello.pl> <008701c94f22$94d773c0$15844c59@chello.pl> <492D105C.8010805@skynet.be><002501c94fed$21e0e4c0$15844c59@chello.pl> <492E5900.1060401@skynet.be><001d01c95079$bd5e2b60$15844c59@chello.pl> <492E7929.7020405@skynet.be><002b01c950af$78a8d840$15844c59@chello.pl> <492ED52D.40209@skynet.be><006801c950bc$1b7cb080$15844c59@chello.pl> <492FB014.9060308@skynet.be> <002b01c9513c$391d9f20$15844c59@chello.pl> Message-ID: <492FC4EE.6070108@skynet.be> Bystry wrote: > Hi, > >>> L40C3 states that the TD can penalise CE. >> Only in repeated violations! Apart from HDW, no players have yet been >> warned for giving CEs - so how can you punish them? > > And so what? Herman, are you 5 years old? Do I have to treat you as such? > > Even if every player in the world will choose the dWS two times in his whole > career and remains unpunished, how does it compare to the number of > situations in which classic UI usage goes unpunished? We're talking about > *the statistical deviation*, nothing more. And in those cases it usually > costs the NOS to have 70% instead of 100%, big deal. > But Maciej, it is ME who is against UI. I avoid UI at all costs! In my cases, there is no UI that can go unpunished, as there is no UI! >> True, but if you don't give a measure - how can you be certain that the >> "benefit" of breaking your proposed law is countered by the PP. > > Now you're treating the TD as small children? There are many serious > infractions which deserve to be punished. The TDs are to judge how severe > this punishment should be. They use their judgment in the other cases, they > will use it here. > But we are not talking about punishment - we are talking about a benefit. A benefit which comes from not giving the UI which you insist must be given. A benefit from partner waking up to his own misunderstanding from sources which are AI. That benefit! You have created a law which obliges a player to give UI. UI which you don't want to leave unpunished. So you will insist that partner is not allowed to wake up. But now I'm talking about an innocent player who, not knowing that new law of yours, has not given the UI that you insist he must. And you think that a PP will help in this? You create a situation in which opponents are entitled to know about the misunderstanding. They are entitled to play 5Di redoubled minus seven, and 24 IMPs in their credit. Yet with an innocent infractor you give 3IMPs penalty, and with me as infractor 6IMPs. And the opponents get nothing? That is what I am talking about. You create a law, but then refuse to apply it. So that people who meekly follow the law are worse off than those who deliberately break it, and much worse than those that innocently break it because they are not yet in the class of HDW who should have known better. Now please don't misunderstand me. I am not advocating augmenting your law with penalties of 24IMPs. I am merely saying that your law is not workable, since there are no workable penalties available for it. >> That is what I am bothered about: YOU create a situation in which the >> breaking of a law is potentially beneficial, and yet you don't take away >> that benefit. PPs are to stop people from doing things, over and above >> the taking away of any benefit they may get from it. > > I don't create anything. I'm not a masked man behind the curtain who pulls > the strings of the lawmakers' puppets. If you want to unleash you anger > write to WBFLC. I happen to (partially) agree with them but I'm not their > representantive and I don't have any will to stand the gaff of your constant > needling. > I know you are not. But you are the one defending this proposed law to me. By calling it "your law" I hope to make you think about it and join me in telling the WBF that "their law" is unworkable. Also I want to make you see that this law is not yet written. I don't consider the 2008 interpretation as law yet. It lacks a penalty and some necessary interpretations about when it does and does not apply. >> Herman. > > Regards > > Maciej > Herman. From darkbystry at wp.pl Fri Nov 28 12:19:53 2008 From: darkbystry at wp.pl (Bystry) Date: Fri, 28 Nov 2008 12:19:53 +0100 Subject: [blml] The Nebraskan issue (really about the methods of change) References: <4926FB3B.7060408@skynet.be><003901c94f16$f0976be0$15844c59@chello.pl> <492D0FA7.1000407@skynet.be><001d01c94fe9$22069200$15844c59@chello.pl> <492E5673.9020401@skynet.be><001301c95076$846f00c0$15844c59@chello.pl> <492EBC70.30801@skynet.be><005201c950ba$ec9fbb00$15844c59@chello.pl> <492FAE6A.7070808@skynet.be> Message-ID: <003101c9514b$3cac6d60$15844c59@chello.pl> Hi, > > The Laws don't state what should you do when you're uncertain. So here your > > view is as valuable as mine. My opinion (that it's better to keep to one's > > own version unless certain it's incorrect) is based on those arguments: > > > > - it's easier to keep to one's own version because that's what you clearly > > know and you are not forced to guess > > That's just plain silly. When you are certain partner is correct, you > also know what the next bid means. When you are partly certain, that's > still the case. Besides, you can always have an exception for the times > when you are uncertain of the version partner is playing. In all other > cases, your argument does not hold. I'll try to refrain from answering by rudeness to rudeness. I saw many of your contributions to this list and you've always used phrases like "you're wrong, you're so so wrong", "that's silly" and similar. That is a bad way of discussing. It only tries to create a mirrage to the other readers that one of the discutants has to be surely right. But only the power of arguments should count, not the power or sneakiness of voice. I'd rather you've threwn this bad manner away but since it didn't happen in all the years, I think there is no more hope. Now to the logic (I remind you that we're talking only about consistent strategies, not guessing what to do in every case): a) keeping to one's own version is effective in every case b) changing the story is effective in many cases, but there are some (distinctive minority) in which it won't be effective - add to this the fact that the initial bidder is much more often convinced that he's right and it's clear that taking all the possible situations into consideration the player has an easy road in a) but not always in b). a) works in every case, b) will sometimes force you to revert to a). Now, can I expect an apology for your statements? Or you'll still try to muddy the waters and wrongly evaluate my arguments? Decide carefully. > > - if you are consitent, it looks ok in the eyes of the opponents and the TD. > > If you change your version and later it turns out you were initially right, > > it may look as you were trying something unfair > > Ehm, please use the word consistent in one direction only. Consistent > is the word I use for explanations which are consistent with the > explanation partner has given. > I do not change my version, since I have not explained my first version > (partner has done that, differently). I do change the system I "know" we > are playing, yes, but I cannot hide the first system, since it > corresponds to the cards I hold. You may think that is "trying something > unfair", I just call it "I made a misbid". Herman, it's very hard for me to comment on this. I'm trying to refrain myself from writing the words that come to my mind. Do you really believe in what you are writing? Or are you trying everything, even the worst of grips to support your ideas? You make any call. You believe it is your systemic call. Partner misexplains your bid. Now you change your version. And *that* you call consistent? And you dare to tell me how should I use an English word which meaning I do perfectly know? Later you even admit that you change your system from that moment. How much hipocrisy do you want to include in your statements? And you call it "I made a misbid"? You are *lying with the straight face* that you'd made a misbid. > > - there is no need for a TD during the auction (his arrival frequently > > engenders emotions and the future auction may be influenced by this) > > Here it's my mistake. I wasn't in full concetration (watching the soccer game) and I confused this with the other situation (when your partner wants to change his initial MI after hearing your SE). Sorry for that, forget about this statement. > Ehm, again, get your facts straight. In my cases, there is no TD during > the auction. When a call is explained consistently, no-one (except the > first bidder) knows there is anything wrong. Only in _your_ system does > the TD arrive, after such strange happenings as "4NT?" "Blackwood" "5Di" > "diamond preference". Look above. But also in CS there is usually no need for the TD during the auction. And certainly not after the auction. In dWS the TD has to be called to correct great amounts of MI at the end of the bidding. And I tried to explain in other emails why it would be hard for the NOS to overcome following confusion (not to say about waste of time comparing to CS). > > Of course I may easily accept other, contradictory arguments. For me it just > > looks fine. > > > Well, try it at the table sometime and see which of the two "looks fine". Well, try to say "Good Morning" to your neighbours. You do? So don't write anything you have no clue about. > > And so what? Maybe you should demand total banishment of questions? That > > would be consinsent because the first question already creates UI for your > > side and only without it the final result would be good enough to satisfy > > you. > > > > No Maciej, you miscomprehend completely. When my partner misexplains my > bid, I am in for a bad score already. So are you. Then, when the next > bid comes, and they don't ask, we are still in the same situation. Still > bad, still as bad as each other. But when they ask us what the next bid > means, our situations diverge. My result will still be the same, bad > one, as before. Your result will deteriorate immensily. Now I don't mind > for you to be masochistic, but I do mind it for you to blame me for not > being masochistic. That just means you're sadistic as well. Herman, you are in a need of instant logic lessons. a) (no questions) -> initial score b) (first question) -> worsened score c) (second question) -> more worsened score You are constantly sheding tears about having to wear the consequences of c) instead of b). So I only advised you that to be consistent you should shed tears because of having to wear the consequences of b) instead of a). > My point is that my result is not affected by the extra question. Yes, not affected by the *extra* question. And so what? If the *first* question can change the result, so can the *second*. I don't see anything bad about it. > is. That means that my system is a correct one, while yours has a > terrible flaw: No, *in your opinion* it may look like that. You have no right to decide which is correct. You consider yourself more clever than me? Do you want to test it? And bet some money on the final result? > the final result at the table is function of something > that has no bearing on the bridge itself: the asking of a question. Moonshine. Questions have significant bearing on bridge. E.g. this first question which causes UI to be present for one opponent. This topic is closed for me. > > Herman, accept that it is your side who is responsible for your supposed > > poor score. Not the opponents, who have every right to get full disclosure. > > But I don't touch that right - they do get full disclosure - at the end. > Which is what your opponents get as well - on the first misexplanation! > Bridge has always been played that way - MI is allowed to exist, and is > corrected at the end. And how does it help bridge to be played normally? Not by TD after the end of the deal? > What you are giving your opponents, masochistically, is not just correct > information about the second bid, but also correct information about the > first bid, something which L20F5a tells us they are not -yet- entitled > to, and also the information that you are having a misunderstanding, > something case law tells us they are not entitled to, ever. Yes, they are not "entitled" to this by the Laws. But as well they are not "entitled" to write super scores when you pass partner's cuebid or make a defensive blunder. But they sometimes get such gifts, that is normal in bridge. And so your mixup is another kind of gift. If you don't want to give them any gifts - learn your system. [sniped, as promised] > > M: if the player breaks any Law he should do all he can to ensure that his > > infraction will have the least possible effect on the innocent side, even if > > it costs him more than it could > > H: such player should accept the penalty (present or future) but he can do > > all that is possible to lessen the effects of this penalty on himself, even > > if it harms the innocent side > > > > And I think that principle is in the laws: L10C3 and L10C4. I'm aware of L10C4. But it doesn't force the player to behave like that. It only allows for such behavior. So my own idea is not conflicted with Laws. Remember that I'm not trying to prove that all my ideas are objectivly better. I was just showing the differences between our goals. > > Yes, the TD has more work but the board is played normally (the score will > > rarely be adjusted) and only the offenders have to explain their actions > > (and are ruled against UI), the NOS can calmly concentrate on the following > > boards and they don't have to waste their time persuading the TD (AC) that > > they would do something or not. > > > > This only depends on what you define by "normally". | | V > Maybe, in your system, there are less score adjustments. You've answered that on your own. > But is that really such an advantage? For me it's a great advantage. The greatest of the greatest. It is a base of all my expectations regarding Bridge Laws and the authorities' main goals. The least TD interventions that possible. The players making their own decisions, bad or good, having the results based solely on their own actions, not on the virtual actions of their peers, TDs or AC members. Utopia? Certainly, but aiming for good utopia makes the things better. Deciding to stay in place (because utopia is unreachable) only preserves the bad things. > Herman. Regards Maciej From darkbystry at wp.pl Fri Nov 28 12:43:55 2008 From: darkbystry at wp.pl (Bystry) Date: Fri, 28 Nov 2008 12:43:55 +0100 Subject: [blml] The real text. References: <49227C4E.3050007@skynet.be><000001c94a0b$1cc70330$56550990$@com><000d01c94ad2$b54bc880$15844c59@chello.pl><4925C17B.1010706@talktalk.net> <49266B37.90306@skynet.be><000f01c94bba$ecc985c0$15844c59@chello.pl><4926C496.6080904@skynet.be><003901c94c47$a1541140$15844c59@chello.pl><4927E7CC.4030303@skynet.be><001f01c94cad$4bf57340$15844c59@chello.pl> <008701c94f22$94d773c0$15844c59@chello.pl> <492D105C.8010805@skynet.be><002501c94fed$21e0e4c0$15844c59@chello.pl> <492E5900.1060401@skynet.be><001d01c95079$bd5e2b60$15844c59@chello.pl> <492E7929.7020405@skynet.be><002b01c950af$78a8d840$15844c59@chello.pl> <492ED52D.40209@skynet.be><006801c950bc$1b7cb080$15844c59@chello.pl> <492FB014.9060308@skynet.be><002b01c9513c$391d9f20$15844c59@chello.pl> <492FC4EE.6070108@skynet.be> Message-ID: <006101c9514e$9870d480$15844c59@chello.pl> Hi, >> And so what? Herman, are you 5 years old? Do I have to treat you as such? >> >> Even if every player in the world will choose the dWS two times in his whole >> career and remains unpunished, how does it compare to the number of >> situations in which classic UI usage goes unpunished? We're talking about >> *the statistical deviation*, nothing more. And in those cases it usually >> costs the NOS to have 70% instead of 100%, big deal. > But Maciej, it is ME who is against UI. I avoid UI at all costs! In my > cases, there is no UI that can go unpunished, as there is no UI! Herman, it cares me equally as much as a color of the dress of the housewifes in Mozambique. I'm really tired from those pointless discussions with you. I was only proving that the overall number of unpunished dWS cases is nothing in comparison to an overall number of the other unpunished infractions. And you don't try to put any counter-arguments (you don't have even one) but instead you try to muddle your defeat by nitpicking and writing about things that have *nothing*, I repeat *nothing* in common with the topic of the discussion. > >> True, but if you don't give a measure - how can you be certain that the > >> "benefit" of breaking your proposed law is countered by the PP. > > > > Now you're treating the TD as small children? There are many serious > > infractions which deserve to be punished. The TDs are to judge how severe > > this punishment should be. They use their judgment in the other cases, they > > will use it here. > > > But we are not talking about punishment - we are talking about a > benefit. A benefit which comes from not giving the UI which you insist > must be given. A benefit from partner waking up to his own > misunderstanding from sources which are AI. That benefit! Herman, when somebody steals 10000E, is caught, and pays 20000E fine, is that a *benefit* for him? Really, your arguments are sometimes worse than those of a 10-year-old child. > You have created a law which obliges a player to give UI. UI which you > don't want to leave unpunished. So you will insist that partner is not > allowed to wake up. But now I'm talking about an innocent player who, > not knowing that new law of yours, has not given the UI that you insist > he must. And you think that a PP will help in this? I won't give a PP to the first-time offender. Next time he won't be innocent. And he'll get a big PP. So big that he'll lose some tournament places thanks to it. And he'll learn that the first duty of every player is to obey the Laws, not to boost one's own score at all costs. [sniped, reductio ad absurdu] > > I don't create anything. I'm not a masked man behind the curtain who pulls > > the strings of the lawmakers' puppets. If you want to unleash you anger > > write to WBFLC. I happen to (partially) agree with them but I'm not their > > representantive and I don't have any will to stand the gaff of your constant > > needling. > > > > I know you are not. But you are the one defending this proposed law to > me. By calling it "your law" I hope to make you think about it and join > me in telling the WBF that "their law" is unworkable. Sorry to disappoint you but your hope is vain. This Law works very fine in my country for the last 10 years, even without recent clarification. Your delusions do not interest me. [sniped, as promised] > Herman. Regards Maciej From darkbystry at wp.pl Fri Nov 28 12:56:04 2008 From: darkbystry at wp.pl (Bystry) Date: Fri, 28 Nov 2008 12:56:04 +0100 Subject: [blml] Two alternatives for the WBF? No, three References: <492BF30E.8020808@skynet.be> <002101c95139$1b17cc60$15844c59@chello.pl> <492FBC5E.4020705@skynet.be> Message-ID: <006701c95150$4a8c1de0$15844c59@chello.pl> Hi, > These bridge players will be very surprised to hear the following > conversation: > > "4NT?" "Blackwood" "5Di?" "diamond preference". > "OK, but how many aces does he have" > "I don't have to tell you". Herman, I'm very close to write something really bad. *I* know those players, *you* don't. Your idiosyncratic fantasies have zero merit. And that's the last time I bother to comment on them. [sniped shedding tears] Full disclosure is very important. But more important is bridge. The game played from the beginning on the fundamental basis - that any player is *not entitled* to know the contents of hand of any other player. That is an essence of bridge - to discover the layout of cards with the help of solving the "riddles", counting, interpreting, signaling, logic. That's the game I want to play. If you don't understand it, that's your problem. > Herman. Regards Maciej From darkbystry at wp.pl Fri Nov 28 14:01:26 2008 From: darkbystry at wp.pl (Bystry) Date: Fri, 28 Nov 2008 14:01:26 +0100 Subject: [blml] pugio tuus est interfector tuus [SEC=UNOFFICIAL] References: Message-ID: <001101c95159$6c488140$15844c59@chello.pl> Richard, > Addendum to addendum: If East-West must immediately cash > two hearts in order to prevent a discard of a heart loser, > and if East-West cashing three rounds of hearts allows a > ruff-sluff discard of a diamond or club loser, then Law > 26A2 and Morton's Fork mean that 2S is cold. >From West's point of view it is not probable that the hearts have to be drawn instantly. If a heart can be thrown on a club than the same goes for a diamond. It would require very specific layout to make a diamond lead worse. Much more often a diamond will free partner from a throw-in or will result in a diamond ruff. > The truth was somewhat more prosaic; West leading the > three of hearts rather than the king of hearts made it > easier for East to misdefend (since East thought it might > be possible that South held KJ doubleton of hearts). > > The complete deal: > > 54 > Q4 > AQT985 > J75 > 952 A3 > KT3 A98752 > 43 KJ > T9864 AK2 > KQJT76 > J6 > 762 > Q3 > > After West's opening lead of the three of hearts was won > by East's ace, and I used Law 26A2 to prohibit East from > leading another heart while East retained the lead, then > East's 100% best line of defence is to immediately stop > retaining the lead by switching to the trey of trumps. > Instead East opted for the 0% defensive line of ace, king > and deuce of clubs. Too bad. Defensive errors happen. But I don't think that many imps were lost by EW. > Note that Maciej Bystry, who as West preferred changing the > insufficient 2H to 3H, would have been a big winner at the > table, since 4H does not need any Law 26A2 rectification to > make. And it is no surprise to me. If N has at most 11HCP and S at most 9, than partner has at least 17. Probably more. So if he has nothing wasted in spades 4H will at least has some serious chances. x AQJ9x AQxx Axx - 17 points and the game requires only the diamond finesse. > Best wishes Regards Maciej From Hermandw at skynet.be Fri Nov 28 14:08:54 2008 From: Hermandw at skynet.be (Herman De Wael) Date: Fri, 28 Nov 2008 14:08:54 +0100 Subject: [blml] The Nebraskan issue (really about the methods of change) In-Reply-To: <003101c9514b$3cac6d60$15844c59@chello.pl> References: <4926FB3B.7060408@skynet.be><003901c94f16$f0976be0$15844c59@chello.pl> <492D0FA7.1000407@skynet.be><001d01c94fe9$22069200$15844c59@chello.pl> <492E5673.9020401@skynet.be><001301c95076$846f00c0$15844c59@chello.pl> <492EBC70.30801@skynet.be><005201c950ba$ec9fbb00$15844c59@chello.pl> <492FAE6A.7070808@skynet.be> <003101c9514b$3cac6d60$15844c59@chello.pl> Message-ID: <492FED66.80303@skynet.be> Hello Maciej, Bystry wrote: > Hi, > >>> The Laws don't state what should you do when you're uncertain. So here > your >>> view is as valuable as mine. My opinion (that it's better to keep to > one's >>> own version unless certain it's incorrect) is based on those arguments: >>> >>> - it's easier to keep to one's own version because that's what you > clearly >>> know and you are not forced to guess >> That's just plain silly. When you are certain partner is correct, you >> also know what the next bid means. When you are partly certain, that's >> still the case. Besides, you can always have an exception for the times >> when you are uncertain of the version partner is playing. In all other >> cases, your argument does not hold. > > I'll try to refrain from answering by rudeness to rudeness. I saw many of > your contributions to this list and you've always used phrases like "you're > wrong, you're so so wrong", "that's silly" and similar. That is a bad way of > discussing. It only tries to create a mirrage to the other readers that one > of the discutants has to be surely right. But only the power of arguments > should count, not the power or sneakiness of voice. I'd rather you've threwn > this bad manner away but since it didn't happen in all the years, I think > there is no more hope. > I apologise for the rudeness. I am trying to convey that some of the things my colleagues are saying are more wrong than others. When I say that something is silly - it is because I find no other word to convey that they are very very wrong. In a sense, it is an acknowledgment that some of the other arguments are not so bad at all. I would like to get some of the bad arguments out of the way, so that the good arguments can remain. The above is one of the worst arguments you are using. It is possible, indeed, that you don't really know which system partner thinks he is playing. But that does not make it impossible for you to give an answer which can be considered a CE. Your argument is that since it is difficult, sometimes, to give a CE, CE must be outlawed. That is false logic. If it proves impossible for one player in one case to give a CE answer, that does not necessitate the WBF to outlaw all other CE answers. Do you see the fallacy in your argument? Then forgive me for having used the word "silly" on it, but please understand why I used it. > Now to the logic (I remind you that we're talking only about consistent > strategies, not guessing what to do in every case): > > a) keeping to one's own version is effective in every case That depends on what you call effective. Since it gives partner UI and opponents information they are not entitled to, I call it most ineffective. > b) changing the story is effective in many cases, but there are some > (distinctive minority) in which it won't be effective > - add to this the fact that the initial bidder is much more often convinced > that he's right > There is no talk about changing any story. > and it's clear that taking all the possible situations into consideration > the player has an easy road in a) but not always in b). a) works in every > case, b) will sometimes force you to revert to a). > Again, even if it proves impossible to follow B in some cases, why does this imply that B ought to be outlawed? > Now, can I expect an apology for your statements? Or you'll still try to > muddy the waters and wrongly evaluate my arguments? Decide carefully. > I have apologised for the words, I cannot apologise for my sentiments. Your logic is sadly totally absent. >>> - if you are consitent, it looks ok in the eyes of the opponents and the > TD. >>> If you change your version and later it turns out you were initially > right, >>> it may look as you were trying something unfair >> Ehm, please use the word consistent in one direction only. Consistent >> is the word I use for explanations which are consistent with the >> explanation partner has given. >> I do not change my version, since I have not explained my first version >> (partner has done that, differently). I do change the system I "know" we >> are playing, yes, but I cannot hide the first system, since it >> corresponds to the cards I hold. You may think that is "trying something >> unfair", I just call it "I made a misbid". > > Herman, it's very hard for me to comment on this. I'm trying to refrain > myself from writing the words that come to my mind. Do you really believe in > what you are writing? Or are you trying everything, even the worst of grips > to support your ideas? > I could say the same about you. > You make any call. You believe it is your systemic call. Partner misexplains > your bid. Now you change your version. And *that* you call consistent? And > you dare to tell me how should I use an English word which meaning I do > perfectly know? > yes, of course that is what I call consistent - the second explanation is consistent with the first. Agreed? If you start using the same word for both schools, you are the one who is muddying the waters, no? As about changing my version - isn't that totally logical. You make a call, believing that to be your system. Your partner explains it differently. From this, you remember that he is right and you were wrong. Why should you not then change your idea of what your system is? Only Stephanie believes that she should continue the wrong explanation even if 100% certain that partner was right. You yourself have admitted that you would explain according to partner's explanation, when convinced that you were wrong before. There is nothing wrong with that. > Later you even admit that you change your system from that moment. How much > hipocrisy do you want to include in your statements? > But I don't change my system. I change my belief in what my system actually was. Have you never been wrong in your system? Have you never made a misbid? And if you are one of those players who have never made a misbid (there appear to be thousands of them) then can you please put yourself in the shoes of someone who has? > And you call it "I made a misbid"? You are *lying with the straight face* > that you'd made a misbid. > Precisely. I would of course never do such a thing, and nor would you, but we are directors, and we are faced with people who tell us that they were absolutely certain they made a misbid. Do we believe them? You will have to make a decision on that. I won't. Because I won't care, since I accept a CE even when the player is absolutely certain he made a correct bid. >>> - there is no need for a TD during the auction (his arrival frequently >>> engenders emotions and the future auction may be influenced by this) >>> > > Here it's my mistake. I wasn't in full concetration (watching the soccer > game) and I confused this with the other situation (when your partner wants > to change his initial MI after hearing your SE). Sorry for that, forget > about this statement. > OK, apology accepted. >> Ehm, again, get your facts straight. In my cases, there is no TD during >> the auction. When a call is explained consistently, no-one (except the >> first bidder) knows there is anything wrong. Only in _your_ system does >> the TD arrive, after such strange happenings as "4NT?" "Blackwood" "5Di" >> "diamond preference". > > Look above. But also in CS there is usually no need for the TD during the > auction. And certainly not after the auction. In dWS the TD has to be called > to correct great amounts of MI at the end of the bidding. And I tried to > explain in other emails why it would be hard for the NOS to overcome > following confusion (not to say about waste of time comparing to CS). > You have two things wrong here. Sadly, many players will not follow the UI laws and you do have to make an adjustment. And even when they say they have tried to follow the UI laws, you still have to check that they in fact have done so. So you still have to make a ruling. That's the first error. The second error is that you seem to believe that MI rulings are difficult and UI rulings are easy. I happen to think the other way. Which proves only one thing: they are both difficult. Now all over the lawbook, the lawmakers have tried to limit the amount of UI that is being given. Why? Maybe because they feel that prevention is better than curing. My proposal for this law also limits the UI. What can be so wrong about that? >>> Of course I may easily accept other, contradictory arguments. For me it > just >>> looks fine. >>> >> Well, try it at the table sometime and see which of the two "looks fine". > > Well, try to say "Good Morning" to your neighbours. You do? So don't write > anything you have no clue about. > So I am telling you to try something because I think you have no clue about it and you say that I haven't got a clue? How does that rhyme with your intorduction about beiong polite? >>> And so what? Maybe you should demand total banishment of questions? That >>> would be consinsent because the first question already creates UI for > your >>> side and only without it the final result would be good enough to > satisfy >>> you. >>> >> No Maciej, you miscomprehend completely. When my partner misexplains my >> bid, I am in for a bad score already. So are you. Then, when the next >> bid comes, and they don't ask, we are still in the same situation. Still >> bad, still as bad as each other. But when they ask us what the next bid >> means, our situations diverge. My result will still be the same, bad >> one, as before. Your result will deteriorate immensily. Now I don't mind >> for you to be masochistic, but I do mind it for you to blame me for not >> being masochistic. That just means you're sadistic as well. > > Herman, you are in a need of instant logic lessons. > No Maciej, you are. > a) (no questions) -> initial score This case is not under discussion. We are discussing cases where the first question was answered wrongly. This logic stinks. > b) (first question) -> worsened score Yes, but this is the basis upon which we are judging. > c) (second question) -> more worsened score > No! c1) CE answer -> same worsened score c2) SE answer -> much worsened score > You are constantly sheding tears about having to wear the consequences of c) > instead of b). So I only advised you that to be consistent you should shed > tears because of having to wear the consequences of b) instead of a). > I am indeed shedding tears about having to wear the consequences of c2), because there are situations in which I will not suffer those consequences. And the situations differ according to something totally extraneous to the real bridge case. It is as if you wrote a law: If you revoke - and it's before the half hour, you lose one trick; - ant it's after the half hour, you lose two tricks. That's what I'm talking about. I can live with a revoke law that costs me one trick, or two, but not with this one. Your law gives opponents an extra benefit if they ask a question, but not if they look up that same question on the SC. That is why your law is a bad one, IMO. Now I agree that if I change the above example to giving no penalties in the first case, then you can say that my law is bad, since it does not punish the infraction. But that is not the case of the DWS law: the infraction (the MI) is punished, equally so when asking or when looking up the answer. But what you are punishing in addition is not the second MI, but the first one. And not by 2 penalty tricks, but by seven (IMO). >> My point is that my result is not affected by the extra question. > > Yes, not affected by the *extra* question. And so what? If the *first* > question can change the result, so can the *second*. I don't see anything > bad about it. > Nor do I, but I don't see anything good about it either. And the construction is bad, because the second question will sometimes not come. The revoke will sometimes happen before the half hour, and it won't get punished equally heavy then. And the 1 trick penalty is enough. >> is. That means that my system is a correct one, while yours has a >> terrible flaw: > > No, *in your opinion* it may look like that. You have no right to decide > which is correct. You consider yourself more clever than me? Do you want to > test it? And bet some money on the final result? > No, of course I won't. (Well, considering I am a better quizzer than bridger, I might accept such a wager :)) But I am allowed to argue in favour of one alternative and you of the other, no? >> the final result at the table is function of something >> that has no bearing on the bridge itself: the asking of a question. > > Moonshine. Questions have significant bearing on bridge. E.g. this first > question which causes UI to be present for one opponent. This topic is > closed for me. > No Maciej, questions are merely a means to an end. A player can want to know how many aces are shown and have many other ways than a question for finding out. If some of these ways are treated differently than others, you have created a bad set of laws. >>> Herman, accept that it is your side who is responsible for your supposed >>> poor score. Not the opponents, who have every right to get full > disclosure. >> But I don't touch that right - they do get full disclosure - at the end. >> Which is what your opponents get as well - on the first misexplanation! >> Bridge has always been played that way - MI is allowed to exist, and is >> corrected at the end. > > And how does it help bridge to be played normally? Not by TD after the end > of the deal? > Do you consider the hand to be played normally if the opponent remembers what kind of Blackwood his opponents are playing and feels he doesn't need to ask the question? Well, then, is the hand not equally normally played if he does ask and receives the answer "one"? Of course the TD then has to sort out the MI about 4NT being explained as Blacwood when it wasn't. But by writing L20F5a, the WBF have shown us that they don't mind about that. >> What you are giving your opponents, masochistically, is not just correct >> information about the second bid, but also correct information about the >> first bid, something which L20F5a tells us they are not -yet- entitled >> to, and also the information that you are having a misunderstanding, >> something case law tells us they are not entitled to, ever. > > Yes, they are not "entitled" to this by the Laws. But as well they are not > "entitled" to write super scores when you pass partner's cuebid or make a > defensive blunder. But they sometimes get such gifts, that is normal in > bridge. And so your mixup is another kind of gift. If you don't want to give > them any gifts - learn your system. > Yes, and the benefit they get from the misbid, they can have. But I don't need to reveal the misbid, nothing in the (current) laws says I have to. Why do you want to give them even more benefit, when the current laws say they are not entitled to this? And can we please look at this standing two paces back? Imagine there is no L20F5a. You believe it would be better if that law be written differently, don't you? Well, we've been playing bridge for more than 30 years with a law like that, and no-one has complained about it. > [sniped, as promised] > (2 Ps - a sniper shoots people) (forgive me for correcting your english, it's just that it's so good that I believe you'd appreciate correcting the small amount of mistakes left) >>> M: if the player breaks any Law he should do all he can to ensure that > his >>> infraction will have the least possible effect on the innocent side, > even if >>> it costs him more than it could >>> H: such player should accept the penalty (present or future) but he can > do >>> all that is possible to lessen the effects of this penalty on himself, > even >>> if it harms the innocent side >>> >> And I think that principle is in the laws: L10C3 and L10C4. > > I'm aware of L10C4. But it doesn't force the player to behave like that. It > only allows for such behavior. So my own idea is not conflicted with Laws. > hahaha. You don't force the player to chose the best option. :) But you criticise me for chosing the best option! > Remember that I'm not trying to prove that all my ideas are objectivly > better. I was just showing the differences between our goals. > So am I. But if we're not trying to prove that our ideas are better than the other's, why are we discussing. Of course you are trying to make me see that your legal system is better than mine. As I am to you. And we're not succeeding. >>> Yes, the TD has more work but the board is played normally (the score > will >>> rarely be adjusted) and only the offenders have to explain their actions >>> (and are ruled against UI), the NOS can calmly concentrate on the > following >>> boards and they don't have to waste their time persuading the TD (AC) > that >>> they would do something or not. >>> >> This only depends on what you define by "normally". > | | > V >> Maybe, in your system, there are less score adjustments. > > You've answered that on your own. > >> But is that really such an advantage? > > For me it's a great advantage. The greatest of the greatest. It is a base of > all my expectations regarding Bridge Laws and the authorities' main goals. > > The least TD interventions that possible. The players making their own > decisions, bad or good, having the results based solely on their own > actions, not on the virtual actions of their peers, TDs or AC members. > But I was not talking about interventions. I was talking of adjustments. You've stepped in too quickly. And then you've snipped the rest of my argument. Yes, there may be less adjustments, but not less rulings. And if you look at a player who is forced to act differently because of L16 as an adjustment of sorts, then there are just as many adjustments in your system as in mine. Porbably more, because you have 2 cases of UI and one of MI, and I have 1UI and 2MI, but one of those MI is not causing damage. And surely as professional directors we should welcome some more cases, shall we not? Herman. > Utopia? Certainly, but aiming for good utopia makes the things better. > Deciding to stay in place (because utopia is unreachable) only preserves the > bad things. > >> Herman. > > Regards > > Maciej > > > _______________________________________________ > blml mailing list > blml at amsterdamned.org > http://www.amsterdamned.org/mailman/listinfo/blml > From Hermandw at skynet.be Fri Nov 28 14:17:22 2008 From: Hermandw at skynet.be (Herman De Wael) Date: Fri, 28 Nov 2008 14:17:22 +0100 Subject: [blml] The real text. In-Reply-To: <006101c9514e$9870d480$15844c59@chello.pl> References: <49227C4E.3050007@skynet.be><000001c94a0b$1cc70330$56550990$@com><000d01c94ad2$b54bc880$15844c59@chello.pl><4925C17B.1010706@talktalk.net> <49266B37.90306@skynet.be><000f01c94bba$ecc985c0$15844c59@chello.pl><4926C496.6080904@skynet.be><003901c94c47$a1541140$15844c59@chello.pl><4927E7CC.4030303@skynet.be><001f01c94cad$4bf57340$15844c59@chello.pl> <008701c94f22$94d773c0$15844c59@chello.pl> <492D105C.8010805@skynet.be><002501c94fed$21e0e4c0$15844c59@chello.pl> <492E5900.1060401@skynet.be><001d01c95079$bd5e2b60$15844c59@chello.pl> <492E7929.7020405@skynet.be><002b01c950af$78a8d840$15844c59@chello.pl> <492ED52D.40209@skynet.be><006801c950bc$1b7cb080$15844c59@chello.pl> <492FB014.9060308@skynet.be><002b01c9513c$391d9f20$15844c59@chello.pl> <492FC4EE.6070108@skynet.be> <006101c9514e$9870d480$15844c59@chello.pl> Message-ID: <492FEF62.5070304@skynet.be> Bystry wrote: > Hi, > >>> >> But we are not talking about punishment - we are talking about a >> benefit. A benefit which comes from not giving the UI which you insist >> must be given. A benefit from partner waking up to his own >> misunderstanding from sources which are AI. That benefit! > > Herman, when somebody steals 10000E, is caught, and pays 20000E fine, is > that a *benefit* for him? Really, your arguments are sometimes worse than > those of a 10-year-old child. > IMO your counter-example is flawed. If someone steals a bicycle, and he is fined 500E, but he gets to keep his bike, then it depends on the value of the bike if he's had any benefit of it. But if he has to return the bike, even a fine of 5? means he's worse off. And even a slap on the wrist is enough. What you are doing is having the kid keep the bike, and try to tell him with a slap on the wrist that he shall never do it again. Unless he's called VdS, in which case you give him an oscar, fine him 500E, and let him keep the bike. You are defending your stance by saying that I am better off than I should be, but you won't take away my bike. That's wrong. >> You have created a law which obliges a player to give UI. UI which you >> don't want to leave unpunished. So you will insist that partner is not >> allowed to wake up. But now I'm talking about an innocent player who, >> not knowing that new law of yours, has not given the UI that you insist >> he must. And you think that a PP will help in this? > > I won't give a PP to the first-time offender. Next time he won't be > innocent. And he'll get a big PP. So big that he'll lose some tournament > places thanks to it. And he'll learn that the first duty of every player is > to obey the Laws, not to boost one's own score at all costs. > Yes, and Polish never steal MP3 players (sorry, unfair slander on your nationality). > [sniped, reductio ad absurdu] > >>> I don't create anything. I'm not a masked man behind the curtain who > pulls >>> the strings of the lawmakers' puppets. If you want to unleash you anger >>> write to WBFLC. I happen to (partially) agree with them but I'm not > their >>> representantive and I don't have any will to stand the gaff of your > constant >>> needling. >>> >> I know you are not. But you are the one defending this proposed law to >> me. By calling it "your law" I hope to make you think about it and join >> me in telling the WBF that "their law" is unworkable. > > Sorry to disappoint you but your hope is vain. This Law works very fine in > my country for the last 10 years, even without recent clarification. Your > delusions do not interest me. > You have never applied this law in your life - I urge you to give me examples, with bidding, please. > [sniped, as promised] > >> Herman. > > Regards > > Maciej > Herman. From agot at ulb.ac.be Fri Nov 28 15:06:42 2008 From: agot at ulb.ac.be (Alain Gottcheiner) Date: Fri, 28 Nov 2008 15:06:42 +0100 Subject: [blml] The Nebraskan issue (really about the methods of change) In-Reply-To: <492FED66.80303@skynet.be> References: <4926FB3B.7060408@skynet.be><003901c94f16$f0976be0$15844c59@chello.pl> <492D0FA7.1000407@skynet.be><001d01c94fe9$22069200$15844c59@chello.pl> <492E5673.9020401@skynet.be><001301c95076$846f00c0$15844c59@chello.pl> <492EBC70.30801@skynet.be><005201c950ba$ec9fbb00$15844c59@chello.pl> <492FAE6A.7070808@skynet.be> <003101c9514b$3cac6d60$15844c59@chello.pl> <492FED66.80303@skynet.be> Message-ID: <492FFAF2.8020400@ulb.ac.be> Herman De Wael a ?crit : > >> Now to the logic (I remind you that we're talking only about consistent >> strategies, not guessing what to do in every case): >> >> a) keeping to one's own version is effective in every case >> > > That depends on what you call effective. Since it gives partner UI and > opponents information they are not entitled to, I call it most ineffective. > > AG : apart from occasionnally giving MI, because you could hardly be 100% sure your version is the right one. From agot at ulb.ac.be Fri Nov 28 15:33:10 2008 From: agot at ulb.ac.be (Alain Gottcheiner) Date: Fri, 28 Nov 2008 15:33:10 +0100 Subject: [blml] reverse hesitation - how would you rule ? Message-ID: <49300126.5030802@ulb.ac.be> Hi everybody. Let's set aside philosophy or a moment and try a practical case, would you ? Pairs, vul vs not, moderate level, the bidding goes : 1NT pass 2C 2H pass pass Dbl pass 2NT pass 3NT There was a BIT before the double - undisputed facts. a) The pais's style is that a double in the seat before the long suit is never really for penalties, although this one can readily be passed. Of eight players who were asked their feelings about the double, seven, including YT, agree with this meaning, which can therefore be considered standard in this circle. Is it right to rule that, even within this context, a slow double will be taken out more readily than a spontaneous one, whence some UI was still used, and pull the contract back to 2H (for only 500) ? Opener had a perfectly normal 1NT hand including Kx of hearts. b) It now appears that responder's hand looked like a genuine penalty double, and that his hesitation was about whether he had any hope of getting them in 2H. Nobody would pretend this player would have hesitated to provoke partner's pass. It just happens that he wanted to make a penalty double, and that their 10 points in diamonds were worth only one trick, so that he was wrong in his valuation, but he's been wrong before. So the UI considered in point a) is actually inexistent, but opener still pulled a slow double. Do you still adjust, if only to make him learn ? Thank you for your precious opinion. Alain From darkbystry at wp.pl Fri Nov 28 15:38:40 2008 From: darkbystry at wp.pl (Bystry) Date: Fri, 28 Nov 2008 15:38:40 +0100 Subject: [blml] The real text. References: <49227C4E.3050007@skynet.be><000001c94a0b$1cc70330$56550990$@com><000d01c94ad2$b54bc880$15844c59@chello.pl><4925C17B.1010706@talktalk.net> <49266B37.90306@skynet.be><000f01c94bba$ecc985c0$15844c59@chello.pl><4926C496.6080904@skynet.be><003901c94c47$a1541140$15844c59@chello.pl><4927E7CC.4030303@skynet.be><001f01c94cad$4bf57340$15844c59@chello.pl> <008701c94f22$94d773c0$15844c59@chello.pl> <492D105C.8010805@skynet.be><002501c94fed$21e0e4c0$15844c59@chello.pl> <492E5900.1060401@skynet.be><001d01c95079$bd5e2b60$15844c59@chello.pl> <492E7929.7020405@skynet.be><002b01c950af$78a8d840$15844c59@chello.pl> <492ED52D.40209@skynet.be><006801c950bc$1b7cb080$15844c59@chello.pl> <492FB014.9060308@skynet.be><002b01c9513c$391d9f20$15844c59@chello.pl> <492FC4EE.6070108@skynet.be><006101c9514e$9870d480$15844c59@chello.pl> <492FEF62.5070304@skynet.be> Message-ID: <000d01c95167$01a14f80$15844c59@chello.pl> Hi, >> Herman, when somebody steals 10000E, is caught, and pays 20000E fine, is >> that a *benefit* for him? Really, your arguments are sometimes worse than >> those of a 10-year-old child. > IMO your counter-example is flawed. It is not. In this point we are only discussing the OS, not the consequences for the NOS. We're discussing criminal laws, not civil ones. The offender has, say, two "free-shots". Instead of 0% he gets 40% (A- is maximum he can ever get). So he's up 80%. Now in the next case he is up 40%, but I give him 200% PP. So overall he is 80% down. And in every future instance he will be more and more dissipative. So he'll have no choice but to grin his teeth, curse the lawmakers but change his behavior and start to obey the Laws. And please, don't start to talk about civil laws. The NOS are disadvantaged but they are equally so in many other cases which appear much more frequently. Here the price must be paid to straighten things up. Besides, they are positioned better than in the dWS anyway (counting all incomes from nailing the mixups). >> I won't give a PP to the first-time offender. Next time he won't be >> innocent. And he'll get a big PP. So big that he'll lose some tournament >> places thanks to it. And he'll learn that the first duty of every player is >> to obey the Laws, not to boost one's own score at all costs. > Yes, and Polish never steal MP3 players (sorry, unfair slander on your > nationality). They do. They kill people, rob and kidnap them, steal, cheat, take and give bribes and break the law in many other ways. And so what? You see, the difference is in detection. Most of the criminals believe that their misdeeds won't be detected and rarely that the penalty is so small that they profit from breaking the law (this shouldn't happen but the lawmakers are not perfect). In case of the dWS virtually all misdeeds will be detected. So severe penalties will be a sufficient deterrent. >> Sorry to disappoint you but your hope is vain. This Law works very fine in >> my country for the last 10 years, even without recent clarification. Your >> delusions do not interest me. > You have never applied this law in your life - I urge you to give me > examples, with bidding, please. Nobody had to apply the penalties because people do not use the dWS. How can I give you any examples? That is *you* who is afraid of benefited offenders. I'm not. > Herman. Regards Maciej From jfusselman at gmail.com Fri Nov 28 15:57:42 2008 From: jfusselman at gmail.com (Jerry Fusselman) Date: Fri, 28 Nov 2008 08:57:42 -0600 Subject: [blml] reverse hesitation - how would you rule ? In-Reply-To: <49300126.5030802@ulb.ac.be> References: <49300126.5030802@ulb.ac.be> Message-ID: <2b1e598b0811280657m409e5bc0m6eb11b6bc8ae6a48@mail.gmail.com> Can you give the table result and hands? Or at least, please make them up. From agot at ulb.ac.be Fri Nov 28 16:22:50 2008 From: agot at ulb.ac.be (Alain Gottcheiner) Date: Fri, 28 Nov 2008 16:22:50 +0100 Subject: [blml] reverse hesitation - how would you rule ? In-Reply-To: <2b1e598b0811280657m409e5bc0m6eb11b6bc8ae6a48@mail.gmail.com> References: <49300126.5030802@ulb.ac.be> <2b1e598b0811280657m409e5bc0m6eb11b6bc8ae6a48@mail.gmail.com> Message-ID: <49300CCA.30109@ulb.ac.be> Jerry Fusselman a ?crit : > Can you give the table result and hands? Or at least, please make them up. > 3NT made. I don't remember whether it was = or +1. It made (= or +1) at every table, except one where the contract was something very different. Even my blind partner made 10 tricks (he really is !). The hands are something like : J10xxx xx xxxx 10x Qxx xx Kx J10xx Qxxx AKJx AKQx Jxx AKx AQ98x x xxxx It looks like that will be 5 tricks in 2HX, and some possibility of 6, so that if we adjust we'd probably give the NOS South 6 tricks (i.e. 6 tricks are probable enough to consider it a plausible result); but anyway -300 or -500 will be the same 10/12 for NS. From rfrick at rfrick.info Fri Nov 28 17:06:29 2008 From: rfrick at rfrick.info (Robert Frick) Date: Fri, 28 Nov 2008 11:06:29 -0500 Subject: [blml] reverse hesitation - how would you rule ? In-Reply-To: <49300126.5030802@ulb.ac.be> References: <49300126.5030802@ulb.ac.be> Message-ID: On Fri, 28 Nov 2008 09:33:10 -0500, Alain Gottcheiner wrote: > Hi everybody. Let's set aside philosophy or a moment and try a practical > case, would you ? > > Pairs, vul vs not, moderate level, the bidding goes : > > 1NT pass 2C 2H > pass pass Dbl pass > 2NT pass 3NT > > There was a BIT before the double - undisputed facts. > > a) The pais's style is that a double in the seat before the long suit is > never really for penalties, although this one can readily be passed. Of > eight players who were asked their feelings about the double, seven, > including YT, agree with this meaning, which can therefore be considered > standard in this circle. > > Is it right to rule that, even within this context, a slow double will > be taken out more readily than a spontaneous one, whence some UI was > still used, and pull the contract back to 2H (for only 500) ? > Opener had a perfectly normal 1NT hand including Kx of hearts. It seems to me that one reason (but just one reason) for a BIT would be having good hearts. I thought that before reading below (though I didn't get the reason right). So that makes even less reason to pull the double. But it makes more reason to bid no trump with Kx. If the double is for take-out, implying short hearts, and the no trump opener is short in hearts, why isn't he looking for a suit fit in what seems to be a competitive part-score auction? (Unless he has a long good diamond suit.) This assumes 2C is not forcing to game. And I am guessing that over any suit bid, the 2C bidder then jumps to 3NT. > > b) It now appears that responder's hand looked like a genuine penalty > double, and that his hesitation was about whether he had any hope of > getting them in 2H. Nobody would pretend this player would have > hesitated to provoke partner's pass. It just happens that he wanted to > make a penalty double, and that their 10 points in diamonds were worth > only one trick, so that he was wrong in his valuation, but he's been > wrong before. > > So the UI considered in point a) is actually inexistent, but opener > still pulled a slow double. Do you still adjust, if only to make him > learn ? You can make people learn? If this is the real question, it is not my teaching style. And probably going beyond your authority as director. From darkbystry at wp.pl Fri Nov 28 18:33:20 2008 From: darkbystry at wp.pl (Bystry) Date: Fri, 28 Nov 2008 18:33:20 +0100 Subject: [blml] The Nebraskan issue (really about the methods of change) References: <4926FB3B.7060408@skynet.be><003901c94f16$f0976be0$15844c59@chello.pl> <492D0FA7.1000407@skynet.be><001d01c94fe9$22069200$15844c59@chello.pl> <492E5673.9020401@skynet.be><001301c95076$846f00c0$15844c59@chello.pl> <492EBC70.30801@skynet.be><005201c950ba$ec9fbb00$15844c59@chello.pl> <492FAE6A.7070808@skynet.be><003101c9514b$3cac6d60$15844c59@chello.pl> <492FED66.80303@skynet.be> Message-ID: <001501c9517f$68433060$15844c59@chello.pl> Hi, > > I'll try to refrain from answering by rudeness to rudeness. I saw many of > > your contributions to this list and you've always used phrases like "you're > > wrong, you're so so wrong", "that's silly" and similar. That is a bad way of > > discussing. It only tries to create a mirrage to the other readers that one > > of the discutants has to be surely right. But only the power of arguments > > should count, not the power or sneakiness of voice. I'd rather you've threwn > > this bad manner away but since it didn't happen in all the years, I think > > there is no more hope. > > > > I apologise for the rudeness. I am trying to convey that some of the > things my colleagues are saying are more wrong than others. When I say > that something is silly - it is because I find no other word to convey > that they are very very wrong. Apologies accepted. But it's not that I feel offended. It's not about the words. It's about the style. I simply do not find any sense in discussing with someone who is so convinced that he, and only he, is right, that the whole world around must be totally dumb not to agree with him. You are trying to create such impression and you don't have any basis for it. > In a sense, it is an acknowledgment that > some of the other arguments are not so bad at all. I would like to get > some of the bad arguments out of the way, so that the good arguments can > remain. The above is one of the worst arguments you are using. But you're not to decide which arguments are good and which are bad. Look, Herman, automaticly rejecting the arguments with which you don't agree would result in a draw at the start of every discussion (both sides would reject all the other side's arguments). Maybe if our discussion was fully private it would be the best solution - to end it instantly without exchange of the arguments. But you see, I'm not trying to convince you. I don't think it's possible at all. We are discussing publicly and the other people appraise our arguments for themselves (if they wish so). And I'm doing my best to represent my "side". I want to achieve the situation where all the people, who didn't already form their own opinions, have all the possible pros and cons available. And I'd wish you acted in the same way. You are not a judge, you are a contestant. Try to understand that and we may discuss forever, I like it. If you don't, I'll simply give up. > It is possible, indeed, that you don't really know which system partner > thinks he is playing. But that does not make it impossible for you to > give an answer which can be considered a CE. Your argument is that since > it is difficult, sometimes, to give a CE, CE must be outlawed. That is > false logic. If it proves impossible for one player in one case to give > a CE answer, that does not necessitate the WBF to outlaw all other CE > answers. No, I want to have CE outlawed because of my philosophical approach. The mere fact that sometimes CE is impossible (and you have to revert to SE) wouldn't be certainly enough for me. It's just one of the drawbacks, not the most important one. > Do you see the fallacy in your argument? Then forgive me for having used > the word "silly" on it, but please understand why I used it. I don't see any fallacy. You can't take my arguments separately and claim that any one of them is not enough to make the dWS worse. Only the totality of those arguments is for me enough. > > Now to the logic (I remind you that we're talking only about consistent > > strategies, not guessing what to do in every case): > > > > a) keeping to one's own version is effective in every case > > That depends on what you call effective. Since it gives partner UI and > opponents information they are not entitled to, I call it most ineffective. Herman, we are talking about easiness of the explanations. So "effective" has to be understood in this context. > > b) changing the story is effective in many cases, but there are some > > (distinctive minority) in which it won't be effective > > - add to this the fact that the initial bidder is much more often convinced > > that he's right > > > > There is no talk about changing any story. The story was changed. After the board (or the auction) it will come into light. start - you make a call meaning something middle - you are explaining your partner's calls as if your call meant something else end - you correct your explanation and we're back at start So the story was changed although this change was later corrected. We are talking about the story in absolute terms, not only in the terms of explanations given. You are creating some kind of illusion for the time between your partner's explanation and the end of the auction (board). And it's not important the opponents do not know at that time they're operating under this illusion. Later they get to know and don't you think it looks fishy for them? > > and it's clear that taking all the possible situations into consideration > > the player has an easy road in a) but not always in b). a) works in every > > case, b) will sometimes force you to revert to a). > > > > Again, even if it proves impossible to follow B in some cases, why does > this imply that B ought to be outlawed? It doesn't. I repeat, it's only one of the numerous arguments. > > You make any call. You believe it is your systemic call. Partner misexplains > > your bid. Now you change your version. And *that* you call consistent? And > > you dare to tell me how should I use an English word which meaning I do > > perfectly know? > > > yes, of course that is what I call consistent - the second explanation > is consistent with the first. Agreed? If you start using the same word > for both schools, you are the one who is muddying the waters, no? I'm not using this word for any school. I'm using a dictionary meening for it. You are not consistent in absolute terms - you are only temporarily consistent (for the time between first mistaken explanation and the end of auction/board) and *only* with your partner's (erroneous) explanation, not with your perception of your system. I am *absolutely* consistent - I explain according to my systemic agreements all the time and do not allow partner's delusions to influence my consistency. > As about changing my version - isn't that totally logical. You make a > call, believing that to be your system. Your partner explains it > differently. From this, you remember that he is right and you were > wrong. Why should you not then change your idea of what your system is? I should. And I would. But only when I was firmly certain. Otherwise it's better to keep with one's own version or the opponents could have (justified) feeling that I was trying to avoid UI consequences at their cost. > Only Stephanie believes that she should continue the wrong explanation > even if 100% certain that partner was right. And she's not crazy in her beliefs. I would approve of them but for the fact that it would be forcing the players to lie, in good faith, but still lie. > > Later you even admit that you change your system from that moment. How much > > hipocrisy do you want to include in your statements? > > > > But I don't change my system. I change my belief in what my system > actually was. You don't change your belief. You make out the change of your belief to avoid the troubles. I was taught by my parents that acting like that is unethical. > Have you never been wrong in your system? Have you never > made a misbid? And if you are one of those players who have never made a > misbid (there appear to be thousands of them) then can you please put > yourself in the shoes of someone who has? I have been wrong with my system. Very rarely, but I was. And I've never tried to avoid the consequences. > > And you call it "I made a misbid"? You are *lying with the straight face* > > that you'd made a misbid. > > > > Precisely. I would of course never do such a thing, and nor would you, > but we are directors, and we are faced with people who tell us that they > were absolutely certain they made a misbid. Do we believe them? > You will have to make a decision on that. Right. And the TDs of this world make such decisions all the time. In cases of misbid vs misexplanation. In cases of fielding a psyche. In cases of determination of facts. Why are you concetrating on this diddy aspect of the game? Why aren't you bothered with much more frequent situations when one side says "he was hesitating" and the other "I was certainly not hesitating"? > > Look above. But also in CS there is usually no need for the TD during the > > auction. And certainly not after the auction. In dWS the TD has to be called > > to correct great amounts of MI at the end of the bidding. And I tried to > > explain in other emails why it would be hard for the NOS to overcome > > following confusion (not to say about waste of time comparing to CS). > > > > You have two things wrong here. Sadly, many players will not follow the > UI laws and you do have to make an adjustment. And even when they say > they have tried to follow the UI laws, you still have to check that they > in fact have done so. So you still have to make a ruling. That's the > first error. I don't care about making a ruling. I'm shocked that you don't understand this after all my emails. I'm not concerned with the TD work. I'm *only* concerned with the NOS. UI rulings do not require their involvement. > The second error is that you seem to believe that MI rulings are > difficult and UI rulings are easy. I happen to think the other way. > Which proves only one thing: they are both difficult. What do you need in UI rulings? Confirm UI (here no problem) and work with "demonstrably suggested" and LAs. So you poll some players and give a ruling. For TD it may require some work, certainly, but for the players (at least for the NOS) this process is neutral. What do you need in MI rulings? You have to ask the NOS "what would you do in different scenario?". You have to find their agreements, signaling, their level of skill. You have to believe them or disbelieve (the same goes for the offenders but here I don't care). They have to waste their time responding and sometimes to swallow bitterness of being disbelieved. Finally the will only get their "equity" - statistical appraisal of the events without the infraction. *Now* you understand why I prefer UI rulings to MI rulings? > Now all over the lawbook, the lawmakers have tried to limit the amount > of UI that is being given. Why? Maybe because they feel that prevention > is better than curing. My proposal for this law also limits the UI. What > can be so wrong about that? Yes, prevention is better than curing. But in this case by preventing UI they don't prevent MI. And to be fair I don't understand why the lawmakers don't want to allow for an instant correction of MI. Maybe they care more for poor TDs than me. Anyway, ask them, not me. > >> Well, try it at the table sometime and see which of the two "looks fine". > > > > Well, try to say "Good Morning" to your neighbours. You do? So don't write > > anything you have no clue about. > > > So I am telling you to try something because I think you have no clue > about it and you say that I haven't got a clue? How does that rhyme with > your intorduction about beiong polite? "because I think you have no clue about it" - here you've got your answer. You know nothing about me. But yet you pretend to know better than me which things I made in my life and which not. Enough said. > > a) (no questions) -> initial score > > This case is not under discussion. We are discussing cases where the > first question was answered wrongly. This logic stinks. We are discussing things globally. I know that you want to restrict it because it fits you well. But no bonus. > > b) (first question) -> worsened score > > Yes, but this is the basis upon which we are judging. > > > c) (second question) -> more worsened score > > > > No! > > c1) CE answer -> same worsened score > c2) SE answer -> much worsened score The logic above was to demonstrate it to you that there are situations exactly identical in which you have no choice. It had nothing to do with CE and SE. Simple - first question causes the worsening of your result and the second one does exactly the same. Now you've found a way to counter the results of the second question. Fine. *But why don't you try to find a way to counter the results of the first question?* Answer it, Herman. Why do you detest so strongly this second worsening but accept trouble-freely the first? I repeat, the situations are exactly the same - the opponent asks the question and one of the players has UI since that time. Now I can guess what will you say. That this second part of UI is additional, that the first is already present and so on. But if you want to destroy the Tree of Evil you should cut its roots, not its leaves. So try to find a way to avoid the UI caused by the first question (the root), than the problems with the second question (the leaf) will diminish automaticly. > > You are constantly sheding tears about having to wear the consequences of c) > > instead of b). So I only advised you that to be consistent you should shed > > tears because of having to wear the consequences of b) instead of a). > > > > I am indeed shedding tears about having to wear the consequences of c2), > because there are situations in which I will not suffer those > consequences. And the situations differ according to something totally > extraneous to the real bridge case. Herman, that is luck. Nothing more. Luck is a great part of bridge. You can be lucky when the first question isn't asked. You can be lucky when the second question isn't asked. But you have to accept the consequences of both because that was *you*, nobody else, who invited luck to the party by forgetting your system. > It is as if you wrote a law: > > If you revoke > - and it's before the half hour, you lose one trick; > - ant it's after the half hour, you lose two tricks. Funny example, I'll follow it. What is our "revoke"? Not your call but *partner's misexplanation*. After it we have two possibilities: - the opponents don't ask follow-up question - you get "one trick penalty" - the opponents ask the follow-up question - you get "two trick penalty" What's so funny with it? That's similiar to how the revoke law worked till 2007. There was one trick penalty but sometimes two depending on quite random (although defined) circumstances. Of course this Law is now changed (maybe there's hope for you) but I haven't heard anybody to say that the previous Law was unjust. Only too complicated. Second thing - one of the important principles in bridge is disclosure. There are no perfect means of reaching this principle. There are side-effects which result in something that maybe shouldn't be part of bridge. You are creative, Herman. Try to find a better way of disclosure than questioning. Than convince the lawmakers to use it. But for now accept that questions are integral part of F2F bridge. And don't forget that they much more often cause troubles with UI for the questioners than for the responders. > That's what I'm talking about. I can live with a revoke law that costs > me one trick, or two, but not with this one. > Your law gives opponents an extra benefit if they ask a question, but > not if they look up that same question on the SC. > That is why your law is a bad one, IMO. If your logic applies for the second question then it has to apply as well *for the first question*. First opponents look at the CC and see that 2D is natural, the second ones ask. You get two different results. And in some strange way that doesn't bother you. Instead you're ranting for many years about the results of follow-up question which is only the consequence of the first one. Sorry, I don't see any logic in it. > >> My point is that my result is not affected by the extra question. > > > > Yes, not affected by the *extra* question. And so what? If the *first* > > question can change the result, so can the *second*. I don't see anything > > bad about it. > > > > Nor do I, but I don't see anything good about it either. > And the construction is bad, because the second question will sometimes > not come. The revoke will sometimes happen before the half hour, and it > won't get punished equally heavy then. And the 1 trick penalty is enough. Herman, I'm not trying to convince you that there is no difference between getting one kick at somebody's b**t and getting two kicks. I'm merely claiming that it's more logical to struggle to prevent both kicks, not only the second one. But you're right with one thing - I think that when you did something bad then it's better if you get two kicks and somebody else is fully compensated for your bad deed than if you get one kick and somebody else isn't fully compensated. > >> is. That means that my system is a correct one, while yours has a > >> terrible flaw: > > > > No, *in your opinion* it may look like that. You have no right to decide > > which is correct. You consider yourself more clever than me? Do you want to > > test it? And bet some money on the final result? > > > > No, of course I won't. (Well, considering I am a better quizzer than > bridger, I might accept such a wager :)) > But I am allowed to argue in favour of one alternative and you of the > other, no? Of course. But you are not allowed to decide which is better. Try to use some phrases like "in my opinion", "I think", "from my experience I..." and so on. > > Yes, they are not "entitled" to this by the Laws. But as well they are not > > "entitled" to write super scores when you pass partner's cuebid or make a > > defensive blunder. But they sometimes get such gifts, that is normal in > > bridge. And so your mixup is another kind of gift. If you don't want to give > > them any gifts - learn your system. > > > > Yes, and the benefit they get from the misbid, they can have. But I > don't need to reveal the misbid, nothing in the (current) laws says I > have to. Why do you want to give them even more benefit, when the > current laws say they are not entitled to this? You are not hiding a misbid. You are hiding your partner's misexplanation and deliberately give them another misexplanation for the time of the auction. That is bad. Realize that in SE they won't know for sure that your partner misbid. They can assume it but sometimes his hand will be similar to the correct one, sometimes following their assumption they will get worse score. That's just a function of luck. But they have all the important information and only they decide what to do with it. > And can we please look at this standing two paces back? Imagine there is > no L20F5a. You believe it would be better if that law be written > differently, don't you? Well, we've been playing bridge for more than 30 > years with a law like that, and no-one has complained about it. Are you sure that nobody complained? I have different feelings. The players constantly complain about the Laws. Mostly in the situations when they are ruled against, but not only. And I saw (in real life and in the casebooks) some complaints concerning our topic. The situation is - during the auction the call was misexplained, there were no follow-up questions and MI was only corrected after the auction (board). Now the player states that he would double the final contract (or bid to his own contract) knowing the real meaning. And he is sometimes believed, sometimes disbelieved. In the second case he certainly complains about the verdict. With L20F5a abolished he wouldn't. And the TD wouldn't have to guess what would have happened. > > [sniped, as promised] > > > > (2 Ps - a sniper shoots people) > (forgive me for correcting your english, it's just that it's so good > that I believe you'd appreciate correcting the small amount of mistakes > left) Thanks, I'm happy you've corrected me. I confused two verbs - to snip and to snipe. > > I'm aware of L10C4. But it doesn't force the player to behave like that. It > > only allows for such behavior. So my own idea is not conflicted with Laws. > > > > hahaha. > You don't force the player to chose the best option. :) > But you criticise me for chosing the best option! Yes. Laws and justice are sadly two different things. But don't be surprised that I advocate more justice in the laws :-) > > The least TD interventions that possible. The players making their own > > decisions, bad or good, having the results based solely on their own > > actions, not on the virtual actions of their peers, TDs or AC members. > > > > But I was not talking about interventions. I was talking of adjustments. > You've stepped in too quickly. And then you've snipped the rest of my > argument. Yes, there may be less adjustments, but not less rulings. And? I don't care about rulings. The TDs are paid for their work. The players have to pay for the play. Who's for whom? > if you look at a player who is forced to act differently because of L16 > as an adjustment of sorts, then there are just as many adjustments in > your system as in mine. Porbably more, because you have 2 cases of UI > and one of MI, and I have 1UI and 2MI, but one of those MI is not > causing damage. But nearly always these adjustments will burden the OS, not the NOS. That's the difference. I have nothing against the offenders being forced to waste their time and having poor scores. Maybe that will teach them to obey the Laws. I have much against the non-offenders being forced to waste their time. They did nothing wrong, were fouled, so they should be left in peace and get indemnity from the TD. > Herman. Regards Maciej From darkbystry at wp.pl Fri Nov 28 18:53:51 2008 From: darkbystry at wp.pl (Bystry) Date: Fri, 28 Nov 2008 18:53:51 +0100 Subject: [blml] reverse hesitation - how would you rule ? References: <49300126.5030802@ulb.ac.be> Message-ID: <002901c95182$46cfed80$15844c59@chello.pl> Hi, > Hi everybody. Let's set aside philosophy or a moment and try a practical > case, would you ? > > Pairs, vul vs not, moderate level, the bidding goes : > > 1NT pass 2C 2H > pass pass Dbl pass > 2NT pass 3NT > > There was a BIT before the double - undisputed facts. > > a) The pais's style is that a double in the seat before the long suit is > never really for penalties, although this one can readily be passed. Of > eight players who were asked their feelings about the double, seven, > including YT, agree with this meaning, which can therefore be considered > standard in this circle. > > Is it right to rule that, even within this context, a slow double will > be taken out more readily than a spontaneous one, whence some UI was > still used, and pull the contract back to 2H (for only 500) ? > Opener had a perfectly normal 1NT hand including Kx of hearts. That's the worst kind of double from the UI point of view. The bidding after it is not easy either. In my system this double is takeout (same is the opener's double) so the hesitation would suggest pass. If this double were penalty then the hesitation would suggest bidding. But what with "cards" double? In this position I'll assume that the hesitation suggests bidding because with poor hearts the player can be afraid of playing 2Hx while with good hearts and points he'll have to double anyway (pass will rarely be good choice and you can always play 2NT/3NT if partner takes the double out). Nevertheless I'm not sure that this logic is universal, much depends on the specific hands with which this double is used. I would question the pair thoroughly about their style but I'm inclined to adjust to 2Hx. > b) It now appears that responder's hand looked like a genuine penalty > double, and that his hesitation was about whether he had any hope of > getting them in 2H. Nobody would pretend this player would have > hesitated to provoke partner's pass. It just happens that he wanted to > make a penalty double, and that their 10 points in diamonds were worth > only one trick, so that he was wrong in his valuation, but he's been > wrong before. Ok, it happens that the hesitation is without merit. Here double could cost nothing, if it's passed - fine, if it isn't - you will play normal 3NT. What alternative did the player have? To bid 3NT at once? > So the UI considered in point a) is actually inexistent, but opener > still pulled a slow double. Do you still adjust, if only to make him learn ? No, not to make him learn. The real reason for the hesitation is not important, only what is suggested. So if you agree with my logic from the above passage you should adjust. If not, then you should let the result stand. > Thank you for your precious opinion. > > Alain Regards Maciej From Hermandw at skynet.be Fri Nov 28 19:38:11 2008 From: Hermandw at skynet.be (Herman De Wael) Date: Fri, 28 Nov 2008 19:38:11 +0100 Subject: [blml] The real text. In-Reply-To: <000d01c95167$01a14f80$15844c59@chello.pl> References: <49227C4E.3050007@skynet.be><000001c94a0b$1cc70330$56550990$@com><000d01c94ad2$b54bc880$15844c59@chello.pl><4925C17B.1010706@talktalk.net> <49266B37.90306@skynet.be><000f01c94bba$ecc985c0$15844c59@chello.pl><4926C496.6080904@skynet.be><003901c94c47$a1541140$15844c59@chello.pl><4927E7CC.4030303@skynet.be><001f01c94cad$4bf57340$15844c59@chello.pl> <008701c94f22$94d773c0$15844c59@chello.pl> <492D105C.8010805@skynet.be><002501c94fed$21e0e4c0$15844c59@chello.pl> <492E5900.1060401@skynet.be><001d01c95079$bd5e2b60$15844c59@chello.pl> <492E7929.7020405@skynet.be><002b01c950af$78a8d840$15844c59@chello.pl> <492ED52D.40209@skynet.be><006801c950bc$1b7cb080$15844c59@chello.pl> <492FB014.9060308@skynet.be><002b01c9513c$391d9f20$15844c59@chello.pl> <492FC4EE.6070108@skynet.be><006101c9514e$9870d480$15844c59@chello.pl> <492FEF62.5070304@skynet.be> <000d01c95167$01a14f80$15844c59@chello.pl> Message-ID: <49303A93.2030901@skynet.be> Bystry wrote: > Hi, > >>> Herman, when somebody steals 10000E, is caught, and pays 20000E fine, is >>> that a *benefit* for him? Really, your arguments are sometimes worse than >>> those of a 10-year-old child. > >> IMO your counter-example is flawed. > > It is not. In this point we are only discussing the OS, not the consequences > for the NOS. We're discussing criminal laws, not civil ones. > Why? I am discussing civil laws, not criminal ones ... > The offender has, say, two "free-shots". Instead of 0% he gets 40% (A- is > maximum he can ever get). So he's up 80%. Now in the next case he is up 40%, > but I give him 200% PP. So overall he is 80% down. And in every future > instance he will be more and more dissipative. So he'll have no choice but > to grin his teeth, curse the lawmakers but change his behavior and start to > obey the Laws. > This only works if you can catch him twice. If you let the kid keep the bike - and only fine him three times the value of the bike if you catch him again - he might steel again. This all depends of course on the numbers of times we think this will happen. But it is guesswork, and who says your guess is better than mine? > And please, don't start to talk about civil laws. The NOS are disadvantaged > but they are equally so in many other cases which appear much more > frequently. Here the price must be paid to straighten things up. Besides, > they are positioned better than in the dWS anyway (counting all incomes from > nailing the mixups). > Yes, but they will only complain about this particular case. And they'll complain to you, the director, who punishes (or not) their opponents for something they did wrong, but did not give them the advantage you apparently think they are due. All in all, not a very good set of laws, don't you think? And why? Just so as to get your win over Herman? What is so terribly wrong with my set of laws? >>> I won't give a PP to the first-time offender. Next time he won't be >>> innocent. And he'll get a big PP. So big that he'll lose some tournament >>> places thanks to it. And he'll learn that the first duty of every player > is >>> to obey the Laws, not to boost one's own score at all costs. > >> Yes, and Polish never steal MP3 players (sorry, unfair slander on your >> nationality). > > They do. They kill people, rob and kidnap them, steal, cheat, take and give > bribes and break the law in many other ways. And so what? You see, the > difference is in detection. Most of the criminals believe that their > misdeeds won't be detected and rarely that the penalty is so small that they > profit from breaking the law (this shouldn't happen but the lawmakers are > not perfect). > > In case of the dWS virtually all misdeeds will be detected. So severe > penalties will be a sufficient deterrent. > That is of course true. But the misdeeds will not be registered. You will not know, when you catch your bicycle thief, if he has stolen any bikes before. Because the previous time he'll have played somewhere else, under a different director, who also just gave him a slap on the wrist. >>> Sorry to disappoint you but your hope is vain. This Law works very fine > in >>> my country for the last 10 years, even without recent clarification. Your >>> delusions do not interest me. > >> You have never applied this law in your life - I urge you to give me >> examples, with bidding, please. > > Nobody had to apply the penalties because people do not use the dWS. How can > I give you any examples? That is *you* who is afraid of benefited offenders. > I'm not. > >> Herman. > > Regards > > Maciej > > Herman. From rfrick at rfrick.info Fri Nov 28 20:25:13 2008 From: rfrick at rfrick.info (Robert Frick) Date: Fri, 28 Nov 2008 14:25:13 -0500 Subject: [blml] Two alternatives for the WBF? No, three In-Reply-To: <002101c95139$1b17cc60$15844c59@chello.pl> References: <492BF30E.8020808@skynet.be> <002101c95139$1b17cc60$15844c59@chello.pl> Message-ID: On Fri, 28 Nov 2008 04:10:06 -0500, Bystry wrote: > Hi, > >> Most bridge players seem to think that after the auction is over, the >> opponents are entitled to both SE and CE. Most of the exceptions don't >> particular want to play bridge with the opponents entitled to one. > > Did you make any official polls? You're using the phrase "most bridge > players" but what are your grounds to make such assumptions? Thanks for asking. MY POLL I polled blml and got 50 answers, all clear. Then it was just using stratified sampling to get an estimate of the population mean. In reality, I got maybe 8 answers, ranging in clarity. The majority, it seemed to me, thought that players were required to divulge the meaning of their Blackwood responses. That's public record but almost impossible to retrieve. I also asked on rec.games.bridge. That you can see, hope: http://groups.google.com/group/rec.games.bridge/browse_thread/thread/81060f565cd972ce?hl=en I got a lot of immediate answers that the responses to Smithtown Blackwood had to be divulged. Riaja came in to claim that they did not legally. But Riaja was going to volunteer the information. Eventually two people who appeared who are pretty confident they they like to play bridge without divulging that information. The nature of these discussion groups is that people don't join in unless they disagree. So there were probably a lot of people who agreed with the first answers and just didn't post. rec.games.bridge is probably closer to your typical bridge player (if there is such a thing). Most of the ACBL Laws Committee apparently thought that the meaning of the Blackwood responses should be divulged. They would be a really select population, and the ACBL tends to be pro-divulgence. AM I GOOD OR WHAT? So there you have it -- a haphazard poll. But the only poll on this issue that I know of. POLLING THE POLES I weigh your opinion about Polish players. But it would mean a lot more if you actually polled some of them. Roughly, your opinion of their opinion is not something most people would weigh heavily. In revised form: You bid 4NT. Partner incorrectly describes it as being Smithtown Blackwood. You play Smithtown Blackwood, but not in that situation. It says on your convention card when you play Smithtown Blackwood. Partner responds 5 Di and becomes declarer. Before the opening lead, the opponents would like to know the meaning of a 5 Di response in Smithtown Blackwood. Are you required to tell them? Categorize the answers as A. No, of course not B. No (but they don't seem fond of playing bridge that way. For example, they say they will volunteer the information.) C. Yes. Bob, who does feel guilty about that pun but could not resist > > In Poland we have less than 7000 of PBU members (Bridge is reportedly > played > by 2 milion but mainly privately, at homes, the players are not bound by > any > Laws). From these 7000 of players the majority (I estimate it to be circa > 6500) plays recreationary Bridge in lower leauges and some local > tournaments. They have no idea what do the Laws state, they play in a way > learned many years ago (e.g popular conscious "body play") or were only > taught the basics of bridge by their friends, frequently their games are > played without any qualified TDs. From the remaining 500 of players who > play > in open tournaments and have at least vague ideas about the Laws, I saw, > say, 350-400 "in action". They simply exlain what does their system > state. > Of course there are some who may do it according to dWS or Bob's School > (not > every of those players had a bidding misunderstanding playing against me) > but I'm certain they are in minority. > > So I'm entitled to say that most *Polish* bridge players choose SE > (counting > only those who, being asked, could give any reasonable answer). But I > have > no idea what do the players from other countries choose. > > You're from the ACBL which has many more members. Against how many did > you > happen to play? How many did you see explaining? Do you have any > knowledge > about the other countries? > > Regards > > Maciej > > > _______________________________________________ > blml mailing list > blml at amsterdamned.org > http://www.amsterdamned.org/mailman/listinfo/blml From darkbystry at wp.pl Fri Nov 28 21:03:38 2008 From: darkbystry at wp.pl (Bystry) Date: Fri, 28 Nov 2008 21:03:38 +0100 Subject: [blml] The real text. References: <49227C4E.3050007@skynet.be><000001c94a0b$1cc70330$56550990$@com><000d01c94ad2$b54bc880$15844c59@chello.pl><4925C17B.1010706@talktalk.net> <49266B37.90306@skynet.be><000f01c94bba$ecc985c0$15844c59@chello.pl><4926C496.6080904@skynet.be><003901c94c47$a1541140$15844c59@chello.pl><4927E7CC.4030303@skynet.be><001f01c94cad$4bf57340$15844c59@chello.pl> <008701c94f22$94d773c0$15844c59@chello.pl> <492D105C.8010805@skynet.be><002501c94fed$21e0e4c0$15844c59@chello.pl> <492E5900.1060401@skynet.be><001d01c95079$bd5e2b60$15844c59@chello.pl> <492E7929.7020405@skynet.be><002b01c950af$78a8d840$15844c59@chello.pl> <492ED52D.40209@skynet.be><006801c950bc$1b7cb080$15844c59@chello.pl> <492FB014.9060308@skynet.be><002b01c9513c$391d9f20$15844c59@chello.pl> <492FC4EE.6070108@skynet.be><006101c9514e$9870d480$15844c59@chello.pl> <492FEF62.5070304@skynet.be><000d01c95167$01a14f80$15844c59@chello.pl> <49303A93.2030901@skynet.be> Message-ID: <000b01c95194$679382e0$15844c59@chello.pl> Hi, [snipped all] Herman, I've made myself clear. People who deliberately break the rules are cheats. The cheats are very small minority of bridge players. Some of them will be caught and PPed. Some will slize through and get their 40%, not a high profit for cheating. If you still worry about this micro-number of cases, than I can't help you. Regards Maciej From wjburrows at gmail.com Fri Nov 28 21:26:44 2008 From: wjburrows at gmail.com (Wayne Burrows) Date: Sat, 29 Nov 2008 09:26:44 +1300 Subject: [blml] reverse hesitation - how would you rule ? In-Reply-To: <49300126.5030802@ulb.ac.be> References: <49300126.5030802@ulb.ac.be> Message-ID: <2a1c3a560811281226i631e1f90wbc43cf6859cad63a@mail.gmail.com> 2008/11/29 Alain Gottcheiner : > Hi everybody. Let's set aside philosophy or a moment and try a practical > case, would you ? > > Pairs, vul vs not, moderate level, the bidding goes : > > 1NT pass 2C 2H > pass pass Dbl pass > 2NT pass 3NT > > There was a BIT before the double - undisputed facts. > > a) The pais's style is that a double in the seat before the long suit is > never really for penalties, although this one can readily be passed. Of > eight players who were asked their feelings about the double, seven, > including YT, agree with this meaning, which can therefore be considered > standard in this circle. This is irrelevant. What others consider is standard has no bearing on what this particular pair play. I had a ruling against me earlier this year "most play penalty doubles...so the slow double suggests ... and our ruling is ...". This despite us making a clear statement that our double was not penalties and we had documented evidence to support this. > > Is it right to rule that, even within this context, a slow double will > be taken out more readily than a spontaneous one, whence some UI was > still used, and pull the contract back to 2H (for only 500) ? > Opener had a perfectly normal 1NT hand including Kx of hearts. If double is not penalties I am not leaving it in with only Kx of hearts. > > b) It now appears that responder's hand looked like a genuine penalty > double, and that his hesitation was about whether he had any hope of > getting them in 2H. Nobody would pretend this player would have > hesitated to provoke partner's pass. It just happens that he wanted to > make a penalty double, and that their 10 points in diamonds were worth > only one trick, so that he was wrong in his valuation, but he's been > wrong before. This illustrates well the point that so often we can not really be sure of what the hesitation was about. If we consider this then we must come to the conclusion that actions are not suggested by a slow bid that could be slow for a multitude of reasons that do not have a common direction. > > So the UI considered in point a) is actually inexistent, but opener > still pulled a slow double. Do you still adjust, if only to make him learn ? No way!!!! > > > Thank you for your precious opinion. > > Alain > > > > _______________________________________________ > blml mailing list > blml at amsterdamned.org > http://www.amsterdamned.org/mailman/listinfo/blml > -- Wayne Burrows Palmerston North New Zealand From hirsch9000 at verizon.net Fri Nov 28 22:35:27 2008 From: hirsch9000 at verizon.net (Hirsch Davis) Date: Fri, 28 Nov 2008 16:35:27 -0500 Subject: [blml] reverse hesitation - how would you rule ? In-Reply-To: <49300126.5030802@ulb.ac.be> References: <49300126.5030802@ulb.ac.be> Message-ID: <4930641F.3050809@verizon.net> Alain Gottcheiner wrote: > Hi everybody. Let's set aside philosophy or a moment and try a practical > case, would you ? > > Pairs, vul vs not, moderate level, the bidding goes : > > 1NT pass 2C 2H > pass pass Dbl pass > 2NT pass 3NT > > There was a BIT before the double - undisputed facts. > > a) The pais's style is that a double in the seat before the long suit is > never really for penalties, although this one can readily be passed. Of > eight players who were asked their feelings about the double, seven, > including YT, agree with this meaning, which can therefore be considered > standard in this circle. > > Is it right to rule that, even within this context, a slow double will > be taken out more readily than a spontaneous one, whence some UI was > still used, and pull the contract back to 2H (for only 500) ? > Opener had a perfectly normal 1NT hand including Kx of hearts. > No. What is the UI conveyed? Why do you assume an in-tempo double will be less easy to pull? The double is value-showing, but leaves the decision to partner. So, the question is whether the BIT actually says anything that could actually suggest pulling or not pulling. What kinds of hands could produce a BIT? Well, the actual hand, with long hearts leaning towards a penalty, is one. Another might be short hearts, where the player might not want his partner to leave the double in. Double in this system says " "Partner, I have values but don't know whether to defend or go forward". Does the BIT convey any additional information not conveyed by the bid? Personally, I think the BIT in this situation is so ambiguous that it doesn't really suggest anything one way or another. > b) It now appears that responder's hand looked like a genuine penalty > double, and that his hesitation was about whether he had any hope of > getting them in 2H. Nobody would pretend this player would have > hesitated to provoke partner's pass. It just happens that he wanted to > make a penalty double, and that their 10 points in diamonds were worth > only one trick, so that he was wrong in his valuation, but he's been > wrong before. > > So the UI considered in point a) is actually inexistent, but opener > still pulled a slow double. Do you still adjust, if only to make him learn ? > > Learn what? He took the action dictated by the auction and his hand. With only two hearts, passing the optional double does not appear to be a logical alternative for the opener under the system that the pair was playing. I'd be more suspicious if the double was left in, but would still need to be convinced that the UI suggested the pass before considering an adjustment. It is not illegal to pull a slow double. It is illegal to select a logical alternative suggested by UI over another that is not suggested. If the UI was actually non-existent, or did not suggest one logical alternative over another, then there was no infraction to adjust. You do NOT adjust the score as a object lesson. > Thank you for your precious opinion. > > Alain > > No infraction. No adjustment. Hirsch From ardelm at optusnet.com.au Sat Nov 29 00:16:38 2008 From: ardelm at optusnet.com.au (Tony Musgrove) Date: Sat, 29 Nov 2008 10:16:38 +1100 Subject: [blml] pugio tuus est interfector tuus [SEC=UNOFFICIAL] In-Reply-To: References: Message-ID: <6.1.0.6.2.20081129101226.01d13cd0@mail.optusnet.com.au> At 03:05 PM 28/11/2008, you wrote: > >>If West had carefully considered the authorised > >>information from the Director's explanation of Law 26A2, > >>West might have chosen as his opening lead the one card > >>that defeats 2S -- the king of hearts! > > >Addendum: West choosing the opening lead of the king of > >hearts is necessary but not sufficient to defeat 2S. > > > >West must be careful not to lead a heart to East's ace at > >trick two, since then I could exercise my other Law 26A2 > >option and demand that East give me a ruff-and-sluff by > >playing a third round of hearts. > >Addendum to addendum: If East-West must immediately cash >two hearts in order to prevent a discard of a heart loser, >and if East-West cashing three rounds of hearts allows a >ruff-sluff discard of a diamond or club loser, then Law >26A2 and Morton's Fork mean that 2S is cold. > >The truth was somewhat more prosaic; West leading the >three of hearts rather than the king of hearts made it >easier for East to misdefend (since East thought it might >be possible that South held KJ doubleton of hearts). > >The complete deal: > > 54 > Q4 > AQT985 > J75 >952 A3 >KT3 A98752 >43 KJ >T9864 AK2 > KQJT76 > J6 > 762 > Q3 > >After West's opening lead of the three of hearts was won >by East's ace, and I used Law 26A2 to prohibit East from >leading another heart while East retained the lead, then >East's 100% best line of defence is to immediately stop >retaining the lead by switching to the trey of trumps. >Instead East opted for the 0% defensive line of ace, king >and deuce of clubs. I think I must have this wrong. I believe that when L26A says "for as long as the offender's partner retains the lead", it does not mean "whenever offender's partner is on lead". So a small heart to the Ace, heart return forbidden, Ace and another trump, and later East will have no logical alternative to playing partner for King of hearts, Regards Tony (Sydney) From grandaeval at tiscali.co.uk Sat Nov 29 00:56:30 2008 From: grandaeval at tiscali.co.uk (Grattan) Date: Fri, 28 Nov 2008 23:56:30 -0000 Subject: [blml] Gamekeeper turmed poacher. References: <492BF30E.8020808@skynet.be> Message-ID: <001201c951b5$029ace40$0302a8c0@Mildred> Grattan Endicott To: "Bridge Laws Mailing List" Sent: Friday, November 28, 2008 7:15 AM Subject: Re: [blml] Two alternatives for the WBF? No, three >> >> I may have believed that most of you advocate option A. You have often >> told us why you believe the CE is unethical and wrong. But maybe you >> don't think it should be outlawed per se. After all, maybe some of you >> have meanwhile seen that apart from applying the UI and MI laws on the >> CE, nothing else can be done. >> >> Which is why I think we should work to see the WBF accept alternative C, >> which is to allow players to make their own choices, standing ready to >> apply MI and UI laws over the choices made. >> >> Comments? >> +=+ I am having an intriguing thought put into mind by the above. If it is not actually illegal to make extraneous remarks. I wonder where we might get to down this avenue? Without commitment, let us browse a little. And I am unable to say what the WBFLC stance would be, What is specifically wrong is to provide CE only. If the player gives SE only he has complied with the requirement of the law and is not called upon to do more. . If he volunteers more, this is extraneous; he must not do so in a way to violate Law 75B or 20F5(a), or indeed any law. To give extraneous information in such a way incurs risk; if the opponent becomes confused or goes wrong in consequence it is the explainer who is in the hot seat. Partner, too, must be careful not to use the UI. It is truly tricky to explore how far one can go in this direction, if at all, without going directly against a law in the book. Any ideas, using my law book not Herman's? ~ G ~ +=+ From nigelguthrie at talktalk.net Sat Nov 29 01:02:15 2008 From: nigelguthrie at talktalk.net (Nigel Guthrie) Date: Sat, 29 Nov 2008 00:02:15 +0000 Subject: [blml] Prior argument [SEC=UNOFFICIAL] In-Reply-To: <492FB8C2.2080506@skynet.be> References: <492E75A5.4040407@talktalk.net> <492E889F.9030303@ulb.ac.be> <492EF02A.60700@talktalk.net> <492FB8C2.2080506@skynet.be> Message-ID: <49308687.8040409@talktalk.net> [Herman de Wael] There are always a number of things that you know about your partner. Those things, the opponents should know them too. But it is impractical, illegal even, to tell them these things. So what you must do is wait until the hand is over (or until the correction period, if of application) and tell them those things at that time. Then ask them if they feel damaged and call the TD if they do (or call the TD anyway). The TD can then correct if he feels that they would in fact have acted differently if they had known about the possibility of the psyche. Take you example. You know that your partner sometimes psyches, and you know how. You hear them bidding, and you have some points yourself. You conclude that he does not have his bid, and you pass. Now they play all the points in his hand, and you score a few queens. If they had known he is a more regular psycher, they could have played the hand differently and the TD may adjust for that. I have no problems with not telling them "my partner is a regular psycher" and I will try to get that message accross in some other way (mentioning on the SC seems OK), but if I fail, I'll accept my AS and not feel bad about it. Some people object to my not feeling bad about it. OK, I do feel bad about it, I accept my AS and then my bad feelings dissappear. [Nigel] IMO *Either* 1. Information about *prior* partnership agreements and partner's *idiosyncrasies* is *unauthorised* to you *or* 2. You should disclose it to opponents. I prefer the 1st alternative; and this is also my reading of the law. In the example, you know - - Partner has enjoyed psyching in the past. - *Where* he likes to psych (His favourite contexts eg NV v V, over a strong artificial bid, when losing a match by a critical margin) - *How* he likes to psych (eg opening 1H when he holds 0-3 HCP). This could help you to diagnose when a psych occurs. It could also help you in the subsequent bidding and play. IMO the information is unauthorised to you. Similarly, DWS is based on *using* UI to give MI. From nigelguthrie at talktalk.net Sat Nov 29 01:11:31 2008 From: nigelguthrie at talktalk.net (Nigel Guthrie) Date: Sat, 29 Nov 2008 00:11:31 +0000 Subject: [blml] reverse hesitation - how would you rule ? In-Reply-To: <002901c95182$46cfed80$15844c59@chello.pl> References: <49300126.5030802@ulb.ac.be> <002901c95182$46cfed80$15844c59@chello.pl> Message-ID: <493088B3.3030001@talktalk.net> [Maciej] No, not to make him learn. The real reason for the hesitation is not important, only what is suggested. So if you agree with my logic from the above passage you should adjust. If not, then you should let the result stand. [Nigel] I'm sure that Maciej is right. From ardelm at optusnet.com.au Sat Nov 29 01:37:24 2008 From: ardelm at optusnet.com.au (Tony Musgrove) Date: Sat, 29 Nov 2008 11:37:24 +1100 Subject: [blml] Taught how to psych Message-ID: <6.1.0.6.2.20081129112950.01d94448@mail.optusnet.com.au> I was talking to a pair at the local club the other day. They are both taking intermediate lessons from a local teacher, and wanted to know whether it is was legal to psyche. "Of course", I replied, "as long as no CPU, opponents no more likely etc. etc." "I suppose you mean like 1C, they double, and you psych 1S?" "Yes, we were taught that, but we were told we had to alert: could be pyschic" "Well", says I, "that's not a psych, that's part of your system, so you need to put it on the SC, and then it is probably OK" My concern is having a pair taught how to psych, and presumably in what circumstances. Is that OK with you guys? Cheers, Tony (Sydney) From blml at spiske.net Sat Nov 29 01:41:22 2008 From: blml at spiske.net (Robert Spiske) Date: Sat, 29 Nov 2008 01:41:22 +0100 Subject: [blml] reverse hesitation - how would you rule ? In-Reply-To: <49300126.5030802@ulb.ac.be> References: <49300126.5030802@ulb.ac.be> Message-ID: <49308FB2.9010800@spiske.net> Hi! Alain Gottcheiner schrieb: > Hi everybody. Let's set aside philosophy or a moment and try a practical > case, would you ? > > Pairs, vul vs not, moderate level, the bidding goes : > > 1NT pass 2C 2H > pass pass Dbl pass > 2NT pass 3NT > > There was a BIT before the double - undisputed facts. > > Does 2C establish a gameforcing for this partnership? Doesn't that make openers pass over 2H some kind of forcing? If dbl is agreed to be takeout in this gameforcing sequence, than pass is not an option. Openers pass (no 4 card major) and the 2NT bid, describe a hand that can play in both minors and has some sort of stopper in H.. I would assume that opener bidding a minor would deny a stopper. So I don't really see an LA to 2NT. Robert From wjburrows at gmail.com Sat Nov 29 01:45:46 2008 From: wjburrows at gmail.com (Wayne Burrows) Date: Sat, 29 Nov 2008 13:45:46 +1300 Subject: [blml] Taught how to psych In-Reply-To: <6.1.0.6.2.20081129112950.01d94448@mail.optusnet.com.au> References: <6.1.0.6.2.20081129112950.01d94448@mail.optusnet.com.au> Message-ID: <2a1c3a560811281645q55a38efcie1b461d12d7b8995@mail.gmail.com> 2008/11/29 Tony Musgrove : > I was talking to a pair at the local club the other day. > > They are both taking intermediate lessons from a > local teacher, and wanted to know whether it is was > legal to psyche. "Of course", I replied, "as long as > no CPU, opponents no more likely etc. etc." > "I suppose you mean like 1C, they double, and you > psych 1S?" > "Yes, we were taught that, but we were told we had > to alert: could be pyschic" > "Well", says I, "that's not a psych, that's part of your > system, so you need to put it on the SC, and then it is > probably OK" > > My concern is having a pair taught how to psych, and presumably > in what circumstances. Is that OK with you guys? > Why not its part of the game - you have to learn somehow. -- Wayne Burrows Palmerston North New Zealand From grandaeval at tiscali.co.uk Sat Nov 29 02:20:36 2008 From: grandaeval at tiscali.co.uk (Grattan) Date: Sat, 29 Nov 2008 01:20:36 -0000 Subject: [blml] Gamekeeper turned poacher. References: <492BF30E.8020808@skynet.be> <001201c951b5$029ace40$0302a8c0@Mildred> Message-ID: <003c01c951c0$b5fac110$0302a8c0@Mildred> Grattan Endicott To: "Bridge Laws Mailing List" Sent: Friday, November 28, 2008 11:56 PM Subject: Re: [blml] Gamekeeper turmed poacher. > > ----- Original Message ----- > From: "Robert Frick" > To: "Bridge Laws Mailing List" > Sent: Friday, November 28, 2008 7:15 AM > Subject: Re: [blml] Two alternatives for the WBF? No, three > > > I may have believed that most of you advocate option A. You have often > told us why you believe the CE is unethical and wrong. But maybe you > don't think it should be outlawed per se. After all, maybe some of you > have meanwhile seen that apart from applying the UI and MI laws on the > CE, nothing else can be done. >> > Which is why I think we should work to see the WBF accept alternative C, > which is to allow players to make their own choices, standing ready to > apply MI and UI laws over the choices made. > > Comments? >>> +=+ I am having an intriguing thought put into mind by the above. If it is not actually illegal to make extraneous remarks. I wonder where we might get to down this avenue? Without commitment, let us browse a little. And I am unable to say what the WBFLC stance would be, What is specifically wrong is to provide CE only. If the player gives SE only he has complied with the requirement of the law and is not called upon to do more. . If he volunteers more, this is extraneous; he must not do so in a way to violate Law 75B or 20F5(a), or indeed any law. To give extraneous information in such a way incurs risk; if the opponent becomes confused or goes wrong in consequence it is the explainer who is in the hot seat. Partner, too, must be careful not to use the UI. It is truly tricky to explore how far one can go in this direction, if at all, without going directly against a law in the book. Any ideas, using my law book not Herman's? ~ G ~ +=+ ....................................................................................................... tidied subject! From brian at meadows.pair.com Sat Nov 29 06:13:20 2008 From: brian at meadows.pair.com (brian) Date: Sat, 29 Nov 2008 00:13:20 -0500 Subject: [blml] Taught how to psych In-Reply-To: <6.1.0.6.2.20081129112950.01d94448@mail.optusnet.com.au> References: <6.1.0.6.2.20081129112950.01d94448@mail.optusnet.com.au> Message-ID: <4930CF70.8000507@meadows.pair.com> Tony Musgrove wrote: > I was talking to a pair at the local club the other day. > > They are both taking intermediate lessons from a > local teacher, and wanted to know whether it is was > legal to psyche. "Of course", I replied, "as long as > no CPU, opponents no more likely etc. etc." > "I suppose you mean like 1C, they double, and you > psych 1S?" > "Yes, we were taught that, but we were told we had > to alert: could be pyschic" > "Well", says I, "that's not a psych, that's part of your > system, so you need to put it on the SC, and then it is > probably OK" > > My concern is having a pair taught how to psych, and presumably > in what circumstances. Is that OK with you guys? > Even if you are totally anti-psyching, it is very difficult to envisage how you could reasonably teach an intermediate pair how to DEAL with possible psyches by their opponents, which is unquestionably their right, without their being able to derive from the lesson HOW to psyche in the first place. At least if they decide to try some psyches for themselves, and if the teacher is doing his/her job properly, they should gain some initial understanding of what constitutes a legal psyche, rather than find their own way by trial and error through a set of CPU penalties. Assuming that the teacher is not trying to bring them too far too fast, I think the idea is commendable. Brian. From darkbystry at wp.pl Sat Nov 29 17:15:26 2008 From: darkbystry at wp.pl (Bystry) Date: Sat, 29 Nov 2008 17:15:26 +0100 Subject: [blml] Two alternatives for the WBF? No, three References: <492BF30E.8020808@skynet.be> <002101c95139$1b17cc60$15844c59@chello.pl> Message-ID: <002101c9523d$b1275f40$15844c59@chello.pl> Hi Bob. > MY POLL > I polled blml and got 50 answers, all clear. Then it was just using > stratified sampling to get an estimate of the population mean. > > In reality, I got maybe 8 answers, ranging in clarity. The majority, it > seemed to me, thought that players were required to divulge the meaning of > their Blackwood responses. Please cite this "majority". I can't believe that any TD could misread the Laws to this extent. > That's public record but almost impossible to retrieve. > > I also asked on rec.games.bridge. That you can see, hope: > > http://groups.google.com/group/rec.games.bridge/browse_thread/thread/81060f5 65cd972ce?hl=en I've read this thread. > I got a lot of immediate answers that the responses to Smithtown Blackwood > had to be divulged. Riaja came in to claim that they did not legally. But > Riaja was going to volunteer the information. Eventually two people who > appeared who are pretty confident they they like to play bridge without > divulging that information. As far as I can estimate without counting, circa 15 people posted, some of them didn't write anything regarding your poll (as Pavlicek), so the real results were, say, 8-4 (as to what the Laws state). But later most of this 8 voters seemed to admit that the Laws are as Raija (and Gordon) explained it to them (using similar, logical arguments as me). I' m sure that you are convinced now what do the Laws state? Now, the second question of what is fair and what could be changed in the future. Here, at least on RGB (but notice that only a few people answered and many nations weren't present at all), the distinctive majority seems to be of opinion that the explanation of the virtual "auction" should be mandatory. I'll try to counter this feelings. First of all I must say that the changed Law (to fulfill your expectations) would be at variance with some principles embodied in TFLB: - often you would be forced to disclose something which is not at all related to your partnership agreements and experience - often you would be forced to disclose the contents of your hand after the misbid This Law would have another major flaw. We wouldn't be able to determine what this second, virtual system is. What if the explainer misinterpreted 3S in this virtual system? New virtual "MI"? Or should the bidder say what he thought? And if he claimed that he's not sure himself? Should he describe his hand? Too many troubles without any purpose. Second thing - fairness is undoubtly a noble goal. As you know from my discussions with Herman, I'm very harsh - all the worst for the OS, all the best for the NOS. But even my harshness has its border. The game of bridge has to be played, not practice on 4 open hands. Let's go back to your example (2D [natural, misexplained as Multi] - 3S [natural, after Multi asks for stopper] - 3NT). What is the position of the NOS? - UI has forced the OS to play the inferior contract (probably 5D or 6D would be laydown) - the NOS knows that the opener doesn't have spades (otherwise he would bid 4S or cuebid) - the NOS knows that 3S was bid under the misconception and the bidder may easily have nothing in spades - the NOS has access to pre-alerts, CC, their knowledge about this pair, their general knowledge Now you are trying to tell me that the entirety of this extra information (which wouldn't be available if 3S was a simple misbid or psyche) isn't enough for everybody (excluding possibly total beginners) to conclude for himself that leading a spade should be a good idea? Do we really have to go to the next step - the OS explains 3S as an asking for stopper to achieve "fairness"? Do we have to eliminate any, even the slightess guess for the NOS? Should we put the cards back to the box, write the top for the NOS and go to the next board? Now you can understand why don't I support the posited change. And now you can understand why would I give an additional explanation only playing against beginers. > POLLING THE POLES [snipped] I promise to arrange this poll at the first possible occasion, probably in January (I don't have emails of most of the players, I have to go to the national tournament). > Bob, who does feel guilty about that pun but could not resist Regards Maciej From Hermandw at skynet.be Sun Nov 30 12:16:24 2008 From: Hermandw at skynet.be (Herman De Wael) Date: Sun, 30 Nov 2008 12:16:24 +0100 Subject: [blml] Two alternatives for the WBF? No, three In-Reply-To: <002101c9523d$b1275f40$15844c59@chello.pl> References: <492BF30E.8020808@skynet.be> <002101c95139$1b17cc60$15844c59@chello.pl> <002101c9523d$b1275f40$15844c59@chello.pl> Message-ID: <49327608.2080303@skynet.be> Maciej, Bystry wrote: > Hi Bob. > >> MY POLL >> I polled blml and got 50 answers, all clear. Then it was just using >> stratified sampling to get an estimate of the population mean. >> >> In reality, I got maybe 8 answers, ranging in clarity. The majority, it >> seemed to me, thought that players were required to divulge the meaning of >> their Blackwood responses. > > Please cite this "majority". I can't believe that any TD could misread the > Laws to this extent. > If you can point me to the law that clearly states what you believe it states, then I'll eat a hat or two. If you think this is _clear_ then you are reading a different lawbook than I do. Herman. From Hermandw at skynet.be Sun Nov 30 12:23:56 2008 From: Hermandw at skynet.be (Herman De Wael) Date: Sun, 30 Nov 2008 12:23:56 +0100 Subject: [blml] Gamekeeper turmed poacher. In-Reply-To: <001201c951b5$029ace40$0302a8c0@Mildred> References: <492BF30E.8020808@skynet.be> <001201c951b5$029ace40$0302a8c0@Mildred> Message-ID: <493277CC.6040101@skynet.be> Grattan wrote: > > Grattan Endicott also ************************************ > "We desire truth and find within > ourselves only uncertainty." > [Pascal] > ''''''''''''''''''''''''''''''''''''''''''''''''''''''''''''''''''''''''''''''''''''''''''''''''' > > ----- Original Message ----- > From: "Robert Frick" > To: "Bridge Laws Mailing List" > Sent: Friday, November 28, 2008 7:15 AM > Subject: Re: [blml] Two alternatives for the WBF? No, three > >>> I may have believed that most of you advocate option A. You have often >>> told us why you believe the CE is unethical and wrong. But maybe you >>> don't think it should be outlawed per se. After all, maybe some of you >>> have meanwhile seen that apart from applying the UI and MI laws on the >>> CE, nothing else can be done. >>> >>> Which is why I think we should work to see the WBF accept alternative C, >>> which is to allow players to make their own choices, standing ready to >>> apply MI and UI laws over the choices made. >>> >>> Comments? >>> > +=+ I am having an intriguing thought put into mind by the above. If it is > not actually illegal to make extraneous remarks. I wonder where we might > get to down this avenue? Without commitment, let us browse a little. And > I am unable to say what the WBFLC stance would be, > What is specifically wrong is to provide CE only. If the player gives SE > only he has complied with the requirement of the law and is not called > upon to do more. . If he volunteers more, this is extraneous; he must not > do so in a way to violate Law 75B or 20F5(a), or indeed any law. > To give extraneous information in such a way incurs risk; if the opponent > becomes confused or goes wrong in consequence it is the explainer who > is in the hot seat. Partner, too, must be careful not to use the UI. It is > truly tricky to explore how far one can go in this direction, if at all, > without going directly against a law in the book. Any ideas, using my > law book not Herman's? > ~ G ~ +=+ > What I find remarkable is that the same people who believe that MI is a worse crime than giving UI, would also believe that there can be something that players know about their system but are _not_ required to tell their opponents. I have also frequently commented on responses being confusing. Which of the following two conversations is the more confusing: "4NT?" "Blackwood" "5Di?" "1 ace" or "4NT?" "Blackwood" "5Di?" "diamond preference" anyone who calls the first conversation confusing must be looking through tinted glasses. Herman. From darkbystry at wp.pl Sun Nov 30 12:51:33 2008 From: darkbystry at wp.pl (Bystry) Date: Sun, 30 Nov 2008 12:51:33 +0100 Subject: [blml] Two alternatives for the WBF? No, three References: <492BF30E.8020808@skynet.be> <002101c95139$1b17cc60$15844c59@chello.pl> <002101c9523d$b1275f40$15844c59@chello.pl> <49327608.2080303@skynet.be> Message-ID: <000b01c952e1$fe3d01a0$15844c59@chello.pl> Hi, > > Please cite this "majority". I can't believe that any TD could misread the > > Laws to this extent. > > > > If you can point me to the law that clearly states what you believe it > states, then I'll eat a hat or two. > If you think this is _clear_ then you are reading a different lawbook > than I do. I don't have to do anything. You tell me where it is written that any player is *obliged* to disclose to the opponents something that is *not a part of his partnership understandings and partnership experience*. My conversation with Bob isn't about the dWS, it's about the explanations *after the auction*. So next time check what the topic is and don't jump in with something not connected to it. > Herman. Regards Maciej From darkbystry at wp.pl Sun Nov 30 13:44:09 2008 From: darkbystry at wp.pl (Bystry) Date: Sun, 30 Nov 2008 13:44:09 +0100 Subject: [blml] Gamekeeper turmed poacher. References: <492BF30E.8020808@skynet.be> <001201c951b5$029ace40$0302a8c0@Mildred> <493277CC.6040101@skynet.be> Message-ID: <001101c952e9$572d1640$15844c59@chello.pl> Hi, > What I find remarkable is that the same people who believe that MI is a > worse crime than giving UI, would also believe that there can be > something that players know about their system but are _not_ required to > tell their opponents. Maybe because that "something" is *not what the players know about their system*? Herman, I really advise you to take some logic lessons. There is *only one system* that the partnership is playing, not two at the same time. *No* call can't be explained in rending of the actual auction. 3S in the auction 1S-pass-3S means something else than in the acution 1H-pass-3S. So if in the first case your opponent asks about 3S and you answer "3S is preemtive", in effect you answer "3S *in response to 1S* is preemptive", not that 3S is defined absolutely. Now apply this logic to your favorite 4NT example. Yes, in some auctions 5D means "one ace", but in this one, 1S-4NT-pass-5D, *it means "diamond preference"*. You are not allowed by the Laws to have two, different, meanings for this call in the same sequence (it could only mean something else if 1S meant something different). There is nothing like "4NT - Blackwood" in rending of the specific auction. 4NT - Blackwood is a convention but it has its real, bridge meaning only when specific partnership agrees to use it in specific auctions (even if defined by some meta-rules). So, yes, your opponents are entitled to know all your methods but the Laws specify the means of disclosing those methods (precisely this is left to the Regulating Authorities). And the Laws state that the players are *not entitled* to inquire about methods not related to the actual auction *during the auction and play period*. At this time the only source of the unrelated information may be System Card (CC). Read 40A1(b), 40B2(a), 40B2(c), 20F1, 20F2. > I have also frequently commented on responses being confusing. Which of > the following two conversations is the more confusing: > > "4NT?" "Blackwood" "5Di?" "1 ace" or > "4NT?" "Blackwood" "5Di?" "diamond preference" > > anyone who calls the first conversation confusing must be looking > through tinted glasses. Anyone who isn't looking at something from the overall point of view must be looking through tinted glass. The second conversation may seem confusing but every conscious opponent will realize that the partnership is in the middle of the mixup. And he will know precisely what is going on. He may use this knowledge reasonably or not, that is a matter of his bridge skills. But since that time he perfectly knows what happened and this knowledge serves him for the whole board. The first converation is certainly not confusing. The player feels comfortable, builds up an image of the deal, has an illusion that he knows what is going on. Now, after the auction, all this is ruined in one moment. He is shocked by the correction of *all* his assumptions and has to do his work for the second time under the time pressure and in the confused state of mind. Don't you really realize that it's much easier to make a simple mistake in such a state? And for this mistake there will be no redress because there is no more MI. Additionally he won't get redress for something that could have happened *during the auction*, e.g. for a double he'd otherwise make. I'm coming to sad conclusion that you have no idea how does the real bridge work. For the man who plays and directs this game for many years it is a bit strange. > Herman. Regards Maciej From Hermandw at skynet.be Sun Nov 30 14:03:29 2008 From: Hermandw at skynet.be (Herman De Wael) Date: Sun, 30 Nov 2008 14:03:29 +0100 Subject: [blml] Two alternatives for the WBF? No, three In-Reply-To: <000b01c952e1$fe3d01a0$15844c59@chello.pl> References: <492BF30E.8020808@skynet.be> <002101c95139$1b17cc60$15844c59@chello.pl> <002101c9523d$b1275f40$15844c59@chello.pl> <49327608.2080303@skynet.be> <000b01c952e1$fe3d01a0$15844c59@chello.pl> Message-ID: <49328F21.8020900@skynet.be> Bystry wrote: > Hi, > >>> Please cite this "majority". I can't believe that any TD could misread > the >>> Laws to this extent. >>> >> If you can point me to the law that clearly states what you believe it >> states, then I'll eat a hat or two. >> If you think this is _clear_ then you are reading a different lawbook >> than I do. > > I don't have to do anything. You tell me where it is written that any player > is *obliged* to disclose to the opponents something that is *not a part of > his partnership understandings and partnership experience*. > But that is precisely where we differ in opinion. How can Balckwood responses (for a pair who do actually play Blackwood, even if not in the present situation) be *NOT a part of ... experience*? They both know how many aces are shown, so they have an understanding! > My conversation with Bob isn't about the dWS, it's about the explanations > *after the auction*. So next time check what the topic is and don't jump in > with something not connected to it. > Who says it is not connected? >> Herman. > > Regards > > Maciej > > > _______________________________________________ > blml mailing list > blml at amsterdamned.org > http://www.amsterdamned.org/mailman/listinfo/blml > From Hermandw at skynet.be Sun Nov 30 14:04:49 2008 From: Hermandw at skynet.be (Herman De Wael) Date: Sun, 30 Nov 2008 14:04:49 +0100 Subject: [blml] Two alternatives for the WBF? No, three In-Reply-To: <000b01c952e1$fe3d01a0$15844c59@chello.pl> References: <492BF30E.8020808@skynet.be> <002101c95139$1b17cc60$15844c59@chello.pl> <002101c9523d$b1275f40$15844c59@chello.pl> <49327608.2080303@skynet.be> <000b01c952e1$fe3d01a0$15844c59@chello.pl> Message-ID: <49328F71.1060209@skynet.be> Bystry wrote: > > My conversation with Bob isn't about the dWS, it's about the explanations > *after the auction*. So next time check what the topic is and don't jump in > with something not connected to it. > Please YOU read the question Bob asked again. If that is not the DWS case question, I don't know what it is. Besides, this is the thread I started. Of course it's about DWS. >> Herman. > > Regards > > Maciej > > > _______________________________________________ > blml mailing list > blml at amsterdamned.org > http://www.amsterdamned.org/mailman/listinfo/blml > From darkbystry at wp.pl Sun Nov 30 15:12:58 2008 From: darkbystry at wp.pl (Bystry) Date: Sun, 30 Nov 2008 15:12:58 +0100 Subject: [blml] Two alternatives for the WBF? No, three References: <492BF30E.8020808@skynet.be> <002101c95139$1b17cc60$15844c59@chello.pl> <002101c9523d$b1275f40$15844c59@chello.pl> <49327608.2080303@skynet.be><000b01c952e1$fe3d01a0$15844c59@chello.pl> <49328F71.1060209@skynet.be> Message-ID: <001101c952f5$bf880f40$15844c59@chello.pl> Hi, > Please YOU read the question Bob asked again. If that is not the DWS > case question, I don't know what it is. Besides, this is the thread I > started. Of course it's about DWS. *I* had a long private discussion with Bob and *I* know the topic of this discussion. *You* know nothing about it. The scenario was: 2D (natural, misexplained as Multi) - 3S (natural, misbid as an asking for spade stopper, *not questioned*) - 3NT and now, after the auction the opponents ask you about an explanation. Bob wanted to know what should be told about 3S - only the systemic meaning or both - the systemic meaning and the meaning of 3S after Multi. So it has *nothing* to do with classical dWS. Later Bob made a poll on RGB putting similar question: 4NT (minors, misexlained as Blackwood) - 5D (better minor, misbid as 1 ace, *not questioned*) - auction ends and now, *after the auction* opponents ask for an explanation. Herman, I don't want to be rude, I respect you, but it was very hard for me to refrain from writing some unnecessary words. Bob even gave a link to the discussion on RGB. You didn't bother to check it, to understand the topic that is discussed, but you did have a gall to quarrel with me. Hopeless. Regards Maciej From darkbystry at wp.pl Sun Nov 30 15:47:37 2008 From: darkbystry at wp.pl (Bystry) Date: Sun, 30 Nov 2008 15:47:37 +0100 Subject: [blml] Two alternatives for the WBF? No, three References: <492BF30E.8020808@skynet.be> <002101c95139$1b17cc60$15844c59@chello.pl> <002101c9523d$b1275f40$15844c59@chello.pl> <49327608.2080303@skynet.be><000b01c952e1$fe3d01a0$15844c59@chello.pl> <49328F21.8020900@skynet.be> Message-ID: <001701c952fa$96793fc0$15844c59@chello.pl> Hi, > > I don't have to do anything. You tell me where it is written that any player > > is *obliged* to disclose to the opponents something that is *not a part of > > his partnership understandings and partnership experience*. > > > > But that is precisely where we differ in opinion. How can Balckwood > responses (for a pair who do actually play Blackwood, even if not in the > present situation) be *NOT a part of ... experience*? [L40B6(a)] When explaining the significance of partner's ***call***or play in reply to opponent's enquiry ***(see Law 20)*** a player shall disclose all ***special information*** conveyed to him through partnership agreement or partnership experience but he need not disclose inferences drawn from his knowledge and experience of matters generally known to bridge players. [L20F1] During the auction and before the final pass, any player may request, but only at his own turn to call, an explanation of the opponents' prior auction. He is entitled to know about ***calls actually made***, about ***relevant*** alternative calls available that were not made, and about ***relevant*** inferences from the choice of action where these are matters of ***partnership understanding***. Except on the instruction of the Director replies should be given by the partner of the player who made the call in question. The partner of a player who asks a question may not ask a supplementary question until his turn to call or play. Law 16 may apply and the Regulating Authority may establish regulations for written explanations. Call actually made - 5D in a sequence 1S-4NT-pas-5D. Partnership agreement - better diamonds. Relevant alternative calls - e.g. 5C - better clubs. Relevant inferences - partner doesn't have a good hand, otherwise he could bid 6D. Special information - partner, having 3-3 in minors, tends to bid clubs too often although our agreement is bidding the better of the suits. What you are doing is misusing the clear, obvious, logical definitions to suit your idiosyncratic ideas. 4NT Blackwood is *not relevant* to this auction. The special information conveyed to somebody through partnership experience *has to be connected* to the questioned call, 5D. E.g. you should disclose that partner is prone to forget that with xxx Kxx he should bid 5D, not 5C (here it doesn't matter but after 5C response it could be important for the defenders). *That* is partnership experience regarding the chosen call, not the fact that your partnership plays Blackwood or Burkina Faso Convention or Multi in other auctions. The auction consists of calls, *not* of explanations. The Laws require you to give correct explanations of *the* calls in *the* auction, not any calls in any auctions. Misexplantion "4NT is Blackwood" is extraneous to the auction and has nothing in common with "correct explanations of *the* calls". And this misexplantion *do not* create a new auction, nor a new system. So nothing has to be explained which is connected with the misexplanation. For me the logic is so simple that I have really very hard times to explain it to you. Maybe it would require spoken words, common mother language, manual demonstration, I don't know. All I know that it's the last time I'm trying to explain it. If you want to remain stubborn and continue to claim logical heresy, suit yourself. I'm washing my hands. > Herman. Regards Maciej From rfrick at rfrick.info Sun Nov 30 16:30:26 2008 From: rfrick at rfrick.info (Robert Frick) Date: Sun, 30 Nov 2008 10:30:26 -0500 Subject: [blml] Two alternatives for the WBF? No, three In-Reply-To: <001701c952fa$96793fc0$15844c59@chello.pl> References: <492BF30E.8020808@skynet.be> <002101c95139$1b17cc60$15844c59@chello.pl> <002101c9523d$b1275f40$15844c59@chello.pl> <49327608.2080303@skynet.be> <000b01c952e1$fe3d01a0$15844c59@chello.pl> <49328F21.8020900@skynet.be> <001701c952fa$96793fc0$15844c59@chello.pl> Message-ID: On Sun, 30 Nov 2008 09:47:37 -0500, Bystry wrote: > Hi, > >> > I don't have to do anything. You tell me where it is written that any > player >> > is *obliged* to disclose to the opponents something that is *not a >> part > of >> > his partnership understandings and partnership experience*. >> > >> >> But that is precisely where we differ in opinion. How can Balckwood >> responses (for a pair who do actually play Blackwood, even if not in the >> present situation) be *NOT a part of ... experience*? > > [L40B6(a)] > > When explaining the significance of partner's ***call***or play in reply > to opponent's enquiry ***(see Law 20)*** a player shall disclose all > ***special > information*** conveyed to him through partnership agreement or > partnership > experience but he need not disclose inferences drawn from his knowledge > and > experience of matters generally known to bridge players. > > [L20F1] > > During the auction and before the final pass, any player may request, > but only at his own turn to call, an explanation of the opponents' prior > auction. He is entitled to know about ***calls actually made***, about > ***relevant*** > alternative calls available that were not made, and about ***relevant*** > inferences from the choice of action where these are matters of > ***partnership > understanding***. Except on the instruction of the Director replies > should > be > given by the partner of the player who made the call in question. The > partner of a player who asks a question may not ask a supplementary > question until his turn to call or play. Law 16 may apply and the > Regulating Authority may establish regulations for written explanations. > > Call actually made - 5D in a sequence 1S-4NT-pas-5D. Partnership > agreement - > better diamonds. Relevant alternative calls - e.g. 5C - better clubs. > Relevant inferences - partner doesn't have a good hand, otherwise he > could > bid 6D. Special information - partner, having 3-3 in minors, tends to bid > clubs too often although our agreement is bidding the better of the > suits. > > What you are doing is misusing the clear, obvious, logical definitions to > suit your idiosyncratic ideas. 4NT Blackwood is *not relevant* to this > auction. The special information conveyed to somebody through partnership > experience *has to be connected* to the questioned call, 5D. E.g. you > should > disclose that partner is prone to forget that with xxx Kxx he should bid > 5D, > not 5C (here it doesn't matter but after 5C response it could be > important > for the defenders). *That* is partnership experience regarding the chosen > call, not the fact that your partnership plays Blackwood or Burkina Faso > Convention or Multi in other auctions. > > The auction consists of calls, *not* of explanations. The Laws require > you > to give correct explanations of *the* calls in *the* auction, not any > calls > in any auctions. Misexplantion "4NT is Blackwood" is extraneous to the > auction and has nothing in common with "correct explanations of *the* > calls". And this misexplantion *do not* create a new auction, nor a new > system. So nothing has to be explained which is connected with the > misexplanation. > > For me the logic is so simple that I have really very hard times to > explain > it to you. Maybe it would require spoken words, common mother language, > manual demonstration, I don't know. All I know that it's the last time > I'm > trying to explain it. If you want to remain stubborn and continue to > claim > logical heresy, suit yourself. I'm washing my hands. The WBFLC did not say why they ruled on this issue. If the laws are clear, why would they rule? Why would they have to? I have been thinking about this. One possibility is that they were clarifying the meaning of "relevant" in "about relevant inferences from the choice of action where these are matters of partnership agreement." You can infer from your partner's choice of action that partner has the ace of diamonds (or whatever your agreement is for Smithtown Blackwood or whatever). So, it seems obvious to me, the issue hinges on relevant. The information will potentially influence the opponents choice of plays. To me, that is the standard definition of relevant. The opponents are certainly going to think that their question and your answer are relevant. So to me, this law clicks in. Another way of seeing this is that partner's misunderstanding of your 4NT called has made the Smithtown Blackwood responses relevant. So you can see the WBFLC as redefining relevant. But not in an obvious way. What is the new definition of relevant? Another possibility is that the WBFLC was resolving a conflict between laws. This would imply that Law 20 does seem to require the divulgence of this information and that the WBFLC was opining that other laws took precedence. But then you are back to Law 20 at least suggesting that the information about Smithtown Blackwood has to be divulged. Bob From Hermandw at skynet.be Sun Nov 30 17:12:03 2008 From: Hermandw at skynet.be (Herman De Wael) Date: Sun, 30 Nov 2008 17:12:03 +0100 Subject: [blml] Two alternatives for the WBF? No, three In-Reply-To: <001101c952f5$bf880f40$15844c59@chello.pl> References: <492BF30E.8020808@skynet.be> <002101c95139$1b17cc60$15844c59@chello.pl> <002101c9523d$b1275f40$15844c59@chello.pl> <49327608.2080303@skynet.be><000b01c952e1$fe3d01a0$15844c59@chello.pl> <49328F71.1060209@skynet.be> <001101c952f5$bf880f40$15844c59@chello.pl> Message-ID: <4932BB53.3070407@skynet.be> Maciej, Bystry wrote: > Hi, > >> Please YOU read the question Bob asked again. If that is not the DWS >> case question, I don't know what it is. Besides, this is the thread I >> started. Of course it's about DWS. > > *I* had a long private discussion with Bob and *I* know the topic of this > discussion. *You* know nothing about it. The scenario was: > > 2D (natural, misexplained as Multi) - 3S (natural, misbid as an asking for > spade stopper, *not questioned*) - 3NT > > and now, after the auction the opponents ask you about an explanation. Bob > wanted to know what should be told about 3S - only the systemic meaning or > both - the systemic meaning and the meaning of 3S after Multi. So it has > *nothing* to do with classical dWS. > > Later Bob made a poll on RGB putting similar question: > > 4NT (minors, misexlained as Blackwood) - 5D (better minor, misbid as 1 ace, > *not questioned*) - auction ends > > and now, *after the auction* opponents ask for an explanation. > > Herman, I don't want to be rude, I respect you, but it was very hard for me > to refrain from writing some unnecessary words. Bob even gave a link to the > discussion on RGB. You didn't bother to check it, to understand the topic > that is discussed, but you did have a gall to quarrel with me. Hopeless. > And I have the same gall again. I think you are wrong, and all the words I wrote are of application to both cases you just mentioned. It is incomprehensible to me that anyone would seriously consider that it is good for the game of bridge that something that both partners know (and applies to their methods) needs not be divulged to the opponents. > Regards > > Maciej > Herman. From Hermandw at skynet.be Sun Nov 30 17:26:40 2008 From: Hermandw at skynet.be (Herman De Wael) Date: Sun, 30 Nov 2008 17:26:40 +0100 Subject: [blml] Two alternatives for the WBF? No, three In-Reply-To: <001701c952fa$96793fc0$15844c59@chello.pl> References: <492BF30E.8020808@skynet.be> <002101c95139$1b17cc60$15844c59@chello.pl> <002101c9523d$b1275f40$15844c59@chello.pl> <49327608.2080303@skynet.be><000b01c952e1$fe3d01a0$15844c59@chello.pl> <49328F21.8020900@skynet.be> <001701c952fa$96793fc0$15844c59@chello.pl> Message-ID: <4932BEC0.2020803@skynet.be> Bystry wrote: > Hi, > >>> I don't have to do anything. You tell me where it is written that any > player >>> is *obliged* to disclose to the opponents something that is *not a part > of >>> his partnership understandings and partnership experience*. >>> >> But that is precisely where we differ in opinion. How can Balckwood >> responses (for a pair who do actually play Blackwood, even if not in the >> present situation) be *NOT a part of ... experience*? > > [L40B6(a)] > > When explaining the significance of partner's ***call***or play in reply > to opponent's enquiry ***(see Law 20)*** a player shall disclose all > ***special > information*** conveyed to him through partnership agreement or partnership > experience but he need not disclose inferences drawn from his knowledge and > experience of matters generally known to bridge players. > Well? He said 4NT was Blackwood, I know which kind of Blackwood we are playing, and I know what 5Di means in that system: 1 ace. There is a high probability he has 1 ace. Is that not **special information** that is **conveyed to me** ** through partnership agreement**? It is not a matter of experience, and our particular version of Blackwood is not generally known to bridge players. I really don't understand how you can use a law that says that everything needs to be divulged, as justification for not divulging something! > [L20F1] > > During the auction and before the final pass, any player may request, > but only at his own turn to call, an explanation of the opponents' prior > auction. He is entitled to know about ***calls actually made***, about > ***relevant*** > alternative calls available that were not made, and about ***relevant*** > inferences from the choice of action where these are matters of > ***partnership > understanding***. Except on the instruction of the Director replies should > be > given by the partner of the player who made the call in question. The > partner of a player who asks a question may not ask a supplementary > question until his turn to call or play. Law 16 may apply and the > Regulating Authority may establish regulations for written explanations. > And even that sentence is open to interpretation. The WBF have found it necessary to write down that interpretation. Very bafflingly, they have found it advisable to write down an interpretation that limits the amount of information that is to be divulged to opponents. If you want to "win" this argument, then point to that interpretation. But don't go round saying that it is the logical and only possible interpretation of the Lawbook. > Call actually made - 5D in a sequence 1S-4NT-pas-5D. Partnership agreement - > better diamonds. Relevant alternative calls - e.g. 5C - better clubs. > Relevant inferences - partner doesn't have a good hand, otherwise he could > bid 6D. Special information - partner, having 3-3 in minors, tends to bid > clubs too often although our agreement is bidding the better of the suits. > > What you are doing is misusing the clear, obvious, logical definitions to > suit your idiosyncratic ideas. Clear, obvious, logical. Waw - and yet the majority on this list believes that an interpretation was needed, and I dare say that a lot of people are not happy with the result. How anyone can argue for two months and still believe his own views are "logical, obvious and clear" is beyond me. You really must have an enourmously big impression of your own capabilities, and an enourmously low one of all the others'. > 4NT Blackwood is *not relevant* to this > auction. Of course it is - he said it was that - that makes it relevant. > The special information conveyed to somebody through partnership > experience *has to be connected* to the questioned call, 5D. E.g. you should > disclose that partner is prone to forget that with xxx Kxx he should bid 5D, > not 5C (here it doesn't matter but after 5C response it could be important > for the defenders). *That* is partnership experience regarding the chosen > call, not the fact that your partnership plays Blackwood or Burkina Faso > Convention or Multi in other auctions. > Well, he is prone to forget that 4NT is not always Blackwood, in which cases he replies with his number of aces - the system of which is clearly relevant to this. > The auction consists of calls, *not* of explanations. The Laws require you > to give correct explanations of *the* calls in *the* auction, not any calls > in any auctions. Misexplantion "4NT is Blackwood" is extraneous to the > auction and has nothing in common with "correct explanations of *the* > calls". And this misexplantion *do not* create a new auction, nor a new > system. So nothing has to be explained which is connected with the > misexplanation. > I agree. But surely you also agree that the version of Blackwood employed by the partnership is part of their PU, hence disclosable to the opponents. The only small argument you have for your view is the writing of L20 which says that opponents are only allowed to ask about calls actually made. Indeed it makes sense to disallow questions like "what kind of Blackwood do you play?" when opponents have just passed 5 times in your own auction. The wording used by the lawmakers might suggest that the same question is also disallowed when 4NT was bid by them, but not in a Blackwood meaning. I feel that this wording is badly chosen in that case, and an interpretation should have enlarged the allowance. The WBF have decided, in their infinite wisdom, to go along another route. But to maintain, against all arguments, that the knowledge of "1 ace" is not an entitlement to opponents, defies all logic. To say moreover that this logically follows from the laws denies the argumentation which has gone before. If that were the case, then the WBFLC interpretation would have been written quite differently. > For me the logic is so simple that I have really very hard times to explain > it to you. Maybe it would require spoken words, common mother language, > manual demonstration, I don't know. All I know that it's the last time I'm > trying to explain it. If you want to remain stubborn and continue to claim > logical heresy, suit yourself. I'm washing my hands. > For me the logic is equally simple. If you want to remain stubborn, suit yourself. I'm washing mine. >> Herman. > > Regards > > Maciej > Herman. From Hermandw at skynet.be Sun Nov 30 17:47:21 2008 From: Hermandw at skynet.be (Herman De Wael) Date: Sun, 30 Nov 2008 17:47:21 +0100 Subject: [blml] Gamekeeper turmed poacher. In-Reply-To: <001101c952e9$572d1640$15844c59@chello.pl> References: <492BF30E.8020808@skynet.be> <001201c951b5$029ace40$0302a8c0@Mildred> <493277CC.6040101@skynet.be> <001101c952e9$572d1640$15844c59@chello.pl> Message-ID: <4932C399.8030204@skynet.be> Bystry wrote: > Hi, > >> What I find remarkable is that the same people who believe that MI is a >> worse crime than giving UI, would also believe that there can be >> something that players know about their system but are _not_ required to >> tell their opponents. > > Maybe because that "something" is *not what the players know about their > system*? Herman, I really advise you to take some logic lessons. There is > *only one system* that the partnership is playing, not two at the same time. No Maciej, there are indeed two systems. The one the 4NT player thinks he is playing, and the one the other one thinks he is playing. And even if only one of them is the correct system, the "correct" system also includes the version of Blackwood actually played by this pair in some other sequences. Please remember that in our example, the pair do actually use Blackwood in some sequences, and that both players of the pair know which version of Blackwood they are playing. So in fact there ARE two systems. And both should be divulged. It is relevant information, known to both players through partnership understanding. How you can maintain that it isn't is really beyond me. > *No* call can't be explained in rending of the actual auction. 3S in the > auction 1S-pass-3S means something else than in the acution 1H-pass-3S. So > if in the first case your opponent asks about 3S and you answer "3S is > preemtive", in effect you answer "3S *in response to 1S* is preemptive", not > that 3S is defined absolutely. > Indeed - and all I am asking here is that player A answers "5Di as a response to Blackwood is 1 ace". > Now apply this logic to your favorite 4NT example. Yes, in some auctions 5D > means "one ace", but in this one, 1S-4NT-pass-5D, *it means "diamond > preference"*. You are not allowed by the Laws to have two, different, > meanings for this call in the same sequence (it could only mean something > else if 1S meant something different). There is nothing like "4NT - > Blackwood" in rending of the specific auction. 4NT - Blackwood is a > convention but it has its real, bridge meaning only when specific > partnership agrees to use it in specific auctions (even if defined by some > meta-rules). > Sorry, no, I look at this in a different way. If 4NT is explained as "Smithtown Blackwood" then that convention needs to be fully explained. That full explanation includes what it asks for, and what the relevant responses mean. Maybe we should ask the question of the explainer then. He has explained 4NT as Blackwood, and this is an incomplete answer. He should have said something like RKCB, and if that is unknown to opponents, he should also give the complete answering scheme with it. > So, yes, your opponents are entitled to know all your methods but the Laws > specify the means of disclosing those methods (precisely this is left to the > Regulating Authorities). And the Laws state that the players are *not > entitled* to inquire about methods not related to the actual auction *during > the auction and play period*. At this time the only source of the unrelated > information may be System Card (CC). Read 40A1(b), 40B2(a), 40B2(c), 20F1, > 20F2. > Please don't patronise me - I've read all those laws and I know what they say. How you can imagine that a reasonable person like me (yes I know that is debatable) would hold an opinion which is contrary to logic and the letter of the law, and still maintain that your opinion is logical and the only sensible one, is beyond me. >> I have also frequently commented on responses being confusing. Which of >> the following two conversations is the more confusing: >> >> "4NT?" "Blackwood" "5Di?" "1 ace" or >> "4NT?" "Blackwood" "5Di?" "diamond preference" >> >> anyone who calls the first conversation confusing must be looking >> through tinted glasses. > > Anyone who isn't looking at something from the overall point of view must be > looking through tinted glass. > > The second conversation may seem confusing but every conscious opponent will > realize that the partnership is in the middle of the mixup. And he will know > precisely what is going on. He may use this knowledge reasonably or not, > that is a matter of his bridge skills. But since that time he perfectly > knows what happened and this knowledge serves him for the whole board. > You said it - it is confusing. > The first converation is certainly not confusing. The player feels You said it. > comfortable, builds up an image of the deal, has an illusion that he knows > what is going on. Now, after the auction, all this is ruined in one moment. OK. That may well be called confusing. But isn't that confusion exactly the same as the one which is given off by the second conversation at once? > He is shocked by the correction of *all* his assumptions and has to do his > work for the second time under the time pressure and in the confused state > of mind. Don't you really realize that it's much easier to make a simple > mistake in such a state? Yes, I can imagine that - and I would be lenient towards it when adjusting for the MI given. > And for this mistake there will be no redress > because there is no more MI. Additionally he won't get redress for something > that could have happened *during the auction*, e.g. for a double he'd > otherwise make. > Of course he will get redress for that - if the double would be made on the entitled information. If the double is made on the basis of the knowledge of the misunderstanding, then we are in a different discussion. IMO this information is not entitled. So to compare your situation with mine, and call your situation better, is to compare apples and oranges. You give people information they are not entitled to, I don't. Your opponents will be better off than mine. That is the crux of this whole problem. Now, IF you are going to rewrite the laws in such a way that my actions become outlawed, and IF you are going to write into your laws an adjustment, not just a PP, THEN you have changed the entitlement that the opponents have. You have made them entitled to the knowledge of the misunderstanding. Now some people actually want to do that, and it will prove an interesting discussion when the WBF get around to that for 2017, but it will apply globally. Until then, it is not fair to compare our two tables on the basis of "doubles that could have been made". Those doubles are based on information that the opponents are not entitled to have, and I haven't given them that information. And actually, all this was not my point. My point was that Grattan thinks the second conversation is confusing and the first one isn't. That is so totally wrong as to defy belief. I am glad you agree with that part at least. > I'm coming to sad conclusion that you have no idea how does the real bridge > work. For the man who plays and directs this game for many years it is a bit > strange. > I don't kow how many years you play and direct bridge, and at what level, so I cannot comment on how strange I find your absolute convictions that yours is the only possible way bridge can be played. >> Herman. > > Regards > > Maciej > Herman. From darkbystry at wp.pl Sun Nov 30 17:44:07 2008 From: darkbystry at wp.pl (Bystry) Date: Sun, 30 Nov 2008 17:44:07 +0100 Subject: [blml] Two alternatives for the WBF? No, three References: <492BF30E.8020808@skynet.be> <002101c95139$1b17cc60$15844c59@chello.pl><002101c9523d$b1275f40$15844c59@chello.pl><49327608.2080303@skynet.be><000b01c952e1$fe3d01a0$15844c59@chello.pl><49328F21.8020900@skynet.be><001701c952fa$96793fc0$15844c59@chello.pl> Message-ID: <000d01c9530a$dd1e5220$15844c59@chello.pl> Hi, > The WBFLC did not say why they ruled on this issue. If the laws are clear, > why would they rule? Why would they have to? I have been thinking about > this. They simply clarified that the words "in any manner" should mean "in any manner except when realizing the obligations put upon the player by the other Laws". That was what they always meant but some people (like Herman) found this as a hole and used to their own ideas. And actually I admit that Herman was in part right. Such holes are unnecessary and not every TD in the world has access to BLML or private clarifications from WBFLC members. So it would be better to have this words corrected in the Laws, not by the Minute. But that doesn't change the actual meaning of L20F1 which now undoubtly makes dWS illegal. [snipped] > Another possibility is that the WBFLC was resolving a conflict between > laws. This would imply that Law 20 does seem to require the divulgence of > this information and that the WBFLC was opining that other laws took > precedence. But then you are back to Law 20 at least suggesting that the > information about Smithtown Blackwood has to be divulged. It doesn't suggest anything. Bob, try this. There is an auction: 2NT-pass-3H (transfer)-pass-3S. Now the opponent asks at his turn: "what would 3S mean in an auction 2NT-pass-3H (natural)-pass-3S?". Answer for yourself - are the players obliged to give an answer to such question? Read the Laws, think about it and answer. If your answer is "no" (as I hope) then tell me why should anybody be obliged to answer the question "what would 5D mean if 4NT was Blackwood instead of minors?" > Bob Regards Maciej From darkbystry at wp.pl Sun Nov 30 17:54:06 2008 From: darkbystry at wp.pl (Bystry) Date: Sun, 30 Nov 2008 17:54:06 +0100 Subject: [blml] Two alternatives for the WBF? No, three References: <492BF30E.8020808@skynet.be> <002101c95139$1b17cc60$15844c59@chello.pl> <002101c9523d$b1275f40$15844c59@chello.pl> <49327608.2080303@skynet.be><000b01c952e1$fe3d01a0$15844c59@chello.pl> <49328F71.1060209@skynet.be><001101c952f5$bf880f40$15844c59@chello.pl> <4932BB53.3070407@skynet.be> Message-ID: <001601c9530c$42111ae0$15844c59@chello.pl> Hi, > And I have the same gall again. I think you are wrong, and all the words > I wrote are of application to both cases you just mentioned. All right, that's enough. Are you ready to support your words? I've proposed a contest in logic. Now I'm repeating this offer. I demand an apology, if not, then have enough courage to take part in the challenge. Prize - 10000 Euro. Time - whenever you wish. Contest - to be chosen. I'm waiting. I won't let anybody to have a gall to claim that he knows better what I write than myself. > Herman. Regards Maciej From svenpran at online.no Sun Nov 30 18:00:21 2008 From: svenpran at online.no (Sven Pran) Date: Sun, 30 Nov 2008 18:00:21 +0100 Subject: [blml] Two alternatives for the WBF? No, three In-Reply-To: References: <492BF30E.8020808@skynet.be> <002101c95139$1b17cc60$15844c59@chello.pl> <002101c9523d$b1275f40$15844c59@chello.pl> <49327608.2080303@skynet.be> <000b01c952e1$fe3d01a0$15844c59@chello.pl> <49328F21.8020900@skynet.be> <001701c952fa$96793fc0$15844c59@chello.pl> Message-ID: <000f01c9530d$214833b0$63d89b10$@no> On Behalf Of Robert Frick ........... > So, it seems obvious to me, the issue hinges on relevant. Sure: If a player during the auction could request explanation on "calls not made" he could request complete information on the opening bid of 1NT in the Vienna system even from a partnership that had never heard of Vienna, and he could request complete information from a partnership using the strong club system (e.g. Precision) on a one club opening bid even after a preemptive opening bid of three clubs. The addition of the word "relevant" limits the range of calls for which an explanation can be requested during the auction to such calls that could have been used in the actual situation when a player had selected a different call. Law 20F1 does not open any door for requesting general bridge lessons. So explanations of 4NT Blackwood (and the systemic answers to this bid) can only be requested after a situation in which 4NT Blackwood was available to the player. This has obviously not been the case when 4NT was a request for preference in minors whatever partner to the 4NT bidder explained. A separate issue is that opponents are entitled to disclosure of partnership understandings before play begins as specified in Law 40A1(b), but this is an entirely different matter. Sven From dalburn at btopenworld.com Sun Nov 30 18:07:06 2008 From: dalburn at btopenworld.com (David Burn) Date: Sun, 30 Nov 2008 17:07:06 -0000 Subject: [blml] Two alternatives for the WBF? No, three In-Reply-To: <001601c9530c$42111ae0$15844c59@chello.pl> References: <492BF30E.8020808@skynet.be> <002101c95139$1b17cc60$15844c59@chello.pl> <002101c9523d$b1275f40$15844c59@chello.pl> <49327608.2080303@skynet.be><000b01c952e1$fe3d01a0$15844c59@chello.pl> <49328F71.1060209@skynet.be><001101c952f5$bf880f40$15844c59@chello.pl> <4932BB53.3070407@skynet.be> <001601c9530c$42111ae0$15844c59@chello.pl> Message-ID: <001a01c9530e$1380b760$3a822620$@com> One small piece of advice: people who challenge one another to contests with large amounts of a named currency at stake are likely to have their messages end up in the junk email folders of other people. The reason for this is, I hope, obvious. David Burn London, England From darkbystry at wp.pl Sun Nov 30 18:09:30 2008 From: darkbystry at wp.pl (Bystry) Date: Sun, 30 Nov 2008 18:09:30 +0100 Subject: [blml] Two alternatives for the WBF? No, three References: <492BF30E.8020808@skynet.be> <002101c95139$1b17cc60$15844c59@chello.pl> <002101c9523d$b1275f40$15844c59@chello.pl> <49327608.2080303@skynet.be><000b01c952e1$fe3d01a0$15844c59@chello.pl> <49328F21.8020900@skynet.be><001701c952fa$96793fc0$15844c59@chello.pl> <4932BEC0.2020803@skynet.be> Message-ID: <003801c9530e$688eb400$15844c59@chello.pl> Hi, > > The auction consists of calls, *not* of explanations. The Laws require you > > to give correct explanations of *the* calls in *the* auction, not any calls > > in any auctions. Misexplantion "4NT is Blackwood" is extraneous to the > > auction and has nothing in common with "correct explanations of *the* > > calls". And this misexplantion *do not* create a new auction, nor a new > > system. So nothing has to be explained which is connected with the > > misexplanation. > > > > I agree. But surely you also agree that the version of Blackwood > employed by the partnership is part of their PU, hence disclosable to > the opponents. The only small argument you have for your view is the > writing of L20 which says that opponents are only allowed to ask about > calls actually made. Small argument? Maybe the clue of the matter? You are so ridiculous that it makes you pathetic. > Indeed it makes sense to disallow questions like "what kind of Blackwood > do you play?" when opponents have just passed 5 times in your own auction. > The wording used by the lawmakers might suggest that the same question > is also disallowed when 4NT was bid by them, but not in a Blackwood meaning. > I feel that this wording is badly chosen in that case, and an > interpretation should have enlarged the allowance. The WBF have decided, > in their infinite wisdom, to go along another route. Bravo. So you admit that but you still had a gall to claim that I was wrong stating what the Laws mean? Hopeless - the mildest word I found to decsribe your behavior. > But to maintain, against all arguments, that the knowledge of "1 ace" is > not an entitlement to opponents, defies all logic. To say moreover that > this logically follows from the laws denies the argumentation which has > gone before. If that were the case, then the WBFLC interpretation would > have been written quite differently. Their interpretation was only connected with the words "in any manner". They didn't address any other words in Laws 20 and 40. So it's obvious they recognize their contents as appropriate and clear. Please, chill out. Send your last message (to satisfy your ego that yours is the last one) and we are ending our discussion. I'm starting to become rude and I really hate it. > Herman. Regards Maciej From darkbystry at wp.pl Sun Nov 30 18:33:21 2008 From: darkbystry at wp.pl (Bystry) Date: Sun, 30 Nov 2008 18:33:21 +0100 Subject: [blml] Two alternatives for the WBF? No, three References: <492BF30E.8020808@skynet.be> <002101c95139$1b17cc60$15844c59@chello.pl> <002101c9523d$b1275f40$15844c59@chello.pl> <49327608.2080303@skynet.be><000b01c952e1$fe3d01a0$15844c59@chello.pl> <49328F71.1060209@skynet.be><001101c952f5$bf880f40$15844c59@chello.pl> <4932BB53.3070407@skynet.be><001601c9530c$42111ae0$15844c59@chello.pl> <001a01c9530e$1380b760$3a822620$@com> Message-ID: <005d01c95311$bdd879c0$15844c59@chello.pl> Hi, > One small piece of advice: people who challenge one another to contests with > large amounts of a named currency at stake are likely to have their messages > end up in the junk email folders of other people. The reason for this is, I > hope, obvious. You are right David, that was unnecessary. Maybe I should simply ignore things that make me mad. Thanks for your remainder. I apologize Herman and everyone who could feel in any way offended or disgusted by my messages. > David Burn > London, England Regards Maciej From darkbystry at wp.pl Sun Nov 30 18:50:38 2008 From: darkbystry at wp.pl (Bystry) Date: Sun, 30 Nov 2008 18:50:38 +0100 Subject: [blml] Gamekeeper turmed poacher. References: <492BF30E.8020808@skynet.be> <001201c951b5$029ace40$0302a8c0@Mildred> <493277CC.6040101@skynet.be><001101c952e9$572d1640$15844c59@chello.pl> <4932C399.8030204@skynet.be> Message-ID: <005e01c95314$28016080$15844c59@chello.pl> Hi, > Maybe we should ask the question of the explainer then. He has explained > 4NT as Blackwood, and this is an incomplete answer. He should have said > something like RKCB, and if that is unknown to opponents, he should also > give the complete answering scheme with it. That is at least a constructive proposal. But to allow for something like that would create many problems - imagine the opponents asking about Blackwood and the responder giving the set of answers to remind his partner of them. The creation of UI would be to great to cope with it (it would be hard to prove that the partner earlier forgot the responses). But it seems for me the best way to introduce the dWS through the kitchen doors. If you are able to expand this topic it may be worth considering. > > comfortable, builds up an image of the deal, has an illusion that he knows > > what is going on. Now, after the auction, all this is ruined in one moment. > > OK. That may well be called confusing. > But isn't that confusion exactly the same as the one which is given off > by the second conversation at once? There is some confusion but the amount is significantly reduced. You have to cope only with one piece of correction of MI. But after the auction you get a whole set of corrections - *all* the explanations are changed. > > He is shocked by the correction of *all* his assumptions and has to do his > > work for the second time under the time pressure and in the confused state > > of mind. Don't you really realize that it's much easier to make a simple > > mistake in such a state? > > Yes, I can imagine that - and I would be lenient towards it when > adjusting for the MI given. But Herman, there won't be any MI anymore. That's the crux of the matter. Every mistake made after the correction will count. You can't redress that, there is no Law that allows for it. > And actually, all this was not my point. My point was that Grattan > thinks the second conversation is confusing and the first one isn't. > That is so totally wrong as to defy belief. I am glad you agree with > that part at least. I can't speak for Grattan but I doubt that he thinks like that. Maybe he thinks along my lines? > I don't kow how many years you play and direct bridge, and at what > level, so I cannot comment on how strange I find your absolute > convictions that yours is the only possible way bridge can be played. That is not dependent on my playing career. My logic derives from the fundaments of bridge - no player is entitled to know the contents of the hand of any other player. So I can't find any reason to tell to the opponents how many Aces are shown by 5D. They know about mixup, they know about pre-alerts, CC, sometimes about our methods, about most popular versions of Blackwood. Why should I reveal anything more? Maybe 1 ace is not so important but look at Bob's example - there your approach would automatically result in the top for the NOS without even making the opening lead. Yes, it may seem inconsistent with my approach (all the worst for the OS) but I still want bridge to be played and without any, even the smallest, risk the game is played in open hands and that I can't approve of. > Herman. Regards Maciej From dalburn at btopenworld.com Sun Nov 30 18:57:09 2008 From: dalburn at btopenworld.com (David Burn) Date: Sun, 30 Nov 2008 17:57:09 -0000 Subject: [blml] Two alternatives for the WBF? No, three In-Reply-To: <005d01c95311$bdd879c0$15844c59@chello.pl> References: <492BF30E.8020808@skynet.be> <002101c95139$1b17cc60$15844c59@chello.pl> <002101c9523d$b1275f40$15844c59@chello.pl> <49327608.2080303@skynet.be><000b01c952e1$fe3d01a0$15844c59@chello.pl> <49328F71.1060209@skynet.be><001101c952f5$bf880f40$15844c59@chello.pl> <4932BB53.3070407@skynet.be><001601c9530c$42111ae0$15844c59@chello.pl> <001a01c9530e$1380b760$3a822620$@com> <005d01c95311$bdd879c0$15844c59@chello.pl> Message-ID: <001b01c95315$10e743a0$32b5cae0$@com> >You are right David, that was unnecessary. Maybe I should simply ignore things that make me mad. It's not a question of discourtesy. But messages involving offers of vast sums of money will be treated by many modern spam filters as if they came from an unknown African banker or a non-existent lottery. Most of my BLML messages go into my BLML folder, but your wager with Herman went straight into the junk mail. David Burn London, England From darkbystry at wp.pl Sun Nov 30 19:11:37 2008 From: darkbystry at wp.pl (Bystry) Date: Sun, 30 Nov 2008 19:11:37 +0100 Subject: [blml] Two alternatives for the WBF? No, three References: <492BF30E.8020808@skynet.be> <002101c95139$1b17cc60$15844c59@chello.pl> <002101c9523d$b1275f40$15844c59@chello.pl> <49327608.2080303@skynet.be><000b01c952e1$fe3d01a0$15844c59@chello.pl> <49328F71.1060209@skynet.be><001101c952f5$bf880f40$15844c59@chello.pl> <4932BB53.3070407@skynet.be><001601c9530c$42111ae0$15844c59@chello.pl> <001a01c9530e$1380b760$3a822620$@com><005d01c95311$bdd879c0$15844c59@chello.pl> <001b01c95315$10e743a0$32b5cae0$@com> Message-ID: <000f01c95317$163b2d60$15844c59@chello.pl> Hi, > >You are right David, that was unnecessary. Maybe I should simply ignore > things that make me mad. > > It's not a question of discourtesy. But messages involving offers of vast > sums of money will be treated by many modern spam filters as if they came > from an unknown African banker or a non-existent lottery. Most of my BLML > messages go into my BLML folder, but your wager with Herman went straight > into the junk mail. O, I see. I misread your message because of the premature assumption as to its aim. And I'm a programmer, for God's sake :-( But something good went out of my misunderstating. My head cooled down and I apologized. > David Burn > London, England Regards Maciej From rfrick at rfrick.info Sun Nov 30 19:26:25 2008 From: rfrick at rfrick.info (Robert Frick) Date: Sun, 30 Nov 2008 13:26:25 -0500 Subject: [blml] A question In-Reply-To: <000f01c9530d$214833b0$63d89b10$@no> References: <492BF30E.8020808@skynet.be> <002101c95139$1b17cc60$15844c59@chello.pl> <002101c9523d$b1275f40$15844c59@chello.pl> <49327608.2080303@skynet.be> <000b01c952e1$fe3d01a0$15844c59@chello.pl> <49328F21.8020900@skynet.be> <001701c952fa$96793fc0$15844c59@chello.pl> <000f01c9530d$214833b0$63d89b10$@no> Message-ID: North East South West 1C 1C/2C The insufficient bid of 1C is corrected to 2C. Is partner of the insufficient bidder required to divulge the partnership agreements, etc., of an opening 1C bid? Sorry if this is obvious. Your answer is interesting only if you think that not all partnership agreements, etc., must be divulged upon request. Bob From svenpran at online.no Sun Nov 30 20:02:49 2008 From: svenpran at online.no (Sven Pran) Date: Sun, 30 Nov 2008 20:02:49 +0100 Subject: [blml] Two alternatives for the WBF? No, three In-Reply-To: <005d01c95311$bdd879c0$15844c59@chello.pl> References: <492BF30E.8020808@skynet.be> <002101c95139$1b17cc60$15844c59@chello.pl> <002101c9523d$b1275f40$15844c59@chello.pl> <49327608.2080303@skynet.be><000b01c952e1$fe3d01a0$15844c59@chello.pl> <49328F71.1060209@skynet.be><001101c952f5$bf880f40$15844c59@chello.pl> <4932BB53.3070407@skynet.be><001601c9530c$42111ae0$15844c59@chello.pl> <001a01c9530e$1380b760$3a822620$@com> <005d01c95311$bdd879c0$15844c59@chello.pl> Message-ID: <001401c9531e$3d4d2500$b7e76f00$@no> On Behalf Of Bystry ............ > I apologize ....... everyone who could feel in any way offended or disgusted by my messages. Not YOUR messages pal, I have since long ago ignored all messages from Herman, and only very occasionally do I waste time by peeping if there could be anything of interest from him. There are also several distinguished contributors to blml that simply have quit this list. I consider that a pity because the list still has much to offer. For a while I feared that you should feel mobbed out and quit; I do hope that you will just filter and continue your interest in those discussions that are of value, Regards Sven From svenpran at online.no Sun Nov 30 20:41:17 2008 From: svenpran at online.no (Sven Pran) Date: Sun, 30 Nov 2008 20:41:17 +0100 Subject: [blml] A question In-Reply-To: References: <492BF30E.8020808@skynet.be> <002101c95139$1b17cc60$15844c59@chello.pl> <002101c9523d$b1275f40$15844c59@chello.pl> <49327608.2080303@skynet.be> <000b01c952e1$fe3d01a0$15844c59@chello.pl> <49328F21.8020900@skynet.be> <001701c952fa$96793fc0$15844c59@chello.pl> <000f01c9530d$214833b0$63d89b10$@no> Message-ID: <001501c95323$9cd0a510$d671ef30$@no> On Behalf Of Robert Frick > Sent: 30. november 2008 19:26 > To: Bridge Laws Mailing List > Subject: [blml] A question > > North East South West > 1C 1C/2C > > The insufficient bid of 1C is corrected to 2C. Is partner of the > insufficient bidder required to divulge the partnership agreements, etc., > of an opening 1C bid? > > > Sorry if this is obvious. Your answer is interesting only if you think > that not all partnership agreements, etc., must be divulged upon request. It IS obvious. If a partnership has failed its "duty to make available its partnership understandings to opponents before commencing play against them" (L40A1) then of course such questions may be asked and must be answered during the auction provided the questioned call is relevant in the actual situation. Regulations usually require for instance all opening bids to be disclosed under this law. I think a reminder is in order that opponents may assume, but only at their own risk, that the insufficient bid was intended as an opening bid. They have no redress if it should develop that the bid actually was a misbid and that the opener failed to correct this misbid legally under Law 25A. The only information available to opponents when the Director allows the auction to continue without restrictions on the offender's partner is that one or both (but not which!) conditions in L27A was satisfied. Consequently opponents are only entitled to a description of the 2C bid in this situation. Regards Sven From PeterEidt at t-online.de Sun Nov 30 21:00:52 2008 From: PeterEidt at t-online.de (Peter Eidt) Date: Sun, 30 Nov 2008 21:00:52 +0100 Subject: [blml] =?iso-8859-15?q?A_question?= In-Reply-To: References: <492BF30E.8020808@skynet.be> <002101c95139$1b17cc60$15844c59@chello.pl> <002101c9523d$b1275f40$15844c59@chello.pl> <49327608.2080303@skynet.be> <000b01c952e1$fe3d01a0$15844c59@chello.pl> <49328F21.8020900@skynet.be> <001701c952fa$96793fc0$15844c59@chello.pl> <000f01c9530d$214833b0$63d89b10$@no> Message-ID: <1L6sTM-2GqtCS0@fwd04.aul.t-online.de> From: "Robert Frick" > North ?East ?South ?West > 1C ? ? 1C/2C > > The insufficient bid of 1C is corrected to 2C. Is partner of the > insufficient bidder required to divulge the partnership agreements, > etc., of an opening 1C bid? Law 20 F1: "During the auction and before the final pass, any player may request, but only at his own turn to call, an explanation of the opponents? prior auction. He is entitled to know about calls actually made, about relevant alternative calls available that were not made, and about relevant inferences from the choice of action where these are matters of partnership understanding. " From darkbystry at wp.pl Sun Nov 30 21:57:11 2008 From: darkbystry at wp.pl (Bystry) Date: Sun, 30 Nov 2008 21:57:11 +0100 Subject: [blml] Two alternatives for the WBF? No, three References: <492BF30E.8020808@skynet.be> <002101c95139$1b17cc60$15844c59@chello.pl> <002101c9523d$b1275f40$15844c59@chello.pl> <49327608.2080303@skynet.be><000b01c952e1$fe3d01a0$15844c59@chello.pl> <49328F71.1060209@skynet.be><001101c952f5$bf880f40$15844c59@chello.pl> <4932BB53.3070407@skynet.be><001601c9530c$42111ae0$15844c59@chello.pl> <001a01c9530e$1380b760$3a822620$@com><005d01c95311$bdd879c0$15844c59@chello.pl> <001401c9531e$3d4d2500$b7e76f00$@no> Message-ID: <001701c9532e$376a8aa0$15844c59@chello.pl> Hi, > > I apologize ....... everyone who could feel in any way offended or > disgusted by my messages. > > Not YOUR messages pal, > > I have since long ago ignored all messages from Herman, and only very > occasionally do I waste time by peeping if there could be anything of > interest from him. There are also several distinguished contributors to blml > that simply have quit this list. I consider that a pity because the list > still has much to offer. For a while I feared that you should feel mobbed > out and quit; I do hope that you will just filter and continue your interest > in those discussions that are of value, Don't be afraid, I have quite strong character and it wouldn't be easy to mob me out. Unfortunately I'm very nervous (health reasons) and it sometimes comes out unnecessairly (regretably even at the bridge table). So although I totally don't agree with Herman and his assumptions, that he knows better than me what I do, think and write, are offensive and unjustified, I still shouldn't attack him personally and state any too vicious comments. Therefore my apologies were necessary. That doesn't mean that I'm withdrawing anything of substance. > Regards Sven Regards Maciej From rfrick at rfrick.info Sun Nov 30 23:15:32 2008 From: rfrick at rfrick.info (Robert Frick) Date: Sun, 30 Nov 2008 17:15:32 -0500 Subject: [blml] A question In-Reply-To: <1L6sTM-2GqtCS0@fwd04.aul.t-online.de> References: <492BF30E.8020808@skynet.be> <002101c95139$1b17cc60$15844c59@chello.pl> <002101c9523d$b1275f40$15844c59@chello.pl> <49327608.2080303@skynet.be> <000b01c952e1$fe3d01a0$15844c59@chello.pl> <49328F21.8020900@skynet.be> <001701c952fa$96793fc0$15844c59@chello.pl> <000f01c9530d$214833b0$63d89b10$@no> <1L6sTM-2GqtCS0@fwd04.aul.t-online.de> Message-ID: On Sun, 30 Nov 2008 15:00:52 -0500, Peter Eidt wrote: > From: "Robert Frick" >> North ?East ?South ?West >> 1C ? ? 1C/2C >> >> The insufficient bid of 1C is corrected to 2C. Is partner of the >> insufficient bidder required to divulge the partnership agreements, >> etc., of an opening 1C bid? > Law 20 F1: > "During the auction and before the final pass, any player > may request, but only at his own turn to call, an explanation > of the opponents? prior auction. He is entitled to know about > calls actually made, about relevant alternative calls available > that were not made, and about relevant inferences from the > choice of action where these are matters of partnership > understanding. " Right. Then what? Suppose you think that players have to explain the responses to Smithtown Blackwood even though your 4NT bid wasn't Smithtown Blackwood. (Partner incorrectly thought it was, explained it as being that, responded 5 Di, and is now declarer.) Then I am pretty sure you will say that the players have to divulge the meaning of a 1C opener. I get that. But there is a way of reading L20F1 so that some people think it doesn't require you to divulge that information. Really. Trust me. I am guessing that those people will also say that the player's don't have to divulge the meaning of the 1C bid. Or if they don't, I am eager to hear how they make that distinction. Then you have the WBFLC opinion that the law should be read the second way. So how should this law be read? Sven wrote: "The addition of the word 'relevant' limits the range of calls for which an explanation can be requested during the auction to such calls that could have been used in the actual situation when a player had selected a different call." and "So explanations of 4NT Blackwood (and the systemic answers to this bid) can only be requested after a situation in which 4NT Blackwood was available to the player." Do people agree with this? The player could not open 1C, so the meaning of the 1C opening does not have to be divulged? Is that what Sven meant when he said the answer was obvious? Bob From richard.hills at immi.gov.au Sun Nov 30 23:35:33 2008 From: richard.hills at immi.gov.au (richard.hills at immi.gov.au) Date: Mon, 1 Dec 2008 09:35:33 +1100 Subject: [blml] A question [SEC=UNOFFICIAL] In-Reply-To: <1L6sTM-2GqtCS0@fwd04.aul.t-online.de> Message-ID: Robert Frick asked: >North ?East ?South ?West >1C ? ? 1C/2C > >The insufficient bid of 1C is corrected to 2C. Is partner of the >insufficient bidder required to divulge the partnership agreements, >etc., of an opening 1C bid? Peter Eidt quotes Law 20F1: "During the auction and before the final pass, any player may request, but only at his own turn to call, an explanation of the opponents' prior auction. He is entitled to know about calls actually made, about relevant alternative calls available that were not made, and about relevant inferences from the choice of action where these are matters of partnership understanding." Richard Hills asks: Is an insufficient bid a call actually made? Or is an insufficient bid a call merely potentially made, becoming actually made if and only if LHO accepts it under Law 27A? Is a non-accepted insufficient bid a relevant alternative call? Or is a non-accepted and/or hypothetical illegal call never relevant? Best wishes Richard Hills, Aqua 5, workstation W550 Telephone: 02 6223 8453 Email: richard.hills at immi.gov.au Recruitment Section & DIAC Social Club movie tickets -------------------------------------------------------------------- Important Notice: If you have received this email by mistake, please advise the sender and delete the message and attachments immediately. This email, including attachments, may contain confidential, sensitive, legally privileged and/or copyright information. Any review, retransmission, dissemination or other use of this information by persons or entities other than the intended recipient is prohibited. DIAC respects your privacy and has obligations under the Privacy Act 1988. The official departmental privacy policy can be viewed on the department's website at www.immi.gov.au. See: http://www.immi.gov.au/functional/privacy.htm --------------------------------------------------------------------- From picatou at picatou.com Mon Nov 24 13:25:45 2008 From: picatou at picatou.com (Picatou) Date: Mon, 24 Nov 2008 07:25:45 -0500 Subject: [blml] Law 27 Message-ID: <000c01c94e2f$c6a8caf0$53fa60d0$@com> Hi all, I spent some nights trying to set a flow chart of the new Law 27 on one single page and think I finally succeeded, but I still have a question: 27B1(a) reads: if the insufficient bid is corrected by the lowest sufficient..... the auction proceeds without further rectification. Law 16D does not apply, but see D below. 27B1(b) : If, except as in (a) above, the insufficient bid is corrected....... the auction proceeds without further rectification, but see D below. Does the fact that "Law 16D does not apply" is not written in 27B1(b) means something ? Laval Du Breuil Quebec City PS: if some of you would like to see this flow chart, send me a private message. The 2008 edition of Laws of Duplicate bridge Made Easier is now ready for printing (French and English versions) and I will put the included flow charts (38) on my WEB as soon as possible. -------------- next part -------------- An HTML attachment was scrubbed... URL: http://www.amsterdamned.org/pipermail/blml/attachments/20081124/2491b618/attachment-0001.htm From picatou at picatou.com Thu Nov 27 17:14:26 2008 From: picatou at picatou.com (Picatou) Date: Thu, 27 Nov 2008 11:14:26 -0500 Subject: [blml] TR: Law 27 Message-ID: <001301c950ab$3842ee70$a8c8cb50$@com> To moderator I sent this email to BLML, some time ago, and received this automatic answer Your mail to 'blml' with the subject Law 27 Is being held until the list moderator can review it for approval. The reason it is being held: Message has a suspicious header Either the message will get posted to the list, or you will receive notification of the moderator's decision. If you would like to cancel this posting, please visit the following URL: As a member of the list since many years, I dont know why my emails are now sent to the moderator and why my messages have "suspicious header". Would you please allow my message to the list and help by being again a "normal" participant. Laval Du Breuil ACBL TD Hi all, I spent some nights trying to set a flow chart of the new Law 27 on one single page and think I finally succeeded, but I still have a question: 27B1(a) reads: if the insufficient bid is corrected by the lowest sufficient..... the auction proceeds without further rectification. Law 16D does not apply, but see D below. 27B1(b) : If, except as in (a) above, the insufficient bid is corrected....... the auction proceeds without further rectification, but see D below. Does the fact that "Law 16D does not apply" is not written in 27B1(b) means something ? Laval Du Breuil Quebec City PS: if some of you would like to see this flow chart, send me a private message. The 2008 edition of Laws of Duplicate bridge Made Easier is now ready for printing (French and English versions) and I will put the included flow charts (38) on my WEB as soon as possible. -------------- next part -------------- An HTML attachment was scrubbed... URL: http://www.amsterdamned.org/pipermail/blml/attachments/20081127/b722112c/attachment-0001.htm